Sei sulla pagina 1di 1616

ORGANIC

CHEMISTRY
SECOND EDITION

BHUPINDER MEHTA
Associate Professor
Department of Chemistry
Swami Shraddhanand College
University of Delhi
and
MANJU MEHTA
Associate Professor
Department of Chemistry
Maitreyi College
University of Delhi
ORGANIC CHEMISTRY, Second Edition
Bhupinder Mehta and Manju Mehta

© 2015 by PHI Learning Private Limited, Delhi. All rights reserved. No part of this book may be
reproduced in any form, by mimeograph or any other means, without permission in writing from the
publisher.

ISBN-978-81-203-5126-4
The export rights of this book are vested solely with the publisher.

Ninth Printing (Second Edition) . . . . . . . . . July, 2015

Published by Asoke K. Ghosh, PHI Learning Private Limited, Rimjhim House, 111, Patparganj
Industrial Estate, Delhi-110092 and Printed by Mohan Makhijani at Rekha Printers Private Limited,
New Delhi-110020.
To all our Family Members for their affection
and
our loving
daughter Ananta and son Sarthak
for being patient and supportive
Table of Contents
Preface
About the Cover Image
Acknowledgements
About the Authors
What This Book Is About
1. Organic Molecules: Structure, Bonding and Properties
1.1 ORGANIC CHEMISTRY—An introduction
1.2 Electronic structure and chemical bonding in organic compounds
1.3 LEWIS STRUCTURE and Chemical Bonds
1.3.1 Electronegativity
1.3.2 Ionic Bond
1.3.3 Covalent Bond
1.3.4 Atomic Radius, van der Waals Radius, Bond Length, and Bond
Angle
1.3.5 Formal Charge
1.3.6 Bond Polarity and Dipole Moment
1.4 Concept of Hybridization and Covalent Bonding
1.4.1 sp3 Hybridization
1.4.2 sp2 Hybridization
1.4.3 sp Hybridization
1.5 Writing the structural Formula for Organic Molecules
1.6 CONCEPT OF RESONANCE (Mesomerism)
1.7 INTERMOLECULAR FORCES (van der Waals Forces)
1.7.1 Melting Point and Boiling Point
1.8 Purification and Identification of Organic Compounds
1.9 Classification of Organic Compounds
1.10 ISOMERISM in Organic Molecules
1.10.1 Constitutional Isomers (Formerly Structural Isomers)
1.10.2 Resonance versus Tautomerism
1.11 ACIDS AND BASES
1.11.1 Bronsted and Lowry Definition
1.11.2 Lewis Definition
Exercises**
Answers to selected exercises
2. IUPAC Nomenclature of Organic Compounds
2.1 Introduction
2.2 IUPAC Nomenclature
2.2.1 Rules for Naming the Organic Compounds
2.3 Selected Examples Of Monofunctional And Polyfunctional
Organic Compounds
2.3.1 Writing the Structure of an Organic Compound from its IUPAC
Name
2.4 COMMON ERRORS IN WRITING IUPAC NAMES
Exercises*
Answers
3. Stereochemistry
3.1 INTRODUCTION
3.2 Configurational Isomerism
3.2.1 Concept of Chirality [Asymmetry]
3.2.2 Chirality in Organic Molecules: Enantiomers and
Diastereoisomers
3.2.3 Fischer Projection
3.2.4 Number of Stereoisomers of a Compound
3.3 Optical Activity
3.4 Absolute Configuration (R And S Configuration)
3.4.1 Assigning R and S Configuration
3.4.2 Relative Configuration (D- and L- Nomenclature)
3.4.3 Chirality in a Molecule with no Stereogenic (Chiral) Centre
3.5 GEOMETRICAL ISOMERISM
3.6 CONFORMATIONS
3.6.1 Conformations of Ethane
3.6.2 Conformations of Propane
3.6.3 Conformations of Butane
3.7 cycloalkanes: conformations and Geometrical Isomerism
3.7.1 Conformations of Cyclohexane
3.7.2 Conformations of Monosubstituted Cyclohexane
3.7.3 Conformations of Disubstituted Cyclohexane
Answers to selected exercises
4. Fundamentals of Organic Reactions
4.1 Electronic Displacements
4.1.1 Inductive Effect
4.1.2 Electromeric Effect
4.1.3 Resonance Effect [or Mesomeric Effect]
4.1.4 Hyperconjugation (No bond resonance)
4.2 REACTIVE INTERMEDIATES
4.2.1 Carbocations
4.2.2 Carbanions
4.2.3 Free Radicals
4.2.4 Carbene
4.2.5 Nitrene
4.3 ReAgent Types
4.3.1 Electrophiles and Nucleophiles
4.4 Types of Reactions
4.5 CHEMICAL ENERGETICS
4.5.1 Thermodynamics and Kinetics of Chemical Reactions
4.5.2 Chemical Equilibrium
4.5.3 Rate of Reaction
4.5.4 Energy Diagrams (or Energy Profile) of Chemical Reactions
4.6 STERIC EFFECT
4.7 Solvents in Organic Reactions
4.8 Organic Compounds as Acid and Bases
EXeRCISES
EXPLORE MORE (Set-I)
5. lkanes and Cycloalkanes
5A.1 Introduction
5A.1.1 Physical Properties
5a.2 Preparation of Alkanes
5A.2.1 Catalytic Hydrogenation of Alkenes and Alkynes
5A.2.2 From Haloalkanes (Alkyl halides)
5A.2.3 From Carbonyl Compounds (Aldehydes and ketones)
5A.2.4 From Sodium Salt of Carboxylic Acids
5A.3 Chemical Properties of Alkanes
5A.3.1 Halogenation
5A.3.2 Nitration
5A.3.3 Sulfonation
5A.3.4 Chlorosulfonation
5A.3.5 Oxidation Reactions
5A.3.6 Other Reactions
5A.4 Petroleum
5A.4.1 Petrochemicals
5A.4.2 Coal
5B.1 Introduction
5B.2 Strain in Ring Compounds: Baeyer’s Strain Theory
5B.3 Preparation of Cycloalkanes
5B.4 Chemical Properties Of Cycloalkanes
5B.4.1 Halogenation
5B.4.2 Catalytic Hydrogenation
5B.4.3 Effect of Heat
5B.4.4 Reaction with Hydrogen Halides
Selected Solved Examples
Exercises
6. Alkenes
6.1 Introduction
6.1.1 Physical Properties
6.2 Preparation Of Alkenes
6.2.1 Reduction of Alkynes: Formation of cis and trans Alkenes
6.2.2 Elimination Reactions [Saytzeff’s and Hofmann’s rule]
6.2.3 Other Methods
6.3 Chemical Properties of Alkenes
6.3.1 Stability of Alkenes
6.3.2 Electrophilic Addition Reactions
6.3.3 Free Radical Addition Reaction
6.3.4 Oxidation Reactions
6.3.5 Allylic Substitution Reactions
6.3.6 Polymerization
Selected Solved Examples
EXERCISES
7. Alkadienes
7.1 Introduction
7.2 Buta-1,3-Diene
7.2.1 Molecular Orbital Picture of Buta-1,3-diene
7.3 Preparation of Buta-1,3-diene
7.4 Chemical Properties of Buta-1,3-diene
7.4.1 Electrophilic Addition Reactions
7.4.2 Free Radical Addition Reactions
7.4.3 Diels–Alder Reaction [Cycloaddition Reaction]
7.4.4 Reduction and Oxidation Reactions
7.4.5 Polymerization
7.5 Isoprene (2-MethylButa-1,3-diene)
7.5.1 Preparation
7.5.2 Chemical Properties
7.6 Chloroprene (2-chlorobuta-1,3-diene)
Exercises
8. Alkynes
8.1 Introduction
8.1.1 Physical Properties
8.2 PREPARATION OF ALKYNES
8.3 CHEMICAL PROPERTIES OF ALKYNES
8.3.1 Addition of Hydrogen
8.3.2 Electrophilic Addition Reactions
8.3.3 Nucleophilic Addition Reactions
8.3.4 Reactions Involving Acetylenic Hydrogens
8.3.5 Polymerization Reactions
8.3.6 Isomerization (Acetylene Allene Rearrangement)
8.3.7 Oxidation Reactions
Selected Solved Examples
EXERCISES
9. Concepts of Aromaticity, Benzene and its Derivatives
A. Concepts of Aromaticity
9A.2 STRUCTURE OF BENZENE
9A.2.1 Kekule Structure
9A.2.2 Resonance Structure
9A.2.3 Orbital Picture of Benzene
9A.3 RESONANCE ENERGY: STABILITY OF BENZENE
9A.4 HUCKEL’S RULE AND AROMATICITY
9A.5 Aromaticity in BENZENE AND other CYCLIC systems
9A.5.1 Aromaticity and the Three Membered Ring Systems
9A.5.2 Aromaticity and Four Membered Ring Systems
9A.5.3 Aromaticity and Five Membered Ring Systems
9A.5.4 Aromaticity and Six Membered Ring Systems
9A.5.5 Aromaticity and Seven Membered Ring Systems
9A.5.6 Aromaticity and Eight Membered Ring Systems
9A.5.7 Aromaticity and Annulenes
9A.5.8 Aromaticity and Other Ring Systems
B. Benzene and its Derivatives
9B.1.1 Coal Tar: Source of Aromatic Hydrocarbons
9B.2 Nomenclature of aromatic compounds
9B.3 Physical properties of aromatic hydrocarbons
9B.4 PREPARATIONS AND CHEMICAL PROPERTIES OF
BENZENE
9B.4.1 Preparations
9B.4.2 Chemical Properties
9B.5 ARENES: Methylbenzene (Toluene) and styrene
9B.5.1 Methylbenzene
9B.5.2 Styrene
Selected Solved Examples
EXERCISES
10. Aromatic Electrophilic Substitution Reactions—Mechanism,
Orientation and Reactivity
10.1 INTRODUCTION
10.2 Mechanism Of Electrophilic Substitution Reactions Of Benzene
10.2.1 Halogenation
10.2.2 Nitration
10.2.3 Friedel–Crafts Alkylation
10.2.4 Friedel–Crafts Acylation
10.2.5 Sulfonation
10.3 ORIENTATION AND REACTIVITY IN Monosubstituted
Benzene: EFFECT OF SUBSTITUENTS ON ELECTROPHILIC
SUBSTITUTION REACTIONS
10.3.1 Nature of Groups
10.3.2 Effect of Group on the Reactivity
10.3.3 Effect of Groups on Orientation
10.4 ORIENTATION IN DISUBSTITUTED BENZENE
Selected Solved Examples
EXERCISES
EXPLORE MORE (Set-II)
11. Haloalkanes
11.1 INTRODUCTION
11.1.1 Physical Properties
11.2 PREPARATION OF HALOALKANES
11.3 CHEMICAL PROPERTIES of Haloalkanes
11.3.1 Nucleophilic Substitution Reactions: A General Discussion
11.3.2 Reactions of Haloalkanes
11.3.3 Elimination Reactions: A General Discussion
11.3.4 Substitution Versus Elimination
11.4 ORGANOMETALLIC COMPOUNDS –AN OVERVIEW
[Reaction of Haloalkanes with Metals]
11.4.1 General Characteristics
11.5 Allyl Chloride and Vinyl Chloride
11.5.1 Preparations
11.5.2 Chemical Reactivity of Vinyl and Allyl Halides
11.6 POLYHALOGENATED COMPOUNDS: A BRIEF ACCOUNT
11.6.1 Dihalogen Derivatives
11.6.2 Trihalogen Derivatives
Selected Solved Examples
EXERCISES
12. Haloarenes
12.1 INTRODUCTION
12.1.1 Physical Properties
12.2 PREPARATION of Haloarenes
12.3 CHEMICAL PROPERTIES of Haloarenes
12.3.1 Nucleophilic Substitution Reactions
12.3.2 Electrophilic Substitution Reactions
12.3.3 Reactions of Haloarenes with Metals
12.4 SIDE CHAIN HALOGEN SUBSTITUTED AROMATIC
COMPOUNDS
12.4.1 Preparations
12.4.2 Chemical Properties
Selected Solved Examples
EXERCISES
13. Monohydric Alcohols (Alkanols)
13.1 INTRODUCTION
13.1.1 Physical Properties
13.2 PREPARATION of Monohydric alcohols
13.3 CHEMICAL PROPERTIES of Monohydric Alcohols
13.3.1 Alcohols as an Acid as well as Base: A General Discussion
13.3.2 Reactions Involving the Acidic Characteristics of Alcohols
13.3.3 Reactions Involving the Basic Characteristics of Alcohols
13.3.4 Reactions involving the entire alcohol molecule
13.4 More about alcohols
Selected Solved Examples
Exercises
14. Ethers, Epoxides, Thiols and Thioethers
14A.1 Introduction
14A.1.1 Physical Properties
14A.2 PREPARATION OF ETHERS
14A.2.1 From Alcohols
14A.2.2 From Halo Compounds
14A.2.3 Alkoxymercuration–demercuration of Alkenes
14A.3 CHEMICAL PROPERTIES of Ethers
14A.3.1 Reactions due to Etheral Oxygen
14A.3.2 Reactions Involving Ether Linkage [Cleavage of Carbon–
Oxygen Bond]
14A.3.3 Other Reactions
14A.3.4 Common Uses of Ethers
14A.4 Crown ethers
14B.1 INTRODUCTION
14B.2 PREPARATION OF EPOXIDES
14B.3 CHEMICAL PROPERTIES of Epoxides
14B.3.1 General Mechanism for Ring Opening in Epoxides
14B.3.2 Ring Opening in Unsymmetrical Epoxides
14C.1 GENERAL INTRODUCTION—SULFUR COMPOUNDS IN
ORGANIC CHEMISTRY
14C.2 THIOLS [MERCAPTANS]
14C.2.1 Physical Properties
14C.3 PREPARATION OF THIOLS
14C.4 CHEMICAL PROPERTIES of Thiols
14C.5 THIOETHERS [ALKYL SULFIDES]
14C.6 PREPARATION OF THIOETHERS
14C.7 CHEMICAL PROPERTIES of Thioethers
Exercises
15. Polyhydric Alcohols (Diols and Triols)
15.1 INTRODUCTION
15.1.1 Physical Properties
15.2 ETHYLENE GLYCOL [ETHANE-1,2-DIOL]
15.2.1 Preparations
15.2.2 Chemical Properties
15.2.3 Uses of ethylene glycol
15.3 PINACOLS
15.3.1 Preparation of 2,3-dimethylbutane-2,3-diol
15.3.2 Pinacol–Pinacolone Rearrangement
15.4.1 Physical Properties
15.4.2 Preparations
15.4.3 Chemical Properties
Exercises
16. Phenols
16.1 INTRODUCTION
16.1.1 Physical Properties
16.2 PREPARATION OF PHENOLS
16.3 CHEMICAL PROPERTIES of Phenol
16.3.1 Acidic Nature of Phenol: A General Discussion
16.3.2 Reactions due to Acidic Nature of Phenol
16.3.3 Electrophilic Substitution Reactions of Phenols
16.3.4 Other Reactions
16.4 POLYHYDRIC PHENOLS
16.4.1 Dihydric Phenols
16.5 Quinones
Selected Solved Examples
Exercises
EXPLORE MORE (Set-III)
17. Aliphatic Carbonyl Compounds (Alkanals and Alkanones)
17.1 INTRODUCTION
17.1.1 Physical Properties
17.2 PREPARATION of ALiphatic Carbonyl Compounds
17.2.1 Formation of both Aldehydes and Ketones
17.2.2 Formation of Aldehydes
17.2.3 Formation of Ketones
17.3 CHEMICAL PROPERTIES of Carbonyl COmpounds
17.3.1 Reactivity of Carbonyl Group: A General Discussion
17.3.2 Nucleophilic Addition Reactions
17.3.3 Oxidation Reactions
17.3.4 Reduction Reactions
17.3.5 Reactions Involving α-Carbons of Carbonyl Compounds
17.3.6 Polymerization Reactions of Formaldehyde and Acetaldehyde
17.4 Reaction of α,β-Unsaturated carbonyl compounds
Selected Solved Examples
Exercises
18. Aromatic Aldehydes and Ketones
18.1 INTRODUCTION
18.1.1 Physical Properties
18.2 PREPARATION of Aromatic Aldehydes and Ketones
18.2.1 Oxidation Reactions
18.2.2 By Heating Calcium Salts of Carboxylic Acids
18.2.3 From Grignard Reagent
18.2.4 Electrophilic Substitution of Aromatic Hydrocarbons
18.2.5 Reactions Used for the Preparations of Aromatic Aldehydes
18.3 CHEMICAL PROPERTIES of Aromatic Aldehydes and Ketones
18.3.1 Reactivity of Carbonyl Group: A General Discussion
18.3.2 Nucleophilic Addition Reactions
18.3.3 Oxidation
18.3.4 Reduction
18.3.5 Typical Name Reactions of Aromatic Carbonyl Compounds
18.3.6 Electrophilic Substitution Reactions
Selected Solved Examples
Exercises
19. Aliphatic Carboxylic Acids and Their Derivatives
19.1 INTRODUCTION
19.2 PREPARATION of Aliphatic Carboxylic Acids
19.2.1 Oxidation Reactions
19.2.2 Hydrolysis
19.2.3 Carboxylation of Oganometallic Compounds (Carbonation)
19.2.4 Carbonylation Reactions
19.3 CHEMICAL PROPERTIES of Aliphatic Carboxylic Acids
19.3.1 Acidic Character of Carboxylic Acids: A General Discussion
19.3.2 Reactions Involving Acidic Hydrogen
19.3.3 Reactions Involving Replacement of –OH Group
19.3.4 Reactions Involving –COOH Group
19.3.5 Reactions Involving α-Carbon of Carboxylic acids
19.3.6 Some Typical Chemical Properties of Formic Acid
19.4 ACID DERIVATIVES
19.4.1 Acid Halides
19.4.2 Acid Anhydrides
19.4.3 Ketene—An Intramolecular Anhydride of Carboxylic Acid
19.4.4 Esters
19.4.5 Amides
19.4.6 Urea—A Derivative of Carbonic Acid
Selected Solved Examples
Exercises
20. Bifunctional Carboxylic Acids and Their Derivatives
INTRODUCTION
20A.1 DICARBOXYLIC ACIDS [Saturated and unsaturated]
20a.1.1 Preparations of Saturated Dicarboxylic Acids
20A.1.2 Preparation of Unsaturated Dicarboxylic Acids
20A.1.3 Chemical Properties
20A.2 Substituted Carboxylic Acids
20A.2.1 Hydroxy Acids
20A.2.2 Preparations of Hydroxy Acids
20A.2.3 Chemical Properties
20A.2.4 Halocarboxylic Acids
20A.2.5 Preparations of Halocarboxylic Acids
20A.2.6 Chemical Properties
20B.1 INTRODUCTION
20B.2 ETHYL ACETOACETATE
20B.2.1 Preparation
20B.2.2 Chemical Properties
20B.3 DIETHYL MALONATE
20B.3.1 Preparation
20B.3.2 Chemical Properties
Exercises
21. Aromatic Carboxylic and Sulfonic Acids
A. AROMATIC CARBOXYLIC ACIDS 729
21A.2 Preparation of Aromatic Carboxylic Acids
21A.3 CHEMICAL PROPERTIES of Aromatic Carboxylic Acids
21A.3.1 Acidic Character of Aromatic Carboxylic Acids: A General
Discussion
21A.3.2 Reactions Involving –OH Group: Nucleophilic Acyl
Substitution
21A.3.3 Reduction Reaction
21A.3.4 Reactions Involving –COOH Group
21A.3.5 Electrophilic Substitution Reactions
21A.3.6 Reactions of Some Important Substituted Carboxylic Acids
21B.1 INTRODUCTION
21B.2 PREPARATION of Aromatic Sulfonic Acids
21B.2.1 Electrophilic Substitution Reactions
21B.3 CHEMICAL PROPERTIES
21B.3.1 Acidic Character of Arenesulfonic Acid: A General
Discussion
21B.3.2 Electrophilic Substitution Reactions
21B.4 Aromatic Sulfonic Acid Derivatives
Selected Solved Examples
EXERCISES
Mini Essay I
A. LIPIDS—FATS, OILS, AND WAXES
B. SOAPS AND DETERGENTS
22. Aliphatic Nitrogen Containing Compounds [Amines, Nitro,
Nitriles, Isonitriles, Isocyanates and Thiocyanates]
A. AMINES [ALKANAMINES] 767
22A.2 PREPARATION of Alkanamines
22A.2.1 Reactions, which Yield Mixture of 1°, 2°, and 3° Amines
22A.2.2 Reactions Specific to the Individual Preparation of 1°, 2°, and
3° Amines
22A.3 CHEMICAL PROPERTIES
22A.3.1 Basicity of Amines: A General Discussion
22A.3.2 Reactions of Amines
22B.1 INTRODUCTION
22B.2 Preparation Of Nitroalkanes
Substitution reactions
22B.3 Chemical Properties Of Nitroalkanes
22B.3.1 Reactions at α-Carbon
22B.3.2 Reactions Involving Nitro Group
22B.3.3 Reactions Differentiating Nitroalkanes and Alkylnitrites
22C.1 Preparation and Properties
22D.1 INTRODUCTION
22D.2 Preparation Of Alkanenitriles
22D.2.1 Substitution Reactions
22D.2.2 Dehydration Reactions
22D.2.3 Other Reactions
22D.3 Preparation Of Isonitriles
22D.4 Chemical Properties Of Nitriles And Isonitriles
22E.1 INTRODUCTION
22E.2 Preparation Of Isocyanates
22E.3 Chemical Properties Of Isocyanates
22E.4 Preparation Of Isothiocyanates
22E.5 Chemical Properties Of Isothiocyanates
Selected Solved Examples
EXERCISES
23. Aromatic Nitrogen Containing Compounds (Nitro, Amines and
Diazonium Salts)
23A.1 INTRODUCTION
23A.2 Preparation Of Aromatic Nitro Compounds
23A.3 Chemical Properties Of Aromatic Nitro Compounds
23A.3.1 Nucleophilic Substitution Reactions
23A.3.2 Electrophilic Substitution Reactions
23A.3.3 Reduction
23B.1 INTRODUCTION
23B.2 CHEMICAL PROPERTIES OF AROMATIC AMINES
23B.2.1 Basicity in Aromatic Amines— A General Discussion
23B.2.2 Nucleophilic Substitution Reactions
23B.2.3 Electrophilic Substitution Reactions
23B.3 ARENEDIAZONIUM SALTS
23B.3.1 Chemical Properties of Diazonium Salts
Selected SOlved Examples
EXERCISES
24. Polynuclear Hydrocarbons
24.1 INTRODUCTION
24.2 NAPHTHALENE
24.2.1 Structure of Naphthalene
24.2.2 Aromaticity in Naphthalene
24.2.3 Preparations
24.2.4 Chemical Properties
24.3 DERIVATIVES OF NAPHTHALENE
24.3.1 Naphthols
24.3.2 Naphthylamines
24.3.3 Naphthoic Acids
24.3.4 Naphthoquinones
24.4 ANTHRACENE
24.4.1 Structure of Anthracene
24.4.2 Preparations
24.4.3 Chemical Properties of Anthracene
24.5 PHENANTHRENE
24.5.1 Preparation
24.5.2 Chemical Properties of Phenanthrene
EXERCISES
EXPLORE MORE (Set-IV)
25. Review Part I: Organometallic Compounds Part II: Oxidation–
Reduction Reactions
25.1 INTRODUCTION
25.2 ORGANOMAGNESIUM HALIDES—GRIGNARD REAGENT
25.2.1 Reactions of Grignard Reagent
25.2.2 Limitations
25.3 ORGANOLITHIUMS
25.3.1 Reactions of Organolithiums
25.4 Lithium dialkylcuprates—Gilman Reagent
25.4.1 Reactions of Lithium dialkylcuprates (Gilman Reagent)
25.5 Introduction
25.6 REDUCTION REACTIONS
25.6.1 Reduction by Catalytic Hydrogenation
25.6.2 Reduction by Metal Hydrides
25.7 OXIDATION REACTIONS
26. Heterocyclic Compounds
26.1 INTRODUCTION
26.2 FIVE MEMBERED HETEROCYCLIC COMPOUNDS
26.2.1 Pyrrole, Furan, and Thiophene
26.2.2 Structure and Aromaticity
26.2.3 Preparations of Pyrrole, Furan, and Thiophene
26.2.4 Chemical Properties of Pyrrole, Furan, and Thiophene
26.3 SIX-MEMBERED HETEROCYCLIC COMPOUND
26.3.1 Pyridine
26.3.2 Structure and Aromaticity
26.3.3 Basicity of Pyridine
26.3.4 Preparations of Pyridine
26.3.5 Chemical Properties of Pyridine
26.4 FUSED HETEROCYCLIC SYSTEMS
26.4.1 Quinoline and Isoquinoline
26.4.2 Preparations of Quinoline
26.4.3 Chemical Properties of Quinoline
26.4.4 Preparations of Isoquinoline
26.4.5 Chemical Properties of Isoquinoline
EXERCISES
27. Carbohydrates
27.1 INTRODUCTION
27.2 D- AND L-SUGARS: RELATIVE CONFIGURATION OF
SUGARS
27.3 D-(+)-GLUCOSE: Chemical Properties and Structure
27.3.1 Open Chain Structure
27.3.2 Cyclic Structure of Glucose
27.3.3 Mutarotation
27.3.4 Synthesis and Degradation of Aldoses
27.3.5 Epimerization and Ene–Diol Rearrangement
27.3.6 Osazone Formation: Reaction with Phenylhydrazine
27.3.7 Interconversion Involving Aldoses and Ketoses
27.3.8 Configuration of Glucose
27.3.9 Writing Haworth Formulae and Conformations for Sugars
27.3.10 Evidence for Cyclic Structure of Glucose
27.4 D(-) FRUCTOSE: Chemical Properties and Structure
27.4.1 Cyclic Structure of Fructose
27.5 DISACCHARIDES
27.5.1 Sucrose
27.5.2 Lactose
27.5.3 Maltose and Cellobiose
27.6 POLYSACCHARIDES
EXERCISES
28. Amino Acids, Peptides and Proteins
28.1 Introduction
28.2 Classification of Amino Acids
28.3 STEREOCHEMISTRY of AMINO ACIDS
28.4 PHYSICAL PROPERTIES
28.4.1 Electrophoresis
28.5 SYNTHESIS OF AMINO ACIDS
28.6 CHEMICAL PROPERTIES of Amino ACIDs
28.6.1 Reactions Due to Amino Group
28.6.2 Reactions Due to Carboxylic Group (–cooh)
28.6.3 Reactions Due to Both –Nh2 and –Cooh Groups
28.7 PEPTIDES and THeir Synthesis
28.7.1 Use of Protecting Groups in Synthesis of Polypeptides
28.7.2 Synthesis of Peptides Using Protected Amino and Carboxylic
Acid Ends
28.7.3 Solid Phase Polypeptide Synthesis
28.8 Determination of Sequence of Amino Acids in a Given
Polypeptide
28.8.1 End Group Analysis
28.8.2 Sequence Analysis
28.9 PROTEiNS
28.9.1 Structure of Proteins
28.9.2 Fibrous and Globular Proteins
28.9.3 Denaturation of Proteins
EXERCISES***
Answers to selected exercises
MINI ESSAY II
ENZYMES AND NUCLEIC ACIDS
Enzyme Action
B. NUCLEIC ACIDS
29. Drugs and Pesticides
A. DRUGS
29A.2 ANALGESICS, ANTIPYRETICS AND ANTI-
INFLAMMATORY DRUGS
29A.3 SULFA DRUGS—ANTIBACTERIAL AGENTS
29A.4 CHLORAMPHENICOL (CHLOROMYCETIN)—AN
ANTIBIOTIC
29A.5 CHLOROQUINE—ANTIMALARIAL DRUG
29A.6 ANTIHISTAMINES (Antihistaminics)
29B.1 INTRODUCTION
29B.2 INSECTICIDES
29B.2.1 Organochlorine Compounds
29B.2.2 Organophosphorous Compounds
29B.3 Herbicides
29B.4 Fungicides
EXERCISES
Pesticides
MINI ESSAY III
A. Alkaloids
B. TERPENes
C. Steroids
30. Dyes
30.1 Introduction
30.1.1 Theory of Dyeing
30.1.2 Classification of Dyes
30.2 COLOUR AND STRUCTURE
30.2.1 Witt Theory
30.2.2 Quinonoid Theory
30.2.3 Modern Theory
30.3 SYNTHESIS AND APPLICATION OF DYES
30.3.1 Azo Dyes
30.3.2 Triphenylmethane Dyes
30.3.3 Phthalein Dyes
30.3.4 Anthraquinone Dye
30.3.5 Indigotin Dye
EXERCISES
31. Synthetic Polymers
31.1 INTRODUCTION
31.2 Classification Of Polymers
31.3 PHYSICAL PROPERTIES AND CLASSIFICATION
31.3.1 Stereochemistry of Polymers
31.4 POLYMERIZATION REACTIONS
31.4.1 Addition Polymers
31.4.2 Condensation Polymers
31.5 RUBBERS
31.5.1 Natural Rubber
31.5.2 Vulcanization of Rubber
31.5.3 Synthetic Rubber
EXERCISES
32. Spectroscopy and Structure
32.1 INTRODUCTION
32.2 ULTRAVIOLET–VISIBLE SPECTROSCOPY
Woodward–Fieser rules
32.3 INFRARED SPECTROSCOPY
32.4 NUCLEAR MAGNETIC RESONANCE (NMR)
SPECTROSCOPY
32.5 MASS SPECTROMETRY—AN INTRODUCTION
EXERCISES
Glossary
Index
Preface

Audience
It has been more than ten years since this book was published in January,
2005. As an author there is always a temptation to include everything in a
book. But, we have to limit ourselves. Over the past decade our students,
friends and colleagues have been our best critics. In fact, these are the
people who have carefully gone through the text and appreciated the
contents and also made valuable suggestions and a few corrections.
Considering the scope of the book, an attempt has been made to incorporate
all important suggestions and necessary corrections. The most obvious
revision in the second edition is the introduction of a new set of problems to
enhance the conceptual vision on the various fundamental aspects of organic
chemistry. Further, there has been additions and modifications in the
contents of some of the chapters, wherever required.
The second edition of the book continues to provide the pedagogical
features with which it was written in the beginning. This helps students not
only to acquire a sound knowledge and understanding of organic chemistry,
but also makes their study interesting, stimulating and relevant as much as
possible. We conceived the idea of writing this book with the objective to
provide clarity on the core concepts of organic chemistry that will help
students in their studies and practice as well. The book presents a thorough
treatment of the principles in a new and exciting way that makes it easier for
students to learn and evokes interest of teachers to teach. The book also
introduces significant developments that have taken place in the field.
Organization of the Text
We have organized the text in a manner that the students develop interest in
this fascinating yet challenging subject. The book comprises:
Thirty-two chapters with abundance of solved examples throughout.
Four sets of especially designed solved problems named ‘Explore More’.
Three ‘Mini Essays’ for providing introductory knowledge of natural
products.
Chapters 1–4
Chapter 1 offers coherent presentation of fundamental aspects of chemical
bonding, structure and properties of molecules. It also covers the concept of
hybridization, resonance, isomerism, and intermolecular forces in organic
molecules. A brief discussion on the qualitative and quantitative analysis is
also incorporated.
IUPAC nomenclature has been discussed in Chapter 2 with latest
recommendations. Over 275 examples/solved problems are included with
suitable explanations. The rules for naming the organic compounds are
clarified by giving appropriate structures that are loaded with key points. In
this particular chapter we have endeavoured to share our classroom teaching
experience with the prospective readers of the book.
The treatment of stereochemistry in Chapter 3 is simple and illustrative. The
structures in this chapter are aesthetically drawn to give a feeling of real
three-dimensional visualization.
A brief explanation of the mechanisms of organic reactions and related
topics like reactive intermediates, electronic factors in organic reactions,
steric factors, role of solvents and chemical energetics has been offered in
Chapter 4.
A new problem set (Explore more-I) has been added after Chapter 4 in the
second edition to provide an in-depth view of fundamentals of organic
chemistry pertaining to
Chapters 1–4. The special emphasis is given to stereochemistry.
Chapters 5–24
The chemistry of hydrocarbons and functional groups has been dealt with in
detail in chapters 5 through 24. We have followed the strategy of discussing
a particular functional group, first for aliphatic compounds followed by that
for aromatic compounds. We feel that this strategy will help the students to
know the preparations and properties, which are different (or similar) in
nature. At the same time, utmost care has been taken to ensure that where
the underlying principles for both the aliphatic and aromatic compounds are
same, these are not repeated and instead bridged by appropriate tie-ins to
earlier material. This way a continuity is established and the overall text
stands simplified without diluting the quality of the contents.
Many new problems have been added to Explore More-II based on the
contents discussed in Chapters 5–10. Similarly, more problems have been
added in Explore More-III and IV as well.
Chapters 25—A Review
This chapter is one of the key features of the text. It presents an overall view
of major topics in two parts:
Part I: Organometallic Compounds
Part II: Oxidation-Reduction Reactions
The readers might wonder as to why we have only just given a review
instead of devoting a separate chapter to each of these topics. The simple
reason is to avoid repetition of the chemistry and at the same time
consolidate the much-scattered chemistry under one head, as almost all the
reactions pertaining to these topics are an integral part of the functional
group chemistry discussed in previous chapters.
Chapters 26–32
The topics like heterocyclic compounds, carbohydrates, amino acids,
peptides and proteins, drugs and pesticides, dyes, synthetic polymers,
spectroscopy and structure are discussed in chapters 26 through 32.
However, we have restricted our discussion to a level, generally covered in
the undergraduate curriculum.
In this edition some text has been added/modified in the chapters on
carbohydrates, amino acids, peptides and proteins, spectroscopy and
structure. Whereas in chapter 32 (Spectroscopy and Structure) Woodward-
Fieser rules along with a number of examples have been added to
Ultraviolet-Visible spectroscopy.
What is Notabilia?
It is an old English word which means, “things worthy of notice”. When we started writing this
book, we thought of putting some additional and important information under the head of
‘infobox’. However, it was creditable on the part of our editor to come up with a word that was not
only catchy but also went along completely and exactly with the contents. We overwhelmingly
accept this beautiful change.

Explore More
The four sets of ‘Explore More’ are a supplement to the text, designed to
provide in-depth knowledge in the form of some typical solved problems
(the number of problems included is 141).
Aim: Chemistry is taught to undergraduates at varying degrees of depth.
These four sets are a sort of bridging chapters for the students who wish to
explore the various aspects of the organic chemistry with a little more
attention to details. This material is based on the philosophy of ‘study or
skip’—students with chemistry as a major subject will find the problem sets
more meaningful. Other students studying organic chemistry as their
qualifying subject may ‘skip’ these problem sets.
Scope: The sets of ‘Explore More’ deal with the topics discussed in chapters
1 through 24. These problems (with solutions) are in addition to the
‘selected solved examples’ included at the end of various chapters. Each set
appears after a few chapters and contains the problems based on the
chemistry of the previous chapters.
Set I Set II Set III Set IV
Related to the text from chapters 1–4 5–10 11–16 17–24
Appears after chapter 4 10 16 24
Number of solved problems 1–31 32–71 72–98 99–141
Since problems are related to fundamental aspects of reactions, it is very
difficult to put the boundaries of chapters between the problems of each set.
The above division of problems must be considered in a broader sense
where the explanation for the problems may include the information given
in the earlier chapters. At appropriate places, section numbers are
incorporated for further reference.
Hope: We hope that our approach of adding these pages in the name of
‘Explore More’ will encourage students to learn more about chemistry, over
and above the text included in the chapters.
Mini Essays
A student of organic chemistry must have the knowledge of natural products
and related chemistry, for example, topics such as lipids (fats, oils and
waxes), nucleic acids, enzymes, terpenes, alkaloids and steroids. These
topics are taught to undergraduates only at an introductory level. We have
introduced these topics in the book in the form of three mini essays.
Mini Essay I Part A Lipids—Fats, Oils and Waxes
(After Chapter 21) Part B Soaps and Detergents
Mini Essay II Enzymes and Nucleic Acids
(After Chapter 28)
Mini Essay III Alkaloids, Terpenes and Steroids
(After Chapter 29)
In Mini Essay I, soaps and detergents are covered in Part B, because the
soaps are manufactured from fats/oils and detergents are complementary to
soaps.
Appendix and Glossary
In the appendix some important and useful data has been given in the form
of tables like characteristics of different solvents, energy data for homolytic
and heterolytic bond cleavage. The glossary includes ‘Name reactions’,
‘Reagents’ and some important terms so that the students can revise them at
a glance (page number(s) are also given for quick reference).
Finally
We hope that the students will find this second edition of the book
interesting, easy to read, and easy to learn from. We would very much
appreciate comments from both teachers and students to help us improve the
text further. Please draw our attention to any errors remained undetected that
you might discover.

BHUPINDER MEHTA
MANJU MEHTA
Acknowledgements

It is a pleasure to acknowledge the enthusiastic support and assistance given to


us by many people at various stages of the development of the manuscript. We
express our gratitude to Professor J.M. Khurana, Department of Chemistry,
University of Delhi, for a quick and thoughtful review of the book and also for
his appreciation for a new feature that has been introduced in the second edition.
Our cordial thanks to Dr. G.K. Parashar (Associate Professor, Swami
Shraddhanand College, University of Delhi) for being a wonderful listener and
advisor, at every stage of manuscript development in both the editions.
We owe our sincere thanks to Dr. (Mrs.) Sulekha Dogra (Associate Professor,
retired, Maitreyi College, University of Delhi) for a thorough review of the
manuscript in its initial stages.
We especially wish to thank our friends, Dr. Sanjeev Kumar (Associate
Professor, School of Sciences, Indira Gandhi National Open University), Dr.
(Mrs.) Lalita S. Kumar (Associate Professor, School of Sciences, Indira Gandhi
National Open University), and Dr. Pankaj Sharma (GM at Research Centre of a
multinational pharmaceutical company) for their patience in listening to our
concepts and ideas and for healthy criticism of the topics of their interest. We are
also indebted to Dr. Hema Rawat
(a postgraduate in medical sciences) for her valuable suggestions related to drugs
(in Chapter 29) and for her help in simplifying the specialized text for easy
understanding by students without diluting the meaning of medical
terminologies. We are thankful to our ‘computer savvy’ nephew Mayank Mehta
for his assistance in the preparation of initial draft of the manuscript.
We owe a great debt of gratitude to the staff of PHI Learning, Delhi. Their
unwavering support and enthusiasm helped make the book a reality. Further, we
appreciate the extraordinary efforts of Mr. Deepak Pandey of the production
department for working out an excellent representation of our ideas in the second
edition.
Finally, we are grateful to our colleagues and students — our interaction with
them has been a constant source of inspiration for the creation of this text. We
acknowledge our debt to everyone who helped and encouraged us.
BHUPINDER MEHTA
MANJU MEHTA
About the Authors

Dr. BHUPINDER MEHTA and Dr. MANJU MEHTA (husband and wife)
Enjoying the moments after accomplishing the venture of second edition.
Bhupinder Mehta received his bachelor’s degree in Chemistry (Honours) in
1981, Master’s degree in 1983 and Ph.D. in 1988, all from University of Delhi.
He was awarded National Merit Scholarship during his postgraduate studies and
later awarded fellowship by the Council of Scientific and Industrial Research,
New Delhi, for pursuing research. His main research work is in the field of
sulfur and nitrogen containing heterocyclic compounds.
He started his teaching profession in 1988 and at present is Associate professor
in Department of Chemistry, Swami Shraddhanand College, University of Delhi.
He has over twenty seven years of experience of teaching organic chemistry.
Manju Mehta, née Rawat received her bachelor’s degree in Chemistry
(Honours) in 1987, Master’s degree in 1989 and Ph.D. in 1993, all from
University of Delhi.
She was declared by the Central Board of Secondary Education, among the top
one per cent students (26th rank) in her Senior School Examination in 1984. She
was awarded Undergraduate Science Merit Award by University of Delhi and
later awarded fellowship by University Grants Commission, New Delhi, for
pursuing research. Her main research work is in the field of sulfur and nitrogen
containing heterocyclic compounds.
She started her teaching profession in 1993 and at present is Associate professor
in Department of Chemistry, Maitreyi College, University of Delhi. She has over
twenty two years of experience of teaching organic chemistry.
What This Book Is About

The second edition of the book continues to offer a range of pedagogical


features maintaining the balanced approach of the text.The attempts have been
made to further strengthen the conceptual understanding by introducing more
ideas and a number of solved problems.
Comprehensive in approach, this text presents a rigorous treatment of organic
chemistry to enable undergraduate students to learn the subject in a clear, direct,
easily understandable and logical manner. Presented in a new and exciting way,
the goal of this book is to make the study of organic chemistry as stimulating,
interesting, and relevant as possible.
Beginning with the structures and properties of molecules, IUPAC nomenclature,
stereochemistry, and mechanisms of organic reactions, proceeding next to
detailed treatment of chemistry of hydrocarbons and functional groups, then to
organometallic compounds and oxidation-reduction reactions, and ending with a
study of selected topics (such as heterocyclic compounds, carbohydrates, amino
acids, peptides and proteins, drugs and pesticides, dyes, synthetic polymers and
spectroscopy), the book narrates a cohesive story about organic chemistry.
Transitions between topics are smooth, explanations are lucid, and tie-ins to
earlier material are frequent to maintain continuity.
The book contains over 500 solved problems from simple to really challenging
ones with suitable explanations. In addition, over 275 examples and solved
problems on IUPAC nomenclature, with varying levels of difficulty, are
included.

About some key features of the book


l EXPLORE MORE: Four sets of solved problems provide in-depth knowledge
and enhanced understanding of some important aspects of organic chemistry.
l MINI ESSAYS: Three small essays present interesting write-ups to provide
students with introductory knowledge of chemistry of natural products such as
lipids, terpenes, alkaloids, steroids along with nucleic acids and enzymes.
l NOTABILIA: Twenty-two ‘notabilia boxes’ interspersed throughout the text
highlight the key aspects of related topics, varying from concepts of chemistry
to the chemistry related to day-to-day life.
l STRUCTURES AND MECHANISMS NOT IN ORDER: Cites examples of
common errors made by students while drawing structural formulae and
displaying arrows in reaction mechanisms and helps them to improve on
language of organic chemistry by teaching appropriate drawings and their
significance.
l GLOSSARY: Includes ‘Name reactions’, ‘Reagents’, and some important

terms for quick revision by students.


Clearly written and logically organized, the authors have endeavoured to make
this complex and important branch of science as easy as possible for students to
learn from and for teachers to teach from.
Chapter1
Organic Molecules: Structure,
Bonding and Properties

1.1 ORGANIC CHEMISTRY—AN


INTRODUCTION

In earlier days, the branch of chemistry, devoted to the study of carbon


compounds, whose source was related to the living system, was termed as
organic chemistry (organic means life). Essentially, all the chemical reactions
that take place in the living system are organic reactions. This is because the
molecules of life such as proteins, vitamins, lipids, carbohydrates, chlorophyll,
haemoglobin, nucleic acids, and so on contain carbon. In fact, our dependency
on nature for food, clothing (cotton, wool, and silk) and energy (natural gas and
petroleum) is, basically, the dependency on organic compounds.
Friedrich Wohler carried the very first synthesis of an organic compound,
called urea, way back in 1828, by heating ammonium cyanate (a compound from
non-living source).

The synthesis of urea marked the beginning of a new era in organic chemistry,
thereby, starting off the process of identification of natural products and their
synthesis. Further, with the advancement in chemistry, a variety of new organic
compounds, catering the needs of mankind, were synthesized. This endless list
of compounds include plastic, rubber, fiber, medicines, dyes, agrochemicals, and
so on. The synthesis of natural and synthetic organic compounds has become a
continuous process and each day new compounds are added to the literature of
organic chemistry.
This development needed a new definition of organic chemistry, which is not
restricted only to the carbon compounds of living system but also includes, the
synthetic organic compounds under its umbrella. Thus, the simplest definition of
organic chemistry is the study of compounds that contain carbon, that is it is
chiefly, the study of compounds where carbon is covalently bonded to carbon,
hydrogen, halogens, oxygen, nitrogen and sulfur. The following text is devoted
to the study of structure, bonding and physical nature of these classes of organic
compounds.
1.2 ELECTRONIC STRUCTURE AND CHEMICAL
BONDING IN ORGANIC COMPOUNDS

We presume that you are already familiar with fundamentals of electronic


structure of atoms and therefore, the keypoints of electronic structure of an atom
are enumerated in this section.
(1) The simplest picture of an atom is represented by a nucleus (protons and
neutrons) with a positive charge surrounded by negatively charged
electrons. In a neutral atom, the number of electrons is equal to number of
protons. When two atoms approach each other, the electrons of one are
attracted by the nucleus of second atom and viceversa. Coulomb’s law best
describes the forces of attraction and repulsion and is given by

where, q1 and q2 are two charges separated by distance r.


If q1 and q2 are opposite charges then F is the force of attraction and if q1
and q2 are similar charges then F is the force of repulsion.
(2) Electrons occupy the region of space called principal energy levels,
identified by principal quantum numbers 1, 2, 3 and so on. Each principal
energy level can contain upto 2n2 electrons, where n is the principal
quantum number. Thus, the first energy level can contain 2 electrons,
second 8 electrons, the third 18 electrons, fourth 32 electrons and so on.
(3) Each principal energy level is subdivided into region of space called
orbitals; each orbital can contain maximum two electrons with spin +½ and
–½ (Pauli exclusion principle).
(4) The first principal energy level contains single orbital, called 1s orbital.
The second principal energy level contains one s orbital and three p orbitals;
these orbitals are called 2s, 2px, 2py, and 2pz. The third principal energy
level contains one 3s orbital, three 3p orbitals, and five 3d orbitals.
(5) A set of orbitals which have equivalent energies are said to be degenerate
orbitals. For example, three p orbitals are degenerate and similarly, set of
five d orbitals are also degenerate, as they are equivalent in energies.
(6) According to Hund’s rule, when a number of degenerate orbitals are
available, they are first filled singly and then pairing of electrons takes
place.
(7) The filling of electrons in atomic orbitals is done according to the Aufbau
Principle, that is, electrons occupy the orbitals successively in order of
increasing energy.
1.3 LEWIS STRUCTURE AND CHEMICAL BONDS

Lewis structure represents the valence electrons (electrons in the outermost


shell) of an atom by placing dots around the symbol of an atom, as represented
in Table 1.1.

The Lewis structure of a compound, also known as the electron-dot formula,


involves the representation of symbols of elements surrounded by dots, which
indicates the electrons taking part in the bond formation as well as the non-
bonding (or free) electrons. In accordance with Lewis model, during bond
formation each atom attains a complete valence shell (either two (as in He) or
eight electrons (referred as octet)) resembling that of its nearest noble gas
configuration. This can be achieved by transfer of electrons (ionic bond) or by
mutual sharing of electrons (covalent bond).

1.3.1 Electronegativity

Electronegativity is a measure of the tendency of an atom to attract the electrons


it shares with another atom. It is a periodic property, which increases from left to
right in a period and decreases on moving down a group. Fluorine is the most
electronegative element. Similarly, lithium and sodium with low
electronegativity values of 1.0 and 0.9 respectively are often termed as
electropositive elements. Table 1.2 illustrates the electronegativity values of
some elements.
Table 1.2 Electronegativity values (on Pauling scale) for selected elements
Element H Li Na C N O F Cl Br I
Electronegativity 2.1 1.0 0.9 2.5 3.0 3.5 4.0 3.0 2.8 2.5

1.3.2 Ionic Bond

An ionic bond is formed when atoms participating in bond formation involve


transfer of electrons from one atom to the other. The transfer of electron(s)
results in the formation of ions. The electrostatic force of attraction holds these
oppositely charged ions close together resulting in the formation of a strong
ionic bond. For example,

The ionic bond is formed between atoms having an appreciable difference it


electronegativities.

1.3.3 Covalent Bond

Two atoms which have same electronegativity or a small difference in


electronegativity involve mutual sharing of electrons to form a bond known as
covalent bond. In general, Lewis structure is used for the representation of a
covalent bond. The shared pair of electrons is represented by a dash (–) between
two atoms and represents a bond. The bond formed between atoms of similar
electronegativities is a non-polar covalent bond such as H2, Cl2 O2, N2, CH4,
and so on. The bond formed between atoms having a small difference in
electronegativities is a polar covalent bond such as HCl, H2O and so on. The
degree of ionic or covalent character can be predicted from electronegativities
of the involved atoms by the formula:

For example, the calculations using this formula (refer to Table 1.2 for
electronegativity) indicate that the covalent bond in HCl possesses 30% ionic
character.
Depending upon the number of shared electron pairs, the covalent bond may be
characterized as a single, double or triple covalent bond.

1.3.4 Atomic Radius, van der Waals Radius, Bond


Length, and Bond Angle

Atomic radius. The distance from the nucleus of the atom to the outermost
electrons is known as the radius of an atom. The atomic radius is the half of the
closest distance of the approach of the atoms in the structure of symmetrical
molecule. For covalent molecules such as H–H and Cl–Cl, the atomic radius is
often termed as covalent radius.
van der Waals radius. The distance from the nucleus of an atom beyond which
the approach of another atom causes repulsion is known as van der Waals radius
of that atom. If two atoms are brought closer than the sum of their van der Waals
radii, they repel each other and this phenomenon is known as van der Waals
repulsion.
Bond length. In a covalently bonded molecule, the distance between the nuclei
of the two atoms is known as bond length. Atomic radii and bond lengths are
measured in angstrom (1 Å = 10–10 m) units.
Bond angle. In a covalently bonded molecule having more than two atoms, the
bonds form an angle with each other, which is termed as bond angle.
Fig. 1.1 (a) Atomic radius and bond length in hydrogen; (b) van der Waals radii of atoms; (c) bond angles,
and bond lengths in methane and water molecule.

1.3.5 Formal Charge

The charge present on an atom, in a molecule or in an ion is known as formal


charge. The formal charge is calculated by taking into consideration, the number
of valence electrons of the Lewis structure as follows.

As an example, let us calculate the formal charge on oxygen in H3O+ and OH–.
The Lewis structure for these species are

1.3.6 Bond Polarity and Dipole Moment

In covalent bonds where two atoms have a difference in electronegativity, a


polarity develops. For example in HCl molecule, chlorine being more
electronegative attracts the shared pair of electrons nearer to itself and acquires a
partial negative charge ($delta$$–) and hydrogen a partial positive charge
($delta$$+).
These two separated opposite charges, which are a result of uneven distribution
of electrons, constitute a dipole. The dipole moment, $mu$$ is a quantitative
measure of polarity and is expressed in ‘Debye unit’ D.

The dipole moment is represented by $map$$, where head of the arrow is always
towards the more electronegative atom and tail is towards less electronegative
atom.

In a molecule, the dipole moment is the vector sum of all the individual bond
dipoles. The individual bond dipole may not be zero but their vector sum on the
whole may be zero since they cancel each other being vector quantities.

Dipole moment values give an idea about the structure of a molecule. A zero
dipole moment indicates that the molecule is symmetrical or linear.
1.4 CONCEPT OF HYBRIDIZATION AND
COVALENT BONDING

As mentioned earlier, an orbital describes the region in space which has the
maximum probability of finding the electrons. In accordance with the modern
theory of bonding, covalent bond formation between two atoms involves the
overlap of atomic orbitals. This overlap results in the formation of new orbitals
termed molecular orbitals.

As you already know that based on principal energy levels, the orbital may be s,
p, d, or f. In present text our emphasis will be on s and p atomic orbitals as
carbon (atomic number 6) involves atomic orbitals of second principal energy
level.

The structure of a molecule can be explained by using the concept of


hybridization. It is a theoretical concept, which states that atomic orbitals of an
atom may combine among themselves to form new set of orbitals called ‘hybrid
orbitals’ having equivalent energies. These new orbitals are highly directional
and interact with the orbitals of other atoms to form bonds. The molecule formed
by the overlap of these hybrid orbitals exhibits the structure and bond angle in
accordance with the observed data.
The structure of a molecule is directly related to the type of hybridization
exhibited by atom(s). The type of hybridization depends on the number and type
of atomic orbitals participating in hybridization and is summarized in Table 1.3.
1.4.1 sp3 Hybridization

Strucutre of methane (CH4)


Experimentally, it has been observed that methane (CH4), the simplest carbon
compound, has same bond length (1.09Å) for all the four carbon-hydrogen
bonds and has a bond angle of 109.5°. The ground state electronic configuration
of carbon has two unpaired electrons in the 2p orbital. In the excited state, one of
the 2s electrons moves to 2p orbital (see Fig. 1.2).

Fig. 1.2 Excitation and sp3 hybridization of the atomic orbitals of carbon.

The four atomic orbitals of carbon which differ in their energies (one 2s and
three 2p orbitals), undergo a sort of intermixing to form four new hybrid orbitals
having equivalent energies. The new hybrid orbitals are highly directional in
nature and are more effective in overlapping compared to pure atomic orbitals.

The four hybrid orbitals are arranged along the sides of a regular tetrahedron
(bond angle 109.5°) to have minimum electronic repulsions.

Fig. 1.3 Tetrahedral arrangement of four sp3 hybrid orbitals of carbon and formation of methane.

The four sp3 hybrid orbitals overlap with 1s orbital of four hydrogens to form
four carbon-hydrogen $sigma$$ bonds (sp3-s overlap) with same bond lengths
(1.09 Å) and same bond angle. The methane molecule is tetrahedral in shape.
Structure of ethane (C2H6)

In ethane, two carbons undergo sp3 hybridization. Each carbon has four sp3
hybrid orbitals arranged in tetrahedral manner, that is, in all, eight hybrid orbitals
are available. The overlap of two sp3 hybrid orbitals (one from each carbon)
results in the formation of a carbon-carbon $sigma$$ bond (sp3-sp3overlap).
The remaining six sp3 hybrid orbitals overlap with 1s orbitals of hydrogens to
form six carbon-hydrogen $sigma$$ bonds (sp3-s overlap).

Fig. 1.4 Structure of ethane showing overlap of sp3 hybrid orbitals of carbons.
1.4.2 sp2 Hybridization

Structure of ethene (C2H4)


The combination of one 2s and two 2p orbitals of carbon results in the formation
of three sp2 hybrid orbitals. These hybrid orbitals have trigonal planar
arrangement (bond angle close to 120o) to minimize electronic repulsions. In
case of ethene, the two carbons are sp2 hybridized.

Fig. 1.5 Excitation and sp2 hybridization of the atomic orbitals of carbon.

Each carbon has three planar sp2 hybrid orbitals, that is, in all six hybrid orbitals
are available. The sp2-sp2 overlap results in C–C $sigma$$ bond formation and
sp2-s overlap results in four C–H $sigma$$ bonds (1.08 Å).
Fig. 1.6 Structure of ethene showing formation of $sigma$$ and $pi$$ bonds.

Each carbon has a pure p orbital (does not participate in hybridization) and
overlap of these two p orbitals results in the formation of a $pi$$ bond. Since
$pi$$ bond is formed by collateral (sideways) overlapping of p orbitals of
carbon, it is relatively weaker than $sigma$$ bond.
Structure of formaldehyde (CH2O)

In general, sp2 hybridization results in the formation of double bond. For


example, formaldehyde (H2C=O) where both carbon and oxygen are sp2
hybridized.

Carbon has three hybrid orbitals (each having single electron). Oxygen too, has
three hybrid orbitals (two having paired electrons and one with single electron).
The carbon-oxygen $sigma$$ bond is formed by the overlap of one of the sp2
orbitals of carbon and sp2 orbital of oxygen containing single electron. The
remaining two sp2 hybrid orbitals on carbon form two C–H $sigma$$ bonds by
overlaping with 1s orbitals of the two hydrogens. The hybrid orbitals of oxygen
contain two lone pairs of electrons. The overlap of pure p orbitals of carbon as
well as of oxygen forms a carbon–oxygen $pi$$ bond.

Fig. 1.7 Hybridization and formation of $sigma$$ and $pi$$ bond in formaldehyde.

1.4.3 sp Hybridization

Structure of ethyne (acetylene, C2H2)


The combination of one s and one p orbital results in the formation of two sp
hybrid orbitals. The two sp hybrid orbitals are linearly arranged (bond angle
180o) to have minimum electronic repulsion. In acetylene, the two carbons are
sp hybridized. Each carbon has two hybrid orbitals, the sp-sp overlap from each
carbon results in carbon-carbon $sigma$$ bond formation whereas sp-s overlap
results in the formation of two C–H $sigma$$ bonds. Each carbon has two pure
p orbitals (mutually perpendicular) and their overlap results in the formation of
two $pi$$ bonds.
Fig. 1.8 Hybridization and formation of $sigma$$ and $pi$$ bonds in ethyne.

Effect of Hybridization on Bond Length and Bond


Strength
The s character in hybrid orbitals increases as one moves from sp3

(25% s character) to sp2 (33% s character) to sp (50% s character).


The s orbitals are spherical and electrons are closer to nucleus. Thus,
the higher the s character of hybrid orbital, more effective its overlap
with the other atomic orbitals or hybrid orbitals is. This causes an
increase in bond strength and decrease in the bond length of the
system with an increase in ‘s’ character of overlapping orbitals.
Order of bond strength and bond length of C–H bond

Order of bond strength and bond length of C–C bond


1.5 WRITING THE STRUCTURAL FORMULA
FOR ORGANIC MOLECULES

The Lewis structure makes use of dots to represent bonding and nonbonding
electrons. Kekule proposed the simplest representation for organic molecules by
taking into consideration the valency of atoms. These chemical formulas were
represented by dash (—), which symbolizes bonding pair of electrons between
atoms and lone pair(s) of electrons is shown as pair of dots around the atom
concern in a molecule.

However, writing bigger structures using Kekule representation is time


consuming as well as space consuming. To avoid this, condensed formulas are
used where single bonds and lone pairs are omitted. Adding vertical lines shows
substituents present in a molecule.
The simplest space saving representation is bond-line formula (also known as
line angle formula), where carbon chain in organic molecule is represented by
zigzag lines and hydrogens are omitted. The terminal represents a methyl group
and each apex of this line represents a carbon. The hydrogen attached to each
carbon is understood taking in consideration the tetravalency of carbon. A
branched line represents a branched alkyl chain.
The following examples represent the various bond-line formulas along with
corresponding Lewis structures and condensed formulas.
The Table 1.4 give the condense formulas for some molecules and corresponding
Bond-Line formulas by taking the examples of variety of functional groups.

The lone pair(s) of electrons is not shown in the condensed and bond-line
formula unless or until it has a role to play in understanding the mechanistic
details in the reactions or explaining the other electronic factors associated with
concerned molecule.
1.6 CONCEPT OF RESONANCE (MESOMERISM)

The Lewis structure of a molecule or ion can be used for predicting the structural
properties and reactivity of that molecule. A number of molecules or ions can be
represented by more than one correct Lewis structures. For example, following
two Lewis structures can be written for nitrite ion (NO2–).

In accordance with these structures, the nitrite ion should have two different
nitrogen-oxygen bond lengths (N–O and N== ). Experimentally, it has been
observed that both nitrogen-oxygen bond lengths are same and their value lies
between that of a single and a double bond value. Any one of these Lewis
structures alone do not represent correctly the bonding in NO2– ion. The actual
structure is a hybrid of these two Lewis structures. The two Lewis structures are
known as resonance forms, resonance contributors, or contributing structures
and are represented by placing a double-headed arrow ($harr$$) between them.
These contributing structures are hypothetical structures. The actual resonance
hybrid structure cannot be represented by a Lewis structure and is shown by
placing dashes, which indicates delocalization of negative charge between two
oxygen atoms.

The energy of a resonance hybrid structure (a real structure) is


always less compared to energy calculated for any of the hypothetical
contributing structures and this difference in energy is termed as
resonance energy. High resonance energy indicates higher degree of
delocalization and thus, higher stability of the system.
Similarly for carbonate ion (CO32–), the contributing structures are represented
as follows:

Here, the negative charge is delocalized among all the three oxygens.

Rules for Writing Contributing Structures (Resonance


Forms)
• The contributing structures should be correct Lewis structures.
• Writing different contributing structures involves the movement of
electrons only, the nuclei or atoms hold the same position
throughout.
• The number of paired and unpaired electrons should be same in all
contributing structures.
• Different resonance forms do contribute towards the real resonance
hybrid structure, but not all of them contribute equally. In general,
(i) More the number of contributing structures, more stable is the
resonance hybrid structure.
(ii) Most stable resonance form contributes the most towards
resonance hybrid structure.

Stability of Contributing Structures (Resonance Forms)


• The structures with maximum number of covalent bonds are most
contributing towards hybrid structure.
• A structure with formal charges contributes less compared to neutral
structure.
• If different structures bear formal charges then a structure with
negative charge on an electronegative atom and positive charge on
an electropositive atom contributes most.
• Structures with similar formal charges on adjacent atoms are least
contributing.
• Structures with electron deficient positively charged atoms are least
contributing.
NOTABILIA 2
Let us write the different resonance forms for formate ion and discuss their
relative contribution.

Structure [I] and [II] are equivalent and contribute equally, however in structure
[III] the number of covalent bonds is three (less than [I] and [II] which have
four). Also, structure [III] has an electron deficient carbon with positive charge
and thus it is least contributing.
The concept of resonance provides an adequate explanation for electron
delocalization in certain molecules, ions and radicals. In general, delocalization
involves lone pair (non-bonding) of electrons and $pi$$-electrons (from multiple
bonds).
Delocalization of electrons during resonance is shown to occur in three ways:
1. Shifting of electrons from $pi$$ bond to adjacent bond.
In the following examples the shaded portion indicates the shifting of electrons
from $pi$$ bond to adjacent bond.
2. Shifting of electrons from $pi$$ bond to an immediately next placed atom.
In the following examples the shaded portion indicates the shifting of
electrons from $pi$$ bond to an immediately next placed atom.

3. Shifting of lone pair of electrons from an atom to an adjacent bond.


In the following examples the shadded portion indicates the shifting of lone
pair of electrons from an atom to an adjacent bond.
1.7 INTERMOLECULAR FORCES (VAN DER
WAALS FORCES)

The physical properties of organic compounds such as their existence as solids,


liquids, or gases, melting point, boiling point, solubility, and so on are attributed
to intermolecular forces of attraction. These forces are electrostatic in nature and
provide stability to the system. There are four major categories of intermolecular
forces, which are collectively known as van der Waals forces of attraction.
They are:
(i) Ion-dipole interaction
(ii) Dipole-dipole interaction
(iii) Hydrogen bonding
(iv) London forces
Ion-dipole interaction
The attraction between an ion and a polar molecule is termed as ion-dipole
interaction. The polar molecule exhibits charge separation and thus a positive
ion is attracted by negative end of the polar molecule and a negative ion is
attracted by positive end of the polar molecule. This interaction is weak
compared to interaction between two oppositely charged ions (ion-ion
interaction). The solubility of potassium chloride in water is an example of ion-
dipole interaction. In aqueous solution, the water molecules surround positive
potassium ions and the negative chloride ions as follows:

This ion-dipole interaction is also known as ion-solvation.


Dipole-dipole interaction
The forces of attraction between negative end of one polar molecule and positive
end of another polar molecule and viceversa are termed dipole-dipole
interactions.

Since charges on polar molecules are small, dipole-dipole interactions are


relatively weaker compared to ion-dipole interactions.
Hydrogen bonding
This is a special type of strong dipole-dipole interaction. The hydrogen attached
to an electronegative atom, like halogens, nitrogen, oxygen or sulfur, is highly
electropositive and can combine readily with negative end of polar molecule or
with molecules having a lone pair of electrons. The hydrogen bonding is shown
by dashed lines (| | | | | | | | | | |). The hydrogen bonding may occur between different
molecules (intermolecular) or within a molecule (intramolecular).

Both ether and alcohol are polar in nature. However, alcohols have higher
boiling point compared to ether because of intermolecular hydrogen bonding.

The lower alcohols are soluble in water. This is due to hydrogen bonding among
alcohol and water molecules (Intermolecular hydrogen bonding). However, with
an increase in the nonpolar hydrocarbon chain of alcohols, the solubility in water
decreases.
Compared to other intermolecular forces, the hydrogen bonding occurs through
specific atoms, (having high electronegativity) on adjacent molecules, for
example, hydrogen with oxygen, hydrogen with nitrogen, hydrogen with
fluorine, and so on. Hydrogen bonding interactions are strong compared to ion-
dipole and dipole-dipole interactions.
London forces
The intermolecular interactions, which are observed in nonpolar molecules, are
known as London forces. The electrons are in continuous motion and induce
temporary polarization in one molecule, which in turn induces polarization in an
opposite direction in adjacent molecule. This momentary induction of dipole
results in attraction between otherwise nonpolar molecules (induced dipole
interaction). The London forces are very weak intermolecular forces. The
attraction among nonpolar molecules of alkanes is attributed to London
forces.

Thus, the overall increasing order of strength for intermolecular forces of


attraction is as follows:

1.7.1 Melting Point and Boiling Point


Melting point. The temperature at which a solid gets converted to a liquid is
termed as the melting point of that solid. In solid state, the particles are arranged
in highly ordered manner (regular and symmetrical). On heating, the particles
acquire thermal energy and move to a random arrangement as in liquid. The
temperature at which thermal energy of particle overcomes the forces of
attraction holding them in an orderly arrangement is referred to as the melting
point.
In general, ionic compounds have much higher melting points compared to
covalent compounds. In ionic compounds, the strong electrostatic forces of
attraction hold the oppositely charged ions in a crystal lattice. To overcome the
strong forces, we have to break the ionic bonds between oppositely charged ions
and for that a very high temperature is required.
In covalent compound, the molecules are held together in crystal through weak
van der Waal forces of attraction (dipole-dipole or induced dipole interactions).
To overcome these forces, relatively low temperature is required. Unlike ionic
compounds, no bonds are broken in covalent compounds rather weakly held
molecules are to be separated from each other. In general, the melting point of
organic compounds increases with an increase in molecular mass.
Boiling point. The temperature at which vapour pressure of a liquid equals the
atmospheric pressure is termed as the boiling point of that liquid. In other words,
the somewhat orderly arranged molecules in liquid state escape from the surface
of liquid to highly random gaseous phase. On heating, the thermal energy
overcomes the forces of attraction between molecules and they escape to vapour
state. In organic compounds, the boiling point increases with an increase in
molecular mass. The hydrogen bonding in an organic molecule increases the
boiling point tremendously as the forces of attraction increase. For example,
ethanol (boiling point 78°C) has higher boiling point compared to dimethyl ether
due to the presence of intermolecular hydrogen bonding.
1.8 PURIFICATION AND IDENTIFICATION OF
ORGANIC COMPOUNDS

The identification of an organic compound involves four basic steps.


1. Purification of the compounds. The organic compounds, which are procured
from natural sources or are obtained by synthesis, contain a lot of impurities.
The purification of an organic compound is one of the most important steps for
its characterization. The liquid compounds are purified by distillation process
and sometimes for complex mixture of liquids fractional distillation is used. If it
is a solid then repeated crystallization using a suitable inert solvent is carried out
for purification. One of the extensively used methods for purification of organic
compounds is chromatography. Chromatography is a technique, which is used
for isolation, separation, and purification of organic compounds.
2. Qualitative analysis. The pure compound is next analyzed for nitrogen, sulfur
and halogens by Lassaigne sodium fusion test. The method involves the
conversion of covalently bonded nitrogen, sulfur, or halogens present in the
organic compounds to corresponding water-soluble ions, in the form of sodium
salts. For this purpose, organic compound is fused with sodium metal and the
fused mixture is dissolved in distilled water. The resultant solution is knows as
Lassaigne’s extract and is used for qualitative analysis of nitrogen, sulfur and
halogens present in organic compounds.
The nitrogen, if present, gets converted into sodium cyanide, which on reaction
with ferrous sulfate and ferric ions gives Prussian blue colour.

Sulfur, if present, gets converted into sodium sulfide, which on reaction with
lead acetate in acetic acid gives black precipitate.
The halogens, if present, in organic compounds are converted into corresponding
water-soluble sodium halides, which on reaction with silver nitrate give
corresponding silver halide precipitates.

3. Quantitative elemental analysis. (Carbon, Hydrogen, Nitrogen, and


Halogens). The qualitative analysis tells about the elements present in an organic
compound. The quantitative analysis is finally carried out to determine the
proportions in which different elements are present in an organic compound.
Different methods for the quantitative analysis of C, H, N, and X are discussed
here.
Determination of carbon and hydrogen by Liebig-Pregl method. An accurately
weighed sample of an organic compound is heated at 600–800°C, with excess of
oxygen, in presence of copper oxide. The complete combustion results in the
formation of carbon dioxide and water. The amount of water and carbon dioxide
is determined by passing the effluent gases through pre-weighed tubes
containing magnesium perchlorate and soda-lime, respectively. Magnesium
perchlorate, a dehydrating agent, absorbs water vapours whereas soda-lime
absorbs carbon dioxide.
Determination of nitrogen
By Dumas method. An accurately weighed sample of organic compound is
heated in the presence of copper oxide. This oxidizes the compound and
converts the combined nitrogen to nitrogen gas. The effluent gases are then
passed over hot copper gauze, which decomposes the oxides of nitrogen that
may be formed, to nitrogen gas. The volume of nitrogen gas produced is
measured, from which the percentage of nitrogen in the sample is calculated.
By Kjeldahl method. A known amount of sample of organic compound is
digested with concentrated H2SO4, which converts combined nitrogen to
ammonium sulfate. The solution is then heated with strong alkali to liberate
ammonia. The amount of ammonia liberated is treated with a known volume of
standard solution of H2SO4. The unreacted H2SO4 is estimated by titrating
against a standard alkali solution. Thus, one can calculate the percentage of
nitrogen in the sample. The method is used mainly for estimation of nitrogen in
foodstuff, fertilizers, and manure.
The Dumas method is a high precision method and is generally preferred over
Kjeldahl’s method.
Determination of halogens by Carius method. An accurately weighed sample of
organic compound is heated with fuming nitric acid in a sealed tube. This
converts the covalently bonded halogens to halide ions. The treatment with silver
nitrate forms silver halides, which are weighed to calculate the percentage of
halogens in organic compounds.
This method can be used for estimation of sulfur and phosphorous except that
treatment with AgNO3 is not carried out. The sulfur and phosphorous are
analyzed from H2SO4 and H3PO4 formed during the treatment of sample with
fuming HNO3.
4. Calculation of empirical formula. Empirical formula indicates the relative
number of different kinds of atoms in a molecule. The quantitative analysis gives
the percentage of different elements present in a compound. The following
example explains the calculation of empirical formula from the given data.
Illustrative Example 1. The quantitative analysis of an organic compound gave
50.7% carbon, 7% hydrogen, and 19.7% nitrogen. What is the empirical formula
of the compound?
Solution. The empirical formula is calculated as follows:
The percentage of C + H + N = 50.7 + 7 + 19.7 = 77.4%. The rest is oxygen, that
is, 100 – 77.4 = 22.6%. Now from these percentages of elements, number of
moles of each element is calculated by dividing percentage by atomic mass. As
the number of atoms cannot be in fraction so the simplest whole number ratio is
calculated by dividing all the values with the lowest value of number of moles.
This gives the empirical formula of organic compound. The result of the
calculation is tabulated as follows.
Hence, the empirical formula is = C3H5NO.
5. Determination of molecular mass and molecular formula. This can be
determined by various methods, but the most commonly used method is vapour
density measurement.
The molecular mass of substance = Vapour density of substance $times$$ 2
The molecular formula indicates the actual number of different kinds of atoms
present in a molecule. If molecular mass is known, the molecular formula can be
calculated as follows:

Illustrative Example 2. The quantitative analysis of an organic compound gave


40% carbon and 6.7% hydrogen. The vapour density of compound is 30.
Calculate the empirical formula and molecular formula of the compound.
Solution. The empirical formula is calculated as follows:
The percentage of C + H = 40 + 6.7 = 46.7% and as the rest is oxygen, the
percentage of oxygen is 100 – 46.7 = 53.3%.

Hence the empirical formula is CH2O.


The empirical formula mass = 12 + 2 + 16 = 30.
The molecular mass = Vapour density $times$$ 2
= 30 $times$$ 2 = 60
The molecular formula = [Empirical formula]n
where, n = molecular mass/empirical mass
= 60/30 = 2
Hence, molecular formula = [CH2O]2 = C2H4O2.
Once molecular formula of a compound is known, its structure can be
determined by various chemical and physical methods. One of the most
important, convenient and precise methods is spectroscopy, which is discussed,
in detail, in Chapter 32.
1.9 CLASSIFICATION OF ORGANIC
COMPOUNDS

Organic compounds based on certain characteristics are classified into the


following categories.

Acyclic compounds
The linear or branched, open chain compounds of carbon are referred as acyclic
or aliphatic compounds. These compounds may be saturated (containing single
bonds) or unsaturated (containing double or triple bonds).
Cyclic compounds or ring compounds
These compounds have their atoms arranged in cyclic manner. They may be
divided into following two classes:
(a) Carbocyclic (or isocyclic) compounds. In these compounds, the ring
consists of carbon atoms only. This includes alicyclic and aromatic compounds.
Alicyclic compounds may be saturated, e.g. cycloalkanes or may be unsaturated,
e.g. cycloalkenes.
(b) Heterocyclic compounds. In these compounds, the ring consists of other
atoms in addition to carbon atoms such as, O, S, N, and so on which are referred
to as heteroatoms.
The replacement of one or more hydrogen atoms in a hydrocarbon with other
atoms or group of atoms gives rise to a characteristic structural unit known as
functional group. A functional group defines the class and reactivity of an
organic compound. Some of the functional groups are listed in Table 1.5.
1.10 ISOMERISM IN ORGANIC MOLECULES

The compounds having same molecular formula are called isomers. Earlier, we
have discussed that carbon is tetravalent in nature and it may form single,
double, or triple covalent bonds with other atoms. Different compounds of
carbon differ because of the way in which the constituent atoms are covalently
bonded to each other. At the same time, two compounds show different
properties even if they have same atoms or groups present and have same
covalent bonding. How can one account for these observations? Two important
aspects describe the structure of a molecule, namely.
Bond connectivity. It describes the sequence in which different atoms or groups
are bonded to each other in a molecule. The isomers that differ in their bond
connectivity are known as constitutional isomers (formerly *structural isomer).
Spatial arrangement. It describes the three-dimensional arrangement of atoms
or groups in a molecule, that is, configuration of a molecule. The branch of
chemistry dealing with the study of three-dimensional nature (spatial
arrangement) of molecules is known as stereochemistry. Chapter 3 is
completely devoted to stereochemistry which is one of the most important
aspects in the study of organic chemistry.
The isomers may be categorized as:
The discussion in this chapter is restricted to constitutional isomers only.

1.10.1 Constitutional Isomers (Formerly Structural


Isomers)

Constitutional isomers have same molecular formula but different bond


connectivity, that is, the sequence in which atoms are bonded in a molecule, is
different. These are further classified as
Chain isomers. These isomers differ in the way the carbon atoms are bonded to
each other in a carbon chain. For example, butane and 2-methylpropane have
molecular formula C4H10 but they differ in their carbon skeleton.

Position isomers. These isomers differ in the position of functional group in a


carbon chain, for example, propan-1-ol and propan-2-ol. Similarly, 1-
bromobutane and 2-bromobutane are also position isomers.

Functional isomers. These isomers have same molecular formula but they differ
in the nature of functional groups, for example, alcohols and ethers with same
molecular formula. Similarly, aldehydes and ketones with same molecular
formula are also functional isomers. Carboxylic acids and esters also represent
functional isomers.
Metamers. These isomers have same functional group but they differ in the
arrangement of alkyl groups around the functional group. For example,

Tautomers. These are the two forms of same compound, which arise due to
migration of a hydrogen atom in a compound. The two forms are readily inter-
convertible and exist in dynamic equilibrium with each other. This phenomenon
is known as tautomerism. For example,
(a) keto-enol tautomerism

(b) Nitro-aci tautomerism

(c) Amido-amidol tautomerism


1.10.2 Resonance versus Tautomerism
Resonance is a mathematical concept based on quantum mechanical approach.
As discussed earlier (refer section 1.6) the resonance indicates that actual
structure of molecule is represented by a hybrid of different structures known as
contributing structures (resonance structures). The concept of contributing
structures is hypothetical and they do not exist in reality. In fact the actual
molecules do not oscillate back and forth between different resonating structures.
The contributing structures are not the isomeric forms. In contrast the tautomers
are real structures, which are related as isomeric forms and two tautomeric
forms exists in equilibrium.
Resonance involves the displacement of $pi$$-electrons or lone pair of electrons
only. The tautomerism involves the displacement of hydrogen atom.
In writing contributing structures, the double-headed arrows ($harr$$) are placed
between different structures. On the other hand, in writing tautomeric structures
(or forms), we place double arrows (), between two tautomeric forms.
1.11 ACIDS AND BASES

Organic molecules may either behave as an acid or a base which is explicitly


discussed at various places in the text. Here, we give a brief discussion on the
concept of acids and bases.

1.11.1 Bronsted and Lowry Definition


An acid is defined as a proton donor and base as a proton accepter. In a
reaction, an acid looses a proton to form a conjugate base whereas a base
accepts a proton to form conjugate acid. For example,

A strong acid has a weak conjugate base and weak acid has a strong conjugate
base. Similarly, a strong base has a weak conjugate acid and a weak base has a
strong conjugate acid.
pKa and pKb values
The relative strength of acids or bases is expressed in terms of their dissociation
constant values Ka and Kb or in terms of pKa and pKb values.
For an acid HA, the dissociation constant, Ka is expressed as

Larger the value of Ka, the stronger is the acid, that is, Ka $prop$$ acidic
strength.
The negative logarithm of Ka is expressed as pKa.
pKa = –log Ka
Higher the value of pKa, lower is the strength of acid. This can be expressed

as
In a similar way, for a base B, the dissociation constant Kb is expressed as:

Larger the value of Kb, the stronger is the base, that is, Kb $prop$$ basic
strength
The negative logarithm of Kb is expressed as pKb
pKb = –log Kb
Higher the value of pKb, lower is the strength of base, i.e. weak base. This
can be expressed as

1.11.2 Lewis Definition


It is general concept which is used to define acid and bases as substances which
provide or take up electron pair to form a covalent bond.
A Lewis acid is defined as an electron pair acceptor while a Lewis base is
defined as an electron pair donor.
The examples of Lewis acid include H+, BF3, ZnCl2, AlCl3, FeCl3, SnCl4 etc.
These species have unfilled valence orbitals and thus can accept electron pair.
EXERCISES**

1. Draw the Lewis structures of the following molecules/ions and calculate the
formal charge on the atoms indicated in bracket.
(a) H2SO4 (sulfur)
(b) HSO4– (oxygen)
(c) NH4+ (nitrogen)
(d) NH2– (nitrogen)
(e) CO2 (carbon)
(f) CH3OH (carbon)
(g) CH3O– (oxygen)
(h) CH3COO– (oxygen)
2. In the following molecules, indicate the positive and the negative end of the
dipole using the symbol ($map$$ ).
(a) CH3Cl
(b) H2O
(c) CCl4
(d) CO2
3. In the following molecules, indicate the bond angle about the indicated atom.

4. Assign orbital hybridization for the indicated atoms in the following


molecules.
5. What is the hybridization of carbon-oxygen bond (in bold) in the following
molecules?

6. In the following pairs of contributing structures, which one is more important


and has greater contribution?
7. How many types of carbon–carbon $sigma$$ bonds are present in CH3–
CH2–C$equiv$$CH.
8. Describe the orbital hybridization in HC$equiv$$N.
9. What are intermolecular forces? Describe the different types of
intermolecular forces and indicate, which one of them is the strongest?
10. Two compounds A and B have pKa values 7.9 and 3.9 respectively. Which
one is a stronger acid?
11. What are Lewis acids and Lewis bases? Can a species behave both as a
Lewis acid and a Lewis base?

Answers to selected exercises


1. (a) 0; (b) –1; (c) +1; (d) –1; (e) 0; (f) 0; (g) –1; (h) –1
3. (a) 120°; (b) 180°; (c) 109.5°; (d) 120°
4. I. a = sp3, b = sp3; II. a = sp2, b = sp3; III. a = sp, b = sp;
IV. a = sp2, b = sp2, c = sp3; V. a = sp2, b = sp, c = sp2.
5. (a) sp3-sp3; (b) sp2-sp2; (c) sp3-sp3.
6. (a) I , No formal charge.
(b) II, Negative charge on electronegative oxygen.
(c) II, More number of covalent bonds.
(d) I, More number of covalent bonds.
(e) I, No formal charge.
(f) I, Negative charge on electronegative oxygen.
7. Three types; sp-sp, sp-sp3, sp3-sp3.
10. B (low pKa indicates high acidic strength).
* The term ‘Structure’ is used in a broader sense involving not only the constitution but also the
configuration, that is, spatial arrangement and thus, the term structural isomer is no longer used.
** Answers to selected exercises are given at the end.
Chapter2
IUPAC Nomenclature of Organic
Compounds

2.1 INTRODUCTION
Each and every object in this universe is identified by its name. When organic
chemistry was in stage of infancy and not many compounds were known, the
compounds were named based on their sources or the name of their discoverer.
These names are called common or trivial names, a few such examples are listed
below in Table 2.1.

With a limited number of compounds, it was easy to identify them by their


common or trivial names but as evident from the table, common names do not
provide any indication for writing the structure of a compound. As more and
more new compounds were synthesized and procured from nature, a common
concern grew to name them (in 1892) in correlation with their structures.
A set of rules was thus, formulated by IUPAC (International Union of Pure and
Applied Chemistry) for systematic nomenclature of organic compounds which is
revised from time to time. This chapter describes the IUPAC nomenclature
system in accordance with the latest recommendations for various classes of
organic compounds.
2.2 IUPAC NOMENCLATURE
The IUPAC name of any organic compound essentially consists of three parts
which can be described as follows.
Stem name. It is the basic unit of the systematic name which describes the
carbon skeleton. It is also referred as the ‘word root’. The first four stem names
were chosen from the trivial system, for example, but-, which appears in butyric
acid. The stem name for five or more carbon atoms are derived from Greek
numerals and a few of them are listed in Table 2.2.

Prefix. The stem name may have certain groups or atoms attached to it which
are termed substituents. The names for these substituents are prefixed (added
before) to the stem name. Some of the substituents are listed in Table 2.3 and
Table 2.4. If a molecule contains more than one functional group than as per the
priority order, lower priority group is treated as substituent and written as a
prefix, as mentioned in the Table 2.5.
Suffix. A suffix denotes the functional group present in a compound and is
always added after the stem name (word root) as mentioned in Table 2.5.
2.2.1 Rules for Naming the Organic Compounds
IUPAC has laid down certain rules to be followed while naming different classes
of organic compounds. This subsection contains a detailed description of these
rules.
Alkanes
The procedural steps for the IUPAC nomenclature of alkanes are as follows:
1. For unbranched alkanes, nomenclature is done by adding suffix -ane to the
stem name. The stem name indicates the number of carbons in a compound
refer Table 2.2).
A few selected examples of the IUPAC names for some straight chain
(unbranched) alkanes are as follows.
CH4 Methane CH3(CH2)7CH3 Nonane
CH3CH3 Ethane CH3(CH2)8CH3 Decane
CH3CH2CH3 Propane CH3(CH2)9CH3 Undecane
CH3(CH2)2CH3 Butane CH3(CH2)10CH3 Dodecane
CH3(CH2)3CH3 Pentane CH3(CH2)18CH3 Eicosane
CH3(CH2)4CH3 Hexane CH3(CH2)19CH3 Heneicosane
CH3(CH2)5CH3 Heptane CH3(CH2)28CH3 Triacontane
CH3(CH2)6CH3 Octane CH3(CH2)98CH3 Hectane
For branched chain alkanes, the rules for naming are:
2. Longest possible continuous chain (parent chain) of carbon atoms is first
selected and the compound is named as a derivative of alkane
corresponding to the number of carbon atoms in the selected chain.
3. Selected chain is then numbered from the end which gives the lowest
number to the carbon atom carrying the alkyl group. For numbering, we use
Arabic numerals 1, 2, 3,… and so on.
4. The position of alkyl group in the chain is designated by the number given
to carbon in the chain, to which it is attached. The number (locant) is
written before the alkyl group. In this system, entire name is written as a
single word. A hyphen is used between a number and the word, and a
comma is used between two numbers. The use of n-, iso-, neo-, tert, sec-
and so on, is avoided as far as possible.
For example,
The wrong way of numbering of the carbon chain is indicated by encircled arabic numerals and the wrong
name given to organic compound is indicated by encircled ‘Not’ throughout the text.
5. The different alkyl groups, present in a compound, are written in an
alphabetical order, irrespective of their position in the carbon chain.

6. If two or more similar alkyl groups are present in a compound, the words
di, tri, tetra, and so on are used to specify the number of times these alkyl
groups appear in the chain. However, these prefixes themselves are not
considered for writing the alphabetical order. For example,

7. If two different alkyl groups are present at an equal distance from either end
of selected chain, the alkyl group which comes first in the alphabetical
order gets the lowest possible rank. For example,

8. If more than two alkyl groups are present and branching occurs at an equal
distance from either end, the numbering is done from the end which gives
minimum possible number to all the substituents. For example,

9. If the substituent is a branched chain alkyl group then it is numbered


separately. The numbering of the branched substituent starts from the point
of attachment to parent chain. The branched chain alkyl group is always
written in a bracket as a prefix.

Alkenes
The basic rules for naming alkenes are similar to those for alkanes but include
the indication for position of double bonds. Additional rules pertaining to
alkenes can be summarized as follows:
1. The longest possible continuous carbon chain should include the carbons of
double bond.
2. The longest chain is given the name by replacing suffix -ane (in case of
alkanes) with
-ene.
3. The numbering is done in such a way that first carbon of double bond gets
the lowest number.
4. The carbon atoms of the double bond get the preference over the other
substituents present in the parent chain.
5. In an alkene, the position of double bond is indicated by placing the number
before the suffix -ene (refer Notabilia 3).

6. If more than one double bond is present, the location of each double bond is
indicated just before the suffix. The suffix used, for two double bonds is -
diene, for three double bonds it is -triene, and so on. The general name for
dienes is ‘Alkadiene’.
[Note: While naming the organic compounds, two vowels are never placed together.
But, in alkadiene and alkatriene vowel a is separated from ‘e’ (of -ene) by -di and -tri,
thus ‘a’ is retained in the name.]
Alkynes
1. In alkynes, the longest possible chain should include the carbons of triple
bond.
2. The naming of alkyne is done by replacing the suffix -ane of the
corresponding alkane with -yne.
3. Lowest numbering is given to first carbon of the triple bond.
Other rules are same as those for alkanes. A few examples of alkynes are as
follows.

4. If a compound contains two triple bonds, it is named as Alkadiyne. The


position of triple bonds is indicated by writing numbers just before -diyne.
For example,
Cyclic aliphatic hydrocarbons
1. The aliphatic cyclic hydrocarbons are named by prefixing cyclo- to alkane.
2. If cyclic structure contains the alkyl group as substituent, it is named as
alkylcycloalkane.
3. If cyclic structure contains two or more alkyl groups then their position and
name are given by applying the rules (stated for alkanes) such as to follow
the alphabetical order in naming, lowest possible numbering to the
substituents and so forth.
4. If the cyclic structure is unsaturated, the carbon atoms bearing the double
bond get the priority over the other substituents present in the cyclic
structure.

5. If alkyl group has more number of carbon atoms than the cyclic structure,
the compound is named as cycloalkylalkane (cyclic structure becomes the
substituent). Otherwise the alkyl group is named as substituent of
cycloalkane.
Alkyl halides (RX)
1. Halogens are always treated as substituents and while naming, written as
prefix to the name of a compound.
2. The IUPAC name for alkyl halides is Haloalkane.

3. The carbon having halogen is given the lowest possible numbering.

4. If different halogens are present at an equal distance from either end then
numbering is done according to their alphabetical order.
A few examples of mono and polysubstituted haloalkanes are as follows:
Alcohols (ROH)
1. The selected longest continuous chain of carbon atoms must include
hydroxy group.
2. The alcohols are named by suffixing -ol for the -e of the corresponding
alkane. Thus alcohols are named as Alkanols.
3. The carbon bearing the hydroxyl group gets the lowest possible numbering.
For example,
4. Alcohols containing two or more hydroxyl groups are commonly known as
polyhydric alcohols and in IUPAC system, suffixes -diol, -triol, and so on
are used instead of -ol, to indicate the number of hydroxyl groups. In such
cases the vowel ‘e’ of alkane is retained in the name (see note on p. 41).
Thus, the general name for an alkane containing two hydroxyl groups is
Alkanediol.

5. The unsaturated alcohols are named by replacing -ane by suffix-enol (ene +


ol) or -ynol (yne+ol) of the corresponding alkanes.

Ethers (ROR′)
1. The general IUPAC name for ethers is alkoxyalkane.
2. The longest continuous carbon chain is named as alkane, whereas the chain
with lesser number of carbon atoms is named alkoxy and is always written
as substituent with the number specifying its position. For example,
3. The cyclic ethers are named in two ways. That is,
(i) Using epoxy as a prefix, which is commonly used for 3-membered cyclic
ethers.
(ii) As heterocyclic compounds. This is commonly used for higher membered
cyclic ethers. For example,
Naming the three membered cyclic ethers

Naming the higher cyclic ethers


The higher cyclic ethers are generally named as being heterocyclic systems. For
example,

Aldehydes (RCHO)
1. The longest continuous carbon chain contains —CHO group and the carbon
of —CHO group gets the lowest number.
2. The aldehydes are named by suffixing -al in place of -e of the
corresponding alkane. Thus in general, an aldehyde is named as Alkanal.
3. The chain is numbered from the aldehydic carbon but the position of the
aldehyde is not specified by number ‘1’, as it is understood that aldehydic
carbon is the terminal carbon.
4. Compounds with two aldehydic groups are named Alkanedial and so on,
where vowel ‘e’ of alkane is retained (p. 41).
5. The unsaturated aldehydes are named Alkenal (ene + al) and Alkynal (yne
+ al).
Rest of the rules for naming substituents remain the same as mentioned already.
For example,

Ketones (RCOR′)
1. The longest continuous carbon chain must contain ketonic group.
2. The ketones are named by suffixing -one in place of -e of the corresponding
alkane. Thus, the name Alkanone.
3. The carbon chain is numbered in such a way that ketonic group gets the
lowest number.
4. Further, numbering the positions of the substituents is carried out and these
are placed as prefix according to the rules explained earlier.
5. The compounds containing two ketonic groups are termed as Alkanediones
along with the position number of groups. The vowel ‘e’ of the alkane is
retained in this case.
6. The unsaturated ketones are called Alkenones (ene + one) or Alkynones
(yne + one). For example,
Carboxylic acids (RCOOH)
1. The longest continuous carbon chain must contain the carboxylic group
which gets the lowest number.
2. The naming of carboxylic acids is done by adding the suffix -oic acid in
lieu of -e of the corresponding alkane. Thus, the name Alkanoic acid.
3. The name of carboxylic acid is written as two words (unlike others, written
as single word).
4. The general name for the family of unsaturated carboxylic acids is
Alkenoic acid and Alkynoic acid.
5. In case of dicarboxylic acids, the longest chain should include both
carboxylic groups. The numbering starts from one carboxylic group and
terminates at the other. Thus, names of dicarboxylic acids are Alkanedioic
acid. It is written without specifying the position of the carboxylic group.
A few examples of carboxylic acids are as follows:
Acid derivatives (RCOCl, (RCO)2O, RCOOR′, RCONH2)
The acid derivatives are named in the following manner:
1. Acid halides are named as Alkanoyl halides.
2. Acid anhydrides are termed Alkanoic anhydrides.
3. Esters are named as Alkyl alkanoates, where alkyl refers to alcoholic part
and alkanoate refers to carboxylic part of the molecules.
4. Acid amides are named as Alkanamides.
5. The names of acid halides, acid anhydrides and esters are written as two
words, whereas the names for acid amides are written as a single word.
Some illustrative examples of IUPAC names for the acid derivatives are:

Alkyl cyanides (RCN)


1. Alkyl cyanide is named as Alkanenitrile.
2. The longest chain must include the carbon of cyanide group (–CN) which
gets the lowest number.
Alkyl isocyanides (RNC)
1. Isocyanides are named as Alkaneisonitriles (earlier they were referred to as
carbylamine derivatives).
2. The carbon of isonitrile is not included while selecting the parent chain. For
example,

Nitroalkanes (RNO2)
1. The nitro group is always treated as substituent and written as prefix along
with its position on the longest continuous carbon chain. For example,

Amines (RNH2)
1. The longest continuous carbon chain includes the carbon attached to –NH2
group.
2. Amines are named by suffixing -amine in place of -e of the corresponding
alkane. Thus, the general IUPAC name for amines is Alkanamine.
3. While numbering the parent chain, the lowest possible number is given to
the carbon attached to –NH2 group. For example,
4. In case of secondary amines, the longer chain is selected as a parent chain.
The other alkyl group is named as prefix and written as N-alkyl before the
parent chain. Thus, they are named N-alkylalkanamines.
5. In case of tertiary amines, the longest chain is selected as a parent chain and
other two alkyl groups are written as prefixes as N,N-dialkyl (if both alkyl
groups are the same) or as N-alkyl-N-alkyl (if both are different).
The examples of secondary and tertiary amines are as follows.

Thioalcohols (R-SH)
1. The IUPAC name for thioalcohol is Alkanethiol.
2. The lowest possible number is given to the carbon bearing the –SH group.
For example,

Sulfonic acids (RSO3H)


1. The longest continuous carbon chain includes the carbon having –SO3H
group.
2. The general IUPAC name is given as Alkanesulfonic acid.
3. The carbon having –SO3H group gets the lowest possible order of
numbering.

Organometallic compounds
The organometallic compounds are generally named by writing organic groups
as prefixes followed by the name of the metal without leaving space in between
the group and metal. In case anionic ligands are present in organometallic
compounds, they are named by writing the organic group followed by the name
of metal and then name of anion. The anion is written as a separate word. For
example,
2.3 SELECTED EXAMPLES OF
MONOFUNCTIONAL AND POLYFUNCTIONAL
ORGANIC COMPOUNDS
In this section, some typical examples are discussed that will help in applying
the rules studied for IUPAC nomenclature of organic compounds.
The general guidelines to be followed are as follows:
1. Select the longest possible carbon chain where the lowest possible number
is assigned to the principal functional group.
2. If more then one functional groups are present, select the principal
functional group in accordance with the preference order given in Table 2.5.
The other functional groups present are treated as substituents and these
functional groups are written as prefixes as given in Table 2.5.
3. The IUPAC name is always written in an alphabetical order irrespective of
the position of substituents.
Selected examples are given to explain the rules studied and wherever
neccessary suitable remarks are given along with.
Example 1

Remarks. When both the double bond and triple bonds are equidistant from both
the ends (Example 1b), preference is given to the double bond.

Remarks. Preference is given to triple bond as it is nearer to one end [refer (a)
and (b)] but name is always written as -enyne with their respective number
indicating positions.
Example 2. The following are the examples of bifunctional compounds, where
in accordance with the priority order (see Table 2.5), one of the functional group
is selected as principal functional group and the other functional group is treated
as a prefix (here indicated by shaded portion in the structure and name).

Example 3

Remarks.
(1) The prefix oxo is used when carbon of aldehyde group is a part of the
longest chain (Example 3a).
(2) The prefix formyl is used when –CHO cannot be included in the longest
continuous chain (Example 3b).

Remarks. Carbon of –C≡N is not a part of the parent chain as it is present as a


substituent (compare with the name of nitriles).
Example 4

Remarks
(1) Numbering is undesirable as two carboxylic groups are always at terminal.
(2) Vowel ‘e’ is retained in the name as ‘e’ and ‘o’ (of oic) are separated by di.
Example 5

Remarks. It is not possible to include all the three carboxylic groups in the
longest continuous chain. Since all the three carboxylic groups are the principal
functional groups, so naming one of the carboxylic groups as a prefix ‘carboxy’
would be wrong. In such a situation, an alternative
suffix (refer Table 2.5, column II) ‘carboxylic acid’ is used for all the three
groups, where
carbons of three carboxylic groups are not a part of the parent chain yet they are
the principal functional groups.

Remarks. Explanation for Examples 5(b) and 5(c) is similar to that given for
Example 5(a).
Example 6. In cyclic compounds the principal functional group can not be
included in the carbon chain and they are named by using alternative suffixes
(Table 2.5).

Example 7

Remarks. All the three –CN groups can not be expressed as principal functional
groups,
hence prefix cyano is used for one of the –CN groups as it cannot be included in
the longest continuous chain. The same explanation holds true for Example –7b
[compare with Examples
5(b) and (c)].
Example 8
Remarks. In Example 8(c), Iodo and methyl are equidistant, so numbering is
done from the end nearer to iodo, as iodo has alphabetical precedence over
methyl. [Compare with Examples 8(a)
and (b)].
Example 9

Remarks
(1) Similar groups are attached to nitrogen (N,N-) as well as carbon (C-3). So
the prefix used is N,N,3-trimethyl.
(2) Branched chain substituent present on nitrogen is written in bracket
[Example 9(b)].

2.3.1 Writing the Structure of an Organic Compound


from its IUPAC Name
Following steps are to be followed to write the structure of an organic compound
from its name. For example, consider 4-Chloro-5-methylheptan-3-ol
Step 1. Write the carbon skeleton as indicated by the parent chain name. In the
present case it is hept, a seven carbon skeleton. Now, count and number the
carbon skeleton from one end.

Step 2. The principal functional group is then identified and placed as indicated
by the name. In this case the principal functional group is alcohol (–OH), present
at third carbon.
Step 3. Rest of the substituents are placed at the indicated positions. In this case
the substituents are -chloro and -methyl, present at positions 4 and 5
respectively, on the carbon skeleton.

Step 4. Now, hydrogen(s) are placed to satisfy valency of the carbon skeleton.
Thus, the structure of organic compound can be written as:

Use of Greek numerals in naming organic compounds


A trivial but popular system of indicating the position of substituent(s) is the use
of Greek
numerals α-, β-, γ-, δ-, and so on. The carbon to which the functional group is
attached is
termed α-carbon and the subsequent carbons in the chain are termed β-, γ-, δ-
respectively. The substituents attached to these carbons are accordingly given the
position number α-, β-, γ-, δ-, etc. For example,
2.4 COMMON ERRORS IN WRITING IUPAC
NAMES
The most common errors observed in writing the IUPAC names of organic
compounds are:
(1) A wrong selection of longest carbon chain, that is, parent chain.
(2) Numbering from the wrong end for the substituents, present in the parent
carbon chain.
(3) Placing the substituents (prefixes) in an incorrect alphabetical order.
(4) Wrongly placed (or missing) comma(s) and hyphen(s).
(5) Not writing the name as single word (exceptions are specified in the text,
e.g. carboxylic acids).
(6) Two vowels (a, e, i, o, u) should never be placed together except when
they are separated by some other alphabet.
A few picks in the IUPAC nomenclature
Following examples are picked up from students notebooks. The students were
asked to write the IUPAC names for the given structures. These examples are
cited with a hope to make the very common errors more visible. Look carefully,
how, some of the learners write the IUPAC names for organic compounds (and
as a result loose ‘a few’ marks/grades in the examination).
Given structure (1)

Student’s answerbook
Correct name: 4-Ethyl-2,2-dimethylhexane
Given structures (2)

(Note: Both structures are same but written in different ways.)


Student’s answerbook
Correct name: 3-Ethylhexane
Given structure (3)

Student’s answerbook
Correct name: 3-Bromo-5-chloro-2,4-dimethyl-2-nitrohexane
Given structure (4)

Student’s answerbook

Correct name: Butanedioic acid


EXERCISES*
1. Give the IUPAC name for each compound.

2. Write the structural formula for each compound.


(a) Pent-2-yne
(b) 3-Hydroxyheptanoic acid
(c) N-Phenylbutan-1-amine
(d) 2-Aminoethanol
(e) 3-Oxobutanenitrile
(f) Ethanediazonium chloride
(g) 2-Methylpropanesulfonic acid
(h) 2-Methyl-5-nitrosohexane
(i) Cyclohexa-1,4-diene
(j) 2-Methylpropanoic anhydride
(k) Methyl cyclopentanecarboxylate
(l) Ethynylcyclohexane
3. Following names are not according to the IUPAC nomenclature system.
Indicate the error in the names and write the correct IUPAC names.
(Hint: First write the structures as per the name given and then assign the name
according to IUPAC rules.)
(a) Pent-4-yne
(b) 2-Chloro-3-bromobutane
(c) Prop-2-enal
(d) Hydroxymethyl methyl ether
(e) Hexa-1,6-diene
(f) N-ethyl-N-propylmethanamine
(g) Formylacetic acid
(h) Pent-2-en-4-one
(i) Pentane-1,5-dioic acid
(j) 2,3-dimethyl-4-(2-butyl)heptane
(k) 2-chloro-5-bromo-9-hydroxy-7-iodo-3-nitro-6-methyl-8-oxononaoic acid
ANSWERS
1. (a) 3-Hydroxybutanal
(b) 1-Bromo-3-chloro-2-fluoropropane
(c) 5-oxopentanoic acid
(d) 4-Oxopent-2-enal
(e) Pent-4-en-2-one
(f) 3-Aminopentanoic acid
(g) 4-Sulfanylbutanoic acid
(h) Hept-5-en-1-yne
(i) Butanedial
(j) 1-Iodo-2-methylcyclohexane
(k) Butane-1,2,4-triamine
(l) 4-Oxobutanenitrile
(m) 5-Bromo-2-chloro-9-hydroxy-7-iodo-6-methyl-3-nitro-8-oxononanoic
acid
(n) 4-Hydroxy-2-methylpentanoyl chloride
(o) Ethyl cyclohexanecarboxylate
(p) 2,5,5-Trimethylhepta-1,6-diene
(q) Cyclohexanecarboxamide
(r) 5-(-1-methylethyl)-3-methyloctane
(s) 4-Cyano-4-hydroxy-2-oxobutanoic acid
(t) 3-Methylenepentanedioic acid
(u) 2-Carbamoylethanesulfonic acid
(v) 4-Ethenylheptanoic acid

2.
3. Correct name (In brackets, hints for error rectification)
(a) Pent-1-yne
(Hint: Lowest number to triple bond.)
(b) 2-Bromo-3-Chlorobutane
(Hint: Numbering and naming as per alphabetical order.)
(c) Propenal
(Hint: Numbering is undesirable.)
(d) Methoxymethanol
(Hint: Parent compound is an alcohol with ether linkage as substituent.)
(e) Hexa-1,5-diene
(Hint: Carbons bearing double bonds get lowest possible numbers.)
(f) N-ethyl-N-methylpropanamine
(Hint: Named as a derivative of longest carbon chain.)
(g) 3-Oxopropanoic acid
(Hint: Aldehydic carbon is a part of longest carbon chain.)
(h) Pent-3-en-2-one
(Hint: Lowest number to the ketonic functional group.)
(i) Pentanedioic acid
(Hint: Numbering is undesirable.)
(j) 2,3,5-trimethyl-4-propylheptane
(Hint: Longest chain must include maximum number of substituents.)
(k) 5-Bromo-2-chloro-9-hydroxy-7-iodo-6-methyl-3-nitro-8-oxononanoic acid
(Hint: Name to be written in alphabetical order.)
* Answers to the exercises are given at the end. It is suggested that the readers first try themselves then
check the answers. Use prefixes and suffixes carefully (as given in tables in the text) and apply the rules
systematically.
Chapter3
Stereochemistry

3.1 INTRODUCTION
The isomers which have same bond connectivity but different arrangement of
groups or atoms in space are termed stereoisomers. The branch of chemistry
dealing with the study of three-dimensional nature (spatial arrangement) of
molecules is known as stereochemistry. Living organisms, their metabolic
activities, natural syntheses, and drug synthesis incorporate various aspects of
stereochemistry. Let us learn some facts about stereoisomers of different
compounds.
• D-glucose is metabolized by animals and fermented by yeast unlike L-
glucose.
• (–) Adrenaline has more hormonal activity than (+) adrenaline.
• (–) Nicotine is more poisonous than (+) nicotine.
• One stereoisomer of Caravone has caraway (spice similar to cumin seeds;
Jeera in Hindi) flavour while other isomer has flavour of spearmint.
• One stereoisomer of Limonene has smell of lemon while the other smells
like oranges.
• Of the several possible isomers of BHC, only one isomeric form has
insecticidal property, that is, gamma–BHC (or Lindane).
• All naturally occurring fatty acids have cis-configuration.
• One stereoisomer of monosodium glutamate (MSG) is used as an active
flavour enhancer (in Chinese recipes such as chowmein).
These facts clearly indicate that a difference in spatial arrangement may affect
the properties of isomers drastically.
The various aspects of stereoisomers will be discussed in context of their
following classification:
3.2 CONFIGURATIONAL ISOMERISM
The carbon is tetravalent in nature and the non-planar tetrahedral geometry in
carbon compounds gave birth to the concept of stereochemistry. The four atoms
(or groups) attached to a carbon are present in different planes as shown in Fig.
3.1 through the dashed-wedged-line structure, and ball and stick model. The
arrangement of different groups or atoms in space is termed as configuration.

Fig. 3.1 (a) Ball and stick model and (b) corresponding dashed-wedged-line structures for a tetrahedral
carbon atom joined to four different group.

3.2.1 Concept of Chirality [Asymmetry]


The term ‘chiral’ is taken from Greek word cheir meaning handedness (chiral
pronounced as ‘kiral’ and in rhyme with spiral). The human hands being
unsymmetrical, a person can shake his right hand only with the right hand of
another person but not with left hand of another person. Asymmetry is best
understood through the concept of an object and its mirror image which is
fundamental to stereochemistry. Consider Fig. 3.2 which depicts some objects
and their mirror images. Try to slide the objects over its image in the same
orientation. An object that is not superimposable on its mirror image is called
chiral and is unsymmetrical while the object that is superimposable (exactly
overlaps its image) on its mirror image is called achiral and is symmetrical. In
the present text when we refer to symmetry, we mean the plane of symmetry.
Fig. 3.2 Chirality as depicted (a) in an unsymmetrical object and (b) in human hand. [Both (a) and (b) have
nonsuperimposable mirror images.]
Thus, hands, gloves, ears, screw, spiral staircase are chiral (unsymmetrical)
where as fork, ball, nose, cup, socks, are achiral (symmetrical). Chirality is
depicted in the Fig. 3.3.
Fig. 3.3 Objects possessing planes of symmetry. Symmetrical objects are superimposable on their mirror
image.

3.2.2 Chirality in Organic Molecules: Enantiomers


and Diastereoisomers

The stereoisomers of a compound, which are non-superimposable mirror


images of each other, are termed as enantiomers (Greek; enantios-opposite). A
molecule with one stereogenic centre always exists in enantiomeric forms.
Fig. 3.4(a) Dashed-wedged-line structure and ball and stick model of chiral (unsymmetrical) molecule. Ball
and stick model is used to illustrate the non-superimposability in chiral molecules.
The stereoisomers of a compound, which are not mirror images of each other
are termed as diastereomers and discussed in subsequent section 3.2.4. The
diasteromers exist only when a compound has two or more stereogenic centres.
Meso compounds. A compound having stereogenic centres but optically
inactive (achiral), due to presence of a plane of symmetry, is termed as meso
compound or a meso form. In meso compounds the plane divides the molecule
into halves that are mirror images of each other. This internal compensation
makes meso compounds optically inactive.
Fig. 3.4(b) Dashed-wedged-line structure and ball and stick model of achiral
(symmetrical) molecule. Ball and stick model is used to illustrate the
superimposability in achiral molecules.

3.2.3 Fischer Projection

[Representation of Three Dimensional Structures in Two Dimensions]


The three dimensional structure of carbon compounds is due to their tetrahedral
geometry. Let us consider the dashed-wedged-line structure, and ball and stick
model for a carbon attached to four different atoms a, b, d, and e.
A simplified approach is to represent the three-dimensional structure (dashed-
wedged-line structure) in two dimensions making use of Fischer projection (Fig.
3.5). A Fischer projection is represented by an additive cross (+), where
horizontal line represents the bonds above the plane and vertical line represents
the bonds below the plane. The point of intersection of two lines represents the
chiral carbon (stereogenic centre) which lies in the plane of paper.

Fig. 3.5 The different representations for dashed-wedged-line structure [A] as shown by the ball and stick
model [B] and its equivalent structure [C]. The orientation of groups as shown in [C] are direct
representation of 2-Dimensional Fischer Projection [D].

The three-dimensional structure may be written in different ways and writing


these structures as corresponding Fischer projection is illustrated through Fig.
3.6.
Fig. 3.6 Illustrations depicting conversion of tetrahedral structures (having different orientations) to
corresponding Fischer projections.
Writing the Fischer projection
Step 1. The structure of a compound, whose Fischer projection is to be written, is
first written in a vertical manner. The carbon with the lowest rank, as per IUPAC
nomenclature, is written on the top.
Step 2. The chiral carbon in the structure is assumed to lie in the plane of the
paper while the groups attached vertically and horizontaly to chiral carbon are
assumed to lie below and above the plane of paper, respectively. For example,
for lactic acid CH3CH(OH)COOH, the Fischer projections are written as

Similarly, Fischer projections for 2-Hydroxypropanal are written as

The Fischer projection of a compound with more than one chiral centre can be
written in the same way using more than one horizontal lines. This is illustrated
through the Fischer projection of 2-bromo-3-chlorobutane as follows:
Step 3. A Fischer projection can be rotated through 180° in the plane of the paper
since this does not change the configuration, that is, spatial arrangement of
groups around chiral carbon. In this manner, same stereoisomer can be
represented in different ways. A rotation by 180° does not involve the
interchange between horizontal and vertical lines as visible in the Fig. 3.7.

Fig. 3.7 Rotation of Fischer projection by 180° does not change the configuration of stereoisomer. The
Fischer projection IA and IB (of lactic acid) represent the same stereoisomer. Similarly, IIA and IIB
(of 2-hydroxypropanal) represent the same stereoisomer.

3.2.4 Number of Stereoisomers of a Compound


The number of stereoisomers possible for a compound is related to the number
of chiral centres (n) present in it as
Number of stereoisomers = 2n
Where n is the number of stereogenic or chiral centre(s) present in the
compound.
Consider the case of lactic acid CH3CH(OH)COOH that contains one chiral
centre. It has two possible stereoisomers (21 = 2), and these are represented by
Fischer projections I and II as follows:
Both I and II differ only in their arrangement of atoms and groups in space, that
is, configuration and exhibit a non-superimposable object mirror image
relationship. The stereoisomers I and II are termed as enantiomers (Greek;
enantios-means opposite).
Next, consider 2-bromo-3-chlorobutane which has two chiral centres. It has four
possible stereoisomers (22 = 4)
Of which, the pairs I–II and III–IV (Fig. 3.8) depict non-superimposable object–
mirror image relationship and are referred to as pair of enantiomers.
A word about Threo and Erythro prefixes
While writing the Fischer projection of a stereoisomer having two
different chiral centres (may or may not be on the adjacent carbon
atoms), if two similar groups or atoms are on the same side of carbon
chain, the isomer is designated as erythro. On the other hand, if two
similar groups or atoms are on opposite sides of the carbon chain, the
isomer is designated as threo.
3.3 OPTICAL ACTIVITY
Ordinary light consists of light waves vibrating in all planes. When passed
through a ‘Polaroid lens’ or ‘Nicol prism’ (made up of calcite, a crystalline form
of calcium carbonate), light moves in one plane only and is called plane-
polarized light (PPL). An unsymmetrical or chiral compound, placed in the path
of plane-polarized light, will rotate the plane of light. The compounds exhibiting
such a behaviour are termed optically active.
A compound rotating the plane of polarization towards right (clockwise) is
termed dextrorotatory [Latin: dexter—right] whereas a compound rotating the
plane of polarization towards left (anti-clockwise) is termed laevorotatory
[Latin: laevus—left]. The dextrorotatory and laevorotatory compounds are
denoted by (+) and (–) respectively.
The optical activity is measured in the laboratory by polarimeter. A polarimeter
consists of a light source (usually D–line of sodium), two polaroid lenses (one
polarizer and one analyzer), and a sample tube placed between polarizer and
analyzer lenses (refer Fig. 3.10). If the sample tube is empty and polarizing axes
of two lenses are parallel, the intensity of light reaching the observer is
‘maximum’. Once the solution of sample is placed in the tube, the emerging light
is observed from the analyzing lens. If there is no change in the intensity of light,
the sample is said to be optically inactive. On the other hand, if the sample
rotates the plane of polarization, the intensity of the light observed through
analyzer lens is found to decrease and the sample is said to be optically active.
The analyzer lens is then rotated to observe the maximum intensity of light. If
the rotation of the plane and hence, of the analyzer lens is towards right, the
sample is dextrorotatory and if rotation is towards left the sample is
laevorotatory. The rotation of plane of polarization is measured in degrees and is
known as observed optical rotation (α).
Fig. 3.10 Polarimeter: A schematic representation.

Specific rotation
Specific rotation [α], is the rotation caused by a sample at a concentration of 1.0
g/mL in a sample tube of length 1.0 dm (10.0 cm). The value of specific rotation
depends on the concentration of sample, its structure, wavelength of light source,
temperature, length of the sample tube, and solvent. The temperature t (in oC)
and wavelength λ of light source are indicated as superscript and subscript
respectively while writing the specific rotation.
Racemic mixture
A mixture containing equal amounts of a pair of enantiomers is termed as
racemic mixture (also called racemic modification). A racemic mixture is
optically inactive. As the enantiomers exhibit same specific rotation in opposite
directions, the specific rotation values of two isomers cancel each other,
resulting in zero optical activity. The racemic mixture is denoted by (+). For
example, (+) lactic acid.

Stereoisomers and their Optical Activity


(1) Enantiomers show identical physical properties but they
may differ in their smell. Diastereomers show different
physical properties, for example, melting point, boiling point,
solubility, refractive index, density etc.
(2) Enantiomers show same specific rotation values but rotate
the plane of polarized light in opposite directions.
Diastereomers differ in their specific rotation values.
(3) Enantiomers exhibit identical chemical properties except in
their reactions with optically active reagents. Diastereomers
show different chemical properties
(4) A racemic mixture is optically inactive due to external
compensation.
(5) A stereoisomer with a plane of symmetry (meso structure) is
optically inactive due to internal compensation. In meso
structure, one half of the molecule is mirror image of the
other half.
3.4 ABSOLUTE CONFIGURATION (R AND S
CONFIGURATION)
The most important task for an organic chemist is to assign configuration to
stereoisomers. Although we can distinguish enantiomers by the sign of optical
rotation, assigning the configuration to (+) and (–) isomers is not easy. The
actual configuration of the molecules, that is, absolute configuration is assigned
by a set of sequence rules given by three scientists Cahn, Ingold, and Prelog
(referred as CIP system). These sequence rules give nomenclature of
stereoisomers in terms of R and S configurations.
Sequence rules
Rule 1. Write the stereoisomer in terms of Fischer projection formula.
Rule 2. Identify the atoms attached (or the atom through which group is
attached) to chiral carbon (stereogenic centre). Now assign priority order in
accordance with their atomic numbers.
Let us consider the application of these rules in the following two example.

As a result, for convenience, in the Fischer projection instead of atoms we write


their priority numbers.
Example B

The chiral carbon is attached to –CH3, –NH2, –OH, and an H atom. The groups
–CH3, –NH2, and –OH are attached through C, N, and O atoms respectively.
The atom with the highest atomic number is assigned the highest priority (1) and
the atom with lowest atomic number is assigned the lowest priority (4).

Rule 3. If in a molecule different groups are attached to chiral centre through


similar atoms, priority order cannot be assigned on the basis of atomic number of
the atoms directly attached to chiral centre. In such a case, we look at the next
set of atoms with higher atomic number present in groups, and priority is
assigned at the first point of difference in accordance with the atomic number.
For example, consider

The priority order to the groups, attached through similar atoms, is assigned as
follows:
Rule 4. In case of groups having multiple bond, for assigning priority order a
multiple bond (double or triple) is considered to be bonded to an equivalent
number of similar atoms through single covalent bonds. For example, two
carbons attached through a double bond and considered as if each carbon is
joined through two single covalent bonds to two carbons. The priority order is
then decided in accordance with the above discussed rules.
The priority order of different groups with multiple bonds is assigned as follows:

3.4.1 Assigning R and S Configuration


Based on above sequence rules, (1–4), priority order is assigned to different
groups and atoms attached to chiral carbon. The group or atom with least
priority is placed at a position vertically downward in the Fischer projection.
Then the arrangement of rest of the three groups or atoms is considered
according to their decreasing priority order.
If the groups or atoms are ordered in a clockwise manner, the absolute
configuration assigned is R [Latin: Rectus—clockwise] and if the order of the
groups or atoms is anticlockwise, the absolute configuration assigned is S [Latin:
Sinister—anticlockwise]. This is clearly understood by the following examples:
To assign R and S configuration to the chiral molecule, the least priority group
should be vertically downward in the Fischer projection. If it is not so, then
interchanges in the positions are carried out to bring it to that position but care
needs to be taken so that the interchanges do not change the actual configuration
of the molecule. This is done by following two rules which state—
1. The interchange is to be carried out only between adjacent positions in the
Fischer projection.

2. An even number of interchanges must be carried out, as it


does not change the actual configuration of the molecule. It
is to be noted that an odd number of interchanges cause a
change in the configuration of the molecule.

Following is an exercise which assigns configuration to a molecule in


accordance with the sequence rules and interchange rules mentioned so for.
Illustrative Example 1. Assign the R or S configuration to the following
molecules:
Solution. The following steps are to be followed for assigning the configuration
of given examples:
(i) Assigning the priority order to the group(s) and or atom(s) attached to the
chiral carbon according to the rules explained earlier.
(ii) Writing the Fischer projection for the molecules (for b and c).
(iii) Bringing down the lowest priority group to vertically downward in the
Fischer projection, if needed, by applying the set of rules as mentioned in
the text.
(iv) Finally, assigning the configuration based on the arrangement of group(s)
and or atom(s).
For (a): The priority order of attached groups and atoms to the chiral carbon is

Thus, the configuration to the molecule is assigned as

For (b): The priority order of attached groups and atoms to the chiral carbon is

Now converting the given molecule to Fischer projection (refer Fig. 3.6)

Now bringing the lowest priority group vertically downwards by even number of
interchanges (p. 79) will give the configuration, which is as follows:
For (c): The priority order of attached groups to the chiral carbon is assigned as
mentioned earlier (p. 78)

Illustrative Example 2. Assign the R or S configuration to the following


molecule having two stereogenic (chiral) centres.

Solution. In molecules with two chiral centres, the configuration is to be


assigned at each chiral centre. First step is to number the carbons and thus chiral
centres in the molecule, as per IUPAC guidelines. In this example, C2 and C3
are chiral centres.
Configuration at C2: C2 is attached to four groups (or atoms), namely CH3, H,
Br and –CH(Br)CH2CH3. Priority order of groups attached to C2 is:
Br > –CH(Br)CH2CH3 > CH3 > H

Configuration at C3: C3 is attached to four groups, namely Br, CH2CH3, H, and


CH(Br)CH3.

Thus, the configuration of above molecule is 2S, 3S (since at C2 and C3, the
configurations are S and S repectively) and this stereoisomer will be named (2S,
3S)-2,3-Dibromopentane.
Illustrative Example 3. Assign the R and S configuration to the following
molecule

Solution. The configuration in this case is to be assigned at both stereocentres,


C2 and C3
Configuration at C2: C2 is attached to four groups, namely H, Br, COOH, and –
CH(OH)CHO. Priority order of groups or atoms attached to C2 is: Br > –COOH
> –CH(OH)CHO > H.
[Note that atomic number of Br is highest.]
For –COOH and –CH(OH)CHO, the priority order is assigned (refer pp. 77-78)
as follows:

Configuration at C3: C3 is attached to four groups, namely H, OH, CHO, and –


CH(Br)COOH.
Priority order of groups or atoms attached to C3 is –OH > –CH(Br)COOH > –
CHO > H
[Note that –CH(Br)COOH has higher priority than –CHO as bromine has higher
atomic number compared to oxygen.] The arrangement of the groups/atoms
(refer pp. 77–78) is as follows
Thus, the configuration of above molecule is 2S, 3R (since the configurations at
C2 and C3 are S and R respectively) and this stereoisomer is named as (2S, 3R)–
Bromo-3-hydroxy-4-oxobutanoic acid.

3.4.2 Relative Configuration (D- and L-


Nomenclature)
The term relative configuration is used when a molecule is assigned
configuration with respect to glyceraldehydes. In glyceraldehydes if –OH group
is present on the right hand side, it is assigned D-configuration and if –OH group
is on the left hand side, it is assigned L-configuration.

Compounds prepared from or converted into D-glyceraldehydes belong to D-


series. Similarly, compounds prepared from or converted into L-glyceraldehydes
belong to the L-series.
• The D- and L- nomenclature is independent of dextro- and laevo-. A
compound of D-series may be dextro (+) or laevo (–) rotatory. Similarly, L-
series compounds may show (+) or (–) optical activity.
• This nomenclature system is used in carbohydrates and amino acids.

3.4.3 Chirality in a Molecule with no Stereogenic


(Chiral) Centre
As discussed earlier, the most common cause of asymmetry in a molecule is
presence of an asymmetric carbon (stereocentre). The molecules with one
stereocentre exist in two steroisomeric forms, which are optically active.
However in some cases like tartaric acid (a molecules with two stereocentres),
meso structure exhibits a plane of symmetry and does not show optically activity.
There are several examples in which stereocentre (chiral carbon) is not present
yet they exhibit optical activity. These molecules possess asymmetry for entirely
different reasons. The allene and biphenyls are few such examples where the
molecule does not have a chiral centre yet they show optical activity.
In allenes, CH2=C=CH2, the two double bonds are present on the same carbon.
The central carbon atom is sp hybridized and linear, while the two outer carbons
are sp2 hybridized and trigonal. In order to have an effective overlap the p
orbitals of terminal CH2 groups will overlap with p orbitals of central atom.
Since the two p orbitals on central carbon are mutually perpendicular it clearly
indicates that p orbital on each terminal CH2 are perpendicular. In other words,
in allenes, the terminal carbons are present in perpendicular plane. Thus the
whole molecule is not in a plane and exhibit asymmetry.

Similarly, in substituted biphenyl, the two phenyl rings are not present in same
plane. The substituents present adjacent to bond joining two benzene rings
restrict the rotation around this bond and the molecule is said to be
conformationally locked. The molecules are highly strained and cannot achieve
symmetry because of the steric hindrance or strained structure. Thus, the
substituted biphenyls exist in enantiomeric forms, which are optically active.
3.5 GEOMETRICAL ISOMERISM
The compounds having similar molecular formulae but different arrangement of
atoms or groups in space around the double bond are called geometrical isomers
and the phenomenon is known as geometrical isomerism. The geometrical
isomerism arises due to restricted rotation about a carbon–carbon double bond.
A complete rotation around carbon–carbon double bond causes the breaking of π
bond (refer Notabilia 4).
Necessary and sufficient condition for geometrical isomerism
An alkene of the type abC==Cxy exhibits geometrical isomerism if a ≠ b and x ≠
y. In an alkene, if either of the double bonded carbons is attached to two
identical groups or atoms, no geometrical isomerism will exist.

There are three different ways in which groups a, b, x, and y can be arranged
around carbon–carbon double bond. The configuration of the geometrical
isomers is designated by
• Cis–trans system
• E–Z system
Cis–trans system
This system is used for designating the alkenes in which the two olefinic carbons
have at least one similar group or atom present on them. For example, the
alkenes of the type abC==Cab and abC==Cax exhibit cis–trans isomerism. It is
to be noted that
(i) the term cis- is used when two similar atoms or groups are present on same
sides across the double bond and
(ii) the term trans- is used when two similar atoms or groups are present on
opposite sides across the double bond.
The cis- and trans- isomers differ in their physical properties. In general,
trans- isomers are more stable compared to cis- isomers. In cis- isomers, the
groups present on the same side experience van der Waals repulsive forces due
to steric factors.

Dipole moment values can be used effectively for distinguishing cis- and
trans-isomers. The trans-isomers have zero dipole moment as the bond dipole on
opposite sides cancel each other.

E–Z system
The alkenes of the type abC==Cxy exhibit geometrical isomerism but they
cannot be classified as cis- or trans- isomers, as all the substituents attached to
olefinic carbons are different. The configuration about any carbon–carbon
double bond is specified using E–Z system. The sequence rules as discussed
earlier (see Sec. 3.5) are used for this purpose.
• The atoms or groups attached to each olefinic carbon are given priority as
per sequence rules described earlier.
• If higher priority groups are present on same sides across the double bond, the
geometrical isomer is said to have Z-configuration [German: Zusammen—
same side].
• If higher priority groups are present on opposite sides across the double bond,
the geometrical isomer is said to have E-configuration [German: Entegegen
—opposite side].
All geometrical isomers (including cis- and trans-) are designated by E–Z
system.

Geometrical isomerism is not restricted to carbon–carbon


double bond [ C=C ] but is also exhibited by compounds having a
carbon–nitrogen double bond [ C=N–] as in oximes, or nitrogen–
nitrogen double bond [–N=N–] as in azo compounds.
According to the sequence rules a lone pair gets the least priority
and is ranked below hydrogen. Let us consider the isomeric oximes
and azo compounds and designate them as E–Z isomers.

NOTABILIA
3.6 CONFORMATIONS
In alkanes, the carbon atoms are connected through single bonds. The atoms
connected by a single bond can rotate about that bond without breaking it and
the energy required for this rotation is very low. Molecules possess kinetic
energy due to a state of continuous motion. The energy is transferred among
molecules during collisions and is sufficient to bring about rotation about single
bond and for this reason the rotation is termed free rotation. Different three-
dimensional arrangements of atoms that result due to free rotation about carbon–
carbon single bond are known as conformations. The individual structures
arising due to free rotation are known as conformers or conformational isomers.
A free rotation about carbon–carbon single bond does not change the basic
skeleton of the molecule.

3.6.1 Conformations of Ethane


In case of ethane molecule, the free rotation about σ bond results in two
conformations, namely staggered and eclipsed conformations.

In staggered conformations, the carbon–hydrogen (C–H) bonds on each carbon


are at a maximum distance and thus, have minimum repulsion.
In eclipsed conformation, the carbon–hydrogen (C–H) bonds on each carbon are
at a minimum distance, that is, very close to each other and thus, experience
maximum repulsion.

Fig. 3.11 Ball and stick model showing C–C free rotation in Ethane.

Representation of conformations
Sawhorse and Newman projections better represent the staggered and eclipsed
conformations of ethane besides dashed-wedged-line structure or ball and stick
model.
Sawhorse projection. In sawhorse projection, the two carbons attached through
σ bond are represented by points where four lines intersect. The bond through
which two carbons are joined is considered in the plane whereas the atoms (or
groups) attached to carbons may be above (shown by thick lines) or below the
plane (shown by dotted lines).
Newman projection. In Newman projection, a dot and a circle represent the two
carbons, attached through σ bonds. The dot represents the front carbon whereas
circle represents the rear carbon. The represents the two carbons attached
through σ bond. The three bonds attached to front carbon are represented as full
lines , whereas the three bonds attached to rear carbon are represented by line
arising from the surface of the circle .
Thus, the staggered and eclipsed forms of ethane can be represented as shown in
the following structures. In a staggered conformation, if we hold the front carbon
and rotate the rear carbon through an angle of 60° around carbon–carbon axis, it
results in an eclipsed conformation; a further rotation by 60° will result in a
staggered conformation.
Sawhorse projection

Newman projection
The study of the energy associated with different conformations is known as
conformational analysis. A plot between energy (along Y-axis) and angle of
rotation (along X-axis) depicts that in the energy diagram the lowest energy
conformations are staggered conformations. The maximum energy is associated
with eclipsed conformations. The difference in energy between the most stable
staggered and least stable eclipsed conformation is referred to as torsional strain
of the molecule. For ethane, the torsional strain is 3.0 kcal mol–1.
The energy required to overcome torsional strain is known as energy of
activation (Ea) and is supplied by molecular collision. The lower the value of
Ea, faster is the rotation about carbon–carbon single bond.
Fig. 3.13 Conformational analysis of ethane.

3.6.2 Conformations of Propane


A rotation about carbon–carbon single bond in propane results in two extreme
conformations, namely staggered and eclipsed, and the conformational analysis
is similar to ethane.

However, the torsional strain in propane is a little higher (3.3 kcal mol–1) than
ethane due to steric hindrance caused by methyl group in eclipsed conformation.
In the eclipsed conformation of propane, van der Waals repulsion occurs
between the methyl group and hydrogen.
Fig. 3.14 Conformations in propane and its conformational analysis.

3.6.3 Conformations of Butane


Butane, CH3CH2CH2CH3 can be written as

If we view the molecule of butane through C2–C3 bond, each carbon is seen
attached to one methyl group and two hydrogens. A rotation about C2–C3 bond
results in the formation of a number of conformations. The Newman projections
(I-VI) for different conformers of butane, as obtained through 60° rotation
around C2–C3 bond, are as follows:

Totally staggered or anti-conformation [conformation–I ]


In an anti-conformation, the two CH3 groups are farthest from each other and
have minimum interaction. Thus, the anti- form is the most stable conformation.
Totally eclipsed conformation [conformation–IV ]
In totally eclipsed conformation, the methyl group on front carbon is exactly in
front of methyl group on the rear carbon. The two bulkier methyl groups are
very close to each other and this steric crowding causes van der Waals repulsion.
The torsional strain and van der Waals repulsion together make the totally
eclipsed conformation least stable.
Skew conformations [conformations–II, III, V, and VI]
Different conformations in butane which result from rotation about carbon–
carbon bond as one moves from totally staggered to totally eclipsed
conformation, are known as skew conformations. Skew conformations may exist
in both staggered or eclipsed forms.
(i) Skew staggered conformation (Conformations III and V): The staggered
conformations in which two methyl groups are not farthest apart but are at
an angle of 60°, are known as Gauche conformations. For example,
conformations III and IV above. In gauche conformations, the two methyl
groups are relatively close compared to anti-form and thus, experience van
der Waals repulsions. This makes gauche conformations to have higher
energy than anti-conformations.
The gauche forms III and V are related as non-superimposable mirror images
and are termed as conformational enantiomers.
(ii) Skew eclipsed conformations (Conformations II and VI): In these
conformations, methyl group and hydrogen on adjacent carbon are present
exactly in front of each other. The eclipsed conformation experiences
torsional strain as well as the van der Waals repulsion between methyl
group and hydrogen. The skew eclipsed conformations represent ‘energy
maxima’ but have relatively low energy compared to totally eclipsed
conformation where van der Waals repulsion is large due to interaction of
two bulkier methyl groups.
The skew eclipsed conformations II and VI are mirror images of each other
and are termed conformational enantiomers.
The skew eclipsed and skew staggered conformations do not exhibit object–
mirror image relationship and are termed conformational diastereomers. The
conformational analysis of butane is represented in Fig. 3.15.
The decreasing order of stability of different conformations of butane is
Anti (Totally staggered) > Gauche (Skew staggered) > Skew eclisped > Totally
eclipsed
3.7 CYCLOALKANES: CONFORMATIONS AND
GEOMETRICAL ISOMERISM
Cycloalkanes have sp3 hybridized carbons and thus, they should have a bond
angle of 109.5°. The simplest cycloalkane is cyclopropane which has a shape of
regular triangle with bond angles of 60°. This reduction in bond angle by 49.5°
(from 109.5° to 60°) causes an angle strain in the molecule (refer Baeyer strain
theory, section 5B.2). The sp3–sp3 overlap in cyclopropane is not as effective as
in open chain compounds. The carbon–carbon bonds are bent and relatively
weak which makes the ring less stable. Cyclopropane is a planar molecule and
all the six hydrogens are present in an eclipsed state. Cyclopropane exhibits
torsional strain as well as angle strain and these two strains together cause ring
strain in cyclopropane. The Newman projection of cyclopropane is represented
as:

In a planar cyclic structure, all the hydrogens are eclipsed which leads to
torsional strain. The more the number of ‘eclipsed hydrogens’, higher is the
torsional strain. Thus, cyclobutane and cyclopentane with a planar structure will
have considerable torsional strain. Acquiring non-planar structure relieves this
torsional strain. A non-planar structure however causes an increase in angle
strain but this increase in angle strain is compensated by the decrease in torsional
strain.

The non-planar cyclopentane ring has a negligible angle strain and is therefore
more stable compared to cyclobutane and cyclopropane.
cis, trans isomerism in cycloalkanes
As discussed earliar in the chapter, cis-, trans-isomerism arises due to restricted
rotation in double bonds. Cyclic compounds can also have cis-, trans-isomerism
because the cyclic system prevents free rotation about single bond. In a cyclic
compound, if the substituents are present on the same side of the ring, it is said
to be cis isomer and if substituents are present on opposite sides of the ring it is
termed as trans isomer. The substituted cycloalkanes posses chiral centre and
exist as enantiomers. Some of the isomers posses the plane of symmetry as
shown in the figure below and thus, do not exist as enantiomers.

3.7.1 Conformations of Cyclohexane


Cyclohexane is as stable as its open chain counterpart hexane. Cyclohexane
exists as a puckered or non-planar ring. The twisting about carbon–carbon bond
results in the formation of different conformations of cyclohexane. The four
main conformations of cyclohexane are:

Chair conformation
This is the most stable conformation of cyclohexane as it is free from torsional
strain. All the twelve hydrogens are in staggered state as evident form Newman
projection. The bond angle is nearly 109.5° and thus, it is free from angle strain
also.

Boat conformation
Twisting about carbon–carbon single bond of the chair form results in the
formation of boat conformation. Boat conformer is free from angle strain.
However, in boat conformation the hydrogens are in eclipsed state, which causes
torsional strain in the molecule. Along with this the hydrogen at C1 and C4 are
close to each other and experience van der Waals repulsion known as flagpole
interaction. The torsional strain and flagpole interaction make boat conformation
less stable compared to chair conformation.

Twist boat conformation


The boat conformation is flexible and a slight twist about the bond reduces the
torsional as well as flagpole interactions which makes the twist boat
conformation a little more stable than the boat conformation.
Half chair conformation
This is the least stable conformation of cyclohexane because carbon atoms at one
end of the ring are planar.

The stability order of different conformations of cyclohexane is:

Chair conformation >> twist boat conformation > boat conformation > half chair
conformation

The conformational anaylsis of cyclohexane is given in Fig. 3.16 that follows.

Fig. 3.16 Conformational analysis of cyclohexane.

Axial and equatorial hydrogens in chair conformation


In the chair conformation of cyclohexane, all the twelve hydrogens are not
equivalent. The chair conformation has two types of hydrogens, axial (a) and
equatorial (e).
Six hydrogens are present perpendicular to the plane of the ring and are termed
as axial hydrogens while the remaining six hydrogens project out sideways,
along the plane of the ring and are termed as equatorial hydrogens.
Each carbon has one axial and one equatorial hydrogen which point in opposite
directions. The three axial hydrogens are perpendicular in upward direction and
three axial hydrogens are perpendicular in downward direction. The axial
hydrogens point alternatively in upward and downward directions in accordance
to the vertices of the cyclohexane ring. If the carbon (vertex) of the chair
conformation is in upward direction, the axial hydrogen will also be in upward
direction, however the equatorial hydrogen will be in downward direction,
however the equatorial hydrogen will be in downward direction. A flip in chair
conformation interconverts the axial and equatorial hydrogens.
3.7.2 Conformations of Monosubstituted Cyclohexane
In the replacement of hydrogen of a cyclohexane ring by a substituent, the
substituent can occupy either an axial or an equatorial position. For example,
methylcyclohexane can be represented by following two chair conformations:

If methyl group occupies an axial position, it is close to axial hydrogens at C3


and C5 (as C5 and C3 are equidistant from C1, these positions are also referred
to as 3 and 3′). The van der Waals repulsion occurs due to steric crowding of
methyl group and two hydrogens. This causes transannular strain. The
transannular strain is the strain produced in a ring due to steric repulsions. This
effect occurs due to axial substituents and is known as 1,3-diaxial interaction. A
substituent at equatorial position does not experience any such repulsion as
equatorial bonds project out sideways. The Newman projection of axial and
equatorial conformers of methylcyclohexane clearly depicts the stability of
equatorial conformers.
The equatorial and axial conformers exist in equilibrium. However, equatorial
conformer is more stable due to absence of 1,3-diaxial interactions. Thus, a
substituent prefers to occupy an equatorial position. With an increase in the
bulkier substituent (bigger group), the ratio of equatorial conformer increases
considerably and a very little of axial conformer is present at equilibrium.

3.7.3 Conformations of Disubstituted Cyclohexane


Two substituents present on same carbon
The substituent present in a cyclohexane ring prefers to occupy an equatorial
position. In case two different substituents are present on same carbon, a bulkier
substituent occupies equatorial position. For example in 1-ethyl-1-
methylcylohexane, a conformation with ethyl group at equatorial position and
methyl group at axial position is more stable than a conformation with methyl
group at equatorial position and ethyl group at axial position.

Two substituents present on different carbons


However, if two substituents are present at two different carbons then depending
upon the position occupied by each substituent, the different conformers are
designated as: equatorial–equatorial(e,e); equatorial–axial(e,a); axial–
equatorial(a,e); and axial–axial(a,a). If two similar substituents are present, (e,a)
and (a,e) represent the same conformers.
Cis- and Trans- isomerism in disubstituted cyclohexanes
Disubstitued cyclohexanes exhibit cis–trans isomerism due to restricted rotation
in cyclic system. There are three possible disubstituted cyclohexanes:
(i) 1,2-disubstituted cyclohexanes,
(ii) 1,3-disubstituted cyclohexanes, and
(iii) 1,4-disubstituted cyclohexanes.

Cyclohexane exists in non-planar chair conformation. A planar form of


cyclohexane (that is, hexagon) is used for a convenient representation of
cis–trans isomers. If two substituents are present on the same side, it represents a
cis- isomer. Thus, cis- and trans- forms of 1,2-, 1,3- and 1,4-
dimethylcyclohexane are represented in the following manner:

Representing cis- and trans- isomers in chair conformation


In chair conformations, the two substituents may occupy (e,e), (e,a), (a,e), and
(a,a) conformation. The substituents may be attached through an axial or an
equatorial bond. The equatorial and axial bonds may point in upward or
downward direction.
• If both the bonds, through which substituents are attached, point in same
direction, that is, both upward or both downward, the conformation
represents a cis- isomer.
• If both the bonds, through which substituents are attached, point in opposite
directions, that is, one upward and one downward, the conformation
represents a trans- isomer.
The cis–trans isomerism and the stability of conformers in disubstituted
cyclohexanes is explained by considering the examples of 1,2- , 1,3- and 1,4-
dimethylcyclohexane as follows:
1,2-Dimethylcyclohexane
Different chair conformations possible for 1,2-dimethylcyclohexane are:

In the above conformations, (a,a) and (e,e) represent trans- isomers, whereas
(a,e) or (e,a) represent cis- isomers.
1,3-Dimethylcyclohexane
Different chair conformations possible for 1,3-dimethylcyclohexane are:

In these conformations, (a,a) and (e,e) represent cis- isomers, whereas (a,e) or
(e,a) represent trans- isomers.
1,4-Dimethylcyclohexane
Different chair conformations possible for 1,4-dimethylcyclohexane are as
follows:
In the conformations above, (a,a) and (e,e) represent trans-isomer whereas (a,e)
or (e,a) represent cis-isomers.
Stability of conformers in dimethylcyclohexanes
In 1,2-, 1,3- and 1,4-dimethylcyclohexane, the stability of conformers is decided
on the basis of the following:
(i) The (e,e) conformer is the most stable and the most preferred conformation
because of the absence of 1,3-diaxial interactions.
(ii) The (a,a) conformer is the least stable because of 1,3-diaxial interactions
that are experienced by both methyl groups.
(iii) The (a,e) or (e,a) conformers are more stable than (a,a) but less stable
than (e,e) because one of the methyl group at axial position experiences
1,3-diaxial interactions.
Exercises*
1. Classify the following as ‘chiral’ and ‘achiral’.
(a) Scissors
(b) Shoe
(c) Hammer
(d) Nail
(e) Screw
(f) T-shirt
(g) Foot
(h) Fork
(i) Nose
2. In the following compounds, indicate the stereogenic (or chiral) centre by
putting an asterisk mark over it.
(a) CH3CH(Br)CH2OH
(c) CH3CH(Cl)CH(Br)CH3
(b) C6H5CH(OH)CH3
(d) CH3CH2CH(Cl)CH(CH3)CH3
3. What are stereoisomers?
4. Define the terms enantiomers and diastereomers.
5. Comment on the physical and chemical properties of enantiomers and
diastereomers.
6. What is optical activity? What is the necessary condition for a molecule to be
optically active?
7. What are meso compounds? Are the meso compounds optically active or
inactive? Explain your answer through a suitable example.
8. What is general formula for determining the number of stereoisomers
possible for a given compound? Indicate the number of stereoisomers
possible for each of the following:
(a) CH3CH(NH2)COOH
(b) CH3CH2CH(OH)CH2CH3
(c) CH3CH2CH(Br)CH(Cl)CH3
(d) CHOCH(OH)CH(OH)CH(OH)CH2OH
9. Draw the Fischer projections for all possible stereoisomers of:
(a) C6H5CH(NH2)COOH
(b) HOOCCH(CH3)CH(CH3)CH2OH
10. In accordance with the sequence rules, assign the decreasing order of
priority to following groups/atoms.
(a) –CH3, –OH, –NH2, Cl
(b) –Br, –CH2Br, –CH2CH3, —SH
(c) –Cl, –Br, –F, –CH2I
(d) –CH==CH2, –CH2CH==CH2, —CH2CH2CH3, —COOH
(e) –Cl, –COCH3, CONH2, COCl
(f) –COOH, –CH2COOH, –CH(Cl)COOH, —CH3
(g) –OCH3, –OH, –NH2, –F
(h) –H, –D, –CH3, –T
(i) –CH2CH3, –CH3, –CH2OH, –H
(j) –CH2OH, –CH2COOH, –CH3, –H
11. Assign the R and S configuration to the following:

12. What are conformations? How does it differ from configuration?


13. Draw the Newman projection for different conformations possible for
butane. Give the conformational analysis for butane.
14. What is dihedral angle? What is the dihedral angle in staggered and eclipsed
Newman projections of ethane?
15. What are ‘skew’ conformations? Can all the skew conformations be termed
as ‘gauche’ conformations?
16. What do you understand by the following terms:
(a) Torsional strain
(b) Angle strain
(c) Ring strain
17. Draw the conformations for cyclopropane. Comment on the low stability of
cyclopropane.
18. Draw the different conformations for cyclohexane and arrange them in
increasing order of stability.
19. In monosubstituted cyclohexanes, why does a substituent prefer to occupy
an equatorial position?
20. Draw the chair conformations of:
(a) cis-1,2-dibromocyclohexane
(b) trans-1,3-dimethylcyclohexane
(c) cis-1,3-dichlorocyclohexane
(d) cis-cyclohexane-1,3-diol
(e) trans-1,4-diethylcyclohexane
(f) cis-1-propyl-4-methylcyclohexane
21. In cyclohexane-1,3-diol, the (a,a) conformation is more stable compared to
(e,e) conformation. Explain.
22. Designate ‘E’ & ‘Z’ to the following double bonded compounds:

Answers to selected exercises


1. Chiral: (a), (b), (e), (g)
Achiral: (c), (d), (f), (h), (i)
8. 2n where n = number of chiral centres
(a) 2 (b) None, its achiral (c) 4 (d) 8
9.
10. (a) Cl > OH > NH2 > CH3
(b) Br > SH > CH2Br > CH2CH3
(c) Br > Cl > F > CH2I
(d) COOH > CH==CH2 > CH2CH==CH2 > CH2CH2CH3
(e) Cl > COCl > CONH2 > COCH3
(f) –CH(Cl)COOH > COOH > CH2COOH > CH3
(g) F > OCH3 > OH > NH2
(h) CH3 > T > D > H
(i) –CH2OH > –CH2CH3 > –CH3 > H
(j) CH2OH > CH2COOH > CH3 > H
11. (a) S (b) R (c) R (d) R (e) 2R, 3R

(f)
22. (a) E (b) E (c) E (d) Z (e) Z
* Answers to selected exercises are given at the end.
Chapter4
Fundamentals of Organic Reactions

4.1 ELECTRONIC DISPLACEMENTS


The organic compounds are the compounds of carbon where different atoms are
held together through covalent bonds. The movement of electrons in a molecule
has a great impact on the reactivity of that molecule. The different ways through
which electronic displacements occur in a molecule are as follows:
1. Inductive effect
2. Electromeric effect
3. Resonance or Mesomeric effect
4. Hyperconjugation

4.1.1 Inductive Effect


The hydrocarbons contain carbon and hydrogen atoms and are thus non-polar in
nature as there is not much difference in the electronegativity of carbon and
hydrogen. For example, in alkanes carbon–carbon and carbon–hydrogen σ bonds
are present. If we replace the hydrogen by some other atom which differs
appreciably in its electronegativity compared to carbon, then it results in the
induction of polarity in the alkane molecule.
If we replace hydrogen by an atom like fluorine, which is more electronegative
than carbon, fluorine will carry a negative charge, as it will attract shared
electrons towards itself and thus, the carbon skeleton will carry positive charge.
This carbon, in turn, induces a small positive charge on the adjacent carbon.
Similarly, when hydrogen is replaced by an atom such as lithium which is more
electropositive than carbon, the lithium will carry a positive charge. Now, carbon
being more electronegative attracts the shared electrons towards itself and will
carry a negative charge. This induces a small negative charge on adjacent
carbon. This induction of polarity in a bond through another bond is termed
inductive effect.
The atoms or groups which are more electronegative than carbon withdraw
electrons from carbon and are said to exhibit –I effect (minus I effect). For
example, F, Cl, Br, I, –OH, –NO2 and so on.
The atoms or groups more electropositive (less electronegative) compared to
carbon release electrons towards carbon and are said to exhibit +I effect (plus I
effect). For example, Li, Na, K, alkyl group and so on.

As the attachment of the number of groups, exhibiting either +I or –I effect, on a


carbon increases it experiences the corresponding inductive effect to a greater
extent. For example.

The characteristic features of inductive effect include


(a) It is a permanent effect.
(b) This effect operates through σ bonds.
(c) It is observed for a maximum of upto second carbon atom of the chain and
ceases to operate beyond fourth carbon.
Being a permanent effect, the inductive effect affects the physical and chemical
properties of molecules such as dipole moment, rate of reaction and their acidic-
basic characteristics. For example the acidic behaviour of alcohols, phenols,
carboxylic acids, sulfonic acids and basicity of amines, is affected by the
inductive effect of substituents and are discussed in respective chapters
subsequently.
4.1.2 Electromeric Effect
This is a temporary effect which is observed in multiple bonded systems, in
presence of a polar reagent. It involves the shifting of π-electrons towards one of
the atom held through a multiple bond e.g. alkenes, alkynes, and carbonyl
compounds.

Alkenes, as such, exist as non-polar species but in presence of a reagent, the π-


electrons shift towards one of the carbon and the system behaves as a highly
polar single bonded system as shown in structure [B]. A similar behaviour is
observed in carbonyl compounds where π-electrons move towards oxygen in
presence of a reagent (refer structure [D]). As soon as the polar reagent is
withdrawn from the vicinity of these molecules, the transferred π-electrons come
back to their original position and the molecules behave as π-bonded systems
(see structure [A] and [C]).
The electromeric effect is denoted by symbol E. It has no fixed direction but
occurs in a direction, which favours the reaction. However, if a group attached to
multiple bonded carbon has +I or –I effect, it affects the direction of
electromeric effect directly.

Being a temporary effect, it does not affect the physical properties of a molecule.

4.1.3 Resonance Effect [or Mesomeric Effect]

[Delocalization of π electrons]
In Chapter 1, we have discussed the concept of resonance in detail (refer section
1.6). The resonance effect (or mesomeric effect) is a permanent effect which
involves delocalization of π electrons in a system. The electrons involved in
delocalization may be π-electrons of a double bond, or π electrons of the
aromatic system (as in benzene), or lone pair of electrons present on an atom.
In other words, delocalization may involve
(1) Overlap of p orbitals involved in the formation of π bonds (π–π overlep)
or
(2) Overlap of p orbital of an atom (vacant or filled) with p orbitals involved
in π bond formation (p–π overlap)
Delocalization of π electrons through π–π overlap
Delocalization of π electrons through π–π overlap occurs in conjugated systems,
that is, a system where π bonds are separated by one σ bond only. For example,
in buta-1,3-diene, each carbon is sp2 hybridized and thus each carbon has a p
orbital. These four p orbitals overlep with each other to form π bonds. The π
bonds are not localized between two carbons (C1 and C2, or C3 and
C4) rather π electrons are delocalized (or distributed equally) over all the four
carbons. This delocalization of π electrons makes all the carbon–carbon bond
lengths in buta-1,3-diene same and the value of carbon carbon bond length in
butadiene lies in between that of carbon–carbon single bond and carbon–carbon
double bond length (refer also section 7.2). The delocalization lowers the
resonance energy (more appropriately delocalization energy—refer section 1.6
for this terminology) and thus stabilizes the system.

Similarly in benzene, the π electrons are delocalized over all the six carbons of
the ring (refer Chapter 9 for details).
The groups containing π bonds such as

and so on when attached directly to benzene ring withdraw electrons from ring
by participation in delocalization through π–π overlap. The π bonds of such
groups are in conjugation with π electrons of the benzene ring system. Such
groups are said to exhibit minus resonance effect or minus mesomeric effect
(–R or –M effect).

Delocalization of π electrons through p–π overlap


Delocalization of π electrons through p–π overlap occurs in system where p
orbital of an atom (vacant or filled) is separated from a π bond through one σ
bond only. In other words the p oribital may contain a lone pair of electron or a
single electron or may be vacant.
The cases where delocalization involves the overlap of lone pair containing p
orbital with the orbitals forming π bond can be illustrated by taking examples of
vinyl chloride and chlorobenzene as follows:
In these examples, the lone pair of electrons is present in p orbital of chlorine.
Although chlorine is highly electronegative (–I effect), the lone pair present in p
orbital of chlorine is in conjugation with π bond(s). As a result, it participates in
delocalization by giving its lone pair towards the π bonded systems of vinyl or
benzene ring and stabilizes the system. The chlorine behaves as an electron
donor when it is in conjugation with the π bond.
Similarly, in aromatic systems like chlorobenzene, phenol, aniline, and so on,
delocalization occurs through p–π overlap. The oxygen, nitrogen, and chlorine
have lone pair of electrons present in their orbitals, which participate in
delocalization with π bonds of the benzene ring system (for details, refer to
Chapter 10).
The groups or atoms, which during delocalization release electrons towards
carbon are said to exhibit +R effect or +M effect, for example, halogens

and so on). These groups in conjugation with


π bonds of aromatic systems or unsaturated alkyl groups donate their lone pair.

The delocalization further explains the low reactivity of vinyl chloride and
chlorobenzene due to strengthening of carbon–chlorine bond. The +R effect in
chlorobenzene and vinyl chloride results in the development of a double bond
character between carbon and chlorine, which provides the extra stability to this
bond (refer Sections 11.5.2 and 12.3.1).
Further in p–π overlap, the delocalization of π-electrons may occur through
vacant or half filled p orbitals. This explains the stability of carbocations and
free radicals through resonance effect and is discussed later in this chapter.

4.1.4 Hyperconjugation (No bond resonance)

[Delocalization involving σ electrons]


In resonance effect, the delocalization of electrons occurs through π–π and p–π
overlap, that is, π electrons are involved in delocalization. Now, we discuss the
delocalization of σ electrons through σ–π or σ–p overlap, known as
hyperconjugation. The σ bond in conjugation with the π bond, or σ bond in
conjugation with p orbital (vacant or filled) participates in delocalization and
thus stabilizes the system. The process is similar to resonance but is extended to
σ bonds and therefore named as hyperconjugation.
For example, in propene, the three C–H σ bonds of CH3 group are in
conjugation with π bond. The σ electrons are transferred in the process to
adjacent carbon-carbon bond, in turn π electrons are shifted to terminal CH2
carbon.
There are three C–H σ bonds so three contributing structures are possible for
propene. Due to participation of σ electrons in delocalization, the bond between
carbon and hydrogen does not exist in contributing structures and for this reason,
hyperconjugation is also referred as No bond resonance.

Hyperconjugation explains the stability of substituted


alkenes. A more substituted alkene is more stable due to availability
of more number of conjugated
C–H σ bonds which can participate in hyperconjugation. In ethene,
hyperconjugation does not occur due to non-availability of
conjugated C–H σ bonds.
(a) Hyperconjugation in propene explains its stability compared to
ethene
[Note that more the number of contributing structures, more
stable is the system].
(b) The hyperconjugation also explains the shortening of σ bond
between C2 and C3 in case of propene. As evident from the
contributing structures, a double bond character develops
between carbons at the 2nd and 3rd position which causes
shortening of C2–C3 σ bond.
Similarly in case of toluene, the carbon–hydrogen σ bonds of methyl group
participate in delocalization with π bonds of benzene ring. The presence of three
C–H σ bonds in conjugation with π bonds of benzene ring results in formation of
nine contributing structures.

In case the number of covalent bonds in the contributing structures during


hyperconjugation is less than the number of covalent bonds in the original
structure (as in case of propene and toluene), hyperconjugation is referred to as
sacrificial hyperconjugation.
Another type of hyperconjugation that occurs in carbocations and free radicals is
referred to
as isovalent hyperconjugation since the number of covalent bonds in
contributing structures
is same as the number of covalent bonds in the structure of carbocation or free
radical.
This hyperconjugation occurs though delocalization involving p orbitals (vacant
or half-filled)
and a conjugated σ bond (σ–p overlap), and explains the stability of carbocations
and free
radicals. This is discussed in detail in the subsequent sections of this chapter
(Sections 4.2.1
and 4.2.3).
Sacrificial hyperconjugation
4.2 REACTIVE INTERMEDIATES
The organic compounds are covalent in nature. A chemical reaction basically
involves the breaking of existing bonds and formation of new bonds. A bond
represents a shared pair of electrons between two atoms. The breaking (or
cleavage) of a bond can occur in two ways as
1. Heterolytic cleavage. During cleavage of bond, the shared pair of electrons
is taken away completely by one of the atoms (relatively electronegative
one). This results in the formation of charged species, that is, positively and
negatively charged ions.

2. Homolytic cleavage. During cleavage of bond, the shared pair of electrons


is distributed equally between the two atoms, that is, each atom departs with
one electron each. This results in the formation of free radicals, that is,
species with a single unpaired electron.

Defining Bond Dissociation Energy and Bond Energy


During bond formation, energy is liberated or released. In a similar
way, for cleavage of a bond, certain amount of energy is required.
The amount of energy required to break a bond is termed as bond
dissociation energy. This energy is expressed in

kJ mol–1 or kcal mol–1.


In a molecule, the average of bond dissociation energies of all the
bonds is termed as bond energy.
For example, in methane there are four carbon–hydrogen bonds.
The bond dissociation energy is the energy required to break a
single C–H bond. Bond energy of methane molecule is the average
of bond dissociation energies of all the four
C–H σ bonds. Some important points regarding bond cleavage are:
(a) Bond dissociation energy involved in homolytic cleavage is
generally less than that of heterolytic cleavage.
(b) In heterolytic cleavage, ions are formed and separation of
oppositely charged ions requires greater energy compared to
separation of uncharged species.
(c) In solution phase where polar or ionizing solvent is present, the
heterolytic cleavage is favoured. While in the gaseous phase,
homolytic cleavage is favoured.

4.2.1 Carbocations
Heterolytic cleavage in an organic molecule where carbon donates the shared
pair of electrons to the leaving group results in the formation of carbocation,
where, carbon carries positive charge. The carbocation is the general term used
for the positively charge carbon. Based on the type of structure and type of
hybridization involved in the formation of carbocation some other terms used for
the purpose are carbenium ion and carbonium ion (refer Notabilia 5).
Carbocations are formed as a consequence of heterolytic cleavage. For example,
in case of haloalkanes, the removal of halide ion results in formation of
carbocation. Similarly, alkenes and alcohols in acidic medium result in the
formation of carbocation.
In a carbocation, carbon is bonded to three atoms or groups (trivalent) and has
only six electrons (sextet) so it behaves as an electron deficient species. A
carbocation is sp2 hybridized with a vacant p orbital and has a planar geometry.
A carbocation once formed may undergo reaction with an electron rich species
i.e. nucleophile, or may undergo deprotonation to form an alkene, or may
undergo rearrangement (you will come across a number of reactions where
carbocations are formed and the reactions carbocations undergo, at various
places throughout the text).
Stability of carbocations
A carbocation is classified as 1o, 2o, or 3o depending upon the number of
carbons (1, 2 or 3 respectively) attached to positively charged carbon. Further,
the carbocations may be categorised as alkyl, allyl, benzyl or vinyl cations. For
example

The stability aspects of different categories of carbocations may be explained


through inductive, hyperconjugation or resonance effect.
Stability of alkyl Carbocations: Explanation through inductive effect
Carbocation is a positively charged and electron deficient species. If the groups
attached to positively charged carbon are electron releasing in nature they will
decrease the intensity of positive charge on carbon. In other words, the electron
releasing groups delocalize (or disperse) the positive charge on carbon, thereby
decreasing its electron deficiency. The alkyl groups are electron releasing in
nature due to inductive effect (+I effect). More the number of alkyl groups
attached to a carbocation, more is the delocalization of positive charge, and more
will be the stability of that carbocation.
The stability order of methyl substituted carbocations is

Stability of alkyl carbocations: Explanation through hyperconjugation


The stability of alkyl carbocations can be explained not only through inductive
effect but also through hyperconjugation. In case of carbocations, the σ bond, in
conjugation with vacant p orbital on carbon carrying positive charge, participates
in delocalization. For example, in case of ethyl cation (CH3–C+H2), the three
C–H σ bonds (of CH3 group) are in conjugation with vacant ‘p’ orbital on
C+H2, as shown below

In (CH3)3C+, there are nine C–H σ bonds which participate in delocalization


with carbon carrying positive charge (overlap of C–H σ orbital with vacant p
orbital on carbon carrying positive charge). Thus for hyperconjugation in tertiary
butyl cation, nine possible contributing structures can be obtained. In case of
(CH3)2CH+, there are only six C–H σ bonds available for participation in
delocalization and only six contributing structures are possible. Thus, (CH3)3C+
is more stable than (CH3)2CH+ and for the same reason, (CH3)2CH+ is more
stable than CH3CH2+. The CH3+ is least stable as no C–H σ bonds are present
to overlap with the vacant p orbital of carbon.

Thus, the overall stability of carbocation can be written as

(CH3)3C+ > (CH3)2C+H > CH3C+H2 > C+H3


Stability of allyl and benzyl cations: Explanation through resonance
Let us consider allyl and benzyl cations

Both are primary (1o) carbocations but are highly stable. The stability of these
carbocations
is attributed to the resonance effect. In both the cases, the vacant p orbital on
carbon
bearing positive charge is in conjugation with π bonds and thus delocalization
occurs through
p-π overlap.

In allyl cation, delocalization occurs through resonance effect (–R effect) and it
is a resonance hybrid of two contributing structures. Since, in general, resonance
effect dominates inductive effect, the carbocation which involves delocalization
through resonance effect are more stable than those where inductive effect
causes the delocalization.
In benzyl cation, the positive charge on carbon is in conjugation with π electrons
of benzene ring and undergoes a –R effect. Both, allyl and benzyl cations are
stabilized by resonance but benzyl cation is more stable compared to allyl
cation because in benzyl cation, the number of contributing structures are more
as compared to allyl cation (Recall more the number of contributing structures,
more is the stability).
In benzyl cation, the replacement of hydrogen by phenyl group results in the
formation of diphenyl and triphenylmethyl cations.

The order of stability of these resonance stabilized cations is [C] > [B] > [A]. In
[A], the positive charge is delocalized over one benzene ring while in [B] and
[C], the positive charge is delocalized over two and three benzene rings
respectively. The number of contributing structures is maximum for [C] i.e. nine
while it is six and three for [B] and [A] respectively.
The carbocations are reactive intermediates which are short lived. One of the
most stable carbocation, namely the ‘tropylium ion’ has been extensively
studied. The stability of tropylium ion (cycloheptatrienyl cation) is attributed to
extensive delocalization.

Thus, we can summarize the stability order of different carbocations discussed


so far, as follows:
Overall stability order of cations (stabilized through inductive (or
hyperconjugation) and resonance) is

The allyl and benzyl cations which are substituted at carbon carrying positive
charge are found to be more stable than 3° alkyl cations.

4.2.2 Carbanions
In an organic molecule heterolytic cleavage of a bond where carbon retains the
shared pair of electrons, results in the formation of carbanion. In this case,
carbon carries negative charge. In general, carbanions are formed by the removal
of hydrogen from a carbon in presence of a strong base. The formation of
carbanion is facilitated if the carbanion is attached to an electron withdrawing
group. For example,

In a carbanion, carbon is bonded to three atoms or groups (trivalent) and has


eight electrons (octet). So, it behaves as an electron rich species. A carbanion is
sp3 hybridized where one of the hybrid orbital has a lone pair of electrons. A
carbanion has a pyramidal geometry.

If a carbanion is attached to three different groups or atoms


then it is expected to be optically active. However, an asymmetric
carbanion shows loss in optical activity because of rapid oscillation
of lone pair and carbon from one side of the plane to the other.

In organometallic compounds, the alkyl group behaves as a carbanion, due to


electropositive nature of metal. Once a carbanion (electron rich species) is
formed, it may react with an electron deficient species i.e. electrophile.
Stability of Carbanions
A carbanion may be 1o, 2o, or 3o depending upon the number of carbons
attached to negatively charged carbon. Further, the carbanions may be
categorised as alkyl, allyl, benzyl or vinyl anions. For example

The stability aspects of different categories of carbanions may be explained


through inductive and resonance effect.
Stability of alkyl carbanions: Explanation through inductive effect
In a carbanion, the carbon carries negative charge. If the groups attached to
carbanion are electron releasing in nature, they intensify (increase) the negative
charge on carbon and destabilize the system.
The alkyl groups are electron releasing in nature due to inductive effect (+I
effect). More the number of alkyl groups attached to carbanion, more is the
intensification of negative charge on carbon and lesser will be the stability. This
is the reason why 3o carbanion is least stable and methyl carbanion is most
stable.

Thus, the order of stability of methyl substituted carbanions is as follows:

Stability of allyl and benzyl anions: Explanation through resonance


Let us consider the case of allyl anion and benzyl anion. As compared to primary
alkyl carbanions, the allyl and benzyl anions are relatively more stable. These
carbanions represent conjugated system where the lone pair on carbon
participates in delocalization with π bond (p–π overlap). Thus, all the carbons in
allyl and bezyl anions are sp2 hybridized.
The alkyl carbanions have pyramidal structure as the carbon is sp3
hybridized.
In conjugated carbanions like allyl and benzyl anions the negative charge is
stabilized through resonance.
The conjugated carbanions like allyl and benzyl anions are planar in
structure (sp2) as planarity ensures an effective overlap of orbital carrying
lone pair with π-bond(s).

The negative charge delocalization occurs in allyl and benzyl anions through
resonance effect
(+R effect). The resonance effect contributes towards the stability of these
anions. A benzyl anion is more stable compared to allyl anion because of
more number of contributing structures.
Similarly, the diphenylmethyl anion and triphenylmethyl anion are more stable
compared to benzyl anion. In diphenyl methyl anion the negative charge is
delocalized over two benzene rings whereas in triphenylmethyl anion the
negative charge is delocalized over three benzene rings. The order of stability of
phenylsubstituted methyl anions is thus,

4.2.3 Free Radicals


Free radicals are formed in the organic reactions when homolytic cleavage of
bond takes place. In general, the free radicals are formed in the reactions which
occur in the presence of light or at high temperature or in the presence of organic
peroxides such as benzoylperoxide. Free radicals are the species with a single
electron and thus, they are electron deficient in nature (a species with single
electron always tends to pair up with another electron and thus, looks for an
electron rich site).
A carbon free radical is sp2 hybridized with a p orbital carrying single unpaired
electron. A free radical has a planar geometry. The carbon in a radical is
trivalent and has seven electrons (septet).

A radical, once formed, reacts immediately to extract another radical from a


bond and thus, results in generation of new radical. For this reason, the reactions,
which involve free radicals as intermediates, are chain reactions. Such reactions
terminate only when two free radicals combine with each other.
Stability of free radicals
A free radical may be 1o, or 2o, or 3o, depending upon the number of carbons
attached to carbon carrying single electron. Further, the free radicals may be
categorised as alkyl, allyl or benzyl radicals.
For example
The stability aspects of different categories of free radicals may be explained
through inductive, hyperconjugation and resonance effect.
Stability of alkyl free radicals: Explanation through inductive effect
Free radicals are electron deficient species where carbon carries a single
unpaired electron. The alkyl groups release electrons through inductive effect (+I
effect). More the number of alkyl group attached to a carbon radical, more is the
availability of electrons and more is the stabilization of free radical. This
electron deficiency in carbon radicals is compensated to maximum in 3o radicals
because of the presence of three electron releasing groups. Thus, the order of
stability of methyl substituted free radicals is as follows:

Stability of alkyl free radicals: Explanation through hyperconjugation


The stability of free radicals may be explained not only through inductive effect
but also through hyperconjugation. The C–H σ bond which is in conjugation
with the p orbital carrying a single electron, participates in delocalization. For
example, in case of ethyl radical (CH3 H2), the three C–H σ bonds (of CH3
group) are in conjugation with the p orbital on H2 (carrying a single electron),
as shown below:
In (CH3)3C•, there are nine C–H σ bonds which participate in delocalization
with p orbital of the carbon carrying single unpaired electron, thus, nine
contributing structures can be obtained. In case of (CH3)2C•H, there are only
six C–H σ bonds available for participation in delocalization and only six
contributing structures are possible. There are three C–H σ bonds in CH3CH2•,
available for participation in delocalization and only three contributing
structures are possible. Thus, (CH3)3C• is more stable than (CH3)2CH• which
in turn is more stable than CH3CH2•. The CH3• is least stable as there is no C–
H σ bond available for participation in delocalization with p orbital of the carbon
carrying single unpaired electron.
Thus, the overall stability of free radicals can be given as
(CH3)3C• > (CH3)2C•H > CH3C•H2 > C•H3
Stability of allyl and benzyl radicals: Explanation through resonance
The allyl and benzyl radicals are 1° free radicals but in comparison to 1° alkyl
radicals the allyl and benzyl radicals are more stable. The stability of these
radicals is attributed to resonance effect. In both the cases the p orbital, on
carbon, carrying a single electron is in conjugation with π bond and thus
delocalization occurs through p-π overlap.

The benzyl radical is more stable compared to allyl radical, because in


benzylic radical, number of contributing structures are more as compared to
those in allylic radical. Due to same reasons the diphenylmethyl radical (6
contributing structures) and triphenylmethyl radical
(9 contributing structures) are more stable compared to benzyl radical (3
contributing structures).

Due to resonance effect, the allyl and benzyl radicals are more stable than 1o,
2o, or 3o alkyl free radicals. Thus, we can summarize the stability order of free
radicals discussed so far, as follows:
Benzylic > allylic > 3o > 2o > 1o > C•H3
4.2.4 Carbene
The neutral species which have divalent carbon with an unshared electron pair
are termed Carbenes. In carbenes, carbon has six electrons (sextet) and hence,
carbenes are electron deficient in nature. The carbene carbon may be sp2 or sp
hybridized.

The carbene with sp2 hybridized carbon has planar shape. The hybridized carbon
has a vacant p orbital and electron pair is present in sp2 hybrid orbital. Thus,
ground state of a planar carbene has a singlet state.
The carbene with sp hybridized carbon has a linear shape. The hybridized carbon
has two pure p orbitals each of which carries an unpaired electron. Thus, ground
state of a linear carbene has triplet state.

One of the most familiar reaction, that is, Reimer Tiemann reaction involves
carbene as a reactive intermediate.

Carbene is also formed by reaction of diiodomethane and zinc-copper alloy


(Simmons–Smith reaction) as discussed in section 5B.3.1. Carbene can insert
itself into a sigma (σ) or pi (π) bond during chemical reactions.

4.2.5 Nitrene
Nitrene can be considered as the nitrogen analogous of carbene. Nitrene is
monovalent and has two unshared pairs of electrons on nitrogen. In nitrene, six
electrons (sextet) are present and it is electron deficient. Nitrene is an important
reactive intermediate in Hofmann degradation and other related reactions.
4.3 REAGENT TYPES

4.3.1 Electrophiles and Nucleophiles


An electrophile is an electron deficient species. It may be an electron deficient
positively charged species or an electron deficient neutral molecule. In a
reaction, the electrophiles get bonded to a site, which is rich in electrons.

The H4 is a positively charged species but is not an

electrophile. The NH4+ has complete octect and thus charge on


nitrogen is not due to its electron deficiency but is due to higher
valency. Similarly, oxonium ions

and
are not electrophiles as positive charge on oxygen is due to higher
valency.
Nucleophiles are electron rich species and in general any nucleophile bears
atleast one lone pair of electrons. The nucleophiles may be negatively charged or
neutral molecules. In a reaction, they always seek a site to which electrons can
be donated, that is, nucleophiles get bonded to electron deficient centres.
Electrophiles and nucleophiles can be classified under two categories, namely
charged and neutral. The classification of electrophiles and nucleophile has been
summarized in Table 4.1.
Electrophilicity describes the relative strength of electrophiles, that is, the speed
with which they react during a reaction while nucleophilicity is the term used for
describing the relative strength of nucleophiles, that is, the swiftness with which
a nucleophile reacts during a reaction.
4.4 TYPES OF REACTIONS
Chemical reaction basically involves the cleavage of existing bonds and
formation of new bonds. As we have already discussed the structure of molecule,
their reactive sites, electronic displacements, reactive intermediates, and type of
reagents (electrophiles and nucleophiles), now we are in a position to classify the
general organic reactions into the following categories:
1. Substitution reactions
2. Addition reactions
3. Elimination reactions
4. Rearrangement reactions
5. Oxidation and reduction reactions
Here we are giving a brief introduction to the different types of reactions
mentioned above. The chemistry of functional groups which has been discussed
in subsequent chapters is mainly the study of the reactions of these categories. A
detailed discussion on these reactions is dealt separately at appropriate places
through out the text.
Substitution reactions
These reactions involve the displacement of an atom or a group by another atom
or group. Substitution occurs in highly stabilized systems. For example, it takes
place at
(i) sp3 hybridized carbons as in alkanes, haloalkanes, and alcohols.
(ii) sp2 hybridized carbons of benzene and its homologous aromatic
compounds.
These reactions may further be classified as free radical, electrophilic, or
nucleophilic substitution reactions.
Addition reactions
These reactions occur in unsaturated systems where
(i) sp2 hybridized centre gets converted into sp3 hybridized centre
(ii) sp hybridized centre gets converted to sp2 or sp3 hybridized centre.
A number of reactions in alkenes, alkadienes, alkynes, carbonyl compounds etc.
fall in this category. Further, these reactions may be categorised as free radical,
electrophilic or nucleophilic addition reactions.
Elimination reactions
These reactions involve removal of two atoms or groups from a molecule. The
elimination of two atoms (or groups) may occur from same atom or different
atoms in molecules and accordingly the elimination reactions may be classified
as
(i) α-Elimination or 1,1-elimination: For example formation of carbene from
chloroform in presence of a base involves a 1,1-elimination of HCl from
chloroform molecule.

(ii) β-Elimination or 1,2-elimination: The elimination of two atoms or groups


from the adjacent atoms in a molecule as in case of haloalkanes and
alcohols. This results in the formation of unsaturated π bonded system.
(iii) 1,3- and 1,4-eliminations: These types of eliminations cause the formation
of cyclic compounds.
Rearrangement reactions
The rearrangement reactions may be considered as internal substitution reactions
where in the course of reaction the atoms or groups migrate within the molecule
to form the product. The migratory aptitude of groups is in accordance with the
stability of intermediates formed during the course of reaction.
Oxidation and reduction reactions
Oxidation reactions involve the addition of oxygen or removal of hydrogen
while the reduction reactions involve the addition of hydrogen or removal of
oxygen. Further, the change in the oxidation state of carbon in the course of
organic reactions may also be classified as oxidation or reduction reaction.
Following chart provides a tabulated summary of different types of reactions and
the location where related information can be found in the book.
The type of chemical reactions mentioned above involve bond breaking that
may occur through heterolytic fission (involve ions) or homolytic fission
(involve free radicals). Now we will introduce in brief another type of reaction
known as pericyclic reactions.
Pericyclic reactions
In pericyclic reactions, the bond breaking and bond formation proceed
simultaneously in a single step through a cyclic transition state. Such reactions
are said to be concerted and do not involve ions or radicals as reaction
intermediates. For this reason, at times these reactions are also termed as no
mechanism reactions. Polar reagents, solvents or catalysts do not affect the
pericyclic reactions. These reactions are affected by heat (thermal) or light
(photochemical) and are highly stereoselective.
The details of pericyclic reactions are beyond the scope of this book. However
some of these reactions are introduced at appropriate places in the text. These
include mainly,
(i) Diels Alder reaction (refer section 7.4.3)
(ii) Claisen rearrangement (refer section 16.2, p. 531)
4.5 CHEMICAL ENERGETICS
Every system tries to attain a lowest energy state. The energy is inversely related
to the stability of system. A system with least energy is most stable whereas a
system with highest energy is least stable. Chemical reactions involve the
interaction of reactant molecules to form products. The overall process involves
cleavage as well as formation of bonds. The product formation occurs only if it
is energetically favoured.
A number of questions may arise in our mind regarding a chemical reaction such
as whether a reaction is feasible or not; how fast or slow the reaction rate is;
what factors affect the progress of a reaction; which pathway is followed during
a reaction, and so on. The study of thermodynamics and kinetics of a reaction are
the tools which answer all these queries.

4.5.1 Thermodynamics and Kinetics of Chemical


Reactions
The chemical reactions are governed by two important factors, namely
thermodynamics (this deals with the ‘change in energy’ during a reaction) and
kinetics (this deals with the rate and mechanism by which a reaction proceeds).
To develop a better understanding of the thermodynamics of a reaction, one must
be familiar with certain terms. These can be enumerated as follows:
(a) Internal energy: It is the stored energy of a system which includes all
possible forms of energy attributed to that system. The internal energy may
include the attraction and repulsion between atoms, molecules, ions or
particles that make up the system as well as the kinetic energy and potential
energy of the system.
(b) Enthalpy, H: Enthalpy is the heat content of a system at constant pressure.
For convenience, the relative energy of a system is described in terms of its
heat content or enthalpy.
(c) Enthalpy change, ΔH: This is the heat change in which is calculated as the
difference in the enthalpies of reactants (HR) and products (HP) involved in
a reaction i.e. ΔH = HP –HR
The ΔH value depends on the strength of the bonds of the reactants and
products and can be calculated from bond dissociation energy, DE as
follows:
ΔH = (Sum of bond dissociation energies, DE of bonds broken) – (Sum
of bond dissociation energies, DE of bonds formed)
ΔH value is negative—if bonds formed during reactions are stronger than
bonds broken, i.e. HR > HP. Heat is released during such reactions and the
reactions are termed Exothermic.
ΔH value is positive—if bonds broken during reactions are stronger than
bonds formed, i.e. HP > HR. Heat is supplied during such reactions and the
reactions are termed Endothermic
(d) Entropy, S: Entropy is the measure of degree of randomness (disorder) in a
system.
(e) Entropy change, ΔS: It is the difference in entropies of reactants, SR and
products SP.
Therefore, ΔS = SP –SR
(f) Energy of activation, Ea: The minimum amount of energy required for a
reaction to occur is called its energy of activation. A lower value of Ea
indicates a fast reaction.

4.5.2 Chemical Equilibrium


All chemical reactions are reversible in nature where the interconversion of
reactants and products takes place to various extents. A state of equilibrium is
reached when the concentration of reactants and products no longer changes
even though reaction continues to occur in forward and backward direction. For
a reversible reaction,
aA + bB cC + dD
The reaction equilibrium is expressed by equilibrium constant K.

A large value of K indicates that the formation of products is favoured. If the


equilibrium lies exclusively towards the side of the products, the reaction is said
to undergo completion (in such cases, the reverse arrow is omitted). A small
value of K indicates that formation of reactants is favoured.
A + B ↦ C + D 99.9% of equilibrium shifts towards the products.
The equilibrium constant is related to free energy change, ΔG by the relation
ΔG° = – 2.303 RT log K
where, R is a molar gas constant (8.314 JK–1mol–1) and T is absolute
temperature.
The equilibrium constant K varies with temperature.
At constant temperature, the free energy change, ΔG is related to enthalpy and
entropy of a reaction by the equation
ΔG = ΔH – TΔS where T is absolute temperature.
The value of ΔG calculated from this equation gives important information
regarding the reaction which can be given as
ΔG < 0 Indicates a spontaneous reaction.
ΔG = 0 Indicates reaction is at equilibrium.
ΔG > 0 Indicates reaction is not feasible in forward direction.

4.5.3 Rate of Reaction


The energy change during a reaction tells us about the feasibility of a reaction
but it does not say anything about how fast or slow a reaction will occur, that is,
about rate of a reaction. During a reaction, the reactants interact and form
products. As the reaction progresses, the concentration of the reactants decreases
and that of the products increases. The rate of a reaction is defined as the rate of
change in concentration of reactant or product with respect to time. For an
elementary reaction,

In this rate equation, ‘a’ is order of reaction with respect to reactant ‘A’ and ‘b’
is the order of reaction with respect to reactant ‘B’. The overall order of this
reaction is ‘a + b’.
In a multi-step reaction, the slowest step is the rate determining step. The
collision of reactant molecules results in the formation of products. The number
of reactant species colliding in a rate determining step determines the
molecularity of a reaction. If only one species reacts, the reaction is
unimolecular; if two species collide and react in the rate determining step it is
bimolecular. The factors affecting the rate of reaction are:
(a) Energy of activation, Ea: Lower the value of energy of activation, Ea,
higher is the rate of reaction.
(b) Concentration of reactants: The higher the concentration of reactants in
the rate determining step, higher is the rate of reaction.
(c) Temperature, T: The rate of reaction increases with an increase in
temperature.
(d) Nature of solvent: Rate of the reaction may increase or decrease depending
upon the type of solvent used.
(e) Use of catalyst: A catalyst provides an alternate path for a reaction with
lower Ea value and thus, increases the rate of reaction.

4.5.4 Energy Diagrams (or Energy Profile) of


Chemical Reactions
The energetics of a reaction describes the energy associated with different
components (reactants and products) during the progress of reaction. Energy
diagram (or energy profile) is a plot of energy along Y axis and reaction
coordinate (progress of reaction) along X axis. This is used for studying the
energetics of reaction.
A reaction proceeds when the reactant molecules collide and have a proper
orientation. The pathway from reactants to products involves a transition state. A
transition state is a hypothetical state which gives a clear picture of the
orientation of reactant molecules during collision, that is, a process of the bonds
formed and broken during the reaction. The transition state is the highest energy
state of a reaction.
The energy diagrams for a reaction can be represented as follows:

Fig. 4.1 (a) Energy change in a chemical reaction; energy profile of exothermic (b) and endothermic (c)
reactions.

Fig. 4.2 Energy profile of a Fast (low Ea) and slow (high Ea) reaction.
Fig. 4.3 Energy profile for two step reactions showing transition states (T.S. I and T.S. II) and the
intermediate.
4.6 STERIC EFFECT
The presence of groups or atoms in close proximity of each other causes van der
Waal repulsions and this phenomenon is termed steric effect (also known as
steric hindrance). Steric effect largely depends upon the size (bulkiness) of the
group or atom and arises from their arrangement in reacting species.
This effect has a marked impact on the rate of reaction, its nature and the extent
to which a reaction proceeds under a given set of conditions. Even if the reaction
conditions are favourable, there may be a change or inhibition of reaction due to
steric effect. Similarly, steric effect may also alter the acidic and basic strengths
of organic molecules.

The stability of molecules their chemical reactivity and few of the physical
properties (like acidity and basicity) are greatly influenced by steric effect and
some common examples are tabulated in Information Chart-II.
4.7 SOLVENTS IN ORGANIC REACTIONS
A homogeneous mixture of two or more components (a solute and a solvent) is
known as solution. In organic chemistry, almost all the reactions are carried out
in an appropriate solvent. In a solution, solvent molecules surround the
molecules or ions and their interaction is known as solvation. The nature of
solvent plays an important role in carrying out a reaction. Solvents are broadly
classified into two categories, namely
(a) Protic and aprotic solvents, and
(b) Polar and non-polar solvents.
Protic solvent
A solvent which contains a hydrogen attached to an electronegative atom such as
nitrogen and oxygen, is termed protic solvent. For example,

Aprotic solvent
A solvent in which hydrogen is not attached to an electronegative atom such as
nitrogen and oxygen, is termed aprotic solvent. For example,

Polar solvents
The solvents having dielectric constant, ε value of 15 and above are termed as
polar solvents. A polar solvent may be protic or aprotic in nature. For example,

Non-polar solvents
The solvents which possess dielectric constant ε value below 15 are termed non-
polar solvents. For example,

The solubility follows the thumb rule ‘like dissolves like’, that is, a polar
substance is soluble in polar solvent and non-polar substance is soluble in non-
polar solvent. A solvent may solvate either the reactants or reaction
intermediates and thereby, cause an increase or decrease in the rate of reaction.
4.8 ORGANIC COMPOUNDS AS ACID AND
BASES
In Chapter 1 (Sec. 1.10) two concepts are explained for defining the compounds
as acid and base. One is Bronsted and Lowry concept according to which an acid
is a proton donor and base is proton acceptor. The second is Lewis concept
according to which an acid is an electron pair accepter and base is an electron
pair donor. The organic compounds like carboxylic acids, phenols, alcohols
show acidic character where as amines, amides etc. exhibit basic character.
Further terminal alkynes, active methylene compounds, α-hydrogen containing
nitrile and nitro compounds also exhibit acidic character.

These factors are discussed at appropriate places in the text along with the
chemistry of concerned functional groups in the subsequent chapters.
EXERCISES
1. What is the hybridization state of carbon in carbocation, carbanion and free
radical?
2. Write short note on heterolytic and homolytic cleavage.
3. How does inductive effect differs from electromeric effect?
4. What is hyperconjugation? How does it differ from resonance?
5. Explain the stability of following carbocations in terms of inductive effect
and hyperconjugation.

6. What do you understand by isovalent and sacrificial hyperconjugation?


7. What is the order of stability of following reactive intermediates?

8. Why is is less stable than ?


9. Label the energy diagram (Fig. 4.4) in terms of (a) reactants, (b) products, (c)
transition state, (d) energy of activation, and (e) ΔH.
10. In the energy diagram (Fig. 4.5) of the reaction from A to E
(a) label the reactants and the products.
(b) label the number of transition states.
(c) label the intermediates.
(d) indicate the fastest step of the reaction.
11. What are carbenes? Are carbenes electrophilic or nucleophilic in nature?
12. Select the electrophiles and nucleophiles from the following.

13. Name the category (type) of reaction to which the following belong.
Problem 1. Which of the following compounds are related as constitutional
isomers? Write also IUPAC names for these isomers.

Solution. Constitutional isomers have same molecular formula but different


bond connectivity (refer p. 28)
Compounds with molecular formula C4H8 along with IUPAC names are:
(f) 2-methylprop-1-ene (l) cyclobutane (m) methylcyclopropane (j) but-2-
ene
Compounds with molecular formula C5H8 along with IUPAC names are:
(b) bicyclo[2.1.0]pentane (e) penta-1,4-diene (k) cyclopentene
Compounds with molecular formula C5H10 along with IUPAC names are:
(a) 2-methylbut-1-ene (i) methylcyclobutane
Compounds with molecular formula C6H10 along with IUPAC names are:
(d) cyclohexene (g) methylidenecyclopentane
Compounds with molecular formula C7H12 along with IUPAC names are:
(c) spiro[3.3]heptane (h) bicyclo[3.2.0]heptane
Problem 2. What functional groups will be present in constitutional isomers
with molecular formula C3H6O? Write the structure for all the possible
constitutional isomers.
Solution. There will be nine possible isomers for the molecular formula C3H6O.
Since molecular formula C3H6O has one oxygen, the possible functional groups
are:
(i) Alcohols
(ii) ethers
(iii) aldehydes/ketones and
(iv) enols (tautomers of aldehydes of ketones).
The given molecular formula corresponds to CnH2nO , which indicates an
unsaturated or cyclic alcohol. The corresponding ethers will be cyclic or
unsaturated ethers. The aldehydes and ketones are functional group isomers
corresponding to molecular formula C3H6O and both will have corresponding
tautumeric forms also.

Problem 3. Using curved arrows draw the contributing structures for the
following. Indicate which structure is more contributing and why?

Solution.
Problem 4. Classify the following as Lewis acid, Lewis base or neither:

Solution. Lewis acid is an electron deficient species that accepts an electron pair
to form a covalent
bond. A Lewis base on the other hand donates a pair of electron during the
formation of covalent bond. Thus, among the given examples the Lewis acids
and bases are as follows:
Problem 5. 2,4-dinitrophenol has pKa = 4.1 and nitromethane has pKa = 10.2.
Which one is a stronger acid? Also explain whether reaction given below will go
to right or left?

Solution. The acidic strength is inversely related to pKa. A small pKavalue


indicates strong acid. Thus 2,4-dinitrophenol (pKa = 4.1) is stronger acid than
nitromethane (pKa = 10.2).
A strong acid has weak conjugate base.
A weak acid has strong conjugate base.
The conjugate base for given acids can be written as:

The given reaction will go to left to form weak acid and weak base from strong
acid and strong base (on right side)
Problem 6. Give the IUPAC nomenclature for the following bridged
hydrocarbons.

Solution. The bicyclic bridged compounds have two or more carbon atoms
common to both the rings. The common atoms are designated as bridgeheads.
These are named as bicyclo [x.y.z] alkanes, where x, y and z are the number of
carbons in each of the three bridges connecting the bridgeheads. The following
steps are considered for naming the bridged molecules in accordance with the
IUPAC system:
(1) The total number of carbon atoms in all the rings including bridge atoms
is counted from which the parent name of the compound is derived.
(2) Further the total number of cyclic systems present in the molecule is
located and the system is designated as bicyclo, tricyclo, and so on
depending on the number of cyclic systems.
(3) Now the number of carbon atoms, around the bridged carbon atoms, is
counted individually. Then these numbers are written in the decreasing
order inside a square bracket.
(4) The numbering starts from one of the bridgehead and proceeds through
the larger ring to other bridgehead.
Keeping these steps in consideration, the IUPAC nomenclature of structure (i)
can be done as:
The total number of carbon atoms in the rings is six and it is an alkane. Hence,
its parent name is hexane. Further the number of carbon atoms in a skeleton
around the bridged carbons are 2, 2, and 0 respectively and it is bicyclic. Thus,
its IUPAC name is bicyclo[2.2.0]hexane.
Similarly, The IUPAC name for structure (ii) and (iii) can be written as

The IUPAC name for (iv) is bicyclo[2.2.1]heptane, already given for structure
(ii), (Only the structure is written in a different way).
Problem 7. Give the IUPAC name for the following spiral compounds

Solution. The spiral compounds are bicyclic compounds that have single carbon
common to both the rings. This common carbon is designated as spiro atom.
These compounds are named as
spiro [x. y] alkane. The x and y are the number of carbon atoms in each ring
joining the spiro atom and are written in square bracket followed by the parent
name derived from the total number of carbons in a system.
The numbering of such systems starts from the ring atom next to spiro atom
and proceeds from this ring through spiro atom to another ring.
If spiro compound is substituted then numbering starts from the ring carrying
the substituent and continues further to other ring.
Thus, spiro compounds [I] and [II] can be given the IUPAC names as follows:

If two rings are of different size than numbering starts next to spiro atom and
proceeds around the smaller ring first then through the spiro atom proceeds
around the second bigger ring.
If this type of spiro compound is substituted, even then numbering starts from
the smaller ring and continues further to other ring. The position of
substituent is numbered accordingly.
Thus, spiro compounds [III] and [IV] can be given the IUPAC names as follows:

Problem 8. In each of the following structures, identify the number of allylic,


benzylic and vinylic hydrogens present.

Solution.
Hydrogen attached to double bonded (olefinic) carbon is termed vinylic
Hydrogen attached to carbon present next to double bonded (olefinic) carbon
is termed allylic
Hydrogen attached to the carbon directly attached to benzene ring is termed
benzylic
Number of allylic, benzylic and vinylic hydrogens present in the given structures
are indicated below.
Each allylic hydrogen is indicated by grey screen, vinylic hydrogens are circled
and hydrogens with square around them indicate benzylic hydrogens.
Problem 9. Indicate whether the following two structures are identical or related
to each other as enantiomers? Can the stereoisomers of a molecule with one
chiral centre (stereogenic centre) be related as diastereomers?

Solution. The three dimensional structure (i) is first written in the Fischer
projection form. Further the priority is assigned to the group/atoms according to
sequence rule (see section 3.4). The decreasing order of priority is Br > CH3 > D
> H.
Now configuration is assigned to both structures (i) and (ii) by writing the
Fischer projection in such a way that least priority H atom lies at a vertically
downward position (see section 3.4.1).

The configuration of (i) is S and for (ii) is R. Hence, they are related as
enantiomers.
It should be noted that the molecules with one chiral centre can have only two
possible, stereoisomeric forms which are mirror images of each other. Thus, they
always exist as enantiomers and not as diastereomers.
Problem 10. Write the Fischer projection for (a) R-2,3-dihydroxypropanal
(glyceraldehyde)
(b) S-2-chloropropanoic acid.
Solution. To draw the Fischer projection from the given name of compound,
with desired configuration, proceed as follows:
Step 1: Write the structure of compound
Step 2: Identify the stereocentre
Step 3: Identify the four groups/atoms attached to the stereocentre and assign the
priority order in accordance with CIP rules (refer p. 75)
Step 4: Draw a cross (+) for Fischer projection and write the least priority group
vertically downward
Step 5: Now place the top three priority groups in clockwise manner (for R) or
in an anti-clockwise manner (for S) as desired
For “R” configuration
Groups/atoms can be arranged in the following manner across the vertical and
horizontal line on a cross.
(a) Following the steps mentioned above, the Fischer projection for R-2,3-
dihydroxypropanal (glyceraldehyde) can be written as
(b) Following the steps mentioned above, Fischer projection for S-2-
chloropropanoic acid can be written as

Problem 11. Indicate the number of stereoisomers possible for pent-3-en-2-ol


along with their structure.
Solution. The structure for pent-3-en-2-ol is,
As evident from structure the compound has only one stereocentre (n = 1) which
is C2. The number of stereoisomers will be two (2n = 21 = 2), which will be pair
of enantiomers having R-/S- configuration. At the same time it has the double
bond at C3-C4 that will result in geometrical isomers (E-/Z-). Thus total
stereoisomers will be four having configuration (2R, 3Z), (2R, 3E), (2S, 3Z) and
(2S, 3E), Structures are given below:

Problem 12. Give the R and S configuration, and their IUPAC names, for the
following compounds:
Solution. Assigning configuration at each stereocentre in (a):
The compound (a) has two stereocentres and procedure for assigning the
configuration at these stereocentres is as follows:

Assigning configuration at each stereocentre in (b):


The compound (b) has two stereocentre and procedure for assigning the
configuration at these stereocentres is as follows:

Assigning configuration at each stereocentre in (c):


The compound (c) has two stereocentre and procedure for assigning the
configuration at these stereocentres is as follows:

Assigning configuration at stereocentre in (d):


The compound (d) has one stereocentre and the configuration at this stereocentre
is as follows:

Thus the IUPAC name for compounds (a to d) along with configuration around
stereocentre are as follows:
(a) (1R, 2S)-1,2-dimethylcyclohexane (c) (1S, 2R)-1-bromo-2-
chlorocyclohexane
(b) (1R, 2R)-1,2-dimethylcyclohexane (d) (3R)-3-methylcyclopentene
Problem 13. In the above problem (no. 12) the compound (a) is named as (1R,
2S)-1,2-dimethyl-cyclohexane. Can it be named as (1S, 2R)-1,2-
dimethylcyclohexane?
Solution. The answer is NO.
In compounds when the same substituents are present at different stereocentres,
in such a way that in IUPAC the lowest possible number can be on either of the
carbon then the configuration decides the numbering for nomenclature.
In 1,2-dimethylcyclohexane numbering can start from either of the carbon
carrying methyl substituent. However, once configuration is assigned at these
carbons (stereocentre) then carbon having ‘R’ configuration gets preference in
numbering over carbon having ‘S’ configuration. The numbering can start only
from carbon with ‘R’ configuration and not from carbon with ‘S’ configuration.
Thus, the compound (a) is named as (1R, 2S)-1,2-dimethylcyclohexane and not
as (1S, 2R)-1,2-dimethylcyclohexane.

Correlating Dashed-wedged-line, Sawhorse, Newman and Fischer


projections (Problems 14–18)
Problem 14. Draw the Dashed-wedged-line, Sawhorse, and Newman projections
for all the possible staggered conformations of following compounds:
(a) 1,2-Dichloroethane
(b) 1-Bromo-1,2-dichloroethane
Solution. The Dashed –Wedged –Line structure may be present in the staggered
or eclipsed conformation. The group/atom on the wedge ( ) are placed on the
right side of the sawhorse projection. The group/atom on the dashed line (
) is written on the left side of the sawhorse projection. The group/atom on line (
) are placed on the top or bottom of sawhorse projection depending upon the
staggered or eclipsed conformation.
In a similar manner Dashed –Wedged –Line structure can be inter-converted to
Newman projection and vice versa.
For compound (a) one staggered conformation and for compound (b) three
staggered conformations b-(i), (ii) and (iii) are possible. The different possible
projections are as follows:
Problem 15. Indicate whether the following two sawhorse representations for
the compound CH3CHBrCH(OH)CH3, are related to each other as enantiomers,
diastereomers, or identical structures?

Solution. The sawhorse representations are written in a form where CH3 groups
are in the eclipsed position. The structure with eclipsed methyl group are then
written directly in Fischer projection form, which indicates that the two
structures are related as enantiomers in the manner as shown here.

Problem 16. The Newman projections for 3-bromo-2-chlorobutan-2-ol are


given. Write their Fischer projection and identify whether the two projections
exhibit enantiomeric, diastereomeric, or identical relationship.
Solution. The given staggered Newman projections are written in eclipsed form
with methyl groups eclipsing each other. The eclipsed form is then directly
written in sawhorse form, which is further written as a Fischer projection.

The Fischer projections clearly indicate that the two forms (i) and (ii) are
diastereomers.
Problem 17. (i) Name the molecule and indicate the carbon-carbon bond around
which free rotation is occurring as depicted by the Newman projection given
below.
(ii) Which of the following does not represent a conformer of C5H12?
Identify the molecules and indicate the carbon-carbon bond around which
free rotation occurs as depicted by following Newman projections?

Solution. (i) The given Newman projection indicates the presence of four
carbons (molecular formula C4H10) and thus, the molecule is butane. The
Newman projection represent the rotation around C1-C2 bond of butane as
shown:
(ii) As explained in the earlier problem, the given Newman projections (a-d)
indicate the presence of five carbons (molecular formula C5H12) whereas
the Newman projection
(e) indicates the presence of four carbons (molecular formula C4H10).
The Newman projections (a-d) represent conformers of isomeric pentanes.
Their structures and the carbon-carbon bond around which free rotation
occurs are given as follows:

The Newman projection (e) represents conformer of butane (section 3.6.7) and
indicates the rotation around C2-C3 bond of butane as shown:

Problem 18. What will be the Newman projection for the following:
Solution. In case of alkenes the rotation around C–C bond is restricted due to
double bond. More over the molecule has the planar structure because of sp2
hybridization. Thus, there is only one Newman projection possible for each of
the given alkenes.
Newman projections

Problem 19. How many staggered conformations are possible for 1,2-
dichloroethane? Why does dipole moment of this compound increase with
temperature?
Solution. 1,2-dichloroethane has three staggered conformations, namely anti
(staggered) conformation [I] (μ = 0), and gauche (skew staggered) conformations
[II] and [III] (μ ≠ 0). Here μ refer to the dipole moment of the molecule.

As temperature increases, the molecules gain energy and move from lower
energy (most stable) conformation [I] to higher energy (less stable)
conformations [II] and [III] and thus, the dipole moment of molecule increases
with increasing temperature.
Problem 20. Draw structures of given dimethylcyclobutanes and indicate which
one will be optically active ?
(a) 1,1-dimethylcyclobutane
(b) Cis-1,2-dimethylcyclobutane
(c) Trans-1,2-dimethylcyclobutane
(d) Cis-1,3-dimethylcyclobutane
(e) Trans-1,3-dimethylcyclobutane
Solution. Only (c) is optically active.
Structures of all dimethylcyclobutanes (a-e) are as follows:

Problem 21. Indicate which of the following substituted cyclohexanes are


optically active?
(a) Methylcyclohexane
(b) Cis-1,2-dimethylcyclohexane
(c) Trans-1,2-dimethylcyclohexane
(d) Cis-1,3-dimethylcyclohexane
(e) Trans-1,3-dimethylcyclohexane
(f) Trans-1,4-dimethylcyclohexane
(g) Cis-1,4-dimethylcyclohexane
Solution. Only (c) Trans-1,2-dimethylcyclohexane and (e) Trans-1,3-
dimethylcyclohexane are optically active. Rest (a,b,d,f and g) have plane of
symmetry and are optically inactive.
(c) Trans-1,2-dimethylcyclohexane is optically active as explained below:
Problem 23. Although 1,3-disubstituted cyclohexanes exist in chair
conformation, but trans-1,3-di-tert.butylcyclohexane does not exist in chair
form. Explain why, and write the stable conformation for the same.
Solution. In trans-1,3- disubstituted cyclohexanes, the chair conformation has
substituents occupying axial-equatorial (a,e) positions (refer p. 101). In trans-
1,3-di-tert.butylcyclohexane the chair form will have one of the tert.butyl groups
at axial position and other at equatorial position. For a bulkier tert.butyl group to
occupy an axial position will cause a large increase in potential energy due to 1,
3-diaxial interactions (refer p. 99). The trans-1,3-di-tert.butylcyclohexane,
instead of chair, exist in twist boat conformation, where both the tert.butyl
groups occupy nearly equatorial positions.
Problem 24.
(a) A sample of compound ‘X’ in chloroform (0.6 g/mL) at 25°C, when taken
in a cell of pathlength 1.0 dm, shows rotation of +3.0°. What is the specific
rotation of sample?
(b) What will be the change in observed value of α if (i) concentration of
sample ‘X’ is reduced to half (ii) the sample is taken in a cell of pathlength
5.0 cm.

Problem 25. A sample of aspartic acid has specific rotation +4.5° mL/g.dm. If
αobs = 0.9°, when taken in cell of pathlength 0.8 dm, calculate the concentration
of sample.

Problem 26. A sample of 2-butanol having concentration of 0.15 g/mL when


taken in a cell of pathlength 0.6 dm shows αobs = –0.49°. For (+) 2-butanol the
[α], is known to be 13.52° mL/g dm.
(i) Calculate the enantiomeric excess.
(ii) Calculate the percentage of (+) 2-butanol and (–) 2-butanol.

Problem 27. A mixture of (R)-2-bromooctane and (S)-2-bromooctane has a


specific rotation of +10°. The specific rotation of (R)-2-bromooctane is +21.6°.
Calculate the percentage of (R)- and
(S)-isomer of 2-bromooctane.

Problem 28. Arrange the following in the order of their increasing acidic
strength:

Solution. The acidic strength of these compounds can be explained on the basis
of stabilities of their corresponding conjugate base. A compound with most
stable conjugate base is most acidic.
The loss of proton (H+), in these cases, result in the formation of corresponding
carbanions (a conjugate base). The triphenylmethyl carbanion (Ia) is much more
stabilized as delocalization involves three benzene rings. In case of
diphenylmethyl cabanion (IIa) the delocalization of electron involves the two
benzene rings and in methylbenzene (IIIa) it involves only one benzene ring (see
p. 124). Thus the stability of conjugate base follows the order:
Thus, the acidic strength follows the order as
I > II > III
Problem 29. Give the decreasing strength of following species as a base:

Solution. All the three species are derived from triphenylmethane, that is,
triphenylmethyl
anion (I), triphenylmethyl radical (II) and triphenylmethyl cation (III). All these
species I, II, and III, are equally stabilized through delocalization. Thus the
Lewis acid and base concept explain the basicity, which states that the
availability of electrons is directly related to the basic strength of that species. A
carbanion, I has a pair of electron and hence most basic in the given examples.
Free radical, II and carbocation, III are electron deficient species. However, free
radical-II is a stronger base (because of the presence of an electron) as compared
to carbocation (III), which is electron deficient. In other words carbanion, I can
accept the H+ readily and thus is a strong base. A positively charged species has
least affinity for accepting a proton.
Thus, the basic strength follows the order as
I > II > III
Problem 30. Identify each of the given carbocations as primary (1º), secondary
(2º), or tertiary (3º) along with their vinylic or allylic nature (wherever possible).
Also give the IUPAC names for these carbocations.
Comment, if for these carbocations one writes corresponding carbanion or free
radicals, will their nature as primary, secondary, or tertiary will change or will
remain the same?
Solution.
(a) A primary carbocation is one in which the carbon bearing the positive
charge, is attached to one carbon.
(b) A secondary carbocation is one in which the carbon bearing the positive
charge is attached to two carbons.
(c) Likewise, a tertiary carbocation is one in which the carbon bearing the
positive charge is attached to three carbons.

Further in corresponding carbanions and free radicals there is no change in


nature such as methyl, primary, secondary, or tertiary because there is only
change in the charge on the carbon which is not affecting any carbon-carbon
bond.
Problem 31. In each of the following set of carbocations (a) and (b) explain
which one is more stable?
Solution.
(a) Carbocation II is more stable than I. Both carbocations I and II are having
double allylic system. However, carbocation II is further stabilized through
donation of lone pair on oxygen, which results in additional contributing
structure during delocalization.
Contributing structure for I

Contributing structure for II

(b) Carbocation II is more stable than I.


Carbocation I is allylic cation and donates its double bonded pair to stabilize
through delocalization.

Carbocation II is 2º in nature, the oxygen donates its lone pair and stabilizes
carbocation through delocalization.

Compared to bonderd pair donation in carbocation I, the lone pair donation in


carbocation II results in the formation of highly stable contributing
structure. Thus carbocation II is more stable.
Chapter5
Alkanes and Cycloalkanes

A. ALKANES
5A.1 INTRODUCTION
Alkanes belong to the class of hydrocarbons where all the carbons present in the
molecule are linked through single bonds (σ bonds) only and thus, referred as
saturated hydrocarbons. Alkanes follow the general formula CnH2n+2. The first
member of this class is methane (CH4). The carbon in alkanes is in sp3 hybrid
state and thus, has 4 hybrid orbitals. These hybrid orbitals of carbon overlap with
‘s’ orbital of hydrogen to form carbon–hydrogen σ bond or hybrid orbital on one
carbon may overlap with hybrid orbital on another carbon (sp3–sp3 overlap) to
form carbon–carbon σ bond. The sp3 hybridization of carbon in alkane provides
a tetrahedral shape to molecule (see Section 1.4.1).

Due to saturation, the alkanes are highly stable and show little reactivity, thus,
they are popularly known as paraffins (Latin: Para means little, affins means
affinity).
1°, 2°, 3° carbons. In alkanes (C3 and onwards), all the carbons are not
equivalent and are referred as primary (1°), secondary (2°), or tertiary (3°)
carbons depending upon the number of carbons attached.
The hydrogens attached to 1o, 2o, 3o carbons are referred to as 1°, 2°, 3°
hydrogens respectively.
The IUPAC nomenclature of alkanes has been discussed in detail in Chapter 2.
Few representative examples of alkanes are as follows.

Isomerism
Alkanes exhibit chain isomerism as well as stereoisomerism (conformations).
The conformations of alkanes have already been discussed in Chapter 3. Alkanes
containing four or more carbon atoms show chain isomerism. For example, the
chain isomers of C4H10 and C5H12 hydrocarbons are:
Isomeric alkanes of molecular formula C4H10

5A.1.1 Physical Properties


Lower alkanes (C1–C4) are gases, alkanes from C5–C17 are liquids with petrol
like odour whereas higher alkanes (C17 onwards) exist as colourless, odourless,
waxy solids. Alkanes are less denser than water.
Two most important physical properties of alkanes, namely the melting and
boiling point are briefly discussed here. Table 5.1 gives the boiling and melting
points of some alkanes.
Boiling point. The boiling point of alkanes increases with an increase in
molecular mass, that is, higher the number of carbons in an alkane, higher is its
boiling point.
For isomeric alkanes where the molecular mass is same, the boiling point
decreases with increase in branching. An increase in branching causes a
decrease in surface area and as a result the intermolecular forces of attraction
decreases thereby decreasing the boiling point (see also selected solved example
1).
Melting point. Alkanes do not show a smooth variation in melting points with
increased molecular mass and branching. Melting point depends on lattice
structure. It has been observed that in general, linear alkanes with even number
of carbon atoms possess higher melting point than those with odd number of
carbon atoms. The carbons in linear alkanes are in fact, not attached in a straight
chain but in a zigzag manner.
In the alkanes with even number of carbon atoms, the terminal carbons are on
opposite sides, this increases the surface area and hence the melting point while
in alkanes with odd number of carbon atoms, the terminal carbons lie on the
same side of the zigzag chain, this decreases the surface area and hence, the
melting point.
In isomeric alkanes the melting point decreases for branched chain alkanes (due
to decreased surface area). however, in highly branched but symmetrical alkanes
the melting point is exceptionally higher. A symmetrical structure fits better in a
crystal lattice thereby increasing the melting point. For example, neopentane (–
20°C) has higher melting point than isopentane (–159°C) due to its symmetrical
structure.
5A.2 PREPARATION OF ALKANES
Alkanes can be prepared by a number of methods. Some common and important
methods include catalytic hydrogenation of alkenes and alkynes, preparation
from haloalkanes, alcohols, carbonyl compounds, salts of carboxylic acids and
alkylboranes. This section offers an explicit discussion on various preparative
methods of alkanes.

5A.2.1 Catalytic Hydrogenation of Alkenes and


Alkynes
An unsaturated hydrocarbon (alkene or alkyne) does not react with hydrogen as
such but in the presence of a metal catalyst, addition of hydrogen occurs to form
alkane. This hydrogenation in presence of a catalyst is known as catalytic
hydrogenation of alkenes or alkynes.

Role of catalyst: As explained earlier, the minimum amount of energy required


for a reaction to occur is the energy of activation, Ea. Energy of activation for
hydrogenation is very high. For this reaction to proceed rapidly, a catalyst is used
which lowers the energy of activation.
The metal catalysts used for this process are platinum (Pt), palladium (Pd), or
nickel (Ni). Platinum and palladium are used in finely divided state adsorbed
over charcoal (Pt/C or Pd/C). Platinum is also used in the form of oxide PtO2
and is then known as Adam’s catalyst. Nickel is generally used at a higher
temperature (~300° C) and the hydrogenation under these conditions is called
Sabatier Senderens reaction.
5A.2.2 From Haloalkanes (Alkyl halides)
Haloalkanes are used as a convenient and versatile source to produce alkanes, as
mentioned in the subsequent sections.
By reduction
Reduction of haloalkanes in presence of different reducing agents directly results
in formation of alkane.

Different reducing agents used for this purpose are:


H2/Pt, Pd or Ni; Zn/HCl or CH3COOH; Na/ethanol; Zn-Cu couple, or LiAlH4.
For example,
Via Grignard reagent
Haloalkanes react with magnesium metal, in presence of dry ether, to give
alkylmagnesium halide (also known as Grignard reagent). In Grignard reagent,
the alkyl group is directly attached to magnesium metal and because of high
electropositive character of magnesium metal, the alkyl group exhibits a
carbanion character. Any compound which contains easily replaceable hydrogen
(i.e. with hydrogen attached to electronegative atoms such as oxygen, nitrogen,
or sulfur) as in case of water, alcohol amines, thiol, and so on, reacts with
Grignard reagent to produce alkane.

For example,

As water is most readily available, it may be used in the reaction with


Grignard reagent to produce alkanes. However this reaction has limited synthetic
utility. For the some reason, preparation of Grignard reagent and its reactions
(refer chapter 25) are carried out in anhydrous conditions, to avoid the formation
of alkane.
Wurtz reaction
The reaction of haloalkanes with sodium metal in presence of dry ether results in
the coupling of two molecules of alkyl halide to give an alkane.

Limitations of Wurtz reaction: Wurtz reaction, when applied to the synthesis of


unsymmetrical alkanes (alkane with odd number of carbon atoms like propane,
pentane and so on) by taking two different haloalkane molecules, results in the
formation of a mixture of alkanes. The mixture of alkanes, thus obtained, is not
easy to separate making the method impractical. For example, in an attempt to
synthesize propane, one ends up with a mixture of alkanes as shown here.

Mechanism. The reaction proceeds by an ionic mechanism in the steps that


follow.
Step 1. Formation of alkyl sodium

Step 2. Formation of alkane (Reaction of alkyl sodium with second molecule of


haloalkane).

In alkyl sodium, the alkyl group attains a carbanion character as it is attached


to highly electropositive sodium metal. The second molecule of haloalkane has a
positive charge on alkyl group being attached to highly electronegative halogen.
The reaction of second molecule of haloalkane with alkyl sodium results in the
formation of an alkane and is a nucleophilic substitution reaction.
Corey–House synthesis
Haloalkanes on reaction with lithium metal in inert solvent (dry ether) give
alkyllithium, which on reaction with cuprous iodide yields lithium dialkylcuprate
also known as Gilman reagent.

The Gilman reagent so produced, undergoes a coupling reaction with an alkyl


halide (same or different) where the alkyl group of Gilman reagent couples with
alkyl group of haloalkane to give an alkane.

The overall reaction to synthesize alkane as mentioned above is known as


Corey–House synthesis. This is a versatile method for the synthesis of all types
of alkanes, that is, symmetrical or unsymmetrical, in high yields. For example,

The
Lithium dimethylcuprate can further be used for preparation of ethane
(symmetrical) or propane (unsymmetrical) as follows
Via alkyllithiums
Alkyllithiums react with compounds containing active hydrogen such as water,
alcohol, and so on, to give corresponding alkane. These reactions of
alkyllithiums are similar to that of Grignard reagent (p. 173). For example

Further, the preparation and various reactions of alkylithium are also discussed
in Chapter 25.

5A.2.3 From Carbonyl Compounds (Aldehydes and


ketones)
The reduction of carbonyl compounds, converts C=O group to CH2 group and
results in the formation of alkanes.
Clemmensen reduction
The carbonyl compounds on reduction with zinc amalgam (Zn-Hg) in presence
of hydrochloric acid result in the formation of alkanes. For example
Wolff–Kishner reduction
The reaction involves the treatment of carbonyl compound with hydrazine
followed by heating with a base like potassium ethoxide which results in the
formation of an alkane.

5A.2.4 From Sodium Salt of Carboxylic Acids


By heating with soda-lime (Duma’s decarboxylation)
Decarboxylation results in the formation of alkanes when sodium salt of
carboxylic acids is heated with soda-lime.
Kolbe’s electrolytic process
The sodium or potassium salts of monocarboxylic acid on electrolysis produce
alkanes. This reaction results in the formation of symmetrical alkanes which
have higher number of carbon atoms than the parent carboxylic acid salt.
In general,

During electrolysis, carboxylate ion loses electrons and undergoes


decarboxylation to produce symmetrical alkane at anode as follows:
5A.2.5 Some Other Methods
By reduction with HI/red P
Monohydric alcohols, aldehydes, ketones and monocarboxylic acids undergo
reduction with HI in presence of red phosphorous to yield alkanes.

Role of red phosphorous. Under high temperature conditions, the iodine


produced during the reaction may cause iodination of alkanes. The red
phosphorous reacts with iodine and generates more of HI, which is used in the
reaction.
2P + 3I2 &MAP; 2PI3
PI3 + 3H2O &map; H3PO3 + 3HI

By hydrolysis of alkylboranes
Alkenes on reaction with borane form trialkylborane, which on hydrolysis with
acetic acid give corresponding alkane. For example,
5A.3 CHEMICAL PROPERTIES OF ALKANES
This section discusses various chemical properties of alkanes including free
radical substitution reactions (halogenation, nitration, sulfonation,
chlorosulfonation), oxidation reactions, aromatization, isomerization, and
pyrolysis reactions.

FREE RADICAL SUBSTITUTION REACTIONS


(Halogenation, Nitration, Sulfonation, Chlorosulfonation)

5A.3.1 Halogenation
The alkanes react with halogens, at high temperature or in presence of light, to
form haloalkanes.

The order of reactivity of halogens is


F2 > Cl2 > Br2 > I2
This subsection extensively discusses the reactions of alkanes with chlorine and
bromine. The various aspects of halogenation in alkanes that are discussed in
this section are:
(a) Chlorination
(b) Proof of free radical mechanism
(c) Bromination
(d) Fluorination and iodination
(e) Energy calculation in halogenation reaction of alkanes
(f) Reactivity and selectivity (chlorination versus bromination)
Chlorination
Reaction of alkanes with chlorine in presence of light or at high temperature
results in the formation of chloroalkanes.

Chlorination of methane. Reaction of methane with chlorine results in the


formation of chloromethane. The reaction does not stop at this stage and further
chlorination occurs till all the hydrogens are replaced by chlorine. This reaction
is shown below.

Mechanism. Chlorination (in general halogenation) is a substitution reaction and


follows the free radical mechanism. This involves three main steps as
discussed here.
Step 1. Chain initiation
Homolytic cleavage occurs in chlorine molecule to produce free radicals and the
energy for this cleavage is supplied by light or high temperature.

Step 2. Chain propagation


The chlorine radical once formed, propagates the reaction further by reacting
with methane to form methyl radical. The methyl radical further reacts with
chlorine molecule to give chloromethane and chlorine radical is also generated.

The highly reactive chlorine radical can react with chloromethane to abstract a
hydrogen radical similar to reaction (1) above. The presence of electronegative
chlorine in CH3Cl makes the abstraction of hydrogen much easier as compared
to methane.
The abstraction of hydrogen from dichloromethane and trichloromethane is even
easier compared to chloromethane. This is because of the introduction of more
number of electronegative chlorine atoms.

Step 3. Chain termination


The radicals in propagation step may be consumed by other radical, resulting in
the termination of chain reaction, that is, no further propagation occurs as free
radicals are not available.

Proof for free radical mechanism


The chlorination of methane in the presence of an organic peroxide (e.g.
benzoylperoxide) in dark results in the formation of a product, similar to one
obtained by chlorination of methane in presence of light.

The peroxides are known to be good free radical generators. They contain a
weak
oxygen–oxygen single bond, which gets homolytically cleaved to produce
radicals. Benzoylperoxide undergoes homolytic cleavage to produce phenyl
radicals as follows:

The phenyl radical combines with chlorine molecule to give chlorobenzene and
chlorine radical. The chlorine radical so generated reacts with methane leading
to chain propagation.

6
Reactions involving peroxides proceed through a free radical mechanism and
this clearly supports a free radical mechanism for reaction of methane and
chlorine in presence of light since in both the cases, same product is obtained.

NOTABILIA 6

The reaction can be made to halt at the chloromethane stage by taking higher
amount (concentration) of methane compared to chlorine. Similarly, chlorination
of ethane gives chloroethane.

Chlorination of propane. The chlorination of propane results in the formation


of two types of products, namely 1-chloropropane and 2-chloropropane. Unlike
methane and ethane, propane does not have equivalent hydrogens. In propane,
there are six 1° and two 2° hydrogens. The substitution of 1° hydrogen gives 1-
chloropropane, whereas the substitution of 2° hydrogen gives
2-chloropropane.

The propagation step of the radical mechanism explains the formation of two
products as

The order of stability of free radicals is

3o > 2o > 1o > H3


2° Radical being more stable, is formed faster compared to 1° radical and thus,
the product through a 2° radical (2-chloropropane) is formed in higher
percentage than that formed through 1o radical (1-Chloropropane). Thus, the
ease of abstraction of hydrogen from an alkane follows the order 3° > 2° > 1°
which is in accordance with the stability of alkyl free radicals formed by
abstraction of hydrogen.
Bromination
In the same way as chlorine does in chlorination, bromine also reacts with alkane
in the presence of light or at high temperature to produce bromoalkanes.
1-Bromopropane 2-Bromopropane

Fluorination and Iodination


The reaction of fluorine with alkanes is a highly exothermic reaction and is
practically uncontrolled at room temperature conditions.
R—H + F—F &map; R—F + HF + Heat [a violent reaction]
On the other hand, the reaction of iodine with alkane is highly endothermic and
thus, does not occur.

Energy calculation in halogenation reaction of alkanes


The two main factors affecting the possibility of a reaction (i.e. why a particular
reaction should occur and how readily will it occur) are as follows:
I. Energy of activation, Ea: This is the minimum amount of energy which must
be supplied for a reaction to occur. A low value of Ea indicates that reaction will
occur readily.
II. Heat of reaction, ΔH: Basically, a chemical reaction involves formation of
new bonds and breaking of existing bonds. During bond formation, energy is
released whereas to break a bond, energy is to be supplied. Bond formation and
bond breaking do not occur simultaneously in the transition state and in general,
bond formation occurs a little late. Energy released during formation is not
available for breaking of bond.
The heat of reaction, for the process involving both bond breaking and bond
formation, is calculated by the following formula:

For an exothermic reaction, ΔH has a negative value, i.e. energy is released


during overall process. For a process where ΔH has a positive value, the energy
is to be supplied during the reaction and the reaction is called endothermic
reaction. A relatively high endothermic process does not occur readily.
We now discuss the readiness of occurence of the halogenation reaction of
alkanes. The halogenation of alkanes involves two main steps, namely the chain
initiation step and the chain propagation step. And, it entirely depends on these
steps whether halogenation will occur or not. Let us calculate the energy
involved in these steps using Ea values and also the formula for calculating ΔH.
Table 5.2 depicts the dissociation energies for C–H, C–X, and X–X bonds.

Chain initiation step. This step involves a homolytic cleavage of halogen—


halogen bond and formation of halogen radical. The minimum amount of energy
required to start this reaction is the energy of activation and for this reason,
irradiation (light/UV) is necessary though the reaction occurs at room
temperature. The Ea of this step is same as ΔH as it involves dissociation of only
X–X bond.

The Lower the Ea value, more readily the reaction occurs to form free radical.
Surprisingly, the Ea value for chlorine is ‘maximum’ and as the data suggests,
the radical formation occurs relatively faster in case of fluorine and iodine. At
the same time, a free radical
once formed (irrespective of the ease with which it is formed) is sufficient to
carry out further reaction.
Thus, the Ea values in chain initiation step are not the only factor, which govern
the proceeding of halogenation reaction. So, along with Ea, we have to consider
the heat of reaction, ΔH involved in chain propagation.
Calculation of ΔH for chlorination of methane during chain propagation step
In chain propagation,

Thus, for reaction (2), ΔH2 = 243 – 352 = –109 kJ mol–1


The overall heat of reaction for the chain propagation step is the sum of ΔH1
values of reaction (1) and reaction (2), and is therefore evaluated as
ΔH (Chain propagation) = ΔH1 + ΔH2 = (+4) + (–109) = –105 kJ mol–1
In all, chlorination is an exothermic reaction. The calculation of ΔH values for
fluorination, bromination, and iodination can be carried out in the same way as
chlorination. The Ea and ΔH values for the halogenation of methane are
tabulated in Table 5.4.
Reactivity and selectivity (chlorination versus bromination)
The halogenation of alkanes containing different types of hydrogens (1°, 2°, and
3°) results in the formation of mixture of haloalkanes. The ease of abstraction of
hydrogen follows the order 3° > 2° > 1° > CH3. Let us consider the following
reactions:
The percentage of monochloro and monobromo products so formed indicates
that bromine is more selective in its reaction with alkanes as compared to
chlorine. Let us consider the energy profile of the chain propagation steps given
as

During chlorination and bromination reactions it is observed that:


Thus, chlorine is more reactive in its reaction with alkanes compared to bromine,
whereas, bromine is more selective in its reaction with alkanes compared to
chlorine.

5A.3.2 Nitration
The reaction of alkanes with nitric acid results in the formation of nitroalkanes.
Nitration may be carried out at a high temperature of 400–500°C (vapour phase
nitration) or it may be carried out at a low temperature of 150°C (liquid phase
nitration).

Nitration of alkanes forms all possible mononitro products (by abstracting


different type of hydrogens) as well as nitro compounds formed through all
possible carbon–carbon chain fission reactions.

The order of reactivity of alkanes is 3° > 2° > 1°.


Mechanism. The mechanism for nitration of ethane showing the formation of
nitroethane and nitromethane is as follows:
5A.3.3 SULFONATION
Alkanes react with fuming sulfuric acid (SO3 enriched H2SO4) to form
alkanesulfonic acid. Sulfonation reaction occurs only upto monosulfonation
stage.
R—H + HOSO3H &map; RSO3H + H2O
Lower alkanes do not udergo sulfonation easily unless 3° hydrogen is present.

Higher alkanes (C6 onward) undergo sulfonation which generally occurs at the
second carbon atom.

5A.3.4 Chlorosulfonation
The chlorosulfonation of alkanes results in the formation of alkanesulfonyl
chloride. The two reagents used for carrying out sulfonation are:
(a) Chlorosulfonic acid
(b) Mixture of chlorine and sulfurdioxide (Cl2 + SO2) in presence of light or
peroxide at a temperature of 40°–60°C.
5A.3.5 Oxidation Reactions
Combustion
Complete combustion of alkanes in the presence of air results in the formation of
carbondioxide and water as the final products. A general formula for combustion
of hydrocarbons is
Oxidation in presence of oxidizing agents
(i) Catalytic oxidation: In lower alkanes, a selective oxidation may result in the
formation of alcohol, aldehyde/ketone, or carboxylic acid. Specific reagents are
used for this purpose.
For example, methane results in the formation of methanal on oxidation with
MoO2 (at 400°C) and oxidation with Cu (at 200°C) results in the formation of
methanol in the following manner:
5A.3.6 Other Reactions
Insertion of carbene to alkanes
The alkanes undergo reactions with carbene (a reactive intermediate, section
4.2.4), and in the process C–H bond of alkane is converted into C–CH3 bond,
that is, insertion of carbene occurs at C–H bonds of alkane. The insertion of
carbene leads to the formation of a higher alkane and the reaction occurs
readily in a tert. hydrogen containing C–H bond.
The ease of insertion of carbene to C–H bonds follows the order 3° > 2° > 1°.

The insertion of carbene may follow a direct insertion mechanism or a free


radical mechanism. Direct insertion mechanism
(In case of singlet carbene)

Dehydrogenation
Lower alkanes (upto C5), on heating in presence of catalysts such as Al2O3,
ZnO, or V2O5, undergo dehydrogenation to produce alkenes.

Aromatization
Straight chain alkanes (C6 and onwards), on heating in presence of Al2O3 and
Cr2O3 at 500oC–600oC, result in cyclization followed by dehydrogenation. The
reaction results in aromatization of alkanes.
For example, n-Hexane undergoes aromatization to give benzene.
Similarly, n-heptane gives toluene under identical conditions. The reaction is
shown as

Cracking or pyrolysis
The cracking process involves thermal decomposition of an alkane into fractions
of lower molecular weight by fission of C–C and C–H bonds. The cracking
process is widely used in petroleum industry. Cracking may be of the following
types:
(i) Thermal cracking: Cracking is carried out at high pressure and high
temperature
(> 500°C).
(ii) Catalytic cracking: This process involves the use of catalysts such as
SiO2, Al2O3, and natural or synthetic aluminium silicate zeolites. The
process improves both quality and quantity of gasoline.
(iii) Hydro cracking: This process is a combination of both catalytic cracking
and hydrogenation and is used to produce gasoline in high yields.
(iv) Steam cracking: Cracking process carried out in presence of steam by
heating at
700–900°C followed by rapid cooling is termed steam cracking.
For example,

Mechanism (Free radical mechanism)

Isomerization
Isomerization of alkanes (C4 and onwards) occurs in presence of HCl/AlCl3 or
HBr/AlBr3 at a temperature of 300°C.
5A.4 PETROLEUM
Petroleum is a decomposed product of the remains of marine organisms and
plants formed over the ages, under high pressure and temperature conditions. It
is a dark viscous oil found in interstices in rocks under high pressure. Petroleum
along with natural gas serves as a major source of alkanes (upto C50–C70),
cycloalkanes, and aromatic hydrocarbons along with organosulfur compounds.
Natural gas is always found along with petroleum.
Natural gas
The chief constituent of natural gas is methane (95%) though along with this a
little fraction of ethane (~4%), propane, butane and isobutene are also present.
The uses of natural gas can be summarized as follows:
(i) It is used as fuel and is commercially supplied as CNG (compressed natural
gas).
(ii) Incomplete combustion of natural gas results in the formation of carbon
black which is a fine powder. Carbon black is used as a pigment in ink,
paints, shoe polish and also as a filler in rubber tyres to increase their
strength.

(iii) Acetylene is formed by partial oxidation of natural gas.

Petroleum refining
Crude petroleum is a viscous mixture of number of hydrocarbons. Refining is
the process to obtain useful products of commercial and industrial importance
from crude petroleum. This involves fractional distillation, which separates the
mixture into a number of fractions in increasing order of boiling point and
molecular mass. In refineries, the mixture is introduced into tall fractionating
column and heated which results in the separation of different fractions. More
volatile fractions with lower boiling point are collected at the top of the column
whereas the less volatile fractions having higher boiling point get collected at the
bottom of the column. The residue of this refining is bitumen, which is used as a
binding material for road surfaces and also as a roofing material.
Table 5.5 enlists different fractions along with their composition and uses.
Octane number
One of the commercially important fractions of petrol refining is gasoline (or
petrol). The efficiency of gasoline as a fuel depends upon its octane number. The
higher the octane number, the better is the fuel.
The octane number of a fuel is taken to be the percentage of isooctane in a
mixture of isooctane and n-heptane, which has the same knocking property as
the fuel. For example, if knocking of a fuel is same as the knocking of the
mixture having 15% n-heptane and 85% of isooctane, then the octane number of
the fuel is 85.
In an internal combustion engine as the piston moves down the cylinder, a
mixture of fuel and air is introduced. The mixture gets compressed during the
return stroke of the piston. At the point of high compression, the mixture is
ignited by a spark plug and a smooth burning of fuel takes place. If premature
ignition of fuel occurs in the engine, that is, the fuel–air mixture explodes during
compression itself without any ignition from the spark plug, it results in the
knocking (cracking sound) of engine. Knocking decreases the efficiency of fuel
besides damaging the engine.
Branched chain hydrocarbons exhibit good antiknock properties, whereas
straight chain hydrocarbons exhibit poor antiknock properties. For rating a fuel,
an arbitrary reference scale has been chosen in which 2,2,4-trimethylpentane
(isooctane) is assigned an octane number 100 and
n-heptane is assigned an octane number 0 (zero).
Tetraethyllead (TEL) has good antiknock properties and is added to petrol for
increasing its octane number. Addition of 3.0 g of TEL per gallon of petrol may
raise the octane number by
15–20 units. However, due to environmental hazards of lead, unleaded petrol is
used now a days. The octane number of unleaded petrol is enhanced by addition
of a mixture of benzene, toluene, and xylene (BTX). Aromatic hydrocarbons also
show good antiknock properties.
5A.4.1 PETROCHEMICALS
These are the compounds obtained from fractions isolated from petroleum by
using different processes such as distillation, cracking, reforming, and
isomerization. Petrochemicals are major sources of industrially important
chemicals. These are used chiefly in the production of polymers, aromatic
hydrocarbons solvents, plastics detergents, alcohols, and so on.
Cracking (pyrolysis)
Higher fractions of petroleum refining which constitute less volatile or solid
hydrocarbons are heated at high temperature resulting in the formation of small
chain hydrocarbons including lower alkanes and alkenes as discussed earlier (pp.
194–95).
Uses of cracking:
1. Cracking process is used in the industries to obtain gasoline from higher
fractions of petroleum.
2. The alkenes obtained during cracking are used commercially for production of
(i) Isooctane (2,2,4-trimethylpentane) by alkylation (p. 235)
(ii) Alcohols and glycols.
(iii) Polymers, for example polyethene (Chapter 31).
Reforming and Isomerization
Reforming is the conversion of straight chain compounds to cyclic and aromatic
compounds. This process is used for the production of fuels with high octane
rating. For example hexane and heptane having, low octane number, on
aromatization form benzene and toluene (having high octane number)
respectively (pp. 193–94).
Isomerization of lower alicyclic hydrocarbons to higher ones followed by
dehydrogenation (aromatization) results in the formation of aromatic
compounds.
The xylenes obtained from isomerization of cyclooctane derivatives have the
immense importance in chemical industries. As these are oxidized to produce
benzenedicarboxylic acids (phthalic, isophthalic, terphthalic acids) which are
further used for the synthesis of various organic compounds.

5A.4.2 Coal
Coal is an important source of alkane and other hydrocarbons which are of
commercial importance. Different processes which are used to prepare synthetic
fuels from coal are discussed here.
Bergius process. Destructive hydrogenation of coal under high temperature and
high pressure conditions results in the formation of alkanes.

This method is used for production of gasoline from coal, producing


approximately 0.5 ton gasoline from 1.0 ton of coal.
Fischer–Tropsch process. At high temperature, passing steam over coal results
in the formation of carbon monoxide and hydrogen. This mixture of gases
(synthesis gas) when passed over cobalt-thorium catalyst under high temperature
and pressure condition results in the formation of a mixture of straight chain
(linear) alkanes and alkenes.

This process is used for the production of gasoline (synthetic fuel). The
reduction of synthesis gas (obtained from coal) results in the formation of
alcohol, which may be used as a fuel.

A mixture of gasoline and alcohol, known as gasohol, is used successfully as an


alternative fuel for petrol driven vehicles. These synthetic fuels can be of great
help in overcoming the shortage of gasoline.

B. CYCLOALKANES
5B.1 INTRODUCTION
Cycloalkanes are the simplest type of alicyclic compounds. The general formula
for cycloalkanes is CnH2n. The first few members of the series are

The boiling points of alicyclic hydrocarbons increase regularly as the ring size
increases. Cyclic structures are rigid thus boiling points of cycloalkanes are
higher than the corresponding alkanes. Table 5.6 compares the boiling points of
a few cycloalkanes with correspoinding alkanes.

IUPAC nomenclature and stereochemistry (conformations of cycloalkanes) have


already been discussed in details, in Chapters 2 and 3 respectively.
5B.2 STRAIN IN RING COMPOUNDS: BAEYER’S
STRAIN THEORY
Earlier in the study of organic chemistry, it was observed that most of the cyclic
compounds occurring in nature were five or six membered ring systems.
Baeyer’s theory presumed the cyclic compounds to be planar and suggested that
angle strain is experienced by ring compounds due to deviation in bond angle
from normal tetrahedral carbon bond angle of 109.5o. As the deviation from
bond angle increases, the ring experiences more strain and becomes unstable.
In cyclopropane, the bond angle is 60o and there is a deviation of bond angle by
49.5° (109.5°–60°). Similarly, planar molecules of cyclobutane, cyclopentane,
and cyclohexane have bond angles of 90°, 108°, and 120° respectively. Thus, the
angle strain in cyclobutane, cyclopentane, and cyclohexane should be 19.5°,
1.5°, and 11.5° respectively. Baeyer suggested that cyclohexane should
experience certain amount of strain and further, as one moves to seven, eight, or
higher ring systems stability decreases , as the strain increases due to increased
deviation from the bond angle of 109.5°.

Although Baeyer’s theory explained the instability of three and four membered
ring systems, it failed to explain the stability associated with some larger rings.
Sachse suggested non-planar structure for cyclohexane (chair and boat form),
which was further supported by E. Mohr. The Sachse Mohr theory confirms the
existence of cycloalkanes as non-planar (puckered) strain free rings. This is
evident from the experimental data, cyclohexane ring is the most stable system
and the larger ring systems such as C12, C13, and so on show considerable
stability. Strain in cycloalkanes can be calculated quantitatively by comparing
the heat of combustion values per CH2 group. The unusual stability of
cyclohexane due to non-planar structure, has already been discussed in Chapter
3.
5B.3 PREPARATION OF CYCLOALKANES
Some important methods of preparation of cycloalkanes are discussed as the
subject matter of this section.
By the addition of carbene to alkenes
Carbenes are neutral, divalent species. Their electrophilic addition to alkenes
results in the formation of cyclopropane derivatives.

The reaction of an alkene with diiodomethane in presence of zinc–copper alloy


to produce cyclopropane is known as Simmons–Smith reaction. The reaction
involves the formation of carbene complex which adds on π bond of alkene to
give cyclopropane as follows:

Action of metallic sodium on dihaloalkane


Dihaloalkanes (two halogens are present at two terminals) on reaction with
sodium or zinc metal, undergo intramolecular cyclization to produce
cycloalkanes. The reaction can be used for the preparation of cyclic rings
containing upto six carbon atoms.
From calcium salts of saturated dicarboxylic acids
Calcium salts of saturated dicarboxylic acids on heating give corresponding
cyclic ketones. The Clemmenson reduction of cyclic ketones results in the
formation of corresponding cycloalkanes. The reaction is best used for the
synthesis of cyclopentane, cyclohexane, and cycloheptane.
From diethylmalonate
The reaction of sodium salts of diethylmalonate with dihaloalkanes (1,3-; 1,4-
and 1,5-) results in the formation of cycloalkanes in a series of reactions as
follows.

The use of diethylmalonate in the synthesis of alicyclic compound has also been
discussed in Chapter 20.
By Dieckmann condensation
The esters of dicarboxylic acids (C6, C7, or C8) in presence of a base such as
sodium ethoxide undergo an intramolecular condensation to give cyclic β-
ketoesters (Dieckmann condensation), which by a series of reactions may be
converted to cycloalkanes. The reaction is used for the preparation of
cyclopentane, cyclohexane, and cycloheptane. For example, diethyladipate is
used for the preparation of cyclopentane as follows.
Diels–Alder reaction
The thermal or photochemical addition of alkenes to buta-1,3-diene produces
cyclohexene. It is a 1,4- cycloaddition reaction involving a 4 π-system (diene)
and an unsaturated compound having 2-π-system (dienophile) to give a cyclic
adduct. This is one of the best methods for the preparation of cyclohexane
derivatives (for details refer section 7.4.3).

By reduction of aromatic compounds


Catalytic reduction of benzene under pressure using nickel as catalyst, results in
the formation of cyclohexane.
5B.4 CHEMICAL PROPERTIES OF
CYCLOALKANES
Cyclopropane and cyclobutane are small ring compounds and experience a large
strain as compared to cyclopentane and cyclohexane. Due to this, cyclopropane
and cyclobutane are highly reactive and undergo ring-opening reactions to
relieve the strain and thus, get converted to more stable open chain derivatives.
In cyclopentane and cyclohexane, the reactions are analogous to open chain
alkanes and they do not undergo ring-opening reactions because of their stability.

5B.4.1 Halogenation
Photohalogenation
Cycloalkanes react with halogens in presence of light to produce corresponding
halocycloalkanes.

Similarly, cyclopentane and cyclohexane result in corresponding


halocycloalkane derivatives.

Catalytic halogenation
The halogenation in presence of a catalyst or at high temperature results in ring-
opening of cyclopropane and cyclobutane to yield dihaloalkanes.
5B.4.2 Catalytic Hydrogenation
Cyclopropane and cyclobutane on catalytic hydrogenation undergo ring-opening
to give propane and butane respectively. Cyclopentane and cyclohexane are
stable even at high temperature conditions and do not undergo ring-opening.

5B.4.3 Effect of Heat


Cycloalkanes (C3 and C4) on heating result in the formation of alkenes.

5B.4.4 Reaction with Hydrogen Halides


Cyclopropane and cyclobutane on reaction with hydrogen halides undergo ring-
opening to give haloalkanes.
No reaction occurs when cyclopentane and cyclohexane react with hydrogen
halides.
SELECTED SOLVED EXAMPLES
Example 1. Arrange the following alkanes in the increasing order of their
boiling points.
(a) 2-methylhexane, (b) pentane, (c) 2,2-dimethylpropane, (d) 2-methylbutane,
(e) heptane,
(f) octane, and (g) hexane.
Solution. The boiling point of members of homologous series increases with
increasing molecular mass. If isomerism is involved, the highly branched isomer
possesses lowest boiling point
(refer section 5A.1.1).
In this example, 2,2-dimethylpropane, 2-methylbutane, and pentane are isomers
of pentane with the order of branching as (c) > (d) > (b). Therefore, the order of
increasing boiling points among these is (c) < (d) < (b).
Similarly, 2-methylhexane and heptane are isomers and they follow the
increasing order of boiling point as (a) < (e).
In terms of homologous series, the increasing order of boiling point can be
written as
Pentane (including isomers) < Hexane < Heptane (including isomers) < Octane.
Taking all the above facts into account, the increasing order of boiling points for
all the given compounds can be concluded as (c) < (d) < (b) < (g) < (a) < (e) <
(f).
Example 2. Can water be used as an extinguisher for burning Octane?
Solution. Water can not extinguish the fire on burning octane because octane (in
general alkanes) being less dense than water, remains at surface and burning
continues.
Example 3. Starting from 1-bromobutane, list three methods for the preparation
of butane.
Solution. Butane can be prepared from 1-bromobutane by the following
methods:
(a) reduction with HI/ P
(b) reduction with Zn in acidic medium
(c) formation of Grignard reagent followed by hydrolysis with water.
Example 4. Name the products formed when water reacts with the following
Grignard reagents
(a) Pentylmagnesium bromide
(b) 2-Pentylmagnesium bromide
(c) 3-Pentylmagnesium bromide.
What products are expected, if instead of water, reaction was carried out with
ethanol, ethanamine, or ethanoic acid?
Solution. Pentane (CH3CH2CH2CH2CH3) is obtained in all cases (refer p.
173).
Example 5. Making use of Corey–House synthesis, how will you prepare 2-
methylpropane in good yield?
Solution. There are two approaches for synthesizing 2-methylpropane, i.e.,
(CH3)2CHCH3.
They are
(a) Prepare lithium di(2-propyl)cuprate from 2-bromopropane and couple it with
bromomethane.
(b) Prepare lithium dimethylcuprate from bromomethane and couple it with 2-
bromopropane.
As mentioned in the text (p. 175), lithium dialkylcuprate (could be 1°, 2° or 3°)
should be coupled with halomethane or 1° haloalkanes ((a) above), for better
yield.

Example 6. Two isomeric compounds A and B on reduction with Zn–Cu couple


yield n-butane. Another compound C, sodium salt of a carboxylic acid, on
electrolysis also yields n-butane. Isomers A and B undergo Wurtz reaction to
yield n-octane and 3,4-dimethylhexane respectively. However, C on fusion with
soda-lime yields ethane. Identify A, B, and C and write the reactions involved.
Solution. Reduction of both A and B, with Zn–Cu couple to yield butane,
indicates that A and B are halobutanes. Since Wurtz reaction of A yields n-
octane, it must be 1-halobutane. As B yields, 3,4-dimethylhexane, so it must be
2-halobutane. Fusion of sodium salt of a carboxylic acid with soda-lime results
in decarboxylation (formation of alkane with one carbon less). Since ethane is
produced from C, compound C is sodium propanoate which also produces
butane by electrolysis (Kolbe’s electrolysis).
The reactions involved are

Example 7. Carry out the following conversions.


(a) Ethane to Butane
(b) Propane to 2,3-Dimethylbutane
Solution.

Bromination of propane gives exclusively 2-bromopropane (refer Reactivity and


Selectivity
(p. 186). Chlorination cannot be used in this conversion as the products will be
mixture of 1-chloro and 2-chloropropane.
Example 8. In order to prepare a mono-bromoalkane by substitution, should one
add bromine to alkane or should the alkane be added to bromine?
Solution. Bromine should be added to alkane. For mono-bromination, the alkane
should be in excess compared to bromine. If alkane is added to bromine then
excessive localized concentration of bromine causes polysubstitution.
Example 9. What major monosubstituted product is formed when 2-
methylbutane and chlorine react in presence of sun light?
Solution. In presence of sun light, chlorination follows a free radical substitution
mechanism.
The order of stability of free radicals is 3° > 2° > 1°. Thus, major product formed
is
2-chloro-2-methylbutane, because 3° C–H bond is more reactive than 2o or 1o
C–H bonds.
EXERCISES
1. Write the structural formula of n-pentane, isopentane, and neopentane. Which
of these have the highest boiling point and why?
2. Discuss the variation in melting point of alkanes.
3. Explain the following with suitable examples, in context with the preparation
of alkanes.
(a) The Wurtz reaction
(b) Grignard reagent
(c) Corey–House synthesis
4. What are the limitations of Wurtz reaction? How Corey–House synthesis
overcomes these limitations?
5. What product would be formed by the action of metallic sodium on (a)
isopropyl iodide and (b) 2-bromobutane?
6. How is it proved that the chlorination of methane occurs via free radical
mechanism?
7. Which isomer of pentane forms only one monochloro-substitution product?
8. Giving examples, write a short note on the following:
(i) Free radical substitution reactions
(ii) Cracking of alkanes
(iii) Nitration of alkanes
(iv) Chlorosulfonation of alkanes
(v) Role of carboxylic acid in preparation of alkanes
(vi) Isomerisation of alkanes
9. Discuss briefly the relative reactivities of halogens in the halogenation of
alkanes.
10. Explain: Bromine is less reactive but more selective whereas chlorine is
more reactive and less selective in its reaction with alkanes.
11. Write a short note on
(a) Cracking of petroleum
(b) Reforming (aromatization)
(c) Octane number
12. Explain the principle of Fischer–Tropsch process for synthetic petrol.
13. Discuss in brief, Baeyer’s strain theory.
14. How can the cyclopropane derivatives be synthesized using carbene?
15. How can cyclohexane be synthesized using
(i) Diels-Alder reaction
(ii) Dieckmann condensation
(iii) Calcium dicarboxylate
16. What products will be obtained from cyclopropane and cyclobutane on
(i) Heating
(ii) Catalytic hydrogenation
(iii) Reaction with HBr and HI.
17. Write the products formed during the halogenation of cyclopropane,
cyclobutane, and cyclohexane.
Chapter6
Alkenes

6.1 INTRODUCTION
The class of hydrocarbons containing a carbon–carbon double bond is termed
alkenes. Alkenes, the unsaturated hydrocarbons, have the general formula
CnH2n.They contain two hydrogens less than corresponding alkanes with same
number of carbon atoms. The first member of this series is, H2C=CH2, ethene
commonly known as ethylene. The alkenes are also called olefins (Latin: oleum
means oil and fiant means producing).
The carbons in ethene are sp2 hybridized. The sp2-sp2 orbital overlap results in
carbon–carbon σ bond formation while sp2-s overlap results in carbon-hydrogen
σ bond formation. The pure p orbitals on each carbon overlap to form carbon–
carbon π bond. Ethene has a planar structure (see details in section 1.4.2)

The IUPAC nomenclature of alkenes has already been discussed in Chapter 2.


Some examples of alkenes are illustrated in Table 6.1 along with their IUPAC
and common
names.
Isomerism. Alkenes having more than three carbon atoms in a molecule, exhibit
constitutional isomerism (see Chapter 1) and geometrical isomerism.
In constitutional isomerism, they exhibit position isomerism as well as chain
isomerism.
But-1-ene and But-2-ene are the examples of position isomerism while 3-
Methylbut-1-ene and
2-Methylbut-1-ene depict chain isomerism (refer Table 6.1).
In alkenes, due to restricted rotation around carbon–carbon double bond, the
groups or atoms present on the carbons attached through double bond differ in
their spatial arrangement, which gives rise to geometrical isomerism (for details,
see Chapter 3). For example, the geometrical isomers of but-2-ene and pent-2-
ene are as follows:

6.1.1 Physical Properties


Ethene, propene, and butenes are gases under ordinary conditions. Alkenes with
five or more carbons are liquids. Alkanes and alkenes are much alike in their
melting and boiling points. Their boiling point generally increases with an
increase in molecular mass.

The branching in alkenes lowers the boiling point compared to isomeric linear
alkenes (Table 6.2). Alkenes are less denser than water. Alkenes are insoluble in
water but soluble in non polar solvents. In comparison to alkanes which are non
polar, the alkenes show slight polarity due to presence of weakly held electrons
in the π-bond.
6.2 PREPARATION OF ALKENES
In nature, alkenes do not occur in free form. Commercially, the lower alkenes are
obtained by cracking of petroleum. The higher alkenes have little difference in
their boiling points and cannot be separated by fractional distillation, thus,
cracking cannot be used for the preparation of higher alkenes. The various
methods are used for the preparation of alkenes. Some important methods are
reduction of alkynes and elimination reactions of haloalkanes, alcohols and
quaternany ammonium hydroxides. This section offers an explicit discussion on
various preparative methods of alkenes.

6.2.1 Reduction of Alkynes: Formation of cis and trans


Alkenes
In general, the reduction of alkynes in presence of a metal catalyst (Pt, Pd, or Ni)
results in the formation of alkane, as the final product. The alkenes which are
difficult to isolate, are formed as intermediary products.

However, partial hydrogenation of alkynes may be carried out using specific


catalysts/reagents, which results in the formation of alkenes with specific
stereochemistry, that is, formation of cis- or trans- alkenes, as discussed in the
subsequent parts of this subsection.

Preparation of cis- alkenes: catalytic hydrogenation


The hydrogenation of alkynes can be easily carried out using deactivated
catalysts such as ‘Lindlar’s catalyst’ (Pd/CaCO3 in lead acetate poisoned with a
small amount of quinoline) or ‘nickel boride’ (known as P-2 catalyst) to produce
cis-alkenes. For example
Preparation of trans- alkenes: dissolving metal reduction (Birch reduction)
The reduction of alkynes with lithium or sodium metal in presence of liquid
ammonia (or ethyl amine) at low temperature, results in the formation of trans-
alkenes.

The reaction proceeds via the formation of three important intermediates,


namely vinyl radical anion, vinyl radical, and vinyl anion as follows:

6.2.2 Elimination Reactions [Saytzeff’s and


Hofmann’s rule]
Dehydrohalogenation of haloalkanes [Saytzeff elimination]
Haloalkanes undergo elimination reaction (dehydrohalogenation) on heating
with potassium hydroxide in presence of ethanol (alcoholic KOH) to form
alkenes.
In haloalkanes, β-elimination occurs, that is, hydrogen is removed from β-carbon
(the carbon adjacent to halogen bearing carbon) during dehydrohalogenation. If
two β-carbons having hydrogen are available, then elimination of hydrogen from
either side leads to the formation of two different alkenes. However, one of the
alkene is formed in higher percentage.

Saytzeff’s rule: This rule states that whenever there is a possibility of formation
of two alkenes by elimination, the formation of more substituted alkene is
favoured. In other words, the hydrogen is removed from β-carbon having lesser
number of hydrogen(s).

Dehalogenation of dihaloalkanes
The vicinal dihaloalkanes undergo dehalogenation in presence of zinc and
methanol to produce alkenes. The other dehalogenating agents used for this
purpose are Zn–Cu couple, magnesium etc.

Dehydration of monohydric alcohols


The monohydric alcohols in acidic medium undergo dehydration to yield
alkenes. The commonly used dehydrating agents are concentrated sulfuric acid
or phosphoric acid. Besides Al2O3, P2O5, or ThO2 may also be used as
dehydrating agents at high temperature.
The elimination of water from alcohols involves the removal of hydrogen from
β-carbon in accordance with Saytzeff’s rule. For example

General Mechanism
The first step of reaction is protonation of alcohol which forms an oxonium ion.
The second step involves cleavage of carbon-oxygen bond that result in removal
of a water molecule and formation of carbocation as an intermediate. The third
step is deprotonation of carbocation to form an alkene. The loss of proton occurs
from the β-carbon of carbocation. The overall mechanism can be represented as
follows:

The reaction proceeds through the formation of a carbocation and it is the


stability of carbocation that governs the order of reactivity of alcohol. In general,
the more stable the intermediate (in this case carbocation), the more reactive is
the substrate (alcohol in this case).
Since, order of stability of carbocations is 3o > 2o > 1o, thus, order of reactivity
of alcohols for dehydration is 3o > 2o > 1o.
One of the limitations of dehydration of alcohol in producing alkene involves
carbocation rearrangement, which may result in the formation of an alkene
different from what is expected by dehydration.
Wagner–Meerwein rearrangement
As discussed above, the mechanism of dehydration involves the formation of
carbocation as an intermediate. The carbocation formed may rearrange itself to a
more stable carbocation through a 1,2-hydride or 1,2-alkyl shift. The
rearrangement of carbocation occurs due to higher stability of
3o carbocation compared to 2o carbocation. Such a rearrangement during
reactions that occurs via carbocation intermediate is known as Wagner–
Meerwein rearrangement. For example, dehydration of 3,3-dimethylbutan-2-ol
results in the formation of 2,3-dimethylbut-2-ene as the major product and not
the expected 3,3-dimethylbut-1-ene.
Mechanism. The dehydration process involves the formation of a 2°
carbocation, that undergoes migration of methyl group from the adjacent carbon
(1,2-methyl shift). In this process the 2° carbocation is rearranged to a more
stable 3° carbocation. This 3° carbocation results in the formation of an alkene,
other than the expected alkene. The mechanism is as follows:

If two β carbons, having hydrogen, are available then elimination of hydrogen


from either side leads to the formation of two different alkenes. However, one of
the alkene is formed in higher percentage in accordance with Hofmann’s rule.
Hofmann’s rule. This rule states that whenever there is a possibility of
formation of two alkenes by elimination, the formation of less substituted alkene
is favoured. For example
This method can also be used for the preparation of 3° amines.

6.2.3 Other Methods


Witting reaction
The reaction of aldehydes and ketones with phosphorous ylide to yield alkenes is
known as Wittig reaction. It is the most useful method for the selective synthesis
of alkenes from carbonyl compounds. The position of double bond in the alkene,
formed by Witting reaction, is unambiguous.

The details and mechanism of the reaction is discussed in Chapter 17 (pp. 582–
83).
Kolbe’s electrolytic method
The concentrated aqueous solution of alkali salts of dicarboxylic acids on
electrolysis results in the formation of an alkene at anode. For example
6.3 CHEMICAL PROPERTIES OF ALKENES
This section disucsses various chemical properties of alkenes such as
Electrophilic addition reactions, free radical addition reactions, oxidation
reactions, isomerization, polymerization,
Diels-Alder cycloaddition reactions and allylic substitution reactions.

6.3.1 Stability of Alkenes


We have earlier mentioned in subsection 6.2.2 that a more substituted alkene is
more stable. Experimentally, stability of alkenes is determined from their heat of
hydrogenation and heat of combustion values.
• The heat of hydrogenation values are used to compare the stability of isomeric
alkenes that on hydrogenation result in the formation of same alkane.
• For alkenes which on hydrogenation do not result in the formation of same
alkane, the heat of hydrogenation values cannot be used for comparing
stability. However, all alkenes on combustion yield carbon dioxide and
water. Thus, for comparing the stability of different alkenes the heat of
combustion values are used.
Heat of hydogenation
The amount of energy released during the hydrogenation of an alkene is termed
as its heat of hydrogenation. A stable alkene releases lesser amount of energy.
Higher the energy released during hydrogenation, lesser is the stability of alkene.
Fig 6.1 Heat of hydrogenation for isomeric butenes and isomeric pentenes.
Heat of combustion
Alkenes on heating in presence of excess of oxygen (complete combustion) form
carbon dioxide and water as the final products. The amount of energy released
during the complete combustion of an alkene is termed as its ‘heat of
combustion’. Higher the heat of combustion, lesser is the stability of alkene. In
other words, a more stable alkene has a lower value of heat of combustion.

Thus, the overall stability of alkyl substituted alkenes is as follows:


6.3.2 Electrophilic Addition Reactions
The carbon–carbon double bond in alkenes consists of a strong σ bond (by
overlap of sp2 hybridized orbital on each carbon) and a weak π bond (by the
overlap of pure p orbital on each carbon). Because of this unsaturation, alkenes
undergo addition reactions, which involves breaking of a weak π bond and
formation of two new σ bonds. During addition reactions, the double bond
provides the loosely held π electrons (thus, behaving as an electron rich centre)
and is readily attacked by electron deficient species (electrophiles).
For this reason the reactions are termed electrophilic addition reactions. In the
addition reactions of alkenes the electrophilic and nucleophilic part of reagent
may add across the double bond from opposite sides and in such case the
addition is known as anti or trans addition. The addition of reagent may occur
from the same side of the double bond and in such case the addition in known as
syn or cis addition.
NOTABILIA 7

Addition of halogens—formation of vicinal dihaloalkanes


Alkenes react readily with chlorine or bromine, in dark, in a solvent such as
CCl4, to produce vicinal-dihaloalkanes as the addition products. Iodine,
generally, does not react with alkenes.

The general mechanism for the electrophilic addition of bromine to alkene is as


follows:
Step 1. Addition of bromine to alkene (Formation of carbocation)
The bromine molecule is non polar in nature. However, as the π electrons of
alkene approach the bromine molecule, the bromine becomes polarized. The
addition of Br+ to alkene occurs in the first step that results in the formation of
carbocation. The carbocation exists in the form of a cyclic intermediate and is
known as bromonium ion.

Step 2. Attack of bromide ion on carbocation (formation of 1,2-dibromoalkane)


The addition of bromide ion to the cyclic carbocation takes place from the
opposite side. Thus, the addition of bromine occurs in an trans- (or anti-)
manner as follows:
Proof for electrophilic addition mechanism:
The above mechanism is further supported by the fact that the reaction of
bromine with ethene in presence of sodium chloride results in the formation of
two products identified as 1,2-dibromoethane and 1-bromo-2-chloroethane.

The formation of 1-bromo-2-chloroethane is possible only when the electrophile


(Br+) attacks first to form a carbocation intermediate which further reacts with
the negatively charged species (Br– from bromine and Cl– from sodium
chloride) to form 1,2-dibromoethane and 1-bromo-2-chloroethane respectively.
Proof for trans- addition of reagent
[Evidence for the stereochemical aspects of addition in alkenes]
In case of open chain alkenes the intermediate carbocation is said to exist in
cyclic form (as cyclic bromonium ion). The rigidity of cyclic structure prevents
the attack of nucleophilic part of reagent from the same side as that of the
electrophile.
Addition of hydrogen halides
[Regioselectivity and Markovnikov’s Rule]
The reaction of alkenes with hydrogen halides (HX, where X = Cl, Br, I) results
in the formation of corresponding haloalkanes. The reaction is carried out in a
solvent such as acetic acid or by passing dry gaseous HX directly over an alkene.

As an example,

In case of an unsymmetrical alkene like propene, reaction with HCl may result in
the formation of two products, namely 1-chloropropane or 2-chloropropane.
However, it is 2-chloropropane which is produced selectively.
Markovnikov’s rule: The addition of polar reagents like HX to unsymmetrical
alkenes follows Markovnikov’s rule, which states that the negative part of the
addendum (polar reagent) adds to that carbon of double bond which has
minimum number of hydrogen(s).

The addition of HX to an unsymmetrical alkene is a regioselective reaction.


Explanation to Markovnikov’s rule: A modern approach
Consider the mechanism of addition of HCl to propene. The attack of the
electrophile H+ on either of the olefinic carbons may result in the formation of a
1o or a 2o carbocation. In general, the attack of electrophile H+ is always
favoured in a direction which results in the formation of more stable carbocation.
In propene, 2-chloropropane forms predominantly through the formation of a
more stable 2o carbocation.

Redefining the Markovnikov’s Rule


In the addition reactions of unsymmetrical alkenes, the attack of
electrophile occurs on that carbon of the double bond which results
in the formation of a more stable carbocation.
This redefined version of Markovnikov’s rule, based on the
stability of carbocation, explains a number of electrophilic addition
reactions in unsymmetrical alkenes, which was not possible with
the earlier definition.
The carbocation formed during electrophilic addition may be
stabilized further through Wagner-Meerwein rearrangement
involving 1,2-hydride or 1,2-methyl shift in the carbocation.
Besides, the nature of substituent also affects the stability of
carbocation.
The electrophilic addition in alkenes based on the stability of carbocations is
discussed as follows:
Case Study I (Stability of carbocation involving 1,2-hydride shift)
Reaction of 3-methylbut-1-ene with HCl
In this reaction, the major product obtained is 2-chloro-2-methylbutane rather
than the expected
2-chloro-3-methylbutane.

This reaction can be explained on the basis of carbocation rearrangement. The


2o carbocation formed as an intermediate undergoes a 1,2-hydride shift to give a
more stable 3o carbocation. This 3° carbocation formed as an intermediate on
reaction with Cl– results in the formation of 2-chloro-2-methylbutane as the
major product.

The reactions proceeding through carbocation rearrangement are known as


Wagner–Meerwein rearrangement reactions (p. 216).
Case Study II (Stability of carbocation involving 1,2-methyl shift)
Reaction of 3,3-dimethylbut-1-ene with HCl
The reaction involves the formation of 2-chloro-2,3-dimethylbutane as the major
product instead of expected 3-chloro-2,2-dimethylbutane.

In this reaction, 2o carbocation is formed which rearranges through 1,2-methyl


shift to give more stable 3o carbocation. Thus, 2-chloro-2,3-dimethylbutane is
obtained as a result of addition of
Cl– to a more stable 3° carbocation.

Case Study III (Effect of electron-withdrawing group on stability of


carbocation)
Reaction of propenenitrile with HCl
In this reaction, instead of expected 2-chloropropanenitrile the product obtained
is
3-chloropropanenitrile.

The addition of electrophile (H+) may result in the formation of a 1° or 2°


carbocation. However, the 2° carbocation is unexpectedly less stable than 1°
carbocation. This is due to the presence of electron withdrawing (–I effect) –CN
group. The –CN group directly attached to carbon bearing positive charge (in
case of 2° carbocation) destablizes it by intensifying the positive charge. Thus,
attack of Cl– occurs at relatively stable 1° carbocation.

Addition of sulfuric acid


The alkene, when passed through cold, concentrated sulphuric acid results in the
formation of alkylhydrogensulfate (ester of sulfuric acid) as the addition product.
The addition is regioselective and follows Markovnikov’s rule. The
alkylhydrogensulphate, on hydrolysis (heating with water), forms alcohol as the
final product.

This method cannot be used for the preparation of 1o alcohols, except in case of
reaction with ethene, which yields ethanol. Further, during the reaction the
carbocation rearrangement may occur through 1,2-hydride shift and 1,2-methyl
shift.
Addition of water (Formation of alcohol)
The acid catalyzed reaction of water with alkenes results in the formation of
alcohol. The addition follows Markovnikov’s rule.
General reaction
Mechanism

For example,

Primary alcohols (except ethanol) cannot be prepared by this method. Further,


carbocation rearrangement may occur. This method is used industrially for
preparation of lower alcohols where there is no possibility of cabocation
rearrangement.
Addition of halogen and water (Formation of haloalcohols (halohydrins))
Treatment of an alkene with bromine or chlorine dissolved in water results in the
formation of bromo- or chloroalcohols. The actual mechanism does not involve
the addition of hypohalous acid by itself, rather it is the reaction of halogen
followed by that of water where an electrophile
(Br+ or Cl+) is provided by halogen and nucleophilic part (HO–) is provided by
water. In all, it is the addition of elements of hypohalous acid, though separately,
as HO–X+. The electrophilic addition follows Markovnikov’s rule.
For example,

Oxymercuration–demercuration reaction (Formation of alcohol)


Alkenes react with mercuric acetate and water in the presence of THF
(tetrahydofuran) as solvent to yield hydroxymercurial compound. This is known
as oxymercuration reaction. The hydroxymercurial compound on reductive
cleavage with sodium borohydride, results in the formation of alcohol and this
process is known as demercuration. The overall process is hydration of alkene in
accordance with Markovnikov’s rule.
For example,
Step 1. Oxymercuration

Step 2. Demercuration
It’s a highly regioselective reaction and occurs readily even at room temperature
to give alcohol (more than 90% yield).
Similarly,

An advantage of the reaction over usual acid catalyzed hydration is that no


carbocation rearrangement occurs and desired alcohols may be prepared easily.
Oxymercuration–demercuration reaction of 3,3-Dimethylbut-1-ene results in the
formation of 3,3-Dimethylbutan-2-ol while its acid catalyzed hydration results in
the formation of
2,3-Dimethylbutan-2-ol due to cabocation rearrangement as explained earlier.

Mechanism. The mercuric acetate acts as a source of electrophile. The


electropositive character of mercury prevents the rearrangement of carbocation.
Hydroboration–Oxidation (Formation of alcohol)
The hydroboration of alkenes using borane results in the formation of
alkylborane that on oxidation with hydrogen peroxide in alkaline medium
produces alcohol. Borane (B2H6) is a dimer of boron hydride (BH3), however in
reactions we represent the reagent as boronyhydride. In boronhydride, the
electrophilic part is boron (electron deficient) and nucleophilic part is hydrogen
(as hydride
H–). The addition of borane occurs from the same side of the alkenes and thus
hydroboration is a syn-addition (or cis-addition). Since, the reagent BH3
furnished three hydride ions, it reacts with three molecules of alkene to form
trialkylborane. For example,
Hydroboration:

Oxidation:

Similarly,
The hydroboration-oxidation reaction is carried out readily and results in high
yields of the product.

The overall hydroboration-oxidation reaction can be


depicted as hydration of alkene where addition of water to alkene
occurs in an anti Markovnikov’s manner to yield alcohol as the
product. The hydroboration-oxidation of alkene is thus used in the
preparation of alcohols that cannot be prepared by usual hydration
of alkenes where addition of water occurs in accordance with
Markovnikov’s rule.
Addition of peroxy acid (Formation of epoxide)
The addition of peroxy acid to alkene results in the formation of epoxide. The
reaction follows the electrophilic addition mechanism.
Mechanism

Addition of alkenes (Dimerization)


Under acidic conditions, two molecules of an alkene react to give another alkene
as the product which contains double the number of carbon atoms compared to
initial alkene. This reaction is known as dimerization of alkene and occurs in the
presence of sulfuric or phosphoric acid as catalyst.
When two molecules of 2-methylpropene react in the presence of an acid
catalyst, two isomeric alkenes (dimer), that differ in their position of double
bond, are formed. The reduction of these alkenes results in the formation of
2,2,4-trimethylpentane (isooctane), a commercially important gasoline, with
high octane number.
Mechanism
Step 1. Protonation of alkene (formation of carbocation).

Step 2. Addition of carbocation to second molecule of alkene.

Step 3. Loss of proton (deprotonation) (Formation of alkene).


The carbocation formed in Step 2 may undergo deprotonation from either of the
adjacent carbons (–CH2 or –CH3) to form isomeric alkene. A more substituted
alkene is preferred and is formed as the major product.

Addition of alkanes (Alkylation)


The reaction of an alkane and an alkene in the presence of an acid catalyst at low
temperature results in the formation of a higher alkane. This method is used for
industrial preparation of isooctane taking 2-methylpropane (isobutane) and 2-
methylpropene (isobutene) as the reactants. The reaction may be carried out in
the presence of HF or H2SO4.
Mechanism. The reaction proceeds though the formation of a carbocation and
the first two steps are similar to dimerization mechanism.
Step 1. Protonation of alkene (Formation of carbocation).
The addition of proton from an acid to alkene forms a stable 3o carbocation.

Step 2. Addition of carbocation to second molecule.


The addition of 3o carbocation to alkene results in the formation of dimeric
carbocation (i.e. new C–C bond is formed).

Step 3. Intermolecular hydride transfer from alkane to dimeric carbocation


(Formation of alkylated alkane).
The dimeric carbocation undergoes an intermolecular hydride shift (hydride
coming from alkane). This results in the formation of a substituted alkane (2,2,4-
trimethylpentane). The alkane 2-methylpropane looses a hydride ion and gets
converted to tertiary butyl cation. The (CH3)3C+ formed due to intermolecular
hydride shift may get attached to another molecule of
2-methylpropene (Step 2) to continue the chain.
Addition of carbene (Formation of cyclopropane derivatives)
[Stereospecific reaction]
The carbenes are neutral divalent carbon compounds and are basically electron
deficient in nature. The addition of carbenes to alkenes is a highly stereospecific
reaction, which results in the formation of cyclopropane derivatives. If alkyl
groups in alkene are cis-, then they will be present in the same way (i.e. only cis-
from) in the cyclopropane derivatives. In the same way, if alkyl groups are trans-
, the product shall also be having the alkyl groups in trans-configuration.
The above reactions are stereospecific and the carbene in these reactions is
present in the singlet state. The reactions of alkene with carbene in triplet state
are non-stereospecific.

6.3.3 Free Radical Addition Reaction


Addition of hydrogen bromide (Peroxide Effect)
The reaction of HBr with an alkene in the presence of peroxide, results in the
formation of bromoalkane where addition of HBr occurs anti to Markovnikov’s
rule. For example, reaction of HBr with propene gives 1-bromopropane, an anti-
Markovnikov’s addition product, in the presence of peroxides.

The anti-Markovnikov’s addition in the presence of peroxide is also known as


Peroxide Effect or Kharasch Effect.
Note: The addition of HBr in the presence of peroxide is a free radical addition
whereas in absence of peroxide, the usual electrophilic addition of HBr takes
place.
Free radical addition mechanism
Peroxides are very good free radical generators. In peroxides, the oxygen–
oxygen single bond undergoes homolytic cleavage to produce radicals. Thus,
above mentioned reaction proceeds through the following steps:
Step 1. Generation of free radicals.
Here, a more stable free radical (2°) is preferentially formed which further reacts
with HBr to form 1-Bromopropane.

The Peroxide Effect is Observed Only in Case of HBr and


Not with HF, HCl, or HI. Why?
The order of bond strength of halogen acids is HF > HCl > HBr > HI.
HF and HCl have high bond strength and polarity, so homolytic
cleavage does not occur easily. In HBr and HI, homolytic cleavage
occurs readily to produce bromine and iodine free radicals. However,
iodine radical undergoes coupling reaction with another iodine radical
to produce iodine.
I. + I. ↔ I2
As a result iodine radicals are not available for addition on alkene.
Thus, only HBr can provide bromine free radical which is formed
easily and is available for addition to olefinic carbon.

6.3.4 Oxidation Reactions


Hydroxylation of alkenes (Formation of 1,2-diols (vicinal diols))
[cis-addition]
Oxidizing agents such as KMnO4 or OsO4 (osmiumtetroxide) react with alkenes
and result in the addition of two hydroxyl groups to the double bond. The final
products of addition of these oxidizing agents are 1,2-diols, commonly known as
glycols.
The addition of two hydroxyl groups occurs form the same side of the double
bond and is therefore, called cis-addition.
With cold alkaline KMnO4 (Baeyer’s reagent)
The alkene and the aqueous alkaline solution of potassium permanganate on
stirring at room temperature, result in cis-hydroxylation of the alkene.

For example,
The reaction of alkene with alkaline KMnO4 is known as
Baeyer’s test and is used to test unsaturation. Unsaturated
compounds react with alkaline KMnO4 (purple in colour) to form
diols (colourless). Thus, decolourization of KMnO4 by an organic
compound indicates the presence of double bond therein.
With OsO4
Alkenes react with osmiumtetroxide to form an osmate ester, which on
hydrolysis produces cis-diols.

Oxidative cleavage with acidified or hot KMnO4 or acidified K2Cr2O7


(Formation of carboxylic acids and/or ketones).
Strong oxidizing agents such as acidified KMnO4, or K2Cr2O7 (or Na2Cr2O7)
react with alkenes and in this oxidation process, cleavage (breaking) of carbon–
carbon double bond occurs to give two smaller oxidized molecules.
A cold, aqueous KMnO4 solution oxidizes an alkene to a diol. However,
acidified or hot KMnO4 solution is a much powerful oxidizing agent and
oxidizes diol further to give carboxylic acids or ketones through the cleavage of
carbon–carbon double bond. The terminal =CH2 group oxidizes completely to
form CO2 and H2O. Another reagent used for such oxidative cleavage is
acidified K2Cr2O7 or Na2Cr2O7.

Similarly

Olefinic carbon with no hydrogen attached to it gets oxidized only upto ketones,
which are very difficult to be oxidized further to carboxylic acids.

Ozonolysis
Ozone exhibits the electrophilic character and reacts rapidly with alkene in
presence of a solvent (such as carbon tetrachloride or light petrol) to produce
cyclic peroxide known as ozonide.
The reaction of ozone with alkene results in the cleavage of carbon-carbon
double bond.
The ozonide being explosive in nature are not isolated as such. Further the
decomposition of ozonide is carried by reduction (known as reductive
ozonolysis) or by oxidation (known as oxidative ozonolysis).
• The reductive ozonolysis with Zn, H2O results in the formation of aldehydes
and/or ketones.
• The oxidative ozonolysis with H2O2 results in the formation of carboxylic
acids and/or ketones.

Examples of oxidative ozonolysis (Formation of Carboxylic acids and/or


ketones)
The products of overall ozonolysis reaction, aldehydes and ketones, are used for
determination of structure of alkenes, that is, to determine the position of double
bond.
Identification of alkene through ozonolysis products
The structure of an alkene is identified by writing the products (obtained by
ozonolysis of alkene followed by reductive cleavage) so that for adjacent
compounds, the carbonyl groups face each other. Then we remove the two
oxygen atoms and instead put a double bond between carbon atoms (which were
initially attached to oxygens). The resulting structure gives the required alkene.

For example, an unknown alkene on ozonolysis followed by reductive cleavage


results in the formation of propanone and ethanal, the structure of this alkene is
identified as follows in the manner as explained above:

Mechanism. Ozone is a hybrid of following contributing structures:


It reacts with alkenes to form an unstable Molozonide at the first stage which
rearranges to form an ozonide.

The oxidation reactions discussed above are summarized in Fig. 6.2, which
depicts the fate of various olefinic carbons during oxidation process.

Fig. 6.2 Oxidation of various types fo alkenes at a glance.

6.3.5 Allylic Substitution Reactions


[Free radical substitution reaction]
Carbon adjacent to double bonded carbon is known as allylic carbon and
hydrogen(s) attached to it are called allylic hydrogen(s). The double bond in an
alkene may enhance the reactivity at allylic position. Thus, in allylic
substitutions the double bond of alkene remains intact but it influences the
reactivity of neighbouring sp3 hybridized carbon where substitution occurs.
Alkenes undergo allylic halogenation via free radical mechanism. Two such
important reactions are reaction of alkenes with halogen at high temperatrue and
the other is reaction of alkenes with N-bromosuccinimide. The reactions are
discussed as follows.
Reaction with Cl2 or Br2 at high temperature
Alkenes with allylic hydrogen react with low concentration of chlorine or
bromine at high temperature (500°–600°C) or in the presence of ultraviolet light
to produce alkenes having the allylic position substituted with chlorine or
bromine.

Mechanism: The reaction follows a free radical mechanism.


Step 1. Formation of chlorine free radicals

Step 2. Abstraction of hydrogen radical from alkene


The chlorine radical can abstract a hydrogen radical from either of the olefinic
carbons or from allylic carbon to generate a new free radical.

Step 3. Abstraction of chlorine radical by allyl radical (formation of allyl


chloride)
Reaction with N-bromosuccinimide
The allylic bromination may be carried out by reaction of alkene with N-
bromosuccinimide in presence of light (or peroxide) using CCl4 as a solvent.

N-bromomosuccinimide provides a low concentration of bromine, which favours


substitution of bromine in alkenes rather than addition. The reaction occurs
through a free radical mechanism as follows:

The HBr released reacts with NBS to produce low concentration of bromine.
6.3.6 Polymerization
Polymerization is the process of joining of small molecular units (known as
monomers) to form large molecules (called polymers) with high molecular
weight.
Alkenes can form large molecules by repetitive addition where monomer units
are joined together without loss of any atom(s). Such a process is termed
Addition polymerization. For example, a number of ethene molecules may join
together to form polyethene.

The preceding example illustrates the process of polymerization by considering


only a few monomer units. In actual, polymerization involves several thousands
of monomer units. Commonly, the polymerization reaction is written as follows:

Many addition polymers or alkenes possess high commercial importance. Some


of them are
Isomerization
Alkenes at high temperature (600–700oC) or on treatment with Al2(SO4)3 at
250–300oC undergo isomerization to form stable isomers. For example, but-1-
ene on heating isomerizes to but-2-ene, which is more stable.
SELECTED SOLVED EXAMPLES
Example 1. How will you differentiate between
(i) Hex-1-ene, hex-2-ene, and hex-3-ene
(ii) Hex-1-ene and hexane
Solution
(i) Since all the given compounds are alkenes, the reductive ozonolysis
product will indicate the structure. After reductive ozonolysis,
Hex-1-ene gives pentanal and methanal (formaldehyde).
Hex-2-ene gives butanal and ethanal (acetaldehyde).
Hex-3-ene gives propanal as the only product.
(ii) Following tests can be used to distinguish hex-1-ene from hexane:
Reaction with cold dilute KMnO4 (Baeyer’s test): Hex-1-ene (in general, any
alkene) on reaction with cold dilute KMnO4 solution (a purple colour solution)
decolourizes it. On the other hand, hexane does not react, so no colour change is
observed.
Reaction with Br2/CCl4: Hex-1-ene (in general, any alkene) on reaction with
Br2/CCl4, (a red coloured solution) decolourizes it immediately. Reaction with
hexane is slow and occurs only at high temperature or UV light.
Example 2. Despite starting with different reactants, the following two reaction
sequences result in the same product. Explain.
Sequence 1

Sequence 2

Solution. Addition of water takes place according to Markovnikov’s rule


followed by elimination of water (dehydration) according to Saytzeff elimination
leading to the formation of same product in both cases.
Example 3. The acid catalyzed addition of water to CH3CH2CH=CHCH3 leads
to the formation of approximately equal amount of Pentan-2-ol and Pentan-3-ol.
Explain.
Solution. Both carbons of the double bond have one hydrogen each on them;
thus, both are equally likely to be attacked by an electrophile (H+) leading to the
formation of two alcohols almost in equal proportions.
Example 4. Write down the structure of corresponding alkene from the carbonyl
compounds listed below obtained by reductive ozonolysis.
(a) Butanal and formaldehyde
(b) Acetaldehyde and propanal
(c) Acetaldehyde and acetone (propanone)
(d) 2 moles of propanal
(e) 2 moles of acetone
(f) Acetone and 2,2-dimethylpropanal
Solution. The corresponding alkenes are:
(a) Pent-1-ene
(b) Pent-2-ene
(c) 2-Methylbut-2-ene
(d) Hex-3-ene
(e) 2,3-Dimethylbut-2-ene
(f) 2,4,4-Trimethylpent-2-ene
Example 5. An alkene on reductive ozonolysis gives hexanedial as the only
product. Deduce the structure of alkene.
Solution. The alkene is identified to be a cyclohexene as follows:

Example 6. An alkene on reductive ozonolysis gives one mole of glyoxal and


two moles of acetaldehyde. Deduce the structure of alkene.
Solution. The alkene is hexa-2,4-diene. The reaction can be written as follows:
Example 7. An alkene on reductive ozonolysis gives 6-Oxoheptanal as the only
product. Deduce the structure of alkene.
Solution. The alkene should be 1-Methylcyclohexene as shown by the following
reaction.

Example 8. An alkene having molecular formula C8H12, on reductive


ozonolysis gives acetaldehyde and glyoxal. Deduce the structure of the alkene.
Solution. The structure of alkene is identified to be that of 2,4,6-Octatriene by
the following reaction.

Example 9. The heat of hydrogenation for four alkenes is as follows. What


conclusion can be drawn about the nature of these alkenes?

Solution. Heat of hydrogenation indicates the stability of alkenes. Higher the


heat of hydrogenation, lesser is the stability of the alkene (refer section 6.3.1).
Thus, the order of stability of alkenes is (a) > (c) > (b) > (d).
Example 10. Carry out the following conversions.
(a) 2-Bromo-2-methylpropane to 1,2-dibromo-2-methylpropane
(b) 2-Bromo-2-methylpropane to 1-bromo-2-methylpropane
(c) 2-Methylbut-2-ene to 3-methylbutan-2-ol
(d) But-1-ene to but-2-ene
(e) 2-Bromopropane to 1-bromopropane
(f) 2-Methylbut-2-ene to 2-methylbutan-2-ol
Solution

Oxymercuration–demercuration process does not involve carbocation


rearrangement.
EXERCISES
1. Discuss the hybridization in ethene giving the various bond angles and bond
lengths. How will you account for the fact that carbon–carbon double bond is
smaller than carbon–carbon single bond?
2. Write a short note on geometrical isomerism.
3. What is Saytzeff’s rule? Explain, giving suitable examples.
4. Describe the preparation of alkenes by dehydration of alcohols. Also explain
the mechanism of reaction.
5. How are the alkenes obtained by the dehydrohalogenation of haloalkanes?
Explain the mechanism and comment on the reactivity of haloalkanes towards
dehalogenation.
6. How can you obtain alkenes by the
(a) Reduction of alkynes
(b) Dehydrogenation of alkanes
(c) Dehalogenation of 1,2-dihaloalkanes (vic-dihalides)
7. How will you prove that addition reactions in alkene are ‘electrophilic
addition reactions’?
8. Explain the following.
(a) Markovnikov’s rule
(b) Peroxide effect
9. The peroxide effect (anti-Markovnikov’s rule or Kharasch effect) in alkenes is
observed only in the addition of HBr and not HCl or HI. Explain why?
10. Explain the following:
(a) Ozonolysis of alkenes
(b) Hydroxylation reactions
(c) Hydroboration
(d) Alkylation of alkenes
11. Describe the mechanism for the addition of carbenes on alkenes.
12. What product is formed by the action of chlorine on propene at high
temperature?
13. Write a short note on polymerization in context with alkenes.
14. Complete the following equations.
15. Starting from propene, how will you prepare the following compounds
(a) 2-bromopropane
(b) 1,2-dibromopropane
(c) 1-chloropropan-2-ol
(d) Allyl chloride
16. Two isomeric compounds A and B have molecular formula C5H10.
Reduction of A gives
2-methylbutane whereas on reductive ozonolysis, it forms acetone and
acetaldehyde. Compound B on reductive ozonolysis forms acetaldehyde and
propanal. Deduce the structures of A and B.
(Hint: A is 2-methylbut-2-ene and B is pent-2-ene.)
Chapter7
Alkadienes

7.1 INTRODUCTION
The class of hydrocarbons containing two carbon–carbon double bonds are
termed dienes and possess a general formula, CnH2n–2. These have the same
molecular formula as alkynes and are thus, isomeric with alkynes. Based on the
relative position of the two double bonds, dienes can be classified as isolated
dienes, conjugated dienes, and cumulated dienes.
Isolated dienes. The two double bonds are separated by at least one sp3
hybridized carbon. For example,

Conjugated dienes. The double bonds are present alternatively, i.e. a double
bond followed by a single bond and then the other double bond. For example,

Cumulated dienes. The two double bonds are present on same carbon. These
dienes are popularly known as Allenes.

Stability of dienes
Earlier we have discussed in Chapter 6 that stability of alkenes are determined
form their heat of hydrogenation values. The comparsion of heat of
hydrogenation (Table 7.1) values of different diene systems indicate the order of
stability of dienes as follows:
Conjugated diene > Isolated diene > Cumulated diene
Table 7.1 Heat of hydrogenation for some diene systems

In the present discussion we will be focusing on conjugated dienes, their


structure, stability and charcteristic chemical properties.
7.2 BUTA-1,3-DIENE
The simplest example of conjugated diene is ‘buta-1,3-diene’,
H2C=CHCH=CH2. In buta-1,3-diene, the two double bonds are separated by a
single bond. All the four carbons are sp2 hybridized. Each carbon has a pure p
orbital (with unpaired electron). The p orbitals are present in the same plane and
overlap side by side to have an extended π system. The four unpaired electrons
are delocalized among four carbons. This delocalization provides stability to
conjugated dienes.

Inactual buta-1,3-diene is hybrid of following contributing structures:

Comparison of bond lengths


The carbon–carbon single bond in buta-1,3-diene is much shorter than a pure
single bond in butane because of high sp2 character of carbons. This shortening
of bond length results in greater orbital overlap and hence, an increase in bond
strength, which imparts higher stability to conjugated dienes.

In short, the factors involved in stability of buta-1,3-diene are


(1) Shortening of carbon-carbon single bond [C2–C3 bond] due to sp2–sp2
overlap, which makes the single bond much stronger.
(2) Delocalization of π-electrons over all the four carbons of diene system.
7.2.1 Molecular Orbital Picture of Buta-1,3-diene

In buta-1,3-diene, all the four carbons are sp2 hybridized and each carbon has a
p orbital (with single electron). The four p orbitals are present in same plane. A p
orbital has two lobes with opposite signs (+ and –) that represent opposite phases
of wave function. In general a lobe with + sign is represented as shaded lobe and
a lobe with – sign as an empty lobe. The lobes with same phase overlap to give
bonding molecular orbitals (π) while the overlap of opposite phase lobes give
antibonding orbitals (π*). In case of overlap of opposite phase lobes there is a
node which represents region of zero electron density, that is, at the node the +
and – lobes exactly cancel each other. The bonding molecular orbitals (π) are
lower in energy than the antibonding molecular orbitals (π*). The total number
of molecular orbitals formed are always equal to the number of p atomic orbitals
that partici-pate in bond formation. For example, in case of ethene there are two
p orbitals that overlap and result in the formation of two molecular orbitals one
bonding and one antibonding. The electrons are first filled in lower energy
bonding molecular orbitals and then to higher energy antibonding molecular
orbitals. In case of ethene there are two electrons thus they are filled in bonding
molecular orbital, π, whereas the antibonding molecular orbital, π*, is empty.
Thus π is the Highest-Occupied Molecular Orbital (HOMO) of ethene and π* is
the Lowest Unoccupied Molecular Orbital (LUMO) of ethene. The molecular
orbital picture of ethene is represented in Fig. 7.1

Fig. 7.1 Molecular orbital picture of ethene.

Now, in case of Buta-1,3-diene there are four p orbitals with single electron each
and their overlap results in the formation of four molecular orbitals. The two
molecular orbitals of butadiene are bonding molecular orbitals (π1 and π2) and
two are antibonding molecular orbitals (π3* and π4*).
For a molecular orbital πn, the number of nodes are (n –1). Thus π1, π2, π3, π4
molecular
orbitals have 0, 1, 2 and 3 nodes respectively. The molecular orbital picture of
buta-1,3-diene (Fig. 7.2) is as follows.

Fig. 7.2 Molecular orbital pircture of buta-1,3-diene.

NOTABILIA 8
7.3 PREPARATION OF BUTA-1,3-DIENE
The various methods used for preparation of buta-1,3-diene are discussed as
follows:
From Cyclohexene
Cyclohexene on heating in the presence of nickel chrome wire results in opening
of the ring which yields buta-1,3-diene and ethene.

From Butane or But-1-ene by Dehydrogenation


Butane or But-1-ene on heating with alumina and chromium oxide at 500–600°C
result in the removal of hydrogen (dehydrogenation) to give buta-1,3-diene.

From Ethyne (Acetylene)


Following two methods are used to carry out the synthesis of butadiene from
ethyne.
(a) Ethyne, on treatment with a mixture of cuprous chloride and ammonium
chloride
(CuCl + NH4Cl), dimerizes to vinyl acetylene, which yields buta-1,3-diene
on partial reduction.

(b) The reaction of ethyne and methanal (formaldehyde) results in the


formation of 1,4-diol which on dehydration yields buta-1,3-diene.
The dehydration of butane-1-4-diol can be carried out in the presence of
concentrated sulfuric acid or with mixture of sodium phosphate and phosphoric
acid.
From Butane-1,3-diol
Heating butane-1,3-diol in presence of sodium phosphate and phosphoric acid
results in the elimination of water molecule (dehydration) to produce buta-1,3-
diene.

Industrial Method (From Ethanol and Ethanal)


Industrially, buta-1,3-diene is prepared by passing a mixture of ethanol and
ethanal over silica gel.
7.4 CHEMICAL PROPERTIES OF BUTA-1,3-
DIENE
In buta-1,3-diene, the electrons are delocalized over all the four carbons and
thus, it behaves as an electron rich site which may be attacked by electrophiles
and radicals to undergo addition reactions. The π-bonds of dienes may also
participate in cycloaddition reactions. Besides, the dienes also undergo oxidation
reactions. The various chemical reactions of buta-1,3-diene are discussed as
follows.

7.4.1 Electrophilic Addition Reactions


Conjugated dienes undergo reactions where electrophilic addition may occur at
one or both the double bonds. If two molar equivalents of reagent are used,
addition occurs at both the double bonds.
However, conjugated dienes show special behaviour when they react with one
molar equivalent of electrophilic reagent.
Reaction with hydrogen halide
Buta-1,3-diene on reaction with one molar equivalent of hydrogen halide, results
in the formation of two products, namely the 1,2-addition product and 1,4-
addition product. For example,

The formation of these two products can be explained by the following


mechanism:
Step 2. Attack of nucleophile [Cl–] on carbocation.
The allylic carbocation is a hybrid of structures [A] and [B]. The Cl– can attack
either [A] or [B] to give two types of addition products.

In a similar manner, the addition of HBr results in the formation of


corresponding 1,2- and
1,4- addition products.

Experimentally, it has been found that at low temperature, 1,2-addition product


is formed as a major product while at high temperature, 1,4-addition product is
formed in larger quantity.
The formation of different products at low and high temperature can be
explained taking into consideration two important factors:
(i) Relative rate of 1,2- and 1,4- addition reactions
(ii) Relative stability of 1,2- and 1,4- addition products.
The attack of electrophile (H+) on buta-1,3-diene results in the formation of allyl
carbocation. The formation of 1,2- and 1,4- addition products via allyl
carbocation intermediate can be explained in view of the energy profile (Fig.
7.3) for the reaction as shown below.

(a) The reaction of allyl carbocation and bromide ion to form 1,2- addition
product involves low Ea while that of allyl carbocation and bromide ion to
form 1,4- addition product involves high Ea.
(b) At low temperature, 1,2- addition occurs more readily due to low
activation energy, Ea. As high activation energy is required for 1,4-
addition, only a small fraction of molecules undergo this reaction.
(c) At low temperature, the product formation is an irreversible process. At
low temperature sufficient energy is not available for products to revert
back, through high energy barrier, to allyl cation. Thus, 1,2- addition
reaction dominates at low temperature and is said to be kinetic or rate
controlled.
(d) At high temperature, sufficient energy is available and even 1,4- addition
(high Ea) is feasible.
(e) 1,4- addition product is more stable compared to 1,2- addition product.
(f) At high temperature, 1,2- addition occurs (because of low Ea) but since
1,4- addition product is more stable, 1,2- addition product reverts back to
form allyl carbocation and bromide ion (reversible reaction is possible only
at high temperature) and forms a more stable 1,4- addition product.
(g) The conversion of 1,2- addition product to 1,4- addition product is
energetically favoured.
(h) The 1,4- addition reaction dominates at high temperature and is said to be
thermodynamic or equilibrium controlled.

Reactions with halogens


(a) With one molar equivalent of halogens (Cl2 or Br2)
The reaction of buta-1,3-diene with one molar equivalent of halogen (for
example, reaction of bromine), results in the formation of 3,4-dibromobut-1-ene
(1,2- addition) and 1,4-dibromobut-2-ene (1,4- addition).

Effect of solvent on addition: At low temperature and in


presence of a non-polar solvent, 1,2- addition product is favoured
while at high temperature and in presence of polar solvent, 1,4-
addition product is formed as the major product. A polar solvent

makes the species +CH2–CH=CH–C–H2 more stable due to charge


separation and thus, 1,4- addition dominates.
(b) With two molar equivalents of halogens
Reaction of Buta-1,3-diene with two molar equivalent of chlorine or bromine
results in the formation of tetrachloro- or tetrabromobutane.

7.4.2 Free Radical Addition Reactions


Buta-1,3-diene, analogous to its electrophilic reactions also undergoes free
radical addition reactions, when the reactions are carried out in the presence of
organic peroxides, light, air, or sodium.
Reactions with halogens
Buta-1,3-diene reacts with halogens (Cl2 or Br2) in presence of an organic
peroxide or light to form 1,2- and 1,4-addition products.

Mechanism
Step 1. Chain initiation (generation of free radical).
Peroxides are good free radical generators as they undergo homolytic cleavage to
produce radicals readily. The radical captures a chlorine radical from chlorine
molecule and in turn generates a chlorine radical.

Step 2. Addition of chlorine free radical to diene (formation of allyl radical).


The addition of chlorine radical to olefinic carbon of diene results in the
formation of stable allyl radical.

The stability of allyl radical is due to delocalization of electrons and is a hybrid


of two contributing structures.
Step 3. Abstraction of chlorine radical by allyl radical (formation of 1,2- and 1,4-
addition
products).

Reaction with bromotrichloromethane


The reaction of buta-1,3-diene with bromotrichloramethane in presence of an
organic peroxide results in the formation of 1,2- and 1,4-addition products.

Mechanism
Step 1. Chain initiation

Step 2. Formation of allyl radical


Step 3. Formation of 1,2- and 1,4- addition products

7.4.3 Diels–Alder Reaction [Cycloaddition Reaction]


The reactions of conjugated dienes with alkenes to form a six membered cyclic
adduct is
known as Diels-Alder reaction. It is a 4 + 2 cycloaddition involving reaction of
diene (4π electrons) with 2π electrons containing alkene (referred as dienophile
means diene lover) to form a cyclic product. This is an example of pericyclic or
concerted reaction (p. 132) where all the bond formations and bond breaking
occur at the same time. During the process of cycloaddition the three π bonds in
the reactants are broken and two new carbon–carbon σ bonds and one new
carbon–carbon π bond are formed. For example, thermal addition of ethene to
buta-1,3-diene produces cyclohexene.

Some other Diels–Alder reactions involving substituted olefinic or acetylenic


compounds (dienophiles) and buta-1,3-diene are as follows:

Characteristic features of Diels-Alder reaction


• In Diels-Alder reaction the diene always reacts in s-cis conformation. The
ends of diene in s-cis conformation can readily overlap with ends of π-
system of dienophile.
• Reaction occurs readily if dienophile has electron withdrawing substituents
attached to it.
• The Diels-Alder reaction is stereospecific. The reaction occurs through syn-
(or cis) addition only. In other words, the dienophile adds to diene through
one side only.
• Stereochemistry of diene as well as of dienophile is retained in the reaction.
The groups that are cis- in the reactants remain cis in the product also.
Similarly, the groups that are trans in the reactant remain trans in the
product also.
• In case of substituted dienophiles, the substituents in the transition state
occupy a position, preferably endo to diene. This is because the polar
substituents at endo position interact with π system of diene and stabilize the
transition state.
• Molecular orbital symmetry in Diels-Alder reaction: For a cycloaddition
reaction the electron flow between diene and dienophile should occur
readily. In terms of molecular orbitals, the molecular orbitals of diene must
overlap with the molecular orbitals of dienophile effectively, that is, the
molecular orbitals of the two systems should have same phase overlap. For
an electron flow it is necessary that one system should be electron rich so
that it can donate electrons and the other system should have vacant orbitals,
so that it can accept electrons readily and an effective overlap of orbitals
take place Since, dienes are electron rich so HOMO of diene overlaps
effectively with LUMO (vacant orbital) of alkene. In actual it has been
observed that overlap of HOMO of diene and LUMO of dienophile or the
overlap of LUMO of diene and HOMO of dienophile are symmetrically
allowed and thus effective overlap occurs in either case.

Fig. 7.4 Symmetry allowed 4 + 2 cycloaddition (Diels–Alder reaction) in 1,3-butadiene and ethene. In (I)
overlap of HOMO of 1,3-butadiene occurs with LUMO of ethene. In (II) overlap of LUMO of 1,3-
butadiene occurs with HOMO of ethene.

7.4.4 Reduction and Oxidation Reactions


Hydrogenation
Partial hydrogenation of buta-1,3-diene with one molar equivalent of hydrogen
gives isomeric butenes, i.e. but-1-ene and but-2-ene. Complete hydrogenation
results in the formation of butane as follows:
Ozonolysis
In buta-1,3-diene, the reaction with ozone occurs at both the double bonds. Due
to conjugated double bonds, one of the products of ozonolysis is a dicarbonyl
compound.

7.4.5 Polymerization
Polymerization of buta-1-3-diene occurs in the presence of organic peroxide or
in the presence of sodium to give a rubber like product. The polymer formed in
the presence of sodium is popularly known as Buna rubber. The product may be
linear or a branched chain polymer.

Mechanism. Polymerization occurs through a free radical mechanism as


described below:
Step 1. Chain initiation (generation of free radical)
Let’s represent the free radical •C6H5 as •Rad.
Step 2. Attack of radical on buta-1,3-diene

The allyl radical so formed is resonance stabilized in the following manner.

Step 3(a). Linear polymerization through [I]


Here, chain propagation occurs through terminal allyl radical.

Step 3(b). Branched chain polymerization through [II]


Here chain propagation occurs through non-terminal allyl radical.

Buna-S and buna-N


Buta-1,3-diene may undergo copolymerization with styrene to give buna-S
rubber. The copolymerization of buta-1,3-diene with acrylonitrile (vinyl nitrile)
results in the formation of
buna-N polymer (for details refer Chapter 31).
7.5 ISOPRENE (2-METHYLBUTA-1,3-DIENE)
Isoprene is a low boiling liquid (boiling point 35oC) which polymerizes even at
room temperature. Natural rubber is an important source which gives isoprene
on distillation. Other methods used for the synthesis of isoprene are discussed in
the subsequent subsection.

7.5.1 Preparation
From isopentane or isopentene (by dehydrogenation)
Isopentane (2-methylbutane) or isopentene (3-metylbut-1-ene) on heating with
alumina and chromium oxide at a temperature of about 600°C undergo
dehydrogenation to form isoprene.

From isoamyl alcohol


3-Methylbutan-1-ol (isoamyl alcohol) is converted to isoamyl chloride which on
reaction with chlorine followed by reaction with soda-lime gives isoprene.

From acetone
The reaction of acetone with sodamide and then with acetylene, in a series of
reactions is used to synthesize isoprene as follows:
7.5.2 Chemical Properties
Isoprene follows the same addition reactions as buta-1,3-diene though the
ozonolysis and polymerization products are different. Reactions of isoprene can
be summarized as follows:
Addition reactions

Ozonolysis
The ozonolysis of isoprene results in the formation of methanal (2 moles) and 2-
oxopropanal (dicarbonyl compound).

Polymerization
Polymerization of isoprene is carried out in the presence of an organic peroxide
and follows a free radical mechanism (refer Chapter 31).
The polymer of isoprene having cis- configuration at all the double bonds is
known as natural rubber. The polymer of isoprene having trans- configuration
at all the double bonds is known as synthetic rubber or gutta percha.

Vulcanization of rubber: The natural and synthetic rubbers are associated with a
drawback that they become sticky during hot weather and become hard in cold.
To overcome this and also to increase the strength, the rubber is heated with 5–
10% of sulfur and this process is known as vulcanization of rubber. Sulfur forms
a cross-link between the linear polymer chains thereby giving a compact
structure, which strengthens the polymer.
7.6 CHLOROPRENE (2-CHLOROBUTA-1,3-
DIENE)
Chloroprene is a liquid which boils at 60oC. Chloroprene can be synthesized
from acetylene as follows:

.
The reactions which chloroprene undergoes are similar to buta-1,3-diene and
isoprene. The polymerization product of chloroprene is popularly known as
neoprene rubber.
EXERCISES
1. Explain the following terms with suitable examples:
(a) Isolated dienes
(b) Cumulated dienes
(c) Conjugated dienes
2. What is conjugation? How can the greater stability of conjugated dienes be
explained?
3. Discuss molecular orbital structure of buta-1,3-diene and the way it can
explain the stability of conjugated dienes systems.
4. Describe the methods of preparation of buta-1,3-diene.
5. Explain with mechanism, the addition of bromine to buta-1,3-diene. Why
buta-1,3-diene undergoes both 1,2- and 1,4- additions?
6. Discuss the mechanism of addition of HBr to buta-1,3-diene.
7. Discuss the effect of temperature on 1,2- and 1,4- addition in buta-1,3-diene.
8. Explain the free radical addition mechanism in butadiene.
9. Write a short note on Diels–Alder reaction.
10. What is the difference between synthetic and natural rubber?
11. Write short notes on
(a) Vulcanization of rubber
(b) Buna-S
(c) Buna-N
12. Give the reductive ozonolysis products of buta-1,3-diene and penta-1,3-
diene.
Chapter8
Alkynes

8.1 INTRODUCTION
The class of organic compounds containing a carbon–cabon triple bond are
called alkynes. Alkynes have the general formula CnH2n-2. Alkynes do not
occur free in nature but are produced during cracking of petroleum. Commonly,
the members of alkyne family are termed acetylenes. The first member of the
series is ethyne or acetylene.
In ethyne, each carbon is sp hybridized and sp–sp overlap results in the
formation of carbon-carbon σ bond while sp-s overlap results in carbon–
hydrogen σ bond. Also, each carbon has two pure p orbitals which are
orthogonal (perpendicular) to each other. Overlap of these p orbitals results in
the formation of two π bonds. Ethyne has a linear structure.

Nomenclature of alkynes has already been discussed in detail in Chapter 2.


Alkynes that have a hydrogen attached to triple bonded carbon (≡CH) are termed
as terminal alkynes. The other alkynes are referred as nonterminal alkynes or
internal alkynes. Further, alkynes with four or more number of carbons exhibit
chain isomerism. Some examples of the IUPAC nonmenclature and isomerism in
alkynes are as follows:
The more substituted alkynes (nonterminal) are more stable.

8.1.1 Physical Properties


Ethyne (acetylene) and propyne are gases. In general, the boiling point of
alkynes increases with increase in the molecular mass. The values of heat of
combustion (Table 8.1) indicate the stability of alkynes. The terminal alkynes
have lower boiling point than corresponding isomeric nonterminal alkynes. The
alkynes are insoluble in water and are less dense than water.
8.2 PREPARATION OF ALKYNES
Various preparative methods of alkynes are discussed as follows.
Preparative methods for acetylene
Acetylene is usually prepared industrially from (i) calcium carbide and (ii)
methane.
From calcium carbide. The hydrolysis of calcium carbide results in the
formation of acetylene. This is a process which can be used to prepare acetylene
in the laboratory and in the industries.

From methane. On heating the mixture of methane and oxygen (1:1 by volume)
at 1500oC, acetylene is obtained. Higher members of the alkyne family can be
conveniently prepared by acetylene.

Dehydrohalogenation of dihaloalkanes (1,1- and 1,2- eliminations)


A geminal or vicinal dihaloalkane on double dehydrohalogenation in presence of
a strong base results in the formation of an alkyne.
The first dehydrohalogenation is a fast reaction that results in the formation of
vinyl halide. The further dehydrohalogenation (removal of second HX molecule
) is a rather slow process that results in the formation of alkyne.

The vinyl halide fomed as an intermediate product is highly stable due to


delocalization (p. 419). Thus, removal of HX from vinyl halide is a difficult
process and requires a strong base for elimination.
The dehydrohalogenation is carried out with concentrated alcoholic KOH
solution or fused KOH at 200°C. The reactions are also carried out with strong
base like sodamide (NaNH2) at relatively low temperatures. For example

Dehydrohalogenating Agents: NaNH2 versus Alcoholic


KOH
The dehydrohalogenation of vicinal or geminal dihaloalkanes with
strong base like NaNH2 (sodamide) results in the formation of
terminal alkynes. Even the nonterminal alkynes are isomerized to
terminal alkynes in presence of sodamide.
On the other hand use of alcoholic KOH results in the formation of
nonterminal alkynes. The terminal alkynes formed during
dehydrohalogenation undergo rearrangement to nonterminal
alkynes in presence of alcoholic KOH.
The isomerization of terminal alkynes to nonterminal alkynes and
vice versa in presence of base is discussed later in the chemical
properties of alkynes (refer section 8.3.6)
From haloforms
In presence of silver powder, two molecules of haloform undergo
dehalogenation to produce acetylene.

From dehalogenation of vicinal tetrahaloalkanes


Vicinal tetrahaloalkanes are dehalogenated by heating with zinc dust in ethanol
to produce alkynes. For example

Kolbe’s electrolytic reaction


Electrolysis of concentrated aqueous solution of alkali salt of an unsaturated
dicarboxylic acid results in the formation of alkyne, at anode.
For example,
Preparation of higher alkynes from lower alkynes
From sodium acetylides by reaction with haloalkanes: Terminal alkynes react
with sodamide to give acetylides which on reaction with haloalkane produce
higher alkynes as follows.

Further alkylation of terminal alkynes results in the formation of higher


nonterminal alkynes

For example

From Grignard reagent: Grignard reagent reacts with alkynes to form


acetylenic Grignard reagent which on reaction with haloalkanes produces
alkyne.
8.3 CHEMICAL PROPERTIES OF ALKYNES
Alkynes being unsaturated undergo addition reactions. The important chemical
reactions of alkynes include electrophilic addition reactions, nucleophilic
addition reactions, reactions due to acetylenic hydrogen, polymerization,
isomerization and oxidation reactions. A detailed discussion of these reactions is
as follows:

8.3.1 Addition of Hydrogen


Complete hydrogenation of alkynes gives corresponding alkane by passing
hydrogen in the presence of metal catalysts. Partial hydrogenation of alkynes
may result in the formation of
cis- or trans- alkenes depending upon the reagent used (already discussed in
section 6.2.1).
For example,

8.3.2 Electrophilic Addition Reactions

Alkynes are Less Reactive than Alkenes Towards


Electrophilic Addition Reactions
The alkynes have higher electron density compared to alkenes, still
the electrophilic additions occur at a slow rate in alkynes. This is
explained on the basis of following factors:
(1) In alkynes the carbon–carbon triple bond (C ≡ C) is a strong
bond because of smaller bond length. As a result electrons in
triple bond are held more tightly and thus it is difficult for an
electrophile to take out the electron pair from triple bond.

The addition of electrophile to alkene proceeds through a cyclic


intermediate (cyclic carbocation Section 6.3.2) whereas in
alkynes the formation of cyclic intermediates is relatively
difficult and thus, alkynes show low reactivity.
(2) In case of alkynes the addition of an electrophile results in the
formation of vinyl carbocation. However, in case of alkenes the
addition of electrophile forms alkyl carbocation. The vinylic
carbocation are generally less stable compared to alkyl
carbocation. As a result, the electrophilic addition in alkynes
occurs at a slower rate compared to alkenes.

Addition of halogens
The addition of one equivalent of halogen to alkynes results in the formation of
haloalkenes. However 2 equivalents of halogens add on to alkynes to form
tetrahaloalkane derivatives.

For example,

Iodine is less reactive and only one mole of it adds to alkyne to form
diiodoalkene.
For example,

Addition of hydrogen halides (formation of dihaloalkanes)


Two equivalents of hydrogen halide add to an alkyne to give dihaloalkane. The
addition of hydrogen halides is regioselective and follows Markovnikov’s rule.
The addition of one equivalent of HX results in the formation of vinyl halide
which further undergoes addition of another equivalent of HX to give
dihaloalkane.
For example,
The reaction with first equivalent of HX proceeds through the formation of
vinylic carbocation. The addition of electrophile (H+) results in the formation of
a more stable carbocation which reacts with halide ion (X–) to from vinyl halide.
Further electrohilic addition of HX forms dihaloalkane.

The reaction of alkyne with HBr in the presence of a peroxide follows anti-
Markovnikov’s rule.

Addition of water (formation of carbonyl compounds)


Alkynes react with water in the presence of acid and mercury salts to produce
carbonyl compounds. Mercury ion (Hg2+; a Lewis acid) acts as a catalyst in
hydration reactions. The addition of water follows Markovnikov’s rule and
proceeds through formation of more stable carbocation.
Mechanism

The products obtained by hydration of different alkynes are as follows:

Thus, hydration of alkynes gives ketones except for ethyne, which on hydration
forms an aldehyde, that is acetaldehyde.
Reaction with hypohalous acid
[Addition of Halogen and water]
The reaction of alkynes with Hypohalous acid result in the formation of geminal
dihalocarbonyl compounds. The mechanism does not involve the addition of
HOX as such to alkyne rather it is the addition of halogen (behaves as
electrophilic part of reagent) followed by reaction of water (behaves as
nucleophilic part of reagent).
Thus, net result is addition of HO–X+ to alkynes. For example

Reaction with boronhydride


The hydroboration of alkynes with bulky dialkyl borane (R2BH) results in the
formation of vinylborane. The syn-addition of dialkylborane occurs in
accordance with Markovnikov’s rule.

The vinylboranes may undergo following reactions:


(i) The vinylborane on oxidation with H2O2 in alkaline medium results in the
formation of an enol that rapidly tautomerizes to produce a carbonyl
compound. For example,

Terminal alkynes on hydroboration-oxidation form aldehydes whereas,


nonterminal alkynes on hydroboration-oxidation form ketones.
(ii) The vinylborane on protonation in acidic medium form an alkene

Vinylborane can undergo oxidation with hydrogen peroxide and NaOH to yield
carbonyl compounds while hydrolysis of a vinylborane in acidic medium
(protonolysis) gives cis- alkenes.

8.3.3 Nucleophilic Addition Reactions


Alkynes undergo nucleophilic addition and have high reactivity towards
nucleophiles. This is to be noted that
(a) Reactivity of alkynes towards nucleophiles is higher than that of alkenes.
(b) The nucleophilic part of the reagent attacks the triple bonded carbon to
form carbanion which then undergoes addition of electrophile to form the
addition product.
Ethyne undergoes nucleophilic addition with active hydrogen containing
compounds like HCN, ROH, RCOOH, RSH etc. to form corresponding vinyl
derivatives. The reactions are catalyzed by base. The general mechanism for
nucleophilic addition is as follows:

A few examples of nucleophilic substitution reactions in acetylene are


The above reactions of acetylene result in the formation of important vinyl
derivatives and these reactions are known as vinylation reactions.

8.3.4 Reactions Involving Acetylenic Hydrogens


Acidic character of alkynes
Alkynes are highly unsaturated compounds and one of the important reactions of
alkynes involves their addition to one or both of the π bonds.

The Acetylenic Hydrogen in Terminal Alkynes (–C≡C–H)


is Acidic in Nature.
The carbon attached through a single bond, double bond, or triple
bond is sp3, sp2, or sp hybridized respectively. The percentage of s

character increases as one moves from sp3 to sp2 to sp hybrid


carbon.
In spherical shaped s orbitals, the electrons are held close to the
nucleus. Greater the s character in a hybrid orbital, greater is the
tendency of hybridized atom to attract the shared pair of electrons
towards itself (i.e. electronegativity). Thus, the electronegativity of

sp hybrid carbon is more compared to sp2 and sp3 hybrid carbon.


In terminal alkynes, the sp hybridized carbon is highly
electronegative and thus, the electrons of C–H bond are held tightly
by carbon. This makes the C–H bond (in
–C≡C–H) relatively weak and removal of hydrogen occurs readily.
In other words, the acetylenic hydrogen is highly acidic in nature.
The overall acidic strength in alkane, alkene and alkyne is as
follows:

Due to high electronegative character and hence, high acidity of acetylenic


hydrogens, these are easily replaced in the presence of strong bases to form
acetylides.
Reactions with sodamide, lithium amide, and Grignard reagent
The terminal alkynes (having —C≡≡CH group) react with sodamide or lithium
amide to form corresponding sodium or lithium acetylides. Terminal alkynes
also react with Grignard reagent to produce acetylenic Grignard reagent.

Synthetic application of metal alkynides


These alkynides are used in organic synthesis where a carbon-carbon bond is to
be introduced. The following reaction sequences result in the formation of
product with higher number of carbons than the starting alkyne.
(i) Formation of higher alkynes by the reaction with haloalkanes. These
reactions take place as follows.

For example,

(ii) Formation of acetylenic acids by reaction with carbon dioxide.


Acetylenic acids are formed when alkynes react with CO2 as under.
The reaction of alkynes with CO2 results in the formation of acetylenic acids.
For example,
Reaction of terminal alkynes with ammonical cuprous chloride
Terminal alkynes react with ammonical cuprous chloride solution to give red
precipitate of corresponding copper acetylide.

For example,

Reaction of terminal alkynes with ammonical silver nitrate


Reaction of terminal alkynes with ammonical silver nitrate results in the
formation of white precipitate of corresponding silver acetylide.

For example,

The reaction of alkynes with ammonical silver nitrate or


ammonical cuprous chloride is used as a qualitative test to
distinguish between terminal and non-terminal alkynes. For
example, But-1-yne forms a red precipitate with ammonical
cuprous chloride and a white precipitate with ammonical silver
nitrate while no such precipitation is observed with But-2-yne.

8.3.5 Polymerization Reactions


One of the important reactions of alkynes is polymerization, that is combining of
more than two units of alkynes. Polymerization may be (i) linear or (ii) cyclic.
Linear polymerization
The molecules of acetylene undergo linear polymerization in presence of
cuprous chloride and ammonium chloride as follows.

The dimerization of acetylene gives vinylacetylene which further combines with


third molecule of acetylene to give divinylacetylene as the linear polymerized
product.

Cyclic polymerization
Alkynes polymerize when passed over a red hot tube to form cyclic (at times
aromatic)
compounds. For example, three molecules of acetylene undergo cyclic
polymerization to produce benzene. In a similar manner, three molecules of
propyne undergo cyclic polymerization to produce 1,3,5-trimethylbenzene
popularly known as mesitylene. The polymerization of four molecules of ethyne
in presence of catalyst results in the formation of cyclooctatetraene.
8.3.6 Isomerization (Acetylene Allene Rearrangement)
Terminal alkynes may be converted to non-terminal alkynes and vice versa
through an acetylene allene rearrangement. For example, but-1-yne is converted
to but-2-yne using ethanolic potassium hydroxide. However, but-2-yne can be
converted to but-1-yne (again through an allene intermediate) by using NaNH2.

The following mechanism, involving allene as an intermediate, in presence of a


base is followed.
As shown above, in presence of base like KOH, the nonterminal alkyne is the
rearranged product. However, in presence of a strong base like NaNH2 the
rearranged product is a terminal alkyne, as once formed this can be removed
irreversibly from the mixture through formation of sodium salt.

8.3.7 Oxidation Reactions


With alkaline potassium permanganate (formation of carboxylic acids)
The alkynes on treatment with alkaline KMnO4 undergo oxidative cleavage at
triple bond to form carboxylic acids. The terminal (≡≡CH) acetylene group
however, oxidizes to carbon dioxide and water.

Ozonolysis of alkynes
Alkynes on treatment with ozone at low temperature in CCl4 solvent, followed
by hydrolysis result in the formation of carboxylic acids.
Structure Determination of Alkynes
The ozonolysis–hydrolysis of alkynes results in the cleavage at
triple bond. The cleaved parts contain carboxylic groups at the
carbon atoms, which were initially joined through triple bond. The
ozonolysis–hydrolysis products of alkynes are used for determining
the structure of alkyne.
For example, the ozonolysis–hydrolysis of an alkyne gives
propanoic acid and ethanoic acid. To determine the structure of
alkyne, we write the products side by side with their carboxylic
groups facing each other and instead of carboxylic groups, place
carbon–carbon triple bond to determine the structure of alkyne.

Oxidative coupling
This reaction is undergone by terminal alkynes (except ethyne). Terminal
alkynes react with cuprous chloride and ammonium hydroxide to give copper
alkynides which can be coupled together by oxidation to form dialkynes. The
oxidation is carried out in presence of oxygen and acetic acid.

For example,

The above oxidative coupling reaction is known as Cadiot–Chodkiewick


oxidative coupling reaction.
SELECTED SOLVED EXAMPLES
Example 1. Which of the following reagents can be used for distinguishing
between propene and propyne? Justify your answer.
(a) Br2/CCl4 (b) Dilute KMnO4 (c) Cu2Cl2 (d) Ag(NH3)2OH
Solution. Cu2Cl2 and Ag(NH3)OH can be used to differentiate between propene
and propyne. Propyne gives red precipitate with Cu2Cl2 and white precipitate
with Ag(NH3)2OH.
Br2/CCl4 and dilute KMnO4 are used to test the unsaturation (of both alkenes
and alkynes).
Example 2. Which of the following compounds forms a white precipitate with
ammonical silver nitrate?
(a) CH3CH2C≡≡CCH3
(b) CH2==CHCH2CH3
(c) CH3CH2C≡≡CH
(d) CH3CH==CHCH3
Solution. Only terminal alkynes give white precipitate with ammonical silver
nitrate.
So (c) but-1-yne reacts with ammonical silver nitrate to give a white ppt.
Example 3. How will you convert pent-2-yne to trans-pent-2-ene?

Solution
Example 4. How will you distinguish hex-1-yne from hex-3-yne?
Solution. Cu2Cl2 and Ag(NH3)OH can be used to distinguish hex-1-yne (an
terminal alkyne) from hex-3-yne. Hex-1-yne gives red precipitate with Cu2Cl2
and white precipitate with Ag(NH3)2OH while hex-3-yne does not react.
Example 5. Complete the following sequence of reactions.
Solution
A. But-2-ene
B. 2,3-Dibromobutane
C. But-2-yne
D. Butanone
Example 6. Carry out the following conversions (for preparation of
corresponding alkynes).
(a) Pent-2-ene to pent-2-yne
(b) But-1-ene to but-2-yne
(c) 1-Bromo-4-methylpentane to 4-methylpent-1-yne
Solution

Example 7. Carry out the following conversions.


(a) Ethyne to 2-chlorobuta-1,3-diene (chloroprene)
(b) Propene to 4-methylpent-2-yne
(c) Propyne to cis-but-2-ene
Solution
EXERCISES
1. Draw the structures of all isomers of alkynes having molecular formula C5H8
and give their IUPAC names.
2. What is the hybridization state of carbon in acetylene? Compare it with
ethene and ethane, and comment on the bond angles and bond lengths.
3. How will you prepare alkynes by
(a) Dehydrohalogenation of 1,2-dihalides (vic-dihalide).
(b) Dehalogenation of tetrahalides.
(c) Reaction of acetylides with haloalkanes.
4. Explain why alkynes are less reactive than alkenes towards electrophilic
addition reactions?
5. Explain the acidic nature of acetylenic hydrogen. Compare its acidic strength
with ethene and ethane.
6. Write a short note on
(a) Addition mechanism of halogen acids on alkynes
(b) Hydrogenation of alkynes
7. How will you distinguish between pent-1-yne and pent-2-yne?
8. Explain why alkynes undergo nucleophilic addition reactions but alkene does
not.
9. Explain the mechanism of nucleophilic addition for alkynes.
10. Give the product obtained by the reaction of dil. H2SO4/HgSO4 with (a)
acetylene,
(b) prop-1-yne, and (c) but-2-yne. Also give the mechanism of reaction with
any one of them.
11. How will you synthesize the following compounds from acetylene?
(a) Propyne
(b) Dichloroacetaldehyde
(c) Benzene
(d) Acrylonitrile
(e) Ethylene dichloride.
12. Write a short note on hydroboration–oxidation reactions of alkynes.
13. What happens when
(a) But-2-yne is treated with hydrogen in presence of Lindlar’s catalyst.
(b) But-2-yne is treated with sodium in liquid ammonia.
(c) But-1-yne is treated with (a) ammonical AgNO3 and (b) ammonical
Cu2Cl2 solution.
14. Explain with suitable examples, the electrophilic and nucleophilic reactions
of acetylene.
Chapter9
Concepts of Aromaticity, Benzene
and its Derivatives

A. CONCEPTS OF AROMATICITY
9A.1 Introduction
In earlier days of organic chemistry, the term ‘Aromatic’ was used for
compounds associated with certain aroma or fragrance. Benzene is the
representative member of the class of aromatic organic compounds which was
isolated in 1825 by Michael Faraday. The term aromatic has been widely used
for benzene and its derivatives since many of them are associated with a distinct
odour. However, there are other organic compounds which are known to be
associated with some odour but are not classified as aromatic. Thus, the
classification of organic compounds as aromatic is not only based on aroma but
on structure and reactivity of the compounds. To study the characteristics of
aromatic compounds and the criteria for aromaticity, let us first study the
structure of benzene.
9A.2 STRUCTURE OF BENZENE
Benzene has a molecular formula C6H6 that suggests a high degree of
unsaturation. However, it does not undergo the usual reactions of unsaturated
compounds such as addition, oxidation, and reduction. It does not decolourize
bromine water or potassium permanganate solution, which is the characteristic
of unsaturated compounds. In fact, benzene undergoes substitutions reactions.
Reaction of benzene with bromine in presence of ferric bromide catalyst results
in the formation of only one compound i.e. C6H5Br. This suggests that all
hydrogens in benzene are equivalent. This unusual behaviour of benzene is
attributed to its structure.

9A.2.1 Kekule Structure


In 1865, Kekule proposed a six membered ring structure for benzene where each
carbon is attached to one hydrogen atom. To satisfy the tetra-valency of carbon,
he further proposed the presence of three alternate double bonds in benzene ring,
which change their position rapidly. He explained that two forms of benzene
exist in equilibrium and cannot be isolated.

Although Kekule structure satisfied the structural features of benzene and also
explained the equivalent nature of hydrogen, it was not able to account for the
unusual behaviour of benzene. However, the Kekule structure of benzene was a
giant step forward and for this reason, the structure is still used but at present,
the explanation for the structure and reactivity of benzene is given in an entirely
different manner.

9A.2.2 Resonance Structure


The resonance theory gave the correct description for the structure of benzene. It
states that whenever a molecule is represented by two or more Lewis structures
(known as contributing structures) any one of the structure alone is not able to
explain the characteristics of the molecule on the whole. The actual structure in
such cases is a hybrid of all the contributing structures. These contributing
structures (or resonance forms) are hypothetical and cannot be isolated. These
are represented by placing a double-headed arrow between them and all such
structures are said to be in resonance with each other (Note that contributing
structures are never in equilibrium as these are not real structures). Thus, for
benzene major contributing structures are written as follows:

The actual hybrid structure has lower energy compared to either of the
contributing structures alone. This decrease in energy of the hybrid structure
stabilizes the molecule and is known as resonance energy. The resonance
energy is also termed as delocalization energy (see Notabilia 2). Benzene has
high resonance energy. In general, all aromatic compounds show high resonance
energy.

9A.2.3 Orbital Picture of Benzene


Experimental evidence indicate the molecule of benzene to be planar with all the
six carbon–carbon bond lengths being same. The bond length value in benzene is
in between that of a single and the double bond. The orbital picture of benzene
explains the behaviour of benzene in an accurate way. The six carbons in
benzene are sp2 hybridized. Each carbon has three hybrid orbitals, two of which
are used in the formation of carbon–carbon σ bonds on either sides and one of
the hybrid orbitals is used in the formation of a carbon–hydrogen σ bond. This
gives a planar skeleton with six carbons attached to each other through σ bonds
in a cyclic manner and further each carbon is attached to one hydrogen. Every
carbon also has a pure p orbital with an unpaired electron. In all, there are six p
orbitals (six unpaired electrons) present in the same plane i.e. plane of the six
membered ring. These six p orbitals overlap sideways to form a π electron cloud
above and below the plane of the ring. Due to this overlap, the six electrons are
shared equally among all the six carbons and this is known as delocalization of π
electrons.

Fig. 9.1 Molecular orbital model of bonding in benzene. (a) C–C sigma bonds are a result of sp2–sp2
orbital overlap, C–H sigma bonds are a result of sp2-1s orbital overlap. The six 2p orbitals, each
containing one electron also combine (b) π-cloud formation due to six 2p orbital overlap.

It is evident that double bonds are not localized between any two carbons rather,
there is a continuous delocalization of π electrons. This delocalization causes all
the six carbon–carbon bond lengths to be same (1.39 Å). This value lies in
between a pure C–C bond length 1.54 Å and C=C bond length (1.33 Å).
Benzene is thus, represented as a hexagon with a circle inscribed, where the
circle represents delocalized π electrons.

Fig. 9.2 Structure of benzene and its representation.


9A.3 RESONANCE ENERGY: STABILITY OF
BENZENE
Experimentally, the heat of hydrogenation of benzene is found to be –208
kJ/mol. We can calculate heat of hydrogenation of benzene from Kekule
structure. This is done by taking into consideration the heat of hydrogenation of
cyclohexene. Table 9A.1 tabulates the actual and calculated values of heat of
hydrogenation of cyclohexene, cyclohexa-1,4-diene, and cyclohexa-1,3,5-triene.
The actual heat of hydrogenation of cyclohexene is –119.6 kJ mol–1, for diene
and triene system it is calculated by following formula:
Calculated Heat of hydrogenation = Number of π-bonds × Heat of hydrogenation
of cyclohexene (–119.6 kJ mol–1)
Table 9A.1 Calculating the heat of hydrogenation of benzene

For benzene, the calculated value is –358.8 kJ mol–1 while the experimentally
observed value is
–208 kJ mol–1. Thus, benzene is stabilized due to lowering of energy by 150.8
kJ mol–1 (i.e.
358.8–208). This difference of energy between the observed heat of
hydrogenation for actual molecule and the one calculated from the hypothetical
structure is known as resonance energy (or delocalization energy). Thus,
benzene has a resonance energy of 150.8 kJ mol–1.
Characteristics of Benzene
The discussion above explains the unusual characteristics of benzene, which is
also known as its aromatic character.
Benzene is a cyclic planar molecule. All the carbons are sp2 hybridized and the
p orbital available on each carbon contributes towards continuous delocalization
of π electrons. This provides stability to benzene and imparts it high resonance
energy. To retain the resonance energy and thereby the stability, benzene
undergoes substitution reactions. As a result, benzene does not behave as an
unsaturated system and does not undergo addition reactions.
9A.4 HUCKEL’S RULE AND AROMATICITY
In 1931, Huckel gave a rule for monocyclic compounds, which states that the
cyclic compounds containing (4n + 2) delocalized π electrons exhibit aromatic
character. Here, n can be zero or a whole number. For example, for n = 0 the
value of (4n + 2) is 2 and for n = 1, the (4n + 2) is 6 and so on. This means that
systems containing 2, 6, 10, 14, 18, 22, and so on, delocalized π electrons show
aromatic behaviour.

A compound is said to exhibit aromaticity if it satisfies all of


the following conditions simultaneously:
(i) It is cyclic.
(ii) It is planar. In general presence of sp2 hybridized carbons in the
system imparts planarity.
(iii) It exhibits continuous delocalization of π electrons (continuous
delocalization is possible if p orbitals are available on each carbon
for overlap).
(iv) It should follow Huckel’s rule, also known as Huckel magic
number, that is, it should have (4n + 2) number of delocalized π
electrons where n is zero or a whole number.
9A.5 AROMATICITY IN BENZENE AND OTHER
CYCLIC SYSTEMS
Aromaticity is not restricted to benzene and its derivatives but is also extended
to other cyclic systems, cyclic ions, fused cyclic systems, and heterocyclic
compounds. Following examples discuss the concept of aromaticity in different
systems.

9A.5.1 Aromaticity and the Three Membered Ring


Systems
Cyclopropene
Characteristics of cyclopropene molecule are as follows:

Thus, cyclopropene is not aromatic as all the conditions are not simultaneously
satisfied. The condition of continuous delocalization is not followed.
Cyclopropenyl cation
Characteristics of cyclopropenyl cation are as follows
(i) It is cyclic
(ii) It is planar
(iii) It exhibits continuous delocalization. All the carbons of cyclic system are
sp2 hybridized, and thus p orbitals are available for delocalization.

(iv) Obeys Huckel’s rule, as the number of delocalized π electrons is two.


Thus, cyclopropenyl cation is aromatic, as all the conditions are simultaneously
satisfied.
Cyclopropenyl anion
Characteristics of cyclopropenyl anion are as follows
(i) It is cyclic
(ii) It is planar
(iii) It exhibits continuous delocalization. All the carbons of cyclic system are
sp2 hybridized, and thus p orbitals are available for delocalization.

(iv) Does not obey Huckel’s rule, as the number of delocalized π electrons is
four.
Thus, cyclopropenyl anion is not aromatic, as all the conditions are not
simultaneously satisfied.

9A.5.2 Aromaticity and Four Membered Ring Systems


Cyclobutadiene

Thus, cyclobutadiene is not aromatic as all the conditions are not satisfied
simultaneously. The condition of Huckel’s rule is not followed.

9A.5.3 Aromaticity and Five Membered Ring Systems


Cyclopentadiene
Characteristics of cyclopentadiene are as follows
Thus, cyclopentadiene is not aromatic as all the conditions are not
simultaneously satisfied. The conditions of continuous delocalization and that of
Huckel’s rule is not followed.

Cyclopentadienyl Ions
The sp3 hybridized carbon in cyclopentadiene is not involved in
delocalization. Removal of a hydrogen (either as a proton (H+) or as a
hydride (H–)) from this carbon results in the formation of an anion or
a cation where all the carbons are sp2 hybridized and thus, continuous
delocalization of π electrons takes place.

Cyclopentadienyl anion
Characteristics of cyclopentadienyl anion are as follows
(i) It is cyclic
(ii) It is planar
(iii) It exhibits continuous delocalization. All the carbons of cyclic system are
sp2 hybridized, and thus p orbitals are available for delocalization.
(iv) Obeys Huckel’s rule, as number of delocalized π electrons is six.
Thus, cyclopentadienyl cation is aromatic, as all the conditions are satisfied
simultaneously.
Cyclopentadienyl cation
Characteristics of cyclopentadienyl cation are as follows
(i) It is cyclic
(ii) It is planar
(iii) It exhibits continuous delocalization. All the corbons of cyclic system are
sp2 hybridized and thus p orbitals are available for delocalization.

(iv) Does not obey Huckel’s rule, as number of delocalized π electrons is four.
Thus, cyclopentadienyl cation is not aromatic, as all the conditions are not
satisfied simultaneously. The condition of Huckel’s rule is not followed.

9A.5.4 Aromaticity and Six Membered Ring Systems


Benzene
9A.5.5 Aromaticity and Seven Membered Ring
Systems
Cycloheptatriene

Cycloheptatrienyl Ions
The sp3 hybridized carbon in cycloheptatriene is not involved in
delocalization. Removal of a hydrogen (either as a proton (H+) or as a
hydride (H–)) from this carbon results in the formation of an anion or
a cation, where all the carbons are sp2 hybridized and thus,
continuous delocalization of π electrons takes place.
Cycloheptatrienyl cation (Tropylium ion)
Characteristics of cycloheptatrienyl cation are as follows
(i) It is cyclic
(ii) It is planar
(iii) It exhibits continuous delocalization. All the carbons of cyclic system are
sp2 hybridized, and thus p orbitals are available for delocalization.

(iv) Obeys Huckel’s rule, as the number of delocalized π electrons is six.


Thus, cycloheptatrienyl cation is aromatic, as all the conditions are satisfied
simultaneously.
Cycloheptatrienyl anion
Characteristics of cycloheptatrienyl anion are as follows
(i) It is cyclic
(ii) It is planar
(iii) It exhibits continuous delocalization. All the carbons of cyclic system are
sp2 hybridized and thus p orbitals are available for delocalization.
(iv) Does not obey Huckel’s rule, as number of delocalized π electrons is
eight.
Thus, cycloheptatrienyl anion is not aromatic, as all the conditions are not
satisfied simultaneously. The condition of Huckel’s rule is not followed.

9A.5.6 Aromaticity and Eight Membered Ring


Systems
Cyclooctatetraene (C8H8)
Characteristics of cyclooctatetraene are as follows
9A.5.7 Aromaticity and Annulenes
Monocyclic systems with conjugated double bonds are termed annulenes.
Bracketting a number indicates the ring size. For example, cyclobutadiene is
[4]annulene, benzene is [6]annulene, and cyclooctatetraene is [8]annulene. The
term annulene is used frequently for large ring size monocyclic systems (C10
and more).

The [10]annulene due to small size of ring cannot accomodate the hydrogens
present at the centre of the ring and this steric hindrance prevents it to attain
planarity. However in [14]annulene and other higher annulenes, due to large size
of ring the hydrogens at the centre are easily accomodated and annulenes attain a
nearly planar structure.
More on the aromatic character in annulenes
A compound is more often said to be aromatic if it is a planar system with
continuous delocalization of electrons and obeys Huckel’s rule.
In any system the delocalization of electrons occurs effectively, if the overlap of
p orbitals is effective. The overlap of p orbitals can occur effectively only when
these orbitals are in the same plane. Any deviation from planarity in a molecule
(system) will retard the delocalization process in it and that will reduce the
aromatic character of the system. Thus, planarity is an essential factor to
describe the aromatic character.
As explained earlier that [10]annulene (structure-I) in spite of obeying Huckel’s
rule, does not exhibit aromatic character because of deviation from planar
structure. The hydrogens present at position 1 and 6 in the ring system prevent it
from attaining a planar structure because of steric hindrance and thus, interfere
in the overlap of orbitals.
However the aromaticity in this molecule can be introduced by eliminating the
interaction of hydrogens (at C1 and C6). One of the best strategy is to bridge 1
and 6 positions as shown in structure-II. The bridging replaces the hydrogens
and prevents the steric hindrance thereby, providing sufficient overlap of
orbitals. This provides aromatic character to bridged molecule.

Similarly, in case of [14]annulene (structure-III) the large ring size somewhat


compensates the steric hindrance due to inner hydrogens and molecule has
nearly planar structure. The molecule shows aromaticity but is not very stable.
However aromaticity and stability in such molecule can be increased effectively
by introducing a triple bond in the system. The introduction of triple bond
replaces the hydrogens and prevents the steric hindrance thereby, providing
sufficient overlap of orbitals. This increases the aromatic character in dehydro
[14]annulene (structure-IV, dehydro means two hydrogen less than the parent
molecule). The two sets of p orbitals in triple bonds are mutually perpendicular.
One of them, which is in the same plane as the p orbitals of the ring system,
participates in delocalization while the other being in perpendicular plane does
not participates in delocalization. Thus the total number of electrons
participating in delocalization in dehydroannulene (structure-IV) remains the
same as in [14]annulene. Relieve in steric strain increases delocalization of
electrons, which in turn increase the aromatic character of the system. Further as
discussed earlier, the bridged [14]annulenes [structure-V] also exhibit more
aromatic character and are stable systems.
9A.5.8 Aromaticity and Other Ring Systems

Azulene
Azulene is a nonbenzenoid fused ring system unlike naphthalene,
anthracene and phenanthrene that are benzenoid systems
(containing fused benzene rings). In azulene a seven and five
membered ring systems are fused together. Azulene is an aromatic
system and readily undergoes electrophilic substitution reactions.

Aromatic heterocyclic compounds

All the above heterocyclic compounds are aromatic. The carbons as well as
nitrogen, oxygen, and sulfur are sp2 hybridized which give a planar structure. In
each case, the total number of delocalized π electrons is six, that is, Huckel’s rule
is followed. In pyrrole, furan, and thiophene, a lone pair of electrons present in p
orbital of nitrogen, oxygen, and sulfur respectively participates in delocalization
with p orbital of sp2 hybridized carbons. In pyridine, the lone pair of electrons
on nitrogen does not participate in delocalization. (For further details on
aromaticity, see Chapter 26 on Heterocyclic Compounds).

Anti-aromatic and Non-aromatic Compounds


The compounds that do not exhibit aromatic character may be
classified as anti-aromatic and non-aromatic.
Anti-aromatic: The cyclic planar molecules with continuous
delocalization but having 4n number of delocalized π electrons are
said to be anti-aromatic. For example, cyclobutadiene (number of
delocalized π electrons = 4; see section 9A.5.2).
Non-aromatic: The non-planar cyclic conjugated molecules having
4n number of delocalized π electrons are said to be non-aromatic. For
example, cyclooctatetraene (number of delocalized π electrons = 8;
see section 9A.5.6).
B. BENZENE AND ITS DERIVATIVES
9B.1 Introduction
So far, we have discussed the aromaticity and stability of organic compounds.
One of the characteristic of aromatic compounds is to undergo electrophilic
substitution reactions. The present part includes the nomenclature of aromatic
compounds in general and also the general methods of preparation, and
characteristic reactions of benzene and its derivatives.
Petroleum and coal tar are the main sources of various aromatic hydrocarbons.
Aromatic hydrocarbons are obtained from petroleum by the process of reforming
(aromatization, refer
Chapter 5). Coal tar, a major source of aromatic hydrocarbons, is discussed here.

9B.1.1 Coal Tar: Source of Aromatic Hydrocarbons

Destructive distillation of coal at a high temperature of 1000oC results in the


formation of coal gas, ammonia, and coal tar. Coal tar is rich in aromatic
hydrocarbons and the products obtained on its fractional distillation are given in
Table 9B.1 as follows.
9B.2 NOMENCLATURE OF AROMATIC
COMPOUNDS
The IUPAC nomenclature of organic compounds has been discussed in Chapter
2. Keeping in view the general rules for naming the compounds, the
nomenclature of aromatic compounds is discussed here in brief. As mentioned
earlier, common names are still used for some organic compounds and are also
accepted by IUPAC. Aromatic hydrocarbons are grouped under a class called
arenes. Some representatives of arenes are

Monosubstituted benzenes are named as derivatives of benzene. Some of them


are

Examples of some fused ring aromatic compounds are


In disubstituted benzene three isomeric structures are possible which have
substitution at
1,2-; 1,3-; and 1,4- positions. They are designated by prefixes ortho- (for 1,2-),
meta- (for 1,3-), and para- (1,4-) and are abbreviated as o-, m- and p-
respectively. For example

If two different alkyl groups are present, they are numbered in alphabetical order
as in the following case.

If more than two substituents are present, their position is designated by


numbering the ring in such a way that all the substituents get the minimum
possible number.
When attached as a substituent, the benzene ring is named as phenyl. The
compounds in which benzene ring is attached to branched alkyl chain or
unsaturated alkyl groups, are named phenylalkanes, phenylalkenes, or
phenylalkynes.

The IUPAC names of some representative compounds, which are popularly


known by their common names, as mentioned above, are given in Table 9B.2.
The suffixes and prefixes for functional groups and substituents respectively, are
used in the way as discussed in Chapter 2, Table 2.5.
9B.3 PHYSICAL PROPERTIES OF AROMATIC
HYDROCARBONS
Benzene is a colourless liquid with boiling point 80oC. It is immiscible with
water. Due to high carbon content it burns with a sooty flame. It is used as a
solvent, however it exhibits carcinogenic properties. Toluene is a colourless
liquid with boiling point 110oC. It is also immiscible with water. It is widely
used as a solvent in industries. Styrene belongs to the class of alkenylbenzenes.
It is a high boiling liquid (146oC). The physical data (melting point, boiling
point, and density) of some aromatic hydrocarbons is given in Table 9B.3.
9B.4 PREPARATIONS AND CHEMICAL
PROPERTIES OF BENZENE
As mentioned in the beginning, the ‘light oil’ fraction which is obtained upto
170oC by distillation of coal tar contains benzene, toluene, and xylene. Benzene
is separated by further distillation of this fraction. The synthetic methods for the
preparation and the chemical properties of benzene are discussed as follows:

9B.4.1 Preparations
From sodium salt of benzoic acid
Sodium salt of benzoic acid on heating with soda-lime undergoes
decarboxylation to give benzene.

From benzensulfonic acid


Heating benzenesulfonic acids in aqueous acidic medium results in
desulfonation to give benzene.

From benzenediazonium salts


The benzenediazonium salts on treatment with hypophosphorous acid undergo
reduction to give benzene.

From phenol
Distilling phenol over zinc dust results in the formation of benzene.
By Grignard reagent
The Grignard reagent, phenylmagnesium bromide on hydrolysis results in the
formation of benzene.

9B.4.2 Chemical Properties


Electrophilic substitution reactions
Benzene is a π electron rich system and is resonance stabilized. It undergoes
electrophilic substitution reactions to give monosubstituted benzene. The
electrophilic substitution reactions are carried out in presence of catalysts that
help in generation of electrophiles (electron deficient species). Various
electrophilic substitution reactions are as follows:
Halogenation
Benzene reacts with chlorine in the presence of a catalyst like AlCl3 or FeCl3 to
produce chlorobenzene. The catalyst, a Lewis acid, helps in generation of
electrophile (Cl+). In a similar way, reaction of benzene with bromine in
presence of AlBr3 or FeBr3 results in the formation of bromobenzene.

Nitration
Benzene on reaction with fuming nitric acid (HNO3 + H2SO4) results in
formation of nitrobenzene. Sulfuric acid acts as a catalyst and helps in
generating the electrophile (NO2+).
Friedel–Crafts alkylation
Benzene reacts with haloalkanes in presence of a catalyst (AlCl3) to give
alkylbenzenes. The catalyst AlCl3, a Lewis acid helps in generation of
electrophile (alkyl carbocation). For example

Friedel–Crafts acylation
Benzene reacts with acid chloride in presence of aluminum chloride as catalyst
to give aryl ketone as the final product. The reaction can also be carried out with
acid anhydride.

Sulfonation
Benzene reacts with fuming sulfuric acid to give benzene sulfonic acid. The
electrophile in sulfonation reaction is sulfur trioxide. It is a reversible reaction
and desulfonation occurs readily in aqueous acidic medium.

Addition reactions
Addition of hydrogen
In presence of nickel catalyst at a temperature of 200oC, benzene undergoes
hydrogenation to produce cyclohexane. The Birch reduction of benzene gives
cyclohexa-1,4-diene.

Addition of chlorine
In the presence of sun light, the addition of chlorine to benzene gives
benzenehexachloride (BHC, Lindane, or gammaxene). BHC is extensively used
as an insecticide.

Oxidation reactions
With oxygen
Benzene, on heating with oxygen in presence of a catalyst vanadium pentaoxide
(V2O5) at 500oC results in the formation of maleic anhydride. In the process,
carbon dioxide and water are evolved.

With ozone
Benzene adds on three molecules of ozone to form a triozononide, which on
reductive cleavage results in the formation three molecules of glyoxal.
9B.5 ARENES: METHYLBENZENE (TOLUENE)
AND STYRENE

9B.5.1 Methylbenzene
Preparations
Wurtz–Fittig reaction
Reaction of chlorobenzene and chloromethane with sodium metal in presence of
dry ether as solvent results in the formation of methylbenzene (toluene).

From sodium salt of toluic acid


Heating sodium salts of o-, m-, or p- toluic acids with soda-lime results in their
decarboxylation and toluene is formed.

By distillation of cresol with zinc dust (reduction)


Heating o-, m-, or p- cresol over zinc dust results in the formation of toluene.
By Grignard reagent
The Grignard reagent (phenylmagnesium bromide) on reaction with
iodomethane results in the formation of methylbenzene.

Reactions involving benzene ring


Toluene undergoes electrophilic substitution reactions due to delocalized π
electron system. The methyl group behaves as an electron-releasing group
towards benzene ring. The release of electrons occurs through inductive effect
(+I) and also through hyperconjugation (details in Chapter 4). The electron
release effect is observed to be maximum at ortho and para positions. Thus,
electrophilic substitution occurs at ortho and para positions in case of toluene.
The directive effect of the substituents in benzene ring is discussed in detail in
Chapter 10.
Halogenation
Toluene reacts with chlorine in presence of catalysts such as AlCl3 or FeCl3 to
produce
o-chlorotoluene and p-chlorotoluene. The catalyst helps in generation of
electrophile (Cl+).
Similarly, reaction of toluene with bromine in presence of AlBr3 or FeBr3
results in the formation of o-bromo and p-bromotoluene.
Nitration
Toluene reacts with a mixture of concentrated nitric acid and sulfuric acid to
produce o- and
p-nitrotoluenes. Sulfuric acid acts as a catalyst that helps in generation of
electrophile (NO2+).

Sulfonation
The reaction of toluene with fuming sulfuric acid results in the formation of o-
and p- toluenesulfonic acids. The electrophile in sulfonation reaction is sulfur
trioxide.
Side chain reactions
Halogenation
Reaction of toluene with chlorine in presence of light (without catalyst) results in
substitution in the side chain (–CH3). The reaction is similar to substitution
reactions in alkanes. The product formed in the reaction is benzyl chloride that
undergoes further substitution to give benzal chloride and finally
trichloromethylbenzene. The reaction follows a free radical mechanism.

Oxidation
Toluene on oxidation with sodium dichromate in acidic medium or on oxidation
with potassium permanganate results in the formation of benzoic acid.

The isomeric dimethylbenzenes (xylenes) on oxidation with


Na2Cr2O7/H+ or with KMnO4 result in the formation of isomeric
phthalic acids. This reaction is generally used for identification of
isomeric xylenes.
The oxidation of alkylbenzenes with acidified dichromate or alkaline potassium
permanganate results in the formation of benzoic acid as the final product.
9B.5.2 Styrene
Preparations
From ethylbenzene
By dehydrogenation. Heating ethylbenzene in the presence of Al2O3/Cr2O3 at
500o–600oC results in dehydrogenation of ethyl side chain to produce styrene.

Side chain halogenation followed by dehydrohalogenation. Reaction of


ethylbenzene with chlorine in presence of light results in side chain chlorination
to produce 1-chloro-1-phenylethane which on heating with alcoholic KOH forms
styrene.
Chemical properties
Side chain reactions
Addition reactions. The unsaturated side chain in styrene undergoes addition
reactions similar to alkenes.

Oxidation. Oxidation of styrene with hydrogen peroxide and formic acid results
in hydroxylation of side chain to produce 1-phenylethane-1,2-diol. However,
strong oxidizing agents like KMnO4 results in the oxidation of styrene to
benzoic acid.
SELECTED SOLVED EXAMPLES
Example 1. Complete the following sequence of reactions.

Solution
(a) It is Friedel–Crafts alkylation using benzylchloride as the alkylating agent.
The product A is Diphenylmethane, i.e.

(b) In presence of catalyst, the electrophilic substitution reaction occurs in


toluene. The product A is o-chlorotoluene and B is p-chlorotoluene. In
presence of Na2Cr2O7/H+, side chain oxidation occurs. A is oxidized to C
which is o-chlorobenzoic acid and B is oxidized to D which is p-
chlorobenzoic acid.

Example 2. How will you convert:


(a) Toluene to benzene.
(b) Benzene to ethyl phenyl ketone.
(c) Benzene to toluene via Grignard Reagent.
Solution
(a) Hint: For steps involved in conversion, refer Example 4(a)
(b) Benzene on Friedel-Crafts acylation, using propanoyl chloride and AlCl3,
results in the formation of ethyl phenyl ketone.

Example 3. Carry out the following conversions.


(a) Toluene to phenylacetylene. (b) Benzene to p-bromobenzylbromide.
(c) Bromobenzene to 1-chloro-1-phenylethane. (d) 2-phenylethanol to
benzene.
Solution.
Example 4. Complete the following sequence of reactions.
Solution
(a) Side chain oxidation with Na2Cr2O7/H+ will give A, Benzoic acid. The
treatment of benzoic acid with soda-lime will cause decarboxylation to
produce B, which is benzene. The nitration of benzene results in the
formation of nitrobenzene C. The overall reaction is

(b) Friedel–Crafts acylation of benzene gives A, acetophenone. Zinc amalgam


reduces the carbonyl group C=O to –CH2. The product B is ethylbenzene.
In presence of light, side chain chlorination of ethylbenzene occurs where
C, i.e. 1-chloro-1-phenylethane is formed.
(c) Side chain chlorination of ethylbenzene occurs in the presence of light to
give A, 1-chloro-1-phenylethane which on treatment with alcoholic KOH
undergoes dehydrohalogenation to give styrene, B. Styrene undergoes
addition at the side chain in an anti-Markovnikov’s manner, in presence of
peroxide to yield C, 1-bromo-2-phenylethane.

(d) Styrene undergoes side chain hydrogenation to give A, ethylbenzene


which undergoes side chain oxidation in presence of Na2Cr2O7/H+ to give
B, benzoic acid. The same product B is obtained by side chain oxidation of
styrene in presence of potassium permanganate as oxidizing agent.
EXERCISES
1. What is Kekule structure of benzene? What are its limitations?
2. Discuss the molecular orbital structure of benzene.
3. What is aromaticity? What are the various characteristics which differentiate
aromatic compounds from aliphatic compounds?
4. Explain Huckel’s rule with examples.
5. What are annulenes?
6. Define aromatic, anti-aromatic, and non-aromatic compounds.
7. Giving suitable explanation, classify the following compounds as aromatic,
non-aromatic, and anti-aromatic.
(a) Cyclobutadiene
(b) Cyclopentadienyl cation
(c) Cyclopentadiene
(d) Cyclopropenyl anion
(e) Cycloheptatrienyl anion
(f) Tropylium ion
(g) Cyclooctatetraene
(h) Naphthalene
(i) Anthracene
(j) Phenanthrene
8. How does Huckel’s rule explain the aromatic character of heterocyclic
compounds? Explain with examples.
9. How is benzene obtained commercially?
10. How can benzene be prepared in laboratory?
11. Write down the reagents and products obtained in the following electrophilic
substitution reactions of benzene.
(a) Halogenation
(b) Nitration
(c) Friedel–Crafts alkylation
(d) Friedel–Crafts acylation
(e) Sulfonation
12. Describe the ozonolysis reaction of benzene.
13. Describe the addition reaction of benzene with hydrogen and chlorine.
14. How will you prepare ethylbenzene from benzene?
15. Compare the reaction of benzene and toluene with chlorine
(a) In presence of catalyst FeCl3
(b) In presence of light
16. Complete the following:

17. How will you prepare styrene from ethylbenzene?


18. What will be the product formed on reaction of styrene with hydrogen
bromide?
19. Compare the reactivity of styrene with alkenes.
20. How will you prepare ethylbenzene from styrene?
21. What will be the oxidation product of toluene and styrene in presence of
alkaline KMnO4 as an oxidizing agent?

Note: The mechanism and details of these reactions are discussed in Chapter 10.
Chapter10
Aromatic
Electrophilic Substitution Reactions
—Mechanism, Orientation and
Reactivity

10.1 INTRODUCTION
One of the most important and characteristic reactions of benzene as well as of
other aromatic systems are electrophilic substitution reactions. Aromatic systems
have delocalized π electrons and this availability of π electrons makes them
susceptible to the attack of electron deficient species called electrophiles.
Benzene is the representative member of the class of aromatic compounds. The
high resonance energy provides stability to benzene (refer Chapter 9). The
addition of electrophile causes a loss in aromaticity and hence, a loss in the
resonance energy of benzene as delocalization of electrons is discontinued. Thus,
benzene prefers to undergo electrophilic substitution reactions as during
electrophilic substitution the aromaticity is retained and so is the resonance
energy.

Fig. 10.1 Comparing the electrophilic addition and electrophilic substitution in benzene.
The present chapter deals with various aspects of electrophilic substitution
reactions of benzene and substituted benzenes. The mechanisms of the
substitution reactions and the impact of substituents on the orientation and
reactivity of substituted benzenes are discussed in detail under the heads as
tabulated in Information Chart I.
10.2 MECHANISM OF ELECTROPHILIC
SUBSTITUTION REACTIONS OF BENZENE
Benzene undergoes electrophilic substitution reactions such as halogenation,
nitration, sulfonation, alkylation, acylation, and so on in presence of a catalyst.
Substitution results in the formation of a monosubstituted benzene as the only
product.

10.2.1 Halogenation
Benzene reacts with chlorine or bromine in the presence of Lewis acids like
AlCl3, FeCl3, FeBr3 etc. as catalysts to give chlorobenzene or bromobenzene.
The catalyst initiates the reaction by generating the electrophile, that is,
chloronium (Cl+) or bromonium (Br+) ions.

Mechanism. The mechanism for chlorination of benzene in presence of a Lewis


acid as catalyst, is as follows:
Step 1. Generation of Chloronium ion (electrophile)
The Lewis acid forms a complex with chlorine which dissociates further to
generate chloronium ion. The Lewis acid enhances the electrophilic character in
chlorine.

However, it is also belived that from complex the chlorine may get transferred
directly to the benzene ring.
Step 2. Attack of chloronium ion on π-electron rich aromatic system (formation
of arenium ion)

Aromaticity is lost at this stage as one of the carbons becomes sp3 hybridized.
Formation of arenium ion is an endothermic process. The four π-electrons in
arenium ion are delocalized among five sp2 hybridized carbons.

Step 3. Loss of proton (formation of chlorobenzene and regeneration of catalyst)


The arenium ion in order to retain aromaticity, undergoes loss of proton from
sp3 hybridized carbon. This proton is captured by –AlCl4 (from Step 1) to
regenerate AlCl3 with simultaneous loss of HCl to produce chlorobenzene.

The formation of chlorobenzene from arenium ion is an exothermic process.


Resonance energy is regained in this process. The energy profile for various
electrophilic substitution reactions in benzene is depicted in Fig. 10.2.
Fig. 10.2 Energy profile for electrophilic substitution in benzene.

In a similar manner, as discussed above, bromination occurs in the presence of


either AlBr3 or FeBr3 as a catalyst, to form bromobenzene.

10.2.2 Nitration
Benzene, on treatment with fuming nitric acid (or mixture of conc. HNO3 and
H2SO4) results in the formation of nitrobenzene. Concentrated sulfuric acid acts
as a catalyst which helps in generation of electrophile, that is, nitronium ion.
The sulfuric acid protonates the nitric acid, which on loss of water molecule
gives nitronium ion, a strong electrophile.

Step 2. Attack of nitronium ion on benzene ring system (formation of arenium


ion)
The nitronium ion attacks the benzene ring system to form a carbocation
(arenium ion). The formation of arenium ion is an endothermic process. The
aromaticity is lost at this stage as one of the carbons become sp3 hybridized. The
four π-electrons are delocalized among five sp2 hybridized carbons of the
arenium ion.
Step 3. Loss of proton (formation of nitrobenzene and regeneration of catalyst)
The arenium ion, in order to regain the aromaticity, undergoes loss of proton
from sp3 hybridized carbon. The proton is captured by HSO–4 (Step 1) to
regenerate the catalyst (H2SO4) with simulataneous formation of nitrobenzene.
The formation of nitrobenzene from arenium ion is an exothermic process (Fig.
10.2).

NOTABILIA 9 π
10.2.3 Friedel–Crafts Alkylation
Haloalkanes react with benzene in presence of a Lewis acid, as catalyst (AlX3,
FeX3) to produce alkylbenzenes. The reactivity of haloalkanes increases with an
increase in the polarity of C–X bond. Thus, order of reactivity of RX is RF >
RCl > RBr > RI.

For example, reaction of chloromethane with benzene in presence of AlCl3 as


catalyst produces methylbenzene.

Mechanism
Step 1. Generation of electrophile (alkyl carbocation)
The Lewis acid enhances the electrophilic character on carbon of alkyl group in
the complex.
Continuous delocalization ceases as one of the carbon becomes sp3 hybridized,
this results in loss of aromaticity in arenium ion. The four π-electrons are
delocalized among five sp2 hybrid carbons of the ring.

Step 3. Loss of proton (formation of alkylbenzene and regeneration of catalyst)


In order to regain aromaticity of the ring system the arenium ion undergoes loss
of proton. The proton is captured by –AlCl4 which regenerates the catalyst with
simulatneous loss of HCl.

Use of Alkylating Agents Other than Haloalkanes in


Friedel–Crafts Alkylation
The Friedel–Crafts alkylation proceeds through the formation of an
alkyl cabocation and is not limited to haloalkanes only. Reagents,
which can produce cabocations can serve the purpose of alkylation.
Alkenes and alcohols can be used, in the presence of acidic
catalysts as alkylating agents. For example, ethyl benzene can be
prepared by the reaction of benzene with ethene in acidic medium
(H3PO4). Similarly, propene is used in acidic medium for
alkylation of benzene to produce cumene.

The other Lewis acids can also be used in Friedel–Crafts alkylation namely BF3,
HF, SnCl4 or acids (as in above examples).

Limitations of Friedel–Crafts alkylation


1. The Friedel–Crafts alkylation proceeds through the formation of a
carbocation. This carbocation may rearrange to form a more stable
carbocation and the alkylation product of benzene in that case will be
different from that expected. Reaction of 1-chloropropane with benzene
does not produce propylbenzene but gives isopropylbenzene as the final
product.
The carbocation formed in this reaction rearranges to form a more stable 2o
carbocation. The final product is formed from the reaction with more stable
carbocation.

2. Polyalkylation is a major limitation of Friedel–Crafts alkylation reaction.


The reaction involves the introduction of an alkyl group. The alkyl group is
electron-releasing in nature, which increases the electron density in benzene
ring and activates the ring towards further electrophilic substitution. As a
result, more than one alkyl groups are introduced in the ring which results
in the formation of di- or tri- alkylbenzenes.

3. Friedel–Crafts alkylation does not occur if electron-withdrawing groups are


present in the aromatic ring system.
An aromatic ring system bearing an electron-withdrawing group (–NO2, C=O,
–CHO, SO3H, –COOH etc.) or groups like –NH2, –NHR, and –NR2 do not
undergo Friedel–Crafts Alkylation. The electron-withdrawing groups
attached to benzene ring decrease the electron density of the ring system,
thereby making the attack of electrophile difficult.

Although –NH2, –NHR, and –NR2 are electron-releasing in nature, they form
a complex with AlCl3 (a Lewis acid). The development of positive charge
on nitrogen makes them behave as electron-withdrawing groups.

10.2.4 Friedel–Crafts Acylation


The reaction of benzene with acid halides or acid anhydride in the presence of a
Lewis acid catalyst like AlCl3 results in the introduction of an acyl group
(RCO–) in the benzene ring. The product of acylation reaction is an arylketone.
Acylation is carried out in the presence of a solvent such as carbondisulfide or
nitrobenzene.
Mechanism
Step 1. Generation of electophile (acylium ion)
The carbonyl oxygen of acid chloride forms complex with AlCl3. This complex
forms corresponding acylium ion that behaves as strong electrophile.

Step 2. Attack of acylium ion on benzene π-electron system (formation of


arenium ion)
The attack of acylium ion on benzene ring results in the formation of arenium
ion which is stabilized through delocalization.

Step 3. Loss of proton (regenaration of catalyst and formation of ketone)


In order to regain aromaticity, the arenium ion undergoes loss of proton that is
captured by –AlCl4 (Step 1) to form AlCl3 and simultaneous removal of HCl.
The formation of aryl ketone is an exothermic process.
10.2.5 Sulfonation
Benzene reacts with fuming sulfuric acid (sulfuric acid rich in sulfurtrioxide,
SO3 content) to produce benzenesulfonic acid. The electrophile in this case is
sulfurtrioxide, a neutral but electron deficient molecule.

The sulfurtrioxide behaves as an electrophile, as the presence of electronegative


oxygens withdraw the electron density away from sulfur atom. This results in
development of positive charge on sulfur making SO3 a powerful electrophile.
Mechanism
Step 1. Generation of sulfurtrioxide (Electrophile)
The sulfurtrioxide present in sulfonating reagent reacts directly with benzene.
Some of it may be formed by the following reaction.

However, this step is not so important as SO3 may react directly with benzene.
Step 2. Attack of sulfurtrioxide on benzene π-electron cloud (formation of
arenium ion)

Resonance stabilization of arenium ion (formation of σ–complex)

Step 3. Loss of proton (formation of benzenesulfonic acid)


Fig. 10.3 Energy profile for sulfonation of benzene.

Sulfonation is a reversible electrophilic substitution


reaction. Desulfo-nation takes place in aqueous acidic conditions.

The reversibility of this reaction is used frequently in organic


synthesis for protection of a particular position in a ring. This
reaction will be used a number of times in the problems related to
organic conversions, in the subsequent chapters.
10.3 ORIENTATION AND REACTIVITY IN
MONOSUBSTITUTED BENZENE: EFFECT OF
SUBSTITUENTS ON ELECTROPHILIC
SUBSTITUTION REACTIONS
In benzene, the six hydrogens are equivalent and electrophilic substitution
reaction results in the formation of only one monosubstituted product. However,
the situation is entirely different in case of substituted benzene. In presence of a
group attached to a benzene ring system, the remaining five hydrogens are no
longer equivalent and the group influences the reactivity of remaining
hydrogens. Electrophilic substitution on monosubstituted benzene results in the
formation of three possible disubstituted benzenes, that is, substitution of
electorphilie can take place at three different positions with respect to the
substituent ‘G’ already present on benzene ring.

A number of reactions have been studied by the chemists, and based on this a
generalization has been put forward for the effect of substituents on electrophilic
substitution reactions.
As an example, let us compare the nitration reaction of methylbenzene,
nitrobenzene, and benzene.
The nitration of methylbenzene takes place more readily compared to nitration
of benzene and nitro-group gets substituted preferably at ortho and para
positions. On the other hand, the nitration of nitrobenzene takes place less
readily compared to nitration of benzene and nitro- group prefers to be
substituted at meta position.

During any electrophilic substitution reaction, the group


already attached to benzene ring governs two important
factors, namely
(a) The rate of electrophilic substitution, i.e. Reactivity of the
aromatic system.
(b) At what position, the substitution of electrophile takes places
i.e. Orientation of the incoming electrophile.
A group that increases the reactivity of the ring towards
electrophilic substitution reaction as compared to benzene is known
as ring activator.
A group that decreases the reactivity of the ring towards
electrophilic substitution reaction as compared to benzene is known
as ring deactivator.
A group that directs the electrophile at positions ortho and para
to itself is known as an ortho, para director.
A group that directs the electrophile at a position meta to itself is
known as a meta director.

10.3.1 Nature of Groups


The group attached to benzene ring may be electron-releasing due to inductive
(+I) or resonance effect (+R) e.g. –OH, –NH2, –NHR, –NR2, –CH3, –OCH3,
Cl, Br, I, F.
Similarly, due to inductive (–I) or resonance effect (–R) a group may exhibit
electron-withdrawing nature e.g. –NO2, –CN, –CHO, C=O, –COOH, –COOR,
SO3H.
Based on the extensive studies in the field of electrophilic substitution
reactions, it has been found that:
(a) Electron releasing groups are ring activators and ortho, para directors.
(b) Electron withdrawing groups are ring deactivators and meta directors.
(c) Exceptions are halogens, which are weak ring deactivators but ortho, para
directors.

10.3.2 Effect of Group on the Reactivity


The relative reactivity of ring towards electrophilic substitution reaction depends
on the nature of the group attached to it. Let us consider the intermediate formed
during substitution in case of benzene and substituted benzene.

The group attached may be electron-releasing or electron-withdrawing in nature.


An electron-releasing group stabilizes the intermediate carbocation by
delocalizing (or neutralizing) the positive charge. More stable the carbocation,
faster is the reaction. On the other hand, an electron-withdrawing group takes
away the electron from an already electron deficient ring and thus, intensifies
(increases) the positive charge. This destabilizes the carbocation and thus, the
further reaction becomes slower.

Thus electron releasing groups are termed as ring activators and electron
withdrawing groups are termed as ring deactivators. A list of some activator and
deactivator groups is given in Table 10.1.
10.3.3 Effect of Groups on Orientation
Ortho, para directors
Electron-releasing groups activate the ring and direct the electrophile towards
ortho and para positions. The group may release the electrons through Inductive
or Resonance effect.
Let us consider the case of methylbenzene. The attack of electrophile may occur
at ortho, meta, and para positions. The most favourable site of attack is decided
and understood from the stability of the carbocation formed in each case.
In all the three cases, each carbocation is a hybrid of three contributing
structures. The carbocations formed by the attack at ortho and para positions are
relatively more stable, as one of the contributing structures in these cases is
associated with extra stability. In case of meta attack, no such extra stability is
provided to the carbocation. Thus, attack occurs more readily at ortho and para
positions.
The alkyl groups attached to benzene ring, release electrons through inductive
effect as well as through hyperconjugation (p. 113).
As an example, consider the chlorination of methylbenzene (toluene).

Mechanism
Step 1. Generation of chloronium ion (Electrophile)
Step 2. Attack of chloronium ion at ortho- and para- positions of toluene
(formation of arenium ion)
Ortho attack:

para attack:

Step 3. Loss of proton (formation of o- and p- chlorotoluene and regeneration of


catalyst)

The release of electrons can occur through resonance effect and this is
observed in case of atoms or groups where lone pair is present on the atom
through which group is attached to benzene ring.
Nitrogen, oxygen, and halogens are electronegative in nature and withdraw
electrons through inductive effect (–I effect), but presence of lone pair on these
atoms makes them participate in an extended delocalization with the π-electrons
of the benzene ring. Due to this strong resonance effect, they behave as electron-
releasing groups towards benzene ring.

Consider, the case of phenol. The electrophile may attack at ortho, meta and
para positions. The most favourable site of attack is decided from the stability of
the carbocation formed in each case.

In case of ortho and para attack the carbocations are hybrid of four contributing
structures, one of which is associated with extra stability. In case of meta attack,
the carbocation is a hybrid of only three structures. More the number of
contributing structures, more is the stability of carbocation and faster is the
reaction. Carbocations formed by ortho and para attack are more stabilized
compared to those formed by meta attack. Thus, in phenol substitution of
electrophile occurs preferably at ortho and para position.
To conclude, electron-releasing groups activate the ortho and para positions to
greater extent (i.e. electron-releasing effect is observed maximum at ortho and
para positions) as compared to meta and therefore, direct the electrophile
towards ortho and para positions. The –NH2, –NHR, –NR2, –OR, –OCOR, –
NHCOR, –O– and –X are electron releasing groups and behave in a similar
manner as shown in case of phenol.
Meta directors
Electron-withdrawing groups deactivate the ring and direct the electrophile
towards meta position. The group may withdraw electron through inductive or
resonance effect.
Let us consider the case of nitrobenzene. The attack may occur at ortho, meta
and para positions. The most favourable site of attack can be decided from
stability of carbocations formed by ortho, meta, and para attack as follows.

The carbocations formed by attack at o-, m- and p- positions have three


contributing structures each. The attack of electrophile at ortho and para
positions is not favoured as one of the contributing structures is highly unstable.
In comparison to this the carbocation formed as a result of attack of electophilie
at meta position is relatively stable as none of the contributing structures are
unstable. As a result the substitution of the electrophile is directed towards a
relatively stable meta position. Similarly the groups like –CN, –CF3, –CHO, –
CO, –COOH, –SO3H etc. are electron withdrawing in nature and are meta
directors.
Halogens are ortho, para directors but ring deactivators
Halogens show an unusual behaviour towards electrophilic substitution
reactions.
(a) Directive effect of halogens indicates an electron-releasing nature (ortho
and para director).
(b) Reactivity indicates an electron-withdrawing nature (ring deactivation).
In actual, halogens attached directly to a benzene ring exhibit dual nature.
Halogens show an electron-releasing nature due to resonance effect. The lone
pair of electrons present on halogen participates in delocalization with the π-
electrons of the benzene ring. This electron release, towards ring, through
resonance (+R effect), stabilizes the carbocations formed by ortho and para
attack. As a results the subtitution of electrophile is favoured at ortho and para
positions.
At the same time, halogens being most electronegative withdraw electrons
through inductive effect. This strong –I effect cannot be overruled and is
responsible for destabilization of carbocation and hence, deactivation of the ring.
The resonance effect, to some extent, compensates the deactivation at ortho and
para positions by opposing the strong –I effect so that ortho and para positions
experience a weak deactivation. Thus, the resonance effect governs the
orientation towards ortho and para positions, whereas inductive effect governs
the reactivity and weakly deactivates the ring.
10.4 ORIENTATION IN DISUBSTITUTED
BENZENE

Case study II (Both substituents oppose each other)


In case where the directive influence of two substituents opposes each other, an
activating group (electron releasing) decides the orientation of electrophile. That
is, an activating group decides the orientation over a deactivating group. For
example, in case of m-nitrophenol, the substitution of electrophile is directed at
positions ortho and para to –OH group.

Fig. 10.5 Attack of eletrophile on m-nitrophenol (–OH group directs the postion of entering group).

Case study III (Both substituents are activating or deactivating)


(a) In this case where both the substituents are activating, that is, both are
ortho, para directors, a strong activating group governs the orientation of
electrophile. This means that the electrophile will occupy a position ortho
and para with respect to strong activating group. For example, in case of p-
methylphenol, –OH group governs the orientation. Thus, electrophile enters
at a position ortho to –OH group.

Fig. 10.6 Attack of eletrophile on p-methylphenol (–OH group directs the position of entering group being
a strong ring activator over –CH3 group).

(b) In case both the substituents are deactivating (electron-withdrawing) in


nature, a strong deactivator decides the orientation of electrophile, that is,
the electrophile enters a position meta to strong deactivating group. For
example, in case of o-nitrobenzaldehyde and p-nitrobenzonitrile, the
electrophile enters meta to strong deactivating –NO2 group.

Fig. 10.7 Attack of eletrophile on o-nitrobenzaldehyde and p-nitrobenzonitrile (–NO2 group directs the
position of entering group).

Case study IV(a) (Both the substituents are either strong activating or weak
activating)
(a) When both the substituents are either strong activating or weak activating
then all the possible products are formed. This means that the products are
obtained with respect to the directive influence of each substituent.
Fig. 10.8 Attack of electrophile at ortho and para to both the substituents.

Case study IV(b) (Both the substituents are either strong deactivating or
weak deactivating)
(b) Similarly, when both the substituents are either strong deactivating or
weak deactivating then all possible products are formed. The products
obtained are formed by substitution of electrophile at positions that are meta
with respect to each group individually.

Fig. 10.9 Attack of electrophile at a position meta to both the substituents.

Case study V (Steric hindrance)


Due to steric hindrance, the electrophilic substitution is not favoured between
two groups which are positioned meta to each other.
Fig. 10.10 Attack of electrophile does not occur at hindered position in meta substituted benzene.
SELECTED SOLVED EXAMPLES
Example 1. Methylbenzene on nitration results in the formation of two products,
2-Nitromethylbenzene and 4-Nitromethylbenzene. What will be product
obtained on nitration of Trifluoromethylbenzene?
Solution. In methyl benzene, the methyl group is an electron-releasing group,
which releases electrons towards benzene ring through inductive and
hyperconjugation effect. The methyl group, thus, behaves as an activating and
ortho, para director group.
If we replace hydrogens of the methyl group by fluorine, we get
trifluromethylbenzene. The presence of three highly electronegative fluorine
atoms, makes –CF3 group a strong electron- withdrawing group which
withdraws electrons from the benzene ring. Due to this, –CF3 behaves as a meta
directing group as well as a ring deactivator towards electrophilic substitution
reactions.
Thus, trifluromethylbenzene on nitration gives only one product, i.e. m-
nitrotrifluoromethyl-benzene.

Example 2. Categorize the substituents in following compounds as ortho, para,


and meta directors, and complete the following reactions.
Solution. All the above reactions exhibit electrophilic substitution in
monosubstituted benzene. The groups –OCH3, –OH, –OCOCH3, and –
HCOCH3 are electron-releasing and thus, ortho & para directors whereas –
COCH3 and –COOCH3 are electron-withdrawing and thus, meta director. The
respective products of the reactions are

Example 3. What products will be obtained on reaction of benzene with (a)


dichloromethane (CH2Cl2) and (b) chloroform (CHCl3) in presence of AlCl3 as
a catalyst?
Solution. The reaction of benzene with CH2Cl2 and CHCl3 in presence of
AlCl3 is a Friedel–Crafts alkylation reaction, where CH2Cl2 and CHCl3 behave
initially as alkylating agents.
(a)

Now, benzylchloride acts as an alkylating agent and reacts with second molecule
of benzene to form diphenylmethane.

(b) In a similar manner, chloroform first reacts with benzene to give:

Now, benzal chloride behaves as an alkylating agent

After this reaction, diphenychloromethane acts as an alkylating agent and reacts


with third molecule of benzene to produce triphenylmethane.
The overall reactions of benzene with dichloromethane and chloroform may be
summarized as follows:

Example 4. Carry out the following conversions:


(a) Benzene to p-bromonitrobenzene
(b) Benzene to m-nitrobenzoic acid
(c) Benzene to m-bromobenzoic acid
Solution

Note: A carboxylic acid group is meta directing.


Example 5. Arrange the following compounds in increasing order of their
reactivity towards electrophilic substitution reactions.

Example 6. Which of the following compounds will readily undergo Friedel–


Crafts alkylation with CH3Cl and AlCl3? Explain. Also discuss the products
formed on reaction of following compounds with CH3Cl and AlCl3 (Friedel–
Crafts Alkylation).

Solution. Friedel–Crafts alkylation does not occur if electron-withdrawing


groups are attached to benzene ring (p. 336). The electron-withdrawing groups
deactivate the ring towards alkylation. Thus, compound I and V will not undergo
Friedel–Crafts alkylation as –NO2 and –CHO are electron-withdrawing groups.
Compound IV has electron-releasing – HCH3 group but it forms a complex
with catalyst AlCl3 (Lewis acid). Positive charge on nitrogen makes it behave
like a strong electron-withdrawing group and as a result, IV will not undergo the
reaction (p. 337)

Groups like methoxy (OCH3) and chloro (Cl) groups are electron-releasing and
thus compound III and VI will undergo the Friedel–Crafts alkylation. The
products formed are:
Example 8. In the following disubstituted benzenes, indicate the position(s)
where nitration reaction will occur. Also indicate the expected major product.
Solution. The possible positions where nitration will occur are indicated by
arrows. A thick arrow indicates the position where the substituent attaches to
form the major product.

Example 9. How many dibromonitrobenzenes will be obtained on nitration of


each of the isomeric dibromobenzene, that is,
(a) 1,2-dibromobenzene, (b) 1,3-dibromobenzene, and (c) 1,4-
dibromobenzene.
Write the structure & IUPAC name of the products obtained.
Solution. In the isomeric dibromobenzenes, two similar groups are attached to
ring and are ortho, para directors. A mixture of products will be obtained on
nitration, as both the bromines will direct the incoming nitro group.

Example 10. How will you convert


(a) Benzene to 2,5-dibromonitrobenzene.
(b) m-xylene to 4-nitroisophthalic acid.
(c) p-xylene to 2-chloroterphthalic acid.
(d) Benzene to 4-bromo-2-nitrobenzoic acid.
Solution
As similar activating groups are present, nitro group enters ortho and para with
respect to both methyl groups. The position ortho to one methyl group is para to
other, so only one product is obtained. Nitration will not occur in between two
meta substituted methyl groups because of steric hindrance. (see case study V, p.
352)

Both –COOH and –NO2 groups are deactivating in nature. The orientation of
electrophile (Br+) is governed by strong deactivating –NO2 group and thus,
bromine enters at a less sterically hindered meta position with respect to nitro
group.
EXERCISES
1. Give the detailed mechanism of electrophilic substitution reaction in benzene
by taking nitration and sulfonation as examples.
2. Write a note on the following reactions in benzene:
(a) Friedel–Crafts alkylation
(b) Friedel–Crafts acylation
(c) Halogenation
3. How does the nature of a substituent affect the reactivity of aromatic ring
system towards electrophilic substitution reactions? Explain on the basis of
inductive and resonance effects.
4. Explain the directive influence of amino and methyl groups in electrophilic
substitution reactions.
5. Halogens are electron-withdrawing in nature yet they direct the incoming
electrophile to ortho and para positions. Explain.
6. Explain the deactivating and meta directing nature of nitro group towards
electrophilic aromatic substitution.
7. What are σ– and π–complexes?
8. What products will be obtained on the nitration of methylbenzene? Explain
the underlying mechanism.
9. What will be the orientation of an entering electrophile in a disubstituted
benzene when
(a) one substituent is activating and the other is deactivating.
(b) one substituent is strong activating and the other is weak activating.
(c) both the substituents are strong deactivating.
(d) both the substituents are strong activating.
10. Discuss the limitations of Friedel–Crafts alkylation?
11. Arrange the following compounds in decreasing order of reactivity towards
bromination.

12. Indicate the position of the entering electrophile NO2+ in the following
compounds:
SET-II
Problem 32. Coupling of two methyl radicals result in formation of ethane.
However, coupling of two triphenylmethyl radicals does not form the expected
hexaphenylethane but results in the formation of another dimer known as
Gomberg dimer. Explain the formation of dimeric product.
Solution. The coupling of two triphenylmethyl radicals does not form
hexaphenylethane due to high steric strain imposed by presence of bulky phenyl
rings. The triphenylmethyl radical (I) undergoes delocalization to give
contributing structure –II. It is the free radical centre generated at carbon of one
of the phenyl ring in II, that undergoes coupling with another triphenylmethyl
radical (I) to produce “Gomberg dimer”.
The dimer looses aromatic stabilization from one of the phenyl ring but still its
formation is energetically more favourable compared to formation of highly
strained hexaphenylethane.
Thus, this is an example where aromaticity is sacrificed to overcome steric
strain to form the stable product “Gomberg dimer”.
Problem 34. What product (s) will be obtained from the Grignard reagents,
obtained from the isomeric monobromo derivatives of 2-methylbutane, if they
are hydrolyzed by (i) water and
(ii) heavy water, that is, D2O?
Solution. The possible four isomers of monobromo derivatives of 2-
methylbutane on treatment with Mg in dry ether will result in the formation of
corresponding Grignard reagent.
These Grignard reagents on hydrolysis with water will result in the formation
of same alkane that is 2-methylbutane (refer for details Sec. 5A.2.2).
But these four Grignard reagents will lead to the formation of different
products on hydrolysis with deuterated water, that is, isomeric deuterated
2-methylbutanes. The reactions are as follows:

Problem 35. How will you carry out the synthesis of (2H1) cyclohexane (that is,
deuterated cyclohexane)?
Solution. Synthesis of deuterated cyclohexane can be carried out by reaction of
Grignard reagent with deuterated water. The overall synthesis starting from
cyclohexene is as follows:

Problem 36. The chlorination of methane is favoured through pathway (a) and
not (b) as given. Explain this proposition by making use of the bond dissociation
energy (DE) values from
Table 5.2 (p. 184).
Solution. The favourable step for chlorination (in general, halogenation) can be
determined by calculating the values of ΔH (heat of reaction) as:
ΔH = (dissociation energy (DE) of bonds broken) – (dissociation energy (DE) of
bonds formed)
In Pathway (a)
The C–H bond breaks (DE = 435 kJmol–1) and H–Cl bond is formed (DE = 431
kJmol–1)
Thus, ΔH = 435 – 431 = +4 kJmol–1
In Pathway (b)
The C–H bond breaks (DE = 435 kJmol–1) and H3C–Cl bond is formed (DE =
352 kJmol–1)
Thus, ΔH = 435 – 352 = +83 kJmol–1.
Since, pathway (b) is highly endothermic (+83 kJmol–1) it is not favoured over
pathway (a).
Problem 37. Determine which of the following reactions can occur during the
bromination of ethane?

Solution. The occurrence of a reaction can be determined from its ΔH value (as
in the previous problem). For bond dissociation energy values refer to Table 5.2
(p. 184).
ΔH for (A) = 368 kJmol–1 – 293 kJmol–1 = +75 kJmol–1
ΔH for reaction (B) = 410 kJmol–1 – 368 kJmol–1 = +42 kJmol–1
This shows that cleavage of C–C bond as in reaction (A), is relatively more
endothermic and therefore does not occur.
Problem 38. The reaction of 2-methylbutane (iso-pentane) and chlorine in
presence of light gives a mixture of monochloro derivative constituting 50% of
1°, 28% of 2°, and 22% of 3° chloro derivatives. What is the relative reactivity
of primary, secondary, and tertiary C–H bonds? What inference can be drawn
from these values?
Solution. In 2-methylbutane there are nine primary, two secondary, and one
tertiary C–H bonds.

The relative reactivity of different type of C–H bonds is calculated as follows:

The relative reactivity of C–H bonds is therefore given by 1°: 2°: 3° = 5.5:14:22
It clearly indicates that tertiary hydrogen can be substituted much more readily
as compared to secondary and primary hydrogens.
Problem 39. Calculate the percentage of isomers formed during the
monobromination of 2,3-dimethylbutane. The relative reactivity values for
tertiary, secondary, and primary hydrogens are 1600, 82, and 1 respectively.
Solution. In 2,3-dimethylbutane there are twelve primary and two tertiary
hydrogens.

The percentage of isomeric monobromo products is evaluated as follows:


This table shows that the tertiary monobromo isomer will be formed in highest
percentage (major product).
Problem 40. Explain, why in Saytzeff elimination the removal of 3° hydrogen
occurs favorably over 2° hydrogen? Take the following example for your
explanation.

Solution. In the Saytzeff elimination the formation of a substituted alkene takes


place (p. 214). In a carbocation the removal of hydrogen takes place from the
carbon adjacent to the carbon carrying positive charge. In case of present
example the hydrogen removal can occur from the adjacent carbons on either
side. It can be explained by taking into consideration the steric strain present on
adjacent carbons on either side of the carbocation.
The overall situation can be depicted as follows:
Thus, removal of 3° hydrogen reduces the steric strain to a greater extent than 2°.
Hence the formation of more substituted alkene is favoured (also refer p. 221).
Problem 41. Give the structures for cis- and trans- isomers of pent-2-ene.
Giving reasons, assign the heat of hydrogenation values of 115.6 kJ/mol and
119.7 kJ/mol to corresponding isomers.
Solution. On hydrogenation isomeric pent-2-ene (both cis- and trans-) yields
pentane. In
trans- isomer, bulkier groups are on the opposite side of double bond and have
minimum steric hindrance and van der Waals repulsions. This makes the trans-
isomer more stable (lower energy) compared to cis- isomers (higher energy).
Hence, heat of hydrogenation of trans- isomer (115.6 kJ/mol) should be less than
cis- isomer (119.7 kJ/mol).
Problem 42. Giving suitable reasons, explain whether the addition of bromine
will be faster in case it is added to ethene or 2,3-dimethylbut-2-ene.
Solution. The addition of electrophile is the rate-determining step. More the
number of electron releasing groups present at the double bond, more is the
availability of electrons and the faster is the attack of electrophile (despite the
steric hindrances).
Thus, the addition of bromine will be faster in case of alkene with more alkyl
substitutions, which in this case is 2,3-dimethylbut-2-ene.

Problem 43. The addition of bromine to isomeric but-2-ene (that is cis- and
trans-) results in the formation of different stereoisomers of 2,3-dibromobutane.
Explain.
Solution. But-2-ene exhibits geometrical (cis- and trans-) isomerism. The
addition of bromine to alkenes is a stereospecific reaction (see section 6.3.2, p.
223) and occurs in a trans- manner. The trans- addition of bromine to cis-but-2-
ene results in the formation of racemic mixture of 2,3-dibromobutane whereas
trans- addition of bromine to trans-but-2-ene results in the formation of meso-
2,3-dibromobutane.
The trans-addition of bromine to cis-but-2-ene can be represented using
sawhorse projections as follows:

Similarly in trans-but-2-ene, the addition of bromine is represented as follows:

Problem 44. Which isomer of stilbene on addition of bromine in a trans-


manner will result in the formation of threo and erythro dibromo addition
products respectively?
Solution. As explained in the previous problem (Problem 43), the cis-isomer of
alkene on trans-addition of bromine results in the formation of enantiomeric
dibromo- addition products (racemic mixture) whereas a trans-isomer of alkene
on trans-addition of bromine results in the formation of meso-dibromo addition
product. An erythro isomer has identical groups on the same side of the carbon
chain in Fischer projection whereas a threo isomer has identical groups on
opposite side of the carbon chain (See p. 72).
Thus, cis-stilbene will result in the formation of threo addition product
(enantiomeric addition products have identical groups on opposite side) and
trans-stilbene will result in the formation of erythro addition product (meso
addition product).

Problem 45. Stating the reasons, name the products formed when bromine reacts
with ethene in the presence of lithium chloride in methanol?
Solution. When bromine is added to ethene in the presence of LiCl in CH3OH,
the reaction first involves the attack of electrophile, Br+ (refer sec. 6.3.2, p. 224).
The next step is the attack of the nucleophile and in this case besides Br–, the
other nucleophiles are also available namely Cl– (from LiCl) and –OCH3 (from
CH3OH). This results in the formation of three products, namely 1,2-
dibromoethane, 1-bromo-2-chloroethane, and 1-bromo-2-methoxyethane.

Problem 46. What will be the decreasing order of reactivity of the following
alkenes, towards the addition of HBr?
(a) CH2=CH2
(b) CH3CH2CH=CHCH3
(c) CH3C(CH3)=C(CH3)CH3
(d) CH3CH2CH=CH2
Solution. As mentioned earlier (Problem 42), alkenes with more number of alkyl
substituents undergo electrophilic addition readily as compared to less
substituted alkenes. Thus, the decreasing order of reactivity of the given alkenes
towards the addition of HBr is:
(c) > (b) > (d) > (a)
Problem 47. In the addition of hydrogen halides to the alkenes, the ease of
addition to the double bond follows the order HI > HBr > HCl. Explain.
Solution. The addition of electrophile (H+) is the rate-determining step in the
electrophilic addition reactions. The higher the acidic strength of hydrogen
halide, the faster is the addition of H+.
A hydrogen halide, which is more acidic in nature, furnishes H+ (electrophile)
more easily to the alkenes and the addition reaction is therefore quicker.
Thus, the ease of addition is directly related to the acidic strength of hydrogen
halides thereby explaining the order of reaction ease:
HI > HBr > HCl
Problem 48. Keeping in view the electrophilic addition mechanism, give the
product obtained by the addition of iodine chloride (ICl) to propene.
Solution. Since chlorine is more electronegative than iodine, iodine behaves as
an electrophile and addition occurs in accordance with Markovnikov’s rule to
give 2-chloro-1-iodopropane as the product.

Problem 49. What product will be formed in the following reaction?

Solution. The electrophilic addition may result in the formation of primary


carbocation (Pathway-I) or secondary carbocation (Pathway-II). The product
formation will occur through more stable secondary carbocation as depicted in
Pathway-II.
Pathway-I

Pathway-II

Problem 50. In each of the given set of carbocations (a) and (b) indicate which
one is more stable, along with explanation.

Solution.
(a) Carbocation I is more stable than II.
Carbocation I (Cyclobutylmethylium) is a primary carbocation but it is
more stable than carbocation II (1-methylcyclobutylium) which is tertiary
carbocation.
Carbocations are sp2 hybridized with bond angle 120°. The four membered
cyclic system have bond angle of 90° and high angle strain does not favour
the formation of
1-methylcyclobutylium [carbocation II].
(b) Carbocation IV is more stable than III.
The carbocation III (cyclohexylmethylium) is a primary carbocation while
carbocation IV (1-methylcyclohexylium) is tertiary in nature. The
cyclohexane ring systems do not exhibit significant angle strain. Thus
Carbocation IV, a tertiary carbocation, is more stable than III, a primary
carbocation.


Problem 51. What product(s) will be formed on addition of HCl to
(i) methylenecyclobutane and
(ii) methylenecyclohexane
Solution. Addition of HCl to alkene is an electrophilic addition and follows
Markovnikov’s rule.
(Recall: Addition proceeds via the formation of more stable carbocation and
products are formed accordingly, refer pp. 226–28).
(i) Although tertiary carbocations are more stable than primary but in this case
tertiary is unstable due to angle strain (see explanation in previous problem
no. 50).
(ii) Six membered ring systems are free from angle strain, so in this case
tertiary carbocation is formed readily(see explanation in previous problem
no. 50).

Problem 52. What products will be formed in the following sequence of


reactions?

Solution. The sequence of reactions is as follows:


Step 1 The bromocyclohexane on reaction with Mg/ether will result in the
formation of corresponding Grignard reagent.
Step 2 The 2,3-dibromoprop-1-ene has both allylic and vinylic bromine. The
allyl halides are more reactive than vinyl halides (section 11.5.2). Thus,
reaction of Grignard reagent occurs with more reactive allylic bromine.

Step 3 The product formed in step-2 reacts with sodamide to form corresponding
alkyne (refer
pp. 275, 276)

Problem 53. The bromination of 2-methylpent-2-ene with N-bromosuccinimide


results in the formation of 4-bromo-2-methylpent-2-ene as the major product.
Explain the formation of product.
Solution. N-bromosuccinimide (NBS) is a special reagent for allylic substitution
(section 6.3.5) and follows a free radical substitution mechanism. 2-Methylpent-
2-ene has two allylic carbons C1 and C4, which can undergo substitution.
Substitution will occur at 2o allylic position (C4), as it will result in the
formation of more stable 2° allylic free radical.

Problem 54. Name the products obtained on selective addition of one mole of
bromine to the following alkadienes:
Solution. The given alkadienes are the example of isolated dienes (that is,
double bonds are not conjugated). One mole of bromine can attack either of the
double bonds, however it attacks selectively at the double bond, which is
substituted with more number of alkyl groups (also refer Problem 42).

More the number of electron releasing groups (+I effect) attached to C=C, much
easier is the addition of electrophile (in this case, Br+). Thus, electrophilic
addition of bromine occurs preferably at the double bond which is substituted
with more number of alkyl groups.
Problem 55. Giving the mechanism, explain the formation of product(s) when:
(i) Cyclopenta-1,3-diene is treated with 1 mole of HBr.
(ii) Cyclopenta-1,3-diene is treated with 1 mole of Br2.
(iii) 1,4-diphenylbuta-1,3-diene is treated with 1 mole of Br2.
Solution. In all the above three cases (i-iii), the reactants are conjugated dienes,
which on reaction with one mole of reagent undergo electrophilic addition to
give 1,2- and 1,4- addition products (refer section 7.4.1). In general formation of
1,4-addition product is favoured due to thermodynamic stability.
(i) Cyclopenta-1,3-diene is treated with 1 mole of HBr. In this case, 1,2- and
1,4- addition of HBr to cyclopenta-1,3-diene results in the formation of
same product. Thus, Cyclopenta-1,3-diene with 1 mole of HBr gives only
one product, that is, 3-bromocyclopentene.
(ii) Cyclopenta-1,3-diene is treated with 1 mole of Br2. The reaction of
cyclopenta-1,3-diene with 1 mole of Br2 gives two products, that is,
3,4-dibromocyclopentene (1,2- addition product) and
3,5-dibromocyclopentene (1,4- addition product)

(iii) 1,4-diphenylbuta-1,3-diene is treated with 1 mole of Br2. The reaction of


1,4-diphenylbuta-1,3-diene with 1 mole of Br2 gives 1,2-addition product
as the only product, that is, 3,4-dibromo-1,4-diphenylbut-1-ene (structure
-I). 1,2-addition product exhibits conjugation and thus its formation is
favoured over 1,4-addition product, that is, 1,4-dibromo-1,4-diphenylbut-
2-ene (structure -II), which does not exhibit conjugation.
Problem 56. Name the reaction and the product obtained, when buta-1,3-diene
reacts with tetracyanoethene.
Solution. The reaction of conjugated diene (4π-system) with dienophile (2π-
system) to form a cyclic adduct is known as Diels–Alder reaction. In the
presence of electron withdrawing group like –CN in dienophile, Diels–Alder
reaction occurs more readily. The reaction of buta-1,3-diene with
tetracyanoethene is as follows:

Problem 57. Write the S-cis conformation of cis,cis-hexa-2,4-diene and


trans,trans-hexa-2,4-diene. Which one of these two geometrical isomers will
undergo Diels–Alder reaction easily?
Solution. The S-cis conformation of cis,cis- hexa-2,4-diene and trans,trans-
hexa-2,4-diene are as follows:
The Diels–Alder reaction is highly stereospecific and the dienes, in Diels–Alder
reaction, react in S-cis-conformation with the dienophile.
In the S-cis conformation of cis,cis- isomer, the cis- alkyl groups block the diene
from approaching due to steric hindrance. The trans,trans-hexa-2,4-diene in the
S-cis conformation easily undergoes the Diels–Alder reaction.
Problem 58. In the following reactions identify products/reactants (A to F):

Solution. The products/reactants (A to F) are given along with comments,


wherever required.
Problem 59. Why 2,3-dibromobutane on treatment with NaNH2 forms but-2-
yne and not buta-1,3-diene?
Solution. The presence of electronegative bromine on C2 and C3 makes the
hydrogen attached to C2 and C3 acidic in nature. In presence of a base these
hydrogens are eliminated preferentially. However, for the formation of buta-1,3-
diene the hydrogens are to be eliminated from C1 and C4. These hydrogens are
less acidic compared to hydrogens at C2 and C3. The removal of hydrogens at
C2 and C3 by base will occur at relatively fast rate and thus formation of but-2-
yne will be favoured over buta-1,3-diene.
Problem 60. Which product will be obtained in the following reaction? Give a
suitable explanation for its formation.

Solution. On addition of one mole of bromine to pent-1-en-4-yne (allyl


acetylene), the product obtained will be BrCH2CHBrCH2C≡CH.
This can be explained on the basis of fact that the sp hybridized carbon is much
more electronegative than sp2 hybridized carbon, and π-electrons of C≡C bond
are firmly held by the carbon than the π-electrons of C=C bond. Thus, C≡C bond
is less susceptible to the attack by electrophilic reagents than the double bond
(section 8.3.2).

Problem 61. Indicate which of the stereoisomers (cis- or trans-) will be obtained
as products in the following addition reactions:

Solution.
(a) The catalytic hydrogenation is a stereospecific reaction where addition of
hydrogen occurs in a syn manner to give cis-alkene (section 6.2.1).
(b) The addition of bromine to an unsaturated system is a stereospecific
reaction. The reaction follows an electrophilic addition mechanism where
addition of bromine occurs in an anti manner to yield a trans-
dibromoalkene.
Problem 62. What product(s) will be obtained by the reaction of (i) H2O/Hg2+,
H2SO4 and (ii) HCl (one mole) with the following alkynes?
(a) Hex-2-yne (b) Hex-3-yne
Solution. The electrophilic addition to alkyne proceeds via the formation of vinyl
cation as intermediate (section 8.3.2).
The acid catalyzed hydration of alkyne is catalyzed by Hg2+ and results in
the formation of carbonyl compound.
Further the addition of one mole of HCl to alkyne results in the formation of
corresponding vinyl chloride derivatives.
In the given examples, these reactions can be explained as follows:
(a) Hex-2-yne is an unsymmetrical alkyne and there is a possibility of
formation of two types of carbocations (I and II) during the electrophilic
addition reactions.

These carbocations may lead to the formation of different products. Thus, in the
given set of reactions
(i) The hydration of hex-2-yne will result in the formation of two types of
carbonyl compounds
(ii) Addition of HCl will also result in the formation of two types of vinyl
chloride derivatives.

(b) Hex-3-yne is a symmetrical alkyne and it will lead to the formation of only
one type of carbocation (I) during the electrophilic addition reactions. Thus
electrophilic addition to symmetrical alkynes will result in the formation of
one product only.
Thus, reactions of hex-3-yne with (i) H2O/Hg2+, H2SO4 and (ii) HCl (one
mole) are as follows:

Similarly,

Problem 63. The acidic strength of H–C≡N is more compared to H–C≡C–H


(acetylene). Explain.
Solution. The acidic strength (p. 284) is related to the nature of hybridized
carbon. Both
H–C≡C–H and H–C≡N have sp hybridized carbons. Still H–C≡N is more acidic
as –C≡N is an electron withdrawing group which further polarizes the H–C bond
and makes the removal of H+ easy.

Problem 64. What product(s) will be obtained by the reaction of 1-


Phenylpropyne with (a) H2O/Hg2+, H2SO4 and (b) HCl (one mole)?
Solution. 1-Phenylpropyne is an unsymmetrical alkyne and there is a possibility
of formation of two types of carbocations (I) and (II) during the electrophilic
addition reactions. But electrophilic addition reactions of 1-phenylpropyne lead
to the formation of one product predominantly. The formation of carbocation-II
occurs preferably as it is stabilized through resonance and thus, results in the
formation of product.
Also, the phenyl group facilitates the attack of nucleophile at C1. The C1 is
attached to sp2 hybridized carbon of the benzene ring whereas C2 is attached to
sp3 carbon of –CH3 group. The sp2 hybridized carbon is more electronegative
compared to sp3 hybridized carbon and thus, attack of nucleophile is preferred at
C1.
Reaction of 1-Phenylpropyne with H2O/Hg2+, H2SO4
Reaction of 1-Phenylpropyne with HCl (one mole)
Similarly, reaction of 1-Phenylpropyne with HCl (one mole) results in the
formation of one product through the resonance-stabilized carbocation-II.

Problem 65. The following species (A-F) as such are not aromatic. Identify
which of these species
(a) Have contributing structures (resonance contributor) that exhibit aromatic
character.
(b) Have tautomers that exhibit aromatic character.

Solution. All the species A-F as such are not aromatic as they do not follow
Huckel’s rule of
(4n + 2) delocalized π-electrons in a cyclic conjugated system.
(a) Contributing structures (resonance contributor) that exhibit aromatic
character:
The species D, E and F are delocalized (conjugated) systems but this extended
conjugation is not a part of cyclic system. These species undergo resonance
and in their contributing structures the electrons get delocalized in the
cyclic system and satisfy the condition of (4n + 2) π-electrons (Huckel’s
rule). Thus contributing structures of D, E and F, exhibit aromatic
character, as shown below:

(b) Tautomers that exhibit aromatic character: The species A, B and C are
delocalized (conjugated) systems but this extended conjugation is not a part
of cyclic system. These species exhibit tautomerism and their tautomeric
forms are highly stable aromatic species, as they follow all the criteria of
aromaticity, as shown below:

Problem 66. In electrophilic substitution reaction of benzene


(a) With Br2/AlCl3 formation of bromobenzene, and not chlorobenzene, takes
place. Explain.
(b) What will be the impact on the rate of nitration if one uses:
(i) Concentrated HNO3 alone
(ii) Concentrated HNO3 in presence of H2SO4
(iii) Concentrated HNO3 in presence of KNO3
Solution.
(a) In the reaction AlCl3 (Lewis acid) acts as catalyst that helps in generation
of electrophile:

For the formation of chlorobenzene, chloronium ion (Cl+) is required as


electrophile. But Cl+ is not formed in reaction
Thus, electrophilic substitution reaction of benzene with Br2/AlCl3 results in
the formation of bromobenzene and not chlorobenzene.

(b) The nitrating agent in nitration reaction is nitronium ion (+NO2) (refer
section 10.2.2).
(i) The nitric acid alone produces a low concentration of nitronium ion as nitric
acid is ionized to a very little extent (~ 4%) and equilibrium lies to left:

(ii) In presence of H2SO4, the ionization of nitric acid is complete and it is


converted to nitronium ion completely as shown:

Thus, in the presence of sulfuric acid, the concentration of nitronium ion


increases which enhances the rate of nitration.
(iii) The addition of KNO3 increases the concentration of NO3¯ ions and the
reaction equilibrium shifts towards left. This causes decrease in the
concentration of nitronium ion and thus, the rate of nitration decreases.

Problem 67. What will be formed by the reaction of benzene with


(a) chloropropane in presence of AlCl3.
(b) propene in acidic medium.
(c) cyclopropane in presence of AlCl3.
Compare and comment on the products formed during these reactions.
Solution. The reactions of benzene with (a), (b), and (c) are the examples of
electrophilic substitution reactions. The steps involved in the reactions are as
follows:
Step 1. Generation of electrophile

Propyl carbocation (1o) rearranges to stable isopropyl cation (2o)

Step 2. Attack of electrophile on benzene: formation of alkylbenzenes

The product formed in the above reactions clearly indicates that chloropropane
cannot be used for preparation of propylbenzene as carbocation rearrangement
occurs and isopropylbenzene is formed as the product.

However, cyclopropane can be used to prepare propylbenzene, where


rearrangement of carbocation does not occur.
[Note: An alternative method for preparation of propylbenzene using Friedel–
Crafts acylation has already been discussed (refer Chapter 10, p. 339) and also
take hint from next problem.]
Problem 68. Carry out the following conversions:
(a) Benzene to diphenylmethane
(b) Benzene to 1-ethyl-4-methylbenzene
Solution. Both the conversions are carried out in two stages. The first being
Friedel–Crafts acylation (or benzoylation) followed by Clemmenson reduction.
The reactions are as follows:

Problem 69. In the electrophilic substitution reactions of following aromatic


compounds indicate the mono nitro- product (s) formed.

Solution.
(a) In biphenyl, the phenyl group behaves as electron releasing group due to
resonance. Thus phenyl group activates the ortho and para positions of
other phenyl ring towards electrophilic aromatic substitution

(b), (c) and (d)


In following cases (b, c and d) the activating group (ring activator) will direct
the orientation of the electrophilic substitution. Thus phenyl ring attached to
activating group will undergo nitration. Phenyl ring attached to electron
withdrawing group (deactivating group) will not undergo nitration. The
substitution will occur preferably at para position with respect to activating
group. The substitution does not occur at ortho position due to steric
hindrance.

Problem 70. Explain, in brief, Birch reduction along with its important aspects.
Solution. Birch reduction: The reduction of aromatic rings by sodium (or Li or
K) in liquid ammonia, in presence of alcohol, is known as Birch reduction. The
1,4-addition of hydrogen in aromatic ring result in the formation of non-
conjugated cyclohexa-1,4-diene derivatives. This is an example of dissolving
metal reduction.

Mechanism:
The alkali metal Na (or Li or K) provide an electron to system whereas
alcohol provides proton (H+) during reduction.
Ammonia is less acidic compared to alcohol and thus cannot provide proton.

Functional groups affected by Birch reduction:


• Aromatic rings undergo Birch reduction
• Selective reduction, of non-terminal alkynes to alkenes, occur by
Birch reduction (refer section 6.2.1)
• Alkenes are not affected by Birch reduction.
• Conjugated olefins and phenylated olefins undergo Birch
reduction for example;

Birch reduction in substituted aromatic ring systems:


• The presence of electron releasing group on ring decreases the rate
of Birch reduction.
• The ring carbon carrying electron releasing group does not
undergo reduction.
• The presence of electron withdrawing group on ring increases the
rate of Birch reduction.
• The ring carbon carrying electron withdrawing group undergoes
reduction.
Problem 71. Complete the following reactions involving Birch reduction and
comment on the formation of products.

Solution.
Above two reactions are the examples of Birch reduction.
In reaction (a) electron withdrawing group is attached to phenyl ring and in
reaction (b) the electron releasing group is attached to phenyl ring. Nature of
groups largely effects the reduction products as they stabilize/destabilize the
radical anion formed during the course of reaction.
The reactions are as follows:
(a) The nitro group is electron withdrawing in nature and Birch reduction
results in the formation of product as explained below:
(b) The methoxy group, in methoxybenzene, is electron releasing in nature
and Birch reduction results in the formation of product as explained below:
Chapter11
Haloalkanes

11.1 INTRODUCTION
Haloalkanes are the halogen derivatives of hydrocarbons with general formula
CnH2n+1X and are represented as R–X where R is an alkyl group and X
represents halogen. In haloalkanes the carbon–halogen bond is formed by sp3-p
overlap. Being more electronegative compared to carbon, halogens impart a
polar character to C–X bond where halogens acquire a negative charge and
carbon acquires a positive charge.

Depending upon the number of carbons attached to the carbon to which halogen
is attached, haloalkanes may be classified as primary (1°), secondary (2°), or
tertiary (3°).

The IUPAC nomenclature of haloalkanes is similar to alkanes and in doing so


halogen is treated as a substituent. A few examples (for details see Chapter 2) of
haloalkane nomenclature are as follows:
11.1.1 Physical Properties
(a) Bond strength: In haloalkanes bond strength of carbon–halogen bond
decreases with an increase in bond length, as one moves from fluorine to
iodine. This is attributed to the size of p orbital, which increases from
fluorine to iodine and thus makes the sp3-p overlap less effective.

(b) Solubility and state of matter: Haloalkanes are insoluble in water but
soluble in organic solvents. Lower molecular mass haloalkanes are gases at
room temperature while higher molecular mass haloalkanes are liquid at
room temperature.
(c) Boiling point: Boiling point of haloalkanes is higher compared to
corresponding alkanes due to dipole–dipole interaction. With an increase in
molecular mass, there is an increase in boiling point. For the same alkyl
group the boiling point increases from fluoroalkane (R–F) to iodoalkane
(R–I). Iodine has a larger surface area and outer electrons are loosely
bound. This makes iodine a highly polarizable atom. A polarizable atom has
increased London forces of attraction (section 1.7), which causes an
increase in boiling point. The branched chain haloalkanes follow a similar
increasing order of boiling points.
(d) Density: The densities of haloalkanes increase with atomic mass of the
halogen and decrease with increasing size of the alkyl group. In
monohaloalkanes, iodomethane (CH3I) has the maximum density.
The general characteristics of monohaloalkanes can be enumerated as follows:
(a) Fluoro and chloroalkanes are less dense than water whereas bromo and
iodoalkanes are denser than water.
(b) Monofluroalkanes are unstable and on heating, H–F is eliminated to
produce alkene.
(c) Bromo and iodoalkanes are generally photosensitive and are stored in
brown opaque bottles. Otherwise, they liberate free bromine and iodine
respectively.
11.2 PREPARATION OF HALOALKANES
The various methods used for the preparation of haloalkanes are discussed in this
section. Some of the methods for the preparation of haloalkanes are already
discussed in details in other chapters as the chemical properties of other
functional groups from which these are derived. Such reactions are indicated at
appropriate places in the present text.
By halogenation of alkanes
Direct Halogenation of alkanes in presence of light (photohalogenation), heat
(thermal halogenation), or catalyst (catalytic halogenation) results in formation
of haloalkanes. The reaction proceeds through a free radical substitution
mechanism. For example,

Note:
(i) Order of reactivity of halogens is F > Cl > Br > I
(ii) Ease of abstraction of hydrogen from alkanes by halogens is 3° > 2° > 1°
By addition of hydrogen halides to alkenes
The electrophilic addition of hydrogen halides to alkenes (Section 6.3.2; p. 225)
results in the formation of haloalkanes. The reaction is regioselective and follows
Markovnikov’s rule.
The addition of HBr in presence of a peroxide results in the formation of 2°
haloalkane (Kharasch effect). A free radical mechanism is followed in presence
of peroxide. For example,
From alcohols
(i) By reaction with HCl or HBr
Reaction of alcohol with HCl in presence of anhydrous ZnCl2 results in the
formation of a chloroalkane. The reagent, HCl combined with anhydrous ZnCl2,
is known as Lucas Reagent and is used for distinguishing 1°, 2° & 3°alcohols.
Similarly, reaction of alcohols with HBr in presence of H2SO4 results in the
formation of bromoalkanes. For example,

Order of reactivity of alcohols is 3° > 2° > 1°


(ii) By reaction with phosphorous halides and thionyl chloride
Phosphorous pentachloride (PCl5), phosphorous trichloride (PCl3), and thionyl
chloride (SOCl2) react with alcohols to produce corresponding chloroalkanes.
The reaction of alcohols with thionyl chloride to produce chloroalkanes is one of
the best methods for its preparation since both the byproducts SO2 and HCl
being in the gaseous state, can be removed easily. The reactions can be
represented as follows:
The bromo and iodoalkanes are prepared conveniently by reaction of alcohols
with phosphorous tribromide (PBr3) and phosphorous triiodide (PI3)
respectively.

From silver salts of carboxylic acids (Hunsdiecker reaction)


The silver salt of carboxylic acid undergoes reaction with bromine in carbon
tetrachloride resulting in the formation of bromoalkane and evolution of carbon
dioxide. The reaction is known as Hunsdiecker reaction. For example,

Mechanism. The reaction follows a free radical mechanism as follows

From other haloalkanes


(a) Formation of iodoalkanes
(i) Finkelstein reaction. The chloroalkanes or bromoalkanes on treatment
with sodium iodide (or potassium iodide) in presence of acetone results in
the formation of iodoalkane. The replacement of chloride and bromide
occurs readily by iodide ions (halide exchange reaction) because sodium
iodide (or KI) is soluble in acetone. NaCl and NaBr are insoluble in acetone
and get precipitated during the course of reaction. This reaction is known as
Finkelstein reaction.
(ii) via Grignard reagent. Grignard reagent on reaction with iodine results in
the formation of iodoalkane.

(b) Formation of flouroalkanes


The flouroalkanes are prepared by reaction of chloroalkanes with inorganic
fluorides. For example,
11.3 CHEMICAL PROPERTIES OF
HALOALKANES
The reactions undergone by haloalkanes can be broadly classified into two
categories:
(1) Substitution reactions, which involve reaction occuring at sp3 carbon
directly attached to halogen.
(2) Elimination reactions, which involve the β-carbons of haloalkanes.
Besides this, the haloalkanes are key reactants for synthesis of various
organometallic compounds and are also used in the well-known Friedel–Crafts
alkylation reaction. Some of the chemical reactions of haloalkanes are discussed
in details in other chapters at appropriate places and their reference has been
given in the present chapter. The various chemical reactions of haloalkanes are
discussed as follows:

11.3.1 Nucleophilic Substitution Reactions: A General


Discussion
In haloalkanes, the C–X bond is polarized due to high electronegativity of
halogen, and thus carbon is electrophilic in nature (electron deficient). The
substitution reactions in haloalkanes involve the replacement of halogen (leaving
group) by a nucleophile (entering group) and are called nucleophilic substitution
reactions. Halide ions being weak bases are easily displaced by nucleophiles and
thus behave as good leaving groups. The general reaction is expressed as

Depending upon the nature of substrate and to some extent on the nature of
reagent and reaction conditions, the course of the nucleophilic substitution
reactions may follow the following mechanisms (or pathways):
(a) Substitution Nucleophilic Unimolecular mechanism (referred as SN1
mechanism)
(b) Substitution Nucleophilic Bimolecular mechanism (referred as SN2
mechanism)
(c) Substitution Nucleophilic Internal mechanism (referred as SNi
mechanism)
(The SNi mechanism is discussed thoroughly in section 13.3.3; p. 466)

Substitution nucleophilic unimolecular (SN1) mechanism


The nucleophilic substitution in a unimolecular mechanism, that is, the SN1
mechanism is favoured in presence of weak bases or poor nucleophiles. Also, it
occurs in two steps. The first step involves heterolytic cleavage of C–X bond
that results in removal of halide ion and formation of a carbocation. The second
step involves the attack of nucleophile (Nu–) on carbocation to form the
substituted product as

The reaction mechanism can be explained as follows.


Step 1. Heterolytic cleavage in C–X bond: formation of carbocation
In haloalkanes the C–X carbon is sp3 hybridized and heterolytic cleavage results
in the formation of carbocation (sp2 hybridized), a reactive intermediate. This
can be depicted in the reaction form as shown.

Some important points to be kept in mind as regards the formation of


carbocation are as under.
(a) The formation of carbocation is a slow step and is thus the rate
determining step. The rate of reaction is dependent only on the
concentration of haloalkanes. Once a carbocation is formed the attack of
Nu– in next step is very fast.
(b) Since only one molecule interacts in rate determining step and rate is
dependent on the concentration of haloalkanes only the reaction is
unimolecular and follows the first order kinetics. (That’s why it is called
SN1)
(c) The ease of formation of carbocation is directly related to the stability of
carbocation. This means that a more stable carbocation is formed more
readily. The order of stability of carbocations is 3° > 2° > 1° > +CH3.
Hence, the order of reactivity of haloalkanes is
3° > 2° > 1° > CH3X.
(d) Since reaction proceeds through the formation of carbocation, there is a
possibility of rearrangement to form a more stable carbocation.
Step 2. Attack of nucleophile (Nu–) on carbocation (fast step)
The carbocation (sp2 hybridized) has a planar structure and can undergo attack
by nucleophile from front side or rear side resulting in the formation of a
substituted product.
An attack from the front results in the formation of product having same
configuration (retention of configuration) as that of the initial haloalkane while a
nucleophile attack from the back results in the formation of a product having a
configuration opposite (inversion of configuration) to that of haloalkane.
In other words, SN1 mechanism involves retention as well as inversion of
configuration. This can be illustrated as follows:
The hydrolysis of S-3-bromo-3-methylhexane with aqueous NaOH as depicted
here summarizes the overall reaction mechanism:
Substitution nucleophilic bimolecular (SN2) mechanism

In a bimolecular mechanism, the attack of nucleophile on sp3 hybridized carbon


of haloalkane and removal of halide ion, (the leaving group) occurs
simultaneously in a single step. Thus, nucleophile has no other option but to
attack the carbon from a side opposite to halogen (leaving group).
The reaction proceeds through a transition stage, which has partial attachment
of nucleophile
(Nu–) and partial detachment of leaving group X–.
The SN2 mechanism is stereospecific in nature and always leads to the
formation of substituted product with a configuration opposite to that of the
initial haloalkane. In other words, SN2 mechanism involves the inversion of
configuration (also known as Walden Inversion). This has been illustrated by
considering the example of the hydrolysis of 2-bromobutane in aqueous alkali as
follows:

The rate determining step involves interaction of two species, namely the
haloalkane and the nucleophile. Thus, rate is dependent upon the concentration
of these two species and is referred as bimolecular reaction, which follows
second order kinetics. It is to be noted that the mechanism is favoured in
presence of strong nucleophiles.
The order of reactivity of haloalkanes is dependent on the stability of
transition state. The stability of the transition state is governed by the following
factors:
(i) A primary haloalkane preferably undergoes SN2 mechanism because of
least steric hindrance in the transition state, which makes the transition state
more stable.
(ii) A bulkier group attached to carbon, makes the transition state unstable
because of steric interactions. More the number of bulkier alkyl groups,
more is the steric hinderance and lesser will be the stability of transition
state. This in turn causes a slow rate of reaction.
Hence, the order of reactivity of haloalkanes is CH3X > 1° > 2° > 3°.
The effect of substituents on the stability of transition state in SN2 reaction is
illustrated as follows:

Effect of solvent on SN1 and SN2 reactions


The solvent plays an important role in the substitution reactions and polar
solvents have a marked effect on the rate of SN1 and SN2 reactions.
{Recall: A polar solvent where hydrogen is attached to oxygen or nitrogen is
termed as polar protic solvent whereas a polar solvent where hydrogen is not
attached to oxygen or nitrogen is termed as polar aprotic solvent. For example
Water, alcohol, and formic acid are polar protic whereas acetone, DMSO, and
DMF are polar aprotic solvents. (Section 4.7)}
SN1 reactions
In SN1 reactions, the reactants in rate determining step involve neutral molecule
that is haloalkane. A polar solvent stabilizes the intermediate carbocation and
thus enhances the rate of SN1 reactions. A polar protic solvent stabilizes not only
the carbocation but also the leaving group, that is, halide ion which further
enhances the SN1 reactions.

SN 2 reactions
In SN2 mechanism, the reactants in rate determining step involve both
haloalkane and nucleophile (a charged species). A polar protic solvent interacts
with nucleophile and solvolyzes it. As solvent molecules surround the
nucleophile, its approach towards electrophilic carbon gets hindered and the rate
of nucleophilic substitution reaction decreases.
Thus, the SN2 reaction rate is decreased in presence of polar protic solvents and
to increase the rate of SN2 reactions, polar aprotic solvents are used. Polar
aprotic solvents provide a polar medium but do not inhibit the approach of
nucleophile towards electrophilic carbon, as they do not solvolyze the
nucleophile.

More about nucleophiles


As discussed earlier in Chapter 4, a nucleophile is defined as an electron rich
species that may be negatively charged or neutral. The nucleophilicity is the
ability of the nucleophile to form bond with an electrophilic (electron deficient)
centre. The nucleophilicity is dependent on following factors.
(i) The availability of electron density on an atom through which nucleophilic
attack occurs. Higher the electron density, higher is the nucleophilicity.
Thus, the negatively charged ions are more nucleophilic compared to
corresponding neutral species. For example, the nucleophilicity of some
charged ions is mathematically quantified as follows.

In a similar manner, if attacking nucleophilic atoms are different but of


almost same size (that is they belong to the same period of the periodic
table), a strong base behaves as a strong nucleophile.
In other words, if attacking nucleophilic atoms are either same or have a
similar size, the nucleophilcity is directly related to basicity of the species.
That is,

(ii) The polarizability of attacking nucleophilic atom is another important


factor, which is considered when the nucleophilic atoms are different and
also have an appreciable difference in their size. For example, in a set of
nucleophiles belonging to the same group of periodic table (as in case of
halide ions), the larger the size of nucleophilic atom the higher will be its
nucleophilicity. A larger atom can easily accommodate the negative charge
by delocalization and is more stable and thus more nucleophilic. However,
the nature of solvents greatly affects the nucleophilicity. the relative
variation in the nucleophilicity is expressed in the following examples.
More about leaving groups
A good leaving group is one which can accommodate the negative charge
present on it. A species with a larger size can easily accommodate negative
charge. In other words, in a larger size species the dispersion of negative charge
is more. A large sized, negatively charged species thus, behaves as a weak base
and this is the reason that weak bases are generally good leaving groups.
Halide ions are considered to be good leaving groups
The leaving group ability of halide ions follow the order I– > Br– > Cl– > F–.
Iodide ion has a larger size than all the others and can accommodate the negative
charge easily, thus behaving as the best leaving group.
Sulfonates (sulfonic ester groups) are good leaving groups
As negative charge is delocalized among oxygens in the sulfonates, for example,
methanesulfonate CH3SO3– (mesylate), trifluoromethanesulfonate CF3SO3–
(triflate ion), p–methylbenzenesufonate ion (tosylate ion), these groups act as
good leaving groups.

Water is good leaving group


The hydroxide ion is a strong base and is not a good leaving group. On the other
hand water is a neutral molecule and behaves as a good leaving group. Thus,
removal of –OH occurs through formation of oxonium ion as neutral water
molecule (refer p. 463).

Table 11.1 compares both reaction mechanisms in the light of various


characteristics.


11.3.2 Reactions of Haloalkanes
Nucleophilic substitution reactions
1. Reaction with oxygen nucleophiles
The nucleophilic species which attack the electrophilic carbon of haloalkane
through oxygen are termed as oxygen nucleophiles. For example, –OH,
–OC H , RCOO–, and (CH ) S=O.
2 5 32
(i) Reaction of haloalkane with aq. NaOH, aq. K2CO3, or moist silver oxide
(AgOH) results in the formation of alcohol.

(ii) Reaction of haloalkanes with sodium alkoxide or dry silver oxide results in
the formation of ether.
(iii) Reaction of haloalkanes with dimethylsulfoxide results in the formation
of carbonyl compounds. For example,

3. Reaction with carbon nucleophiles

(i) Reaction of haloalkanes with acetylide ions results in the formation of


higher alkynes. The reaction results in the formation of new C–C bond and
is used for lengthening of carbon chain during organic synthesis. For
example
(ii) Reaction of haloalkanes with carbanions: Formation of carbon–carbon
bond
The α-hydrogen in carbonyl compounds is acidic in nature and its abstraction in
presence of a base results in the formation of a carbanion, which on reaction
with haloalkanes results in the formation of alkylated products (i.e. formation of
C–C bond). The reactions of active methylene compounds with haloalkanes are
of immense importance since these types of reactions are used to synthesize a
number of organic compounds. Chapter 20 is devoted to the chemistry of such
type of reactions.

4. Reaction with sulfur nucleophiles


The nucleophiles which attack the electrophilic carbon of haloalkane, through
their sulfur atom, are termed as sulfur nucleophiles.
(i) Reaction of haloalkanes with sodium hydrogen sulfide results in the
formation of alkanethiols. For example,

(ii) Reaction of haloalkanes with sodium sulfide results in the formation of


thioethers.
(iii) Reaction of haloalkane with sodium sulfite results in the formation of
sodium salt of sulfonic acid. For example,

5. Ambident nucleophiles
The nucleophiles which can attack a substrate through two possible sites are
termed as ambident nucleophiles. For example, a cyanide ion can attack as a
nucleophile through carbon as well as through nitrogen. Similarly, nitrite can
attack as a nucleophile through oxygen and nitrogen.

The atom of ambident nucleophile, which attacks the


electrophilic carbon, depends upon the type of substitution reaction,
that is, SN1 or SN2

Attack through more electronegative site of ambident


nucleophile: In reactions resulting in the formation of carbocation
as intermediate (SN1 pathway), the ambident nucleophile attacks
through more electronegative atom. Thus, 3° haloalkanes (as they
follow SN1 pathway) react with ambident nucleophile through
more electronegative atom.
The overall reactivity of haloalkanes following SN1 pathway with
ambident nucleophile is as follows:
(i) In case of nitrite: Attack through oxygen, which results in the
formation of nitrite as the major product.
(ii) In case of cyanide: Attack through nitrogen, which results in
the formation of isonitrile as the major product.
Attack through less electronegative site of ambident nucleophile:
In reactions resulting in the formation of transition state (SN2
pathway), the ambident nucleophile attacks through less
electronegative atom. Thus, 1° haloalkanes (as they follow SN2
pathway) react with ambident nucleophile through less
electronegative atom.
The overall reactivity of 1° haloalkanes with ambident nucleophile
is as follows:
(i) In case of nitrite: Attack through nitrogen, which result in the
formation of nitro compound as major product.
(ii) In case of cyanide: Attack through carbon, which result in the
formation of nitrile as major product.
Reactions with silver salts: A comment

In general Ag+ ions promote SN1 reaction as they can pull the
leaving group effectively and helps in generating the carbocation.
Thus, with silver salts nucleophilic attack occurs mainly through
more electronegative atom in the nucleophilic substitution
reactions.

(i) Reaction of haloalkanes with aqueous alcoholic solution of NaCN or KCN


results in the formation of alkanenitriles (via SN2 mechanism). The attack
of nucleophile occurs through carbon.

The tertiary haloalkanes do not undergo this substitution reaction and instead
undergo an elimination to form alkenes.
(ii) Reaction of haloalkanes with alcoholic solution of AgCN results in the
formation of alkaneisonitrile (via SN1 mechanism). The attack of
nucleophile in this case occurs through nitrogen.
(iii) The reaction of sodium nitrite with primary or secondary haloalkanes (via
SN2 mechanism) result in the formation of nitroalkanes. The ambident
nucleophile (nitrite ion) attacks through less electronegative atom (i.e.
nitrogen).

Alkyl nitrites are formed as byproducts in these reactions.


(iv) In case of primary haloalkanes, reaction with silver nitrite results in the
formation of nitroalkanes as SN2 mechanism is favoured.
The reaction of silver nitrite with tertiary and secondary haloalkanes (via SN1
mechanism) results in the formation of alkylnitrites as the major product. The
ambident nucleophile (nitrite ion) attacks through more electronegative atom
(i.e. oxygen).
The various nucleophilic substitution reactions of haloalkanes that have been
discussed so for, find their synthetic utility in functional group transformation
and are summarized in Fig. 11.1.

Fig. 11.1 Synthetic utility of nucleophilic substitution reactions of haloalkanes: Functional group
transformations from haloalkanes using oxygen, sulfur, nitrogen and carbon nucleophiles.

11.3.3 Elimination Reactions: A General Discussion

In haloalkanes, the reaction with an electron rich species (Nu–) may either be a
nucleophilic substitution reaction or an elimination reaction. In fact, the
substitution and elimination reactions compete with each other. The reaction of
haloalkanes with aqueous sodium or potassium hydroxide results in the
formation of alcohol (substitution reaction). However, the reaction of
haloalkanes with alcoholic potassium hydroxide results in dehydrohalogenation
to produce an alkene (elimination reaction).
In general elimination reaction occurs: (i) in the presence of a strong and/or
bulkier base;
(ii) at high temperature; and (iii) with bulkier haloalkanes.
The alcoholic KOH results in the formation of a strong base, ethoxide ion (
–OC H ). The much stronger ethoxide ion abstracts a proton to form an alkene.
2 5

The elimination may proceed through a unimolecular (E-1) or bimolecular (E-2)


mechanism and the reaction follows Saytzeff elimination (section 6.2.2).
Elimination unimolecular (E-1) mechanism
The unimolecular elimination mechanism is a two step process and proceeds
through the formation of carbocation intermediate. For example, the
dehydrohalogenation in case of 1-bromopropane results in the formation of
propene as follows:
Step 1. Heterolytic cleavage of C–X bond: formation of carbocation.

• The rate determining step involves only the haloalkane. Thus, it’s a
unimolecular reaction and follows first order kinetics.
• The stability of carbocation governs the rate of reaction. The order of
stability of carbocations and hence, the order of reactivity of haloalkanes
towards elimination reactions is 3° > 2° > 1° > CH3X.
• The reaction involves a carbocation intermediate. Therefore, there is a
possibility of carbocation rearrangement.
Step 2. Abstraction of β-proton of the carbocation by strong base: formation of
an alkene

Elimination bimolecular (E-2) mechanism


This is a single step mechanism where abstraction of β-proton by base and
removal of halide ion occur simultaneously. The elimination involves removal of
hydrogen halide and the formation of a π-bond.

The rate determining step (only step) involves the base as well as the haloalkane
so it’s called a bimolecular reaction, which follows a second order kinetics. It
should be noted that
(i) A more substituted alkene is favoured during the reaction.
(ii) An E-2 mechanism involves anti elimination, that is, removal of β-
hydrogen and halide ion occurs from opposite sides.
In general, elimination reactions favour bimolecular (E-2) mechanism, as it does
not involve carbocation rearrangement. The following table (Table 11.2)
compares the E-1 and E-2 mechanism.
E1cB (Elimination, unimolecular, conjugate base) mechanism
The presence of electron-withdrawing substituents on β-carbon of haloalkane
favours E1cB mechanism. This is a step wise mechanism where first step is fast
and involves loss of β-hydrogen of haloalkane, in presence of a base, to give a
carbanion. The loss of halide ion from carbanion intermediate, is a slow, rate
determining step that results in the formation of an alkene.

11.3.4 Substitution Versus Elimination


Both substitution and elimination reactions occur simultaneously. The ratio of
substituted and elimination products formed depends upon two factors, namely
the structure of haloalkane and the nature of base used.
Structure of haloalkane
In the presence of a strong base, with an increase in the bulk of alkyl groups of
haloalkane, the ratio of elimination product increases. In a sterically hindered
haloalkane elimination product is preferred over substitution product.
11.4 ORGANOMETALLIC COMPOUNDS –AN
OVERVIEW
[REACTION OF HALOALKANES WITH METALS]
Haloalkanes (as well as haloarenes) react with metals to form corresponding
organometallic compounds. This section discusses some such formations.
(a) Formation of Grignard reagent. The reaction of haloalkanes with
magnesium metal in anhydrous ether as a solvent results in the formation of
alkylmagnesium halides which are referred more commonly as Grignard
reagents.

(b) Formation of alkyllithiums. The reaction of haloalkane with lithium


metal in anhydrous ether as a solvent results in the formation of
alkyllithiums.

(c) Formation of Gilman reagent via alkyllithium. The reaction of


alkyllithium with cuprous iodide in anhydrous ether as a solvent results in
the formation of lithium dialkylcuprate also known as Gilman reagent.

11.4.1 General Characteristics


The organometallic compounds exhibit some general characteristics that can be
enumerated as:
(a) The organometallic compounds contain carbon–metal bond.
(b) Due to high electropositive character of metal, the carbon bears a negative
charge, that is, carbanion character develops at the organic moiety.

(c) A more electropositive metal forms a more ionic carbon–metal bond and
the electropositive character of metals follows the order: Li > Mg > Cu
(d) The preparation and reactions of organometallic compounds are carried
out in suitable inert solvents like ethers, under anhydrous conditions. In
general, the ethers used are diethyl ether, dimethoxyethane (DME), and
cyclic ethers such as Tetrahydrofuran (THF) and dioxane.

(e) The ethers solvate the organometallic compounds easily, that is, they
remain in dissolved form and generally exist as etherates.

(f) The preparation of organometallic compounds involves the oxidative


insertion of metal into carbon–halogen bond that is, there is a change in the
oxidation state of metal. For example the reaction of magnesium metal with
haloalkanes to form alkyl magnesium halide (Grignard reagent) involves a
change in the oxidation state of magnesium
from Mg (0) to Mg (II). This means that the metal is oxidized and
haloalkane is reduced in turn.
The organometallic compounds such as Grignard reagent, organolithium, and
Gilman reagent are of immense synthetic utility and serve as a versatile tool for
synthesis of almost all types of functional groups.
Note:
(1) The reactions of organometallic compounds are discussed invariably in
most of the chapters on the functional group chemistry.
(2) A REVIEW of the reactions of Grignard reagent, organolithium, and
Gilman reagent is given in Chapter 25, and a number of solved problems
based on organometallic compounds are discussed in the sets of “Explore
more” in the book.
11.5 ALLYL CHLORIDE AND VINYL CHLORIDE
[Unsaturated Halogenated Hydrocarbons]

11.5.1 Preparations
Vinyl chloride [Chloroethene]
Reaction of ethyne (acetylene) with one mole of HCl results in the formation of
vinyl chloride (chloroethene). It is an electrophilic addition reaction (Section
8.3.2).

Allyl chloride [3-Chloropropene]


Reaction of chlorine with propene at 500°C results in the formation of allyl
chloride. It is a free radical substitution reaction (Section 6.3.5).

Allyl bromide is prepared by the reaction of propene with NBS (Section 6.3.5)

11.5.2 Chemical Reactivity of Vinyl and Allyl Halides


The unsaturated halogenated hydrocarbons where halogen is separated from
double bond by three or more carbon atoms show a similar behaviour as
haloalkanes (saturated compounds). However, in case of vinyl and allyl halides
the halogen is separated from double bond by one and two carbons respectively
and thus, they differ in their reactivity compared to saturated haloalkanes. The
double bond affects their properties as follows:
(i) In vinyl halides, the reactivity towards nucleophilic substitution decreases
compared to haloalkanes. In vinyl halide, the lone pair on halogen
participates in delocalization with its conjugated double bond. This +R
effect introduces a double bond character between carbon and halogen,
which decreases the positive charge on carbon. Due to reduced electrophilic
character of carbon in vinyl halide, the nucleophilic reagent does not attack
it readily.
Further the double bond character strengthens the C–X bond thereby making
the removal of halide ion difficult. Also the carbon of C–X bond is sp2
hybridized and thus C–X bond is shorter and stronger compared to
haloalkanes (where carbon of C–X bond is sp3 hybridized).

(ii) In allyl halide, the reactivity towards nucleophilic substitution is high


compared to saturated haloalkanes. In allyl halide, the removal of X–
results in formation of allyl carbocation, which is stabilized by
delocalization.

Thus, allyl carbocations are formed readily and undergo nucleophilic


substitution reaction at a faster rate.
This implies that the nucleophilic reactions like hydrolysis occur at a faster rate
in allyl chloride compared to vinyl chloride. Similarly, the reactions with other
nucleophilic reagents occur faster in case of allyl chloride as compared to
haloalkanes. Vinyl chloride does not undergo nucleophilic substitution reactions
under these conditions.
11.6 POLYHALOGENATED COMPOUNDS: A
BRIEF ACCOUNT

11.6.1 Dihalogen Derivatives


The dihalogen derivatives of general interest are 1,1-dihaloalkanes [also known
as geminal dihalides (alkylidene halides)] or 1,2-dihalides [also known as vicinal
dihalides (alkylene halides)]. The dihalogen derivatives are generally colourless,
sweet smelling liquids with higher boiling points compared to corresponding
monohalogen derivatives. The density of dihaloalkanes is higher compared to
water and these are insoluble in water. However, these compounds are soluble in
organic solvents.
Preparations
Electrophilic addition reactions
(i) Addition of halogens to alkene (Formation of vicinal dihalides). The
addition of halogens to alkenes results in the formation of 1,2-dihaloalkanes
(vicinal dihalides). For example,

(ii) Addition of halogen acids to alkyne (Formation of geminal dihalides): The


addition of halogen acids to alkynes results in the formation of 1,1-
dihaloalkanes (geminal dihalides). For example,
Reactions of phosphorous pentahalides
(i) Reaction with alkanediols (Formation of vicinal dihalides). The
alkanediols on reaction with phosphrous halides result in the formation of
1,2-dihaloalkanes (vicinal dihalides).

(ii) Reaction with carbonyl compounds (Formation of geminal dihaloalkanes).


The aldehydes and ketones on reaction with phosphorous halides result in
the formation of
1,1-dihaloalkane (geminal dihalides). For example,

Chemical properties of dihaloalkanes


The chemical reactions of dihaloalkanes are similar to monohalogen derivatives
as discussed earlier. Some chemical reactions along with their synthetic uses in
organic chemistry have been summarized in this section.
11.6.2 Trihalogen Derivatives
The important trihalogen derivatives are that of methane namely chloroform
(CHCl3), bromoform (CHBr3), and iodoform (CHI3). Chloroform (boiling point
61°C) and bromoform (boiling point 150°C) are liquids and iodoform is a yellow
solid (melting point 119°C). Both chloroform and bromoform have higher
density than water and are insoluble in water.
Preparations of haloforms
(i) Haloform reaction. Heating ethanol or acetone with halogen (Cl2, Br2, or
I2) and aqueous alkali results in the formation of trihalomethane derivatives
(Haloform). For example,

(ii) Industrial preparation of chloroform. Industrially, chloroform is prepared


by heating ethanol or acetone with bleaching powder [Ca(OCl)2].

Chlorine is an oxidizing agent and it oxidizes ethanol to ethanal. Ethanal (with


three α-hydrogens) reacts further with chlorine to form trichloroethanal (chloral)
that on hydrolysis with calcium hydroxide yields chloroform. The overall
reaction is as follows:

(iii) Preparation of chloroform from chloral hydrate. Choral hydrate on


heating with aqueous alkali results in the formation of chloroform. The
chloroform thus obtained is highly pure and used for anesthetic purposes.
Chemical reactions of chloroform
(i) Oxidation of chloroform. On exposure to air and light, chloroform is slowly
oxidized to phosgene (carbonyl chloride) which is a highly poisonous gas.

To prevent the formation of phosgene, chloroform is always kept in dark bottles


with 1% ethanol added to it as an oxidant. Ethanol converts phosgene to ethyl
carbonate.

(ii) Nitration. Reaction of concentrated nitric acid with chloroform results in


the formation of nitrochloroform (popularly known as chloropicrin).
Chloropicrin is a lachrymator and lung irritant. It is used as an insecticide.

(iii) Reaction with ketones. In alkaline medium, ketones undergo addition of


chloroform. The addition of chloroform to acetone results in the formation
of chloretone which is used as hypnotic.

The other reactions of chloroform include (discussed as indicated)


(i) Reaction with chlorine (formation of carbon tetrachloride) (Section 5A.3.1)
(ii) Carbylamine or isocyanide reaction (a test for chloroform and 1° amines)
(p. 786).
(iii) Reaction with silver (formation of acetylene) (p. 276)
(iv) Reimer–Tiemann reaction (p. 543)

NOTABILIA 10
SELECTED SOLVED EXAMPLES
Example 1. Of the following nucleophilic subsititution reactions, which one will
be faster and why?

Solution. Reaction (ii) is faster compared to recation (i) because acetate ion is a
better nucleophile compared to water. In both the cases the attacking site of
nucleophiles is oxygen. A negatively charged ion is more nucleophilic than
corresponding neutral species (refer pp. 402–03). Thus, CH3COO– has more
nucleophilicity than H2O.
Example 2. Arrange the haloalkanes in the following series in an increasing
order of reactivity towards SN1 reations:
(i) CH3CH2CH2CH2Br; CH3CH2CH(Br)CH3; (CH3)3C–Br
(ii) (CH3)2CHBr; (CH3)2CHCl; (CH3)2CHI
Solution. In series (i), all the compounds are bromoalkanes and hence the rate of
reaction will depend upon the stability of carbocations formed by loss of
bromide ions in the first step. The carbocations formed are:

Increasing order of reactivity towards SN1 reactions is:


CH3CH2CH2CH2Br < CH3CH2CH(Br)CH3 < (CH3)3CBr
In series (ii) all compounds are seocndary haloalkanes. In SN1 reaction, the first
step is removal of halide ion to form a carbocation. This step is also the rate
determining step. The halide ion is the leaving group. A better leaving group is
removed readily and thus faster will be the reaction (refer p. 404).
I– is a better leaving group followed by –Br and –Cl. Thus,
Increasing order of reactivity towards SN1 is:
(CH3)2CHCl < (CH3)2CHBr < (CH3)2CHI
Example 3. Arrange the haloalkanes in the following series in an increasing
order of reactivity in SN2 reaction towards a given nucleophile.
(i) (CH3)2CHBr; (CH3)3CBr; CH3CH2Br.
(ii) CH3CH2Br; CH3CH2Cl; CH3CH2I.
Solution. In series (i) the bromoalkanes are respectively 1°, 2°, and 3° in nature.
In SN2 reactions, the order of reactivity of haloalkanes depends on the stability
of transitions state. A bulkier 3° haloalkane will be least reactive due to steric
interactions in transition state. Thus, the order of reactivity of bromoalkanes is:
(CH3)3CBr < (CH3)2CHBr < CH3CH2Br
In series (ii), all are 1° haloalkanes. The order of reactivity will be related to the
ease with which the halide ion breaks away, that is, the ability of halide ion as
good leaving group (see previous example). As the leaving ability follows the
order I– > Br– > Cl–, the order of reactivity of haloalkanes is:
CH3CH2Cl < CH3CH2Br < CH3CH2I
Example 4. Solvolysis of CH2=CHCH2Cl is much faster compared to
(CH3)2CHCl. Explain.
Solution. The solvolysis in the above chloro derivatives occurs through SN1
mechanism. The allyl chloride CH2=CHCH2Cl easily enters an SN1 reaction as
it readily forms allyl carbocation by removal of chloride ion. Similarly, isopropyl
chloride follows SN1 mechanism to form a secondary carbocation. Stability of
allyl carbocation is more compared to isopropyl carbocation since the former is
stabilized through resonance effect (refer p. 419).

Higher the stability of intermediate carbocation, faster will be the solvolysis.


Thus, allyl chloride solvolyzes faster than isopropyl chloride.
Example 5. Complete the following reactions by writing the main products
formed. Give a suitable explanation for the difference in the course of reactions.

Solution

Reactions (i) and (ii) are carried out in the presence of same base, i.e. sodium
ethoxide. In
reaction (i), the substrate is a primary haloalkane which undergoes preferably a
substitution reaction. In reaction (ii), the substrate is a tertiary haloalkane which
undergoes preferably an elimination reaction.
The substitution and elimination reations compete with each other. In presence
of same base, the bulkier haloalkane (3°) undergoes elimination. If the substrate
is same but reation is carried out with different bases, a strong base favours
elimination (see pp. 415–16)
Example 6. Why in the reaction of (CH3)3CCl with –OH, there is no formation
of (CH3)3C–OH?

Solution. In presence of strong base –OH, the bulkier 3° haloalkane preferably


undergoes elimination reaction to give alkene as the major product. The
competitive substitution reaction which gives alcohol does occur but the
substituted product is formed in a very little amount (i.e. less than 1%). So, the
major product formed is elimination product.

Example 7. Why is the conversion of haloalkane to alkane called reduction and


halogenation of alkane oxidation? Explain.
Solution. Reduction is either a complete or relative gain of electrons by an atom
while loss of electrons (complete or relative) is an oxidation process. In an
alkane, the electron pair is held equally by carbon and hydrogen of C–H bond. In
haloalkanes, the electron pair lies relatively more towards halogen in C–X bond
due to high electronegativity of halogens.

Halogenation of an alkane causes shifting of electrons from carbon to halogens,


that is, relative loss of electrons from carbon, which is oxidation. On the other
hand in conversion of haloalkane to alkane, the carbon has a relative gain of
electrons with respect to carbon of haloalkane and the process is known as
reduction.
EXERCISES
1. Give the IUPAC names for the following haloalkanes:

2. Write the structure and IUPAC names for all isomeric chlorobutanes
(C4H9Cl) and bromopentanes (C5H11Br).
3. How are the haloalkanes prepared from alkanes and alkenes? What type of
mechanism is involved in these preparations?
4. Give the convenient method for the synthesis of the following haloalkanes.
Name the type of reaction mechanism involved and also indicate the name
reaction(s), if any?
(a) 2-Chloropropane from alkene
(b) 1-Iodobutane from 1-Chlorobutane
(c) 2-Bromobutane from but-1-ene
(d) 1-Bromobutane from but-1-ene
(e) Bromoethane from carboxylic acid
(f) tert. Butylbromide from tert. butyl alcohol
5. Giving a suitable explanation, arrange the following haloalkanes in increasing
order of boiling points:
(a) CH3Br, CH3Cl, CH3I, CH3F
(b) n-butyl bromide, isobutyl bromide, 2° butyl bromide, and 3° butyl
bromide
6. Giving examples, explain the following terms:
oxygen nucleophiles, nitrogen nucleophiles, sulfur nucleophiles, and carbon
nucleophiles.
7. What product(s) will be obtained when 1-bromobutane is treated with
(a) Aqueous KOH
(b) Alcoholic KOH
(c) Na/ether
(d) Mg/ether
(e) NaI/acetone
(f) NaCN
(g) AgCN
(h) C2H5ONa
8. How will you distinguish between haloalkane, alkenes, and terminal alkynes?
9. How will you distinguish between chloroethane and vinyl chloride?
10. How will you distinguish between chloroform and carbon tetrachloride?
11. Explain, why
(a) Vinyl chloride is less reactive than ethyl chloride.
(b) Allyl chloride is more reactive than vinyl chloride.
12. Discuss and compare the mechanism of SN1 and SN2 reactions of
haloalkanes.
13. Comment on the stereochemical aspects involved in the bimolecular
nucleophilic substitution reactions?
14. Explain the term nucleophilicity by taking suitable examples.
15. Define the role of solvents in nucleophilic substitution reactions.
16. What are leaving groups? Explain with examples.
17. Define ‘elimination reactions’ and explain the conditions under which they
compete with substitution reactions.
18. Give the detailed mechanism of the reaction of bromomethane and 2-bromo-
2-methylpropane with aqueous NaOH.
19. What are organometallic compounds and how are they prepared?
20. What is the effect on the polarity of alkyl group in haloalkane after its
reaction with metal?
21. What are haloforms? Give industrial method of preparation of chloroform.
22. Why is ethanol added to bottles during storage of chloroform?
23. How are geminal and vicinal dihalides prepared from unsaturated
hydrocarbons?
Chapter12
Haloarenes

12.1 INTRODUCTION
The aromatic halogen compounds can be classified into two categories, namely
(i) the compounds where halogen is directly attached to benzene ring called
halobenzenes or haloarenes and (ii) the compounds where halogen is attached to
the side chain of benzene ring. A few examples of both the categories along with
their IUPAC names are given here. This chapter describes the chemistry of
haloarenes in detail.

In haloarenes, the carbon to which halogen is attached is sp2 hybridized. This


has been illustrated by taking examples of chlorobenzene and benzyl chloride as
follows.
12.1.1 Physical Properties
The haloarenes are polar in nature due to the electronegativity of halogens and
are denser compared to water. The boiling point of haloarenes increases with an
increase in molecular mass of the halogen.
The boiling point and densities of few aromatic halogen compounds are as
follows:
12.2 PREPARATION OF HALOARENES
This section deals with the discussion on various methods of preparations for
haloarenes.
Halogenation of benzene
The reaction of Cl2 or Br2 with benzene in presence of a catalyst like FeCl3 or
AlCl3, results in the formation of chlorobenzene. Similarly, Br2 in presence of a
catalyst like FeBr3 or AlBr3 forms bromobenzene.
The halogenation of benzene is an electrophilic substitution reaction and takes
place in the following manner.

From arenediazonium salts


The arenediazonium salts are prepared from aniline by treatment with nitrous
acid at a very low temperature.
Fluorobenzene is prepared by Balz-Schiemann reaction. It involves the reaction
of benzenediazonium chloride with hydrofluoroboric acid (HBF4) to produce
fluoborate, which on thermal decomposition results in the formation of
fluorobenzene.
Chloro and bromobenzene are prepared in good yields by Sandmeyer Reaction,
in which benzenediazonium salt is treated with CuCl or CuBr.
To prepare iodobenzene, the diazonium salt is treated with potassium iodide. The
reactions are as follows:
From silver salt of carboxylic acid (Hunsdiecker reaction)
The silver salt of benzoic acid on treatment with bromine in CCl4 at 70oC
undergoes decarboxylation to produce bromobenzene. The reaction is known as
Hunsdiecker reaction.
12.3 CHEMICAL PROPERTIES OF HALOARENES
The haloarenes undergo various chemical reactions such as nucleophilic
substitution reactions, electrophilic substitution reactions, and reactions with
metals. A detailed discussion on the chemical properties of haloarenes is as
follows:

12.3.1 Nucleophilic Substitution Reactions


Reactivity of haloarenes towards nucleophilic substitution reactions: a
comparison with haloalkanes
Haloarenes are relatively unreactive towards nucleophilic substitution reactions
as compared to haloalkanes.
(i) The carbon–halogen bond in halobenzenes is much shorter and stronger
compared to that in haloalkanes. The carbon in halobenzenes is sp2
hybridized, that is, C–X bond in halobenzenes is formed by sp2–p overlap
whereas in haloalkanes it is formed by sp3–p overlap. The electrons of sp2
hybridized carbon, being closer to nucleus compared to sp3 hybridized
carbon, form a stronger bond.

(ii) In halobenzenes, the lone pair of electrons on the halogen participates in


delocalization with the π-electrons of benzene ring and exhibits +R effect.
During resonance effect, a double bond character develops in C–X bond, which
further strengthens the C–X bond compared to haloalkanes where no such
resonance effect (+R) occurs.
These two factors (discussed above) which strengthen the C–X bond in
halobenzenes, make the removal of halide ion difficult and thus nucleophilic
substitutions do not occur as readily as in case of haloalkanes. The substitution
reactions in haloarenes take place only under drastic conditions of temperature,
pressure, and reagent concentration.
PATHWAY OF NUCLEOPHILIC SUBSTITUTION
[Haloarenes do not follow SN1 or SN2 mechanism: an explanation]
In haloarenes the substitution does not occur through SN1 or SN2 mechanisms
as in case of haloalkanes. It is to be noted that
(i) In both SN1 and SN2 mechanisms, the C–X bond cleavage requires a very
high energy due to high strength of C–X bond.
(ii) The SN1 mechanism is not favoured, since removal of halide from
halobenzenes results in the formation of phenyl cation, which is highly
unstable.

(iii) SN2 reactions are stereospecific where attack of nucleophile occurs from
the back (rear side) to form a product with inversion of configuration. In
halobenzenes, the steric hindrance due to aromatic ring shields the back
side attack of nucleophile.

Thus, substitution in haloarenes instead of SN1 or SN2 occurs through the


following two mechanisms:
Elimination–addition mechanism. To carry out substitution in haloarenes drastic
conditions such as high temperature, high pressure, and strong concentrated
reagents are to be applied. The substitution under these conditions follows an
elimination–addition mechanism via benzyne intermediate.
Addition–elimination mechanism. In case when electron-withdrawing groups are
attached to benzene ring in haloarenes, the nucleophilic substitution reactions
occur readily. The substitution under these conditions follows an addition–
elimination mechanism through the intermediary formation of Meisenheimer
complex.
These mechanisms are discussed as follows:
Elimination–addition mechanism
The chlorobenzene under high temperature and pressure conditions, and in the
presence of concentrated nucleophilic reagents undergoes nucleophilic
substitution reactions as follows:

Mechanism. Let us consider the formation of aniline when reaction of


chlorobenzene is carried out with a strong base, NaNH2, in presence of
ammonia.
Step 1. Abstraction of ortho-proton of chlorobenzene: formation of carbanion
Sodamide (NaNH2) is a strong base. In chlorobenzene, the presence of highly
electronegative chlorine makes the ortho-hydrogen acidic and this hydrogen is
easily taken away as a proton by strong base like amide ion (–NH2).

Step 2. Elimination of chloride ion: formation of benzyne intermediate


The electronegative chlorine stabilizes the carbanion through its –I effect. The
carbanion undergoes loss of a chloride ion that results in the formation of highly
reactive intermediate called benzyne.

In benzyne, sp2 orbitals lie in the plane of the ring and do not interact with the p
orbitals of the aromatic system. The two sp2 orbitals are in same plane but the
overlap is not effective due to improper orientation and thus, they form a very
weak bond. For this reason, the benzyne intermediate is highly unstable and thus
highly reactive.

Step 3. Addition of –NH2 to benzyne: formation of carbanion


The highly reactive benzyne undergoes addition of amide ion and results in the
formation of carbanion.

Step 4. Proton abstraction by carbanion: formation of substituted product

The overall reaction may be written as:


Similarly the Dow’s process for synthesis of phenol involves treatment of
chlorobenzene with concentrated NaOH solution under high temperature and
pressure conditions.
The reaction follows elimination addition mechanism through a benzyne
intermediate as shown in the following sequence:

Proof for elimination–addition mechanism


The first step in elimination–addition mechanism is the abstraction of ortho-
hydrogen of halobenzene by a strong base. If we take an ortho-substituted
chlorobenzene and carry out the reaction with sodamide, no product formation
occurs. This proves that substitution occurs through elimination–addition
mechanism.

Proof for benzyne intermediate


If we start with an isotopically labelled halobenzene, say, isotopically labelled
chlorobenzene (the carbon to which chlorine is attached is isotopically labelled
and is marked by asterik) and treat it with NaNH2/NH3 we get equal amounts of
two types of anilines. In one case, the carbon carrying the –NH2 group is
isotopically labelled and in other case it is unlabelled.
The formation of these two types of anilines is possible only if reaction proceeds
through a benzyne intermediate as shown in the following reaction sequence:

Addition–elimination mechanism
The presence of electron-withdrawing groups on benzene ring of haloarenes
enhances the rate of nucleophilic substitution reactions as evident from the
following reactions. For example, the formation of 2-nitrophenol from 2-
chloronitrobenzene occurs in alkaline medium at about 135°C. On the contrary
2,4,6-trinitrophenol (picric acid) can be prepared from 2,4,6-
trinitrochlorobenzene in alkaline medium at room temperature.
Thus, more the number of electron-withdrawing groups present at ortho- and
para- positions, faster is the nucleophilic substitution reaction.
In the presence of electron-withdrawing groups the reaction follows addition–
elimination mechanism. The nucleophile in the first step adds on to electrophilic
carbon of C–X bond. This is a slow step, which results in the formation of an
anion. The anion is stabilized by delocalization. The delocalization occurs in
presence of electron-withdrawing groups and occurs effectively if the electron-
withdrawing groups are present at ortho- and para- positions.

Mechanism. Let us consider the general mechanism taking the reaction of 4-


nitrochlorobenzene
(p-nitrochlorobenzene) with NaOH as an example.
Step 1. Addition of nucleophile: formation of anion
The addition of nucleophile to the electrophilic carbon of C–Cl occurs to give an
anion. This is a slow step. The anion formed in the process (also called
Meisenheimer complex) is stabilized by delocalization.

The presence of electron-withdrawing group stabilizes the Meisenheimer


complex by delocalization of the negative charge. Delocalization of negative
charge occurs maximum when electron-withdrawing group(s) are present at
ortho- and para- position.
Step 2. Elimination of chloride ion: formation of substituted product

Ipso Substitutions: A definition


Substitution reactions of aromatic ring systems where a group other
than hydrogen is substituted are known as ipso substitutions. In the
reactions discussed so far, the halogen in the benzene ring is
substituted by groups like –NH2, –OH etc. Thus, these reactions are
the examples of ipso substitution.

12.3.2 Electrophilic Substitution Reactions


As we have already discussed (p. 348) that due to +R effect halogens behave as
electron-releasing groups and are thus ortho- and para- directors. But at the
same time the strong –I effect of halogens makes them weak deactivators
towards electrophilic substitution reactions.

The haloarenes undergo usual electrophilic substitution reactions such as


halogenation, nitration, sulfonation, and Friedel–Crafts reaction to yield
corresponding ortho- and para- substituted haloarenes. For example,

The mechanisms for these electrophilic substitution reactions have already been
discussed in details in Chapter 10.

12.3.3 Reactions of Haloarenes with Metals


Besides the substitution reactions (nucleophilic and electrophilic) as already
discussed, the haloarenes undergo reactions with different metals to form
different products as follows:
Reaction with magnesium metal (Formation of Grignard Reagent).
Bromobenzene and iodobenzene react with magnesium metal in presence of dry
ether to form Grignard reagent.

Chlorobenzenes are relatively unreactive and are generally not


used for the preparation of Grignard reagent.
For example, in case of reaction of 1-bromo-3-chlorobenzene and
magnesium metal in dry ether, the formation of Grignard reagent
occurs selectively from the side of aryl bromide.

Thus, the order of reactivity of aryl halides (haloarenes) towards


magnesium metal is as follows:
Aryl iodide > Aryl bromide >> Aryl chloride
Reaction with lithium (Formation of aryllithiums). The haloarenes react with
lithium metal in presence of dry ether to form aryllithiums.
The lithium metal is more electropositive and has higher reactivity compared to
magnesium metal. As already mentioned above, chlorobenzene is least reactive
towards magnesium and not used for preparing Grignard reagent. However, it
readily reacts with lithium metal in presence of dry ether to form phenyllithium.
Reaction with sodium metal [Formation of alkyl benzene (Wurtz–Fittig
reaction)]. Haloarenes react with haloalkanes in the presence of sodium metal to
yield alkylbenzenes. For example

Reaction with copper metal [Formation of biphenyl (Ullmann Reaction)].


Haloarenes on refluxing with copper result in the formation of biphenyl. The
reaction occurs through coupling of phenyl radicals to form biphenyl.
12.4 SIDE CHAIN HALOGEN SUBSTITUTED
AROMATIC COMPOUNDS
The aromatic halogen compounds also include the compounds where halogen is
attached to side chain of the benzene ring. For example, benzyl chloride, benzal
chloride, and so on. Such compounds depict the characteristics reactions of
haloalkanes as well as that of aromatic rings.

12.4.1 Preparations
This section describes the various methods used for the preparation of aromatic
compounds with halogens attached to the side chain.
Direct halogenation
The reaction of toluene with halogens in presence of sunlight or ultraviolet light,
results in its substitution at side chain rather than, at the hydrogen of the
aromatic ring.

The halogenation of side chain occurs by the free radical mechanism, similar to
alkanes via the formation of benzyl free radical.
By the action of N-bromosuccinimide
The reaction of toluene with N-bromosuccinimide (NBS) in the presence of
organic peroxide results in side chain bromination to form benzyl bromide.

From benzyl alcohol


The reaction of benzyl alcohol with HCl in the presence of anhydrous zinc
chloride results in the formation of benzyl chloride.

By action of sulfuryl chloride


The sulfuryl chloride in the presence of organic peroxide reacts with toluene to
form benzyl chloride.

Blanc reaction
The reaction of benzene with formaldehyde and HCl in presence of zinc chloride
or aluminium chloride results in the formation of benzyl chloride. This is a
typical electrophilic substitution reaction, which involves the introduction of
chloromethyl group (–CH2Cl) directly to the nucleus (aromatic ring). The
reaction is commonly known as chloromethylation.

From benzaldehyde
Benzaldehyde on reaction with phosphorous oxychloride results in the formation
of benzal chloride.

12.4.2 Chemical Properties


The side chain halogenated aromatic hydrocarbons resemble haloalkanes but
they are more reactive towards nucleophilic reactions. The high reactivity is
attributed to the resonance stabilization of benzyl carbocation formed during
substitution reactions.

The side chain (–CH2Cl) in benzyl chloride shows nucleophilic substitution


reactions, with oxygen, sulfur, carbon, and nitrogen nucleophiles similar to
haloalkanes. The reactions are summarized as follows:

In benzyl chloride, the chloromethyl group behaves as an electron-releasing


group and thus activates the ring at ortho- and para- positions. The various
electrophilic substitution reactions like nitration, sulfonation and halogenation
are given below:
Friedel–Crafts reaction
Benzyl chloride behaves as an electrophile in presence of Lewis acid in its
reaction with benzene and forms diphenylmethane.

Wurtz reaction
Benzyl chloride reacts with sodium metal to form dibenzyl. Similarly with
haloalkanes in presence of sodium metal, it forms alkylbenzene.

Reaction with magnesium metal


In dry ether, reaction of benzyl chloride with magnesium results in the formation
of Grignard reagent.
Oxidation
Benzyl chloride on oxidation in presence of potassium permanganate results in
the formation of benzoic acid.

Reduction
On reduction with lithium aluminium hydride benzyl chloride results in the
formation of toluene.

Reactions of benzal chloride and benzotrichloride


(i) Benzal chloride undergoes hydrolysis with aqueous NaOH or KOH and
results in the formation of benzaldehyde.

(ii) Benzotrichloride on hydrolysis under similar conditions results in the


formation of benzoic acid.

(iii) Oxidation of benzal chloride in the presence of KMnO4 results in the


formation of benzoic acid.
SELECTED SOLVED EXAMPLES
Example 1. The reaction of 4-chlorotoluene with aq NaOH at high temperature
results in the formation of two isomeric cresols. Name the products formed and
explain their formation using suitable mechanism.
Solution. The reaction with NaOH (aq) results in the formation of two products
as

The nucleophilic substitution in haloarenes occures via an elimination–addition


mechanism through benzyne intermediate. [See p. 436].

Example 2. o-Bromoanisole and m-bromoanisole on treatment with sodamide in


liquid ammonia give the same product. Name the product and explain its
formation.
Solution. Both o-bromoanisole and m-bromoanisole on reaction with NaNH2
result in the formation of m-methoxyaniline.
The reaction proceeds through elimination–addition mechanism via benzyne
intermediate.
Both o-methoxybromobenzene (o-bromoanisole) and m-methoxybromobenzene
(m-bromoanisole) undergo elimination in the first step in presence of amide ion
to form same benzyne intermediate. Since, two different reactants undergo
reaction through same intermediate, they result in the formation of same
product only.

The benzyne intermediate can undergo addition at either carbon to give two
carbanions. A more stable carbanion of the two results in the formation of final
product.
EXERCISES
1. Give the structure and IUPAC names for all isomers possible for
(a) C7H7Br and
(b) C6H4Cl2.
2. Under what conditions, the chlorination of benzene results in the formation of
chlorobenzene?
3. In toluene, chlorination can occur at the aromatic ring as well as at the side
chain. Give the reaction conditions for the formation of both.
4. How can chlorobenzene and bromobenzene be synthesized from benzene
diazonium salt?
5. Why is chlorobenzene less reactive than haloalkanes towards nucleophilic
substitution reactions?
6. How can chlorobenzene be converted into
(a) phenol
(b) aniline
(c) toluene
(d) benzene
(e) phenyllithium
Give, if any, name reaction(s) involved in above conversions.
7. What is the effect of electron-withdrawing group(s), present in
chlorobenzene, towards its nucleophilic substitution reaction?
8. How can picric acid be prepared from chlorobenzene?
9. Explain why 2,4,6-trinitrochlorobenzene is easily hydrolyzed with water but
chlorobenzene is not.
10. Give the electrophilic substitution reactions of chlorobenzene.
11. Chlorobenzene acts as an ortho and para director for electrophilic
substitution reactions but is a weak ring deactivator. Explain.
12. Giving evidence for the formation of benzyne intermediate, discuss the
elimination–addition mechanism for the nucleophilic aromatic substitution
reactions.
13. How can the side chain aromatic halogen compounds be prepared?
14. Give some nucleophilic substitution reactions of side chain aromatic halogen
compounds. Are these nucleophilic reactions different for side chain aromatic
halogen compounds than those for haloalkanes?
15. How can the benzyl chloride be converted into
(a) benzyl amine
(b) benzaldehyde
(c) benzyl alcohol
(d) toluene
(e) diphenylmethane
(f) benzoic acid
Give, if any, name reaction(s) involved in these conversions.
16. What is the directive influence of –CH2Cl in benzyl chloride towards
electrophilic substitution reactions? Give the products for halogenation,
nitration, and sulfonation reactions of benzyl chloride.
17. Which product will be obtained by the reaction of benzyl chloride with
(a) chloromethane in presence of sodium.
(b) benzene in presence of anhydrous AlCl3.
(c) chlorine in presence of sunlight followed by hydrolysis.
18. Give the oxidation reactions of benzyl chloride.
19. How is benzyl chloride converted into benzal chloride and benzotrichloride.
20. How will you distinguish between 2-methylbromobenzene and benzyl
bromide?
Chapter13
Monohydric
Alcohols (Alkanols)

13.1 INTRODUCTION
The class of organic compounds in which hydroxy (–OH) group is attached to a
hydrocarbon is termed as alcohols. Alcohols may be considered as the hydroxy
derivatives of hydrocarbons and have a general formula CnH2n+1OH. The
formula represents a monohydric alcohol, that is, one which contains only one
hydroxyl group. Similarly, alcohols containing two or three hydroxyl groups are
called dihydric or trihydric alcohols and are collectively termed as polyhydric
alcohols. The carbon–oxygen bond in alcohol is formed by sp3–sp3 overlap. An
alcohol may be classified as primary (1°), secondary (2°), or tertiary (3°)
depending upon the number of carbons attached to carbon bearing hydroxyl
group.

According to the IUPAC nomenclature, the alcohols are named as alkanols. The
rules for naming alcohols have already been discussed in detail in Chapter 2. A
few examples of alcohols are
13.1.1 Physical Properties
Alcohols exhibit considerable dipole moment due to the presence of
electronegative oxygen. The alcohol molecules form intermolecular hydrogen
bonds readily. The intermolecular association of alcohol molecules through
hydrogen bonding is responsible for higher boiling point of alcohols as
compared to that of corresponding hydrocarbons and haloalkanes. Generally, the
boiling point increases with an increase in molecular mass. Though methanol
has low molecular mass compared to corresponding haloalkanes, it still exhibits
a high boiling point due to intermolecular hydrogen bonding. In case of
haloalkanes, no such hydrogen bonding occurs and thus they have low boiling
points.

The isomeric alcohols show a decrease in boiling point with an increase in


branching.
The increased branching in alcohols decreases the surface area and thus boiling
point also decreases. A comparison of boiling points of alcohols can be clearly
understood in the following manner:
In a similar way, the solubility of lower alcohols in water is attributed to the
tendency of alcohol molecules to form hydrogen bonding with water molecules.

As the number of carbon atoms in the chain in alcohol increases (C4 and above),
its solubility in water decreases. This is because in higher alcohols the large
alkyl groups cause an enlargement of the nonpolar hydrocarbon part, which
decreases their solubility in water (a polar solvent). The higher order alcohols
are soluble in organic solvents. In general, the monohydric alcohols have lower
density compared to water.
13.2 PREPARATION OF MONOHYDRIC
ALCOHOLS
Alcohols can be prepared conveniently by a number of different methods, some
of which have already been discussed, in previous chapters. In this section
various methods used for the preparation of monohydric alcohols are discussed
and the reference of the section numbers is given for methods which have been
discussed earlier.
From alkenes
The preparation of alcohols from alkenes can be carried out through various
reactions, which have already been discussed in section 6.3.2. These reactions
are —
(a) Addition of water–Hydrolysis of alkenes (p. 229)
(b) Addition of concentrated H2SO4 to alkenes followed by hydrolysis (p.
228)
(c) Oxymercuration–demecuration process in alkenes (p. 230)
(d) Hydroboration oxidaton in alkenes (p. 232)
These reactions can be summarized as

From haloalkanes
The hydrolysis of haloalkanes with aqueous sodium or potassium hydroxide, or
carbonates, or moist silver oxide results in the formation of alcohols. These
nucleophilic substitution reactions have already been discussed in details in
Chapter 11. For example,
In general, hydrolysis is not preferred for preparation of
alcohols from haloalkanes in the presence of strong bases.
• The use of strong base (such as NaOH) may cause elimination (as a
side reaction) in haloalkanes to form an alkene along with the
usual substitution to form alcohol.
• Moist silver oxide and sodium carbonate are mild bases compared to
alkali metal hydroxides. Thus, the use of AgOH or Na2CO3
minimizes the possibility of elimination reaction.
FROM ORGANOMETALLIC COMPOUNDS
Reactions of Grignard reagent
(a) With carbonyl compounds. One of the most convenient and versatile
method for preparation of alcohols involves the reaction of Grignard Reagent
with aldehydes or ketones.
In Grignard reagent (an organomagnesium halide), the alkyl group behaves as a
nucleophile (electron rich) and gets added to electrophilic carbon (electron
deficient) of aldehyde or ketone to give an addition product. Hydrolysis of this
addition product results in the formation of alcohol.

By selecting an appropriate Grignard reagent and an aldehyde or a ketone, one


can prepare any of the 1°, 2°, or 3° alcohol. This can be done in the following
manner:
(i) Reaction with formaldehyde (formation of 1° alcohol). Grignard reagent on
reaction with formaldehyde results in the formation of 1° alcohol. For example,
when methylmagnesium halide reacts with formaldehyde, ethanol is formed.
Similarly, reaction of ethylmagnesium bromide with formaldehyde, followed by
hydrolysis results in the formation of propan-1-ol, a primary alcohol as shown.

Note: It is obvious that methanol cannot be synthesized by using Grignard


reagent.
(ii) Reaction with aldehydes (other than formaldehyde) (formation of 2°
alcohol). The higher aldehydes (C2 onwards) on reaction with Grignard reagent,
followed by hydrolysis, yield
2° alcohols. For example, butan-2-ol (a 2° alcohol) can be prepared by the
reaction of ethylmagnesium bromide with ethanal (acetaldehyde). Alternatively,
butan-2-ol can also be prepared by reaction of methylmagnesium bromide with
propanal.

(iii) Reaction with ketones (formation of 3° alcohols). The reaction of ketones


with Grignard reagent followed by hydrolysis results in the formation of 3°
alcohol. For example, 2-Methylbutan-2-ol can be prepared using Grignard
reagent as follows:
Alternatively,

(b) With epoxides (formation of alcohol). Epoxides on reaction with Grignard


reagent experience ring opening and on hydrolysis result in the formation of
alcohols. For example, ethylene oxide on reaction with Grignard reagent
followed by hydrolysis results in the formation of 1° alcohol.

In case of substituted epoxides, the attack of Grignard reagent occurs at a less


hindered site resulting in the formation of 2° or 3° alcohols. For example,

(c) With esters. The reaction of Grignard reagent with esters may result in the
formation of secondary or tertiary alcohols.
(i) Reaction with formate esters (formation of 2° alcohols). Grignard reagent on
reaction with ethylformate results in the formation of aldehyde, but the aldehyde
produced reacts further with another molecule of Grignard reagent to produce 2°
alcohol as the final product. For example,

(ii) Reaction with esters other than formate esters (formation of 3° alcohols).
The alkyl alkanoates (other than alkyl formate) on reaction with Grignard
reagent result in the formation of ketones which further react with another
molecule of Grignard reagent to produce 3° alcohols. For example,

(d) With acid chlorides (formation of 3° alcohols). The acid chlorides (other
than formyl chloride, HCOCl, which is not stable) on reaction with Grignard
reagent result in the formation of ketone which immediately reacts with another
molecule of Grignard reagent to form 3° alcohol. For example,
Reactions of alkyllithiums
Alkyllithiums are organometallic compounds which are prepared by the reaction
of haloalkanes with lithium using ether as a solvent (Section 11.4). In
alkyllithium, the alkyl group behaves as a nucleophile (electron rich) and reacts
readily with electrophilic carbon (electron deficient). The reactions of
alkyllithiums leading to the formation of alcohols are similar to Grignard
reagent. However, alkyllithiums are more reactive in comparison to
Grignard reagent. By selecting an appropriate alkyllithium, any type of 1°, 2°,
or 3° alcohols can be prepared. The concerned reactions are summarized as
follows:
By reduction
The aldehydes, ketones, carboxylic acids, acid chlorides, esters, and acid
anhydrides, on reduction
result in the formation of corresponding alcohols. Different types of reducing
agents are used for reduction of these classes of organic compounds.
The common methods used for the reduction of aldehydes and ketones to
alcohols are
(i) Bouveault–Blanc Reduction (Na/alcohol)
(ii) Meerwein–Ponndorf–Verley Reduction (Aluminium isopropoxide in
isopropyl alcohol)
(iii) Catalytic Reduction (H2/Pt, Pd or Ni)
Besides, Zn/acetic acid or alkali and Na–Hg/alcohol are also used as reducing
agents.
The reducing agents just mentioned cannot reduce carboxylic acids and their
derivatives. The metal hydrides such as LiAlH4 can be conveniently used for
reducing not only the carbonyl compounds to alcohols but also for reduction of
carboxylic acids and their derivatives to corresponding alcohols. For example,

Lithium aluminium hydride is a selective reducing agent (refer review, Chapter


25), which does not reduce double bond and is thus used for the preparation of
unsaturated alcohols as follows:

By hydrolysis reactions of ethers and esters


(a) Ethers. Hydrolysis of ethers in acidic medium results in the formation of
alcohol in the following manner.

(b) Esters. Esters may be hydrolyzed in acidic or alkaline medium to yield


carboxylic acids and alcohols as the products. The hydrolysis of esters in
alkaline medium is also known as saponification.
The mechanism of this reaction is discussed in Section 19.2.2 (p. 658) in detail.
From primary aliphatic amines
The primary aliphatic amines on reaction with nitrous acid (prepared in-situ by
NaNO2 + HCl) liberate nitrogen and result in the formation of 1° alcohols.
13.3 CHEMICAL PROPERTIES OF
MONOHYDRIC ALCOHOLS
The chemical properties along with the reference to key features of monohydric
alcohols are as follows:

13.3.1 Alcohols as an Acid as well as Base: A General


Discussion
Acidic nature. Alcohols show an acidic character as they release a proton from
hydroxyl group in few reactions. However, alcohols are weaker acids than water.
The presence of alkyl groups (+I effect) destabilizes the alkoxide ion formed by
the removal of proton from alcohols. The decreasing order of acidic strength of
alcohols is: 1° > 2° > 3°
Basic nature. In alcohols, the presence of lone pair of electrons on oxygen
makes them Lewis bases. The basic character of alcohols is further increased by
the presence of alkyl groups attached to carbon bearing –OH group. Alkyl
groups exhibt +I effect and thus, increase the electron density on oxygen thereby
increasing the strength of alcohol as a base. The decreasing order of basic
strength of alcohols is: 3° > 2° > 1°
Thus, alcohols behave both as acids as well as bases. It should be noted that
• In strong acidic medium, alcohols exist as alkyloxonium ions (RO+H2).

• In strong alkaline medium, alcohols exist as alkoxides (RO–).

• In neutral medium, alcohols exist as alcohol (R–OH).


Thus, the reactions of alcohols can be categorized as:
(1) Reactions involving the acidic characteristics of alcohols, that is, reactions
where cleavage of oxygen–hydrogen bond (O–H) takes place. In these
reactions, hydrogen is replaced as proton (H+).
(2) Reactions involving the basic characteristics of alcohols, that is, reactions
in which cleavage of carbon–oxygen bond (C–O) takes place and in these
reactions replacement of –OH group occurs by nucleophiles.
(3) Reactions involving the entire alcohol molecule.

13.3.2 Reactions Involving the Acidic Characteristics


of Alcohols

(Replacement of hydrogen)
When a proton is removed from an alcohol, it results in the formation of
alkoxide ion. The alcohol then behaves as an acid. The acidic strength of
alcohols depends upon the following factors:
(a) Number of alkyl groups. More the number of alkyl groups (+I effect)
attached to carbon, the lesser is the stability of alkoxide ion and lesser will
be acidic strength of alcohol.
(b) Solvation. Alkoxide ion formed is stabilized through hydrogen bonding
with solvent molecules and this is known as solvation. In 3° alcohols, the
alkoxide ion is destabilized as steric factors inhibit the solvation. Thus,
steric hindrance reduces the acidic strength and as a consequence, the 3°
alcohols are least acidic.
Thus, order of reactivity of alcohols when they behave as acids is 1° > 2° > 3°.
Reaction of alcohol with metal (formation of alkoxides)
The alcohols react with active alkali metals like sodium to form corresponding
sodium alkoxides with the release of nascent hydrogen. For example,

Reaction with carboxylic acids and its derivatives (formation of esters)


The reaction of alcohol with carboxylic acid in acidic medium results in the
formation of esters. During ester formation, the removal of water molecule
occurs where hydrogen is removed from alcohol and hydroxyl part from
carboxylic acid. The acidic medium is provided by use of dilute sulfuric acid or
by passing dry HCl gas (Fischer–Speier method). The mechanism of the
reaction is discussed in Section 19.3.3, p. 668. Similarly, the reaction of alcohol
with acid chlorides and acid anhydrides results in the formation of ester.

Reaction with Grignard reagent (formation of alkane)


The reactions of alcohols with Grignard reagent have been discussed in Section
5A.2.2; p. 173.
13.3.3 Reactions Involving the Basic Characteristics of
Alcohols

(Replacement of hydroxyl group)


The alcohols undergo nucleophilic substitution reactions where –OH group is
replaced. The hydroxy group as such is not a good leaving group (refer more
about leaving group on p. 404). Thus in order to make the reaction faster,
substitution reactions are carried out in presence of acid as it converts –OH into
a much better leaving group, that is, H2O through the formation of oxonium ion.

The nucleophilic substitution reaction in alcohols may follow SN1 or SN2


pathway. In general, the primary alcohols undergo substitution through SN2
mechanism whereas secondary and tertiary alcohols follow SN1 mechanism.

Reaction with hydrogen halides (formation of haloalkane)


Alcohols react with hydrogen halides where substitution results in the formation
of haloalkane. Reaction of alcohol with HI occurs readily to form iodoalkane
however reactions with HBr and HCl are carried out in presence of a catalyst as
illustrated in the following reactions.

The order of reactivity of alcohols is 3° > 2° > 1° and the order of reactivity of
hydrogen halides is HI > HBr > HCl.
Reaction with Lucas reagent (HCl + anhydrous ZnCl2)
[Test for distinguishing 1°, 2°, and 3° alcohol]
The reaction of alcohol with HCl in the presence of anhydrous ZnCl2 (called
Lucas reagent) results in the formation of corresponding chloroalkanes. The
alcohols are soluble in Lucas reagent whereas the chloroalkanes formed are
insoluble. The formation of chloroalkane is indicated by the cloudiness, which
appears in the reaction mixture. The time taken for the appearance of cloudiness,
by an alcohol is a measure of its reactivity. Thus, the reaction of alcohols with
Lucas reagent
(HCl/ZnCl2) is used as a test to distinguish between 1° , 2° and 3° alcohols. This
can be done by observing that
(i) in case of 3° alcohols, cloudiness appears immediately.
(ii) in case of 2° alcohols, cloudiness appears after a few minutes.
(iii) in case of 1° alcohols, no cloudiness appears.

Why is anhydrous ZnCl2 used in reaction of alcohols with


HCl?
The reaction of alcohol with HCl involves a nucleophilic substitution
reaction, which results in the formation of chloroalkans. The Cl– is
not a good nucleophile and in alcohol –OH acts a poor leaving group.
Reaction with HCl is thus, carried out in presence of ZnCl2 (Lewis
acid). ZnCl2 forms a complex with oxygen of alcohol and converts
the –OH to a much better leaving group thereby giving way to a rapid
formation of chloroalkane.

Why are 3° alcohols more reactive than 2° and 1° alcohols


towards Lucas reagent?
The high reactivity of 3° alcohols is attributed to formation of a more
stable 3° carbocation. The reaction proceeds through the formation of
carbocation intermediate. Higher the stability of carbocation, the
faster is the reaction. Since order of stability of carbocaton is 3° > 2°
> 1°, the order of reactivity of alcohols will also follow the same
pattern, that is, 3° > 2° > 1°.

Reaction with phosphorous halides and thionyl chloride (formation of


haloalkanes)
Primary and Secondary alcohols react with phosphorous pentachloride (PCl5),
phosphorous trichloride (PCl3), and thionyl chloride (SOCl2) to produce
corresponding chloroalkanes. The reaction with phosphorous tribromide at low
temperature (or phosphorous triiodide) results in formation of bromoalkane (or
iodoalkane). Use of thionyl chloride is the best method for preparation of
chloroalkanes as the side products of this reaction are gases that are eliminated
easily.
Reaction with ammonia
Passing vapours of alcohol and ammonia over Al2O3 or ThO2 results in the
formation of a mixture of amines. For example,

13.3.4 Reactions involving the entire alcohol molecule


Oxidation
(i) Oxidation of alcohol (formation of carboxylic acid). Strong oxidizing
agents such as alkaline KMnO4 and concentrated nitric acid are used for
oxidation of 1° alcohol to carboxylic acid.
The secondary and tertiary alcohols may be oxidized by concentrated HNO3 to
carboxylic acids. However, the carboxylic acids formed from 2° and 3° alcohols
contain lesser number of carbon atoms compared to parent alcohols.

(ii) Selective oxidation of 1° alcohols (formation of aldehydes). The primary


alcohols are highly susceptible to oxidation and lead to the formation of
carboxylic acids via aldehydes. The selective oxidation of 1° alcohols to
aldehydes is carried out in presence of Pyridiniumchlorochromate (PCC) which
cannot oxidize aldehydes further.

PCC is prepared by dissolving CrO3 in HCl followed by treatment with


pyridine.
(iii) Oxidation of 2° alcohol (formation of ketones). Secondary (2°) alcohols
on treatment with Na2Cr2O7 in acidic medium (Na2Cr2O7 + H2SO4 ↔
H2CrO4) results in the formation of ketone. Another reagent, which is used for
oxidation of alcohols, is Jones reagent (CrO3, H2SO4 in acetone). The
following reactions depict the oxidation.

Reaction with hot reduced copper (dehydrogenation)


Treating 1° and 2° alcohols with hot reduced copper at 300° C results in the
dehydrogenation of alcohols to give aldehyde and ketone respectively. However,
tertiary 3° alcohols under these conditions undergo dehydration to produce
alkenes in the manner as follows:

Reaction with concentrated H2SO4 or H3PO4 (dehydration)


Alcohols on heating with concentrated sulfuric or phosphoric acid result in
elimination of water molecule to form alkenes. Order of reactivity of alcohols in
this case is 3° > 2° > 1°.
Reaction of ethanol with sulfuric acid under different temperature
conditions. Ethanol reacts with sulfuric acid under different temperature
conditions to give different products. That is,
(i) At low temperature (110°C), the product formed is diethylsulfate.
(ii) At a temperature of 140°C and in presence of excess ethanol, the
formation of diethyl ether takes place.
(iii) A reaction of ethanol with excess sulfuric acid at 170°C results in the
formation of ethene.
The different reactions and their mechanisms are as follows:
Reactions of ethanol with conc. H2SO4
1. At 110°C

Mechanism. The reaction of ethanol and sulfuric acid at 110°C involves


following steps:
Step 1. Formation of oxonium ion by protonation of ethanol
Step 2. Reaction of sulfuric acid with oxonium ion through SN2 and formation
of ethyl-hydrogensulfate
Step 3. Reaction of ethyl hydrogensulfate with oxonium ion through SN2 and
formation of diethylsulfate
2. At 140°C (Diethylether)
Reaction of ethanol (in excess) with sulfuric acid at 140°C results in the
formation of Diethyl ether. This reaction is known as Williamson ether
synthesis.

Mechanisms

3. At 170°C (Ethene)
Reaction of ethanol with sulfuric acid (in excess) at 170° results in the formation
of ethene.

Mechanism
Victor Meyer test
This test is used for distinguishing 1°, 2°, and 3° alcohols. The alcohols are
converted to corresponding iodoalkanes, which are further treated with silver
nitrite to form corresponding nitroalkanes. The reaction of nitroalkanes with
nitrous acid followed by treatment with aqueous alkali gives following results
which distinguishes the 1°, 2°, and 3° alcohol.
(i) The nitroalkane from 1° alcohol on reaction with nitrous acid gives nitrolic
acid, which dissolves in aqueous alkali to give a red coloured solution.
(ii) The nitroalkane from 2° alcohol on reaction with nitrous acid gives
pseudonitrol, which is insoluble in aqueous alkali and gives blue colour.
(iii) The nitroalkane from 3° alcohol does not react with nitrous acid.

Iodoform reaction
Alcohols on oxidation result in the formation of carbonyl compounds. Alcohols
containing a CH3CH(OH)– group on reaction with iodine in presence of alkali
result in the formation of a yellow crystalline solid, iodoform along with sodium
salt of a carboxylic acid. In the first step, the
CH3CH(OH)– group is oxidized by sodium hypoiodite, NaOI (from NaOH + I2)

to group.
This group undergoes iodination at α-position in presence of a base to form a
triiodocarbonyl compound, which on alkaline hydrolysis forms iodoform.
In alcohols, the iodoform reaction is given by:
(i) Ethanol, the only primary alcohol that gives reaction.
(ii) Secondary alcohols having CH3CH(OH)– group (methyl secondary
alcohols).
Thus, iodoform reaction is used for distinguishing ethanol from other 1°
alcohols and also to distinguish CH3CH(OH)– (methyl secondary alcohols)
from other alcohols (except ethanol).
13.4 MORE ABOUT ALCOHOLS
1. Methyl alcohol or methanol, CH3OH, is also known as wood alcohol
because it was earlier obtained from the destructive distillation of hard
wood in the absence of air. Its common name methyl alcohol can also be
linked with the traditional process of its manufacturing since in Greek
methe means wine and hyle means wood. However now methanol is
manufactured in large scale by reduction of carbon monoxide with
hydrogen. Industrially methanol is used to manufacture formaldehyde and is
a precursor for many other chemicals.
2. Ethanol is also known as grain alcohol. It is obtained from the fermentation
of sugars
and starches by yeast. This process provides alcohol for beverages and
industrial
purposes. Fermentation process produces an aqueous solution of 12–15%
ethanol
because at this concentration of alcohol, yeast cells stop reproducing due to
its antiseptic action.

Direct distillation of the fermented molasses gives ~90% alcohol termed


as rectified spirit (or alcohol). The rectified alcohol is further distilled to
give 95.6% ethanol and 4.4% water as azeotropic mixture (commercial
alcohol). Dehydrated alcohol contains <1% water and is obtained from
commercial alcohol by treating with calcium oxide (quicklime) followed by
distillation or by distillation of commercial alcohol with benzene. Last
traces of water are removed from alcohol by treatment with magnesium to
obtained dry alcohol known as absolute alcohol.

Addition of pyridine or butanone to alcohol makes it unpleasant and


unpalatable, but
this does not affect its use in industrial applications, and is referred as
denatured
alcohol.
SELECTED SOLVED EXAMPLES
Example 1. The nucleophilic substitution occurs readily in haloalkanes.
Whereas nucleophilic substitution in alcohols occurs in presence of strong acids
as catalyst. Explain.
Solution. Halide ions are good leaving groups whereas –OH is a poor leaving
group. In presence of acids, it is converted to a better leaving group, water
(HOH), and then substitution occurs readily through SN1 or SN2 mechanism,
depending upon the nature of alkyl groups.

Example 2. What products will be formed on an acid catalyzed dehydration of


3-methylbutan-
2-ol? Name the major product formed in the reaction.
Solution. The alcohol in presence of acid will form a 2° carbocation. The 2°
carbocation rearranges to more stable 3° carbocation through Hydride shift. The
3° carbocation undergoes loss of proton from β-carbon to form an alkene as the
final product. Since two β-carbons are available, loss of β-hydrogen results in the
formation of two alkenes. As per Saytzeff’s rule, a more substituted alkene will
be formed as a major product.
Example 3. Carry out the following conversions:
(i) Butan-1-ol to But-2-ene
(ii) Propan-2-ol to 2,3-Dimethylbutane
(iii) 2-Methylpropan-1-ol to 2-Methylpropan-2-ol
Solution

Example 4. Prepare the following alcohols using propene as the intial reactant:
(i) Pent-4-en-2-ol
(ii) Allyl alcohol (Prop-2-en-1-ol)
Solution
Example 5. Complete the following sequence of reactions:

Solution

Example 6. Write isomeric alcohols with molecular formula C4H10O. Which


isomer(s) will give a positive iodoform test?
Solution. The isomeric alcohols (C4H10O) have the following structures:
Only butan-2-ol will give positive iodoform test (refer p. 472).
Example 7. Which of the following two reactions will occur readily and why?

Solution. The formation of bromoethane from ethanol [reaction (ii)] occurs


readily compared to reaction of ethanol with HCl [reaction (i)] to form
chloroethane. This is because Br– is a stronger nucleophile compared to Cl– and
thus attacks readily to form the substituted product (refer
pp. 403–04).
Example 8. Carry out the following conversions:
(i) Propan-1-ol to Propan-2-ol (1° alcohol ↔ 2° alcohol with same number of
carbons)
(ii) Propan-1-ol to Butan-2-ol (1° alchohol ↔ 2° alcohol with one carbon
more)
(iii) Propan-1-ol to 2-Methylbutan-2-ol (1° alcohol ↔ 3° alcohol)
Solution
Example 9. Carry out the conversion of given 2° alcohols to the mentioned 1°
and 3° alcohols.
(i) Propan-2-ol to Propan-1-ol
(ii) Propan-2-ol to 2-Methylpropan-1-ol
(iii) Butan-2-ol to 2-Methylbutan-2-ol
(iv) Butan-2-ol to 2-Ethylbutan-2-ol
Solution (i) This is conversion of 2° alcohol to 1° alcohol with same number of
carbon atoms.
This method is used for the conversion of a 2° alcohol to a 1° alcohol with one
more number of carbon atom.
(iii) For the conversion of a 2° alcohol to 3° alcohol with higher number of
carbon atoms, the alcohol is oxidized to ketone which is followed by
reaction with appropriate Grignard reagent.

(iv) Same as in (iii) except that Grignard reagent used is CH3CH2MgI.

Example 10. How will you convert tert. butyl alcohol to 2-Methylpropan-1-ol.
Solution. The conversion of 3° alcohol to 1° alcohol can be carried out as
follows:
EXERCISES
1. Give the IUPAC name for the following alcohols:

2. How many isomeric alcohols are possible for C4H10O. Write the structural
formula and IUPAC names for all the isomeric alcohols.
3. Explain, why alcohols have higher boiling points than alkanes and
haloalkanes of comparable mass.
4. How is ethanol manufactured from molasses or starch?
5. Give the product(s) formed when propan-1-ol reacts with following:
(a) Na
(b) PCl5
(c) Conc. H2SO4
(d) SOCl2
(e) K2Cr2O7/H+
(f) Conc. HBr
6. What are the different products obtained by the treatment of ethanol with
concentrated H2SO4 at 110°, 140°, and 170°C?
7. Give the oxidation products of 1°, 2°, and 3° alcohols.
8. Discuss the acid catalyzed mechanism for the dehydration of alcohol.
9. What happens when:
(a) Ethanol is treated with iodine in a strong alkaline medium.
(b) tert. butanol is passed over the heated copper at 300°C.
(c) Ethanol is treated with ethanoic acid in presence of conc. H2SO4.
10. How are methanol and ethanol manufactured commercially?
11. Complete the following sequence of reactions.
12. Which of the alcohol will give the positive iodoform test?

[Hint: Look for presence of CH3CH(OH)– group.]


13. Carry out the following conversions in a single step:
(a) Propene to propan-1-ol
(b) But-1-ene to butan-1-ol
(c) Propan-1-ol to propanal
(d) Tert. butyl alcohol to 2-methylpropene
14. What is Lucas reagent? How is it used for identification of 1°, 2°, and 3°
alcohols?
15. What is the function of anhydrous ZnCl2 in Lucas reagent?
16. Giving reasons, arrange the following alcohols in their increasing order of
reactivity with HBr.
(i) CH3CH2CH2OH,
(ii) (CH3)3COH,
(iii) (CH3)2CHOH.
17. Are alcohols stronger or weaker acids than water? Explain.
18. Using CH3Li how will you prepare isopropyl alcohol and n-propyl alcohol?
19. Giving an example explain the internal nucleophilic substitution (SNi) in
alcohols?
20. Name the products formed by the following reactions:
Chapter14
Ethers, Epoxides,
Thiols and Thioethers

A. ETHERS
14A.1 INTRODUCTION
Ethers belong to class of oxygen containing organic compounds that can be
considered as derivatives of alcohols where an alkyl or aryl group replaces
hydroxy hydrogen. Structurally they may be considered as dialkyl (or aryl)
derivatives of water. Ethers are isomeric with alcohols but like alcohols they do
not posses replaceable active hydrogen. They may also be regarded as
anhydrides of alcohols as they can be derived by the elimination of water
molecule from two alcohol molecules. If the two groups attached to oxygen are
same in ethers, they are known as symmetrical ethers however if two groups are
different, they are termed as mixed or unsymmetrical ethers. Ethers are further
classified as open chain, cyclic, saturated, unsaturated, and aromatic ethers
depending on the nature of groups attached to oxygen.

Most of the ethers are known by their common names where while naming, the
alkyl or aryl groups attached to oxygen are prefixed before the word ether.
Cyclic ethers are named as oxides or epoxy compounds. In the IUPAC system,
ethers (refer Chapter 2) are considered as alkoxy derivatives of alkanes and
named accordingly as alkoxyalkanes. In case of unsymmetrical ethers, larger
alkyl group is chosen as parent hydrocarbon and the smaller alkyl group is
considered as alkoxy group. The IUPAC names of few ethers are as follows
(with the common names written in parentheses).
Isomerism
Ethers exhibit functional isomerism and metamerism.
(i) Functional isomerism. Ethers are isomeric with alcohols as both have same
general formula (CnH2n+2O).
(ii) Metamerism. Ethers having four or more number of carbons exhibit
metamerism because with same molecular formula they can have different
alkyl groups attached to oxygen atom. Thus, the molecular formula
C4H10O represents three isomeric ethers or metamers as

14A.1.1 Physical Properties


Lower members such as methoxymethane and methoxyethane are gases while
higher members are low boiling, volatile liquids. Ethers have lower density than
water and are polar molecules. Their boiling points increase with an increase in
the molecular mass. The boiling points and density values at 20°C are listed in
Table 14.1 for some ethers.
The ether molecules are not associated through hydrogen bonding due to which
their boiling points are much lower than isomeric alcohols (refer Table 14.2).
Ethers, are readily soluble in organic solvents.
14A.2 PREPARATION OF ETHERS

14A.2.1 From Alcohols


By dehydration
Heating alcohol with concentrated sulfuric acid or phosphoric acid results in
condensation of two molecules of alcohol with elimination of water molecule to
form ether. Lower members of ether family are obtained industrially by this
method.

Reaction is catalyzed by H+ ions to from oxonium ion that undergoes


substitution (SN1 or SN2) by another molecule of alcohol to form ether.

By the action of diazomethane


Alcohols on reaction with diazomethane result in the formation of methyl ethers.
The reaction is carried out in the presence of a catalyst such as boron trifluoride.
For example,
14A.2.2 From Halo Compounds
Williamson ether synthesis
Williamson synthesis is the most important method for laboratory preparation of
ethers and involves heating a haloalkane with sodium or potassium alkoxide.

The reaction occurs by SN2 mechanism and involves the attack of alkoxide ion
(a nucleophile) on polar carbon–halogen bond of haloalkane. The reaction is
most suitable for preparing mixed or unsymmetrical ethers.

In a similar manner in case of haloalcohols, intramolecular Williamson


synthesis, occurs in the presence of hydroxide ions to produce cyclic ethers.

In the Willamson synthesis, reaction of alkoxide ion with 1°


haloalkanes results in the formation of ethers in good yield whereas,
2° and 3° haloalkanes form alkenes predominantly. The formation of
alkene instead of ether may be well understood as alkoxides being
strong bases abstract β-proton from haloalkane that causes
elimination (dehydrohalogenation) in the 2° and 3° haloalkanes
preferably over the substitution.
To illustrate this fact, synthesis of 2-methoxy-2-methylpropane can be
carried out by two possible routes (I and II) though route II is
preferred for the reasons mentioned above. The reactions that take
place are as follows:

A modified procedure to avoid elimination, while using 3°


haloalkane, involves the reaction of alcohol and haloalkane in
presence of silver oxide.

Heating with dry silver oxide


Ethers may be prepared through nucleophilic substitution reactions by heating
haloalkanes with dry silver oxide (Ag2O).

Reaction of halogenated ethers with Grignard reagent


Grignard reagent reacts with haloalkanes to form the carbon–carbon bond. In a
similar way, it reacts with substituted haloalkanes such as halogenated ether to
form higher analogues of ether.

14A.2.3 Alkoxymercuration–demercuration of
Alkenes
The reaction of alkenes with mercuric trifluoroacetate and alcohol in the
presence tetrahydrofuran as solvent results in the formation of alkoxyalkyl
mercury compounds (alkoxy mercuration). The reduction of these compounds
with sodium borohydride (demercuration) results in the formation of ether.

The mechanism is similar to oxymercuration–demercuration of alkenes used for


preparation of alcohol (p. 230).
14A.3 CHEMICAL PROPERTIES OF ETHERS
Ethers are more or less inert in nature. Ethers do not get oxidized or reduced
easily and show inert behaviour towards alkalis, dilute acids, metals,
phosphorous halides, and so on in cold. However, ethers undergo a few
characteristic reactions. Some of the reactions of ethers are similar to those of
alcohols but the ethers are less reactive than alcohols since they do not have the
active hydrogen attached to oxygen as present in alcohols. The chemical
reactions of ethers can be classified as discussed follows:

14A.3.1 Reactions due to Etheral Oxygen


Due to the presence of two unshared lone pairs of electrons, oxygen readily
coordinates with electron deficient molecules or Lewis acids. The reactions due
to etheral oxygen are:
Formation of oxonium salts
Ethers dissolve in strong mineral acids due to the formation of oxonium salts.
Due to the presence of lone pair of electrons on etheral oxygen, it reacts with
proton of an acid and forms oxonium salts, which are stable at low temperature.
The oxonium ions give back ether in aqueous medium.

Ethers behave as Lewis bases and form coordination complexes with Lewis
acids, such as BF3, AlCl3, and the like, popularly known as etherates. Because
of the ability of etheral oxygen to coordinate with the electron deficient species,
they are used as solvents in preparations of organaometallic compounds like
Grignard reagent.
Formation of peroxides
On exposure to air in the presence of light, ethers slowly undergo spontaneous
auto-oxidation and form peroxide derivatives.

These peroxides may cause violent explosion during


distillation of ethers. The peroxides are destroyed by addition of Fe(II)
salts.
The presence of peroxides in ether is detected by shaking ether
solution with ferrous ammonium sulfate and potassium thiocyanate.
The appearance of a red colour (due to the formation of
ferrithiocyanate complex) indicates the presence of peroxides.

14A.3.2 Reactions Involving Ether Linkage


[Cleavage of Carbon–Oxygen Bond]
The stability of carbon–oxygen bond is less than the carbon–carbon bond. Thus,
the cleavage of carbon–oxygen bond occurs more readily in the reactions of
ether as discussed below:
Hydrolysis
Ethers on treatment with steam result in the cleavage of carbon–oxygen bond to
form corresponding alcohols.

Action of sulfuric acid


Ethers on reaction with hot sulfuric acid, form alcohol and alkyl hydrogen
sulfate.

Action of hydrobromic or hydroiodic acid


Ethers react with hydrobromic or hydroiodic acids to form an alcohol and
corresponding haloalkane. However with hot halogen acids, only haloalkanes are
formed. This is because as soon as alcohol is formed, it reacts further with
hydrobromic acid or hydroiodic acid and gets converted into corresponding
haloalkane.
The order of reactivity of halogen acids is HI > HBr > HCl

In case of mixed ethers, halogen attaches itself to smaller of the two alkyl groups
of ether. For example,
This can be explained on the basis of the fact that halide ion (a nucleophile) will
attack readily on a less hindered carbon with high electrophilic character. The
overall mechanism for the reaction of mixed ether with HI is as follows:

ZEISEL’S METHOD—A method for detection and


estimation of alkoxy groups.
The reaction of ethers with hydroiodic acid forms the basis of Zeisel’s
method for the detection and estimation of alkoxy group in a
compound. A known weight of ether is heated with approximately
57% HI. The alkyl halide so formed is volatile and gets absorbed in an
alcoholic solution of AgNO3. Reaction of alkyl halide with silver
nitrate gives the precipitate of silver iodide, which is then washed,
dried, and weighed. The quantity of AgI formed corresponds to the
amount of alkyl halide, which is equivalent to the number of alkoxy
groups present. Thus from the amount of silver iodide, the number of
alkoxy groups can be estimated.
Action of phosphorous pentachloride
With hot phosphorous pentachloride, cleavage of carbon–oxygen bond occurs to
yield chloroalkanes.

Action of acid derivatives


Acid chlorides or acid anhydrides react with ethers in the presence of catalysts
such as
anhydrous aluminium chloride or zinc chloride to form esters and haloalkanes.
The reactions are as folllows:

14A.3.3 Other Reactions


Other reactions of ehters include substitution at alkyl group, dehydration
reaction, and reaction with carbon monoxide as discussed below:
Halogenation (substitution at alkyl groups)
Alkyl groups attached to etheral oxygen undergo substitution reactions similar to
alkanes.
Reaction of ethers with chlorine or bromine in the absence of sunlight, results in
the formation of halogen-substituted ethers. The hydrogen at α-carbon (carbons
attached to oxygen) is displaced most easily. However in presence of sunlight,
all the hydrogens are replaced by halogens to form perhalo ethers.
Dehydration
The ether vapours when passed over heated alumina undergo dehydration to
form alkenes.
Reaction with carbon monoxide
Ethers react with carbon monoxide under high temperature and pressure
conditions to form
esters.
These reactions are summarized as follows taking the example of ethoxyethane
(diethyl
ether).
14A.3.4 COMMON USES OF ETHERS
Ethers find a variety of applications in day-to-day life. Methoxymethane
(dimethyl ether) and ethoxyethane (diethyl ether) are commercially important
ethers. These are synthesized industrially from corresponding alcohols methanol
and ethanol respectively in presence of concentrated sulfuric acid.
(1) Dimethyl ether is a gas (b.p. –24.5°C) and is used mainly in the form of
compressed liquid as a refrigerant and for food storage by freezing on direct
contact, as it leaves no undesirable taste or smell.
(2) Diethyl ether is used extensively as a solvent for the extraction of organic
compounds because it is immiscible with water. Being an inert solvent, it is
a very useful solvent for the reaction of organometallic compounds like
Grignard reagent.
14A.4 CROWN ETHERS
The large cyclic polyethers are termed crown ethers. The name crown is given
due to the shape of these macrocyclic molecules that resembles that of crown.
Nomenclature. Instead of the actual name of these macrocyclic polyethers we
designate them in a simplified way as [x] crown-y , where x = total number of
atoms in the molecule and y = number of oxygen atoms in the molecule. For
example,

Other hetero atoms may replace the oxygen in the crown ethers. In 1987,
Pederson, Cram and Lehn received Nobel Prize for development of crown
ethers. The crown ethers have the unique property of forming strong complexes
with metal ions. These complexes are known as Cryptates and crown ethers
themselves are known as Cryptands. The complex exhibit a Host-Guest
relationship where crown ether plays the part of host for its guest that is metal
ion. The ability to bind the metal ion depends on the size of crown ether and that
of metal ion. For example,
[12]-crown-4 binds Li+ strongly (in other words Li+ forms strong cryptate
with
[12]-crown-4)
[15]-crown-5 binds Na+ strongly (in other words Na+ forms strong cryptate
with
[15]-crown-5)
[18]-crown-6 binds K+ strongly (in other words K+ forms strong cryptate
with
[18]-crown-6)
Because of their property to bind with metal ions they are used as phase transfer
catalysts. The ionic reagents do not dissolve in organic solvents. To carry out the
reaction of organic substrate more effectively with ionic reagents, in a
homogeneous medium, the crown ethers are used. Crown ethers basically behave
as a phase transfer catalysts, which form complex with metal ions and transport
the anion from aqueous phase to organic phase as an ion pair. The anion reacts
with the organic substrate more efficiently in this process.

In similar way, KMnO4 dissolves in water but is insoluble in benzene. If we add


[18]crown–6 ether in benzene the crown ether will form strong complex with K+
ions and will enable the
dissolution of KMnO4 in benzene. The resultant benzene with high
concentration of KMnO4 dissolved in it, is termed as purple benzene and is
useful for the oxidation of alkene in anhydrous organic solvents.
Several drugs also transport selectively the K+ ions (in presence of Na+) across
the cell membrane, as they resemble in their structure with crown ethers and
forms cryptates selectively with K+ ions only.

B. EPOXIDES
14B.1 INTRODUCTION
In general, cyclic ethers with three membered heterocyclic ring (one of the
carbon is replaced by oxygen, hetero atom) are known as epoxides. Carbons and
oxygen are sp3 hybridized in epoxides. These three membered cyclic ethers are
also known as ethylene oxides or oxiranes. Another name of cyclic ethers is
oxacycloalkanes where oxa- indicates replacement of a CH2 group in the ring by
oxygen. Commonly, the cyclic ethers are referred to as epoxyalkanes. A few
examples of epoxides are:

Ethylene oxide is a colourless gas under ordinary conditions (boiling point


10.7°C), soluble in organic solvents. Ethylene oxide finds its use as a fumigant
and insecticide for storing foodgrains. It is also used in the manufacture of non-
ionic surface-active agents.
14B.2 PREPARATION OF EPOXIDES
Some methods of preparation of epoxides are discussed in this section.
From ethene
Heating ethene and oxygen under pressure in the presence of silver catalyst
between 200–400°C results in the formation of ethylene oxide.

From ethylene glycol


Ethylene glycol on heating at 500°C undergoes dehydration to yield ethylene
oxide as follows:

Industrial preparation from 2-Chloroethanol


Industrially, ethylene oxide is prepared from 2-Chloroethanol (ethylene
chlorohydrin) by treating it with concentrated sodium hydroxide solution.

The reaction is an intramolecular synthesis and follows internal SN2


displacement, as follows:
14B.3 CHEMICAL PROPERTIES OF EPOXIDES
Epoxides, being cyclic compounds are highly strained (due to angle strain) and
therefore show a tendency to open up. These compounds undergo a number of
ring opening reactions on addition of various regents. Thus compounds like
water, alcohols, acids, bases, and so on give addition products with ethylene
oxide. Most of the addition reactions of ethylene oxide give bifunctional
compounds, which are widely used in organic synthesis.

14B.3.1 General Mechanism for Ring Opening in


Epoxides
Most of the ‘ring opening reactions’ are acid or base catalyzed and occur in the
presence of electrophiles or nucleophile. The generalized acid and base
catalyzed ring opening in the presence of a nucleophile (Nu–) can be depicted as
follows:
Acid catalyzed ring opening:

Base catalyzed ring opening:

Some of these reactions with ethylene oxide are enlisted here.


(a) Hydrolysis in acidic medium—formation of ethylene glycol
(b) Reaction with halogen acids—formation of ethylene halohydrins
(c) Reaction with hydrogen cyanide—formation of ethylene cyanohydrin
(d) Reaction with organic acids—formation of glycol monoacetate
(e) Reaction with alcohols—formation of glycol monoalkyl ether
(f) Reaction with ammonia and its derivatives—formation of substituted 2-
aminoethanol
(g) Reaction with Grignard reagent or alkyl lithiums followed by hydrolysis—
formation of 1° alcohols
(h) Reaction with hydrogen—formation of ethanol
(i) Action of heat—formation of ethanal
These reactions can be summarized as follows:

14B.3.2 Ring Opening in Unsymmetrical Epoxides


In case of asymmetrical epoxides, there are two sites at which ring opening can
take place. These are—
(a) In acid catalyzed reactions, the attack of nucleophile occurs at a more
hindered carbon.
(b) In base catalyzed reactions, the attack of nucleophile occurs at a less
hindered
carbon.

The general mechanism for acid catalyzed and base catalyzed ring opening
reactions are as
follows:
Acid catalyzed ring opening in unsymmetrical epoxides:

Base catalyzed ring opening in unsymmetrical epoxides:


C. THIOLS AND THIOETHERS
14C.1 GENERAL INTRODUCTION—SULFUR
COMPOUNDS IN ORGANIC CHEMISTRY
Sulfur and oxygen are members of the same group in the periodic table and the
chemistry of sulfur and oxygen compounds is similar in many respects. A series
of organic compounds of sulfur are known which behave in a manner analogous
to oxygen compounds. The C–S bond is weaker compared to C–O bond (as the
electronegativity of sulfur is less then oxygen) which attributes to low reactivity
of sulfur compounds compared to analogous oxygen compounds. The structure
of sulfur compounds, linked through single bonds, is more or less similar to
those of the oxygen analogues.
The organic compounds of sulfur are written in a similar way as the
corresponding oxygen compounds by replacing oxygen with sulfur. The sulfar
organic compounds are named by prefixing thio before the name of the
corresponding oxygen compound. Various classes of sulfur compounds are

This chapter discusses two most important classes of sulfur compounds, namely
thiols (or thioalcohols) and thioethers.
14C.2 THIOLS [MERCAPTANS]
Thiols may be considered as alkyl derivatives of hydrogen sulfide, are analogous
to alcohols, and thus termed as thioalcohols. Since they react with mercuric
oxides to form mercuric salts they are also called alkyl mercaptans (mercurium =
mecury; captans = catching) or simply mercaptans. The
–SH group is known as sulfhydral or mercapto group. The IUPAC nomenclature
of thioalcohols is done by suffixing thiol to the parent alkane with the lowest
possible numbered position for thiol. Thus, the name becomes alkanethiol. Few
examples of thiols are given here.

14C.2.1 Physical Properties


In thiols, the S–H bond is less polar compared to O–H bond in alcohols. Because
of large size of sulfur and low polarity of the –SH bond, thiols do not form
strong hydrogen bonds. Thus, thiols have lower boiling points compared to
corresponding alcohols. Methanethiol is a gas while higher thiols are volatile
liquids having unpleasant odour. The boiling point of methanethiol is 6°C and
that of ethane thiol is 37°C.
14C.3 PREPARATION OF THIOLS
General methods of preparation of thiols are as follows:
From haloalkanes
Haloalkanes on reaction with excess sodium or potassium hydrogensulfide in
alcoholic medium undergo nucleophilic substitution reaction to form
corresponding thiols.

Why is NaSH or KSH used in excess?


If NaSH or KSH is not used in excess, the thiol so formed may react
with initial haloalkane to form thioethers (dialkyl sulfide) as the final
product.

From alcohols
Thiols can be prepared from alcohols either by heating with phosphorous
pentasulfide or by the reaction with hydrogen sulfide in the presence of a
catalyst.

From alkyldisulfides
The reduction of alkyldisulfides with zinc in the presence of acid results in the
formation of thiols as
From Grignard reagent
Reaction of sulfur with Grignard reagent forms an adduct, which on subsequent
hydrolysis yields corresponding thiol.
14C.4 CHEMICAL PROPERTIES OF THIOLS
The low polarity of –SH bond makes it a weak bond, which contributes to high
acidic character of thiols compared to alcohols. The alkane thiolate ion (RS–)
formed by loss of a proton is stabilized to a greater extent due to delocalization
of negative charge over larger sulfur atom. Thus, thiols are stronger acids
compared to alcohols. In their reactions, the thiols are more nucleophilic
compared to alcohols.
Reaction with alkali metals and alkali
Thiols on reaction with alkali metals forms mercaptides with the evolution of
hydrogen.
Thiols behave as weak acids and thus dissolve readily in alkalis also to form
mercaptides. For example,

Reactions with metal salts and metallic oxides


Thiols react with metal oxides and other metal salts to form the following
products.

Reaction with acids and acid derivatives


Thiols react with acids, acid chlorides, acid anhydrides, and so on to form
thioesters.
Reaction with aldehydes and ketones
Like alcohols, thiols also react with aldehydes and ketones to form thioacetals
and thioketals respectively in the presence of hydrochloric acid. For example,

Oxidation
Thiols are oxidized to disulfides with mild oxidizing agents such as hydrogen
peroxide and, sodium hypochlorite. But with strong oxidizing agents like nitric
acid or potassium permanganate, they are oxidized to corresponding sulfonic
acids.
14C.5 THIOETHERS [ALKYL SULFIDES]
Thioethers are the sulfur analogues of ethers. The C–S–C bond angle in thioether
is nearly 105°. In IUPAC nomenclature, symmetrical thioethers are named as
dialkyl sulfides whereas in unsymmetrical thioethers, both the alkyl groups are
written before the suffix sulfide.

Thioethers are colourless, unpleasant smelling, volatile liquids. In general, their


boiling points are more than those of corresponding ethers. The boiling point of
dimethyl thioether is 38°C and that of dimethyl thioether is 92°C. They are
insoluble in water but soluble in organic solvents.
14C.6 PREPARATION OF THIOETHERS
General methods of preparations for thioethers are as follows:
From haloalkanes
Haloalkanes on heating with potassium sulfide or sodium mercaptide result in
the formation of thioethers.

From ethers
Ethers on reaction with phosphorous pentasulfide form thioethers.

From thiols
(i) Thioethers are formed when vapours of thioalcohols are passed over heated
catalysts at 300°C.
(ii) The addition of thiols to an olefin in the presence of peroxide produces
thioethers. For example,

From Grignard reagent


The reaction of Grignard reagent with sulfur followed by reaction with
haloalkane results in the formation of a thioether.
14C.7 CHEMICAL PROPERTIES OF
THIOETHERS
Sulfur in thioethers is more nucleophilic compared to oxygen in ethers and
hence, thioethers undergo reactions more readily than ethers.
Reaction with halogens
Thioethers form addition compounds with halogens to yield corresponding
thioether dihalides. The reaction proceeds preferably by free radical mechanism.
Reaction with haloalkanes
The reaction of haloalkane with thioether results in the formation of sulfonium
salt which is similar to oxonium salt formed by ethers. The sulfonium halides
when treated with moist silver oxide form sulfonium hydroxides which on
heating decompose to give thioethers and alkenes.
Reaction with metal salts
Thioethers form insoluble coordination compounds with metal salts.
Hydrolysis
Thioethers on boiling with alkali get hydrolyzed to alcohols.
Oxidation
Mild oxidizing agents such as hydrogen peroxide, chlorine, water, and so on
oxidize thioethers to sulfoxides which on further oxidation give sulfones. The
strong oxidizing agents such as concentrated nitric acid or potassium
permanganate directly oxidize thioethers to sulfones.
All these reactions can be summarized as follows:
MUSTARD GAS—A Note
Mustard gas, a thioether, is not a gas but an oily liquid (boiling point
217°C) which smells like mustard. It is a highly poisonous compound
and was used in the first world war as a chemical weapon, in the
vapour form, by sprinkling through bombing. Its vapours get absorbed
through skin and cause blisters, itching, and ulceration. Since it was
used in the vapour form, it was referred to as a gas. This chemical
weapon was used in recent times in Iran–Iraq war.
Chemically, mustard gas is 2,2’-dichlorodiethylsulfide and is prepared
from ethene as follows:
EXERCISES
1. Give IUPAC names for the following compounds:

2. Write the structures and names for all the isomeric compounds with molecular
formula C5H12O.
3. Why do ethers have low boiling points compare to isomeric alcohols?
4. Explain, with mechanism, how can ethers be synthesized by following
methods.
(a) Williamson synthesis
(b) Alkoxymercuration–demercuration of alkenes
5. Why are 3° haloalkanes not used in Williamson ether synthesis?
6. How can Grignard reagent be used for the preparation of higher ethers?
7. Why are ethers less reactive compared to alcohols?
8. Why are ethers stored in dark coloured bottles?
9. How are peroxides formed in ethers solution destroyed?
10. What will be the product formed in the following reaction.

11. Why, in the reaction of HBr and HI with unsymmetrical ethers, does the
smaller group form the haloalkane?
12. How will you convert methoxymethane to (i) methanol, (ii) methylacetate,
and (iii) bromomethane?
13. What is Zeisel’s method for the estimation of alkoxy groups?
14. Following methods are basically the methods used for the preparations of
some commercially important ethers. Complete the reactions and identify the
ether formed in each case.
15. Complete the following reactions:

16. What is the product formed when bromoethane reacts with excess of sodium
hydrogen sulfide?
17. Why do thiols have lower boiling points compared to alcohols?
18. Compare the acidic strength of alcohols with thiols.
19. How can diethyl sulfide (C2H5SC2H5) be prepared from a Grignard
reagent?
20. What are the products obtained by oxidation of (a) thiols and (b) thioethers,
with mild oxidizing agents?
Chapter15
Polyhydric Alcohols
(Diols and Triols)

15.1 INTRODUCTION
Polyhydric alcohols contain two or more hydroxy groups present on different
carbons of the hydrocarbon chain. Depending upon the number of hydroxy
groups they are termed as di-, tri-, tetra-, or pentahydric alcohols. The
compounds that contain two hydroxy groups attached to the same carbon atom
are highly unstable and readily eliminate a water molecule to form more stable
carbonyl compounds.
In polyhydric alcohols, the relative position of the –OH group in the molecule
is designated by 1,2-, 1,3-, or 1,2,3- and so on. In IUPAC nomenclature system,
the longest carbon chain containing –OH groups is selected and the hydroxy
groups get the minimum possible number. The dihydric alcohols are termed as
alkanediols (commonly known as glycols) and in a similar manner trihydric
alcohols are called alkanetriols (refer Chapter 2). A few examples of polyhydric
alcohols are as follows:

Dihydric alcohols exhibit positional and functional isomerism. Thus, a glycol


having molecular formula C3H8O2, represents the following four isomeric
compounds:
15.1.1 Physical Properties
The presence of two or more hydroxy groups increases the number of sites
through which hydrogen bonding can take place. Thus, diols and triols exhibit
much stronger intermolecular hydrogen bonding compared to monohydric
alcohol, which is reflected by high boiling point of polyhydric alcohols. For the
same reason, the diols and triols are viscous liquids. The increased
intermolecular hydrogen bonding of diols and triols with water explains the high
solubility of these compounds in water. Table 15.1 enlists the boiling points of
some polyhydric alcohols.

In the present chapter we shall discuss dihydric and trihydric alcohols taking
Ethylene glycol (ethane-1,2-diol ), pinacols, and glycerol (propane-1,2,3-triol) as
the representative examples of this family of compounds.
15.2 ETHYLENE GLYCOL [ETHANE-1,2-DIOL]
It is the first member of the series of alkanediols and is also known as glycol.
Ethylene glycol is a colourless, viscous liquid (boiling point 197°C). It is
hygroscopic in nature and is miscible with water and ethanol in all proportions.
It is sweet in taste but a toxic compound. It forms low freezing mixture with
water.

15.2.1 Preparations
Ethylene glycol may be prepared by the following methods:
From ethene
There are three different methods by which ethylene glycol (ethane 1,2-diol) can
be synthesized from ethene.
(i) Oxidation. The reaction of ethene with cold, dilute alkaline KMnO4 solution
or osmium tetraoxide (OsO4) results in cis-hydroxylation to form ethane-1,2-
diol as the final product.

(ii) Epoxidation of ethene followed by hydrolysis. Heating ethene with oxygen


in the presence of silver catalyst under pressure results in the formation of
epoxide. The acidic hydrolysis of epoxide then gives ethylene glycol.

(iii) Via the formation of 2-chloroethanol (chlorohydrin) followed by


hydrolysis. The reaction of ethene with chlorine and water results in the
formation of chlorohydrin which on hydrolysis with aqueous sodium carbonate
gives ethylene glycol. This is a commercial method for the preparation of
ethylene glycol.
From 1,2-dihaloalkane (Vicinal dihalide)
The hydrolysis of 1,2-dihaloethane with the sodium carbonate solution results in
the formation of ethylene glycol. It is a laboratory method used for the
preparation of ethylene glycol and the yields in this case are generally poor.

What if NaOH or KOH solutions are used in the reactions


discussed so far instead of Na2CO3 solution for hydrolysis?
Hydroxides of sodium or potassium are stronger bases than carbonates
of sodium or potassium. In presence of a strong alkali, chlorohydrin
undergoes ring closure through nucleophilic displacement, eventually
forming an epoxide.

Further, 1,2-dihaloethane (a vicinal dihalide) may result in the


formation of vinyl halide as a side product through elimination
process, if strong bases are used. On the other hand, the use of sodium
carbonate, a mild base, results in the formation of pure ethylene
glycol.

To obtain a good yield of ethylene glycol, 1,2-dihaloethane is treated with


sodium acetate which results in the formation of glycol acetate in a quantitative
yield. Glycol acetate on hydrolysis with aqueous sodium hydroxide results in the
formation of ethylene glycol.

By reduction of oxalic ester, glyoxal, or glycolic aldehyde


Reduction of oxalic ester, glyoxal, or glycolic aldehyde with sodium and ethanol
gives ethylene glycol.

From ethane-1,2-diamine
In general, reaction of aliphatic amines with nitrous acid results in the formation
of corresponding alcohol. By a similar analogy, the reaction of ethane-1,2-
diamine gives ethylene glycol.

15.2.2 Chemical Properties


The chemical reactions of ethylene glycol are similar to those of monohydric
alcohols but the reagent is required in double the amount, as it has to act over
two hydroxyl groups. However in most of the reactions, both these groups are
not equally reactive. Thus, the reaction of one hydroxyl group is completed
before the other one participates in the reaction. Some of the important reactions
are discussed in this section.
Reaction with sodium
Unlike monohydric alcohols, ethylene glycol reacts slowly with sodium and at
about 50°C, it forms monosodium glycolate by the replacement of hydrogen
(indicates the acidic nature of ethylene glycol). However, the replacement of
hydrogen from second –OH group is rather difficult and requires higher
temperature to result in the formation of disodium glycolate.

Why is the replacement of hydrogen from the second


hydroxyl group difficult?
This is because once alkoxide is formed by the replacement of one
hydrogen, it behaves like an electron releasing group which makes the
O–H bond more stronger and thereby reducing its reactivity towards
metals.

Reaction with inorganic acids


Ethylene glycol reacts with inorganic acids to give corresponding products.
(i) With hydrogen chloride. Reaction of ethylene glycol with hydrogen chloride
at 160°C results in the formation of 2-chloroethanol (ethylene chlorohydrin).
However at higher temperature
(~ 200°C), it results in the formation of 1,2-dichloroethane.

Why is the replacement of second hydroxyl group difficult


and requires higher temperature?
This reaction is a nucleophilic substitution reaction where –OH is
replaced by chlorine. In acidic medium the first step is protonation of
–OH to form oxonium ion. Oxonium ion being a good leaving group
is easily replaced by chlorine to form ethanechlorohydrin. Now due to
the strong electron withdrawing nature of chlorine, it exerts –I effect
on the adjacent carbon, which in turn withdraws the electron density
from the oxygen of the second –OH group. This decreases the electron
density on oxygen and reduces its tendency to form an oxonium ion.
Thus, it requires high temperature conditions for replacement
reactions.
(ii) With concentrated nitric acid. On heating ethylene glycol with
concentrated nitric acid in presence of concentrated sulfuric acid (as catalyst), it
results in the formation of ethylene dinitrate. Ethylene dinitrate is also known as
glycol nitrate and is highly explosive in nature.

Reaction with organic acids


Reaction of ethylene glycol with organic acids results in the formation of
corresponding esters.
(i) With monocarboxylic acid. Ethylene glycol reacts with monocarboxylic
acids such as acetic acid to form glycol diacetate as the end product. The
reaction of ethylene glycol with acetic acid derivatives like acetyl chloride or
acetic anhydride also results in the formation of glycol diacetate.

(ii) With dicarboxylic acids. The reaction of ethylene glycol with dicarboxylic
acid generally results in the formation of condensation polymers. For example,
its reaction with terphthalic acid forms a well-known polymer called terylene
(also known as decron or terene), which is extensively used as a fiber.

Reaction with phosphorous halides


Phosphorous pentahalide or phosphorous trihalide react with ethylene glycol to
form
1,2-dihaloethane. In the reaction, halohydrin is formed as an intermediate
product.

Reaction of ethylene glycol with phosphorous triiodide


results in the formation of ethene and not 1,2-diiodoethane.
Initially, like other phosphorous trihalides (chloride and bromide) the
product formed with phosporous triiodide is also 1,2-diiodoethane.
But this product is quite unstable and thus, decomposes to form ethene
as the final product with the elimination of iodine.

Oxidation
Ethane-1,2-diol contains two primary alcoholic groups. The oxidation results in
the formation of different products depending on the nature of the oxidizing
agent.
(i) Oxidation of ethane-1,2-diol with nitric acid results in the formation of
oxalic acid via a series of other oxidation products.
(ii) Ethane-1,2-diol on oxidation with periodic acid, or lead tetraacetate, or
sodium bismuthate results in the carbon–carbon bond cleavage of glycol
producing carbonyl compounds. In case of ethylene glycol, the product
formed is formaldehyde.

(iii) Oxidation of diols with acidic potassium permanganate or potassium


dichromate also results in cleavage of carbon–carbon bond. The product
formed is a carboxylic acid, in case of ethylene glycol, the oxidation
product is formic acid.

Condensation with aldehydes and ketones


Ethylene glycol condenses with aldehydes and ketones in the presence of
mineral acids as catalyst to form cyclic acetals and ketals respectively. The
reaction is used for the protection of carbonyl group (Section 17.3.2, p. 586)

Dehydration
Ethylene glycol undergoes dehydration under different conditions to form
different products. For example:
(i) Heating ethylene glycol up to 500°C results in dehydration to form
ethylene oxide.
(ii) Heating with anhydrous zinc chloride results in the formation of
acetaldehyde.
(ii) Two molecules of ethylene glycol condense in the presence of
concentrated H2SO4 to form cyclic ether known as dioxane.

15.2.3 Uses of ethylene glycol


Ethylene glycol is used
(a) as an antifreeze in automobile radiators because it forms low freezing
mixture with water.
(b) for preventing deposition and formation of ice on aeroplane wings.
(c) as a plasticizer for viscose rayon and as an intermediate in the manufacture
of polyurethane polymers. It is also used in the manufacture of synthetic
polymer fiber terylene.
(d) in the preparation of several commercially important organic compounds,
for example, dioxane, diethyl glycol (used as a solvent), ethylene dinitrate
or nitroglycol (used as an explosive) and glycol stearate (used as a
lubricant).
15.3 PINACOLS
Pinacol is a general name for tertiary 1,2-diols (glycol). This section discusses,
only in brief, the preparations and chemical properties (other than the common
properties of glycols) of a representative example from the pinacol family,
namely 2,3-dimethylbutane-2,3-diol.

15.3.1 Preparation of 2,3-dimethylbutane-2,3-diol


Ketones on reduction with metals like amalgamated magnesium in inert solvents
(benzene, ether, and so on) result in the formation of di-tertiary 1,2-diols known
as pinacols. The synthesis of
2,3-dimethylbutane-2,3-diol can easily be carried out from acetone in the
following manner.

15.3.2 Pinacol–Pinacolone Rearrangement


In the presence of sulfuric acid (moderately concentrated), the di-tertiary 1,2-
diols (pinacol) rearrange to form a carbonyl compound known as pinacolone.
The reaction is commonly called pinacol–pinacolone rearrangement.

The rearrangement of pinacol to pinacolone is a classical example of acid


catalyzed rearrangement of 1,2-diols.
The mechanism for this rearrangement involves following steps:
Step 1. Protonation of diol
In the presence of an acid, the protonation of one of the hydroxyl group occurs
and results in the formation of oxonium ion.
Step 2. Loss of water molecule and formation of carbocation
This is a slow and the rate determining step.
Step 3. 1,2-migration of methyl group (1,2-methyl shift)
The migration of methyl group occurs to an electron deficient centre, that is,
tertiary carbocation through a bridged intermediate. The migrating group enters
at a position trans (anti-manner) to the leaving group. This results in the
development of positive charge on the carbon bearing the hydroxyl group, which
is further stabilized by resonance.
Step 4. Loss of proton (formation of pinacolone)
The overall mechanism for the pinacol-pinacolone rearrangement is as follows:

The Pinacol–pinacolone rearrangement in pinacols where two different


groups are attached to carbons.
In pinacols where carbon is attached to both alkyl and aryl groups the
rearrangement depends on two factors namely stability of carbocation and
migratory aptitude of the groups.
• In the following example the stability of carbocation governs the formation
of product. In Step 1 of mechanism, the protonation of hydroxyl group
occurs. This is the hydroxyl group which on removal of water molecule
results in the formation of a more stable carbocation (Step 2).

• In this example, the migratory aptitude of the groups governs the formation
of product. In Step 3 of mechanism, the migration of an aryl group is
preferred over an alkyl group. In general the migratory aptitude of different
groups follows the order
–Ph > –C(CH3)3 > –CH2CH3 > –CH3 > H

15.4 GLYCEROL [PROPANE-1,2,3-TRIOL]


Propane-1,2,3-triol is the first member of the series of alkanetriols. It is
commonly known as glycerol. It occurs in a combined state with higher fatty
acids usually palmitic (C15H31COOH), oleic (C17H33COOH), and stearic
(C17H35COOH) acids, in the form of esters known as glycerides. Glycerides are
important constituents of almost all vegetable and animal oils and fats.

15.4.1 Physical Properties


Glycerol is a colourless, odourless, viscous, and hygroscopic liquid (boiling
point 290°C). It decomposes on heating at the boiling point. It is sweet in taste
and miscible with water in all proportions due to the availability of three
hydroxy groups for hydrogen bonding with water. It is insoluble in organic
solvents such as ether, benzene, chloroform, and the like.

15.4.2 Preparations
Glycerol is prepared by the following methods:
From fats and oils
As already mentioned, fats and oils are triesters of glycerol (glycerides).
The hydrolysis of glycerides forms glycerol and fatty acids. The fats and oils are
hydrolyzed either with alkali (in soap industry) or by superheated steam (in
candle industry) and glycerol is obtained as a by-product.
(i) From soap industry. In the soap industry fats and oils are hydrolyzed with
sodium hydroxide and the hydrolysis results in the formation of sodium salt
of fatty acids (known as soap) and glycerol is obtained as a by-product.

Soap is salted out (or precipitated) by adding sodium chloride to the reaction
product. The soap is filtered and the filtrate is known as spent lye. It contains
approximately 3–5% glycerol. Spent lye is then allowed to settle in tanks where
most of the suspended impurities settle down. These impurities are then filtered
off, filtrate is distilled under vacuum and futher purified with activated charcoal.
It is again subjected to vacuum distillation, which gives glycerol of 90–95%
purity. Further distillation under reduced pressure yields pure glycerol (specific
gravity 1.26).
(ii) From candle industry. Higher fatty acids like stearic acid are mixed with
paraffin wax in the manufacture of candles. The fatty acids required for this
purpose are obtained by the hydrolysis of fats and oils with superheated
steam using dilute sulfuric acid as catalyst. The fatty acids are usually solids
and can be removed by filtration. Water is evaporated from the filtrate under
vacuum and further redistillation is done to obtain pure glycerol.
By fermentation of sugars
Glycerol is formed as a by-product during fermentation of sugars to obtain
ethanol. Better yields are obtained by adding small amount of sodium sulfite
along with yeast.

Industrial preparation of glycerol


Glycerol can be prepared industrially by a synthetic method. The synthesis
begins with propene (obtained from petroleum cracking) and proceeds through
the following route.

Alternatively, allyl chloride may be treated with hypochlorous acid to form


glyceryl dichlorohydrin, which on hydrolysis gives glycerol.

15.4.3 Chemical Properties


Glycerol molecule contains two 1° and one 2° alcoholic groups. Usually, 1°
alcoholic groups are more reactive. However, glycerol shows the chemical
reactions for both the types of alcoholic groups.
Reaction with sodium
Glycerol reacts with sodium metal (similar to monohydric alcohols), thus,
exhibiting the acidic nature. It is only the 1° alcohol group, which reacts with
sodium metal to form monosodium glycerolate at room temperature. At a higher
temperature, it forms disodium glycerolate.

Reaction with hydrogen halides


(i) With hydrochloric acid. On passing hydrogen chloride gas through
glycerol at 110°C a mixture of two monochloro-derivatives is formed. If the
reaction with HCl gas continues further, it produces a mixture of two types
of dicholrohydrin derivatives.

The products involving replacement of 2° alcohol group are obtained in a


lesser amount compared to the products obtained by replacement of 1°
alcohol group.
(ii) With hydroiodic acid. Glycerol on reaction with small amounts of
concentrated hydroiodic acid forms 1,2,3-triiodopropane, which is a highly
unstable compound and decomposes to give allyl iodide.
In the presence of excess of hydroiodic acid, allyl iodide forms
diiodopropane which being unstable eliminates a molecule of iodine to form
propene. Propene so formed reacts with excess of hydroiodic acid to form
2-iodopropane.
Reaction with phosphorous halides
Glycerol reacts with phosphorous tri- or pentachloride to form glyceryl
trichloride (1,2,3-trichloropropane). A similar reaction is observed when glycerol
reacts with phosphorous tri- or pentabromide.

However with phosphorous triiodide, it forms unstable glyceryl triiodide, which


decomposes to give allyl iodide. If phosphorous triiodide is present in excess, it
results in the formation of
2-iodopropane (as shown above). This is because phosphorous triiodide reacts
with water to generate hydrogen iodide, which participates in reaction with
allyliodide to form 2-iodopropane as the final product.

With nitric acid


Reaction of glycerol with concentrated nitric acid, in the presence of sulfuric
acid, at low temperature results in the formation of glyceryl trinitrate (also
known as trinitroglycerine, TNG).
Glyceryl trinitrate (an oil) is highly explosive. Glyceryl trinitrate when adsorbed
on keiselguhr (a clay type) is termed as dynamite.
Esterification
Glycerol reacts with acetic anhydride or acetyl chloride to form mono-, di-, and
triesters.

With oxalic acid


Glycerol reacts with oxalic acid under different conditions to give different
products as discussed here.
(i) On heating glycerol at 110°C with excess of oxalic acid, it first forms
glyceryl mono-oxalate, which on decarboxylation gives glyceryl
monoformate that reacts further with water to form formic acid and
glycerol.

(ii) Heating glycerol at 260°C with oxalic acid result in the formation of
glyceryl dioxalate which undergoes decarboxylation at high temperature to
give allyl alcohol as the final product.

With potassium hydrogen sulfate


Glycerol on heating with potassium hydrogen sulfate (KHSO4), undergoes
dehydration with the elimination of two water molecules resulting in the
formation of an unsaturated aldehyde known as acrolein (or acrylic aldehyde).

Oxidation
Glycerol contains two 1° and one 2° alcoholic groups and undergoes oxidation to
yield a variety of products. The type of product formed is dependent on the
nature of oxidizing agent.

Oxidation of glycerol with different oxidizing agents gives the following


products:
(i) With bismuth and sodium nitrate, glycerol gives mesoxalic acid.
(ii) With dilute nitric acid, it yields glyceric acid.
(iii) Concentrated nitric acid oxidizes glycerol to glyceric acid and tartronic
acid.
(iv) Fenton’s reagent (H2O2 + FeSO4) or bromine water (in the presence of
sodium carbonate) results in the mixture of glyceraldehyde and dihydroxy
acetone (glycerose).
(v) Oxidation with solid potassium permanganate results in the formation of
oxalic acid and carbon dioxide. It is an explosive reaction.

Condensation with aldehydes and ketones


Glycerol reacts with aldehydes and ketones in acidic medium to form cyclic
acetals or ketals respectively.
Glycerol is used as a sweetening agent in confectionary, beverages, and
medicines. It is also used in the preparation of good quality soaps and cosmetics,
non-dry inks, printing inks, and stamp-pad inks. Further, it is also used for the
synthesis of some useful organic compounds such as formic acid, allyl alcohol,
acrolein, and so on.
EXERCISES
1. How is ethylene glycol prepared commercially?
2. Starting from the following compounds, give the sequence of reactions to
prepare Glycerol.
(a) Fats or oils
(b) Propene (via allyl alcohol)
(c) Acetone
3. Why are polyhydric alcohols generally viscous liquids?
4. Why is the hydrolysis, during the preparation of alkane diols from 1,2-
dihaloalkanes, carried out with mild alkali like Na2CO3 and not with NaOH?
5. Why does, in ethylene glycol, the replacement of first –OH group occur more
readily compared to replacement of second –OH group during reaction with
HCl?
6. Complete the following reactions.

7. What oxidation products will be obtained from following reactions of diols?

8. Starting from glycerol, how can one synthesize:


(a) Allyl alcohol, (b) Nitroglycerine, (c) Acrolein, and (d) 2-lodopropane
9. What are pinacols? Explain, with mechanism, the pinacol–pinoacolone
rearrangement.
10. Carry out the following conversions:
(a) Butan-1-ol to Butane-1,2-diol.
(b) Ethane-1,2-diol to Dioxane.
(c) Glycerol to Formic acid.
(d) Glycerol to Glyceric acid.
11. Complete the following oxidation reactions:
Chapter16
Phenols

16.1 INTRODUCTION
The class of organic compounds where hydoxy group (–OH) is directly attached
to benzene ring is termed as phenols. The carbon–oxygen bond in phenols is
formed by sp2–sp3 overlap. Most of the phenolic compounds are better known
by their common name. The IUPAC names of some phenolic compounds along
with their common name and physical characteristics (boiling and melting
points) are as follows:
16.1.1 Physical Properties
Phenols are generally colourless solids at room temperature. On exposure to air,
phenol becomes reddish in colour. Phenols are polar in nature and form strong
hydrogen bonding, which is reflected by high boiling point compared to
hydrocarbons of same molecular mass.
The substitution of phenol with amino, nitro, or hydroxy groups results in an
increase in melting point. Phenols are slightly soluble in water and their
solubility increases with an increase in temperature. Phenol is also known as
carbolic acid. Generally phenols are toxic but they exhibit antiseptic property.
Liquid phenol is a mixture of 90% phenol and 10% water. Phenols are weakly
acidic in nature.
16.2 PREPARATION OF PHENOLS
Various preparative methods of phenols as follows:
Dow’s process
Heating chlorobenzene with concentrated NaOH solution at 300°C under high
pressure results in the formation of sodium salt of phenol, which in acidic
medium liberates free phenol. The reaction proceeds through an elimination–
addition mechanism via benzyne intermediate (Section12.3.1). The presence of
electron withdrawing group in chlorobenzene enhances the rate of substitution
reaction and the reaction follows an addition-elimination mechanism via
Meisenheimer complex formation as an intermediate.

From aromatic amines


Aniline, an aromatic amine, on treatment with nitrous acid produces diazonium
salt. The diazonium salt, on heating in aqueous acidic medium, liberates nitrogen
and forms phenol.

From sodium benzenesulfonate


Fusion of sodium benzenesulfonate with sodium hydroxide results in the
formation of phenol. This is a nucleophilic aromatic substitution reaction where
the sulfonate group is replaced by hydroxide ion.

Mechanism
Step 1. Friedel–Crafts alkylation

Step 2. Oxidation of cumene to cumene hydroperoxide


Step 3. Hydrolytic rearrangement in cumene hydroperoxide
From allylphenyl ethers (Claisen rearrangement)
Allylphenyl ether on heating undergoes an intramolecular rearrangement to form
o-allyl phenol and the reaction is known as Claisen rearrangement. In the
reaction, the migration of allyl group occurs at the ortho position to give the
product.
16.3 CHEMICAL PROPERTIES OF PHENOL

16.3.1 Acidic Nature of Phenol: A General Discussion


Phenols are acidic in nature. The acidic nature of phenol is attributed to +R
effect in phenol. Due to +R effect, the oxygen develops a positive charge and
thereby registers reduced hold on hydrogen. This results in the weakening of O–
H bond and makes removal of hydrogen as proton (H+) to occur readily, thereby
making the phenol acidic.

Phenol is more acidic than alcohols but less acidic than


carboxylic acids.
The loss of proton results in the formation of conjugate bases, that is,
alkoxide (in alcohols), phenoxide (in phenol), and carboxylate ion (in
carboxylic acids). The acidic strength is directly related to the stability
of the conjugate base.
The alkoxide ions are destabilized by +I effect of alkyl group(s) and
thus alcohols are least acidic. The phenoxide and carboxylate ions on
the other hand are resonance stabilized. Although phenoxide ion has
more number of contributing structures as compared to the
carboxylate ion, carboxylic acid is more acidic than phenol.
The contributing structures in phenoxide ion carry a negative charge
on carbon while in carboxylate ion the negative charge is delocalized
over both the oxygens. The contributing structures with negative
charge on electronegative atom (like oxygen as in case of carboxylate
ions) contribute more towards the resonance stabilization (refer
Chapter 1).

No resonance in alkoxide ions.

Relatively less stable contributing structures for phenoxide ion as negative charge is delocalized
over electropositive carbon of the aromatic ring.

More stable contributing structures for carboxylate ion as negative charge delocalized over
electronegative oxygens.

Effect of substitution on acidic strength of phenol


The electron withdrawing groups on aromatic ring in phenols increase the acidic
strength of phenol. The electron withdrawing groups take away the electrons
from ring system and thus, increase the positive charge on oxygen of O–H bond.
More the positive charge on oxygen, easier becomes the removal of proton and
higher becomes the acidic strength.
The electron releasing groups release electrons towards the ring system and this
neutralizes or reduces the positive charge on oxygen and makes O–H bond
stronger thereby making the removal of proton difficult. Lesser the positive
charge on oxygen, higher the strength of O–H bond and lesser is the acidic
strength of phenol.
The increasing acidic strength of these phenols may be explained on the basis of
the stability of their corresponding conjugate bases (phenoxide ions) as follows:

Comparison of acidic strength


(1) In isomeric nitrophenols (ortho, meta and para isomers)
In case of phenols the acidic strength depends upon two factors:
(i) Inductive effect which depends upon the position of the substituent
(that is, lesser the distance stronger is the inductive effect) and is
experienced in the order: ortho > meta > para.
(ii) Resonance effect of substituents, which is observed maximum at
ortho and para positions.
The order of acidic strength of ortho, meta, and para nitrophenols is
as follows:
The nitro group is electron withdrawing in nature and increases the
acidic strength of phenol through it inductive (–I) and resonance (–R)
effect. The resonance effect is operative more at ortho and para, thus
ortho and para isomers are more acidic compared to meta.
Out of ortho and para, the ortho isomer should be more acidic due to
inductive (–I) effect. However this is not the case because in o-
nitrophenol intramolecular hydrogen bonding takes place, which
makes the removal of proton difficult. Thus, ortho isomer is less
acidic than para isomer due to intramolecular hydrogen bonding. The
overall acidic strength in case of isomeric nitrophenols follows the
order para > ortho > meta.

(2) In isomeric chlorophenols (ortho, meta, and para isomers)


The order of the acidic strength of ortho, meta, and para
chlorophenols is as follows:

Chlorine exhibits +R effect but at the same time it exhibits a strong –I


effect. In this case the only factor, which contributes towards the
acidic strength, is the inductive effect. Chlorine due to its electron-
withdrawing nature, will increase the acidic strength of phenols. As
mentioned earlier the inductive effect is experienced in the order ortho
> meta > para. Thus the same order of acidic strength will be
followed in isomeric chlorophenols as
o-chlorophenol > m-chlorophenol > p-chlorophenol
Reactions of phenols are discussed under following categories:
(1) Reactions due to acidic nature of phenol: In these reactions, the cleavage
of oxygen–hydrogen bond takes place.
(2) Reactions involving aromatic ring: These reactions involve the
electrophilic substitution reactions of benzene ring and O–H group remains
intact during these reactions.
(3) Other reactions include oxidation and reduction reactions of phenols.

16.3.2 Reactions due to Acidic Nature of Phenol


Reaction with sodium hydroxide
Phenol on reaction with aqueous alkali solution results in the formation of a
sodium salt, that is, sodium phenoxide.

The sodium salt of phenol is highly reactive and as a result, most of the reactions
of phenols are carried out as reactions of sodium phenoxide and not that of free
phenol.

Phenols react only with strong bases but not with weak bases
such as sodium carbonate or sodium bicarbonate. This reaction is
often used to distinguish between aromatic carboxylic acid and
phenol.
Reaction with acid chlorides and acid anhydrides (formation of esters)
Phenols react with acid chlorides and acid anhydrides in presence of bases to
form phenyl esters. The reaction and reaction mechanism are illustrated here.

Mechanism

Similar mechanism is followed in case of reaction of phenol with benzoyl


chloride and this reaction is popularly known as Schotten-Bauman reaction.

The reaction is similar to the reaction of alcohol, however,


esterification is slower in phenols compared to alcohols. The lone pair
of electron on oxygen in phenol participates in delocalization with π-
electrons of the benzene ring (+R effect), which decreases the
nucleophilicity of oxygen in phenol, and causes the decrease in the
rate of esterification.
Reaction with dimethylsulfate or halogenated hydrocarbons (formation of
phenolic ethers)
Sodium phenoxide on reaction with dimethyl sulfate, haloalkanes, or ortho or
para substituted halobenzenes result in the formation of phenolic ethers.

Reaction of phenoxide with haloakanes is known as Williamson ether synthesis.


The reaction of sodium phenoxide with ortho and para substituted halobenzene
occurs in presence of copper powder and results in the formation of
corresponding diaryl ethers. The reaction is known as Ulmann reaction.
Reaction with ferric chloride
[Qualitative detection of phenolic functional group]
Phenols on treatment with neutral ferric chloride solution, give violet colour due
to the formation of a complex. This reaction is used as a qualitative test for
detection of phenolic functional group in an organic compound.

16.3.3 Electrophilic Substitution Reactions of Phenols


The +R effect in phenols makes –OH group behave as a strong electron releasing
group. The –OH group activates the ring strongly, at ortho and para positions,
towards electrophilic substitution reactions.
Halogenation
The hydroxyl group in phenols is highly activating due to +R effect. Passing
chlorine through phenol results in the formation of o-chlorophenol and p-
chlorophenol.

In the presence of polar solvent, the bromination of phenol gives a trisubstituted


product. For example, the reaction of an aqueous solution of bromine with
phenol results in the formation of 2,4,6-tribromophenol.
The reaction of phenol with bromine, in a non-polar solvent like CS2, results in
the formation of p-bromophenol as the major product.
Nitration
In phenols, –OH group strongly activates the ring system. As a result, phenols
are susceptible to oxidation in the presence of concentrated nitric acid. Thus,
nitration of phenols is carried out with dilute nitric acid and results in the
formation of o-nitrophenol and p-nitrophenol. The o-nitrophenol is steam
volatile and the mixture of o-nitrophenol and p-nitrophenol is separated by steam
distillation.

Why is o-nitrophenol more volatile compared to p-


nitrophenol?
In p-nitrophenol, intermolecular hydrogen bonding occurs due to
which a large number of molecules are associated together. The
association of such a large number of molecules in the p-isomer
makes it less volatile.
However in o-nitrophenol, intramolecular hydrogen bonding occurs
and thus, it exists in a monomolecular state making it more volatile.
Sulfonation
The reaction of phenols with concentrated H2SO4 results in the formation of
ortho- and para-hydroxybenzenesulfonic acids. At low temperature (25°C), the
ortho isomer is a major product
whereas at high temperature (100°C), para isomer is favoured. The formation of
ortho product is a kinetically controlled reaction whereas that of para product is
a thermodynamically controlled reaction.
A thermodynamically stable product formation is preferred and for this reason,
the o-hydroxy-benzenesulfonic acid on heating gets converted to more stable p-
hydroxybenzenesulfonic acid.

Friedel–Crafts Reaction (with modified procedure)


The Friedel–Crafts reaction does not occur in case of highly activating rings like
phenols. In case of Friedel–Crafts reaction of phenol, the AlCl3 forms a complex
with oxygen. Thus, direct Friedel–Crafts reaction generally does not give good
results. In case of phenols, the Friedel–Crafts reaction is used with modification
where phenolic group is protected by methylation (etherification) before
carrying out the reaction.
Fries rearrangement
Phenols on treatment with an acid chloride or acid anhydride form phenolic
esters. These phenolic esters on heating with a catalyst (AlCl3), followed by
hydrolysis result in a rearrangement to form ortho and para acyl phenols (ortho
and para hydroxy aromatic ketones). The reaction is known as Fries
rearrangement.

The reaction forms same products in the presence of light, using ethanol as a
solvent (and without catalyst) and is then termed as Photo Fries rearrangement.
Mechanism. The fries rearrangement is an electrophilic substitution reaction that
may proceed through an inter- or intramolecular rearrangement. These
rearrangement mechanisms are individually discussed here.
Intermolecular rearrangement
Step 1. Complex formation (phenolic ester) with Lewis acid AlCl3 (formation of
acyl cation, an electrophile)

Step 2. Attack of electrophile at ortho and para positions of the ring & resonance
stabilization of arenium ion

Step 3. Removal of proton from σ-complex (regain of aromaticity)


Step 4. Hydrolysis (formation of o- and p-acyl phenols)
Intramolecular rearrangement
The intramolecular rearrangement explains the formation of ortho isomer.
Step 1. Complex formation (phenolic ester) with Lewis acid
Step 2. Intramolecular shift of acyl at ortho- position
Step 3. Hydrolysis (formation of ortho- acylphenol)

The ortho isomer formed by hydrolysis is stabilized by intramolecular hydrogen


bonding
Kolbe Schmitt reaction (industrial synthesis of salicylic acid)
The sodium salt of phenol on reaction with carbon dioxide at 120–140°C and
high pressure
(5–6 bar) results in the formation of o-hydroxybenzoic acid (salicylic acid). This
is an example of electrophilic substitution reaction where CO2 acts as an
electrophile. In phenoxide ion the aromatic ring is strongly activated and easily
undergo electrophilic substitution at ortho- position even with a weak
electrophile like carbondioxide.

Mechanism. The electrophile gets added to phenoxide ion at ortho- position.


This results in the formation of an intermediate, which is stabilized by chelation
and results in the formation of an substituted ortho- product.
Salicylic acid is the major product formed in the reaction and a small amount of
p-hydroxy benzoic acid is also formed.

Reimer–Tiemann reaction
Sodium phenoxide, on reaction with chloroform in alkaline medium, results in
the formation of
o-hydroxybenzaldehyde (salicylaldehyde) and p-hydroxybenzaldehyde.
The reaction is an electrophilic substitution reaction where the electrophile
involved is dichlorocarbene. Dichlorocarbene is generated by the reaction of
chloroform and sodium hydroxide. The attack of dichlorocarbene at strongly
activated ortho- and para- positions in phenoxide ion results in formation of
ortho and para products. Salicylaldehyde is formed as a major product due to
intramolecular hydrogen bonding. The details of reaction mechanism are as
follows:
Step 1. Generation of electrophile (1,1-elimination)

Step 2. Reaction of electrophile with phenoxide

Step 3. Hydrolysis
In a similar manner, the electrophile can react at para position of phenoxide ion
to give
p-hydroxybenzaldehye as the minor product.
Nitrosation
Phenol reacts with nitrous acid to form p-nitrosophenol along with a small
amount of o-nitosophenol. The product p-nitrosophenol exhibits tautomerism
and thereby forms yellow crystals of
p-benzoquinone monoxime.

Azo-dye formation
The aryldiazonium salt behaves as an electrophile and couples with phenols at
strongly activated para- position to form azo dyes. The reaction occurs in
alkaline medium and is referred to as azo-coupling. In case p-position in phenol
is already occupied by a substituent, the coupling occurs at ortho position.
16.3.4 Other Reactions
Reduction
(i) Phenol on distillation over zinc dust results in the removal of the hydroxy
group and forms benzene.
(ii) Hydrogenation of phenol in presence of metal catalyst like nickel results in
the formation of cyclohexanol.

Oxidation
The oxidation of phenol results in the formation of various products depending
upon the nature of oxidizing agent used. For example, oxidation with hydrogen
peroxide gives a mixture of di- and trihydric phenols.

In the presence of stronger oxidizing agents, the breakdown of ring may occur.
Strong oxidizing agents may also cause the coupling of several benzene rings,
which results in the formation of tarry materials. The oxidation occurs through
the formation of phenoxy radical.

Ammonolysis of phenol
Phenol, when heated with ammonia in presence of zinc chloride as catalyst,
results in the formation of aniline.

Laderer Manasse reaction


The reaction of phenol with formaldehyde in the presence of alkaline medium
results in hydroxymethylation of phenol at ortho- and para- positions. The
reaction is known as Laderer Manasse reaction and is used to manufacture
bakelite (Chapter 31).

Reaction with phthalic anhydride (formation of phenolphthalein)


The condensation of phenol (2 mol) and phthalic anhydride (1 mol) in presence
of sulfuric acid results in the formation of phenolphthalein. The phenolphthalein
is used as an indicator in acid-base tritrations.
The reaction has been described in detail in Section 30.3.3.
16.4 POLYHYDRIC PHENOLS
The class of organic compounds where two or more hydroxy groups are directly
attached to benzene ring system are termed as polyhydric phenols. The examples
of common polyhydric phenols are catechol, resorcinol, quinol, phloroglucinol,
and so on (see Section 16.1).
An increased number of hydroxy groups in aromatic ring has the following
effects:
(1) With an increase in the number of hydroxy groups, the solubility in water
increases due to an increase in the available sites for hydrogen bonding.
(2) An increase in the number of hydroxy groups activates the ring to a much
greater extent and as a result, the polyhydric phenols are more susceptible
to oxidation.
The methods of preparation and chemical reactions of polyhydric phenols are
similar to monohydric phenols. A brief description of polyhydric phenols forms
the subject matter of the subsequent discussion.

16.4.1 Dihydric Phenols


The three important isomeric dihydric phenols are catechol, resorcinol, and
hydroquinone (quinol). The preparation and properties of these phenols are
detailed in this section.
Catechol
(i) Heating o-hydroxysulfonic acid with an alkali results in the formation of
1,2-dihydroxybenzene also called catechol.

(ii) Industrial preparation of catechol involves heating 2-chlorophenol with


alkali at a high pressure in the presence of copper sulfate.
Catechol is crystalline substance with melting point 104°C. It is soluble in water
and gives green colour with ferric chloride, which changes to violet red in
presence of an alkali. It reduces Tollens reagent and Fehling solution, and is used
as a photographic developer.
Resorcinol
Benzene-1,3-disulfonic acid on fusion with alkali results in the formation of
resorcinol as follows:

It is a colourless crystalline substance with melting point 110°C. It is soluble in


water and gives violet colour with ferric chloride. It reduces Tollens reagent and
Fehling solution and is used as an antiseptic. Resorcinol exists in its tautomeric
‘diketo’ form, which on reduction with Na–Hg forms dihydroresorcinol.
Hydroquinone [Quinol]
Reduction of p-benzoquinone with sulfurous acid or nascent hydrogen results in
the formation of hydroquinone.

It is a colourless substance and has a melting point 172°C. It shows reducing


properties and is used as a photographic developer.
16.5 QUINONES
Quinones are characterized by the presence of two double bonds and two
carbonyl groups that are a part of the ring. The quinonoid systems are
responsible for colour in the organic molecules (refer Chapter 30).

Ortho- and para-benzoquinone are obtained by oxidation of corresponding


dihydric phenols. Resorcinol does not form quinone.

The p-benzoquinone is also prepared by the oxidation of aniline with chromic


acid.
SELECTED SOLVED EXAMPLES
Example 1. Alkenylphenyl ether with structure PhOCH2CH=CHC2H5 on
heating undergoes Claisen rearrangement. Give the structure and IUPAC name
of the product formed by the rearrangement.
Solution. The Claisen rearrangement results in the formation of o- substituted
phenol (refer
p. 531). The reaction is as follows:

Example 2. Arrange the following in increasing order of their acidic strength.

Solution. In phenols, presence of electron releasing group(s) (like OCH3, CH3)


on ring decreases the acidic strength of the compound whereas the presence of
electron withdrawing groups (like NO2, SO3H) increases the acidic strength. A
powerful electron withdrawing group further enhances the acidic strength. More
the number of electron withdrawing groups, more will be the increase in acidic
strength.
Therefore, the increasing order of acidic strength is:

Example 3. An organic compound [A] on treatment with CHCl3/NaOH gives


two products [B] and [C]. On Zn-distillation, both [B] and [C] give the same
product [D]. The compound [B] and [C] on oxidation give [E] and [F] which are
isomeric having molecular formula C7H6O3. On heating with soda lime, [E] and
[F] give back [A]. Zinc dust distillation of [A] gives an aromatic hydrocarbon
[G]. Identify the compounds [A] to [G] and explain the reactions involved.
Solution. The reagent CHCl3/NaOH is used in Reimer–Tiemann reaction which
introduces an aldehyde group at ortho- and para- positions of phenolic
compounds. Zinc distillation removes the –OH group of phenol. The oxidation
of aldehyde results in carboxylic acid formation and treating it with soda lime
causes decarboxylation. So the reactions from [A] to [G] can be justified as
follows:

Example 4. Complete the following sequence of reactions:


Solution
Example 5. Carry out the following conversions:
(i) Phenol to anisole
(ii) Phenol to 4-chlorophenetole
(iii) Benzene sulfonic acid to 2-bromobenzaldehyde.
(iv) Phenol to aspirin (acetyl salicylic acid)
Solution
Example 6. Carry out the following conversions:
(i) Phenol to 4-methoxybenzyl alcohol
(ii) Phenol to 2,4-dinitrochlorobenzene
(iii) Benzene to 2-methyl-5-nitrophenol
Solution
EXERCISES
1. How are phenols structurally different from alcohols?
2. Give the structure and name of all possible isomeric phenols having
molecular formula
(a) C6H6O2 and (b) C7H8O.
3. Give the IUPAC nomenclature of the following compounds

4. Giving reasons, arrange the following phenols in increasing order of their


melting points.

5. Why is o-nitrophenol more volatile compared to p-nitrophenol?


6. Why an increase in the number of hydroxy groups on aromatic ring increases
its solubility in water?
7. Explain the Claisen rearrangement for the preparation of phenols?
8. Why phenols are more acidic than alcohols?
9. Arrange the following in the increasing order of their acidic strengths.

10. Why does esterification in phenols occur at a slower rate compared to


alcohols?
11. Complete the following reactions:

12. What is the directive influence of –OH group in phenol towards electrophilic
substitution reactions?
13. Why is nitration of phenol carried out with dilute HNO3 and not with
concentrated HNO3 and H2SO4 mixture?
14. What will be the bromination product of phenol under following reactions
conditions? Explain the formation of product.

15. What is Fries rearrangement? Explain with mechanism.


16. Discuss the mechanism for Reimer–Tiemann reaction.
17. Why does sodium salt of phenol in Kolbe Schmitt reaction form salicylic
acid as the major product?
18. Carry out the following conversions:
(a) Chlorobenzene to salicylaldehyde
(b) Aniline to benzoic acid
(c) Phenol to benzoic acid
(d) Phenol to nitrobenzene
19. Complete the following reactions:
Problem 72. In the following two reactions, the haloalkanes follow different
nucleophilic substitution reaction mechanism, why?

Solution. In reaction (a) primary haloalkane follows SN2 mechanism and is


explained in detail in section 11.3.1.
In reaction (b) the formation of carbocation is favoured as it is stabilized through
delocalization. Thus it follows the SN1 mechanism.

Problem 73. What is the most convenient route for the synthesis of neopentyl
iodide from neopentyl chloride.
Solution. Normally the iodides are conveniently prepared by the halide
exchange reaction known as Finkelstein reaction (Sec. 11.2, p. 396). Neopentyl
chloride (2,2-dimethyl-1-chloropropane) is a 1° haloalkane but has a bulky alkyl
group, so the substitution reactions (both SN1 and SN2) are very slow.
Synthesis of neopentyl iodide from neopentyl chloride using Grignard reagent
(from neopentyl chloride) is the most convenient method. The conversion takes
place as follows:
Problem 74. The hydrolysis of primary cholroalkanes is much faster in the
presence of iodide ions. Explain.
Solution. Hydrolysis of primary chloroalkanes follows SN2 mechanism to form
alcohols. The chloride ion is a poor leaving group and thus, its hydrolysis is
slow.

However in presence of iodide ions (I–), hydrolysis proceeds as two consecutive


SN2 reactions as follows:
First SN2 reaction. The primary chloroalkane is readily attacked by strong
nucleophile I– and gets converted to corresponding iodoalkane.
Second SN2 reaction. Iodoalkane then undergoes hydrolysis at a much faster rate
since I– is a very good leaving group compared to chloride ion.

Iodide ion is a strong nucleophile as well as a good leaving group (p. 404) and
thus, catalyzes the hydrolysis of chloroalkane.
Problem 75. In an attempt to prepare Grignard reagent from 3° haloalkane, a
side reaction occurs frequently. Explain the formation of the side product by
taking as an example, the reaction of
2-chloro-2-methylpropane with magnesium in dry ether.
Solution. The reaction of 2-chloro-2-methylpropane with magnesium in dry
ether results in the formation of tert-butylmagnesium chloride. Since a 3°
haloalkane is used, it may simultaneously undergo dehydrohalogenation to give
2-methylpropene as the side product.

Problem 76. Cyclopentane on heating with bromine gives compound A, which


on reaction with sodium ethoxide in ethanol, gives B. Compound B on reaction
with NBS (N-bromosuccinimide) in presence of peroxide, gives C. Identify A,
B, and C.
Solution. Bromination of cyclopentane gives bromocylcopentane [A], which
undergoes dehydrobromination with NaOC2H5/C2H5OH to form cyclopentene
[B]. The treatment of [B] with NBS/peroxide results in allylic bromination
(Section 6.3.5) to yield 3-bromocyclopentene[C].

Problem 77. In the given series of anions, identify the strongest base and the
strongest nucleophile.
(a) CH3O–, CH3COO–, HO–
(b) HO–, Cl–, CH3O–, I–
Solution. (a) If the attacking atom is the same, nuclophilicity of an anion is
directly related to the basicity of the anion (p. 402). Basicity is the ease of
availability of electrons on a species that can further participate in a reaction.
More is the availability of electrons in an anion, higher is its basicity. As the
increasing order of the basicity of the anions is CH3COO– < HO– < CH3O–, the
same order is followed for nucleophilicity. The nucleophilic centre in all these
anions is same, namely oxygen.
(b) The nucleophilicity order of anions containing different nucleophilic centres
is directly related to the polarizability of anions. An anion with a larger size is
more polarizable and hence will be a strong nucleophile. The iodide ion because
of its largest size will behave as the strongest nucleophile. But at the same time,
the larger size of iodine makes it least basic due to dispersal of negative charge.
The CH3O– is the strongest base as +I effect of –CH3 group increases the
availability of electrons on oxygen.
Problem 78. Of the following pair of anions, indicate which one will be a
stronger nucleophile and why?
(a) CH3O– and CH3S–
(b) CF3COO– and CH3COO–
(c) C6H5O– and CH3O–
Solution. The nucleophilicity is directly related to basicity if different species
have the same attacking nucleophilic atom. However, if nucleophilic atoms are
different, polarizability is taken into consideration (refer Problem 77). Thus,
(a) CH3S– is a stronger nucleophile than CH3O– because sulfur being larger
in size is more polarizable and is a better nucleophile than oxygen.
(b) CH3COO– is a stronger nucleophile than CF3COO– as the presence of
electron withdrawing fluorine decreases the basicity and hence the
nucleophilicity of the anion.
(c) CH3O– is a stronger nucleophile than C6H5O–. In case of C6H5O–, the
negative charge on oxygen participates in delocalization with phenyl ring
that decreases the basicity and hence its nucleophilicity.
Problem 79. Are SN2 reactions stereospecific or stereoselective?
Solution. The SN2 reactions are stereospecific. It proceeds with a rear side
attack of nucleophile to form a substituted product with an inversion of
configuration (refer text). Thus, each stereoisomer of the initial reactant forms a
specific stereoisomer of the product.
Problem 80. Explain, why in an SN2 reaction the rate of reaction of
bromopropane in acetone is much higher than that of 2-bromo-2-methylpropane
(tert.butyl bromide) under similar conditions?
Solution. As already explained in the text (for details, refer to sec 11.3.1), the
formation of highly unstable transition state (as in case of tert.alkyl halides)
slows down the reaction rate in an SN2 reaction. In general, transition state in
SN2 is more stable when lesser number of alkyl groups are attached (as in case
of primary alkyl halides) as it causes lesser steric hindrance.
Thus, in 2-bromo-2-methylpropane (tert.butyl bromide), steric hindrance inhibits
it from undergoing SN2 reactions.

Problem 81. What product will be formed by hydrolysis of neopentyl bromide?


Explain with mechanism.
Solution. Hydrolysis of a haloalkane is a nucleophilic substitution reaction that
results in the formation of an alcohol. In general, 1° haloalkanes follow SN2
mechanism whereas 3° haloalkanes follow SN1 mechanism.
Neopentyl bromide is a 1° haloalkane but the tert.butyl group causes steric
hindrance in the transition state of SN2 mechanism and inhibits it from
following a SN2 mechanism during hydrolysis.
However, the reaction may follow SN1 mechanism during which it undergoes a
carbocation rearrangement (1,2-methyl shift) to form more stable 3° carbocation
that undergoes nucleophilic attack of water to form a 3° alcohol. Thus, although
neopentyl bromide is a primary haloalkane but its hydrolysis does not follow
SN2 mechanism. The SN1 mechanism of its hydrolysis is summarized as
follows:

Problem 82. Explain, what is/are the product(s) formed in the following reaction
if SN1 mechanism is followed?

Solution. The SN1 reaction proceeds through a carbocation intermediate. The


carbocation formed in the above reaction is an allylic carbocation, which
undergoes resonance stabilization (p. 419). Each contributing structure
undergoes attack by nucleophile (–OCH3) resulting in the formation of
corresponding ether. The formation of two products occurs as follows:
Problem 83. Explain, why vinyl bromide has a low dipole moment value
compared to bromoethane?
Solution. The dipole moment of bromoethane is due to –I effect of bromine
which increases the polarity of C–Br bond.
The polarity of C–Br bond in vinyl bromide is decreased by the +R effect
(resonance effect), which acts in a direction opposite to –I effect of bromine.
Thus, vinyl bromide has a lower dipole moment than bromoethane.

Problem 84. Explain, why are the compounds CH3CHF2 and CH3CF3 not
reactive towards nucleophilic reagents?
Solution. Fluorine has a strong –I effect. The presence of more number of
fluorine atoms should increase the electrophilicity of the carbon. However, the
fluorine atoms present on same carbon
atom decreases the polarity of molecule as the dipole moment of C–F bonds
oppose each other. This causes a net decrease in the electrophilic character of the
carbon attached to two or more fluorine atoms. Thus, the compounds CH3CHF2
and CH3CF3 are not reactive towards nucleophilic reagents due to reduced
electrophilicity of the carbon.
Problem 85. Complete the following reactions and indicate, giving reasons,
which of the reactions will be faster?

Solution. The product formed in the Reaction 1 is an ether and in Reaction 2 is a


thioether.

Both reactions are SN2 type. The sulfur atom being larger in size is more
polarizable and hence, more nucleophilic compared to oxygen (refer also
Problem 78). Thus, the rate of reaction in 2, is much faster due to higher
nucleophilicity of the anion CH3CH2CH2CH2S– as compared to
CH3CH2CH2CH2O– anion.
Problem 86. Giving a suitable explanation, arrange the following species in
order of their increasing strength as dehydrohalogenating agents.

Solution. Dehydrohalogenation is an elimination reaction where removal of a


proton occurs in the presence of a base. The higher the basicity of a reagent,
faster will be the dehydrohalogenation. The alkoxides used for
dehydrohalogenation, are very strong bases. The basicity of alkoxides increases
with an increase in the number of alkyl groups (due to +I effect). Thus, the order
of basicity as well as their strength as dehydrohalogenating agents, follows the
order
IV < III < II < I
Problem 87. Explain, how the nature of solvent, (i) water/alcohol and (ii) ether,
affects the hydrolysis of tert.butyl chloride?
Solution. The hydrolysis of tert.butyl chloride follows SN1 mechanism which
involves the formation of carbocation intermediate by the removal of Cl–. The
hydrolysis will be faster in a solvent which solvates the intermediate carbocation
as well as the anion (leaving group).
The polar protic solvents such as water and alcohol can solvate, both cations and
anions. However, a polar aprotic solvent such as ether can solvate only cations.
Thus, the reaction rate is faster in water/alcohol solvent system compared to
ether.

Problem 88. Which is the possible geometrical isomeric alkene formed during
dehydrbromination of 2-bromopentane in alcoholic KOH?
Solution. Dehydrobromination of 2-bromopentane results in the formation of
trans-2-pentene. The reaction follows E2 mechanism (see Section 11.3.3) where
trans- (or anti-) elimination occurs and results in the formation of a more stable
alkene.
Problem 89. Carry out the synthesis of the following alcohols from
ethylmagnesium bromide:
(a) 2,3-Dimethylpentan-2-ol
(b) Pentan-1-ol
Solution. In both the cases, parent chain in alcohol is having five carbon atoms.
Grignard reagent, that is, ethylmagnesium bromide supplies two carbons and
therefore, the other reactant must supply three carbon atoms.
(a) Synthesis of 2,3-dimethylpentan-2-ol: Its synthesis involves the reaction of
ethylmagnesium bromide with substituted epoxy compound (oxacyclopropane
derivative). The ring opening takes place from the less hindered site and the
reaction occurs as follows:

(b) Synthesis of pentan-1-ol: It is a primary alcohol and primary alcohols can be


synthesized by the reaction of corresponding oxacycloalkane with Grignard
reagent (see text and review,
Chapter 25). Thus, for the synthesis of pentan-1-ol, the ethylmagnesium bromide
is treated with trimethylene oxide (oxacyclobutane), which on hydrolysis gives
the desired product.
The high reactivity of oxacyclobutane is associated with the ring
strain in four membered cyclic compounds. As a result, it readily
undergoes ring opening on reaction with Grignard reagent resulting in
the formation of corresponding alcohol.
In contrast to oxacyclobutane, the next homologue oxacyclopentane
(Tetrahydrofuran: THF) does not react with Grignard reagent and in
fact, it is used as a solvent in many reactions. The inert behaviour
(stability) of THF is attributed to the absence of ring strain.
Problem 90. The nitration of benzyl halides results in the formation of ortho-
and para- substituted nitro derivatives along with a little percentage of meta-
nitro derivative. The relative percentage of meta- derivative increases as one
moves from benzyl bromide through benzyl chloride to benzyl fluoride. Explain,
with reasons.
Solution. Halogens are electronegative in nature with a strong –I effect which
follows the order F > Cl >Br. The strong –I effect reduces the activation by –
CH2X group, at the ortho- and para- positions.

The activation of ortho- and para- positions decreases with an increase in the
electronegativity of halogen, which results in a relative increase in the yield of
meta- product.

Problem 91. Explain, the product formation in the following reaction.

Solution. The reaction is an example of nucleophilic substitution in aromatic


halogen compounds through elimination–addition mechanism via benzyne
intermediate (p. 436). In the reaction, the
o-methoxyfluorobenzene undergoes attack of PhLi, an organometallic
compound, where phenyl ring behaves as a nucleophile. The mechanism of the
reaction is as follows:

The –I effect of –OCH3 group stabilizes the negative charge at ortho position
more effectively as compared to negative charge at meta position. This results in
the formation of
m-phenylmethoxybenzene as the final product.
Problem 92. The reaction of ketones with Grignard reagent is a convenient
method for the preparation of 3° alcohols. However, one fails to synthesize a
sterically hindered 3° alcohol by a similar reaction as shown here:

Explain the reason and suggest an alternative method for the preparation of
sterically hindered 3° alcohols.
Solution. The reaction of organometallic compounds with ketones is used as a
tool for synthesizing alcohols. The reaction involves the attack of nucleophlic
alkyl group on electrophilic carbon of keto group. Higher the electropositive
character of metal, faster is the reaction with ketones.
The carbonyl group in the given ketone (2,4-dimethylpentan-3-one) is sterically
hindered. The relatively less electropositive character of magnesium inhibits the
attack of alkyl group of Grignard reagent on the sterically hindered carbonyl
carbon. Thus, Grignard reagent fails to give any reaction with sterically hindered
ketones.
Alkyllithiums are relatively more reactive than organomagnesium compounds
(Grignard reagent) due to high electropositive character of lithium compared to
magnesium. The highly electropositive lithium increases the carbanion character
of alkyl group and as a result alkyllithium reacts readily, even with sterically
hindered ketones to form 3° alcohol.
Problem 93. Give a suitable explanation for the following results obtained
during substitution reactions of 3°butyl alcohol.

Solution. The substitution reaction of 3° butyl alcohol follows a SN1 pathway.


Its reaction with sodium chloride does not occur. The –OH is a very strong base
and it is very difficult to remove –OH in a nucleophilic substitution reaction.
Thus, –OH behaves as a poor leaving group. However, when the reaction of 3°
butyl alcohol with NaCl is carried out in presence of an acid, the protonation of
oxygen occurs to form an oxonium ion (R–+OH2) and the –OH is converted to a
better leaving group (i.e. water, H–O–H). Thus, in the presence of an acid, a
carbocation is formed readily, which then undergoes attack of nucleophile (Cl–)
easily to form 3° butyl chloride.

Problem 94. What major product will be obtained on reaction of 3,3-


dimethylbutan-2-ol with HBr in presence of H2SO4 as catalyst?
Solution. The acid catalyzed nucleophilic substitution of 3,3-dimethylbutan-2-ol,
first results in the formation of a 2° carbocation. The 2° carbocation undergoes
Wagner–Meerwein rearrangement (p. 216) to form a more stable 3° carbocation
through 1,2-methyl shift. The attack of
Br– (nucleophile) on 3° carbocation results in the formation of 2-bromo-2,3-
dimethylbutane.
The product formed through 2° carbocation, that is, 2,2-dimethyl-3-bromobutane
is obtained in minor amounts.
Problem 95. Explain, with mechanism, the formation of alkene by dehydration
of neopentyl alcohol.
Solution. Dehydration of neopentyl alcohol occurs under acidic conditions to
form an alkene. The neopentyl alcohol, on protonation followed by elimination
of water, results in the formation of a 1° carbocation.

The 1° carbocation rearranges itself to a more stable 3° carbocation through 1,2-


methyl shift (Wagner–Meerwein rearrangement). The 3° carbocation undergoes
deprotonation from adjacent carbon to form a more substituted alkene, namely 2-
methylbut-2-ene.

Problem 96. Arrange the following alcohols in increasing order of their


reactivity with HBr.
Solution. The reaction of an alcohol with HBr results in the formation of
bromoalkane through nucleophilic substitution reactions (SN1). The
intermediate so formed is a carbocation. The alcohols, in acidic medium
eliminate –OH group as H2O to form following carbocations:

The increasing order of stability of carbocation is: [II] < [I] < [III] < [IV]
The carbocations are stabilized through resonance. More the number of phenyl
rings attached to carbon with positive charge, more is the delocalization of the
charge and higher will be the stability. The p-nitrobenzyl cation is more stable
than benzyl carbocation due to delocalization of positive charge by –NO2 group.
Thus, order of reactivity is the same as the order of stability of carbocations.
Problem 97. The reaction of methoxybenzene (methylphenyl ether) with
concentrated aqueous HI solution at ~100°C results in the formation of phenol
and iodomethane. However, diphenyl ether under similar conditions (and even at
higher temperature), gives no reaction. Explain.
Solution. In phenolic ethers it is very difficult to cleave the phenolic C–O bond.
The mechanism for the reaction of methoxybenzene with concentrated HI
solution is as follows:
In diphenyl ether, the protonation of oxygen occurs but there is no sp3 carbon
which can be attacked by nucleophile I– to carry out further reaction.

Problem 98. What product will be obtained from following pinacols on heating
them in acidic medium? Explain with mechanism.
(a) 2,3-Diphenylbutane-2,3-diol
(b) 2-Methyl-1,1-diphenylpropane-1,2-diol
Solution. The heating of pinacols in acidic medium results in a rearrangement
reaction to form pinacolone (pinacol–pinacolone rearrangement). The important
steps in pinacol–pinacolone rearrangement (Section 15.3.2) are:
(i) Formation of a stable carbocation by removal of one of the –OH group in
acidic medium.
(ii) Migratory aptitude of groups during 1,2-shift of alkyl/aryl group.
(a) 2,3-Diphenylbutane-2,3-diol (a symmetrical pinacol) in acidic medium
undergoes pinacol–pinacolone rearrangement to produce 3,3-diphenylbutanone.
A symmetrical pinacol can lose either of the –OH group as it leads to the
formation of same carbocation. However during 1,2-shift, a phenyl group
migrates preferably over alkyl group.

Mechanism
(b) 2-Methyl-1,1-diphenylpropane-1,2-diol (an unsymmetrical pinacol) in acidic
medium undergoes pinacol–pinacolone rearrangement to produce 3,3-
diphenylbutanone. In unsymmetrical pinacol, the loss of –OH group occurs from
that carbon atom which results in the formation of stable carbocation. In this
case, the removal of –OH group occurs from carbon atom carrying two phenyl
groups. During 1,2-shift, either of the methyl group can migrate to give the
product.
The Pinancol-pinacolone rearrangement in some cases may
lead to ring expansion or ring contraction of the cyclic system.
Refer Problem 99 of Explore More Set-IV.
Chapter17
Aliphatic
Carbonyl Compounds
(Alkanals and Alkanones)
17.1 INTRODUCTION
The aldehydes and ketones belong to the class of oxygen containing organic
compounds that have a C=O group. Therefore, they are termed as carbonyl
compounds. In carbonyl group, the carbon and oxygen are sp2 hybridized. In
aldehydes, the three sp2 hybrid orbitals of the carbon form three σ-bonds,
namely the bonds with alkyl group (sp2–sp3 overlap), hydrogen (sp2–s overlap),
and oxygen (sp2–sp2 overlap). The p orbitals on carbon and oxygen overlap to
form a carbon–oxygen π-bond. The bond angle is 120° and carbonyl compounds
have planar structure. Aldehydes and ketones have the following general
structure:

In accordance with IUPAC nomenclature, the aldehyde and ketones are named as
alkanals and alkanones respectively. The carbonyl carbon is included in the
longest possible carbon chain and is given the lowest possible number. The
nomenclature of carbonyl compounds has already been discussed in detail in
Chapter 2. A few selected examples follow along with their IUPAC
nomenclature (common names in parentheses).
17.1.1 Physical Properties
The carbonyl compounds are polar in nature as the electronegativities of carbon
and oxygen differ a lot, and oxygen being more electronegative has a greater
electron density around it compared to carbon.

Due to polarity, the intermolecular association of carbonyl compounds occurs


which makes the boiling point of carbonyl compounds higher than the
corresponding alkane of comparable mass. However, the carbonyl compounds
cannot form hydrogen bonding with each other and their boiling points are lower
compared to alcohols (of comparable mass). Methanal and ethanal are gases at
room temperature; the other members of aldehydes and ketones series are
liquids. Higher aldehydes
(C20 onwards) and ketones (C30 onwards) are solid at room temperature.
Methanal, ethanal, and propanone are highly soluble in water due to their
association with water through hydrogen bonding. The higher members (C4
onwards) are either slightly soluble or completely insoluble in water. Table 17.1
enlists the boiling points and densities of some carbonyl compounds.
17.2 PREPARATION OF ALIPHATIC CARBONYL
COMPOUNDS
This section discusses in detail, the methods/reactions grouped under three
categories, namely
(i) The reactions involving the formation of both aldehydes and ketones; (ii)
those concerned only with the formation of aldehydes; and (iii) those concerned
with the formation of ketones only. A few of the methods used for preparation of
carbonyl compounds have already been discussed in the previous chapters as
important reactions of various functional groups. The reference of the section
numbers that are previously discussed in details are given along with.

17.2.1 Formation of both Aldehydes and Ketones


Oxidation reactions
(i) Ozonolysis of alkenes. The alkenes on treatment with ozone followed by
reductive cleavage result in the formation of aldehydes whereas substituted
alkenes on ozonolysis form ketones.

(ii) Oxidation of alcohols. Primary alcohols in presence of oxidizing agents


such as PCC or by catalytic dehydrogenation with Cu at 250°C get oxidized to
aldehydes whereas secondary alcohols result in the formation of ketones.
Catalytic oxidation of methanol and ethanol. Formaldehyde (methanal) and
acetaldehyde (ethanal) are commercially synthesized by oxidizing methanol and
ethanol respectively. The vapours of methanol or ethanol are mixed with air and
passed over heated copper gauge to produce corresponding aldehydes. In case of
methanol, the aldehyde and water vapours are condensed to a liquid solution
giving 37–40% solution of formaldehyde, known as formalin.
An alternative method involves passing the alcohol vapours directly over heated
copper catalyst.

(iii) Oxidation of vicinal diols. Vicinal diols (glycols), on treatment with


periodic acid or lead tetra-acetate, undergo cleavage of carbon–carbon bond and
result in the formation of aldehydes. Oxidation of substituted vicinal diols with
periodic acid or lead tetra-acetate results in the formation of ketones (refer also
p. 514–15).
Hydrolysis reactions
(i) Hydration of alkynes. Ethyne (acetylene), on acid catalyzed hydration in
presence of HgSO4, results in the formation of enol, which tautomerize to yield
ethanal (acetaldehyde). All other alkynes under similar conditions give
corresponding ketones (p. 281).

(ii) Hydrolysis of geminal dihalides (1,1-Dihaloalkanes). The hydrolysis of


geminal dihalides results in the formation of gem. diols, which readily eliminate
water molecule to form aldehydes or ketones. For example,

From Grignard reagent


The addition of Grignard reagent to nitriles followed by hydrolysis results in the
formation of aldehydes and ketones.
(i) With HCN. Reaction of Grignard reagent with HCN followed by hydrolysis
results in the formation of aldehydes.
(ii) With alkanenitriles. Reaction of Grignard reagent with alkanenitrile
followed by hydrolysis results in the formation of ketones.
The preparation of aldehydes and ketones using Grignard reagent are depicted as
follows:
The reaction of Grignard reagent with nitriles is
considered to be the best method for the preparation of carbonyl
compounds.
As mentioned earlier (see Section 13.2) reaction of Grignard reagent
with compounds such as acid chlorides, esters, and so on lead to the
formation of carbonyl compounds but the reaction does not stop at
this stage and proceeds further with the another molecule of Grignard
reagent to give corresponding alcohol as the final product.
With nitriles, In the first stage of addition, an imine is formed as an
intermediate product. This imine is then isolated and hydrolyzed to
give a carbonyl compound as the final product.
Pyrolysis of Calcium Salts of Carboxylic Acids
Heating calcium salts of carboxylic acids results in partial decarboxylation to
form carbonyl compounds.
(i) With calcium formate. Heating calcium formate (calcium methanoate) with
calcium salts of other carboxylic acids results in partial decarboxylation and
aldehydes are formed.
An alternate method for preparation of aldehydes involves passing the vapours
of formic acid and other carboxylic acids over calcium oxide, manganese oxide,
or thorium oxide.
(ii) With calcium salts of carboxylic acids (other than formic acid). Calcium
salts of carboxylic acids (other than calcium formate) are heated to result in the
formation of ketones. Mixed ketones can be prepared by heating calcium salts of
two different carboxylic acids together. For example,

Hydroboration–oxidation of alkynes
Hydroboration of alkynes with borane results in the formation of vinyl
organoboranes, which on oxidation with alkaline hydrogen peroxide forms an
enol that tautomerizes rapidly to corresponding carbonyl compound (p. 282).
While terminal alkynes result in the formation of aldehydes, non-terminal
symmetrical alkynes result in the formation of ketones. However, non-terminal
unsymmetrical alkynes on hydroboration–oxidation result in the formation of a
mixture of ketones.
The vinyl organoborane has a double bond, which may undergo addition of a
second molecule of borane (as in the case of alkenes) to give a mixture of other
side products. Thus to prevent further addition, a bulkier hydroborating agent is
used which is popularly known as 9-BBN (9-Borabicyclo [3.3.1] nonane).

17.2.2 Formation of Aldehydes


Reduction
(i) Rosenmund reduction. Acid halides on treatment with hydrogen in the
presence of Pd catalyst supported over BaSO4 get reduced to aldehydes. The
catalyst is poisoned with little sulfur, which prevents further reduction of
aldehydes to alcohols. For example,

(ii) Stephen reaction. The reaction of alkanenitrile with HCl in the presence of
anhydrous stannous chloride (SnCl2) yields aldimine complex as an
intermediate, which on hydrolysis results in the formation of aldehyde. The main
reducing species is tetrachlorostannic acid, H2 [SnCl4], which is formed by
action of HCl and SnCl2. This is illustrated in the following reaction:
(iii) Reduction of acid chloride with Lithium tri-t-butoxyaluminohydroxide.
The reagent is prepared by treating LiAlH4 with three moles of tert-butyl
alcohol. Treating the acid chloride with this reagent in the presence of diglyme
(CH3OCH2CH2OCH2CH2OCH3) as a solvent results in the reduction of acid
halide to aldehyde.
17.2.3 FORMATION OF KETONES
From Dialkylcadmium
Reaction of acid chloride with dialkylcadmium results in the formation of
ketones. Dialkylcadmium is prepared by treating alkylmagnesium halide
(Grignard reagent) with cadmium chloride.
For example, preparation of butanone using Gilman reagent may be carried out
in the following two ways:
17.3 CHEMICAL PROPERTIES OF CARBONYL
COMPOUNDS
The carbonyl compounds undergo various types of chemical reactions which
include reactions involving the carbonyl ( C=O) group (nucleophilic addition
reactions), oxidation and reduction reactions, reactions involving α-carbon of
carbonyl compounds, and polymerization reactions of aldehydes. The reactions
which have already been discussed are also given in brief, along with the
reference of previous section numbers.

17.3.1 Reactivity of Carbonyl Group: A General


Discussion
The high elctronegativity of oxygen makes the carbon–oxygen double bond
polar in nature. As a result, the carbonyl carbon carries a partial positive charge
and becomes an electrophilic carbon.

Aldehydes and ketones undergo nucleophilic addition reactions. During addition


of a reagent to carbonyl group, the nucleophilic part of reagent adds to the
electrophilic carbon of carbonyl group and thus in carbonyl compounds the
addition reactions are said to be nucleophilic addition reactions. The
electrophilic part of the reagent adds to the oxygen of carbonyl group.
The aldehydes are more reactive than ketones towards nucleophilic addition
reactions. The order of reactivity of aldehydes and ketones in this case may be
generalized as:
This low reactivity of ketones is explained by (i) inductive effect and (ii) steric
hindrance.
The carbonyl carbon is electrophilic in nature. Greater the positive charge on
carbonyl carbon, greater is its reactivity towards nucleophilic addition reactions.
The alkyl group exhibits +I effect. More the number of alkyl groups attached to
the carbonyl carbon, lesser is the positive charge on carbon and lesser will be its
reactivity towards nucleophilic addition reactions. Also the bulkier alkyl groups
hinder the approach of nucleophilic reagent and therefore leading to a decrease
in the reactivity of ketones as compared to aldehydes.

The α-hydrogen in carbonyl carbon is highly acidic due to electron withdrawing


character of the carbonyl group. The carbanion generated by the removal of α-
hydrogen is stabilized by delocalization of negative charge over the carbonyl
group. A number of reactions of carbonyl groups involve the reactivity of α-
hydrogen(s).

The carbonyl compounds also exhibit keto–enol tautomerism where α-hydrogen


atom shifts its position from α-carbon to oxygen.

17.3.2 Nucleophilic Addition Reactions


The carbonyl compounds undergo addition of carbon, oxygen, sulfur, and
nitrogen nucleophiles. In general, the reaction follows two types of mechanisms.
Mechanism 1. Addition of strong nucleophiles
The addition of strong nucleophiles (like –CN) occurs in neutral or alkaline
medium. The general mechanism of this reaction is depicted here.
General mechanism:

In general the reaction of carbon nucleophiles with carbonyl compounds follow


this mechanism. For example, reaction of HCN, witting reaction, Reformatsky
reaction and reaction of organometallic compounds.
Mechanism 2. Addition of weak nucleophiles
Strong acids catalyze the addition of weak nucleophiles to carbonyl compounds.
In presence of acid, the protonation of carbonyl oxygen occurs which increases
the electrophilicity of carbonyl carbon. Therefore, addition of weak nucleophile
occurs readily at carbonyl carbon as shown through the general mechanism that
follows:
General mechanism:

In general this mechanism is followed in reaction of water, alcohol, thiols,


percarboxylic acids and ammonia derivatives with carbonyl compounds.
With carbon nucleophiles
(i) Reaction with HCN. The carbonyl compounds react with hydrogen cyanide
to form cyanohydrin, an addition product.
Mechanism. The carbon nucleophile (–CN), which is a strong nucleophile, first
attacks the carbonyl carbon followed by addition of proton to form cyanohydrin.

The advantage of this method is that hydrolysis of cyanohydrin produces


carboxylic acid. This reaction is frequently used in the synthesis of α-
hydroxycarboxylic acids.

(ii) Wittig reaction. The Wittig reaction is used for the synthesis of alkenes by
reaction of carbonyl compounds with phosphorous ylides. Phosphorous ylide is
prepared by reaction of triphenylphosphine with haloalkane (methyl, 1° or 2°).
This reaction results in the formation of phosphonium salt which on treatment
with strong base (such as PhLi, BuLi, NaOC2H5 ) undergoes deprotonation to
form the phosphorous ylide. The reaction of phosphorous ylide with carbonyl
group results in the formation of betaine, which undergoes elimination to
produce an alkene. The synthesis of alkene (for example, propene) through this
reaction involves the following steps:
Step 1. Preparation of phosphorous ylide
Phosphorous ylide is a resonance hybrid of two structures and the structure with
positive and negative charge on adjacent atoms is the major contributing
structure.
Step 2. Reaction of phosphorous ylide with carbonyl compounds

The formation of alkene is regiospecific, that is, there is no ambiguity in the


position of double bond. In a similar manner, a number of phosphorous ylides
may be prepared using suitable haloalkane. The phosphorous ylides on reaction
with carbonyl compounds, result in the formation of different alkenes. For
example,

(iii) Reformatsky reaction. The carbonyl compounds react with α-haloesters


(esters with halogen at α-carbon) in the presence of zinc to yield β-hydroxy
esters. In general, aldehydes react more readily than ketones to form β-hydroxy
esters.

The reaction proceeds through the formation of organozinc derivative and


follows the mechanism as given here.

(iv) Reactions with organometallic compounds


[Grignard reagent and alkyllithiums]
As discussed already (Section 13.2), the nucleophilic addition of organometallic
compounds to the carbonyl compounds is used as a tool for synthesizing 1°, 2°,
and 3° alcohols. The alkyl group in the organometallic compounds behaves as a
source of nucleophilic carbon which gets attached to the carbonyl carbon (C-C
bond formation) to form an adduct that on hydrolysis results in the formation of
alcohols. For example,
(a) Formaldehyde results in the formation of 1° alcohols.
(b) Aldehyde (other than formaldehyde) form 2° alcohols
(c) Ketones result in the formation of 3° alcohols.
The corresponding reactions are discussed in detail in the preparation of alcohols
(Section 13.2). The general reaction of aldehydes and ketones with
organometallic compounds is as follows:

With oxygen nucleophiles


(i) Reaction with water. The addition of water to a carbonyl compound in acidic
medium results in the formation of a geminal diol. The products formed are
commonly known as hydrates. In general, the hydrates of aldehydes are more
stable than hydrates of ketones due to steric factors as well as due to high
electrophilicity of carbonyl carbon in aldehydes compared to ketones.
(ii) Reaction with alcohols. The reaction of alcohol (weak nucleophile) with
carbonyl compounds in acidic medium results in the formation of addition
products. Addition of alcohols to aldehydes in acidic medium first forms
hemiacetals which on addition of second molecule of alcohol results in the
formation of acetals. Addition of alcohols to ketones to form hemiketals and
ketals does not occur readily because of the steric hindrance.

Reaction of carbonyl compounds with ethylene glycol forms cyclic acetal or


cyclic ketal. This reaction is used for the ‘protection’ of carbonyl group provided
the reaction conditions are not acidic. This is because acetals and ketals
decompose in acidic medium to produce aldehydes and ketones.

(iii) Reaction with peroxyacids (Baeyer–Villiger rearrangement). The


reaction of ketones with peroxyacid in acidic medium results in the formation of
an ester. The oxygen of peroxyacids behaves as a nucleophile and adds to the
carbonyl carbon of ketone. This is followed by a rearrangement that involves
migration of an alkyl group of ketone to form ester.

A cyclic ketone undergoes Baeyer–Villiger rearrangement to form a cyclic ester


called lactone.
The migratory aptitude of different alkyl groups follows the order:
CH3 < 1°alkyl < 2°alkyl~ phenyl < cyclohexyl < 3°alkyl < H
In case of aldehydes, instead of alkyl group, it is the migration
of hydrogen that occurs and results in the formation of carboxylic
acid.

With nitrogen nucleophiles


(i) Reaction with ammonia. The reaction of ammonia with aldehydes results in
the formation of an addition product, which readily undergoes elimination of
water molecule to form an imine. In general, the nucleophilic addition of
ammonia and its derivatives is followed by elimination.

The imine formed by addition of ammonia to carbonyl compounds may further


react to form polyaddition products. For example, reaction of formaldehyde with
ammonia results in the formation of hexamethylenetetramine. However, reaction
of acetaldehyde with ammonia results in the formation of a trimer.
The aldehydes other than formaldehyde and acetaldehyde react with ammonia to
form imines but further reaction with ammonia does not occur due to steric
hindrance.
(ii) Reaction with ammonia derivatives. Derivatives of ammonia react with
carbonyl compounds in acidic medium to give addition products, which undergo
elimination of water molecule to give the final product as imine derivatives:
The reaction of carbonyl compounds with ammonia derivatives in general form
crystalline solid products, which have sharp melting points and are used for
identification of aldehydes and ketones. A few illustrative examples showing
reactions of carbonyl compounds with ammonia derivatives are as follows:
(a) With 1° amines (formation of Schiff’s base). Amines are good nucleophiles
and the addition product formed by the reaction of carbonyl compounds with
amines, further undergoes elimination of water in acidic medium to produce
imines also known as Schiff’s bases. The reaction is catalyzed by mild acidic
conditions. In highly acidic medium, this reaction with carbonyl compounds
does not occur due to protonation of amine (RNH2 + H+↔ RNH3+).
(b) With hydroxylamine (formation of oxime)

Oximes have planar structure ( C=N) and they exhibit geometrical isomerism.
One of the important reactions of oxime is Beckmann rearrangement which is
discussed in
Chapter 18.
(c) With hydrazine (formation of hydrazone)
(d) With phenylhydrazine (formation of phenylhydrazone)

(e) With semicarbazide (formation of semicarbazone)

(f) With 2,4-dinitrophenylhydrazine (formation of 2,4-


dinitrophenylhydrazone)
Reaction of carbonyl compounds with 2,4-
Dinitrophenylhydrazine
This reaction is of special interest and is used for the qualitative
analysis of carbonyl compounds since the product 2,4-
dinitrophenylhydrazone has an intense yellow or orange colour.
Therefore if addition of 2,4-dinitrophenylhydrazine (commonly
referred to as DNP reagent) to an organic compound gives deep
yellow or orange solid, the compound is said to be a carbonyl
compound.
With sulfur nucleophiles
(i) Reaction with sodium bisulfite. The addition of sodium bisulfite to carbonyl
compounds forms a crystalline bisulfite addition product (adduct). The product
is soluble in water but insoluble in cold concentrated sodium bisulfite solution.
The structure of the adduct has been established to be that of a salt of sulfonic
acid. For example,

The reaction does not require any acid catalyst and the nucleophile in this
reaction is SO32–. Sulfur is a more powerful nucleophile compared to oxygen
and therefore the attack is preferred through sulfur.
All aldehydes but not all ketones give bisulfite adduct. This
bisulfite addition product is formed by all aldehydes and only by
methyl ketones. The other ketones due to decrease in electrophilicity
of carbonyl carbon and also because of steric factors due to alkyl
groups, do not react with sodium bisulfite.
This reaction is used for the purification of aldehydes and ketones
because aldehydes and ketones may be recovered from their bisulfite
adducts by warming them up with sodium carbonate solution.

(ii) Reaction with thiols. Thiols are more acidic as compared to alcohols and
thiolate salts behave as powerful nucleophiles. The acid catalyzed addition of
thiols to carbonyl compounds results in the formation of thioacetals. In general,
the catalysts used in this reaction are Lewis acids such as BF3 in diethyl ether or
ZnCl2. For example,

Thioacetals are used to protect carbonyl compounds in various multi-step


organic syntheses. Also the reduction of thioacetals with raney nickel results in
the formation of alkanes and the method is used for the conversion of carbonyl
group ( C=O) to methylene group ( CH2).
17.3.3 Oxidation Reactions
Oxidation reactions of aldehydes
Though both aldehydes and ketones generally show similar nucleophilic addition
reactions, they differ in their oxidation reactions. In presence of strong oxidizing
agents such as acidified K2Cr2O7, acidified KMnO4, or acidified CrO3, the
oxidation of aldehydes results in the formation of carboxylic acids.

Aldehydes are oxidized much more readily compared to ketones and as a result,
aldehydes are able to reduce even mild oxidizing agents as discussed in the
subsequent parts of this section.
Oxidation reactions of ketones
Ketones are fairly resistant to oxidation. The oxidation of ketones occurs
under drastic conditions and requires strong oxidizing agents like acidified
K2Cr2O7 solution or alkaline KMnO4 solution. The oxidation of ketones results
in the formation of carboxylic acid with lesser number of carbon atoms. The
cleavage occurs at carbonyl carbon and hydrocarbon chain and one of the
hydrocarbon parts is oxidized to carbon dioxide and water. Oxidation of
cyclohexanone in presence of concentrated nitric acid results in the formation of
adipic acid.
In case of unsymmetrical ketones, during oxidation the cleavage can occur on
either side of carbonyl group to give a mixture of carboxylic acids.

Distinguishing aldehydes from ketones


In alkaline medium, mild oxidizing agents such as Tollens reagent, Fehling’s
solution, and Benedicts solution oxidize aldehydes to form carboxylic acids.
Ketones do not get oxidized under these conditions.
(i) Tollens reagent. The Tollens reagent is an ammonical silver nitrate solution
which when heated with an aldehyde oxidizes it to carboxylic acid and itself gets
reduced to silver. As the aldehyde produces free silver due to the reduction of
Tollens reagent, the reaction is also referred as silver mirror test for aldehydes.

Since this reaction is only given by aldehyde, it is used to distinguish between


aldehydes and ketones.
(ii) Fehling’s solution. The Fehling’s solution is a mixture of two solutions
known as Fehling solution A (alkaline CuSO4) and Fehling solution B (sodium
potassium tartarate called Rochelle salt in sodium hydroxide). The equivalent
quantities of two solutions , on mixing result in the formation of deep blue
coloured copper tartarate complex, which furnishes cupric (Cu2+) ions. When
this complex reacts with aldehydes, cupric ions are reduced to cuprous oxide
giving a red precipitate.

(iii) Benedict’s solution. This is an alkaline solution containing copper sulfate,


sodium citrate, and sodium carbonate. It furnishes cupric ions and on reaction
with aldehyde forms red precipitate of cuprous oxide. The aldehyde in turn is
oxidized to carboxylic acid. The chemical reaction is similar to that of Fehling’s
solution.

α-Hydroxyketones give positive Tollens and Fehling test


Although ketones do not reduce Tollens reagent and Fehling’s
solution. But in
a-hydroxyketones, the secondary alcoholic group gets oxidized to
ketonic group and in turn the a-hydroxyketone reduces the Tollens
reagent and Fehling’s solution as follows:

17.3.4 Reduction Reactions


Aldehydes and ketones undergo reduction in the presence of different reducing
agents and depending on the nature of reducing agent, the products such as
alcohols, alkanes, or glycol are obtained.
Catalytic reduction
The hydrogenation of carbonyl compounds in presence of metal catalysts like Pt
or Ni results in the formation of alcohol. Aldehydes on reduction yield primary
alcohols while ketones yield secondary alcohols.

Reduction with metal hydrides


Metal hydrides like lithium aluminiumhydride (LiAlH4) and sodium
borohydride (NaBH4) carry out selective reduction of aldehydes and ketones to
primary and secondary alcohols respectively.

Mechanism
It should be noted that these reagents are selective reducing agents, which reduce
only carbonyl group in a molecule. The double bond, if any, is not reduced by
these reducing agents.

Meerwein–Ponndorf–Verley reduction
Aldehydes and ketones are reduced to alcohols by treating them with a solution
of aluminium isopropoxide in isopropanol. The reaction is reversible in nature
and the reverse reaction is known as Oppenauer oxidation. However, oxidation
is carried out preferably using aluminium tert butoxide in tert butanol as a
catalyst.

Wolff–Kishner reduction
Wolff–Kishner reduction involves hydrazine as the reducing agent and reduces
carbonyl group of aldehydes and ketones to methylene group ( CH2). Aldehyde
or ketone is heated with hydrazine in presence of base such as potassium tert.
butoxide, potassium hydroxide, or potassium ethoxide, which results in the
reduction of the carbonyl compound to corresponding alkane. For example.

Since Wolff–Kishner reduction is carried out in alkaline conditions, it can be


used conveniently for reduction of carbonyl compounds, which are sensitive to
acids.
Clemmensen reduction
The reduction of carbonyl group of aldehyde and ketones to methylene group (
CH2) in acidic conditions in presence of zinc amalgam is known as Clemmensen
reduction. This can be illustrated as follows:

Since Clemmensen reduction is carried out in acidic conditions, it can be used


conveniently for reduction of carbonyl compounds, which are sensitive to bases.
Reduction of ketones to pinacols
Ketones on reduction in the presence of magnesium result in the formation of
tert. diols known as pinacols (refer Section 15.3.2). Aldehydes do not form
pinacols.
17.3.5 Reactions Involving α-Carbons of Carbonyl
Compounds
The reactions discussed so far involve the C=O group of aldehydes and ketones
as the reactive centre. It is interesting to note that carbon(s) adjacent to C=O
group also participate in some significant reactions. The carbon adjacent to C=O,
is known as α-carbon and the hydrogens attached to it are called α-hydrogens.

The aldehydes and ketones containing at least one α-hydrogen exhibit keto-enol
tautomerism.

The α-hydrogen of carbonyl compounds is acidic in nature due to the electron


withdrawing nature of C=O group. The α-hydrogen can be abstracted easily by a
strong base, which results in the formation of resonance stabilized carbanion
known as enolate ion. Enolization is enhanced in the presence of a strong base.

The enolate ions are involved in a number of reactions of aldehydes and ketones.
Some of these reactions are popular and happen to be important name reactions.
These are discussed as follows:
Aldol condensation
Two molecules of aldehyde containing α-hydrogen undergo condensation in the
presence of a base to produce β-hydroxy aldehyde known as aldol and the
reaction is known as Aldol reaction. The acetaldehyde (ethanal) molecules
condense in the presence of a base like sodium hydroxide to produce 3-
hydroxybutanal (β-hydroxybutanal). Other bases such as barium hydroxide or
calcium hydroxide may also be used.

Mechanism
Step 1. Abstraction of α-hydrogen of aldehyde by base (formation of enolate
ion):
The α-hydrogen, being acidic, is abstracted by base to produce carbanion.

Step 2. Nucleophilic attack of carbanion on the second molecule of aldehyde:


The carbonyl carbon is electrophilic in nature and is attacked by a nucleophilic
carbanion.

Step 3. Protonation (formation of aldol):

Heating causes elimination of water from aldol to produce an α,β-unsaturated


aldehyde. The overall reaction is known as Aldol condensation. Dehydration can
also be affected by using mineral acids.
The ketones containing α-hydrogen also undergo similar reactions to produce β-
hydroxyketones, that is, ketols. Propanone (acetone) in the presence of a base
undergoes condensation to form
4-hydroxy-4-methylpentan-2-one.

Ketols under acidic conditions or heating undergo dehydration to produce α,β-


unsaturated ketones.

The mesityl oxide may be condensed with another molecule of acetone to form
phorone.

Mixed aldol condensation


Two different aldehydes (both having α-hydrogen) may undergo condensation in
presence of base to give a mixture of different β-hydroxy aldehydes. The
condensation reaction of two different aldehydes is commonly termed as mixed
aldol condensation. For example, ethanal and propanal, in presence of base,
undergo condensation to form a mixture of four products.
• 3-hydroxybutanal (I) formed by self condensation of ethanal.
• 3-hydroxy-2-methylpentanal (II) formed by self condensation of propanal.
• 3-hydroxy-2-methylbutanal (III) by condensation of carbanion from propanal
(CH3– H–CHO) with ethanal.
• 3-hydroxypentanal (IV) by condensation of carbanion from ethanal
(–CH2CHO) with propanal.

Crossed aldol condensation


To avoid formation of a mixture of products, the aldol reaction with different
aldehydes is carried out in such a way that one aldehyde contains α-hydrogen
and the other does not have any α-hydrogen. During reaction, the carbanion is
formed from the α-hydrogen containing aldehyde. For example,

Claisen–Schmidt reaction
The aldol reaction may occur between an aldehyde and a ketone. In a crossed
aldol condensation if one of the component is ketone having α-hydrogen and the
other is an aldehyde with no α-hydrogen, the reaction is called Claisen–Schmidt
reaction.
In general, the reactivity of aldehydic carbonyl group is more compared to the
carbonyl group of ketone. Thus, attack of carbanion occurs on carbonyl carbon
of aldehyde.
Cannizzaro’s reaction
In the presence of alkali, the aldehydes with no α-hydrogen, undergo
intermolecular oxidation and reduction (auto oxidation–reduction or dismutation
of aldehyde). In this case, one molecule of aldehyde is oxidized to sodium salt of
carboxylic acid and the other is reduced to an alcohol.

Cannizzaro’s reaction is a characteristic of aromatic aldehydes as aromatic


aldehyde lacks the α-hydrogen.
Mechanism. The reaction involves hydride shift (intermolecular hydride
transfer) and is explained taking formaldehyde as an example.
Step 1. Attack of base (nucleophile) on carbonyl carbon

Step 2. Intermolecular hydride shift to second aldehyde molecule


In case of dialdehydes (dials) such as glyoxal, the hydride transfer takes place
within the molecule to the adjacent aldehydic group, which results in the
formation of salts of hydroxy acid. This is known as internal Cannizzaro’s
reaction as illustrated.

Mannich reaction
The aldehydes and ketones containing α-hydrogen undergo condensation with
formaldehyde and ammonia (or primary and secondary amines) to produce β-
aminocarbonyl compounds. This condensation reaction is known as Mannich
reaction. The reaction may be acid or base catalyzed. For example,
Tischenko reaction
The aldehydes (with or without α-hydrogen) in presence of aluminium alkoxide
or magnesium alkoxide undergo auto oxidation and reduction. In the process,
one molecule of aldehyde is oxidized to carboxylic acid and the other aldehyde
molecule is reduced to alcohol, which results in the formation of an ester as the
final product of the reaction. For example,
Darzen glycidic ester synthesis
The reaction of a carbonyl compound with an α-hydrogen containing α-haloester
in the presence of a base like sodamide (NaNH2), to produce an α,β-epoxy ester
(known as glycidic ester) is termed as Darzen glycidic ester synthesis. This is an
aldol type condensation reaction and proceeds through an internal nucleophilic
substitution reaction.

Mechanism
Step 1. Abstraction of α-hydrogen from α-haloester by sodamide (formation of
carbanion)

Step 2. Nucleophilic attack of carbanion of haloester on carbonyl carbon (of


aldehyde or ketone)

Step 3. Internal nucleophilic substitution


The nucleophilic oxygen attacks the carbon bearing chlorine and thus the
removal of chloride ion results in the formation of α,β-epoxy ester, that is,
glycidic ester.

Application. The hydrolysis of glycidic esters to carboxylic acid followed by


decarboxylation results in the introduction of –CHO group at the place of C=O
group.

Halogenation in carbonyl compounds


The aldehydes and ketones containing α-hydrogen react with halogens to form α-
halocarbonyl compounds. The electron withdrawing carbonyl group enhances
the acidic strength of α-hydrogen and is thus replaced readily by halogens.

All the α-hydrogens present in the carbonyl compound may be replaced easily
by halogens.

Iodoform reactions (Haloform reaction)


The carbonyl compounds containing CH3CO– group on reaction with a halogen
in presence of an alkali results in the formation of haloform (CHX3) and sodium
salt of carboxylic acid. The halogens used in the reaction may be chlorine,
bromine, or iodine which form corresponding haloforms, namely chloroform
(CHCl3), bromoform (CHBr3), or iodoform (CHI3). Iodoform is a yellow
crystalline solid with sharp melting point whereas chloroform and bromoform
are liquids. The reaction with iodine is popularly known as iodoform reaction
and is used as a qualitative test for detection of CH3CO– group in an organic
compound.
The iodoform reaction is given by all methyl ketones and the only aldehyde,
which gives this reaction, is ethanal (acetaldehyde).

As discussed earlier in Section 13.3.4, p. 472, the iodoform reaction is also given
by ethanol and secondary alcohols which have CH3CH(OH)– group. In the first
step, the CH3CH(OH)– group is oxidized by sodium hypoiodite, NaOI (from
NaOH + I2) to CH3CO– group and then the reaction proceeds as it does with
carbonyl compounds.
Mechanism. The presence of CH3CO– group is necessary for iodoform reaction.
The >C=O group is an electron withdrawing group which makes the α-
hydrogens (CH3– group) acidic in nature.
Step 1. Abstraction of α-hydrogen by base (formation of carbanion)

Step 2. Reaction of carbanion with iodine (formation of α-iodocarbonyl


compound)

Step 3. Formation of triiodocarbonyl compound


The introduction of iodine (an electronegative species) further enhances the
acidic character of α-hydrogens and their abstraction by base occurs more
readily. The repetition of steps 1 and 2 occurs till all the α-hydrogens are
replaced by iodine and a triidocarbonyl compound is formed.

Step 4. Hydrolysis of triiodocarbonyl compound (formation of iodoform)


In presence of an alkali, the cleavage of carbon–carbon bond in C–CO– occurs
and is favoured by the presence of three electron withdrawing iodine atoms
attached to α-carbon of carbonyl compound. As a consequence, iodoform is
finally formed.

17.3.6 Polymerization Reactions of Formaldehyde and


Acetaldehyde
Some of the reactions of carbonyl compounds are specific to aldehydes only.
One such reaction is polymerization reaction. Lower aldehydes are easily
polymerized to form linear chain or cyclic polymers. During polymerization, the
aldehyde units undergo addition at the polar carbonyl group. This section
discusses the polymerization of formaldehyde and acetaldehyde.
Polymers of formaldehyde
Formaldehyde forms three polymers, namely paraformaldehyde, trioxane, and
bakelite.
(i) Linear polymer (Paraformaldehyde). Solution of formalin (38% aqueous
solution of formaldehyde), when evaporated on the steam bath, results in the
formation of a linear polymer called paraformaldehyde having 6–100 monomer
units.
(ii) Cyclic polymer (Metaformaldehyde or Trioxane). In the presence of
sulfuric acid, formaldehyde yields a cyclic trimer called metaformaldehyde
which is also known as symmetrical trioxane.

(iii) Phenol–formaldehyde polymer (Bakelite). In alkaline medium, the


condensation of phenol and formaldehyde results in the hydroxymethylation of
phenol. This is known as Lederer–Manasse reaction. The
hydroxymethylphenol units polymerize to form a polymer known as bakelite
(refer Chapter 31).
Polymers of acetaldehyde
Acetaldehyde forms cyclic polymers. In the presence of concentrated sulfuric
acid, it forms cyclic trimer known as paraldehyde and a tetramer known as
metaldehyde.
17.4 REACTION OF Α,Β-UNSATURATED
CARBONYL COMPOUNDS
The aldol condensation is one of the methods used for preparation of α,β-
unsaturated compounds. The α,β-unsaturated carbonyl compounds undergo
reactions that are characteristics of C=C and C=O systems. These compounds,
may undergo addition reactions at C=C or C=O system which may occur in two
ways to result in 1,2-addition products or 1,4-addition products.
Meaning of 1,2 and 1,4-addition
(Do not confuse with the numbering due to nomenclature)
• 1,2-Addition means the addition of reagent on any adjacent positions in the
substrate.
• 1,4-Addition means the addition of one part of the reagent takes place on the
substrate (position-1) followed by addition of other part of the reagent to the
position-4, with respect to the first addition.
The 1,2-addition may occur as electrophilic addition or nucleophilic addition as
depicted below:

However, in these compounds the C=C participates in delocatization with C=O


group. This delocalization results in 1,4-addition or conjugate addition in α,β-
unsaturated carbonyl compounds.

Due to resonance, during the addition of polar reagent the electrophilic part of
the reagent add to carbonyl oxygen and nucleophilic part to β-carbon. The β-
carbon is electrophilic due to electron withdrawing carbonyl group.
The reactions of α,β-unsaturated carbonyl compounds are summarized below by
taking the examples of propenal (R=H) and but-3-enone (R=CH3)

If the polar reagent has H+ as electrophile part, then 1,4-addition results in enol
that rapidly tautomerizes to give a keto product, which is a 1,2-addition product.
For example,

Strong nucleophilic reagents like Grignard reagent or organolithiums prefer to


attack at carbonyl carbon to give usual 1,2-addition product.
It is interesting to note that Grignard reagent not only attacks at carbonyl carbon
(1,2-addition) but it may also attack at olefinic carbon ( C=C ; 1,4-addition) of
α,β-unsaturated ketones.
On the other hand the organolithium cuprates (Gilman reagent) and enolates
being less basic, attack exclusively at β-carbon. The nucleophilic addition, of
compounds with reactive methylene group like malonic ester or
ethylacetoacetate, to α,β-unsaturated carbonyl compounds in presence of base
(as catalyst) is known as Michael addition. The α,β-carboxylic acid esters or
nitriles also undergo Michael addition reaction.
SELECTED SOLVED EXAMPLES
Example 1. Giving reasons, arrange the following in increasing order of their
boiling points:

Solution. The polar compounds have higher boiling point compared to nonpolar
compounds. Thus, CH3CH2CH2CH3 will have the lowest boiling point.
Alcohol (CH3CH2CH2OH) will have
the highest boiling point due to hydrogen bonding. Carbonyl compounds have
higher boiling points compared to ethers (refer Section 17.1.1). Therefore, the
increasing order of boiling
points is:

Example 2. Arrange the following carbonyl compounds in the decreasing order


of their reactivity towards nucleophilic addition reactions:

Solution. The nueleophilic addition in carbonyl compounds is related to


electrophilicity of carbonyl carbon. A high positive charge on carbonyl carbon
enhances the rate of addition reaction. Another factor which influences the
addition is the steric hindrance. An increased steric hindrance lowers the rate of
nucleophilic addition. The inductive effect and steric factors (Section 17.3.1),
together lead the order of reactivity to be as follows:
Example 3. Explain whether the addition of HCN will be faster with propanal or
propanone. Why is the formation of cyanohydrin much faster in the presence of
a base?
Solution. The addition of HCN will be faster in the case of propanal.

In the presence of a base, formation of the nucleophile –CN is enhanced.

The base (:B) is used as a catalyst in reactions of HCN with carbonyl


compounds.
Example 4. The addition of HCN to carbonyl compounds occurs in presence of
a catalyst, whereas addition of NaHSO3 (sodium bisulfite) does not require any
catalyst. Explain.
Solution. The addition of sodium bisulfite occurs through nucleophilic sulfur.
The –CN is a relatively weaker nucleophile, and thus requires a catalyst (refer
previous problem).
Example 5. Using appropriate carbonyl compounds and Grignard reagents, give
the synthesis of the following alcohols:
(i) Butan-1-ol
(ii) 2-Methylpropanol
(iii) Butan-2-ol
(iv) Cyclohexylmethanol
(v) Hexane-2,5-diol
(vi) 3-Methylpentan-3-ol.
Solution. (i) Primary alcohols are prepared from reaction of Grignard reagent
with formaldehyde as
Example 6. Which of the following carbonyl compounds give a haloform test?
(i) Ethanal
(ii) Propanone
(iii) Propanal
(iv) 2-Methylpentan-3-one
(v) Hexan-2-one
(vi) 2-Methylbutanal
(vii) Pentan-2-one
(viii) Pentan-3-one
Solution. Haloform test is given by carbonyl compounds having CH3CO–
group. The structures of the given compounds clearly indicate that haloform test
will be given by:
(i), (ii), (v) and (vii).
Example 7. A carbonyl compound on reaction with iodine in presence of sodium
hydroxide followed by hydrolysis results in the formation of iodoform and
sodium salt of butanoic acid. Give the structure of carbonyl compound.
Solution. The iodoform test is given by carbonyl compounds containing
CH3CO– group. In a carbonyl compound RCOCH3, the reaction with iodine and
NaOH followed by hydrolysis gives carboxylic acid and iodoform as follows:

Thus, sodium butanoate and iodoform will be obtained from the following
carbonyl compound.

Example 8. Give the structure of the carbonyl compound which on reaction with
iodine and sodium hydroxide followed by hydrolysis gives iodoform and sodium
salt of 3,4-dimethylpentanoic acid.
Solution. The structure of carbonyl compound can be written by writing the
structure of products side by side & replacing the marked part by –CH3 as
shown here:

Example 9. What are the reactions which distinguish aldehydes from ketones?
Solution. The following reactions distinguish aldehydes and ketones (refer
Section 17.3 for details):
(1) Aldehydes reduce Tollens reagent and thus precipitate silver (silver mirror
test).
(2) Aldehydes reduce Fehiling’s solution and Benedict’s solution. Aldehydes
give a red precipitate on reaction with these reagents.
Ketones do not give reactions (1) and (2)
(3) Aldehydes, on oxidation, readily give carboxylic acids with the same
number of carbon atoms.
Ketones are very difficult to oxidize. Oxidation of ketones under drastic
conditions result in formation of carboxylic acids with lesser number of
carbon atoms.
(4) In general, the nucleophilic addition reactions occur at a faster rate with
aldehydes compared to ketones.
(5) Ketones do not form simple addition products with ammonia.
(6) Sterically hindered ketones do not undergo sodium bisulfite addition.
(7) Ketones do not show polymerization. Aldehydes may polymerize to give
different products.
Example 10. Identify the following name reactions and write the products
formed in each case:
Solution. (i) This is an example of Aldol reaction. The reaction occurs as
follows:

(ii) This is an example of crossed aldol condensation where one aldehyde has
no α-hydrogen and other is a ketone with α-hydrogen. The reaction is
known as the Claisen–Schmidt reaction.

(iii) Aldehydes with no α-hydrogen undergo Cannizzaro’s reaction as follows:

Example 11. Complete the following sequence of reactions:


Solution.
EXERCISES
1. Give the IUPAC names for the following compounds:

2. Give the structures and IUPAC names of the isomeric carbonyl compounds
that have the molecular formula C5H10O.
3. Why do the carbonyl compounds have
(a) higher boiling points than alkanes of comparable molecular mass?
(b) lower boiling points compared to alcohols of comparable molecular
mass?
4. How will you prepare the following carbonyl compounds using suitable
Grignard reagents?
(a) Butanal
(b) Pentan-3-one
(c) Pentan-2-one
5. Name the carbonyl compounds obtained by the reductive ozonolysis of
(a) 3-Methylhex-1-ene
(b) 2,3-Dimetylbut-2-ene
(c) Penta-1,3-diene
6. Name the reagent used for the selective oxidation of alcohols to aldehydes.
7. Give the commercial method for the manufacture of formaldehyde and
acetaldehyde.
8. What is formalin?
9. Write the chemical equation and name the product(s) obtained by the
hydrolysis of
(a) But-1-yne
(b) 2,2-Dichlorobutane
(c) 1,1-Dichlorobutane
10. How can aldehydes and ketones be prepared by (a) hydroboration–oxidation
of alkynes and (b) pyrolysis of calcium salts of carboxylic acids?
11. Give the two ‘name reactions’ used for the preparation of aldehydes only.
12. Why dialkylcadmium is preferred over Grignard reagent for preparation of
ketones from acid chloride?
13. Using lithium dialkylcuprate (Gilman reagent), how will you prepare
acetone?
14. Arrange the following carbonyl compounds in the increasing order of their
reactivity towards nucleophilic addition reactions.
HCHO, CH3COCH3, CH3CHO, CH3CH2CHO, CH3CH2COCH3
15. Give the general mechanism for the nucleophilic addition reactions in
carbonyl compounds.
16. What are phosphorous ylides?
17. Giving suitable examples, explain Wittig reaction.
18. Using ethylmagnesium bromide, how will you prepare
(a) Propan-1-ol
(b) Butan-1-ol
(c) 2-Methylbutan-2-ol
(d) Pentan-3-ol
19. Complete the following reactions:

20. What will be the products obtained by Baeyer–Villiger rearrangement of


(a) Propanal
(b) Pentan-2-one
21. Give the reaction of (a) Butanal and (b) Cyclopentanone that occurs when
they react with the following nitrogen nucleophiles:
(a) Hydroxylamine
(b) Semicarbazide
(c) 2,4-DNP
(d) Hydrazine
22. What will be the products obtained by the following reduction reactions:

23. A carbonyl compound in the presence of a base results in the formation of


crotonaldehyde (But-2-enal). Identify the carbonyl compound, indicate the
name reaction, and give its mechanism.
24. Give two examples each of the addition reactions of carbonyl compounds
with:
(a) sulfur nucleophile
(b) carbon nucleophile
(c) oxygen nucleophile
25. How will you distinguish between following compounds:
(a) CH3CHO and CH3COCH3
(b) CH3CH2CHO and CH3CHO
(c) CH3COCH2CH2CH3 and CH3CH2COCH2CH3
26. Complete the following reactions and indicate the name reaction:

27. Explain with mechanism, the iodoform reaction in carbonyl compounds.


28. Which of the following compounds will give a positive iodoform test and
why?

29. Why, in Reformatasky reaction, magnesium or cadmium cannot be used in


place of zinc?
30. Write the structure of linear and cyclic polymers of formaldehyde.
Chapter18
Aromatic
Aldehydes and Ketones

18.1 INTRODUCTION
Aromatic aldehydes and ketones belong to the class of oxygen containing
organic compounds where –CHO or –COR (where R may be alkyl or aryl) group
is directly attached to benzene ring system so they are termed as aromatic
carbonyl compounds. In the carbonyl group carbon and oxygen are sp2
hybridized and carbonyl carbon is bonded to benzene ring through sp2–sp2
overlap. The simplest aromatic aldehyde is benzaldehyde. An aromatic aldehyde
does not contain α-hydrogen. Aromatic ketones may contain alkyl or aryl group.
Di-aryl ketones do not contain α-hydrogen. The nomenclature of aromatic
carbonyl compounds has already been discussed in detail in Chapter 2.

18.1.1 Physical Properties


Generally aromatic aldehydes are colourless liquids or solids and exhibit a
characteristic odour. Benzaldehyde smells like bitter almonds. Aromatic ketones
are generally pleasant smelling liquids and are used in perfumery. Aromatic
aldehydes and ketones are insoluble in water but soluble in alcohol and ether.
The boiling or metting points of some aromatic carbonyl compounds are given
along with their IUPAC nomenclature (and common names) as follows:
18.2 PREPARATION OF AROMATIC ALDEHYDES
AND KETONES

18.2.1 Oxidation Reactions


By the oxidation of aromatic hydrocarbons
The oxidation of aromatic hydrocarbons with manganese dioxide and sulfuric
acid under controlled conditions (~40°C) results in the formation of aromatic
aldehyde. Under high temperature conditions, the oxidized product is an
aromatic carboxylic acid. Toluene on oxidation with manganese dioxide and
sulfuric acid results in the formation of benzaldehyde. Ethylbenzene undergoes
oxidation in oxygen, in the presence of manganese acetate to yield
acetophenone.

Etard reaction
The oxidation of aromatic hydrocarbons to get better yields of aromatic
aldehydes makes use of chromyl chloride as an oxidizing agent and the reaction
is known as Etard reaction.
The reaction results in the formation of an intermediate, which on hydrolysis
gives quantitative yield of aldehyde.
The oxidation of –CH3 group of the aromatic hydrocarbon to aldehyde (–CHO)
group cannot be carried out easily, if oxidizable groups like –OH and –NH2 are
present on the ring system.
Oxidation of aromatic alcohols
Benzyl alcohol, on oxidation with acidified potassium dichromate results in the
formation of benzaldehyde. The oxidation of secondary aromatic alcohols under
similar conditions results in the formation of aromatic ketones.

18.2.2 By Heating Calcium Salts of Carboxylic Acids


Calcium salts of a mixture of aromatic carboxylic acid and aliphatic acid are
heated to form
aromatic aldehydes and ketones (aryl alkyl ketones). For example, heating
calcium benzoate
and calcium formate results in the formation of benzaldehyde, whereas heating
calcium
benzoate and calcium acetate yields acetophenone. Benzophenone is formed
when calcium benzoate is heated alone. The reactions are as follows
However, the yield of ketones obtained by this method is poor.

18.2.3 From Grignard Reagent

18.2.4 Electrophilic Substitution of Aromatic


Hydrocarbons
Gattermann–Koch reaction
An aldehyde group is directly introduced by treating benzene with carbon
monoxide and hydrogen chloride in the presence of aluminium chloride as a
catalyst.

This is an electrophilic substitution reaction (modified Friedel–Crafts reaction),


where the most likely electrophile is acylium ion.

Gattermann aldehyde synthesis


This reaction is a modified Gattermann–Koch synthesis. The reagent used is
hydrogen cyanide and hydrogen chloride in the presence of AlCl3 (as catalyst),
which basically forms imidoformyl chloride. The reaction of imidoformyl
chloride with benzene results in the formation of an aryl imine, which on
hydrolysis produces benzaldehyde.

Mechanism

The modified Gattermann aldehyde synthesis use Zn(CN)2 and and HCl which
makes the reaction easy to carry out.
Gattermann–Koch reaction versus Gattermann aldehyde
synthesis
(a) The Gattermann–Koch reaction does not occur in case of phenol,
phenolic ethers and nitrobenzene.
(b) The Gattermann aldehyde synthesis can be carried out in case of
phenols and phenolic ethers but not with nitrobenzene (or any
other deactivating group).

Reimer–Tiemann reaction
The reaction of phenol with chloroform in presence of sodium hydroxide results
in the formation of o- and p-hydroxybenzaldehydes and the reaction is known as
Reimer–Tieman reaction. This is an electrophilic substitution reaction and
proceeds via the formation of dichlorocarbene as electrophile. Salicylaldehyde
(o-hydroxybenzaldehyde) is the major product of Reimer–Tieman reaction
because of the stabilization that occurs through intramolecular hydrogen bonding
(p. 543).

Friedel–Crafts reaction
One of the most widely used reaction for the preparation of aromatic ketones is
the Friedel–Crafts reaction (Section 10.2.4). Benzene reacts with acetyl chloride
in the presence of AlCl3 (as catalyst) to form acetophenone. This reaction is
known as Friedel–Crafts acylation reaction. Use of benzoyl chloride instead of
acetyl chloride results in the formation of benzophenone.

One of the most economical and frequently used methods for the preparation of
benzophenone involves reaction of two molecules of benzene with
carbontetrachloride in the presence of AlCl3 catalyst.

Benzophenone may also be prepared by the reaction of benzene with phosgene,


COCl2 in presence of AlCl3 as catalyst.

Fries rearrangement
Phenolic esters, on heating with aluminium chloride, undergo rearrangement to
form phenolic ketones (p. 540).
Hoesch condensation
Reaction of polyhydric phenols with alkanenitriles in presence of HCl and
anhydrous ZnCl2 in ether as solvent results in the formation of ketones. For
example, reaction of 1,3,5-trihydroxybenzene (phloroglucinol) with
methanenitrile results in the formation of 2,4,6-trihydroxyacetophenone.

18.2.5 Reactions Used for the Preparations of


Aromatic Aldehydes
Oxidation of benzyl chloride
Benzyl chloride, in the presence of lead nitrate as an oxidizing agent gets
oxidized to benzaldehyde in the manner as shown in the following reaction:

Sommelet reaction
Benzyl chloride on reaction with hexamethylenetetramine, using alcohol as
solvent followed by acid hydrolysis, results in the formation of benzaldehyde.
Hydrolysis of benzal chloride (gem dihalide)
Chlorination of toluene in the presence of sunlight results in the formation of
benzal chloride. The hydrolysis of benzal chloride in alkaline medium then
forms benzaldehyde.

Rosenmund reduction
Benzoyl chloride on catalytic reduction in the presence of Pd/BaSO4 results in
the formation of benzaldehyde.

Stephen reaction
The aromatic nitriles on reduction with stannous chloride in presence of HCl
produce aldimine complex. The main reducing species is tetrachlorostannic acid,
H2[SnCl4], which is formed by reaction of HCl and SnCl2. The hydrolysis of
aldimine complex results in the formation of aromatic aldehyde.
18.3 CHEMICAL PROPERTIES OF AROMATIC
ALDEHYDES AND KETONES
The aromatic carbonyl compounds undergo various types of chemical reactions
which include
(i) reaction involving carbonyl ( C=O) groups, namely the nucleophilic addition
reactions, oxidation reactions, and reduction reactions, (ii) reactions involving α-
carbon of carbonyl compounds, (iii) some typical reactions of aromatic
aldehydes, and (iv) electrophilic substitution reactions. The various reactions are
discussed as follows:

18.3.1 Reactivity of Carbonyl Group: A General


Discussion
The high elctronegativity of oxygen makes the carbon–oxygen double bond
polar in nature, which makes them undergo characteristic nucleophilic addition
reactions. The reactions are similar to their aliphatic counterparts (refer Section
17.3.1). The aromatic aldehydes are more reactive compared to aromatic
ketones, however, the reactivity of aromatic carbonyl compounds is less
compared to aliphatic carbonyl compounds due to steric factor. The order of
reactivity may be generalized as:

Benzaldehyde and benzophenone do not undergo the reactions involving


reactivity at α-carbon of the carbonyl compounds due to absence of α-hydrogen.
However, there are some reactions, which are typical of aromatic aldehydes and
are not exhibited by aliphatic carbonyl compounds. The aromatic carbonyl
compounds undergo electrophilic substitution reactions. Since carbonyl group is
an electron withdrawing group (–R effect and –I effect), the aromatic aldehydes
and ketones deactivate the aromatic ring towards electrophilic substitution and
behave as meta-directors.
18.3.2 Nucleophilic Addition Reactions
The aromatic carbonyl compounds undergo nucleophilic addition reactions
similar to their aliphatic counter parts. The reactions occur with carbon, oxygen,
nitrogen, and sulfur nucleophiles. The general mechanism of nucleophilic
addition reactions is similar to that discussed for aliphatic carbonyl compounds
(for details, refer section 17.3.2).
Reaction with carbon nucleophiles
(i) With HCN. The aromatic carbonyl compounds react with hydrogen cyanide
to form cyanohydrin.

The hydrolysis of cyanohydrin is often used as a method for synthesizing


hydroxy acids.
(ii) With organometallic compounds
[Grignard reagent and organolithium]
The alkyl or aryl group in organometallic compounds behaves as a nucleophile
and adds on to aromatic carbonyl compounds to form an adduct. The hydrolysis
of this adduct results in formation of corresponding alcohols. Aromatic
aldehydes give 2° alcohols while aromatic ketones form
3° alcohols. A few illustrative reactions of aromatic aldehydes and ketones with
methylmagnesium iodide (Grignard reagent) are given as follows:
with benzaldehyde:

with acetophenone:

with benzophenone:

Aromatic aldehydes and ketones undergo similar reactions with methyllithium.


(iii) Wittig reaction. As discussed earlier (Section 17.3.2; p. 582), Wittig
reaction involves reaction of phosphorous ylides with carbonyl compounds to
form alkenes. The reaction proceeds via the formation of the betaine
intermediate in the following manner:
with benzaldehyde:
with acetophenone:

with benzophenone:

(iv) Reformatsky reaction. Reaction of aromatic aldehydes and ketones with α-


haloesters in the presence of zinc to form β-hydroxyesters is known as
Reformatsky reaction. The reaction proceeds via the formation of an
organozinc derivative (for mechanism refer Section 17.3.2; p. 584).
with benzaldehyde:

In aryl aldehydes, at times after the hydrolysis, elimination occurs and the
product is an α,β-unsaturated ester.

with acetophenone:
with benzophenone:

Reaction with sulfur nucleophiles


(i) Addition of sodium bisulfite. Aromatic aldehydes react with sodium
bisulfite to form bisulfite addition product (adduct). The attack of sodium
bisulfite occurs through sulfur, which is a better nucleophile compared to oxygen
(p. 592). The products formed are identified as sulfonates.

Aryl ketones do not give addition product (adduct) with


sodium bisulfite due to steric factors.

(ii) Addition of thiols. The reaction of carbonyl compound with thiols is much
faster compared to reaction with alcohol due to high nucleophilicity of sulfur
compared to oxygen. Acid catalyzed addition of thiols to carbonyl compounds
results in the formation of thioacetals. Thioacetals are used for the protection of
carbonyl group in multi-step organic synthesis. The reduction of thioacetals with
raney nickel results in the formation of hydrocarbons.

Reactions with oxygen nucleophiles


(i) Addition of alcohol. Aromatic aldehydes undergo addition of alcohol in
acidic medium to form hemiacetal, which further adds another molecule of
alcohol resulting in the formation of acetal.

Aryl ketones do not undergo addition of alcohol to form


hemiketals and ketals because of steric reasons.

(ii) Baeyer–Villiger rearrangement. In presence of acid catalyst, the aromatic


ketones react with peroxyacids to form esters. The mechanism (p. 587) involves
the migration of alkyl or aryl group of ketones to form corresponding ester.
Aromatic aldehyde on reaction with peroxyacid results in the
formation of carboxylic acid instead of ester due to the migration of
hydrogen instead of the phenyl group.

Addition of nitrogen nucleophiles


(i) Reaction with primary amines. Reaction of primary amines with aromatic
aldehydes or ketones results in the formation of an addition product, which
readily undergoes elimination of water to form stable compounds known as
Schiff’s bases.
(ii) Reaction with ammonia derivatives. The ammonia derivatives include
hydroxylamine, hydrazine, semicarbazide, phenylhydrazine, and 2,4-
dinitrophenylhydrazine. The nucleophilic addition of ammonia derivatives to
carbonyl compounds in acidic medium results in the formation of addition
products, which undergo elimination of water to give corresponding solid
crystalline products that have sharp melting points. The reactions are as follows:
General reaction with ammonia derivatives
The reaction of carbonyl compounds with 2,4-
dinitrophenylhydrazine (known as DNP reagent) results in the
formation of yellow/orange crystalline product and is used as a
qualitative test for the detection of carbonyl compounds.

Beckmann rearrangement
The reaction of ketones with hydroxylamines results in the formation of
ketoximes. The ketoximes (R2C=NOH) in presence of acidic catalysts such as
PCl5 in ether, H2SO4, P2O5, BF3, SOCl2, SO3, and so on undergo
rearrangement to form N-substituted amides. This reaction is known as
Beckmann rearrangement. The reaction was first observed by Beckmann in case
of benzophenone oxime.

The reaction of oximes of unsymmetrical ketones in acidic


medium revealed the fact that Beckmann rearrangement is
stereospecific in nature. During rearrangement the group that migrates
to nitrogen is the one, which is present at a position anti- to –OH
(hydroxyl) group of oxime.
Mechanism of Beckmann rearrangement. It is an intramolecular
rearrangement (1,2-shift) reaction. The rate of reaction is accelerated in presence
of more nucleophilic migration groups. The mechanism for this oxime-amide
rearrangement, in case of acetophenone oxime, in presence of an acid catalyst is
as follows:

18.3.3 Oxidation
Aromatic aldehydes are less susceptible to oxidation compared to aliphatic
aldehydes. Aromatic aldehydes can reduce Tollens reagent to metallic silver salt
and give positive silver mirror test.

Aromatic aldehydes do not react with Fehling’s solution or Benedict’s solution.


Similar to their aliphatic counterparts, the aromatic ketones are not oxidized by
mild oxidizing agents and show no reaction with Tollens reagent, Fehling’s
solution, and Benedict solution.
Strong oxidizing agents such as acidified potassium dichromate or potassium
permanganate oxidize aromatic aldehydes to carboxylic acids.
These strong oxidizing agents oxidize aryl alkyl ketones to aromatic carboxylic
acids. During oxidation, the cleavage of alkyl group from carbonyl carbon takes
place.

18.3.4 Reduction
The reduction of aromatic carbonyl compounds results in the formation of
different products depending upon the type of reducing agent used.
Catalytic reduction
The hydrogenation of aromatic carbonyl compounds in the presence of metal
catalyst results in the formation of alcohol.

Sodium in ethanol (Bouveault–Blanc reduction) is also used for converting


carbonyl compounds to alcohols.
With metal hydrides
Lithium aluminiumhydride (LiAlH4) and sodium borohydride (NaBH4) reduce
carbonyl compounds, selectively, to alcohols. These reducing agents do not
reduce double or triple bond present along with carbonyl group.

Meerwein–Ponndorf–Verley reduction
Aromatic carbonyl compounds on treatment with a solution of aluminium
isopropoxide in isopropyl alcohol get reduced to alcohols.

This reagent does not reduce other groups, namely the double bond, triple bond,
and the nitro group present in compound. For example,

Clemmensen reduction
The carbonyl ( C=O) group of aromatic aldehydes and ketones can be reduced to
methylene ( CH2) group by zinc amalgam in a manner similar to aliphatic
carbonyl compounds. This reaction is used for reducing carbonyl compounds,
which are sensitive to alkali.

Wolff–Kishner reduction
The carbonyl group ( C=O) of aromatic aldehydes and ketones can be reduced to
methylene ( CH2) group by reaction with hydrazine followed by hydrolysis with
sodium or potassium ethoxide as in the case of aliphatic carbonyl compounds.
The method is used for the reduction of carbonyl compounds that are sensitive to
acids.

Reduction to pinacols
Aliphatic aldehydes do not form pinacols unlike aromatic aldehydes and ketones
that form pinacols (di-tert 1,2-diols) on reduction. The reducing agent used in
the process is zinc in acetic acid. Further in the presence of light and isopropanol
as a solvent, benzophenone yields benzopinacol. The reaction is termed as
photochemical reduction of benzophenone. The reaction with different aromatic
carbonyl compounds are as follows:
with benzaldehyde:

with acetophenone:
with benzophenone:

18.3.5 Typical Name Reactions of Aromatic Carbonyl


Compounds
Cannizzaro’s reaction
Aromatic aldehydes do not contain α-hydrogen and under alkaline conditions
undergo intermolecular oxidation and reduction to form alcohol and salt of
aromatic carboxylic acid as the final products.

The reaction of benzaldehyde with formaldehyde is an example of crossed


Cannizzaro’s reaction where formaldehyde is oxidized and benzaldehyde is
reduced. This occurs because aliphatic aldehydes are oxidized much more
readily compared to aromatic aldehydes.
Mechanism. The mechanism of the Cannizzaro’s reaction involves an
intermolecular hydride shift. The steps so involved are:
Step 1. Attack of base (nucleophile) on electrophilic carbon of carbonyl group

Step 2. Removal of hydride ion and its shift to second molecule of benzaldehyde
(with no
α-hydrogen)

Claisen–Schmidt reaction
The reaction of benzaldehyde with α-hydrogen containing ketone in presence of
a base results in the formation of a β-hydroxy ketone. This reaction is known as
Claisen–Schmidt reaction. The aromatic aldehydes cannot undergo Aldol
condensation due to lack of α-hydrogen. However, they participate in crossed
Aldol condensation reactions (p. 601), where one aldehyde does not contain α-
hydrogen (as in aromatic aldehydes) and the other is a α-hydrogen containing
ketone.
Generally, the dehydration of hydroxy compound takes place to form α, β-
unsaturated carbonyl compounds.
Mechanism. This is a base catalyzed reaction. The step wise Mechanism of the
reaction is as follows:
Step 1. Abstraction of α-hydrogen (formation of carbanion)
In alkaline medium, carbonyl compound containing α-hydrogen looses a proton
and forms an enolate ion (carbanion).

Step 2. Attack of nucleophile (carbanion) on second molecule, that is, aromatic


aldehyde
The aromatic aldehyde does not have any α-hydrogen and therefore it behaves as
a carbanion acceptor.

Step 3. Proton abstraction


Proton is abstracted from water to form β-hydroxyketone which readily
undergoes dehydration to form α,β-unsaturated carbonyl compound.

Benzoin condensation
Aromatic aldehydes, in the presence of cyanide ion condense to form α-hydroxy
ketones. In case of benzaldehyde, the condensation in the presence of cyanide
ion as a catalyst results in the formation of benzoin.
Mechanism
Step 1. Attack of nucleophile (cyanide ion) on the aromatic aldehyde
The –CN is a good nucleophile, which attacks the carbonyl carbon of the first
molecule of benzaldehyde. The –CN is a good electron withdrawing group and
increase the acidic character of C-H bond. This results in the removal of proton,
which is rapidly transferred to oxygen. The cyanide group helps in stabilization
of carbanion thus formed.

Step 2. The addition of carbanion to second molecule of benzaldehyde

Step 3. loss of –CN (formation of benzoin)


The –CN is a good nucleophile as well as a good leaving group. Rapid loss of
–CN results in the formation of benzoin, a 2-hydroxy ketone.
Benzil–Benzilic acid rearrangement
The condensation of benzaldehyde in the presence of cyanide ion (benzoin
condensation) results in the formation of benzoin. Oxidation of benzoin (2-
hydroxy ketone) results in the formation of benzil, a diketone. Benzil, in
presence of a base undergoes a rearrangement reaction to form benzilic acid, an
α-hydroxy acid.

Mechanism
Step 1. Attack of nucleophile (base –OH) on carbonyl carbon

Step 2. Migration of phenyl group to carbonyl carbon (intramolecular carbanion


addition)
The phenyl group along with its shared electron pair migrates to the carbonyl
carbon. Phenyl behaves as carbanion and its addition occurs on carbonyl carbon
to form α-hydroxy acids.

The reaction may be considered as an analogous reaction to the intramolecular


Cannizzaro’s reaction. In contrast to hydride shift in Cannizzaro’s reaction, it is
the phenyl group (with electron pair), which migrates to carbonyl group.
Perkin condensation
Aromatic aldehydes on reaction with aliphatic acid anhydrides, containing two
α-hydrogens, in the presence of sodium salt of same aliphatic acid result in the
formation of α,β-unsaturated carboxylic acid. For example, reaction of
benzaldehyde with acetic anhydride in the presence of sodium acetate results in
the formation of cinnamic acid.

Mechanism
Step 1. Abstraction of α-hydrogen of acid anhydride by base (formation of
carbanion)
The anion of the acid acts as a base and removes α-hydrogen from the anhydride.

Step 2. Attack of carbanion on carbonyl carbon of benzaldehyde

Step 3. Intramolecular acetyl shift


The migration of acetyl group occurs via a cyclic intermediate to alkoxy oxygen.
Step 4. Loss of acetate ion from β-carbon
In the presence of a base, the α-hydrogen is abstracted which in turn causes the
removal of good leaving group CH3COO– to form α,β-unsaturated carboxylic
acid.

Reactions of aryl methyl ketones


The aryl methyl ketones like acetophenone contain α-hydrogen and undergo
reactions similar to aliphatic methyl ketones. Some of these reactions are:
(i) Iodoform reaction [Haloform reaction]. The reaction of acetophenone with
iodine in the presence of an alkali results in the formation of iodoform and
sodium benzoate (for mechanism refer p. 606).

(ii) Mannich reaction. The condensation of acetophenone (containing α-H) with


formaldehyde and amine (1° or 2°) results in the formation of β-aminoketones
known as Mannich bases. (For mechanism refer p. 603).
18.3.6 Electrophilic Substitution Reactions
The aromatic aldehydes and ketones undergo electrophilic substitution reactions.
The carbonyl group due to its negative resonance (–R) effect and negative
inductive (–I) effect behaves as a ring deactivator and meta-director towards
electrophilic substitution reactions.

The aromatic aldehydes and ketones undergo usual halogenation, nitration, and
sulfonation reactions to produce corresponding m-halo, m-nitro, and m-
substituted sulfonic acid derivatives (for mechanism and detailed discussion,
refer Chapter 10).
Being electron withdrawing in nature, the aromatic aldehydes and ketones do not
undergo Friedel–Crafts reaction.
SELECTED SOLVED EXAMPLES
Example 1. Identify the following name reactions and write the product(s)
formed in each case:

Solution. (i) This is an example of crossed aldol reaction:

(ii) Cross Cannizzaro’s reaction; formaldehyde can be easily oxidized as


compared to aromatic aldehydes. An aromatic aldehyde forms alcohol as
the reduction product.

(iii) No reaction takes place as Perkin condensation requires presence of


atleast two α-hydrogens in acid anhydride (see text).
(iv) Benzoin condensation of aromatic aldehyde
(v) “Aldol reaction” in aromatic ketones having α-hydrogen.

Example 2. Carry out the following conversions:


(i) Benzaldehyde to 3-bromo-5-nitrobenzaldehyde
(ii) Chlorobenzene to 2-hydroxy-5-nitrobenzaldehyde.
(iii) Benzene to 2,3-diphenylbutane-2,3-diol.
(iv) Benzaldehyde to diphenylglycolic acid.
Solution.

Example 3. What product(s) will be formed on treating benzalacetone with


bromine and sodium hydroxide?
Solution. The benzalacetone is a ketone with group. It gives haloform
reaction and reaction is as follows:
EXERCISES
1. Give the IUPAC names for the following compounds.

2. Name the carbonyl compounds obtained by the oxidation of


(a) p-Nitrotoluene with chromyl chloride
(b) 3-Ethylnitrobenzene with oxygen in presence of manganese acetate
(c) 1-Phenylethanol with acidified dichromate
3. Using appropriate Grignard reagent, synthesize the following carbonyl
compounds.

4. Complete the following reactions and identify the ‘name reaction(s)’:

5. Discuss two name reactions, which are used for the preparation of aromatic
aldehydes only.
6. Arrange the following carbonyl compounds in increasing order of their
reactivity towards nucleophilic addition reactions.

7. How will you prepare ethyl phenyl ketone from acetophenone?


8. Using appropriate carbonyl compound and CH3MgI, how will you prepare

9. Explain with mechanism, the Reformatsky reaction in aromatic aldehydes and


ketones.
10. Identify the starting aromatic carbonyl compound and phosphorous ylide
which on Wittig reaction yield the following alkenes:

11. Why do aromatic ketones not form addition product with sodium bisulfite?
12. Complete the following reactions:
13. Give the reduction product of following carbonyl compounds in the
presence of indicated reducing agents:

14. Complete the following by substituting appropriate reagent needed for the
reactions to take place.

15. Identify the name reactions and the final products formed in the following
reactions:
16. How will you distinguish between the following carbonyl compounds:

17. Carry out the following conversions:


(a) Benzaldehyde to Mandelic acid (2-hydroxy-2-phenylethanoic acid)
(b) Benzaldehyde to styrene
(c) Acetophenone to ethylbenzene
(d) Benzaldehyde to acetophenone
(e) Benzaldehyde to cinnamaldehyde.
18. Give the reaction mechanism for the following.
(a) Benzoin condensation
(b) Benzil–benzillic acid rearrangement
19. Explain with mechanism, the Beckmann rearrangement reaction in
acetophenone.
20. Give the products formed in following reactions:
Chapter19
Aliphatic
Carboxylic Acids
and Their Derivatives

19.1 INTRODUCTION
Aliphatic carboxylic acids belong to the class of oxygen containing organic
compounds having a caboxyl group. In a carboxyl functional group, a hydroxy
group is attached to carbonyl group. The general formula of carboxylic acids is
CnH2n+1COOH. In general, many of the compounds with this general formula
are obtained by hydrolysis of fats and for this reason carboxylic acids are also
termed as fatty acids. The carboxylic acids have a planar structure due to planar
carbonyl group.
The carboxylic acids in IUPAC system are termed as alkanoic acids (refer
Chapter 2). The common and much familiar names are derived from the source
of these acids and are widely accepted. A few examples of IUPAC name for
carboxylic acids are given in Table 19.1.
Physical properties. The carboxylic acids exhibit high polarity because of the
presence of carbonyl (–CO–) and hydroxy (–OH) group, and are thus able to
form hydrogen bonds with other polar molecules.

The carboxylic acids generally exist in dimeric form where the hydroxy group of
one carboxylic acid molecule is hydrogen bonded to carbonyl oxygen of the
second molecule of carboxylic acid. In a similar manner, the hydroxy group of
second molecule is hydrogen bonded to carbonyl oxygen of the first molecule.
Thus, carboxylic acid molecules are held together strongly through
intermolecular hydrogen bonding in solid, liquid, and in some cases even
gaseous phase. Due to this reason, carboxylic acids have high melting and
boiling points compared to hydrocarbons and alcohols of comparable mass.

Branching in the alkyl chain of carboxylic acids decreases the boiling point.
Boiling points (°C) of a few monocarboxylic acids are given in Table 19.1.

In a similar manner, carboxylic acid molecules can form hydrogen bonding with
alcohol and water. The lower members of carboxylic acids (C4 and below) are
thus, soluble in water. This is depicted in the following structure.
19.2 PREPARATION OF ALIPHATIC
CARBOXYLIC ACIDS
This section discusses various methods used for preparation of monocarboxylic
acids. Some of the methods have already been discussed in details in other
chapters and for such reactions only the section numbers are given for reference.

19.2.1 Oxidation Reactions


Oxidation of ethene (Industrial synthesis of ethanoic acid)
The oxidation of ethene in presence of palladium chloride and copper chloride
catalysts results in the formation of ethanal (acetaldehyde). This is known as
Wackers process. Further oxidation of acetaldehyde with cobalt catalyst results
in the formation of acetic acid.

The following oxidation methods for preparation of carboxylic acids have been
discussed in details in previous chapters and their references are as follows:
• Oxidation of alkanes (Section 5A.3.5)
• Oxidative cleavage of alkenes with hot KMnO4 (Section 6.3.4)
• Oxidation of monohydric alcohols (Section 13.3.4)
• Oxidation of aldehydes and ketones (Section 17.3.3)

19.2.2 Hydrolysis
Alkaline hydrolysis of trihaloalkanes
Trihaloalkanes on treatment with alkali solution (aq NaOH or KOH) undergo
nucleophilic substitution to form gem. triols which readily undergo elimination
of water molecule to form carboxylic acids. For example,
Hydrolysis of alkanenitriles
The hydrolysis of alkanenitriles in hot acidic or alkaline medium results in the
formation of carboxylic acid with liberation of ammonia. This is one of the
convenient and preferred methods for the introduction of carboxylic functional
group. For example,

Mechanism. The hydrolysis of nitrile to carboxylic acid proceeds via formation


of an amide.

Acid catalyzed hydrolysis. This involves the following steps:


Step 1. Protonation of nitrile and nucleophilic attack of water
In presence of an acid, the protonation occurs at nitrogen of nitrile, which
increases the electrophilic character of carbon of –CN. This electrophilic carbon
is attacked readily by nucleophile, that is, water molecule in this case.

Step 2. Loss of proton (formation of amide intermediate)


Step 3. Hydrolysis of amide (formation of carboxylic acid)
Base catalyzed hydrolysis. This involves the following steps:
The nucleophilic attack of base on the carbon of nitrile group followed by the
hydrolysis results in the formation of carboxylic acid. Various steps involved in
the mechanism are as follows:

Hydrolysis of esters
The hydrolysis of esters results in the formation of alcohols and carboxylic
acids. The hydrolysis may be carried out in acidic or alkaline medium. The
hydrolysis in alkaline medium is termed as saponification reaction. This has been
shown as follows:

The alkaline hydrolysis of triglycerides is used to prepare soap. The mechanism


for ester hydrolysis catalyzed by acid or base is as follows:
Mechanism. Acid catalyzed hydrolysis
Mechanism. Base catalyzed hydrolysis

Acid hydrolysis of active methylene compounds


The active methylene compounds have –CH2 (methylene) group sandwiched
between two electron withdrawing groups, for example, ethyl acetoacetate,
malonic acid, and its derivatives. Hydrolysis of these active methylene
compounds, with base, result in the formation of acid as the final product. (Refer
also Section 20B.2 and 20B.3)
19.2.3 Carboxylation of Oganometallic Compounds
(Carbonation)
The organometallic compounds such as Grignard reagents (RMgX) and
alkyllithiums (RLi)
have alkyl groups, which are nucleophilic in nature. These organometallic
compounds undergo addition of carbon dioxide (electrophile) to form salt of
carboxylic acid, which on hydrolysis yield free acid.

(i) Reaction with Grignard reagent

(ii) Reaction with alkyllithiums. The carbonation of alkylithium is similar to


Grignard reagent and occurs in the following manner:

In both the cases, carboxylic acid produced contains one carbon more than the
alkyl group present in organometallic compounds.

19.2.4 Carbonylation Reactions


Koch reaction: Reaction of alkene with carbon monoxide
The alkenes undergo addition of carbon monoxide and water in acidic medium
(sulfuric acid or phosphoric acid) to form carboxylic acids. The mechanism for
the reaction is also explained here as under:

Mechanism
Step 1. Formation of carbocation
In presence of an acid, the alkene undergoes protonation to yield a stable
carbocation.
Step 2. Attack of carbon monoxide on carbocation
Step 3. Addition of water to electrophilic carbon
All these steps are shown in the following reaction sequence:
Industrial methods for preparation of formic acid and acetic acid by
carbonylation reactions
Heating carbon monoxide with sodium hydroxide under pressure results in the
formation of sodium formate, which on hydrolysis yields formic acid.

The addition of carbon monoxide to alcohol in the presence of a catalyst like


Rh3+ or BF3 at 180°C results in the formation of carboxylic acid. This method
is used for industrial preparation of acetic acid from methanol.

The carbonylation of sodium alkoxides also results in the formation of


carboxylic acid.

Haloform reactions
The reader may refer to Section 17.3.5; p. 606 for a detailed discussion on
haloform reactions, as this reaction involves the formation of carboxylic acids
along with haloform (CHX3).
19.3 CHEMICAL PROPERTIES OF ALIPHATIC
CARBOXYLIC ACIDS
The carboxylic acids show acidic as well as basic properties. The important
reactions of carboxylic acids involve the reactions due to carboxylic group,
hydroxy group, carbonyl group, and the alkyl part of the carboxylic acid
depicted as follows:

The various chemical reactions of carboxylic acids are discussed as follows.

19.3.1 Acidic Character of Carboxylic Acids: A


General Discussion
The carboxylic acids are one of the most acidic organic compounds. Their acidic
strength is much higher compared to water and phenol. The loss of proton in
carboxylic acid forms conjugate base, that is, carboxylate ion. The high acidic
strength is attributed to resonance stabilization of carboxylate ion.

Due to delocalization in carboxylate ion, both the C–O bond lengths are same.
However in carboxylic acids, these two C–O bond lengths are different.
Effect of substituents on acidic strength
The presence of electron withdrawing group increases the acidic strength of
carboxylic acids. This effect is more pronounced if electron withdrawing group
is present on α-carbon which is close to carboxylic group. The electron
withdrawing group/atom (–I effect) delocalizes the negative charge thereby
stabilizing the carboxylate ion. For example, chloroacetic acid is more acidic
than acetic acid.

It should be noted that the electron withdrawing nature of halogens (–I effect)
decreases due to decrease in electronegativity and follows the order F > Cl > Br
> I. Thus, in α-haloacetic acids the decreasing order of acidic strength is as
follows:

The presence of electron releasing groups (+I effect) decreases the acidic
strength. This happens as these groups increase or intensify the negative charge
thereby destabilizing the carboxylate ion. As a result, acetic acid is more acidic
than propanoic acid. Similarly, more alkyl groups attached to acetic acid, further
decrease the acid strength. Thus, for alkylated acetic acids the decreasing order
of acidic character is as follows:
19.3.2 Reactions Involving Acidic Hydrogen
Reaction with bases
The carboxylic acids show considerable acidic character and form salts even
with mild alkalis. For example,
The reaction of carboxylic acids with sodium bicarbonate is
used as a qualitative test for detection of carboxylic functional group
in an organic compund. Carboxylic acids on reaction with aqueous
sodium bicarbonate solution produce brisk effervescence. The sodium
salts of carboxylic acids are highly soluble in water.
Also the acids react with metals to form metallic salts with the liberation of
hydrogen.

Reaction with diazomethane (formation of esters)


Carboxylic acids, on reaction with diazomethane form methyl esters. The
reaction gives high yields of methyl esters at room temperature. For example,

Mechanism. The steps involved in the mechanism are as follows:


Step 1. Dissociation of acid
Step 2. Protonation of diazomethane

Step 3. Nucleophilic substitution by carboxylate ion

19.3.3 Reactions Involving Replacement of –OH


Group
Nucleophilic acyl substitution reaction through addition–elimination
mechanism
The carbonyl carbon of carboxylic acid, being electrophilic, undergoes addition
by a reagent with a nucleophilic centre to form a tetrahedral intermediate, which
eliminates water (leaving group) to form a nucleophilic substituted product.

An acid or a base catalyzes this addition–elimination process. The formation of


carboxylic acid derivatives like acid halides, acid anhydrides, esters, and acid
amides involves nucleophilic acyl substitution in the manner discussed through
an addition–elimination mechanism.
Acid catalyzed substitution. The acid catalyzed reaction involves protonation
of carbonyl oxygen which enhances the electrophilic character of carbonyl
carbon and it is therefore attacked readily by nucleophile.
Mechanism
Base catalyzed substitution. In base catalyzed substitution reactions, the base
enhances the nucleophilicity of the attacking reagent thereby facilitating the
addition–elimination process.

Machanism

So far, we have discussed the general mechanism for nucleophilic acyl


substitution. A few examples of such substitution reactions of carboxylic acids
are as follows:
Reaction with thionyl chloride (formation of acid chloride)
Carboxylic acids, on reaction with thionyl chloride result in the formation of
acid chloride. This is one of the best methods for preparation of acid chlorides as
the by products of the reaction (SO2 and HCl) are gases and are easily
eliminated.

Mechanism. The reaction is an acid catalyzed substitution and the nucleophile in


this case is chloride ion.

Formation of acid anhydride


(i) By reaction with acid halides. The carboxylic acids readily attack the
carbonyl carbon of the acid halides to produce acid anhydrides. This is a
convenient method for preparing acid anhydrides in quantitative yield. For
example,

(ii) By dehydration of carboxylic acids. The acid anhydrides are also prepared
by heating carboxylic acid at high temperature in the presence of dehydrating
agents such as phosphorous pentoxide or sodium ammonium hydrogen
phosphate. For example,
Reaction with alcohols: formation of esters
The reaction of carboxylic acid with alcohols in acidic medium results in the
formation of an ester and this is known as Fischer’s esterification. For example

During ester formation, the elimination of water occurs where the removal of –
OH occurs from carboxylic acid and the removal of H occurs from alcohol.

Mechanism. Let us consider the mechanism of esterification by taking the


example of the reaction of ethanoic acid and ethanol in acidic medium.
Step 1. Protonation of carboxylic acid
In the presence of acid catalyst, protonation of carbonyl oxygen occurs which
enhances the electrophilicity of carbon.

Step 2. Attack of alcohol on carbonyl carbon (addition)


Although alcohol is a weak nucleophile, it attacks readily on carbonyl carbon
due to enhanced electrophilicity (refer Step 1).
Step 3. Deprotonation and elimination of leaving group (formation of esters)
The loss of proton occurs from oxonium ion. This proton is captured by the
oxygen of –OH group and hydroxyl group gets converted to a better leaving
group, namely H2O.

Reaction with ammonia and amines


The reaction of carboxylic acids with ammonia forms ammonium salts of
carboxylic acid, which on heating undergoes elimination to produce amide. The
nitrogen in ammonia and its derivatives is a better nucleophile compared to
oxygen in alcohols, so it captures the proton from hydroxy group (of carboxylic
acid) to form ammonium carboxylate salt. Similarly, amines on reaction with
carboxylic acid form N-substituted amides. For example,

Mechanism
Step 1. Addition of ammonia (or amine) to carbonyl carbon. Ammonia (or
amines) act as a nucleophile and attacks the carbonyl carbon.
Step 2. Proton transfer. The loss of proton occurs from nitrogen and this proton
is transferred to oxygen of hydroxyl group.
Step 3. Elimination of water (formation of amide)

Reaction with alkyl lithium: formation of ketone. Reaction of carboxylic


acids with two equivalents of alkyl lithium results in the formation of ketones.
Carboxylic acid on reaction with alkyllithium results in the formation of lithium
carboxylate ion. This reacts further with another equivalent of alkyllithium to
form ketone.

Examples

19.3.4 Reactions Involving –COOH Group


Reduction with lithium aluminium hydride (formation of 1° alcohol)
The reduction of carboxylic acids is carried out in the presence of strong
reducing agents such as lithium aluminium hydride. The lithium aluminium
hydride reduces the carboxylic acid to primary alcohol.

The reaction involves hydride transfer from lithium aluminium hydride


(LiAlH4) and causes reduction at carbonyl part of carboxylic group.
The following reactions involving –COOH group of carboxylic acids have been
discussed in details in previous chapters. For these reactions only section
numbers are indicated for reference.
• Reaction with soda lime: formation of alkane (Section 5A.2.4)
• Hunsdiecker reaction: formation of haloalkane (p. 396)
• Kolbe’s electrolytic reaction: Formation of alkane (Section 5A.2.4)
• Heating calcium salts of carboxylic acids: formation of aldehyde and
ketone (p. 576)
• Reduction with red phosphorous and hydrogen iodide: formation of
alkane
(Section 5A.2.5)

19.3.5 Reactions Involving α-Carbon of Carboxylic


acids
α-Halogenation: Hell–Volhard–Zelinsky reaction (HVZ reaction)
The reaction of carboxylic acid (having α-hydrogen) with bromine or chlorine in
the presence of red phosphorous results in the bromination or chlorination at α-
carbon to yield 2-bromocarboxylic acids or 2-chlorocarboxylic acids.
If two α-hydrogens are present, both of them may be substituted by halogens (Cl
or Br).
Mechanism. The mechanism has been discussed by taking the example of α-
bromination of propanoic acid.
Step 1. Formation of catalyst
The PBr3 is generated in-situ and it catalyses the reaction.

Step 2. Reaction of PBr3 with carboxylic acid: (formation of acid bromide).


Step 3. Acid catalyzed enolization followed by bromination at α-position

Step 4. Exchange reaction (formation of 2-bromopropanoic acid)


The 2-bromopropanoyl bromide then reacts with unreacted propanoic acid. An
exchange reaction between the two results in the formation of 2-bromopropanoic
acid and propanoyl bromide. The acid bromide undergoes step 3 again and
continues the reaction.

19.3.6 Some Typical Chemical Properties of Formic


Acid
The carboxylic group in the formic acid is attached to hydrogen instead of the
alkyl group. It reacts with sodium hydroxide similar to other carboxylic acids
and forms a sodium salt, namely sodium formate. However due to absence of α-
carbon, formic acid shows some typical reactions not given by other carboxylic
acids. These reactions are discussed in this section.
(a) Formic acid forms sodium formate with sodium hydroxide, which on
heating results in the formation of sodium oxalate, a salt of dicarboxylic
acid.

(b) Formic acid, on mild heating with sulfuric acid forms carbon monoxide
and water.
While at a higher temperature (160°C), it results in the formation of carbon
dioxide and hydrogen.

(c) Oxidation: Formic acid due to its structure, gets oxidized with even mild
oxidizing agents and acts as a strong reducing agent.
Formic acid reduces Tollens reagent, Fehling’s solution, and
potassium permanganate. Thus, it is the only monocarboxylic acid,
which gives positive silver mirror test (Tollens test) and a reddish
brown precipitate with Fehling’s solution. Therefore, these reactions
are used for distinguishing formic acid from other carboxylic acids.
with Tollens reagent

with Fehling’s solution


19.4 ACID DERIVATIVES
In the nucleophilic acyl substitution reactions of carboxylic acids, we have seen
how the –OH is replaced by halogen, alcohol, and ammonia to give acid halides,
esters, and amides respectively. Further, the two molecules of carboxylic acid on
intermolecular dehydration result in the formation of carboxylic anhydrides.
The acid halides, anhydrides, amides, and esters are characterized by the
presence of the following groups:

These groups are collectively known as acid derivatives. All these derivatives on
hydrolysis result in the formation of carboxylic acid. In view of this, the nitriles
(–C≡N) are also sometimes considered as derivatives of carboxylic acids.
Nitriles and isonitrites are discussed separately in Chapter 22.
Table 19.2 enlists the IUPAC names, general formulae and common names (in
parentheses) of some carboxylic acids and their corresponding derivatives.
19.4.1 Acid Halides
Acid chlorides are the most common and widely studied acid halides, being
convenient to prepare as compared to acid bromides and acid iodides. This
section therefore focusses on the reactions and properties of acid chlorides only.
Acid chlorides are liquid and have lower boiling point compared to
corresponding carboxylic acids. This decrease in the boiling point is due to the
replacement of –OH of carboxylic acids by –Cl in acid chlorides causing the loss
of hydrogen bonding.
The acid chlorides react rapidly with water (hydrolyzed) to
produce carboxylic acid and hydrochloric acid. This rapid reaction of
acid
chlorides with water makes them Lachrymatory, that is, producing
tears in the
eyes as these react with moisture in eyes to form hydrochloric acid,
which acts as an irritant.

The nucleophilic substitution (acyl substitution) in acid chlorides is


similar to carboxylic acids as discussed earlier (Section 19.3.3).
Acid chlorides are the most reactive of acid derivatives.
The high reactivity of acid chlorides is attributed to the highly
electronegative chlorine atom. Chlorine increases the electrophilic
character
of carbonyl carbon, which is thus attacked readily by nucleophiles.
Also
chloride ion is a good leaving group, which further enhances the rate
of nucleophilic reaction.
The chemical reactions of acid chlorides are used for the synthesis of a number
of functional groups. This has been summarized in the reactions as follows:
19.4.2 Acid Anhydrides
Acid anhydrides are formed by intermolecular dehydration that occurs between
two carboxylic
acid molecules. Acid anhydrides have high dipole moment. The boiling point of
acid
anhydrides is slightly higher than parent acids due to high molecular mass of
anhydrides. The acid anhydrides are less reactive compared to acid chlorides. In
general, the anhydrides are used as acylating agents for hydroxy and amino
groups. The reactions of acid anhydrides are summarized as follows:

19.4.3 Ketene—An Intramolecular Anhydride of


Carboxylic Acid
Ketene having the molecular formula C2H2O, is an intramolecular anhydride of
carboxylic acid. Ketenes have cumulated double bonds, that is, two double
bonds are present on the same carbon. Ketene is highly poisonous and unstable.
It dimerizes readily while diphenylketene (a substituted ketene) is stable at room
temperature.

Preparation
(i) Ketene is prepared by the dehydration of carboxylic acid at a high
temperature in the presence of triethylphosphate.
(ii) Ketene is also prepared by the pyrolysis of acetone at a high temperature
in the presence of catalysts.

(iii) The substituted ketene, say, diphenylketene can easily be prepared by the
reaction of substituted acid chloride with organic base
(dehydrohalogenation).

Chemical properties. Ketene is used for the preparations of number of organic


compounds like acids, esters, thioesters, amides, anhydrides, and a number of
cyclic compounds. The reactions are summarized as follows:
19.4.4 Esters
Esters are formed by the reaction of acids or acid derivatives such as acid
chloride and acid anhydride with alcohol. Esters are pleasant smelling volatile
liquids. Quite a number of esters have fruity odours like that of apples, bananas,
pineapples, apricots, oranges, and so on. These are the most widely distributed
organic compounds in nature. Oils, waxes, and fats are the examples of natural
esters. Many of the esters are used as solvents. The following table (Table 19.3)
tabulates a few esters with the corresponding odours.
The esters have low boiling points due to the absence of hydrogen bonding.
They are insoluble in water due to the hydrophobic alkyl groups. These are less
reactive compared to acid chlorides and acid anhydrides. The reactions of esters
are summarized here:
19.4.5 Amides
Amides are formed when carboxylic acids react with ammonia or its derivatives.

Except methanamide, other unsubstituted amides are all solids. These are
colourless, crystalline compounds and have higher melting points in comparison
to parent carboxylic acids. The high melting point of amides is attributed to the
fact that amide molecules are highly associated through intermolecular hydrogen
bonding.

In case of N-substituted amides, the boiling and melting points decrease as the
sites for hydrogen bonding diminish with increasing number of alkyl
substituents. Amides are less basic compared to aliphatic amines. They exhibit
resonance where the lone pair on nitrogen participates in delocalization with
oxygen of carbonyl group. This reduces the availability of electrons on nitrogen
thereby decreasing the basicity.

The introduction of double bond character between carbon and nitrogen reduces
the C–N bond length to less than the usual C–N single bond. Amides due to
resonance, show a planar geometry with restricted rotation around the C–N
bond. In nature, amide linkages are found in proteins.
Amides are least reactive of all acid derivatives because of delocalization which
reduces the electrophilicity of carbon (of CONH2). The electron releasing –OR
group in esters decreases the electrophilicity of carbon (of –COOR). In acid
chlorides the highly electronegative chlorine increases the electrophilicity of
carbon (of COCl) thereby making them the most reactive.
Based on electronic factors, the order of reactivity of acid derivatives towards
nucleophilic reaction is as follows:

The reactions of amides are generally slow and require prolonged heating or
catalysts (acid or base) for them to take place. A few reactions of amides are
19.4.6 Urea—A Derivative of Carbonic Acid
Urea is a diamide of carbonic acid.

Urea is a stronger base than amides. In urea, the two –NH2 groups are present
that form the amide linkage. In acid amides, the lone pair on nitrogen
participates in delocalization with carbonyl group. However in urea, at a time
only one of the lone pair on nitrogen participates in delocalization while the lone
pair on other nitrogen is free to interact with other species. Thus, the basicity of
urea is high as compared to acid amides.

Commercially, Haber’s process is used to prepare urea, which involves the


reaction of carbon dioxide and ammonia under high temperature and pressure
conditions.

Urea is widely used as a fertilizer and is also used in the manufacture of polymer
resin, for example, urea formaldehyde.
The reactions of urea can be classified in two categories, namely general
reactions and those leading to the formation of heterocyclic compounds.
(a) General reactions

(b) Reactions of urea leading to the formation of heterocyclic compounds


Biuret test. Urea on heating above its melting point, solidifies to biuret with the
evolution of ammonia. Biuret on reaction with copper sulfate in alkaline medium
forms a violet complex

NOTABILIA 13
SELECTED SOLVED EXAMPLES
Example 1. Write the chemical reactions and products formed, when
calciumbutanoate is
(i) heated alone
(ii) heated after the addition of conc. sulfuric acid
(iii) heated along with calcium formate.
Solution.
(i) Heating calcium salt of carboxylic acid results in the formation of ketone in
the following manner.

(ii) Addition of sulfuric acid followed by heating results in the formation of


free carboxylic acid.

(iii) Heating with calcium formate results in the formation of aldehyde.

Example 2. Arrange the following acids in increasing order of their acidic


strength.
Solution. In carboxylic acids the acidic strength is directly related to the stability
of corresponding carboxylate ion. The groups with +I effect destabilize the
carboxylate ion whereas presence of groups with –I effect stabilizes the
carboxylate ion.
(a) II < I < III
In ethanoic acid (II), the +I effect of CH3 group destabilizes the ethanoate ion
and makes II the least acidic in nature. The –I effect of chlorine makes
chloroethanoic acid (III) the most acidic in nature in the given series.
(b) I < III < II
The +I effect of alkyl group makes propanoic acid least acidic. The –I effect
of –CN is much stronger than iodine which makes 2-cyanoethanoic acid (II)
most acidic in the series.
(c) III < II < I
In case of chlorosubstituted carboxylic acids the –I effect of chlorine is
observed maximum when it is nearest to carboxylic group. Thus, 2-
chlorobutanoic acid (I) is most acidic and 4-chlorobutanoic acid (III) is least
acidic in the series.
(d) III < I < II
The +I effect of methyl group makes 2-Methylpropanoic acid (III) least
acidic. The
nitro group is strong electron withdrawing group compared to chlorine
which makes
2-Nitropropanoic acid most acidic in the series.
Example 3. Carry out the following conversions:
(i) Propanoic acid to Propanal
(ii) Propanoic acid to Butanoic acid
(iii) Propanoic acid to 2-Oxopropanoic acid
(iv) Butanoic acid to 2-Aminobutanoic acid
(v) Acetic acid to Hexan-2-one
Solution
Example 4. What product will be obtained on treatment of pentanediamide with
bromine and sodium hydroxide.
Solution. The amides are degraded to amines by bromine in alkaline medium.
(refer Hofmann degredation reaction in Section 19.4.5).

Example 5. Arrange the following acid halides in their increasing order of


reactivity with ethyl magnesium bromide (Grignard reagent).

Solution. The reactivity of acid halides with Grignard reagent is depenendent on


steric factors. Bromine with larger size will have more steric hindrance
compared to fluorine with smallest size (least steric hindrance)
The order of reactivity of acid halides is:
Example 6 Carry out the following conversions:
(i) Ethanoic acid to 3-Aminopropanoic acid
(ii) Allyl chloride to 3,4-Dichlorobutanoic acid
(iii) Ethyl 2-bromo-2-methylpropanoate to Ethyl 2,2-dimethyl-3-
hydroxypentanoate
(iv) Ethanoyl chloride to Hexan-2-one
(v) Propan-1-ol to Butanamide
Solution
EXERCISES
1. Give the IUPAC names for the following compounds:

2. Why boiling points of carboxylic acids are much higher than alcohols of
comparable mass?
3. Why do
(a) Acid chlorides have lower boiling points compared to the corresponding
carboxylic acids?
(b) Acid amides have high melting points compared to parent carboxylic
acids?
(c) Acid anhydrides have high boiling point compared to parent carboxylic
acids?
4. Give Wacker’s process for synthesis of acetic acid.
5. Name the carboxylic acids formed by the hydrolysis of:
(a) 1,1,1-Trichlorobutane in alkaline medium
(b) Acetonitrile in acidic medium
(c) Ethyl propanoate in alkaline medium
6. Complete the following reactions:
7. Give the industrial method of the preparation of formic and acetic acid by the
carbonylation reaction.
8. In carboxylic acids, there are two different C–O bond lengths, however the
carboxylate ion has the same C–O bond lengths, explain.
9. How will you prepare methyl ester of ethanoic acid without using alcohol as a
reagent?
10. Give the mechanism for the alkaline hydrolysis of an ester.
11. Give the mechanism for the acid catalyzed nucleophilic acyl substitution in
carboxylic acids.
12. Why do acid amides show less basicity compared to amines?
13. Why does the presence of base catalyze the nucleophilic acyl substitution in
carboxylic acids?
14. Giving example, explain the synthesis of mixed anhydrides.
15. Give the mechanism for acid catalyzed esterification process.
16. Explain with mechanism Hell–Volhard–Zelinsky reaction in carboxylic
acids.
17. Why are acid chlorides more reactive towards nucleophilic substitution
reactions compared to alkyl chloride?
18. Why does formic acid give a positive Tollens test (Silver mirror test)?
19. How do you explain the lachrymatory nature of acid chlorides?
20. Why is C–N bond length in amide shorter than C–N bond in amines?
21. Complete the following reactions:
22. What is trans-esterification?
23. Giving suitable explanation, arrange the acid derivatives (acid chlorides,
esters, acid anhydrides, and amides) in increasing order of reactivity towards
nucleophilic substitution reaction.
24. Why is urea a stronger base compared to acid amides?
25. Carry out the following conversions:
(a) Urea to Semicarbazide
(b) Ketene to Methanamine
(c) Urea to Thymine
(d) Ethyl acetate to tert. Butyl alcohol
(e) Propanoic acid to Propanenitrile
(f) Urea to Barbituric acid.
Chapter20
Bifunctional
Carboxylic Acids and
Their Derivatives

INTRODUCTION
The organic compounds in general may contain more than one functional group,
which may not be same. The compounds with two functional groups exhibit the
characteristics of each functional group and at same time undergo some typical
reactions that depict a characteristic influence of one functional group over
another. The section A of present chapter deals with the chemistry of
Dicarboxylic and substituted carboxylic acid. The section B is devoted to
chemistry of reactive methylene compounds. This chapter gives an in-depth
analysis of the bifunctional carboxylic acids and their derivatives as tabulated in
Information Chart I.
A. DICARBOXYLIC ACIDS AND
SUBSTITUTED CARBOXYLIC ACIDS
20A.1 DICARBOXYLIC ACIDS [SATURATED AND
UNSATURATED]
The aliphatic dicarboxylic acids are named in IUPAC as alkanedioic acids. Most
of the dicarboxylic acids are better known by their common names. The
unsaturated dicarboxylic acids exhibit geometrical isomerism and named as E-
or Z-alkenedioic acids.

Physical properties. The dicarboxylic acids exhibit two dissociation constants


(K1 and K2) as each carboxylic acid functional group undergoes ionization. The
K1 values are higher compared to Ka value of monocarboxylic acids. The K1
values of different carboxylic acids decrease with an increase in the number of –
CH2 groups between two carboxylic groups. However, the K2 values of different
dicarboxylic acids do not differ appreciably and are low compared to K1 values.
Table 20.1 enlists the melting point and dissociation constants for same
dicarboxylic acids.
In a dicarboxylic acid, the first dissociation constant, K1, is
higher than the second one, K2.
The higher value of K1 is due to electron withdrawing effect (–I
effect) of one carboxylic group on the other, which stabilizes the
negative charge on carboxylate ion. The electron withdrawing effect
decreases with an increase in the distance between two carboxylic
functional groups. The carboxylate ion (–COO–) formed is rich in
electron density and shows electron releasing effect (+I effect), which
makes the removal of hydrogen from –COOH group difficult. Thus,
the second dissociation constant K2, is much lower than the first one.
K1 (first dissociation constant) of maleic acid (pK1= 1.9) is
greater than K1 of fumaric acid (pK1=3.0). But K2 (second
dissociation constant) of maleic acid (pK1= 6.5) is less than K2 of
fumaric acid (pK2=4.5).
Maleic acid. The high K1 value in maleic acid is attributed to the
intramolecular hydrogen bonding (due to cis- configuration) which
stabilizes the carboxylate ion.
However, this intramolecular hydrogen bonding makes the removal of
second proton difficult and thus it has very low values of K2.
Fumaric acid. Fumaric acid has a low K1 value due to trans-
configuration, due to which no hydrogen bonding is possible resulting
in instability of the carboxylate ion.
But this absence of hydrogen bonding makes the removal of second
proton easy and thus has a high value of K2.
The lower members of the dicarboxylic acid series are soluble in water and
alcohol due to intermolecular association through hydrogen bonding. The
dicarboxylic acids are solid and their melting points increase with an increase in
molecular mass. The dicarboxylic acids, with even number of carbons have
higher melting point than those containing odd number of carbon atoms but they
are less soluble in water compared to dicarboxylic acids containing odd number
of carbon atoms.
In dicarboxylic acids containing even number of carbons, terminal –COOH
groups lie opposite to each other. As a result, they fit better in crystal lattice
structure due to intermolecular association and have high melting point.
However, they don’t form hydrogen bonds that easily with water and are
thereofore, less soluble in water.
On the other hand, in carboxylic acids containing odd number of carbons, the
terminal –COOH groups are on the same side and therefore in crystal lattice
structure, intermolecular association is hindered due to which these compounds
have low melting point. The –COOH group is relatively free to form hydrogen
bonding with water and alcohol and thus having higher solubility as compared to
dicarboxylic acids with even number of carbon atoms.

20A.1.1 Preparations of Saturated Dicarboxylic Acids


(1) Oxalic acid [HOOC–COOH]
[Ethanedioic Acid]
The important methods for preparations include:
(1) Heating sodium formate at 360°C produces sodium oxalate from which
oxalic acid is recovered by treatment with calcium hydroxide followed by
concentrated sulfuric acid.
(2) Heating sodium metal in an atmosphere of carbon dioxide at high
temperature results in the formation of sodium oxalate from which oxalic
acid is recovered as mentioned in the previous method.

(2) Malonic acid [HOOCCH2COOH]


[Propanedioic acid]
Malonic acid is synthesized from calcium carbide through a series of reactions as
follows:

(3) Succinic acid [HOOCCH2CH2COOH]


[Butanedioic acid]
(1) Succinic acid is synthesized from calcium carbide as follows:

(2) Alternatively, reduction of maleic acid in the presence of metal catalyst


results in the formation of succinic acid.
20A.1.2 Preparation of Unsaturated Dicarboxylic
Acids
(1) Maleic acid
[(Z)-But-2-enedioic acid]
(1) Maleic acid may be prepared from α-hydroxysuccinic acid (malic acid) in the
following manner.

(2) Fumaric acid


[(E)-But-2-enedioic acid]
(1) The fumaric acid may be prepared from α-bromosuccinic acid or maleic acid
as follows:

(2) An alternate method involves Knoevenagel reaction, in which malonic acid


reacts with glyoxalic acid to form fumaric acid.
20A.1.3 Chemical Properties
Dicarboxylic acids have similar chemical properties as monocarboxylic acids. A
dicarboxylic acid react with two moles of reagent to give corresponding
disubstituted derivatives summarized as follows:

The most important reaction is the action of heat where different products may
be obtained depending upon the structure of dicarboxylic acid.
Action of heat on dicarboxylic acids
The dicarboxylic acids on heating may undergo one of the two important
reactions, namely
(i) decarboxylation and (ii) dehydration.
(i) Decarboxylation. Oxalic acid (ethanedioic acid), malonic acid, and other
dicarboxylic acids where two carboxylic groups are present on same carbon, on
heating, undergo removal of carbon dioxide to produce monocarboxylic acids.
The mechanism of decarboxylation is as follows:

Heating a dicarboxylic acid in the presence of concentrated sulfuric acid results


in the formation of oxides of carbon and water. Heating malonic acid in the
presence of P2O5 results in the formation of carbon suboxide.

(ii) Dehydration. Dicarboxylic acids where two carboxylic groups are separated
by two or three carbon atoms, on heating, undergo elimination of water molecule
to form acid anhydrides. For example,
(iii) Simultaneous decarboxylation and dehydration. Dicarboxylic acids
where two carboxylic groups are separated by four or five carbon atoms undergo
simultaneous removal of carbon dioxide and water to form cyclic ketones. This
is depicted in the following reactions:
Oxidation reactions
The oxidation of oxalic acid with KMnO4 or K2Cr2O7 results in the formation
of carbon dioxide and water.

Maleic and fumaric acid on reaction with alkaline KMnO4 or OsO4 forms
tartaric acid. Maleic acid gives the meso tartaric acid while fumaric acid gives a
racemic mixture of tartaric acid.

Reaction with dihydric alcohols


The dicarboxylic acids usually react with diols such as ethylene glycol to form
polyesters. However, the reaction of oxalic acid with ethylene glycol results in
the formation of a cyclic compound called ethylene oxalate.
Further, oxalic acid reacts with glycerol to form either formic acid or allyl
alcohol (p. 523). One of the important and characteristic reaction of maleic acid
is its reaction with conjugated dienes, known as Diels–Alder reaction (Section
7.4.3).
20A.2 SUBSTITUTED CARBOXYLIC ACIDS

20A.2.1 Hydroxy Acids


The carboxylic acids having a hydroxy group(s) in the molecule are termed as
hydroxy acids or hydroxycarboxylic acids. The molecules in these acids may
contain one or more hydroxy group and also one or more carboxylic group. A
few examples of hydroxy acids are as follows:

Physical properties. Hydroxy acids are soluble in water but insoluble in


alcohol. The hydroxy monocarboxylic acids have high acidic strength compared
to unsubstituted monocarboxylic acids since the –I effect of hydroxy groups
stabilizes the carboxylate ion. The –I effect of hydroxy group, in hydroxy acids,
decreases with the increasing distance from carboxylic group. Thus, the
increasing order of acidic strength of hydroxy acids is as follows:

20A.2.2 Preparations of Hydroxy Acids


(1) Lactic acid [CH3CH(OH)COOH]
[2-Hydroxypropanoic acid]
Lactic acid is obtained from molasses by fermentation. Its synthesis may be
carried out from calcium carbide as follows:

(2) Malic acid [HOOCCH2CH(OH)COOH]


[2-Hydroxybutanedioic acid]
Malic acid is isolated from juice of berries. Its syntheses may be carried out
either from maleic acid by hydrolysis under pressure or from tartaric acid by
reduction, as follows:

(3) Tartaric acid [HOOCCH(OH)CH(OH)COOH]


[2,3-Dihydroxybutanedioic acid]
Tartaric acid is manufactured by fermentation of grapes. Its synthesis may be
carried out from ethyne (acetylene) in the following manner:

An alternative method involves its synthesis from ethene as follows:


Stereochemistry in tartaric acid: Tartaric acid has two stereogenic centres.
However, it exists in three stereoisomeric forms. Tartaric acid exhibits a plane of
symmetry and occurs as a meso form. Details about stereoisomers of tartaric acid
and their optical activity have already been discussed in Chapter 3.
(4) Citric acid [HOOCCH2C(OH)(COOH)CH2COOH]
[2-Hydroxypropane-1,2,3-tricarboxylic acid]
Citric acid is extracted from lemon juice. It is also manufactured by fermentation
of molasses. One of the important methods for citric acid synthesis involves
glycerol as the starting material.

20A.2.3 Chemical Properties


Hydroxy acids undergo reactions, which are characteristics of the –OH group, as
well as reactions that are characteristic of –COOH group. This section briefly
describes these reactions.
Reactions involving –OH group
The reactions occurring at hydroxy group of hydroxycarboxylic acids are similar
to the reactions of alcohols and can be summarized as follows:
Reactions involving –COOH group
The reactions occurring at the carboxylic group of hydroxy acids are similar to
reactions of carboxylic acids and include formation of carboxylate salts, esters,
amides, and anhydrides as summarized below:
Reactions involving both –OH and –COOH groups
These reactions occur at hydroxy and carboxylic group simultaneously in the
following
manner.

Action of heat on hydroxycarboxylic acids


Heating hydroxycarboxylic acids results in the formation of different products
depending upon the position of hydroxy group relative to carboxylic functional
group.

The hydroxy acids on heating may undergo following reactions, namely (i)
intermolecular esterification, (ii) intramolecular esterification, and (iii)
dehydration.
α-Hydroxycarboxylic acids. Two molecules of an α-hydroxy acid, on heating,
undergo intermolecular esterification and result in the formation of cyclic
diesters known as lactides.

β-Hydroxycarboxylic acids. On heating, β-hydroxycarboxylic acids result in


the elimination of water molecule (dehydration) to form α,β-unsaturated
carboxylic acid.

γ-Hydroxycarboxylic acids. The γ-hydroxycarboxylic acids on heating undergo


intramolecular esterification and result in the formation of five-membered cyclic
esters known as
lactones.
δ-Hydroxycarboxylic acids. The δ-hydroxycarboxylic acids on heating undergo
intramolecular esterification to form six-membered cyclic esters known as
lactones.

ε- and higher hydroxycarboxylic acids. The hydroxy carboxylic acids where –


OH and –COOH groups are separated by five or more carbon units, on heating,
undergo linear intermolecular esterification and result in the formation of
polyesters.

Action of heat on citric acid


In citric acid, action of heat may result in the formation of two products,
explained as
follows:
(i) Heating may result in the elimination of water with hydroxyl group at β-
position resulting in the unsaturated acid, which on further decarboxylation
results in the formation of cyclic anhydride as an end product.
(ii) The heating may also result in the decarboxylation to yield acetone as an
end product.

Oxidation of hydroxycarboxylic acids


The oxidation of hydroxy acids may be carried out with oxidizing agents such as
nitric acid, potassium permanganate, Fentons reagent, and Tollens reagent, and
the reactions are summarized as follows:
Reduction of hydroxycarboxylic acids with HI
Reduction of hydroxy acid with hydrogen iodide results in the removal of
hydroxy group and thus resulting products contain only carboxylic groups.
Action of sulfuric acid on hydroxycarboxylic acids
Following reaction products are obtained by the reaction of a hydroxy acid with
sulfuric acid.

20A.2.4 Halocarboxylic Acids


The carboxylic acids containing halogens are termed as halocarboxylic acids.
The acids may be classified as α, β, γ halo acids, and so on depending upon the
position of halogens. The presence of halogen (–I effect) increases the acidic
strength of α-halocaboxylic acids. Whereas –I effect decreases as the distance of
halogen from carboxylic acid increases thereby decreasing the acidic strength.
Thus, the acidic strength of halocarboxylic acids is as follows:

20A.2.5 Preparations of Halocarboxylic Acids


Monocarboxylic acids, on reaction with chlorine or bromine in presence of red
phosphorous form α-chloro or α-bromocarboxylic acids. The reaction is known
as Hell–Volhard–Zelinsky (HVZ) reaction (Section 19.3.5).

The β- and γ-halocarboxylic acids are synthesized by reaction of halogen acids


with α, β-unsaturated aldehydes and the β, γ-unsaturated carboxylic acids
respectively.

20A.2.6 Chemical Properties


The halocarboxylic acids exhibit dual properties of haloalkanes and of
carboxylic acids. The halogen atom in α-halocarboxylic acids is more reactive
compared to that in haloalkane due to –I effect of carboxylic group. The
reactions of halo acids can be summarized as follows, by considering the
reactions of α-chlorocarboxylic acid as an example.

B. REACTIVE METHYLENE
COMPUNDS
[Chemistry of Enolate ions]
20B.1 INTRODUCTION
The hydrogens in methane practically do not exhibit acidic character. However
when two of the hydrogens are replaced by electron withdrawing groups, the rest
of the hydrogens become acidic in nature. The electron withdrawing groups
present on both sides attract the electrons towards themselves and thus, weaken
the C–H bond of methylene ( CH2) group. It results in the easy release of
hydrogen as a proton (H+). Thus a methylene group, which is sandwiched
between two electron withdrawing groups, exhibits acidic character and is
known as reactive methylene group.

The strong electron withdrawing groups may be aceto (CH3CO–), carbethoxy (–


COOC2H5) or cyano (–CN). A few examples of compounds having reactive
methylene groups are listed here.

Keto-enol tautomerism in reactive methylene compounds. The reactive


methylene compounds exhibit keto-enol tatuomerism where hydrogen of
methylene group migrates to the oxygen of carbonyl group. The keto and enol
forms of acetyl acetone, ethyl acetoacetate, and diethyl malonate are:
The enol form of these compounds is chelated through intramolecular hydrogen
bonding. The content of keto and enol forms depends on the stability of these
forms as well as the nature of the solvents. The enol form in acetyl acetone is
stabilized through resonance, however, the stabilization of enol form of ethyl
acetoacetate is relatively less. The resonance stabilization of enol form of diethyl
malonate is almost negligible.

It is to be noted that in ethyl acetoacetate the ester group does not participate in
enolization as it results in the decrease of the resonance energy of the system. On
these grounds, it is quite clear that the percentage of enol form will be lower in
case of ethyl acetoacetate and diethyl malonate compared to acetyl acetone. The
order of the percentage of enol form in reactive methylene compounds is as
follows:

Further the polar solvents increase the keto content whereas the non-polar
solvents increase the enol content.
Acidic character of reactive methylene compounds. In reactive methylene
compounds, the presence of electron withdrawing groups facilitates the removal
of proton from methylene group. In presence of a base, the loss of proton from a
reactive methylene compound results in the formation of a carbanion. The
resonance stabilization of this carbanion accounts for the high acidic strength of
reactive methylene compounds.

Let us take the example of ethyl acetoacetate as a reactive methylene compound.


With a base like sodium ethoxide, its hydrogen gets replaced as a proton (H+)
and forms a carbanion, which is stabilized through resonance. The resonance
stabilization occurs through enolization of carbanion and thus, its salts are
referred to as enolate ions.

The chemistry of enolate ions is of immense importance as these can be used for
the synthesis of a number of organic functional groups. Here we will be
discussing the chemistry of two important reactive methylene compounds,
namely ethyl acetoacetate and diethyl malonate.
20B.2 ETHYL ACETOACETATE
(IUPAC name: Ethyl 3-oxobutanoate); [CH3COCH2COOC2H5]
Ethyl acetoacetate (commonly referred as acetoacetic ester) is a colourless liquid
(b.p. 181°C) with pleasant smell and is sparingly soluble in water. Although it is
neutral to litmus, it is soluble in sodium hydroxide solution. It partially
decomposes on strong heating and is therefore distilled much below its boiling
point under reduced pressure.

20B.2.1 Preparation
Two molecules of α-hydrogen containing esters condense in presence of a base
to form β-keto esters and this reaction is known as Claisen condensation. The
Claisen condensation of ethyl acetate in the presence of sodium ethoxide results
in the formation of ethyl acetoacetate (acetoacetic ester).

Mechanism. Claisen condensation is a nucleophilic substitution reaction.


Sodium ethoxide acts as a catalyst and provides conditions necessary for the
reaction. The reaction follows the following steps:
Step 1. Abstraction of α-hydrogen of ester by base (formation of carbanion)

Step 2. Attack of carbanion on carbonyl carbon of second ester molecule

Step 3. Removal of ethoxide ion (formation of ethyl acetoacetate)


20B.2.2 Chemical Properties
Ethyl acetoacetate exhibits keto-enol tautomerism and behaves as a ketone as
well as an alcohol in the chemical reactions it undergoes.
Reactions due to keto form in ethyl acetoacetate
The ethyl acetoacetate undergoes nucleophilic addition reactions similar to
ketones. Some of the typical reactions due to the presenc of ketonic group are:
(i) Reaction with hydrogen cyanide results in the formation of cyanohydrin.
(ii) Reaction with sodium bisulfite results in the formation of a crystalline
addition compound.
(iii) Hydroxylamine reacts with ethyl acetoacetate to result in the formation of
an oxime, which immediately loses a molecule of alcohol and forms a
heterocyclic compound called methyl isoxazolone.
(iv) Reduction of ethyl acetoacetate with sodium amalgam and alcohol or
hydrogen in presence of nickel, results in the formation of ethyl 3-
hydroxybutanoate, that is, C=O group is reduced to –CHOH group.
However, with lithium aluminium hydride it forms butane-1,3-diol as both
keto and ester groups get reduced.
The following sequence summarizes these reactions that acknowledge the
presence of ketonic group in ethyl acetoacetate:
Reactions due to enolic form in ethyl acetoacetate
The enolic form of ethyl acetoacetate contains an olefinic bond as well as
alcohol functional group. Thus, it undergoes reactions that are characteristic of
olefinic bond as well as of alcoholic group. Following are the reaction that ethyl
acetoacetate undergoes due to the enolic group present:
(i) Reaction with sodium ethoxide results in the formation of sodium salt.
(ii) Acetylation of ethyl acetoacete with acetyl chloride results in the
formation of an acetyl derivative.
(iii) The addition of bromine takes place due to the presence of olefinic
linkage (C=C) in the enolic form and results in the formation of dibromo
addition product.
(iv) Reaction with diazomethane forms methyl ether of ethyl crotonate.
(v) Grignard reagent results in the formation of its corresponding hydrocarbon
as the –OH group in enol form acts as a source of active hydrogen.
(vi) Ethyl acetoacetate reacts with ammonia as well as primary and secondary
amines to form β-amino crotonic esters.
(vii) Ethyl acetoacetate gives a violet colour with ferric chloride due to the
presence of the enolic group –C=C–OH (similar to phenol).
All these reaction are summarized here as follows:

Reactions of synthetic importance


The reactions of ethyl acetoacetate which are widely employed for the synthesis
of other functional group derivatives include:
(i) Reaction of ethyl acetoacetate with base: Formation of enolate. In the
presence of a base like sodium ethoxide, ethyl acetoacetate looses a proton to
form corresponding sodium salt. The carbanion thus formed is stabilized through
resonance (as mentioned earlier).
(ii) Reaction with haloalkanes: Formation of alkyl derivatives of ethyl
acetoacetate (Alkylation)
The sodium salt of ethyl acetoacetate reacts with 1° and 2° haloakanes to form
corresponding alkyl derivatives. The reaction can be used to prepare both
monoalkyl as well as dialkyl derivatives of ethyl acetoacetate.

Both the hydrogens of methylene group cannot be replaced in a single step


simultaneously by the base. Thus, to prepare dialkyl derivative, alkyl groups are
introduced one at a time. In case two different alkyl groups are to be introduced,
the larger group is introduced first due to steric reasons.
(iii) Hydrolysis of ethyl acetoacetate. In presence of potassium hydroxide, the
hydrolysis of ethyl acetoacetate may occur in two ways to give either ketone or
carboxylic acid as the final product. Based on the product obtained, the
hydrolysis can be categorized as ketonic hydrolysis and acid hydrolysis.
Ketonic hydrolysis. Heating ethyl acetoacetate with dilute solution of aqueous
or ethanolic potassium hydroxide results in the formation of a β-keto acid. Acid
on heating undergoes decarboxylation to yield ketone as the final product.

Similarly, ketonic hydrolysis of alkyl derivative of ethyl acetoacetate results in


the formation of ketones. It should be noted that the ketone so formed is always a
methyl ketone (CH3CO–).
It is to be noted that in various synthetic strategies from ethyl acetoacetate,
involving ketonic hydrolysis, the ethyl acetoacetate contributes three carbon unit
(shown as shaded carbons) to the product.
Acid hydrolysis. Heating ethyl acetoacetate with concentrated solution of
ethanolic potassium hydroxide results in the formation of potassium salt of
acetic acid. The acidification gives free acetic acid as the final product. It is to be
noted that in various synthetic strategies from ethyl acetoacetate, involving acid
hydrolysis, the ethyl acetoacetate contributes two carbon unit (shown as shaded
carbons) to the product.

Similarly, alkyl substituted ethyl acetoacetate on acid hydrolysis results in the


formation of acetic acid and substituted acid derivatives.
Applications. Ethyl acetoacetate and its alkyl derivatives react with a number of
other reagents to yield different functional group derivatives. Some examples of
their use in the synthesis of functional group derivatives are mentioned here:
(a) Synthesis of monocarboxylic acids. Ethyl acetoacetate on acid hydrolysis
forms acetic
acid. The monoalkyl and dialkyl derivatives of ethyl acetoacetate on acid
hydrolysis yield corresponding higher monocarboxylic and substituted
monocarboxylic acids respectively. For example,
Preparation of butanoic acid and 2-methylbutanoic acid:

In a similar way, other monocarboxylic acids and substituted derivatives can be


prepared.
(b) Synthesis of dicaboxylic acids
(i) Succinic acid and its derivatives. The monosodium salt of ethyl acetoacetate
on reaction with ethyl chloroacetate followed by acid hydrolysis results in the
formation of succinic acid.

Alternatively, if two molecules of sodium salt of ethyl acetoacetate react with


iodine and the reaction is followed by acid hydrolysis, succinic acid is formed.

Similarly, the reaction of substituted alkyl derivatives of ethyl acetoacetate with


iodine or with α-halogenated substituted esters forms substituted succinic acid.
For example,

(ii) Higher dicarboxylic acids. Higher dicarboxylic acids such as glutaric acid,
adipic acid, pimelic acid, and the like, are prepared by reaction of two molecules
of the sodium salt of ethyl acetoacetate with dihaloalkanes (having halogen at
the terminal carbons) followed by acid hydrolysis. This is shown in the
following reaction:
(c) Synthesis of keto acids. The reaction of sodium salt of ethyl acetoacetate
with α-haloesters followed by ketonic hydrolysis results in the formation of keto
acids as the final product.

(d) Synthesis of α,β-unsaturated acids. The reaction of ethyl acetoacetate with


an aldehyde followed by acid hydrolysis yields α,β-unsaturated carboxylic acids.

(e) Synthesis of ketones. Mono- and dialkyl derivatives of ethyl acetoacetate on


ketonic hydrolysis result in the formation of higher methyl ketones.
(f) Synthesis of acetylacetone (diketones). The reaction of sodium salt of ethyl
acetoacetate with acid chlorides followed by ketonic hydrolysis results in the
formation of 1,3-diketones. For example, reaction with acetyl chloride results in
the formation of acetyl acetone (pentane-2,4-dione).

The 1,4-diketones can be prepared by reaction of two moles of sodium salt of


ethyl acetoacetae with iodine followed by ketonic hydrolysis. For example,
Hexane-2,5-dione can be prepared as follows:

(g) Synthesis of alicyclic compounds. The reaction of sodium salt of ethyl


acetoacetate with terminal dihaloalkanes followed by hydrolysis results in the
formation of cycloalkane derivatives. For example,
(h) Synthesis of heterocyclic compounds. Ethyl acetoacetate is widely used for
the synthesis of number of heterocyclic compounds. For example,
(i) It reacts with urea to form 4-methyluracil (Thymine).

(ii) It reacts with hydroxylamine to form an oxime, which eliminates a


molecule of ethanol to form isoxazolone.

(iii) It reacts with phenyl hydrazine to form phenyl hydrazone derivative,


which on elimination of a molecule of ethanol forms pyrazolone
derivatives. The methylation of pyrazolone derivative results in the
formation of antipyrine.
20B.3 DIETHYL MALONATE
(IUPAC name: Diethyl propanedioate); [H2C(COOC2H5)2]
Diethyl Malonate is a pleasant smelling, colourless liquid (b.p. 199°C). It is
sparingly soluble in water but soluble in organic solvents such as alcohol,
benzene, and chloroform.

20B.3.1 Preparation
Diethyl malonate (malonic ester) is prepared by heating a solution of potassium
cyanoacetate in ethanol with concentrated hydrochloric acid.

20B.3.2 Chemical Properties


Diethyl malonate exhibits keto-enol tautomerism. The methylene group is
sandwiched between two electron withdrawing ester groups and shows high
acidic character. In the presence of a base like sodium ethoxide, it results in the
formation of sodium salt of diethyl malonate, which can further react with
haloalkanes to form alkyl substituted diethyl malonate. Unlike ethyl
acetoacetate, the dialkyl derivatives can be prepared in a single step by reaction
of diethyl malonate with two equivalents of sodium ethoxide followed by two
equivalents of haloalkane.
Alternatively,

Reactions of synthetic importance


Diethyl malonate is also used for the synthesis of some organic compounds,
discussed as follows:
(a) Synthesis of monocarboxylic acids. Hydrolysis of diethyl malonate by
heating with potassium hydroxide forms potassium salt of malonic acid, which
on acidification followed by heating undergoes decarboxylation to form acetic
acid.

In a similar manner, monoalkyl and dialkyl derivatives of diethyl malonate on


hydrolysis followed by decarboxylation result in the formation of higher
monocarboxylic and substituted monocarboxylic acids. For example,

(b) Synthesis of dicarboxylic acids


(i) Succinic acid and its derivatives. The reaction of sodium salt of malonic ester
with ethyl chloroacetate followed by hydrolysis and decarboxylation results in
the formation of succinic
acid.
Alternatively, the reaction of two molecules of sodium salt of malonic ester with
iodine, followed by hydrolysis and decarboxylation results in the formation of
succinic acid.

In a similar manner, the sodium salt of monoalkyl malonic ester forms


dialkylsuccinic acid derivatives when treated with iodine and then hydrolyzed.
For example,

(ii) Higher dicarboxylic acids. Higher dicarboxylic acids such as glutaric acid,
adipic acid,
pimelic acid, and so on are prepared by reaction of two molecules of the sodium
salt of malonic ester with dihaloalkanes (having halogen at the terminal carbons)
followed by hydrolysis and decarboxylation.
(c) Synthesis of keto acids. The reaction of sodium salt of malonic ester with
acid chloride followed by hydrolysis and decarboxylation results in the
formation of keto acids as the final product.

(d) Synthesis of α β-unsaturated acids. The reaction of malonic ester with an


aldehyde followed by hydrolysis and decarboxylation results in the formation of
α, β-unsaturated carboxylic acids. The reaction is known as Knoevenagel
reaction.

(e) Synthesis of alicyclic compounds. The reaction of sodium salt of malonic


ester with a dihaloalkane followed by hydrolysis and then decarboxylation yields
cycloalkane derivatives. For example,

(f) Synthesis of heterocyclic compounds (Barbituric acid and Barbitone).


The reaction of diethyl malonate with urea results in the formation of barbituric
acid (malonyl urea). The diethyl derivative of malonic ester with urea results in
the formation of barbitone ( also known as veronal), which is used as a hypnotic
agent. Luminal is an important derivative of barbituric acid and is prepared in a
similar manner using bromobenzene and bromoethane successively.
EXERCISES
1. Why does first dissociation constant (K1) of dicarboxylic acids have a lower
value than Ka of monocarboxylic acids?
2. Why does the second dissociation constant (K2) have a lower value than first
dissociation constant (K1) in dicarboxylic acids?
3. Predict the increasing trend of K1 values for the following dicarboxylic acids:
HOOC–CH2–COOH, HOOC–CH2CH2CH2–COOH, HOOC–COOH
4. How will you carry out the following syntheses:
(a) Malonic acid from ethyne
(b) Succinic acid from ethyne
(c) Maleic acid from malic acid
(d) Fumaric acid from malonic acid.
5. What is the action of heat on following dicarboxylic acids:
(a) Oxalic acid
(b) Malonic acid
(c) Tartaric acid
(d) Succinic acid
6. Name the products obtained when the following compounds are heated with
sulfuric acid:
(a) Lactic acid (c) Citric acid
(b) Tartaric acid (d) Oxalic acid
7. What happens when
(a) Malonic acid is treated with P2O5.
(b) Oxalic acid reacts with glycerol at 260°C.
(c) Lactic acid is heated.
8. How will you synthesize citric acid from glycerol?
9. Maleic acid has higher K1 value than fumaric acid but fumaric acid has
higher K2 value than maleic acid. Explain.
pK1 for maleic acid = 1.9 pK1 for fumaric acid = 3.0
pK2 for maleic acid = 6.5 pK2 for fumaric acid = 4.5
10. Write the oxidation products obtained in the following reactions:
11. Giving suitable examples, explain the effect of heat on:
(a) α-hydroxy carboxylic acids
(b) β-hydroxy carboxylic acids
(c) γ-hydroxy carboxylic acids
(d) δ-hydroxy carboxylic acids
12. Name the products obtained by heating citric acid.
13. What products will be obtained when the following compounds are reduced
with HI.
(a) Citric acid
(b) Tartaric acid
(c) Lactic acid
(d) Malic acid
14. How will you convert propanoic acid to methylmalonic acid using HVZ
reaction?
15. Complete the following reactions:
16. What do you mean by reactive methylene compounds? Explain with two
examples.
17. Give the preparation of ethyl acetoacetate and discuss the mechanism
involved in it.
18. What is Claisen condensation? Discuss the mechanism in detail.
19. What product will be formed on the ketonic and acidic hydrolysis of ethyl
acetoacetate?
20. How will you synthesize the following from ethyl acetoacetate:
(a) Antipyrine (e) Pentane-2,4-dione
(b) 4-methyluracil (f) Pent-2-enoic acid
(c) Butanoic acid (g) 3-methylpentan-2-one
(d) Propanoic acid
21. Giving suitable reactions, establish that ethyl acetoacetate is a mixture of
keto and enol forms?
22. Give the synthesis of the following from diethyl malonate:
(a) Glycine (d) Succinic acid
(b) Adipic acid (e) Dimethylacetic acid
(c) Crotonic acid
23. Why is enol form of ethyl acetoacetate more stable compared to the enol
form of acetone?
24. How will you convert diethylmalonate to ethyl acetoacetate?
25. Beginning with ethyl acetoacetate or diethyl malonate, how can you prepare
the following cyclic compounds?
(a) Cyclohexanecarboxylic acid
(b) Cyclohexyl methylketone
(c) Barbitone (Veronal)
Chapter21
Aromatic
Carboxylic
and Sulfonic Acids

A. AROMATIC CARBOXYLIC ACIDS


21A.1 INTRODUCTION
The organic compounds where a carboxylic group is attached directly to
aromatic ring are known as aromatic carboxylic acids. Benzoic acid is a
representative of this class. The compounds where carboxylic group is present
on the side chain of aromatic ring system as in phenyl acetic acid and cinnamic
acid show similar behaviour as aliphatic carboxylic acids. In the aromatic
carboxylic acids the carboxylic group is attached to sp2 hybridized carbon of the
aromatic ring system while in case of aliphatic acids the carboxylic group is
directly attached to sp3 hybridized carbon.

In IUPAC nomenclature, these acids are termed benzene carboxylic acid


derivatives. However, the common names of aromatic carboxylic acids are more
familiar and are also accepted by IUPAC. A few selected examples of the family
are as follows:
Physical properties. The aromatic carboxylic acids exhibit polarity due to
presence of polar carbonyl and hydroxyl group. In general, carboxylic acids exist
in the dimer form due to intermolecular hydrogen bonding (as in case of
aliphatic acids). The aromatic carboxylic acids are generally crystalline solids
with high melting point. However, the substituted carboxylic acids may form
intramolecular hydrogen bonding as in case of 2-hydroxybenzoic acid (salicylic
acid).
21A.2 PREPARATION OF AROMATIC
CARBOXYLIC ACIDS
Besides the preparative methods meant exclusively for aromatic carboxylic
acids, some of the methods are similar to those of the synthesis of aliphatic
acids. A reference of section numbers has also been given for the methods that
have already been discussed in previous chapter. Various methods for the
preparation of aromatic carboxylic acids are as follows.
Oxidation reactions
1. Oxidation of alkyl or alkenyl benzenes. The oxidation of alkyl substituted
benzene or benzene with unsaturated carbon chain in the presence of strong
oxidizing agent like acidified potassium or sodium dichromate results in the
formation of benzoic acid, irrespective of the length of carbon chain of the alkyl
group (p. 319). The oxidation of xylenes results in the formation of
corresponding dicarboxylic acids in the following manner:

The oxidation cannot be carried out if groups like –OH and NH2 are present on
aromatic ring as they are highly susceptible to oxidation.
2. Oxidation of primary aromatic alcohol. The oxidation of benzyl alcohol
with acidified sodium dichromate or potassium permanganate results in the
formation of benzoic acid.

3. Oxidation of aromatic carbonyl compounds. The aromatic aldehydes or


alkylaryl ketones on oxidation with acidified dichromate or potassium
permanganate (Section 18.3.3) result in the formation of aromatic carboxylic
acid.

Hydrolysis reactions
1. Hydrolysis of trihaloalkyl benzenes. The chlorination of toluene in the
presence of ultraviolet light results in the formation of trichloromethyl benzene.
The alkaline hydrolysis of this product gives benzoic acid.

2. Hydrolysis of aromatic nitriles. The hydrolysis of benzonitrile in acidic or


alkaline medium results in the formation of benzoic acid via formation of an
amide. The mechanism is similar to that discussed earlier in case of aliphatic
carboxylic acids (refer Section 19.2.2; pp. 656–58).
3. Hydrolysis of ester. The hydrolysis of alkyl or aryl benzoates yields aromatic
carboxylic acid and alcohol or phenol.

The hydrolysis can occur in both acidic or alkaline media (the mechanism has
already been discussed in Section 19.2.2; p. 658 with aliphatic carboxylic acids).
Other methods
1. Carboxylation of organometallic compounds. In organometallic compounds
such as Grignard reagent and phenyllithium, the aryl group behaves as a
nucleophile, which adds carbon dioxide (a weak nucleophile) to form
carboxylate salts. The acidic hydrolysis of addition product results in the
formation of aromatic carboxylic acids.
2. Haloform reaction of acetophenone. The acetophenone is an aryl methyl
ketone, which reacts with halogen in the presence of sodium hydroxide to form
haloform and sodium benzoate.

3. Preparation of hydroxy benzoic acids. Further, the carboxylic group may be


introduced to phenols by using carbon dioxide in the presence of sodium
hydroxide [Kolbe–Schmitt reaction
(p. 542)] and carbon tetrachloride in the presence of sodium hydroxide [similar
to Reimer–Tiemann reaction (p. 543)]. These methods are used for the
preparation of hydroxy benzoic acids.
21A.3 CHEMICAL PROPERTIES OF AROMATIC
CARBOXYLIC ACIDS
Similar to the aliphatic counterparts, the aromatic carboxylic acids show similar
chemical properties except for the reactions involving α-carbon (these are the
reactions shown by aliphatic acids) and electrophilic substitution reactions
(characteristic of aromatic acids).
This section discusses in detail, the reactions that are exclusive to aromatic
carboxylic acids while summarizes others in brief.

21A.3.1 Acidic Character of Aromatic Carboxylic


Acids: A General Discussion
The acidic strength of aromatic carboxylic acid is attributed to resonance
stabilization of carboxylate ion formed by the loss of proton.

The aromatic carboxylic acids have high acidic strength


compared to aliphatic carboxylic acid. In aromatic carboxylic acids, –
COOH group is attached to sp2-hybridized carbon of phenyl ring,
whereas in carboxylic acids the –COOH group is attached to sp3-
hybridized carbon of alkyl group. The sp2-hybridized carbon is more
electronegative compared to sp3-hybridized carbon. Thus, phenyl ring
stabilizes the carboxylate ion more effectively compared to alkyl
group.

Effect of substituents on acidic strength


The presence of electron withdrawing groups increases the acidic strength as
these groups delocalize the negative charge and stabilize the carboxylate ion.
However, the electron releasing groups decrease the acidic strength by
intensifying the negative charge and thus, destabilizing the carboxylate ion.
The order of acidic strength of some substituted carboxylic acids is as follows:

The high acidic strength of p-nitrobenzoic acid is due to both –I and –R effects
of nitro group.

Comparison of acidic strength in isomeric substituted


benzoic acids (ortho, meta and para isomers)
In case of aromatic carboxylic acids the acidic strength depends upon
two factors:
(i) Inductive effect which in turn depends upon the position of the
substituent (lesser the distance between the substituents, stronger
is the inductive effect) and is experienced in the order ortho >
meta > para.
(ii) Resonance effect of substituents, which is observed maximum at
ortho and para positions. The influence of these factors can be
explained by the examples (a), (b), and (c) as follows:
(a) The order of acidic strength of ortho, meta, and para
nitrobenzoic acid is as follows:

The nitro group is electron withdrawing in nature and increases


the acidic strength of carboxylic acid through –I and –R effects.
The resonance effect is operative more at ortho and para
positions; thus ortho and para isomers are more acidic compared
to meta isomer. Among ortho and para, the ortho isomer is more
acidic due to inductive (–I) effect which is more operative for
ortho isomer as already mentioned. Thus, the overall acidic
strength in case of isomeric nitrobenzoic acids follows the order
ortho > para > meta.
(b) The order of acidic strength of ortho, meta, and para-
chlorobenzoic acid is as follows:
Chlorine exhibits +R effect but at the same time it exhibits a
strong –I effect. In this case inductive effect is the only factor,
which contributes towards acidic strength. The impact of
inductive effect follows the order ortho > meta > para (as
mentioned already). The acidic strength thus, follows the same
order.
(c) The order of acidic strength of ortho, meta, and para
hydroxybenzoic acid is as follows:

The –OH group is electron withdrawing in nature due to inductive


(–I ) effect but at the same time it exhibits a +R effect. Thus, the
only factor contributing towards acidic strength in this case is
inductive effect which follows the order ortho > meta > para.
Further, the relatively high acidic strength of ortho isomer is the result
of stabilization of carboxylate ion through intramolecular
hydrogen bonding.
Reactions involving acidic hydrogen
(i) The aromatic carboxylic acids react with bases such as sodium hydroxide,
sodium carbonate, and sodium bicarbonate to form sodium carboxylate. The
reaction with sodium bicarbonate is used for qualitative detection of carboxylic
acid functional group since the reaction occurs with evolution of carbon dioxide
in the form of brisk effervescence.

Sodium benzoate is used as a food preservative and is effective in acidic


medium.
(ii) Reaction with diazomethane (formation of methyl ester). The aromatic
carboxylic acids on reaction with diazomethane form methyl esters. For
example, the reaction of benzoic acid with diazomethane results in a high yield
of methyl benzoate at room temperature. The mechanism is similar to that
discussed for aliphatic acids (Section 19.3.2; p. 665)
21A.3.2 Reactions Involving –OH Group:
Nucleophilic Acyl Substitution
These are the nucleophilic substitution reactions of carboxylic acid, which occur
through an addition-elimination mechanism. The reactions may be acid or base
catalyzed. The general reaction may be written as follows:

The formation of acid derivatives, namely acid chlorides, acid anhydrides, esters,
and amides are the examples of nucleophilic substitution reactions occurring
through addition–elimination mechanism. The mechanism for all these reactions
is similar to that discussed in the aliphatic carboxylic acids (Section 19.3.3).
Preparation of aromatic carboxylic acid derivatives
(i) Acid chlorides. Reaction of carboxylic acid with thionyl chloride is one of
the best methods for preparation of acid chlorides.
(ii) Acid anhydride. Heating aromatic carboxylic acid and acid chloride
together results in the formation of acid anhydride and this method is used
widely for the synthesis of anhydrides.
(iii) Esters. The reaction of aromatic carboxylic acids with alcohol in acidic
medium results in the formation of esters.
(iv) Amides. Aromatic carboxylic acids react with ammonia or amine to form
amides through an addition–elimination mechanism. The ammonia (or amine)
behaves as a nucleophile in the reaction.
These reactions that lead to the formation of aromatic carboxylic acid derivatives
are summarized as follows using benzoic acid as one of the reactants.
21A.3.3 Reduction Reaction
The reduction of carboxylic acids is carried out in the presence of strong
reducing agents like lithium aluminium hydride. The reaction involves the
hydride transfer and reduces the carboxylic acid to alcohol.

21A.3.4 Reactions Involving –COOH Group


The reactions due to carboxylic group are similar to those discussed in aliphatic
acids. These reactions are carried out as salts of carboxylic acids. The reactions
are discussed here for benzoic acid as an example.
(i) Formation of hydrocarbons. Heating sodium salt of benzoic acid with soda
lime results in the formation of benzene.

(ii) Hunsdiecker reaction. Silver salt of benzoic acid is heated with bromine to
yield bromobenzene with the evolution of carbon dioxide.

(iii) Formation of aromatic carbonyl compounds. Calcium salt of benzoic


acid, when heated forms benzophenone while heating calcium benzoate with
calcium formate or calcium acetate results in the formation of benzaldehyde and
acetophenone respectively.

21A.3.5 Electrophilic Substitution Reactions


The carboxylic functional group attached directly to benzene ring behaves as an
electron withdrawing group through –R effect. Due to electron withdrawing
character (–R effect), the functional group behaves as a ring deactivator and
makes the ortho and para positions electron deficient.

Thus, the carboxylic acid group behaves as a ring deactivator and meta director
towards electrophilic substitution reactions such as halogenation and nitration.
Aromatic carboxylic acids do not undergo Friedel–Crafts reaction. [Note:
Friedel–Crafts reaction does not occur in presence of electron withdrawing
groups]

21A.3.6 Reactions of Some Important Substituted


Carboxylic Acids
Reactions of phthalic acid [a dicarboxylic acid]
Most of the reactions of dicarboxylic acids are similar to those of aromatic
carboxylic acids. An important dicarboxylic acid is phthalic acid. As mentioned
earlier, it is prepared by the oxidation of o-xylene. Phthalic acid forms anhydride
while isomeric dicarboxylic acids that is isophthalic acid and terphthalic acid do
not form anhydride.
A few important reactions of phthalic acid can be summarized as under:
Phthalic anhydride is an important derivative of phthalic acid and is used for the
preparations of number of important organic compounds.

Reactions of salicylic acid [2-hydroxybenzoic acid]


Salicylic acid exhibits the reactions of phenols as well as the reaction of
aromatic carboxylic acid. Besides this, it shows usual reactivity towards
electrophilic substitution reactions. This section gives a brief insight into these
reaction:
Reactions involving –OH and –COOH groups

Electrophilic substitution reactions


The orientation in electrophilic substitution reaction is governed by electron
releasing –OH group. Thus, sulfonation occurs at positions para with respect to
the –OH group. However in bromination and nitration, the decomposition in
carboxylic group occurs resulting in ortho and para tri-substituted phenols.
B. AROMATIC SULFONIC ACIDS
21B.1 INTRODUCTION
The class of aromatic compounds where sulfonic (–SO3H) group is directly
attached to aromatic ring system is termed as arenesulfonic acids. The sulfonic
acids are generally colourless crystalline compounds and since they are highly
soluble in water, they are often deliquescent in nature. The sulfonic acids are
most commonly used in the form of their salts.
IUPAC names them by adding the suffix sulfonic acid to the name of the parent
hydrocarbon. For example,
21B.2 PREPARATION OF AROMATIC SULFONIC
ACIDS

21B.2.1 Electrophilic Substitution Reactions


(1) Reaction with sulfuric acid. The sulfonation of benzene results in the
formation of benzene sulfonic acid. Similarly, the reaction of phenol and
nitrobenzene with sulfuric acid results in the formation of corresponding
substituted sulfonic acid

(2) Reaction with chlorosulfonic acid. The reaction of one equivalent of


chlorosulfonic acid with benzene results in the formation of benzenesulfonic
acid. However, the use of two equivalents of chlorosulfonic acid yields
benzenesulfonyl chloride.
21B.3 CHEMICAL PROPERTIES

21B.3.1 Acidic Character of Arenesulfonic Acid: A


General Discussion

Sulfonic acids are stronger acids compared to carboxylic


acids. Their acidic strength is attributed to resonance stabilization of
sulfonate ion. The negative charge is delocalized over three oxygen
atoms and this delocalization imparts stability to the sulfonate ion.

In carboxylic acids, delocalization of negative charge occurs over two


oxygen atoms only.

Effect of substituents on acidic strength


The electron withdrawing groups present on benzenesulfonic acid ring increase
its acidic strength by delocalization of charge. However, electron releasing
groups decrease the acidic strength. For example, the increasing order of acidic
strength for some substituted sulfonic acids mentioned below is given as
This can be explained by the stability of corresponding sulfonate ion (conjugate
base). The methyl group is electron releasing in nature and increases the
negative charge on sulfonate ion thereby destabilizing it. The destabilization
decreases the acidic strength. The nitro group is electron withdrawing in nature,
which delocalizes the negative charge in sulfonate ion and therefore stabilizes it.
More the number of electron withdrawing groups, more is the stabilization of
sulfonate ion and higher is the acidic strength.
Reactions involving acidic hydrogen (Replacement of hydrogen)
(i) Reaction with alkalis (formation of salts). Sulfonic acids are highly acidic
in nature and react with alkalis such as NaOH, Na2CO3, and NaHCO3 to form
sodium salts of sulfonic acid, also called sulfonates. The sulfonates are highly
soluble in water.

(ii) Reaction with dimethylsulfate (formation of methyl esters). The sodium


salt of sulfonic acid on heating with dimethylsulfate results in the formation of
methyl ester.

Reaction involving replacement of –OH group


Reaction with phosphorous pentachloride or thionyl chloride. Aromatic
sulfonic acids or their sodium salts on reaction with phosphorous pentachloride
or thionyl chloride result in the formation of arylsulfonyl chloride. For example

Arylsulfonyl chlorides undergo reactions similar to carboxylic acid chlorides. A


few important reactions may be summarized as follows:

Tosylates (p-Toluenesulfonates, PTS)—An important tool


in Organic Transformations
The p-toluenesulfonyl chloride (Tosyl chloride) on reaction with
alcohol forms sulfonic ester known as sulfonates (Tosylates). Unlike
benzenesulfonates the tosylates, in general, are crystalline derivatives.
The tosylates readily under go nucleophilic substitution reactions as
the tosylate ion is stabilized through delocalization and behaves as a
very good leaving group in nucleophilic substitution reactions.
In contrast, the –OH group of alcohol behaves as a poor leaving
group. Thus, the reaction of tosyl chloride with alcohol transforms the
alcoholic –OH to a better leaving group that is tosylate ion. The tosyl
chloride is conveniently prepared by reaction of thionyl chloride with
p-toluenesulfonic acid.

The reaction of tosylates result in the formation of various functional


groups through nucleophilic substitution reactions. The transformation
of alcohol to various functional groups is facilitated through tosylates
and a few important transformations are summarized below:
Reactions involving replacement of –SO3H group
[Nucleophilic substitution reactions]
The sulfonate group is a very good leaving group. Thus, arenesulfonic acids
undergo nucleophilic substitution reactions at carbon where replacement of –
SO3H group occurs. The reactions
are generally carried out with sodium salt of sulfonic acids. These reactions of
arenesulfonic acids are
(i) Fusion with sodium hydroxide [oxygen nucleophile]: formation of phenols
(ii) Reaction with cyanide ions [carbon nucleophile]: formation of aromatic
nitriles
(iii) Reaction with sodium hydrogen sulfide [sulfur nucleophile]: formation of
thiophenols
(iv) Reaction with sodamide [nitrogen nucleophile]: formation of aromatic
amines
These nucleophilic substitution reactions are summarized with the sodium salt of
benzenesulfonic acid as one prime reactant.
21B.3.2 Electrophilic Substitution Reactions
The sulfonic acid group is electron withdrawing in nature and exhibits –R effect.
Due to this, it behaves as a ring deactivator and meta director towards
electrophilic substitution reaction.

Sulfonic acids undergo the usual halogenation, nitration, and sulfonation


reactions but do not undergo Friedel–Crafts reaction. The summary of
electrophilic substitution reactions is as follows:
Benzenesulfonic acid on heating in aqueous acidic medium at 150–180°C
undergoes desulfonation to give benzene. The sulfonic acid group is replaced by
hydrogen under these reaction conditions.
21B.4 AROMATIC SULFONIC ACID
DERIVATIVES
Toluene on reaction with chlorosulfonic acid results in the formation of ortho
and para-toluene sulfonyl chlorides. The ortho and para isomers are separated
from the mixture and used in the preparation of saccharin and chloramine-T
respectively, which are two useful aromatic sulfonic acid derivatives.

Saccharin (o-Sulfobenzoic acid amide)


The o-toluenesulfonyl chloride on reaction with ammonia results in the
formation of
o-toluenesulfonamide (I). The oxidation with KMnO4 yields o-sulfamoylbenzoic
acid (II) which on heating results in the formation of saccharin.

Saccharin as such is insoluble in water, however, the reaction with sodium


hydroxide converts it to water-soluble sodium salt.
Chloramine-T (N-Chloro-p-toluenesulfonamide salt)
The p-toluenesulfonyl chloride (tosyl chloride) on reaction with ammonia results
in the formation of tosylamide (I) which on treatment with sodium hypochloride
results in the formation of a sodium salt of N-chloro-p-toluenesulfonamide,
popularly known as chloramine-T.
Chloramine-T is used as an antiseptic.

SELECTED SOLVED EXAMPLES


Example 1. Undertake the following conversions in a single step:
(i) Phenylacetic acid to toluene.
(ii) Benzoyl chloride to benzoylperoxide.
(iii) Benzene to 2-benzoylbenzoic acid.
(iv) Benzoic acid to cyclohexanecarboxylic acid.
Solution

Example 2. An organic compound [A] C7H5BrO2 on reaction with an alkali


(NaHCO3) forms a salt. However, on reaction with ferric chloride, it does not
produce any colour. The nitration of [A] gives a single mononitro derivative.
Give the structure of [A].
Solution. The molecular formula suggests an oxygen containing aromatic
compound.
(i) No colour with FeCl3 indicates that it is not a phenol.
(ii) Since it forms salt with a mild alkali NaHCO3, it can be a carboxylic acid.
Therefore, the possible structure may be:

(iii) Nitration of [I] will give two mononitro derivatives (at positions indicated
by arrows). Similarly, compounds [II] will also produce two mononitro
derivatives. A single mononitro derivative will be formed in case of p-
isomer. Thus, the structure of [A] is:

Example 3. Carry out the following conversions:


(i) p-xylene to 2,5-dimethylbenzoic acid.
(ii) Toluene to 4-chlorobenzoic acid.
(iii) Benzyl bromide to 2,4-dinitrophenylacetic acid.
(iv) Benzene to salol (phenyl salicylate).
Solution
Example 4. Arrange the following in increasing order of acidic strength
Solution. The acidic strength of aromatic carboxylic acids is directly related to
stability of corresponding carboxylate ions. Presence of electron withdrawing
groups on benzene ring increase the acidic strength whereas electron releasing
groups attached to benzene ring decrease the acidic strength as compared to
unsubstituted carboxylic acid (Sections 21A.3.1 and 21B.3.1).
(i) II < I < IV < III
The methoxy group is electronreleasing whereas –CN and –CHO groups are
electron withdrawing in nature. Thus, 4-methoxybenzoic acid (II) is least acidic.
The –CN group is stronger electron withdrawing than –CHO group. Thus, 4-
cyanobenzoic acid (III) is most acidic in the series.
(ii) I < II < III
The –I and –R effect of nitro group makes 4-nitrobenzoic acid (III) most acidic.
Halogens exhibit strong –I effect only. As chlorine is less electronegative than
fluorine it makes 4-chlorobenzoic acid (I) least acidic in the series.
(iii) III < I < II
Due to +I effect of methyl group 4-methylbenzoic acid (III) is least acidic. The –
I effect is experienced more when bromine is present meta to carboxylic group
compared to when present at para position. Thus, 4-bromobenzoic acid is least
acidic in the series.
(iv) II < III < I
The –I effect and –R effect of –NO2 group makes p-nitrobenzoic acid (I) most
acidic. The electron releasing effect of –OH (+R effect) is more compared to –
CH3 (+I effect), which makes
p-hydroxybenzoic acid (II) least acidic in the series.
(v) I < III < II < IV
Aromatic sulfonic acids are more acidic compared to aromatic carboxylic acids
due to resonance stabilization of sulfonate ion. Thus, benzoic acid (I) is least
acidic. In case of sulfonic acids, more the number of electron withdrawing
groups attached to benzene ring more is the acidic strength. Thus, 3,5-
dinitrobenzenesulfonic acid (iv) is most acidic in the series.
Example 5. Arrange the following acid amides in increasing order of their base
catalyzed hydrolysis:

Solution. The presence of an electron withdrawing group increases the


hydrolysis rate compared to benzamide. The electron releasing groups reduce the
hydrolysis rate compared to benzamide.
In amides, the “ortho effect” due to steric influence is important. An electron
withdrawing group, if present at ortho position of benzamide causes lowering of
the hydrolysis rate compared to benzamide. The rate of hydrolysis in 2-methyl
benzamide is the least due to:
(a) Presence of electron releasing CH3 group.
(b) Presence of group at ortho position.
Thus, increasing order of the rate of hydrolysis is:
EXERCISES
1. Give the IUPAC names for the following:

2. Name the carboxylic acids formed by the oxidation of


(a) Propylbenzene with acidified sodium dichromate.
(b) m-Xylene with acidified sodium dichromate.
(c) p-Nitrobenzyl alcohol with KMnO4.
3. Complete the following reactions:
4. Why do aromatic carboxylic acids have low acidic strength compared to
aromatic sulfonic acids?
5. Why does 2,6-diethylbenzoic acid on reaction with ethanol not undergo
esterification easily?
6. Carry out the following conversions:
(a) Phenol to aspirin
(b) Phthalic acid to phthalimide
(c) Phthalimide to anthranilic acid
(d) Salicylic acid to phenol
7. How will you prepare saccharin from o-toluenesulfonyl chloride?
8. How will you prepare chloramine-T from p-toluenesulfonyl chloride? ESSAY
I
MINI ESSAY
I
LIPIDS, Soaps AND Detergents
A. LIPIDS—FATS, OILS, AND WAXES
Lipids are one of the most important heterogeneous classes of naturally
occurring organic compounds, which are of immense importance in living
systems. The natural organic compounds, which are completely insoluble in
water but soluble in nonpolar solvents, are classified as lipids.
Fats, oils, waxes, phospholipids, steroids, water insoluble vitamins,
adrenocorticoid hormones, and sex hormones are few of the important examples
which belong to the lipids family. Most of
the lipids are biosynthesized in animal and plant bodies through acetyl esters like
acetyl
coenzymes A.

The following discussion sums up the important characteristics and properties of


fats, oils, and waxes in brief.

(1) Fats and Oils


Fats and oils have become an essential part of our day-to-day life. Fats that are
used in daily life are obtained from the most common dietary sources and can be
classified as:
(a) Animal fat: The major sources of animal fats are ghee (Hindi/(India);
ghee-fat processed from milk and used for cooking) butter, milk, cheese,
eggs, and fat from meat and fish.
(b) Vegetable fat: The main source of vegetable fat (vegetable oils) is plants.
This is stored in the seeds. Some examples are mustard, corn, sunflower,
cottonseed, olive, peanuts, and soyabean oils.
(c) Others: Cereals, pulses, nuts, and vegetables also contain small quantities
of fat. For example, rice and wheat contain approximately 3% fat.
Fat and oils are triesters of glycerols and are also called triglycerides.
Complete hydrolysis of triglycerides yields one molecule of glycerol and three
molecules of fatty acids. Fatty acids are monocarboxylic acids. Table 1 tabulates
the structural formulae and melting points of some common fatty acids. The
alkaline hydrolysis of fat or oil is referred to as saponification reaction.
In general, a particular fat or oil consists of a simple triglyceride, in which all
three fatty acids are identical. Though it may contain a mixture of triglycerides
(of simple triglycerides) which further yields a mixture of different fatty acids
during hydrolysis.

Characteristics of fatty acids

Fatty acids obtained from the triglycerides of plant or animal origin (few are
listed in Table 2) have some common characteristics that can be summarized as
follows:
(1) Most fatty acids in nature are unbranched and have even number of carbon
atoms (although exceptions are known).
(2) If unsaturation, is present in fats, all the double bonds are cis- (or Z) in
nature and are never conjugated.
(3) Saturated fatty acids have high melting points and fully extended carbon
chain.
(4) Unsaturated fatty acids have low melting points and have rigid bent
structure due to presence of double bond.
Dissimilarities between fats and oils
(i) Oils contain a much higher percentage of unsaturated fatty acids in
triglycerides whereas fats contain high percentage of saturated acids.
(ii) Fats are solid at room temperature (~20°C) and are called “oils” if they are
liquid at the same temperature.

Essential fatty acids (EFA)


Essential fatty acids cannot be synthesized in our body. They can be derived only
from food. The most important essential fatty acid (EFA) is Linoleic Acid,
which serves as a basis for the production of other essential fatty acids (namely,
linolenic acid and arachidonic acid). Vegetable oils are abundant in linoleic
acid. It should be noted that not all polyunsaturated fatty acids are essential fatty
acids. The dietary sources of EFA are enlisted in Table 3 as follows:
Rancidity
On exposure to air, most triglycerides develop unpleasant odour and flavour and
are said to become rancid. This phenomenon is called rancidity or
rancidification.
Rancidity causes the production of low molecular weight fatty acids and
carbonyl compounds, which are foul smelling. These products are formed due to
interaction of atmospheric moisture and oxygen with double bonds in the fat
molecules. For example, the odour of rancid butter is largely due to the presence
of butanoic acid formed as a result of the hydrolysis of butter fat.
Hydrogenation
Vegetable oils contain large proportions of unsaturated fatty acids and most of
them are liquid. The hydrogenation of vegetable oils under conditions of
optimum temperature and pressure in the presence of finely divided nickel
(catalytic hydrogenation) results in reduction of some or all of the double bonds,
thus converting the unsaturated oil into saturated oil (also called hydrogenated
oils). As a consequence, oils are converted into semi-solid or solid fat. The
process is called hardening of oils.
The resulting hydrogenated product is known as vegetable ghee, a popular
cooking medium in India. The hydrogenation increases the keeping quality of
oils (longer shelf life) and oils do not become rancid easily after being
hydrogenated.
During the process of hydrogenation, unsaturated fatty acids are converted into
saturated acids and the essential fatty acid content is drastically reduced.
Analyzing Fats and Oils
Acid value. The amount of free fatty acid present in an oil or fat is indicated by
its acid value.
It is defined as the number of milligrams of potassium hydroxide required for
neutralizing one milligram of oil or fat. The acid value is determined by titrating
a solution of oil or fat in pure alcohol against standard potassium hydroxide
solution. A high acid value indicates that the oil is of low quality, that is, stale
and may be due to rancidity it contains high amount of free acid.
Saponification value. The saponification value gives an estimate of the
molecular mass of fat or oil. The smaller the saponification value, higher is the
molecular mass. It is defined as the milligrams of potassium hydroxide required
to completely hydrolyze (saponify), one gram of fat or oil.
Iodine value. It is a measure of unsaturation in an oil or fat and is defined as the
grams of iodine that combine with 100 grams of oil or fat.
For determining the iodine value, iodine cannot be used directly because it does
not react with unsaturated oils. Instead, Wij’s solution (iodine monochloride in
glacial acetic acid) is used which reacts with unsaturated oils readily. Iodine
value is determined by adding excess of Wij’s solution (of known concentration)
to a solution of known weight of oil or fat. The quantity of unreacted iodine is
estimated by titration with standard sodium thiosulfate (hypo) solution.
Reichert–Meissl Value (RM Value). It is a measure of steam volatile fatty acids
present in fats or oils and is defined as the millilitres of N/10 potassium
hydroxide solution required for neutralizing the distillate of 5 g of hydrolyzed fat
or oil. It is mainly used for checking the purity of butter.

(2) Waxes
Waxes are esters of long chain linear fatty acids and long chain linear alcohols.
In nature, waxes are present as protective coating on leaves , feathers of birds,
and animal fur. The fatty acids and alcohols present in waxes are:

Some common examples of waxes are described as follows:


(a) Bee’s wax. It is obtained from bee’s honeycomb. Bee’s wax is an ester of
myricyl alcohol and palmitic acid (myricyl palmitate or
C15H31COOC30H61) and ceryl alcohol and myristic acid (ceryl myristate
or C13H27COOC26H53). It is used in shoe polishes, as water-proof
coatings, and for preparing candles.
(b) Carnauba wax. It is an ester of myricyl alcohol and cerotic acid (myricyl
cerotate or C25H51COOC30H61). It is extracted from the leaves of a
brazilian palm tree and is used for floor and automobile polishing.
(c) Spermaceti. It is obtained from sperm whale. It consists of cetyl palmitate
that is, C15H31COOC16H33. This wax is odourless, tasteless, and white in
colour and is widely used in preparing cosmetics and pharmaceuticals.
Dissimilarities between fats (and oils) and waxes
(i) Fats and oils are triglycerides, that is, they are triesters of fatty acids and
glycerol while waxes are esters of long chain fatty acids and long chain
alcohols.
(ii) Waxes are harder than fats and oils. Waxes are less soluble in common
solvents.
B. SOAPS AND DETERGENTS

Soaps
When a oil or fat is heated with alkali, the triglyceride is converted to glycerol
and salts of fatty acids. The salts (usually sodium salts) of long-chain fatty acids
are referred to as soaps.

Detergents
Synthetic detergents (sometimes called syndets) today dominate the market and
have taken over ordinary soaps. The development of syndets was a response to
two problems associated with the use of ordinary soaps. First being salts of weak
acids, soaps give rather alkaline solutions in water, due to partial hydrolysis and
alkalis can be harmful to certain fabrics. The ordinary soap cannot function well
at low pH (that is in acid) because the long chain fatty acids precipitate in the
solution. The second disadvantage of soaps is that they form insoluble salts with
calcium, magnesium, or ferric ions that may be present in hard water.
The first detergents to be introduced were sodium salts of alkyl hydrogen
sulfonates, such as lauryl hydrogen sulfate.
Lauryl hydrogen sulfate is an excellent detergent. Being salt of a strong acid, its
solutions are nearly neutral. Its calcium, magnesium and iron salts are water-
soluble. With similar properties, the detergents are effective in hard water also
and their cleaning action is not affected. At present, the most widely used
detergents are straight chain alkylbenzenesulfonates.

It is important that in detergents, the alkyl chain should be linear as these are
biodegradable. The chains can be degraded by microorganisms and do not
accumulate in the environment.
The detergents which have branched alkyl chains are non-biodegradable and
cause environmental contamination. Especially, such detergents are a threat to
aquatic species and pollute the water of ponds and rivers to the extent that it
becomes non-usable.
Washing Action of Soap and Detergents
The cleansing action of soaps and detergents is due to an important property of
being an emulsifying agent. The fabric with dirt/oil is dipped in a solution of
soap or detergent. The oil, dirt, or stains are insoluble in water. The nonpolar end
of soap and detergent molecule surrounds the oil (dirt) while the polar end,
which is hydrophilic remains suspended in water. The aggregates of a number of
such molecules with their non-polar segments (lypophilic) surrounding the oil or
dirt and polar part suspended in water are known as micelles. The micelles so
formed dissolve the dirt, which is washed away when fabric is rinsed thoroughly
with water.
Chapter22
Aliphatic Nitrogen
Containing Compounds
[Amines, Nitro, Nitriles, Isonitriles,
Isocyanates and Thiocyanates]

A. AMINES [ALKANAMINES)
22A.1 INTRODUCTION
Nitrogen is one of the essential components of our living system as well as of
our atmosphere. The nitrogen containing organic compounds are of immense
importance. Amines may be considered as organic derivatives of ammonia
where alkyl groups replace the hydrogen atoms of ammonia. Depending upon
the number of alkyl groups directly attached to nitrogen, the amines may be
classified as 1°, 2°, or 3° amines. In addition to this, tetra-substituted nitrogen
derivatives are also known and are termed as quaternary ammonium salts. The
general formulae for these types of amines are:

In IUPAC nomenclature system, amines are refered to as alkanamines. The


nomenclature has already been discussed in Chapter 2. A few selected examples
are as follows:

Structure. In a way similar to ammonia, the amines have a pyramidal structure.


The nitrogen in amines is sp3 hybridized. Due to presence of a lone pair of
electrons in sp3 hybrid orbital of nitrogen, the tetrahedral geometry distorts to
pyramidal geometry.
Isomerism. Amines exhibit two types of isomerism, that is, position isomerism
and metamerism. Position isomerism is exhibited by amines from propanamine
onwards, for example, propan-1-amine and propan-2-amine. Similarly,
metamerism is exhibited by ethanamine onwards due to isomeric structures
arising by the arrangement of carbons around nitrogen. For example,
(a) C2H7N has the following metamers:

(b) C4H11N has the following metamers:


Physical properties. Amines are polar in nature due to presence of
electronegative nitrogen and also form hydrogen bonding. However, their
polarity is less compared to alcohols, as oxygen is more electronegative
compared to nitrogen. This not only makes alcohols more polar but also makes
the hydrogen bonding in alcohols much stronger than amines. Thus, amines have
lower boiling points compared to alcohols. For the same reason, the amines have
lower boiling points compared to carboxylic acid of comparable molecular mass.

The boiling points of primary, secondary, and tertiary amines of the same
molecular mass follow the order 1° > 2° > 3°. The primary and secondary
amines form intermolecular hydrogen bonding through their hydrogens while
there is no hydrogen bonding in tertiary amines since hydrogen is absent. More
the number of hydrogens directly attached to nitrogen, stronger is the hydrogen
bonding. Thus, primary amines show strong hydrogen bonding relative to
secondary amines and thus, have higher boiling point compared to 2° and 3°
amines.
The low molecular mass amines are soluble in water due to the formation of
intermolecular hydrogen bonding with water molecules as shown below.
22A.2 PREPARATION OF ALKANAMINES
Amines can be prepared by a number of methods. The discussion on the various
preparatory methods can be classified under two heads, namely (1) reactions,
which yield mixture of 1°, 2° and 3° amines and (2) reactions specific to the
individual preparation of 1°, 2°, and 3° amines.

22A.2.1 Reactions, which Yield Mixture of 1°, 2°, and


3° Amines
(a) Reactions of haloalkanes with ammonia. The reaction of haloalkane with
ammonia under pressure results in the formation of a mixture of 1°, 2°, and 3°
amines. This is a nucleophilic substitution reaction.

If ammonia is used in excess, one gets primary amine as the major product.
However, excess of haloalkane results in the formation of 3° amine as the major
product.
(b) Reaction of alcohol with ammonia. The reaction of 1° alcohol with
ammonia in presence of Al2O3 or ThO2 at 360°C results in the formation of a
mixture of 1°, 2°, and 3° amines.

The mixture of amines formed by these two reactions is separated into different
amines by two important methods namely Hofmann’s method and Hinsberg
method.
Separation of mixture of 1°, 2°, and 3° amines
Hofmann’s method
The mixture of amines is treated with diethyl oxalate and the following results
are obtained
(i) The 1° amine reacts with diethyl oxalate to form oxamide (a solid
compound)
(ii) The 2° amine reacts with diethyl oxalate to form oxamic ester (a liquid
compound).
(iii) While 3° amine does not react with diethyl oxalate.

The mixture is then distilled, the solid residue is oxamide (from 1° amine) while
the distillate gives two fractions; the low boiling fraction which separates out is
pure 3° amine (because it does not react) while the high boiling fraction contains
oxamic ester (from 2° amine).
The 1° and 2° amines are then recovered from the separated derivatives by
hydrolyzing them with potassium hydroxide solution. The hydrolysis of oxamide
gives 1° amine while hydrolysis of oxamic ester gives 2° amine.
The following flowchart gives an overview of the Hofmann’s method which is
used for the separation of amines.

Hinsberg method (Hinsberg test)


This method is used for separation as well as identification of 1°, 2°, and 3°
amines. The method involves the treatment of the mixture of amines with
benzenesulfonyl chloride (Hinsberg reagent**) in the presence of aqueous
potassium hydroxide. The following results are obtained in the process:
(i) The primary amine reacts with benzenesulfonyl chloride to give
sulfonamide, which on treatment with aqueous potassium hydroxide forms
a water-soluble salt.
(ii) The secondary amine also forms a sulfonamide but on treatment with
aqueous potassium hydroxide it remains as an insoluble derivative.
(iii) The tertiary amine does not react with benzenesulfonyl chloride.

Extracting the mixture with ether, using a separating funnel, is then used to carry
out the separation. The upper ether layer contains insoluble sulfonamide (from
2° amine) and the unreacted 3° amine. The lower aqueous layer contains water-
soluble sulfonamide salt from 1° amine. The lower layer is separated and ether
layer is distilled. The residue of distillation contains insoluble sulfonamide from
2° amine and the distillate contains 3° amine.
Recovery of 1°and 2° amines. The recovery of primary and secondary amines
is carried out by treating the soluble sulfonamide salt (having 1° amine) with
HCl, followed by treatment with alkali, which releases pure 1° amine. Similarly,
acidification of insoluble sulfonamide followed by treatment with alkali gives 2°
amine.
The overall separation process is illustrated with the following flowchart:

22A.2.2 Reactions Specific to the Individual


Preparation of 1°, 2°, and 3° Amines
PRIMARY AMINES
(1) Reduction. The primary amines may be prepared by the reduction of nitro
compounds, nitriles, amides, oximes, and amines.
(i) Nitro compounds when reduced in the presence of Zn/HCl, or Sn/HCl, or
LiAlH4, or
H2/Pt, or Ni result in formation of primary amines.

(ii) Reduction of a nitrile is an important method for the preparation of amines


and is used for increasing the length of carbon chain during the synthesis of
amine from haloalkanes. The reduction is carried out by hydrogenation (in
the presence of a catalyst) or LiAlH4.

(iii) Acid amides on reduction with LiAlH4 form amine. This is one of the
widely used methods for the synthesis of amines since good yields are
obtained with this method without any side products. For example,
(iv) Oximes are formed by reaction of carbonyl compounds with
hydroxylamine. These oximes on reduction with LiAlH4 or Na/C2H5OH or
NaBH4 or H2/catalyst, result in the formation of amines.

(v) Reductive amination of aldehydes and ketone: Aldehydes and ketones on


reductive amination, that is, reduction in the presence of ammonia (or its
derivatives) results in the formation of primary amines. The condensation of
carbonyl compounds with ammonia or amines results in the formation of
imines which on reduction with H2/Pt or NaBH3CN (cyanoborohydride)
result in the formation of 1°amine.

(2) Hydrolysis. The acid catalyzed hydrolysis of alkaneisonitrile and alkaline


hydrolysis of alkaneisocyanates results in the formation of 1° amines.
(3) Hofmann degradation of amides. The Hofmann degradation method is
used for the conversion of amides to amines. The amide on treatment with
bromine in alkaline medium results in the formation of primary amine. An
important characteristic of this reaction is that an amine with one carbon less
than those in the amide is formed.

Mechanism. The mechanism involves a 1,2-migration of the alkyl group and the
reaction therefore is a rearrangement reaction, which proceeds through an
isocyanate intermediate. The reaction at times is referred as Hofmann
rearrangement. For example the mechanism of Hofmann degradation in case of
propanamide is as follows:
Step 1. Reaction with bromine (formation of N-bromoamide)

Step 2. 1,2-migration of alkyl group in alkaline medium (formation of isocyanate


intermediate)
Step 3. Hydrolysis of isocyanate.
The hydrolysis of isocyanate intermediate results in the formation of carbamic
acid, which undergoes decarboxylation to produce amine.

Now, we describe some reactions namely Curtius, Lossen, and Schmidt reactions
which are related to Hofmann reaction in the sense that all these proceed through
formation of isocyanate intermediate.
(4) Lossen rearrangement. The hydroxamic acid on heating in the an inert
solvent in the presence of acetic anhydride undergoes loss of water to form an
acetyl derivative that rearranges, through migration of alkyl group, to form
alkylisocyanate. The hydrolysis of isocyanate results in the formation of 1°
amine.

(5) Curtius reaction. Curtius reaction involves the pyrolysis of acid azide to
form isocyanate which produces 1°amine on hydrolysis. The formation of
isocyanate involves migration of alkyl group.
(6) Schmidt reaction. The reaction of carboxylic acid with hydrazoic acid
(HN3) in the presence of an acid catalyst (H2SO4) results in the formation of 1°
amine through an isocyanate intermediate. The formation of isocyanate involves
migration of alkyl group.

Mechanism

(7) Gabriel phthalimide synthesis. Gabriel phthalimide synthesis is a useful


method for preparing 1° amines. It involves the reaction of potassium salt of
phthalimide with a haloalkane followed by hydrolysis to yield 1° amine.
SECONDARY AMINES
(1) Reduction
(i) Alkane isonitrile on reduction in presence of H2/Ni or LiAlH4 results in
the formation of 2° amines.

(ii) N-substituted amides on reduction with LiAlH4 yield 2° amines.

(iii) The reaction of aldehydes or ketones with primary amines result in the
formation of Schiffs bases which on reduction form secondary amines.
(2) Hydrolysis of substituted cyanamides. One of the useful and convenient
method for preparation of pure 2° amines involves the reaction of calcium
cyanamide (CaNCN) with haloalkane to form disubstituted cyanamide. This
disubstituted cyanamide on hydrolysis followed by decarboxylation yields 2°
amine.

(3) From aniline. The reaction of aniline with haloalkane results in formation of
N-N-disubstituted aniline. Treating N-substituted aniline with nitrous acid
followed by hydrolysis results in the formation of 2°amine.

TERTIARY AMINES
(1) Reduction
(i) The reduction of dialkyl substituted amides with LiAlH4 results in the
formation of 3°amines in the following manner:

(ii) The condensation of aldehydes and secondary amine results in the


formation of iminium ion which on reduction with cyanoborohydride
(NaBH3CN) or H2/Ni, results in the formation of 3° amine.

(2) Hofmann elimination (heating quaternary ammoniun hydroxides). In the


synthesis of alkenes (p. 217) we have discussed that quaternary ammonium
hydroxides on heating undergo β-elimination with a simultaneous C–N bond
fission to produce an alkene (less substituted) and a tertiary amine.

This method is used for elucidation of nitrogen present in natural organic


compounds (Hofmann’s exhaustive methylation).
22A.3 CHEMICAL PROPERTIES
The amines are basic in nature because of the presence of a lone pair of a
electrons on nitrogen, which can be shared with other electron deficient species.
The reactions of amines depict the basicity of amines as well as the nucleophilic
character of amines to participate in substitution reactions.

22A.3.1 Basicity of Amines: A General Discussion


Amines abstract proton from acid to form corresponding ammonium ion.
Alkanamines are more basic compared to ammonia because of the presence of
electron releasing (+I effect) alkyl groups, which increases the availability of
electron on nitrogen. Thus, The expected order of basicity of amines is 3° > 2°
>1°. However it has been found that 3° amines are less basic compared to 2°
amine in aqueous solution as evident from pKb values of 1°, 2° and 3°
methylamines. (Recall, the lower the value of pKb, higher is the basicity.)

In other words, the presence of three alkyl groups (+I effect) in 3° amines
increases the electron density on nitrogen but the crowding due to the presence
of three bulkier alkyl groups hinders the ability of nitrogen to make these
electrons available to other species (like solvent molecules). Thus, the steric
hindrance in 3° amines makes them less basic compared to 2° amines. Thus, in
aqueous solution the order of basicity is (CH3)2NH > CH3NH2 > (CH3)3N.
The basicity of 2° and 3° amines can also be explained through solvation effect.
In aqueous solution, the amines form the ammonium ions. The basicity is
directly related to the stability of ammonium ions (conjugate acid). The
ammonium ions undergo solvation (association with water molecules). Higher
the solvation of ammonium ion, higher is the basicity of amine. In a 3° amine
because of the steric crowding and the availibilty of only one N–H bond (R3N+–
H), the solvation is less compared to that in a 2° amine.
However, in the absence of solvation effect (in non-aqueous medium), that is, in
gaseous phase the order of basicity is (CH3)3N > (CH3)2 NH > CH3NH2.
The basicity of amines is more compared to water.

The mineral acids react with amines and convert them to their respective
ammonium salts.

The amines can be recovered from their salts by treatment with strong alkali
such as hydroxide ion. The hydrochloride salts of amines on heating eliminate
chloroalkane to give the following products:
22A.3.2 Reactions of Amines
(1) Alkylation of amines. The amines react with haloalkanes to form substituted
amines.

(2) Reaction with acid chlorides (formation of amides). The reaction of acid
chlorides (aliphatic or aromatic) or sulfonyl chlorides with 1° and 2° amines
results in the formation of N-substituted and N,N-disubstituted amides. This is a
nucleophilic substitution reaction where amines behave as nucleophiles and react
at carbonyl carbon of acid chlorides. The 3° amines do not react with acid
chlorides.
The reaction of amines with benzoyl chloride in the presence of sodium
hydroxide solution is referred to as Schotten-Baumann reaction. Besides acid
chloride, the amide formation takes place with acid anhydrides also.
(3) Reaction with nitrous acid. Primary and secondary aliphatic amines react
with nitrous acid to give different products whereas 3° amines do not react with
it. The nitrous acid is produced in situ by reaction of sodium nitrite and
hydrochloric acid. The reactive species in this case is nitrosonium ion
(electrophile), which attacks the nucleophilic nitrogen of amines and replaces
proton (H+).
The reaction of 1° amines with nitrous acid yields unstable diazoniumion ion,
which decomposes with expulsion of nitrogen to give alcohol as the main
product along with traces of alkene.

The secondary amines form N-nitrosoamines (yellow oily compounds) on


reaction with nitrous
acid.

Tertiary amines react with nitrous acid to form soluble salts.

Libermann nitroso test (Qualitative detection of aliphatic 2° amines): The test


involves the reaction of secondary amine with nitrous acid to produce N-
nitrosoamine, which on heating with phenol in sulfuric acid, followed by
treatment with an alkali, gives blue colour.
(4) Reaction with phenylisocyanate. The 1° and 2° amines react with
phenylisocyanate to produce substituted ureas.

(5) Reaction with carbon disulfide—Hofmann’s mustard oil test. Addition of


1° amine to carbon disulfide results in the formation of dithiocarbamic acid from
which hydrogen sulfide is eliminated using mercury (II) chloride, to give
isothiocyanate as the final product. The alkyl isothiocyanates give a typical
mustard oil odour, and as a consequence, this reaction is popularly known as
Hofmann’s mustard oil test.

The isothiocyanates are used for the preparations of thiourea derivatives. This
occurs in the following manner:
Secondary amines react with carbon disulfide to produce N-substituted
dithiocarbamic acid but do not form isothiocyanate with HgCl2. It should be
noted that tertiary amines do not react with CS2.

(6) Carbylamine reaction. (A characteristic reaction of primary amines) All


primary amines, on heating with chloroform in presence of alcoholic potassium
hydroxide solution, result in the formation of alkaneisonitrile, which has
characteristic offensive odour. The reaction is known as carbylamine reaction or
isocyanide reaction.
This reaction is used as a qualitative test for detection of primary amine
functional group in organic compounds and is also referred to as carbylamine
test for primary amines.

Mechanism
Step 1. Formation of electrophile
In alkaline medium, chloroform eliminates an HCl molecule to form
dichlorocarbene, an electron deficient species.

Step 2. Attack of dichlorocarbene (electrophile) on nucleophilic nitrogen of 1°


amine

Step 3. Removal of HCl (2 moles): formation of isonitrile

B. ALIPHATIC NITRO COMPOUNDS


22B.1 INTRODUCTION
The organic compounds where nitro group (–NO2) is directly attached to the
carbon of hydrocarbon chain are known as aliphatic nitro compounds. The
electronic structure of the nitro group is a resonance hybrid of two contributing
structures (I and II). The nitro compound should be clearly distinguished from
their isomeric nitrous esters, which are known as alkylnitrites and in these esters,
the nitrogen is bonded only to oxygens.

Depending upon the number of hydrogen atoms present on the carbon to which
nitro group is attached, these are classified as 1°, 2°, or 3° nitro compounds. In
IUPAC nomenclature, they are named as nitroalkanes (refer Chapter 2 also). A
few examples are

Physical properties. As evident from their structure, the nitro compounds are
polar in nature and exhibit high dipole moment. Due to dipole-dipole
interactions, nitroalkanes have high boiling points. Most of the nitroalkanes are
colourless liquids, have pleasant smell, and are sparingly soluble in water.
The primary and secondary nitroalkanes exhibit tautomerism (nitro and aci
forms) as illustrated with the following examples.
22B.2 PREPARATION OF NITROALKANES

Substitution reactions

(1) Nitration of alkanes. Alkanes react with concentrated nitric acid and result
in the formation of nitroalkanes. The reaction occurs in vapour phase at high
temperature and follows free radical substitution mechanism. All possible
mononitroalkanes are produced in the reaction (refer
Section 5A.3.2).

(2) Reaction of haloalkanes with nitrite ion. The primary and secondary
haloalkanes (generally bromides and iodides) on reaction with sodium nitrite
undergo nucleophilic substitution reaction to produce nitroalkanes (refer p. 411).

(3) Reaction of nitrite ions with α-haloacids. The α-chlorocarboxylic acid on


reaction with potassium nitrite results in the formation of α-nitrocarboxylic acid,
which on heating undergoes decarboxylation to produce nitroalkane. For
example,

Oxidation reactions
(1) Oxidation of primary amines. The 1° and 2° alkyl groups containing 1°
amine are oxidized with peracids to nitro compounds. However, a 3° alkyl group
containing 1° amine is oxidized with KMnO4 to give excellent yields of
corresponding nitroalkane. For example,

(2) Oxidation of oximes. The oximes on oxidation in presence of


trifluoroperacetic acid result in the formation of nitro compounds.
22B.3 CHEMICAL PROPERTIES OF
NITROALKANES

22B.3.1 Reactions at α-Carbon


The nitro group is an electron withdrawing group and exhibits –I effect. Due to
–I effect, the hydrogen attached to α-carbon of nitroalkane is highly acidic. The
carbanion formed by removal of proton from α-carbon is resonance stabilized.
Thus in presence of base, the α-hydrogens are abstracted and reaction occurs at
α-carbon.

(1) Reaction with alkali (formation of salt). The acidic character (due to –I
effect of nitro group) is exhibited in reaction with strong alkalis, where salt
formation takes place.

(2) Reaction with halogen (formation of α-halonitroalkanes). The 1° and 2°


nitroalkanes on reaction with halogen in alkaline medium undergo halogenation
at α-position. For example,
In primary nitro group (in nitromethane) all the α-hydrogens
are replaced by halogens in alkaline medium.
In nitromethane the first hydrogen is replaced by halogen atom due to
the –I effect of nitro group. Replacement of hydrogen by halogen
further increases the acidity of α-hydrogen due to enhanced –I effect
which further eases the replacement of next hydrogen by halogen. For
example, nitromethane on reaction with chlorine results in the
formation of chloropicrin (trichloronitromethane).

(3) Reaction with aldehydes (formation of β-hydroxynitroalkane). In alkaline


medium, the condensation of aldehydes and nitroalkanes takes place in a
manner similar to aldol condensation to produce β-hydroxynitroalkanes. The
reaction is given by nitroalkanes that have α-hydrogen. For example,
Mechanism
Step 1. Abstraction of α-hydrogen from nitroalkane (formation of carbanion)

Step 2. Nucleophilic attack (of carbanion) on carbonyl carbon

Step 3. Protonation (formation of β-hydroxynitroalkane)

(4) Reaction with nitrous acid [Test for distinguishing 1°, 2°, and 3°
nitroalkanes]. The 1° and 2° nitroalkanes react with nitrous acid to form nitrolic
acid and pseudonitrol respectively. The 3° nitroalkanes do not react with nitrous
acid. The reaction forms the basis of Victor Meyer’s test for distinguishing 1°,
2°, and 3° alcohols. Nitrous acid is produced in situ from sodium nitrite and
hydrochloric acid.
22B.3.2 Reactions Involving Nitro Group
(1) Reduction (formation of amine and hydroxylamines). The nitroalkanes on
reduction result in the formation of amines or hydroxylamine depending on the
nature of the reducing agent. Reduction of nitroalkane in presence of metal
catalyst (H2/Pt or Pd or Ni or Zn/HCl or SnCl2/ HCl) results in the formation of
amines. On the other hand, reduction of nitroalkane in neutral medium using
Zn/NH4Cl results in the formation of hydroxylamine. The reactions can be
illustrated as:

(2) Action of heat. The primary and secondary nitroalkanes on heating at 300°C
undergo elimination of nitrous acid (HNO2) to produce alkenes.
(3) Hydrolysis of nitroalkanes (formation of carbonyl compounds). The
acidic hydrolysis of sodium salt of 1° or 2° nitroalkanes results in the formation
of aldehyde or ketone. This is known as Nef reaction. For example,

22B.3.3 Reactions Differentiating Nitroalkanes and


Alkylnitrites
As mentioned earlier due to ambident nature of nitrite ion (refer p. 409), the
attachment of an alkyl group may occur in two ways, that is, (i) attachment
through nitrogen yields nitroalkanes and (ii) attachment through oxygen yields
alkylnitrite.

The reactions, which distinguish the isomeric nitroalkanes and alkylnitrites can
be compared as follows:
C. DIAZOMETHANE [CH2N2]
22C.1 PREPARATION AND PROPERTIES
Diazomethane (CH2N2) is a toxic, yellow gas (b.p. –23°C). It is highly
explosive. The structure of diazomethane is the resonance hybrid of following
contributing structures.

Preparation. The solution of N-methyl-N-nitrosotoluene-4-sulfonamide in ether


on treatment with ethanolic potassium hydroxide results in the formation of
diazomethane. Diazomethane is obtained as a yellow etheral solution, which is
stable at 0°C.

Diazomethane can also be prepared from nitrosomethylurea but this synthesis is


not recommended due to the carcinogenic nature of nitrosomethylurea.

Chemical properties. Diazomethane is widely used in organic chemistry as a


methylating agent for acids, alcohols, and enols. Some reactions illustrating their
chemical properties are
Arndt Eistert synthesis. Reaction of acid chlorides with diazomethane results
in the formation of carboxylic acid via the formation of a ketene intermediate.
The carboxylic acid so formed contains one carbon more than acid chloride. The
reaction is as follows:

D. NITRILES AND ISONITRILES


[Alkanenitriles and Alkaneisonitriles]
22D.1 INTRODUCTION
The class of organic compounds where alkyl group is attached to –C≡N through
carbon are referred to as alkanenitriles and those where alkyl group is attached
through nitrogen are called alkaneisonitriles. The cyanide ion is ambident and
can form covalent bond through carbon or nitrogen (Section 11.3.2; p. 409).
Alkanenitrile and alkaneisonitrile may be considered as organic derivatives of
hydrocyanic acid (HCN). IUPAC names (for details, refer Chapter 2) of few
selected compounds have been listed here as follows:

Physical properties
(i) Alkanenitriles have a pleasant smell while alkaneisonitriles have offensive
odour.
(ii) Lower alkanenitriles are soluble in water (form hydrogen bonding with
water) but alkaneisonitriles are insoluble in water because of no hydrogen
bonding.
(iii) Alkanenitriles have higher boiling points in comparison of corresponding
isonitriles.
(iv) Alkanenitriles are less toxic compared to isonitriles
22D.2 PREPARATION OF ALKANENITRILES

22D.2.1 Substitution Reactions


(1) From haloalkanes. The primary and secondary haloalkanes on heating with
potassium cyanide in aqueous ethanolic medium undergo nucleophilic
substitution to form alkanenitriles.

(2) From alkali salts of sulfonic acid. The potassium alkylsulfate on heating
with potassium cyanide results in the formation of alkanenitrile. Some iosnitrile
is also produced during reaction.

22D.2.2 Dehydration Reactions


The alkanenitriles can be prepared by dehydration of ammonium carboxylate,
acid amide, or aldoxime. The reactions for preparation of ethanenitrile are
summarized as follows:
22D.2.3 Other Reactions
(1) From primary amines. Primary amines on heating at 300°C over copper
undergo dehydrogenation to produce alkanenitrile as

(2) From Grignard reagent. The Grignard reagent on reaction with cyanogens
chloride results in the formation of alkanenitrile. This method is used for the
preparation of 3° alkanenitriles.
22D.3 PREPARATION OF ISONITRILES
(1) From haloalkanes. Iodoalkane on heating with silver cyanide in aqueous
ethanolic solution undergoes nucleophilic substitution to form alkaneisonitrile
(Section 11.3.2).

(2) From amines. Primary amines on heating with ethanolic potassium


hydroxide and chloroform result in the formation of isonitrile which has an
offensive odour.

This reaction is used for distinguishing the primary amines from secondary and
tertiary amines in the qualitative analysis and the reaction is popularly known as
carbylamine test (or reaction).
22D.4 CHEMICAL PROPERTIES OF NITRILES
AND ISONITRILES
(1) Hydrolysis. The hydrolysis of alkanenitrile in acidic or alkaline medium
results in the formation of carboxylic acid. The hydrolysis of alkaneisonitrile
results in the formation of amine and formic acid. Isonitriles can be hydrolyzed
only in acidic medium.

(2) Reduction. Alkanenitriles undergo reduction to form primary amines or


aldehydes depending upon the type of reducing agent used. Reduction in the
presence of H2 in presence of metal catalyst or sodium in alcohol results in the
formation of primary amines. On the other hand, reduction in the presence of
SnCl2 and HCl (Stephen’s reaction) results in the formation of aldehydes.

Alkaneisonitriles on catalytic reduction or reduction in the presence of


Na/C2H5OH result in the formation of methyl 2° amines.

(3) Addition of hydrogen halides. The nitriles and isonitriles undergo reaction
with HX to form corresponding addition products.
(4) Reaction with Grignard reagent. Alkanenitriles on reaction with Grignard
reagent followed by hydrolysis result in the formation of ketones. This is the best
method for the preparation of ketones. The alkane isonitriles on reaction with
Grignard reagent result in the formation of amines and aldehydes. For example,

(5) Reactions given by nitriles only. A few reactions given only by


alkanenitriles are
(i) Reaction with hydrogen sulfide (formation of thioamides),
(ii) Reaction with carboxylic acid (formation of secondary amides), and
(iii) Reaction with alcohols (formation of esters)
The reactions are summarized by taking the example of ethanenitrile as the main
reactant in the following manner.

(6) Reactions given by isonitriles only. A few reactions given only by


isonitriles are
(i) Oxidation with HgO (formation of isocyanate)
(ii) Reaction with sulfur (formation of isothiocyanate)
(iii) Pyrolysis (rearrangement to nitriles)
Following are the reactions summarized for methaneisonitrile.

E. ISOCYANATES AND
ISOTHIOCYANATES
22E.1 INTRODUCTION
The isocyanates are nitrogen derivatives of carbonic acid whereas
alkylisothiocyanates are considered as esters of isothiocyanic acids.

Alkylisocyanates are liquids with pungent smell and are toxic in nature. The
isothiocyanates are liquids with pungent odour having strong mustard smell and
are insoluble in water. They are lachrymatory and cause blisters on skin.
22E.2 PREPARATION OF ISOCYANATES
Alkaneisonitriles on oxidation with HgO result in the formation of isocyanates
(refer Section 22D.4). Alkaneisocyanates are also formed as intermediates in
reactions such as Claisen, Hofmanm, and Loosen reaction in decomposition of
amides, azides, and hydroxamic acids. Besides this, isocyanates can be
synthesized by the following methods:
(1) Dimethyl sulfate on heating with potassium cyanate in the presence of
sodium carbonate results in the formation of methylisocyanate.
(2) The reaction of primary amines with phosgene results in the formation of
isocyanates. It is an process used for the synthesis of methyl isocyanate
(MIC; refer Notabilia 15) in the industries.
22E.3 CHEMICAL PROPERTIES OF
ISOCYANATES
Isocyanates are highly reactive compounds due to the cumulated double bond
system present. They react readily with nucleophiles such as water, alcohol,
ammonia, and amines.
(1) Hydrolysis. The alkyl isocyanates on hydrolysis form primary amine and
carbon dioxide. This is a highly exothermic reaction.
(2) Reaction with alcohols or phenols (formation of esters). The isocyanates
react readily with alcohols or phenols to form solid esters (urethanes).
(3) Reaction with ammonia. Isocyanates react readily with ammonia to yield
N-substituted
urea.
(4) Trimerization. In presence of bases, the isocyanates trimerize to form
cyanuric acids.
All these reactions are summarized as follows:
22E.4 PREPARATION OF ISOTHIOCYANATES
1. Hofmann mustard oil reaction (p. 786)
2. Reaction of isonitrile with sulfur (p. 800)
3. Reaction of primary amines with carbondisufide. The reaction of primary
amines with carbondisulfide in the presence of alkali (NaOH) results in the
formation of N-alkyldithiocarbamide, which reacts with ethyl chloroformate to
form alkyl isothiocyanate as
22E.5 CHEMICAL PROPERTIES OF
ISOTHIOCYANATES
The characteristic reactions of isothiocyanates are
(1) Hydrolysis. The alkylisothiocyanates are hydrolyzed on heating in acidic
medium to yield primary amines.
(2) Chlorination. Addition of chlorine to isothiocyanates results in the
formation of isocyanidedichloride.
(3) Reaction with amines. Isothiocyanates on reaction with primary or
secondary amines form corresponding thiourea. Tertiary amines do not react
with isothiocyanates.
These reactions are summarized taking the example of methylisothiocyanate.
SELECTED SOLVED EXAMPLES
Example 1. Giving suitable explanation, arrange the following in increasing
order of basicity.

Solution. Basicity is directly related to availability of electrons on a species.


(i) III < II < I
The +I effect of ethyl group makes ethanamine (I) most basic. The
participation of electrons on nitrogen, in delocalization with carbonyl group
makes ethanamide least basic. Delocalization also occurs in urea (II) but
one of the –NH2 group still has electrons available which makes (II) more
basic compared to III.
(ii) III < I < II
Due to +I effect of methyl group, methanamine (II) is most basic. In N,N-
diacetylamine (III) the lone pair on nitrogen participates in delocalization
with carbonyl groups on either side and is thus least basic.
(iii) I < III < II < IV < V
Sodium hydroxide (V) ionizes completely to furnish OH– ions and is most
basic. The electronegativity of nitrogen is less than oxygen thus basicity of
methanamine (IV) is more compared to ammonia (II) which in turn is more
basic than ethanol (III). Water (I) is least basic in the series.
Example 2. What will be the order of basic strength of nitriles (RCN), imine
(R2C=CN) and ammonia (NH3).
Solution. The order of basicity is
This is explained on the basis of hybridization at nitrogen

The sp hybrid orbitals are more electronegative (electron withdrawing)


compared to sp2 because of higher ‘s’ character. The sp3 hybrid orbitals are least
electronegative (least ‘s’ character). Thus, the availability of electrons in sp
hybrid nitrogen is least and so it is least basic.
Example 3. Guanidine, (NH2)2C = NH, is one of the strongest nitrogenous base.
How will you account for high degree of basicity in guanidine?
Solution. Guanidine undergoes delocalization. This delocalization is not much
effective, as in contributing structures charge separation takes place.

The guanidine accepts a proton readily, to form a cation. The cation undergoes
delocalization and is thus highly stabilized. Higher the stability of cation, higher
is the basicity of corresponding neutral molecule.

Thus, delocalization in cation makes guanidine a strong base.


Example 4. Carry out the following conversions:
(i) 1-Bromopropane to Propan-1-amine
(ii) 1-Bromopropane to Butan-1-amine
(iii) 2-Bromopropane to Propan-2-amine
(iv) Propan-2-amine to 2-Bromopropane
Solution.
Alternatively,

(iii) One method involves Gabriel phthalimide synthesis

Alternatively, through isonitriles:

Example 5
(i) But-1-ene to Pentan-1-amine
(ii) Butanal to Butan-1-amine
(iii) Butan-1-ol to Propan-1-amine
(iv) Propanone to Propan-2-amine
(v) Propan-2-amine to Propanone
(vi) 2-Methylpropanoic acid to Propan-2-amine
Solution

Example 6. Complete the following reactions:


Solution
EXERCISES
1. Give IUPAC names of the following:

2. Write all the possible isomers of C5H13N and give their IUPAC names.
3. Amines attached to different alkyl groups do not show optical activity, why?
4. Why do amines have lower boiling points compared to alcohols?
5. Arrange the following in increasing order of their boiling points.
(CH3CH2)3N , CH3CH2CH2NH2, (CH3CH2)2NH
6. Describe Hofmann’s and Hinsberg’s methods for separation of 1°, 2°, and 3°
amines?
7. What product will be obtained by reduction of CH3CH2CONH2 with
LiAlH4?
8. Give mechanism for the Hofmann degradation reaction.
9. How do Loosen rearrangement, Curtius reaction, and Schmidt reaction result
in the formation of amines?
10. Complete the following reactions:

11. Explain the basic character of amines.


12. Arrange the following in increasing order of basicity.
13. How will you differentiate between CH3CH2NH2 and (CH3)2NH?
14. Complete the following reactions:

15. Give mechanism for carbylamine reaction.


16. What happens when butanamine is treated with peracid?
17. Why do α-hydrogens in nitroalkanes show acidic behaviour?
18. Explain the tautomerism in primary and secondary nitroalkanes.
19. How will you convert nitromethane to chloropicrin?
20. Complete the following reactions:

21. Give reactions that differentiate 1°, 2°, and 3° nitroalkanes.


22. Carry out the following conversions:
(a) Propanenitrile to propanal
(b) Ethanamine to N-methylethanamine
(c) Acetonitrile to N-acetylacetamide
(d) Methanamine to methylisocyanate.
23. Give the method for preparation of diazomethane.
24. What is Arndt Eistert synthesis? Explain giving mechanism.
25. What is Hofmann’s mustard oil reaction?
* Now a days instead of benzenesulfonyl chloride the reagent used is p-toluenesulfonyl chloride, which is
commonly referred to as PTS.
H
Chapter23
Aromatic
Nitrogen Containing Compounds
(Nitro, Amines
and Diazonium Salts)

A. AROMATIC NITRO COMPOUNDS


23A.1 INTRODUCTION
The nitrogen containing organic compounds where nitro group (–NO2) is
directly attached to benzene ring are known as aromatic nitro compounds. The
compounds where nitro group is
attached to side chain of aromatic ring behave similar to aliphatic nitro
compounds. The structure and IUPAC names (along with common names) of
some selected nitro compounds are as
follows:

Physical properties. The nitro compounds are polar in nature and the dipole-
dipole interactions impart them high boiling points. Nitrobenzene has a dipole
moment 3.9 D and is a high boiling pale yellow liquid, which has a characteristic
bitter almond odour. Nitrobenzene is denser than and insoluble in water,
however, it dissolves in organic solvents. The substituted nitro derivatives are
generally solids. The melting and boiling points of aromatic nitro compounds
generally increase with an increase in the number of nitro groups.
23A.2 PREPARATION OF AROMATIC NITRO
COMPOUNDS
The different methods of preparation of aromatic nitro compounds are as
follows:
(1) Nitration of aromatic ring. The nitration of benzene is carried out with a
mixture of
nitric acid and sulfuric acid, known as nitrating mixture at ~50°C, which results
in the
formation of nitrobenzene. At higher temperature (~100°C), one more nitro
group is
introduced resulting in the formation of m-dinitrobenzene. It is an electrophilic
substitution
reaction where nitronium ion (+NO2) acts as an electrophile (for mechanism,
refer
Section 10.2.2).

The nitration in aromatic systems may also be carried out with acetyl nitrate or
NaNO2/CF3COOH.

For the nitration of highly activated aromatic systems such


as phenol and aniline, nitric acid alone is used for nitration. The
nitro group occupies ortho and (or) para position to the activating
group present in the aromatic ring.
This is because the nitrating mixture (HNO3 + H2SO4) may cause
oxidation of the activated aromatic ring system.

(2) From arenediazonium salts. The aromatic amines can be converted to nitro
compounds via diazonium salts. The first step involves reaction of aromatic
amines with nitrous acid in the presence of BF4– and the second step involves
the treatment of diazonium salt with sodium nitrite in the presence of copper
catalyst.

The conversion of diazonium to nitro group occurs even in the absence of


catalyst if the electron withdrawing groups are attached to aromatic ring system.

(3) Oxidation reactions. The nitro compounds can be prepared by oxidation of


amino and nitroso compounds. The oxidation of aromatic amines, with
trifluoroperacetic acid or m-chloroperbenzoic acid, results in the formation of
nitro compounds. The nitroso compounds are relatively easily oxidized to nitro
compounds.
(4) Rearrangement of N-nitroamines. N-nitroamines, in the presence of acids
undergo rearrangement to produce o-nitro and p-nitro aromatic amines. The o-
nitro product is the major product of this reaction.
23A.3 CHEMICAL PROPERTIES OF AROMATIC
NITRO COMPOUNDS
The nitro group is a strong electron withdrawing group and when attached
directly to aromatic ring system, withdraws electrons from aromatic ring through
inductive effect (–I effect) as well as resonance effect (–R effect).

The nitro group deactivates the aromatic ring towards electrophilic substitution
reactions and thus, behaves as a meta-director in these reactions. However, this
deactivation leads to nucleophilic substitution reaction in nitro compounds at
ortho and para positions.
In nitrobenzene, the ring is highly deactivated. As a result, nitrobenzene is stable
towards oxidation by different reagents and is generally used as a solvent in
oxidation reaction of other organic compounds.
The important reactions of nitro compounds include (i) nucleophilic substitution
reactions,
(ii) electrophilic substitution reactions, and (iii) reduction reactions. A discussion
of these reactions is as follows:

23A.3.1 Nucleophilic Substitution Reactions

Nucleophilic substitution reactions occur at ortho or para


positions.
The impact of –I and –R effects of nitro group is observed maximum
at the ortho and para positions in aromatic ring. As a result, ortho and
para positions become relatively electron deficient centres. So ortho
and para positions in nitro compounds are attacked readily by
nucleophiles (electron rich species) to form ortho and para substituted
products.

Reaction with carbon nucleophiles


(i) Alkylation reaction. Alkylation of nitrobenzene with methylsulfinyl
carbanion results in introduction of –CH3 group at ortho and para positions.

(ii) Reaction with organometallic compounds. Reaction of nitrobenzene with


alkyllithium or Grignard reagent (alkyl group behave as nucleophile in
organometallic compounds) followed by treatment with an oxidizing agent like
bromine, results in the introduction of alkyl group at ortho and para positions.
Alkylation using alkyl lithium results in good yields of the product.

Nucleophilic substitution reactions in substituted nitrobenzene derivatives.


In the presence of other substituents in nitrobenzene, the reaction with
nucleophile may cause replacement of nitro group. For example,
(i) Heating m-bromonitrobenzene with aqueous alcoholic KCN results in the
formation of
o-bromobenzoic acid and p-bromobenzoic acid.
(ii) Heating p-bromonitrobenzene with aqueous alcoholic KCN results in the
formation of only m-bromobenzoic acid. In these cases the attack of
nucleophile (CN–) occurs at the available ortho and para positions with
respect to nitro group which is followed by hydrolysis and removal of nitro
group through a complex rearrangement.

Reaction with oxygen nucleophiles


Heating nitrobenzene with potassium hydroxide solution results in the formation
of ortho and para nitrophenols. The HO– acts as a nucleophile in this reaction.

23A.3.2 Electrophilic Substitution Reactions


Nitro group is an electron withdrawing group. It deactivates the aromatic ring
towards electrophilic substitution reactions and directs the electrophile towards
the meta position. As compared to benzene, the electrophilic substitution in
nitrobenzene requires vigorous reaction conditions.

The aromatic nitro compounds do not undergo Friedel–Crafts


reactions as the aromatic system is highly deactivated by nitro group
(refer Section 10.2.3).
For example, the synthesis of m-nitroacetophenone and 1,2-dimethyl-
4-nitrobenzene cannot be carried out using Fridel–Crafts reaction.
This is depicted in the following reactions:
In fact, nitrobenzene is used as a solvent in Friedel–Crafts reaction of
other aromatic compounds.

23A.3.3 Reduction
The reduction of aromatic nitro compounds is a complex process and involves
the production of many intermediate products before the final stage—formation
of aromatic amine, is reached.
(a) The reduction of nitrobenzene in acidic or neutral solution results in the
formation of nitrosobenzene, which on reduction yields
phenylhydroxylamine. Further reduction of phenylhydroxylamine results
in the formation of aniline as the final product.

(b) Reduction of aromatic nitro compounds in alkaline medium yields the


same products as in case of acidic or neutral medium but the nitrosobenzene
and phenylhydroxylamine thus formed in reaction medium react with each
other to yield azoxybenzene.
The azoxybenzene, in turn, may be reduced reversibly to yield azobenzene and
hydrazobenzene. The hydrazobenzene so formed may be reduced to aniline as
a final reduction product. The hydrazobenzene undergoes intramolecular
rearrangement in acidic medium to produce benzidine and this reaction is known
as benzidine rearrangement.

A proper choice of reducing agent and conditions may result in the formation of
most of the above reduction products. The following reaction series summarizes
all these reduction reactions.

(c) Selective reduction. The dinitro compounds can undergo selective


reduction of one of the nitro groups of benzene ring in the presence of
ammonium sulfide or sodium sulfide. For example,

B. AROMATIC AMINES AND


ARENEDIAZONIUM SALTS
23B.1 INTRODUCTION
The aromatic amines belong to the class of nitrogen containing organic
compounds where –NH2 group is directly attached to aromatic ring system.
Similar to aliphatic amines, the aromatic amines may be classified as 1°, 2°, and
3° depending upon the aryl and(or) alkyl groups attached to nitrogen. The
compounds such as benzylamine have an aromatic ring but –NH2 group is
attached to side chain of the aromatic system, namely the alkyl group. Thus,
these amines undergo reactions similar to primary aliphatic amines. Most of the
aromatic amines are better known by their common names, however, in IUPAC
system they are referred to as arenamines. Following are some aromatic amines
along with their IUPAC and common names (in square brackets), and melting or
boiling points.

Physical properties. Aromatic amines are generally colourless liquids or solids,


which turn brown in the presence of light due to oxidation. Aromatic amines are
polar compounds and can form intermolecular hydrogen bonding which renders
high boiling points to them than that compared to aromatic hydrocarbons.
However due to the presence of bulkier phenyl group, they are not readily
soluble in water.
The hydrogen bonds formed with nitrogen have lesser strength compared to
hydrogen bonding formed through oxygen (as in phenol), since oxygen is more
electronegative compared to nitrogen.
23B.2 CHEMICAL PROPERTIES OF AROMATIC
AMINES
The reactions of aromatic amines are quite similar to aliphatic amines. However
in addition to this, the aromatic amines undergo electrophilic substitution
reactions. The aromatic amines exhibit a
+R effect and behave as electron donors to the ring. Thus, they activate the ring
at ortho and para positions towards electrophilic substitution reactions. The
various reactions of aromatic amines are as follows:

23B.2.1 Basicity in Aromatic Amines— A General


Discussion
The basic character of aromatic amines is due to lone pair of electrons present on
nitrogen. However, aromatic amines are less basic compared to aliphatic amines.
In aromatic amines, the –NH2 group is directly attached to sp2 hybridized
carbon of benzene ring whereas in aliphatic amines, the –NH2 group is attached
to sp3 hybrid carbon. The sp2 hybridized carbon is more electronegative
compared to sp3 hybridized carbon and thus, reduces the availability of lone pair
on nitrogen in aromatic amines.

Also, an aromatic amine such as aniline exhibits a resonance effect (+R effect),
due to which the lone pair on nitrogen participates in delocalization with π-
electrons of benzene ring system and is less available to be shared with other
species.
The order of basic strength of amines is as follows:

With an increase in the number of phenyl groups, the delocalization of lone pair
increases. Further, steric hindrance is also increased due to the presence of
phenyl groups and these two factors together reduce the availability of lone pair
of electrons on nitrogen thereby decreasing the basicity. Diphenylamine is a
weaker base than aniline while triphenylamine completely lacks basicity.
Effect of substituents on basicity of aromatic amines
In aromatic amines, the presence of electron releasing group on benzene ring
enhances the electron density on nitrogen and thus, increases the basicity.
However, the presence of electron withdrawing groups decreases the basicity of
amines. The order of basicity of different substituted amines is as follows:

The presence of –NO2 group which depicts –I effect as well as –R effect


enhances the delocalization of lone pair on nitrogen with π-electrons of the
benzene ring system and decreases the basicity of p-nitroaniline compared to
aniline. The +R effect of methoxy group enhances the electron density on
nitrogen thereby increasing its basicity compared to aniline.
Comparison of basicity in isomeric nitroanilines
The order of basicity of ortho, meta, and para nitroanilines is as
follows:

In case of aromatic amines, the basicity depends upon two factors:


(a) Inductive effect, which depends upon the position of the
substituent and is experienced in the order ortho > meta > para.
(b) Resonance effect, which is observed in maximum intensity at
ortho and para positions.
The electron withdrawing substituent on aromatic ring reduces the
basicity of substituted anilines. The –R effect of nitro group is
observed maximum at ortho and para positions. Thus, m-nitroaniline
is relatively more basic compared to ortho and para isomers as it
withdraws electrons through inductive effect (–I effect) only. Out of
o-nitroaniline and p-nitroaniline, –I effect is found to be stronger in
ortho isomer as compared to para. Thus, o-nitroaniline experiences a
strong –R and –I effect and is least basic.
Basicity and steric effects
In general, the steric hindrance in 3° amines reduces the basicity as compared to
1° and 2° amines as already mentioned (and also see, aliphatic amines Section
22A.3.1). However in some cases, the steric hindrance enhances the basicity. For
example, despite of steric crowding, N,N-dimethyl-2,4,6-trinitroaniline is a
much stronger base compared to 2,4,6-trinitroaniline.
Due to steric hindrance, the nitro groups at ortho positions as well as the
dimethyl amino groups move away and are no longer coplanar. This inhibits the
delocalization of lone pair on nitrogen with aromatic ring. In other words, –R
effect due to nitro group is not operative. Thus, the availability of lone pair of
electrons on nitrogen due to steric inhibition of resonance increases the basicity
of N,N-dimethyl-2,4,6-trinitroaniline. In case of 2,4,6-trinitroaniline, no such
steric hindrance is observed and the –R effect due to three –NO2 groups is
highly effective as shown above in the contributing structures and this reduces
the basicity.
Reaction due to basicity of aromatic amines
The aromatic amines are less basic compared to aliphatic amines but are more
basic compared to water. Aromatic amines (1°, 2°, and 3°) on treatment with
aqueous mineral acid or carboxylic acid result in the formation of their
corresponding salts. The amines can be regenerated from their salts by treating
them with alkalis. A few illustrative examples are as follows:
23B.2.2 Nucleophilic Substitution Reactions
The nitrogen of aromatic amines bearing a lone pair of electrons behaves as a
nucleophile. Reactions of amines with haloalkanes and with acid chlorides are
examples of nucleophilic substitution reaction.
(1) Reaction with haloalkanes ( Alkylation of amines )
The reaction of 1°, 2°, or 3° aromatic amines with haloalkanes results in their
alkylation. As a result, 1° amines get converted to 2° amines, the 2° amines to 3°
amines and 3° amines to quaternary ammonium salts. For example,

The quaternary ammonium halide on treatment with silver oxide results in the
formation of quaternary ammonium hydroxide with precipitation of silver halide.
Heating the quaternary ammonium hydroxide results in the formation of 3°
amine and an alkene. This reaction is known as Hofmann elimination (p. 217).
(2) Reaction with acid chlorides (formation of amides)
The nucleophilic substitution of aromatic amines with acid chlorides results in
the formation of
N-substituted amides. The reaction of amines with benzoyl choride is carried out
in alkaline medium. This benzoylation reaction is popularly called Schotten-
Baumann reaction.

Mechanism
Step 1. Attack of aniline on carbonyl carbon of benzoyl chloride
Step 2. Loss of proton
Step 3. Removal of HCl (formation of benzamide)

In a similar manner, the reaction of aromatic amines with aliphatic acid chlorides
(or aliphatic acid anhydride) results in the formation of amides. The acetylation
reaction is best carried out with acetic anhydride rather than acetyl chloride.

The reaction with benzene sulfonyl chloride results in the formation of


sulfonamides. The formation of amides takes place with 1° and 2° amines but 3°
amines do not form amides.

23B.2.3 Electrophilic Substitution Reactions


The –NH2, –NHR, and –NR2 groups in aromatic amines behave as strong ring
activators as well as ortho, para directors towards electrophilic substitution
reactions (Section 10.3.3). This is attributed to +R effect of these groups which
enhance the electron density at ortho and para positions (Section 23B.2.1).
Halogenation
The –NH2 group is such a powerful ring activator that halogenation occurs at all
the possible ortho and para positions. For example, aniline on reaction with
aqueous bromine results in the formation of 2,4,6-tribromoaniline. In a similar
manner, reaction of 4-ethylaniline with aqueous bromine solution yields 2,6-
dibromo-4-ethylaniline.

However, in presence of non polar solvents such as carbon disulfide or carbon


tetrachloride, the reaction of aniline with bromine results in the formation of
monobromoaniline.
Nitration
In the nitration of highly reactive aniline ring using nitric acid,
two major hurdles are encountered:
(i) Nitric acid is both a nitrating agent as well as a strong oxidizing
agent. Thus, besides nitration, it also oxidizes the highly activated
aromatic ring. As a result, the yield of nitro-substituted product is
reduced and much of material is lost as tar.
(ii) Although aniline is a ring activator (an ortho, para director), direct
nitration of aniline results in the meta substitution instead of ortho
and para substitution.
In the presence of mineral acids, the aniline readily forms anilinium
ion. The anilinium ion is a strong electron withdrawing group, which
directs the electrophile towards meta- position.
Sulfonation
Aniline on reaction with sulfuric acid results in the formation of anilinium
hydrogen sulfate. Heating this anilinium salt, yields p-aminobenzenesulfonic
acid, popularly known as sulfanilic acid.

Sulfanilic acid exists as a dipolar or zwitter ion. The acidic character due to
sulfonic acid group is increased as it is present in the form of sulfonate ion
whereas basic character is reduced due to anilinium ion.
It should be noted that amides of sulfanilic acids are known as sulfanilamides.
These compounds are used as potential sulfa drugs.
Reaction with nitrous acid
Nitrosation reaction. One of the most important electrophilic substitution
reactions is the nitrosation reaction, which occurs in the presence of nitrous acid.
This reaction can differentiate 1°, 2°, and 3° amines as they yield different
products with nitrous acid. The nitrous acid is produced insitu by reaction of
sodium nitrite and hydrochloric acid.
Primary aromatic amines react with nitrous acid to form aromatic diazonium
salts (arenediazonium salts). Unlike their aliphatic counterparts, the aromatic
diazonium salts are stable. Formation of diazonium salts and their further
reactions are of immense importance and are discussed in detail in the next
section.
The 2° amines on reaction with nitrous acid yield yellow oily N-nitroso
compounds.

The 3° aromatic amines react with nitrous acid and the nitrosation occurs at p-
position of the ring. The ring substitution by nitrosonium ion (an electrophile)
results in the formation of 4-nitroso-N,N-disubstituted aniline.

Mechanism. The nitrosation reaction is an electrophilic substitution reaction.


The electrophile in the reaction is nitrosonium ion, which is formed as follows:

The nitrosonium ion is a weak electrophile and thus attacks at an electron rich
site. In primary amines, the electron rich site is nitrogen.

For 2° amine: In 2° amines the electron rich site is again nitrogen, which is
attack by +NO, and a proton is displaced. Reaction occurs till the N-nitroso
compound is formed.

3° amine. In case of 3° amines,the presence of alkyl groups on nitrogen activates


the ring towards electrophilic substitution reaction. The ring is an activated
electron rich centre, which is attacked by +NO at para position followed by
removal of H+ to form nitrosoamine. The electrophilic substitution of +NO
occurs in ring systems attached to highly activating groups such as –NR2 or –
OH.
23B.3 ARENEDIAZONIUM SALTS
The reaction of primary aromatic amines with nitrous acid, at low temperature,
results in the formation of diazonium salts. For example,

23B.3.1 Chemical Properties of Diazonium Salts


Diazonium salts are an important class of compounds that help in introducing a
large number of substituents into the aromatic ring system. This helps in
synthesis of all those compounds where a certain substituent cannot be
introduced directly into aromatic ring system through usual electrophilic
substitution reactions. The reactions of diazonium salts are of two types, namely
(a) reactions involving replacement of diazo group (loss of nitrogen) and (b)
coupling reactions in which the diazo group is retained (electrophilic substitution
of diazonium ions).
Reactions involving replacement of diazo group
These reactions involve the attack of a substituent, which replaces the diazonium
group to give substituted product. The substitution of diazonium group results in
the formation of various classes of organic compounds that can be summarized
as follows:
(i) Preparation of aromatic hydrocarbons. The reactions of arenediazonium
salts with hypophosphorous acid result in the replacement of diazo group by
hydrogen. Further, the reaction of diazonium salt with benzene in alkaline
medium results in the formation of biaryl and is known as Gomberg reaction.
For example,
(ii) Preparations of haloarenes. The reaction of benzenediazonium salt with
sodium fluoroborate followed by heating results in the formation of
fluorobenzene. The reaction is known as Balz- Schiemann reaction. The
reaction of benzendiazonium salt with copper (I) chloride in HCl or copper (I)
bromide in HBr is known as Sandmeyer reaction and results in the formation of
chlorobenzene and bromobenzene respectively. The treatment of diazonium salt
with potassium iodide replaces diazo group by iodine to yield iodobenzene.

These reactions are widely used in organic conversion for introduction of


halogens in aromatic ring system at positions, which are not possible otherwise
by direct electrophilic substitution of
halogen (s).
(iii) Preparation of phenol, nitrile, nitrobenzene, and carboxylic acids. The
diazonium group can be replaced by –OH, –CN, NO2 and –COOH groups by
following reactions.
This electrophile can be none other than phenyl carbocation. The phenyl
carbocation is formed by the loss of nitrogen from benznediazonium salt under
reaction conditions.

Coupling reactions in which the diazo group is retained


(Electrophilic substitution of diazonium ions)
The diazonium salts react with powerful activating groups (such as –OH and
NH2 groups) containing aromatic compounds to form azo compounds. This
reaction of formation of azo compounds is known as the coupling reaction.

The coupling reaction is an electrophilic substitution reaction. Here, the


diazonium group behaves as a weak electrophile and thus couples with highly
activated aromatic ring. The powerful electron releasing groups activate the ring
at ortho and para positions. The diazonium group couples preferably at para
position of the ring.
Conditions for diazocoupling. The coupling of diazonium salt generally occurs
with highly activated aromatic ring, that is, with phenols, aniline, and substituted
anilines. The acidic or alkaline medium is to be maintained depending upon the
nature of the substrate with which the diazonium ion couples.

However, if para position is blocked or substituted, the coupling occurs at the


ortho position in the following manner:
SELECTED SOLVED EXAMPLES
Example 1. Arrange the following in increasing order of their basic strength
with suitable explanation.

Solution. Basic strength of amines is directly related to the availability of


electrons on nitrogen of amine group.
(i) III < II < IV < I
cyclohexylamine (I) being an aliphatic amine is most basic. Aromatic amines
are less basic compared to aliphatic amines because of resonance effect.
Presence of nitro group (–I and –R effect) decreases the availability of
electron on nitrogen of amine this makes
4-nitroaniline (III) least basic in the series. 4-methyl aniline (IV) is stronger
base than aniline (II) due to +I effect of methyl group.
(ii) II < III < I
In aniline the lone pair on nitrogen participates in delocalization with π
electrons of benzene ring. As the number of phenyl rings increases the
delocalization also increases and thus basicity decreases. Thus, aniline (I) is
most basic and N,N-diphenyl aniline (II) is least basic in the series.
(iii) III < II < I
Benzylamine (I) being an aliphatic amine is most basic. In acetanilide (III)
the lone pair on nitrogen participates in delocalization with benzene ring as
well as carbonyl group and is least basic.
(iv) III < II < I
N-methylpropanamine (I) being an aliphatic amine is most basic. In aromatic
amines, the presence of electron withdrawing nitro (–NO2) group decreases
the basic strength which makes 2,4-dinitroaniline (III) least basic.
Example 2. Carry out the following conversions:
(i) Nitrobenzene to 1,3-dichlorobenzene
(ii) Nitrobenzene to 4-nitrophenol
(iii) 2-Chloroaniline to 1,3-dichlorobenzene
(iv) Aniline to 4-nitrobenzoic acid
(v) Benzene to 3-fluoronitrobenzene
Solution
Example 3. Complete the following sequence of reactions:

Solution
Example 4. Carry out the following conversions:
(i) Benzene to metanilic acid (3-aminobenzenesulfonic acid)
(ii) Toluene to m-chlorotoluene
Solution
EXERCISES
1. Give IUPAC names of the following compounds:

2. Write the structure for all isomeric aromatic amines with molecular formula
C8H11N.
3. Why do aromatic amines have higher boiling point compared to aromatic
hydrocarbons?
4. Comment on the basic strength of aromatic amines with respect to aliphatic
amines.
5. Select the stronger base in the following pairs (Justify your answers with
reasons).
6. What is Hofmann elimination reaction?
7. What product is formed when aniline reacts with benzoyl chloride in alkaline
medium? Name the type of reaction involved and give suitable mechanism.
8. Although, in aniline the –NH2 group exhibits +R effect, direct nitration gives
meta- nitroaniline instead of expected ortho and para nitro product, explain.
9. What products are formed by reaction of the following amines with nitrous
acid?

10. Bromination of aniline gives 2,4,6-tribromoaniline however nitration of


aniline yields
3-nitroaniline as the major product. Explain.
11. Carry out the following conversions:
(a) Aniline to p-nitroaniline
(b) Aniline to sulfanilic acid
(c) Nitrobenzene to m-nitroaniline
(d) Nitrobenzene to sulfanilic acid
12. What is directive influence of nitro group in electrophilic substitution of
aromatic compounds?
13. Why does nitrobenzene not undergo Friedel–Crafts reaction?
14. Complete the following reactions:

15. Give the mechanism for the formation of arenediazonium salt.


16. Carry out following conversions:
(a) Aniline to benzoic acid
(b) Toluene to p-chlorotoluene
(c) Nitrobenzene to m-nitroaniline
(d) p-Chloroaniline to p-chlorophenol
(e) Aniline to biphenyl
(f) o-Toluidine to toluene
(g) p-Toluidine to p-cresol
(h) Aniline to m-nitrobenzoic acid
17. What is azo coupling reaction?
18. Why azo coupling reaction does not occur in strong acidic or strong alkaline
medium?
Chapter24
Polynuclear
Hydrocarbons

24.1 INTRODUCTION
The class of aromatic hydrocarbons containing more than one aromatic ring are
known as polynuclear hydrocarbons. The polynuclear hydrocarbons in which
two or more aromatic rings are fused together (generally through ortho carbons)
are referred to as condensed polynuclear compounds or fused ring hydrocarbons.
For example,

The present chapter includes the structure elucidation, preparation, and chemical
properties of fused ring system such as naphthalene, anthracene and
phenanthrene.
24.2 NAPHTHALENE
Naphthalene is one of the constituents of coal tar, which is also its industrial
source. Naphthalene is the simplest fused ring system. It is a colourless
crystalline solid (m.p. 80°C) and sublimes on heating. It has a characteristic
odour. It is insoluble in water but soluble in alcohol, ether, and benzene.
Naming the positions in naphthalene: The rings in naphthalene are designated
individually as A and B and are numbered using Arabic numerals or Greek
letters as follows:

In general, the monosubstituted naphthalenes are named using Greek letters α


and β. However, polysubstituted naphthalenes are named using numbers.

24.2.1 Structure of Naphthalene


Naphthalene has a molecular formula C10H8. The structure of naphthalene has
been deduced based on the following chemical reactions undergone by it.
(a) Naphthalene on oxidation gives phthalic acid, which indicates the presence
of an ortho disubstituted benzene ring.
(b) Nitration of naphthalene gives nitronaphthalene which on oxidation results
in the formation of nitrophthalic acid. This further confirms the presence of
benzene ring substituted at ortho position.
(c) Nitronaphthalene on reduction gives aminonaphthalene, which on
oxidation gives phthalic acid. This indicates the presence of ortho
substituted benzene ring. However, this benzene ring is different from the
benzene ring discussed earlier because the oxidation product does not
contain the amino group.

This is to further add that benzene ring containing an –NH2 group is highly
activated and is more susceptible to oxidation whereas benzene ring containing –
NO2 group is deactivated and does not undergo oxidation. These two factors
explain the product formation already discussed in (b) and (c) above.
Thus, naphthalene comprises two benzene ring systems fused at ortho position
and is a condensed polynuclear hydrocarbon. The reactions discussed so far can
be summarized as follows:
24.2.2 Aromaticity in Naphthalene
In naphthalene, the two benzene rings are condensed at ortho carbons where
each carbon is sp2 hybridized. It contains a delocalized π-electron system with
ten π-electrons [Fig. 24.1(a)] and follows Huckel’s (4n + 2) rule. Thus,
naphthalene is an aromatic system (Section 9A.4).

Figure 24.1 (a) Overlap of p-orbitals in naphthalene (hydrogens not shown in the figure); (b) resonance in
naphthalene; and (c) hybridization and bond lengths in naphthalene.
Benzene has resonance energy 36 kcal mol–1. Since in naphthalene the two
benzene rings are fused together, the resonance energy of 2 × 36 = 72 kcal mol–1
is expected. In reality, naphthalene has a resonance energy of 61 kcal mol–1. The
two benzene rings share a pair of electrons and therefore the total number of π-
electrons available for delocalization is not twelve but ten. This means that
delocalization energy per ring is decreased. This decrease in the resonance
energy (delocalization energy) reduces the aromaticity and for this reason, in
general, condensed polynuclear hydrocarbons have relatively less aromatic
character, which is evident from their high chemical reactivity. Naphthalene is a
hybrid of three contributing structures [Fig. 24.1(b)].
In case of naphthalene, the C1–C2 bond length is shorter than the C2–C3 bond
length
[Fig. 24.1(c)].
In two of the contributing structures (I and II in Fig. 24.1(b)), a double bond
character develops between C1 and C2 whereas in C2–C3 bond, it is only in one
of the contributing structure III. Thus, overall double bond character in C1–C2 is
more than that in C2–C3 bonds and this attributes to the shortening of C1–C2
bond.
NOTABILIA 16
24.2.3 Preparations
Naphthalene can be prepared by the following methods:
(i) Haworth synthesis. The Friedel–Crafts acylation of benzene with succinic
anhydride gives
3-benzoylpropanoic acid (I), which on Clemmensen reduction forms 4-
phenylbutanoic acid (II). Heating this product in presence of concentrated
H2SO4 results in ring closure by elimination of water molecule to form α-
tetralone (III). The Clemmensen reduction of α-tetralone gives
tetrahydronaphthalene [tetralene (IV)] which on dehydrogenation with selenium
results in the formation of naphthalene.

Haworth synthesis of 1-alkylnaphthalenes: The reaction of Grignard reagent


with α-tetralone (III) introduces an alkyl group at the first position. When this
product undergoes dehydration in presence of an acid followed by treatment with
selenium, 1-alkylnaphthalene is formed as the final product.

Haworth synthesis of 2-alkylnaphthalenes and other-substituted naphthalenes:


The Haworth synthesis using alkylbenzene and succinic anhydride results in the
formation of 2-alkylnaphthalene. The Friedel–Crafts acylation with succinic
anhydride occurs at para position.

(ii) From 4-phenylbut-1-ene. Passing 4-phenylbut-1-ene over red-hot calcium


oxide results in the formation of naphthalene in the following manner:
(iii) Fittig’s synthesis. The 4-phenylbut-3-enoic acid, on heating with
concentrated H2SO4, undergoes ring closure to form α-naphthol which on
distillation over zinc dust forms
naphthalene.

24.2.4 Chemical Properties


Naphthalene is more reactive than benzene. It undergoes electrophilic
substitution reactions more easily than benzene. It also undergoes addition
reaction and is more susceptible to oxidation and reduction as compared to
benzene. These reactions indicate that naphthalene is less aromatic than benzene.

Reduction of naphthalene
Naphthalene on reduction (addition of hydrogen) by different methods results in
the formation of number of products.
(i) Reduction of naphthalene with sodium and ethanol results in the formation
of 1,4-dihydrodialin (1,4-dihydronaphthalene).
(ii) Reduction of naphthalene with isopentanol (isoamyl alcohol) and sodium
results in the formation of tetralin (1,2,3,4-tetrahydronaphthalene).
(iii) Catalytic reduction (Formation of trans and cis-decalin). Naphthalene on
catalytic reduction with H2/Ni results in the formation of tetralin, which is
further reduced to yield trans-decalin. On the other hand, the catalytic
reduction of naphthalene with H2/Pt results in the formation of cis-decalin.
Trans-decalin is more stable than cis-decalin due to less strained structure.
These reduction reactions of naphthalene are summarized as follows:

Addition of halogens
The addition of chlorine to naphthalene results in the formation of naphthalene
dichloride, which is a 1,4-dichloro addition product. Further addition of chlorine
results in the formation of naphthalene tetrachloride, which is a 1,2,3,4-tetra
addition product. Bromine also results in the formation of di- and tetra-bromides.
In the addition of halogens, all the atoms (of chlorine or bromine) add to the
same ring of naphthalene.

The dehydrohalogenation of di- and tetra-naphthalene halide results in the


formation of substituted halo-naphthalene derivatives. On oxidation, these
addition products result in the formation of phthalic acid.
Oxidation reactions
(i) The oxidation of naphthalene results in the formation of different products
depending upon the oxidizing agent used. Oxidation of naphthalene with
potassium permanganate in acidic medium results in the formation of
phthalic acid. On the other hand, oxidation of naphthalene with potassium
permanganate in alkaline medium yields phthalonic acid.
(ii) Oxidation of naphthalene with chromic acid results in the formation of
1,4-naphthoquinone.
(iii) Oxidation of naphthalene with oxygen in presence of vanadium
pentaoxide catalyst results in the formation of phthalic anhydride. The same
product is obtained on oxidation of naphthalene with H2SO4/HgSO4.
(iv) Ozonolysis reaction. Naphthalene on reaction with ozone forms a
diozonide, which on reductive cleavage in the presence of Zn/H2O results
in the formation of phthalaldehyde.
The oxidation reactions discussed above are summarized as follows:

Electrophilic substitution reactions


In naphthalene, attack of electrophile occurs preferably at
α-carbon
(C1 position)
Naphthalene undergoes electrophilic substitution reactions similar to
benzene.
In naphthalene, electrophilic substitution can occur at C1(α-carbon) or
at C2
(β-carbon). The carbocation formed by attack of electrophile at α-
position is much more stabilized by resonance since it has four
contributing structures in which aromatic character is retained in one
of the ring [refer Fig. 24.2 structures I–IV]. The carbocation formed
by the attack of electrophile at β-position has only two contributing
structures in which aromatic character is retained in one of the rings
(structures V and VI of Fig. 24.2). Thus, electrophilic substitution
occurs preferably at α-carbon that is, C1 position.
Attack at α-position (C1)

Attack at β-position (C2)

Fig. 24.2 Attack of electrophile at α- and β-positions.


(i) Nitration. Nitration of naphthalene at room temperature results in the
formation of
1-nitronaphthalene. The 1-nitronaphthalene on reduction with Fe/HCl results in
the formation of
1-naphthylamine, which on hydrolysis in acidic medium results in the formation
of 1-naphthol
(α-naphthol). When 1-nitronaphthalene reacts with PCl5 it results in the
formation of
1-chloronaphthalene. The reactions can be represented as follows:

(ii) Halogenation. The reaction of chlorine with naphthalene in the presence of


iron catalyst results in the formation of 1-chloronaphthalene as the major product
and 2-chloronaphthalene as a minor product. Similarly, bromination in carbon
tetrachloride results in the formation of
1-bromonaphthalene.

(iii) Friedel–Crafts Alkylation. The fused ring compounds are highly reactive
and are attacked by the catalyst AlCl3 under vigorous conditions. As a result,
poor yields are obtained in Friedel–Crafts reaction. For better yields, the
Friedel–Crafts alkylation of naphthalene is carried out at low temperature (mild
conditions). The Friedel–Crafts alkylation of naphthalene with iodomethane
results in the formation of 1-methylnaphthalene as a major product along with 2-
methylnaphthalene as a minor product. However, Friedel–Crafts alkylation with
bromoethane results in the formation of 2-ethylnaphthalene as the only product.
The reaction of naphthalene with bromopropane yields 2-isopropylnaphthalene
as the propyl cation formed during reaction rearranges to a more stable isopropyl
cation (p. 336).

(iv) Friedel–Crafts Acylation. The reaction of naphthalene with acetyl chloride


in the presence of AlCl3 as a catalyst results in the formation of methyl 1-
naphthyl ketone. The product formation depends on nature of solvent used.
Reaction in the presence of nonpolar solvent, such as carbon disulfide results in
the formation of methyl 1-naphthyl ketone as the major product whereas,
reaction in a polar solvent such as nitrobenzene gives methyl 2-naphthyl ketone
as the major product.

(v) Sulfonation. The sulfonation of naphthalene at a low temperature (<80°C)


results in the formation of naphthalene-1-sulfonic acid as the major product.
Sulfonation at higher temperatures (>120°C) results in the formation of
naphthalene-2-sulfonic acid as the major product.
The formation of naphthalene-1-sulfonic acid is a
kinetically controlled reaction and the formation of naphthalene-
2-sulfonic acid is a thermodynamically controlled reaction.
Sulfonation is a reversible reaction. Equilibrium exists between 1- and
2-substituted sulfonic acids. At low temperatures, substitution occurs
readily at the 1-position and naphthalene-1-sulfonic acid is formed in
larger percentage.
At higher temperatures, naphthalene-1-sulfonic acid readily undergoes
desulfonation to form naphthalene as an intermediate and is converted
into naphthalene-2-sulfonic acid. Naphthalene-2-sulfonic acid is
formed at a slow rate but once formed, being thermodynamically
stable it resists desulfonation. Thus at higher temperatures,
naphthalene-2-sulfonic acid is formed as the major product.

Electrophilic substitution in naphthalene derivatives


The electrophilic substitution in monosubstituted naphthalenes (1- or 2-
substituted), depends upon the nature of group already present on the ring (Fig.
24.3).
In 1-substituted naphthalene:
(i) If substituent is an electron-releasing group, electrophilic substitution is
homonuclear (that is, on the same ring carrying the substituent) and occurs
preferably at C4 or less at C2.
(ii) If substituent is electron-withdrawing group, electrophilic substitution is
heteronuclear (that is, on other fused ring) and occurs at C5 or C8 position.
In 2-substituted naphthalene:
(i) If the substituent is electron-releasing group, electrophilic substitution is
homonuclear and occurs at C1. However if the substituent is a halogen or
amino group, electrophilic substitution is heteronuclear and occurs at C5 or
C8.
(ii) If substituent is electron-withdrawing group, electrophilic substitution is
heteronuclear and occurs at C5 or C8.
24.3 DERIVATIVES OF NAPHTHALENE

24.3.1 Naphthols
Preparations
(i) 1-Naphthol and 2-naphthol (α-naphthol and β-naphthol) occur in small
quantities in coal tar. The industrial preparation of naphthol involves fusion
of corresponding naphthalene sulfonic acid with sodium hydroxide.

(ii) 1-Naphthylamine (α-naphthylamine) on hydrolysis with sulfuric acid at


180°C results in the formation of 1-naphthol.

Chemical properties
The chemical reactions of naphthols are similar to phenols. Formation of esters
and ethers with naphthols is much faster as compared to phenols since naphthols
are more acidic than phenols. The increased acidic strength in case of naphthols
is attributed to the stabilization of naphthoxide ion due to more delocalization as
compared to phenoxide ion. In naphthoxide ion, the delocalization involves both
the rings as shown below:
Naphthols undergo coupling reactions with benzene diazonium salts and this
reaction is used for preparation of azo dyes. 1-Naphthol undergoes coupling at
C4 whereas 2-naphthol undergoes coupling at C1.

With ferric chloride solution, 1-naphthol (α-naphthol) gives violet colour and 2-
naphthol
(β-naphthol) gives green colour. This reaction is used for the qualitative
detection of α- and
β-naphthols.

24.3.2 Naphthylamines
Preparation
Bucherer reaction. Naphthylamines are prepared from naphthols by Bucherer
reaction. This involves heating of naphthols with aqueous solution of
ammonium hydrogen sulfite and ammonia. In the process, 1-naphthol results in
the formation of 1-naphthylamine and 2-naphthol yields 2-naphthylamine.
Mechanism

1-Naphthylamine can be conveniently prepared by reduction of 1-


nitronaphthalene.

Chemical properties
1-Naphthylamine on diazotization forms naphthalene-1-diazonium salt,

which can be used for the preparation of other 1-substituted naphthalene


derivatives as follows:
Naphthalene-2-diazonium chloride undergoes similar reactions to form 2-
substituted naphthalene derivatives.
1-Naphthylamine on heating with sulfuric acid results in the formation of
naphthionic acid, which is used extensively in dye industry.

24.3.3 Naphthoic Acids


Naphthoic acids can be prepared by oxidation of methylnaphthalene in the
presence of sodium dichromate in acidic medium. A better alternative method
involves use of Grignard reagent, naphthylmagnesium bromide, which on
reaction with carbon dioxide followed by hydrolysis results in the formation of
naphthoic acid.

The 2-naphthoic acid is conveniently prepared from 2-acetylnaphthalene by


iodoform reaction.

The sodium salt of 2-naphthol on reaction with CO2 (Kolbe-Schmitt reaction)


results in the formation of 2-hydroxy-1-naphthoic acid (I) at low temperature
and 3-hydroxy-2-naphthoic acid (II) at high temperature. The product I on
heating gets converted to a more stable product II.

24.3.4 Naphthoquinones
There are three types of possible naphthoquinones depending upon the position
of the two carbonyl groups. The two carbonyl groups in naphthoquinones may
be present in the same ring or different rings. The naphthoquinones are yellow
coloured crystalline solids. It should be noted that
(a) 1-Amino-2-naphthol is synthesized by coupling of 2-naphthol and benzene
diazonium chloride, followed by reduction. 1-amino-2-naphthol on
oxidation results in the formation of 1,2-naphthoquinone.

(b) Oxidation of 4-amino-1-naphthol with chromic acid results in the


formation of 1,4-naphthoquinone. 4-amino-1-naphthol is synthesized by
coupling of 1-naphthol and benzenediazonium chloride, followed by
reduction.

(c) Oxidation of 2,6-dihydroxynaphthalene results in the formation of 2,6-


naphthoquinone.
Naphthoquinones have oxidizing properties and among these, 2,6-
naphthoquinone is the strongest oxidizing agent. An important point to
remember is that vitamin K is a 1,4-naphthoquinone derivative.
24.4 ANTHRACENE
Anthracene is a fused ring hydrocarbon where three benzene rings are fused
together. It is a colourless solid (m.p. 218°C) and it shows blue fluorescence in
ultraviolet light. It is obtained
from coal tar distillation along with phenanthrene and carbazole. Anthracene is
isolated from
this mixture by washing with naphtha followed by a treatment with pyridine,
which causes
removal of phenanthrene and carbazole respectively. Anthracene is then purified
by
sublimation.
An alternative method involves removal of phenanthrene from the mixture by
washing with naphtha and then oxidizing the mixture in presence of V2O5 that
removes carbazole in the form of CO2 and anthracene is oxidized to
anthraquinone. The reduction of anthraquinone gives good yields of anthracene.

24.4.1 Structure of Anthracene


Anthracene has a molecular formula C14H10. Its structure can be determined by
following reactions:
(a) Oxidation of anthracene results in the formation of anthraquinone, which
on fusion with KOH results in the formation of two molecules of benzoic
acid.

This indicates the presence of two benzene rings in anthracene.


(b) Bromination of anthracene gives a monobromosubstituted product, which
on fusion with potassium hydroxide gives hydroxyanthracene. Oxidation of
hydroxyanthracene results in the formation of phthalic acid and 2-
benzoylbenzoic acid.
This suggests the basic skeleton of anthracene to be of the form as shown in
structure [I]. Since the oxidation product of anthracene is anthraquinone, the
structure of anthracene can be given as structure II.

Thus, anthracene is a polynuclear hydrocarbon where three benzene rings are


fused at ortho positions.
The numbering in anthracene ring and designation of positions as α, β and γ, is
carried out as follows:

Aromaticity in anthracene
In anthracene, all the carbons are sp2 hybridized and the three fused rings lie in
the same plane, which gives a flat structure to anthracene. The ring has fourteen
π-electron delocalized system (obeys Huckel’s rule) and is aromatic in nature.
The structure of anthracene is a hybrid of following contributing structures:

24.4.2 Preparations
The anthracene can be prepared by Friedel–Crafts reaction in the following
manner:
(a) The acylation of benzene with phthalic anhydride in presence of AlCl3
results in the formation of 2-benzoylbenzoic acid which is converted into a
cyclic compound in presence of concentrated sulfuric acid to form
anthraquinone. The distillation of anthraquinone over zinc dust gives
anthracene.

In a similar manner, use of alkylbenzene in the above reaction results in the


formation of
2-alkylanthracene.
(b) The condensation of two molecules of benzyl chloride in the presence of
AlCl3 results in the formation of anthracene.

(c) Reaction of benzene with 1,1,2,2-tetrabromoethane in the presence of


AlCl3 gives anthracene.

24.4.3 Chemical Properties of Anthracene


Anthracene undergoes electrophilic substitution, oxidation, reduction,
dimerization, addition, and Diels–Alder reactions. All these reactions occur at 9
and 10 positions of carbon atoms.
The reactions in anthracene generally occur at positions 9 and 10. In this case,
the aromatic sextet is retained in two benzene rings, which stabilizes the system
due to high resonance energy.

Electrophilic substitution reactions


Anthracene undergoes electrophilic substitution like nitration, halogenation, and
Friedel–Crafts acylation (in nonpolar solvent) preferably at C9 as it results in the
formation of a stable carbocation where aromaticity is retained in two benzene
rings. The loss of a proton from this carbocation results in the formation of 9-
substituted anthracene.
However in case of polar solvent, Friedel–Crafts reaction occurs at C1. The
sulfonation occurs at C1 and C2 positions to yield corresponding anthracene
sulfonic acid. In presence of excess of sulfuric acid, disubstituted (1,8- or 2,7-)
sulfonic acids are obtained. These electrophilic substitution reactions can be
summarized as follows:
Other reactions
Other reactions such as oxidation, reduction, dimerization, addition, and Diels–
Alder reactions occur at 9, 10 positions of anthracene. These reactions are
summarized as follows:
24.5 PHENANTHRENE
Phenanthrene is a fused ring polynuclear hydrocarbon obtained from coal tar
distillation. It has molecular formula C14H10 and is isomeric with anthracene.
Phenanthrene is a colourless crystalline substance (m.p. 101°C). Its solution in
benzene shows blue fluorescence. The structure of phenanthrene is determined
from the following reactions sequence:
Phenanthrene on oxidation gives phenanthraquinone (II), which further oxidizes
to give diphenic acid (III). The diphenic acid on treatment with soda lime gives
diphenyl (IV). Thus, the basic skeleton of phenantherene is given by the
structure (V) and the accommodation of remaining hydrogens indicates the
structure to be (I) where three benzene rings are fused in an angular manner.

The carbons in phenethrene are sp2 hybridized but unlike anthracene, the three
benzene rings are fused in angular manner. Similar to anthracene, it is a fourteen
π-electron delocalized system and is aromatic in nature.
The numbering in phenanthrene is carried out as follows:

24.5.1 Preparation
Haworth synthesis: Friedel–Crafts acylation of naphthalene with succinic
anhydride results in the formation of 3-(1-naphthoyl)propanoic acid (I). The
Clemmensen reduction of this acid gives 4-(1-naphthyl)butanoic acid (II) which
undergoes cyclization at 2-position of naphthalene to give a cyclic ketone (III).
The ketone III, on Clemmensen reduction followed by dehydrogenation with
selenium or Pd-C results in the formation of phenanthrene.

Use of 1-alkylnaphthalene in the above Haworth synthesis results in the


formation of
9-alkylphenanthrene. The acylation occurs at position-4 of 1-alkylnaphthalene.

24.5.2 Chemical Properties of Phenanthrene


In a manner similar to anthracene, most of the reactions in phenanthrene occur at
C9 and C10 positions. A few important reactions are summarized as follows:
EXERCISES
1. What are polynuclear hydrocarbons?
2. How you will establish that in naphthalene two benzene rings are fused at
ortho positions.
3. Using Haworth’s synthesis, how will you prepare
(a) Naphthalene
(b) 2-Ethylnaphthalene
(c) 1-Ethylnaphthalene
(d) 2-Methoxynaphthalene (methyl 2-naphthyl ether)
4. Comment on the aromatic behaviour of naphthalene, anthracene, and
phenanthrene.
5. Why polynuclear hydrocarbons have relatively less aromatic character as
compared to benzene?
6. Why is C1–C2 bond length in naphthalene shorter than C2–C3 bond length?
7. Why do electrophilic substitution reactions in naphthalene take place at α-
position?
8. Explain why reaction of naphthalene with conc. H2SO4 at 40°C yields
naphthalene-1-sulfonic acid whereas at 160°C naphthalene-2-sulfonic acid is
the major product?
9. Complete the following reactions:
10. Write down the different steps involved in Haworth synthesis of
(a) Anthracene
(b) 2-Ethylanthracene
(c) 1-Methylphenanthrene
(d) Phenanthrene
(e) 9-Ethylanthracene
11. Carry out the conversion reactions to form the following compounds from
naphthalene:
(a) β-naphthylamine
(b) phthalic anhydride
(c) 1,3-Dinitronaphthalene
(d) 2-Naphthoic acid
(e) 2-(N-phenyl)naphythylamine
(f) 2-propylnaphthalene
(g) 1-Hydroxy-2-naphthaldehyde
SET-IV
Problem 99. Giving mechanism, explain the formation of products in the
following reactions of given pinacols.

Solution. These are pinacol–pinacolone rearrangement reactions. This


rearrangement involves migration of an alkyl, aryl group or hydrogen ( section
15.3.2 and also Problem 98 of Explore More Set-III). In the given reactions, the
migration of a bond occurs during rearrangement, that causes ring expansion or
ring contraction in the product.
The mechanism is as follows:
Problem 100. Compare the stability of the hydrates of acetaldehyde and
trichloroacetaldehyde (chloral).
Solution. The carbonyl compounds form hydrates with water (p. 585). The
stability of hydrates depends on the extent of positive charge on carbonyl carbon
(electrophilicity). More the positive charge on the carbonyl carbon, more is the
stability of its hydrate. The presence of electron withdrawing group (–I effect) in
carbonyl compound will increase the electrophilicity of carbonyl carbon and
they form stable hydrates. Thus, the stability of hydrate of trichloroacetaldehyde
(chloral, CCl3CHO) is more compared to acetaldehyde (CH3CHO) due to the
presence of electronegative chlorine atoms.
Problem 101. Identify the name reaction and give the products formed in the
following reactions:

Solution
(i) Dieckmann condensation (ii) Dieckmann condensation
(Intramolecular Claisen condensation)

(iii) Intramolecular Claisen Schmidt condensation


Note: For details and general mechanism of these name reactions, the reader may refer to the text.
Problem 102. What product will be formed on heating hexanedial in the
presence of a base like sodium ethoxide?
Solution. Hexanedial undergoes intramolecular Aldol condensation in the
presence of a base. The base abstracts one of the α-hydrogen to form a carbanion
that attacks the carbonyl carbon of other aldehyde group present within the
molecule to form a cyclic aldol. Heating causes the dehydration of aldol to form
an unsaturated cyclic aldehyde.

Problem 103. What products will be formed on heating a mixture of 4-


methylbenzaldehyde and
4-chlorobenzaldehyde in alcoholic solution containing potassium cyanide.
Solution. This is ‘Benzoin condensation’ (p. 643) of a mixture of aromatic
aldehydes and results in the formation of two products depending upon the
attack of CN– on carbonyl carbon of
4-chlorobenzaldehyde (path 1) or 4-methylbenzaldehyde (path 2) to give the
corresponding benzoin.
Problem 104. What product(s) will be obtained on heating the following
compounds in alkaline solution:

Solution. The above compounds are examples of dicarbonyl compounds, that is,
two carbonyl groups are present in the same molecule. These dicarbonyl
compounds do not contain an α-hydrogen. In presence of base, these dicarbonyl
compounds undergo an intramolecular Cannizzaro’s reaction.
The mechanism is similar to what has been discussed in Cannizarro’s reaction
(refer text). In intramolecular reaction, the hydride transfer occurs to second
carbonyl group present within the same molecule to form hydroxy carboxylic
acids.

The mechanism for intramolecular Cannizzaro’s reaction is as follows:


Similarly, compound [B] undergoes intramolecular Cannizzaro’s reaction as
follows:

Identifying the structures of compounds in a series of reactions (Problems


105–110)
Examine how! A little change in the information (hint) makes a lot of difference to the
structure in the following six problems [105–110] (Carefully watch out for the
similarity of information and the not so appreciable variation.)
Problem 105. An aliphatic open chain compound A (C6H12O) reacts with
C6H5NHNH2 but does not reduce Tollens reagent. On catalytic hydrogenation,
compound A gives B (C6H14O). This compound B, when passed over heated
Al2O3 gives C (C6H12).
Compound C on reductive ozonolysis gives two compounds D and E. Reaction
product D gives positive Fehling’s solution test and negative iodoform test while
E gives negative Fehling’s solution test and positive iodoform test.
Identify A, B, C, D, and E and explain the reactions.
Solution. Compound A (C6H12O) is a ketone since it reacts with
phenylhydrazine and does not respond to Tollens reagent.
(Recall: Both aldehydes and ketones react with ammonia derivatives but ketones
do not reduce Tollens reagent.)
Compound B (C6H14O) is a secondary alcohol as it is obtained by the catalytic
hydrogenation (reduction) of ketone A.
(Recall: Aldehydes yield primary alcohols while ketones yield secondary
alcohols on reduction.)
Compound C (C6H12) is an alkene. Alcohols when passed over heated alumina
result in dehydration to yield alkene. The molecular formula of hydrocarbon C
also indicates it to be an alkene.
(Recall: A more substituted alkene is formed in accordance with Saytzeff’s rule.)

Structure determination. The ozonolysis products of alkene C are carbonyl


compounds D and E. These carbonyl compounds give a direct indication that C
is an alkene, from which, tracing backwards (retroanalysis), the structures for B
and A can be determined.
Compound D gives a positive Fehling’s test and negative iodoform test. Thus,
compound D is an aldehyde other than acetaldehyde.
(Recall: Acetaldehyde is the only aldehyde which gives positive iodoform test.)
Compound E gives negative Fehling’s test but a positive iodoform test. Thus,
compound E is a methyl ketone.
(Recall: Ketones do not respond to Fehling’s test and all methyl ketones
(CH3CO- type) give positive iodoform test.)
Structure of alkene C[C6H12]. There are three possibilities that help decide for
the structure of alkene (having six carbon skeleton) based on the ozonolysis
products. Let us consider them one by one.
Possibility 1 (ruled out): Compound D can be formaldehyde and the
corresponding methyl ketone E is then pentan-2-one. These two carbonyl
compounds correspond to the alkene, 2-methylpent-1-ene, which is a
dehydration product of a primary alcohol, 2-methylpentan-1-ol. As it has already
been established that B is a secondary alcohol, this possibility is ruled out.
Possibility 2 (ruled out): Compound D can be acetaldehyde and the
corresponding methyl ketone E should then be butanone. However, this is ruled
out since compound D cannot be acetaldehyde as mentioned earlier.
Possibility 3 (actual): The compound D can be propanal and corresponding
methyl ketone E will be propanone (acetone). This corresponds to alkene 2-
methylpent-2-ene (C), which in turn is obtained from a secondary alcohol 2-
methylpentan-3-ol (B). This alcohol is obtained from reduction of ketone,
namely 2-methylpentan-3-one (A).
The overall reactions can be schematically summarized as follows:
Take a look at the next problem (problem 106) and compare it with the previous problem. The two
problems look alike except for the only difference, which is indicated through shaded area.
Problem 106. An aliphatic open chain compound A (C6H12O) reacts with
C6H5NHNH2 but does not reduce Tollens reagent. On catalytic hydrogenation,
compound A gives B (C6H14O). Compound B, when passed over heated Al2O3
gives C (C6H12).
Compound C on reductive ozonolysis gives two compounds D and E. The
product D gives positive Fehling’s solution test and positive iodoform test
whereas E gives negative Fehling’s solution test and positive iodoform test.
Identify A, B, C, D, and E and explain the reactions.
Solution. As discussed in problem 105, compound A is a ketone (C6H12O), B is
a secondary alcohol (C6H14O), and C is an alkene (C6H12). But the ozonolysis
products D and E have different characteristics. We now determine the structures
of A, B, and C through D and E.
Compound D gives positive Fehling’s test and positive iodoform test, which
directly confirms the compound D to be ethanal.
Compound E gives negative Fehling’s test and positive iodoform test, which
indicates E to be a methyl ketone (CH3CO- type).
Structure of alkene C. Since compound D is ethanal, the corresponding methyl
ketone E should be butanone (remember alkene has a six carbon skeleton). Thus,
acetaldehyde (D) and butanone (E) correspond to the alkene 3-methylpent-2-ene
(C). The alkene, in turn, is obtained from secondary alcohol 3-methylpentan-2-ol
(B), which is obtained from ketone 3-methylpentan-2-one (A).
The overall reaction can be summarized as:
[Note: The readers are advised to solve the following four problems (Problems 107–110) on their own;
compare with previous problem; and find out the reason for the change in the structures. Consider carefully
the Saytzeff’s rule.]
Problem 107. An aliphatic open chain compound A (C6H12O) reacts with
C6H5NHNH2 but does not reduce Tollens reagent. On catalytic hydrogenation
compound A gives B (C6H14O). Compound B, when passed over heated Al2O3
gives C (C6H12).
Compound C on reductive ozonolysis gives two compounds D and E. The
product D gives positive Fehling’s solution test and positive iodoform test while
E gives positive Fehling’s solution test and negative iodoform test. E on
Clemmensen reduction gives n-butane.
Identify A, B, C, D, and E and explain the reactions.
Answer. A (Hexan-2-one); B (Hexan-2-ol); C (Hex-2-ene); D (Acetaldehyde); E
(Butanal)
Problem 108. An aliphatic open chain compound A (C6H12O) reacts with
C6H5NHNH2 but does not reduce Tollens reagent. On catalytic hydrogenation,
compound A gives B (C6H14O). Compound B, when passed over heated Al2O3
gives C, C6H12.
Compound C on reductive ozonolysis gives two compounds D and E. While D
gives
positive Fehling’s solution test and positive iodoform test, E gives positive
Fehling’s solution test and negative iodoform test. However, E on Clemmensen
reduction gives 2-methylpropane (isobutane).
Identify A, B, C, D, and E and explain the reactions.
Answer. A (4-Methylpentan-2-one); B (4-Methylpentan-2-ol); C (4-Methylpent-
2-ene); D (Ethanal); E (2-Methylpropanal)
Problem 109. An aliphatic open chain compound A (C6H12O) reacts with
C6H5NHNH2 but does not reduce Tollens reagent. On catalytic hydrogenation,
compound A gives B (C6H14O). Compound B, when passed over heated Al2O3
gives C (C6H12).
Compound C on reductive ozonolysis gives two compounds, namely
formaldehyde and D, which gives positive Fehling’s solution test but negative
iodoform test. Clemmensen reduction of D gives symmetrical alkane. Identify A,
B, C, and D and explain the reactions.
Answer. A (3,3-Dimethyllbutan-2-one); B (3,3-Dimethylbutan-2-ol); C (3,3-
Dimethylbut-1-ene);
D (2,2-Dimethylpropanal).
Problem 110. An aliphatic open chain compound A (C6H12O) reacts with
C6H5NHNH2 but does not reduce Tollens reagent. On catalytic hydrogenation,
compound A gives B (C6H14O). Compound B, when passed over heated Al2O3
gives C (C6H12).
Compound C on reductive ozonolysis gives two molecules of compounds D.
This compound D gives positive Fehling’s solution test and negative iodoform
test.
Identify A, B, C, and D and explain the reactions.
Answer. A (Hexan-3-one); B (Hexan-3-ol); C (Hex-3-ene); D (Propanal).
Problems on Grignard reagent (111–114)
Problem 111. The reaction of ketone with Grignard reagent followed by
hydrolysis results in the formation of 3° alcohols. Give a suitable explanation as
to why are acidic conditions avoided during hydrolysis. Take the synthesis of 2-
methylpropan-2-ol as an example.
Solution. The synthesis of 2-methylpropan-2-ol can be carried out easily by
reaction of propanone (acetone) and methylmagnesium bromide. In the first step,
Grignard reagent forms the C–C bond with the carbonyl carbon of propanone.
In the second step, the product thus formed is subjected to hydrolysis. In acidic
conditions, the 3° alcohol formed may undergo an elimination reaction
(dehydration) resulting in the formation of an alkene, namely methylpropene. To
avoid the formation of alkene, hydrolysis is carried out under milder conditions
by using ammonium chloride.
Problem 112. Grignard reagent easily reacts with carbonyl compounds but does
not react with carbon–carbon double bond. Why?
Solution: The carbonyl compounds are highly polar in nature and the alkyl
group of the Grignard reagent readily attacks the electrophilic carbon of the
carbonyl group. The alkyl group in a Grignard reagent has carbanion character
and behaves as a nucleophile in its reactions.
As such, the polarity of C=C is negligible compared to C=O and thus, the
olefinic carbon is not attacked readily by Grignard reagent.

[Note: However, if polarity of C=C is enhanced considerably, it may undergo attack by Grignard reagent.
(refer problem 113)]
Problem 113. The reaction of pent-3-en-2-one with methylmagnesium iodide
(molar ratio 1:1) followed by hydrolysis results in the formation of two products
—a 3°alcohol (as expected) and a minor amount of saturated ketone
(unexpectedly). Write the reactions involved and explain the formation of
saturated ketone.
Solution. The addition of Grignard reagent at the carbonyl group of the
unsaturated ketone followed by hydrolysis results in the formation of unsaturated
alcohol (shown as [I] in the reaction). This reaction is similar to the reaction of
saturated ketone with Grignard reagent.
Although Grignard reagent does not attack C=C (as explained in previous
problem) but in pent-3-en-2-one, the carbon–carbon double bond is in
conjugation with the carbonyl group. Due to this conjugation, the delocalization
of electrons takes place that enhances the polarity of olefinic bond. Thus,
olefinic carbon with enhanced electrophilic nature is attacked readily by
Grignard reagent (Section 17.4) to form saturated ketone (shown as [II] in the
reaction).

To conclude, methylmagnesium iodide reacts with pent-3-en-2-one to give an


unsaturated alcohol (by reaction at carbonyl group) and a saturated ketone (by
reaction at olefinic carbon).

In general, the reaction of Grignard reagent with α,β-


unsaturated aldehyde occurs at carbonyl group (1,2-addition) to give
unsaturated alcohols as the final product.
However, in case of α,β-unsaturated ketone the reaction of Grignard
reagent also occurs at olefinic carbon (1,4-addition) to give β-
alkylated ketone (product II). The β-alkyl ketone is obtained as minor
product along with the usual 1,2-addition product, that is, unsaturated
alcohol (product I).
In case of bulkier and sterically hindered α,β-unsaturated ketones, the
reaction of Grignard reagent occurs preferably at olefinic carbon and
in such cases the β-alkylketone is the major product.
Problem 114. What products will be expected if the reaction of pent-3-en-2-one
is carried out with excess of methylmagnesium iodide.
Solution. The reactions will be similar to as discussed in problem 113. However
in excess of Grignard reagent, the saturated ketone [structure II] formed will
further undergo reaction
with Grignard reagent to form a 3° alcohol as the final product, that is, 2,4-
dimethylpentan-2-ol [structure III].
Grignard reagent does not attack the product [I], that is, unsaturated alcohol 2-
methylpent-3-en-2-ol further. The isolated carbon–carbon double bond (absence
of conjugation) does not react with Grignard reagent (refer problem 112).

Problem 115. In the esterification process of carboxylic acids, mineral acid is


used to catalyze the reactions. However, the use of higher concentration of
mineral acid retards the rate of esterification. Explain.
Solution. The esterification occurs when reaction of carboxylic acid and an
alcohol occurs in the presence of mineral acid. The mineral acid protonates the
oxygen of carboxylic group and enhances the electrophilicity of carboxylic
carbon. As a result, the attack of nucleophile (alcohol) occurs readily on
carboxylic carbon to form an ester (for mechanism, refer text).

However at higher concentration of mineral acid, the protonation of alcohol


also occurs and the alcohol looses its nucleophilicity. Thus, the attack of alcohol
(in oxonium form) does not take place readily on carboxylic acid.

Problem 116. What will be the increasing order of rate of esterification of a


carboxylic acid with following alcohols:
(a) Ethanol, (b) 2-Methylpropan-2-ol, (c) Propan-2-ol
Solution. The rate of esterification of different alcohols with a given carboxylic
acid decreases with an increase in steric hindrance. This means that the bulkier
the groups attached to –OH, slower will be the esterification.
Thus, the increasing order of rate of esterification is

Problem 117. Explain, why


(a) In acylation reaction, benzoyl chloride is less reactive than acetyl chloride?
(b) Hydrolysis of benzoyl chloride is slow as compared to that of acetyl
chloride?
Solution. (a) The acylation reaction involves the attack of a nucleophile on
carbonyl carbon of acid chloride. Higher the electrophilicity of carbonyl carbon,
faster will be the acylation reaction.
In case of benzoyl chloride, the π-electrons of benzene ring participate in
delocalization with π-electrons of carbonyl group. This delocalization of π-
electrons reduces the electrophilic character of carbon and thus decreases the
reactivity of benzoyl chloride compared to acetyl chloride. In acetyl chloride, no
such delocalization takes place.
(b) For the same reason, hydrolysis of benzoyl chloride is slow compared to
that of acetyl chloride.

Problem 118. The cis-cinnamic acid has a relatively high acidic strength
compared to trans-cinnamic acid. Explain.
Solution. The cis-cinnamic acid has phenyl ring and carboxylic group on the
same side of the double bond. The steric hindrance from phenyl group makes
carboxylic group to be in a plane different from the double bond. As a result, no
delocalization of electrons takes place between the double bond and carboxylic
group, which enhances the acidic strength.
Problem 119. Among succinic acid (butanedioic acid) and malonic acid
(propanedioic acid), which will undergo thermal decarboxylation most readily?
Solution. The carboxylic acids having electron withdrawing group (–I effect) at
α-position undergo decarboxylation most readily. Thus, among the given two
dicarboxylic acids, malonic acid will undergo decarboxylation more easily
compared to succinic acid.
In malonic acid, carboxylic group (having –I effect) is at α-position to the other
carboxylic acid. Succinic acid undergoes dehydration to form succinic anhydride
(refer text) rather than decarboxylation.

Problem 120. Which of the carboxylic acid of the following pairs will undergo
decarboxylation readily?
(i) O2NCH2COOH O2NCH2CH2COOH
(ii) CH3COCH2COOH CH3CH2CH2COOH
(iii) CH3CH2COOH CH3CH(Cl)COOH
(iv) CCl3COOH CH2ClCOOH
Solution. As mentioned in the previous problem, carboxylic group having
electron withdrawing group at α-position undergoes decarboxylation more
readily. Thus,
(i) O2NCH2COOH will undergo decarboxylation more readily (–NO2 at α-
position) as compared to O2NCH2CH2COOH (–NO2 at β-position).
(ii) CH3COCH2COOH will undergo decarboxylation more readily (CH3CO
at α-position) as compared to CH3CH2CH2COOH (no electron
withdrawing group is present).
(iii) CH3CH(Cl)COOH will undergo decarboxylation more readily (Cl at α-
position) as compared to CH3CH2COOH (no electron withdrawing group
is present).
(iv) Both the acids have –Cl atom at α-position but in case of CCl3COOH, three
–Cl atoms are present at α-position (enhanced –I effect) as compared to
CH2ClCOOH (only one –Cl at α-position, –I effect is less). Thus,
CCl3COOH will undergo decarboxylation more readily.
Problem 121. Arrange the following diazonium ions in increasing order of
reactivity towards coupling reactions.

Solution. In the coupling reactions, diazonium ion behaves as an electrophile (p.


836). The presence of electron withdrawing groups (like nitro groups) in the
benzene ring increases the electrophilic character of diazonium ions. More the
number of nitro groups present on benzene ring, more is the electrophilic
character of diazonium salt and higher is the reactivity towards coupling
reaction. Thus, the order of reactivity is as follows:
[III] < [I] < [II] < [IV]
Problem 122. What will be the order of reactivity of the following
arenediazonium ions towards coupling reactions?

Solution. The presence of electron releasing group decreases the electrophilic


character of diazonium ions whereas electron withdrawing group increases the
electrophilic character. The strength of electron withdrawing groups follows the
order –NO2 > –SO3H. The –CH3 is an electron releasing group. Thus, the
increasing order of reactivity of arenediazonium ions is as follows:
[III] < [II] < [I]
Problems based on distinguishing the organic compounds (123–125)
Problem 123. How the following set of organic compounds (a–c) can be
distinguished?

Solution
(a) Add Br2 water; the alkene [ii] and alkyne [iii] will decolorize it but alkane
[i] will not decolorize the bromine water. Further the alkyne [iii] (being a
terminal alkyne) gives white precipitate with Tollens reagent, Ag(NH3)2+,
or red precipitate with Cu(NH3)2+ .
(b) These three can be distinguished with AgNO3. 1-Bromobut-2-ene [i] gives
the precipitate of AgBr immediately.1-Bromobutane [ii] gives precipitate on
heating while 1-bromobut-1-ene [iii] does not react.
Remarks: Reason lies with the position of bromine in these compounds that is in
[i] it is allylic, in [ii] it is at primary carbon and [iii] it is vinylic which is
least reactive.)
(c) Acid [i] would give effervescence with NaHCO3 solution (due to
liberation of CO2) and will dissolve in it. Aldehyde [ii] and ketone [iii] both
will give the positive test (yellow/orange ppt.) with 2,4-
dinitrophenylhydrazine (DNP reagent). In addition to this only aldehyde [ii]
gives silver mirror with Tollens reagent and red precipitate with Fehling’s
solution.
Problem 124. How the following set of organic compounds (a–e) can be
distinguished?
Solution
(a) Only ether [ii] is soluble in sulfuric acid
Remarks: it results in the formation of oxonium salts
(b) Phenol [ii] will give violet colour with FeCl3 (a characteristic reaction of
phenols).
Alternatively,
Only Phenol [ii] would dissolve in sodium hydroxide solution.
Remarks: phenol forms sodium phenoxide while alcohol [i] does not react with
sodium hydroxide.
(c) On adding AgNO3 solution the ethanoyl chloride (acetyl chloride; [i]) will
give white precipitate of AgCl immediately.
Remarks: acetyl chloride will liberate HCl with water, which will form
precipitate with AgNO3.
(d) Only Aniline [i] decolorizes the bromine water (because of the formation
of tribromoaniline) Alternatively,
On addition of NaNO2 in HCl at 0°C; Cyclohexylamine [ii] will give
effervescences as the N2 will bubble off from the solution.
Remarks:
• Both [i] and [ii] will give carbylamine test as these are 1° amines
• Aromatic amine forms diazonium salt while aliphatic amine forms mainly
alcohol with the evolution of N2 from the solution.
(e) cis-Cyclopentane-1,2-dicarboxylic acid [ii] on heating will undergo
dehydration to form corresponding anhydride.
Remarks: carboxylic groups are on the same side so the dehydration will occur
readily which is not possible in case of its trans isomer [i].

Problem 125. How the following set of organic compounds [a and b] can be
distinguished?

Solution
(a) Acid [i] would give effervescences with NaHCO3 solution and will
dissolve in it. Aldehyde [ii] and ketone [iii] both will give the positive test
(yellow/orange ppt.) with
2,4-dinitrophenylhydrazine (DNP reagent). But in addition to this only
aldehyde [ii] will give silver mirror with Tollens reagent and red precipitate
with Fehling’s solution.
Remarks:
• Formic acid (methanoic acid) also gives the silver mirror test with Tollens
reagent. So first acid has to be distinguished as mentioned above to
avoid confusion between [i] and [ii].
• Both aldehyde [ii] and ketone [iii] will also give the iodoform test.
Further to add these tests not only confirm the type of functional group but also
confirms the compound, that is
• Aldehyde giving iodofrm test must be ethanal (acetaldehyde)
• Acid responding to Tollens test must be methanoic acid (formic acid).
(b) Amide [i] and urea [iii] liberate ammonia on heating with sodium
hydroxide solution. However only urea will give biuret test.
Compound [ii] is an amino acid and will give violet colour with ninhydrin
(ninhydrin test).
Strategies in organic synthesis (126–137)
Problem 126. How will you carry out the conversion of benzene to p-
methylbenzaldehyde?
Solution

Problem 127. Carry out the following conversions:


(a) Benzene to m-nitrophenol
(b) Benzonitrile to 1,3-dihydroxybenzene (resorcinol)
Solution. In both the conversions, –SO3H group is used to introduce a phenolic
group. This is done because benzenesulfonic acid on fusion with NaOH results
in the formation of phenol.
(a) Benzene to m-nitrophenol

(b) Benzonitrile to 1,3-dihydroxybenzene (Resorcinol)

Problem 128. Carry out the following conversions:


(a) o-Nitroaniline to o-chloronitrobenzene
(b) o-chloronitrobenzene to o-bromochlorobenzene
(c) Benzene to m-bromophenol
(d) o-Nitroaniline to o-dinitrobenzene
Solution. These conversions involve the use of diazonium salt for introducing
other functional groups. This is done as follows:
(a) o-Nitroaniline to o-chloronitrobenzene

(b) o-chloronitrobenzene to o-bromochlorobenzene

(c) Benzene to m-bromophenol

(d) o-Nitroaniline to o-dinitrobenzene

Problems based on selectivity of reagents in the organic transformation


(129–134)
Problem 129. What product(s) will be obtained for the following reactions?
Comment on the formation of each product.
Solution.
[A]: CH3CH=CH–COOH
Ammonical silver nitrate (Tollens reagent) is mild oxidizing agent. It does not
affect the double bond, however it oxidizes the –CHO to –COOH group.
[B]: CH3CH=CH–CH2OH
LiAlH4 reduces only –CHO group to –CH2OH goup without affecting the
double bond.

[C]:
Gilman reagent (lithium dialkylcuprate) is used to introduce the alkyl group
(here it is ethyl group) at β-position of α,β-unsaturated carbonyl
compounds. It does not affect the –CHO group (unlike the other
organometallic compounds such as Grignard reagent and Organolithiums
refer Chapter 25).
Problem 130. Carry out the following conversion only in two steps and justify
the each step involved in it.

Solution. The reaction sequence and the comments on each step are as follows:
The Gilman reagent is prepared from vinyl bromide, CH2=CH–Br (refer for
general preparation; (Chapter 25).
Problem 131. What product(s) will be obtained by the reaction of p-
acetylbenzoic acid with
(1) NaBH4 (2) LiAlH4 and (3) with SOCl2 followed by reaction with
(CH3)2CuLi? Give the comments associated with the specific use of these
reagents.
Solution. The reaction of p-acetylbenzoic acid with the given reagents along
with comments is as follows:
Problem 132. How will you carry out the following transformations starting
from
3-oxocyclohexanecarbaldehyde[X]?
Solution. Aldehydes are more reactive as compared to ketones. In a compound
containing aldehyde and keto group, the reaction occur at –CHO preferentially.
In these transformations selective protection of aldehydic group is carried out
through acetal formation. The reaction of aldehyde with ethylene glycol is
carried out with catalytic amount of acid. However use of excess acid results in
deprotection. The reactions are then carried out selectively at keto group and in
the final step deprotection of aldehydic group is carried through acidic
hydrolysis.
The general reaction of selective protection of aldehyde group is as follows:

The first step in all these transformation is protection of aldehyde group using
ethylene glycol. Further these reactions are shown from protected 3-
oxocyclohexanecarbaldehyde.
Transformation to hydroxy compounds [A] and [B]
The IUPAC names of the products formed in the above sequences of reactions
are as follows:
Cyclohexanecarbaldehyde [C],
Cyclohexanecarboxylic acid [D],
Bromocyclohexane [E],
Cyclohexanecarbonyl chloride [F],
Cyclohexylethanone [G, common name:cyclohexylmethylketone],
Methylcyclohexane [H]
Problem 133. How will you synthesize the following compound using Friedel–
Crafts reaction?

Solution. The immediate solution, which comes to the mind involving Friedel–
Crafts reaction is benzoylation of nitrobenzene (because –NO2 group is electron
withdrawing group and meta-directing) using benzoyl chloride in the presence of
AlCl3. But, this reaction cannot be carried out since Friedel–Crafts acylation
does not occur in case of aromatic systems containing electron withdrawing
group (Section 10.2.4). In fact, nitrobenzene is generally used as a solvent for
Friedel–Crafts acylation reactions.

However, the synthesis can be carried out using m-nitrobenzoylchloride as


acylating agent and benzene as substrate, in the presence of AlCl3. The steps of
synthesis are as follows:
Step 1. Synthesis of m-nitrobenzoylchloride

Step 2. Friedel–Crafts acylation of benzene using m-nitrobenzoylchloride.


Problem 134. Which of the following two paths are favoured for preparation of
methyl ester of
2,6-difluorobenzoic acid?

Solution. In path I, esterification involves the attack of the nucleophile


(methanol) on carboxylic carbon. However, the carboxylic carbon is sterically
hindered due to the presence of fluorine at ortho positions.
In path II, the acid anion acts as a nucleophile and attacks the electrophilic
carbon of iodomethane to produce ester and no steric hindrance is experienced in
this case.
Thus, path II is favoured to form methyl ester of aromatic carboxylic acid.
Problems based on protecting/blocking groups (135–137)

Problem 135. Carry out the following conversions:


(a) Nitrobenzene to 3,4-dinitroaniline
(b) 4-Methylaniline to m-nitrotoluene
(c) m-Xylene to 3,5-dimethylchlorobenzene
Solution. These conversions involve the use of protecting group. In general,
protection of –NH2 group is carried out by acetylation (p. 831) which converts it
to –NHCOCH3 group. The process of deprotection (to get back –NH2 group) is
carried out in aqueous acidic medium after carrying out the substitution at
required position(s).
(a) Nitrobenzene to 3,4-dinitroaniline
(b) 4-Methylaniline to m-nitrotoluene

(c) m-Xylene to 3,5-dimethylchlorobenzene


Problem 136. Carry out the following conversions:
(a) Toluene to o-nitrotoluene
(b) Chlorobenzene to 2,6-dinitroaniline
Solution. In these conversions, substitution is carried out exclusively at ortho
positions by blocking the para position with sulfonic group. Sulfonation is a
reversible reaction and thus, the blocking group (–SO3H) is removed readily in
aqueous acidic medium.
(a) Toluene to o-nitrotoluene
(b) Chlorobenzene to 2,6-dinitroaniline

Problem 137. How will you carry out the conversion of nitrobenzene to o-
nitroaniline?
Solution. This conversion involves the use of a protecting group as well as a
blocking group simultaneously in the following manner:

Problems based on separation of organic compounds from a mixture (138–


141)
A word about separations. The method of separation of organic compounds generally
involves two methodologies, discussed as follows:
1. Transforming one (or more) compound into water-soluble salt by making use of an
acid or a base, which is followed by extraction with ether.
Ether (commonly diethyl ether is used) is immiscible with water and forms the upper
layer (less denser than water). Most of the organic compounds are ether-soluble
whereas the salts of organic compounds are water-soluble. Basic compounds are
transformed to their water-soluble salts (like amines to their hydrochlorides) when
treated with mineral acids. Similarly acidic compounds are transformed to water-
soluble salts (like acids, phenols to their sodium salts) when treated with alkalis.
Thus, in order to separate a mixture of organic compounds, the acid or base is added to
transform one of the compounds to a water-soluble one by converting it into its
salt. Extraction with ether using separating funnel (see figure given below), then,
results in the separation of compound(s) in aqueous phase and organic phase (ether
layer). Pure compound from aqueous layer is recovered by neutralizing it with an
acid or base. Solid product obtained is separated by filtration. A pure compound
from ether layer is recovered by distilling off ether.
2. By making use of chemical properties of a functional group and transforming it into
its suitable derivative that can be separated from rest of the components in a
mixture by extraction.
One of the important aspects of this method is that the original compound should easily
be recovered from the derivative. For example, Hinsberg method for the separation
of amines and bisulfite addition product for the carbonyl compounds.
These processes of separation may be used repeatedly depending upon the number and
nature of the components.

Problem 138. How can the following mixture of organic compounds be


separated by extraction and acid-base reactions?

Solution. The given mixture contains benzoic acid, aniline and biphenyl. These
three can be separated by the following procedure:
Problem 139. How can the mixture of following organic compounds be
separated by extraction and acid-base method?

Solution. The given set of mixture of phenol, benzyl alcohol, and benzoic acid
can be separated by scheme as follows:
Problem 140. How can the mixture of following organic compounds be
separated? The separation by extraction procedure may involve acid-base
method and/or chemical transformation.

Solution. Phenol and benzoic acid are acidic components of the mixture and are
separated from basic components like aniline and N, N-dimethylaniline using
acid-base reactions. The separation of mixture of amines is carried out using
chemical transformation that is Hinsberg method. Benzene is a neutral
compound in this case. The separation scheme is as follows:
Problem 141. How can the mixture of pentanal and pentan-3-one be separated in
pure form?
Solution. Both pentanal and pentan-3-one are carbonyl compounds. So they are
separated by the chemical method indicated as follows:
Chapter25

Review
Part I: Organometallic Compounds
Part II: Oxidation–Reduction
Reactions

AIM
To offer consolidated information about above mentioned topics which have been
discussed under different functional groups interspersed throughout the text. Also
provides additional information (if any) which could not be accomodated earlier to
avoid loss of focus of the targetted topic.
.

SCOPE
To correlate and condense the scattered information under one roof and help
comprehend the vast field of organic chemistry with a broader outlook yet an integrated
approach.
.

HOPE
The unified and organized approach throughout the review will help the reader to
understand the topics in a simple and easily digestable manner.
LOOKING GLASS The review is thoroughly and regularly connected with the
text in the book by corresponding section number(s)[denoted as sec], page
number(s) [denoted as pp.] and solved problems from explore more
[Mentioned as EM followed by problem number] as and where they appear
throughout the book.
Further, key points and additional information (wherever needed) is also
incorporated along with reactions. Appearance of symbol star (↔) in the review
indicates the reactions and informations not discussed earlier in the text.

PART I: ORGANOMETALLIC
COMPOUNDS
25.1 INTRODUCTION
The high reactivity of organometallic compounds makes them a versatile tool to
synthesize a variety of functional groups. In Part I of this review chapter, a
summary of the general characteristics, preparations, reactions, and limitations
related to these compounds is given. Since most of the reactions have already
been discussed in the text in respective chapters, only key information is referred
to.
The organometallic compounds, prepared from organic halides, that are
discussed in this chapter include (in the same order:)
(1) Organomagnesium halides—Grignard Reagent
(2) Organolithium Compounds
(3) Lithium dialkylcuprates–Gilman Reagent
General characteristics (Section 11.4)
The organometallic compounds contain carbon-metal bond. Due to high
electropositive character of metals, the carbon bears a negative charge, that is,
the carbanion character develops at the organic moiety. Therefore, higher the
electropositive character of metal, higher is the ionic character of the carbon–
metal bond in organometallic compound.

The electropositive character of different metals follows the order: Li > Mg > Cu.

The preparation and reactions of organometallic compounds are carried out in


suitable inert solvents, like ethers, under anhydrous conditions. In general,
the ethers used for the purpose are diethyl ether, dimethoxyethane (DME), and
cyclic ethers such as Tetrahydrofuran (THF) and dioxane.
The ethers solvate the organometallic compounds easily, which means that they
remain in dissolved form and generally exist as etherates.

Preparation. The preparation of organometallic compounds involves the


oxidative insertion
of metal into carbon–halogen bond. In other words, this involves a change in the
oxidation
state of metal. For example, the reaction of magnesium metal with haloalkanes
to form alkyl magnesium halide (Grignard reagent) involves a change in the
oxidation state of magnesium from Mg (0) to Mg (II). That is, the metal is
oxidized and in turn reduction occurs at the carbon of haloalkane.
In organometallic compounds, the alkyl or aryl group bonded to metal behaves as a
carbanion. The reactions of organometallic compounds with different functional
groups can be classified into the following categories:
• Electrophilic substitution reactions
• Addition reactions
25.2 ORGANOMAGNESIUM HALIDES—
GRIGNARD REAGENT
General preparation

Similarly,

The alkyl or aryl group in Grignard regent behaves as a carbanion due to


electropositive character of magnesium metal. In general, the Grignard reagent
can be represented as .

• Ar–Cl and CH2=CH–Cl (vinyl chloride) generally do not react readily with
magnesium. However CH2=CH–Br, Ar–I and Ar–Br react with magnesium under
normal conditions to form Grignard reagent.
• In case of Dihaloarenes, if both chlorine and bromine are present, the Grignard
reagent is formed by the replacement of bromine (refer Section 12.3.3 ).
• The preparation of Grignard reagent from tertiary haloalkanes is frequently
accompanied by dehydrohalogenation as the side reaction, that results in the
formation of alkene as a side product (refer EM problem 75).

25.2.1 Reactions of Grignard Reagent


In all the reactions discussed subsequently, the alkylmagnesium halide and
arylmagnesium halide are represented by RMgX only.
(1) With an active source of proton (formation of hydrocarbons)
General reaction
The above reactions are basically electrophilic substitution reactions as the
electrophilic part of Grignard reagent (+MgX) is replaced by H+, an electrophile
from active source of proton.
(2) With terminal alkynes (formation of acetylenic Grignard reagent)
General reaction

(3) With carbonyl compounds (formation of 1°, 2°, and 3° alcohols)


General reaction:
This is an example of addition reaction of Grignard reagent to carbonyl
compounds.
(4) With epoxides (formation of alcohols)
General reaction

(5) With esters (formation of 2° and 3° alcohols (as final product)


General reaction

(6) With acid chlorides (formation of 3° alcohols (as final proudct)


General reaction
(7) With nitriles (formation of aldehydes and ketones)
General reaction

(8) With carbon dioxide (formation of carboxylic acid)


General reaction

(9) With alkyl chloroformates (formation of esters)


General reaction
(10) With cyanogen chloride (formation of alkanenitrile)
General reaction

(11) With halogenated compounds (formation of higher analogues of


ethers and alkenes)
General reaction

(12) With carbon disulfide and sulfur dioxide (formation of dithiocarboxylic


acid, dithio esters, and sulfinic acids)
General reaction

(13) With sulfur (formation of thiols and thio ethers)


General reaction
(14) With isonitriles (formation of aldehydes and amines (as by product)
General reaction

(15) With metal halides (formation of other organometallic compounds)


General reaction

25.2.2 Limitations
The high reactivity of Grignard reagent also poses some limitations in the
preparations of organomagnesium compounds. The organic halide used for the
preparation of Grignard reagent should be selected carefully because
(a) Grignard reagent cannot be prepared from an organic halide having an
acidic/active hydrogen containing functional groups such as –OH, –SH, –
NH2, –COOH, –SO3H. In the preparation of Grignard reagent from such
functional group containing halides, the Grignard reagent formed
decomposes immediately under reaction conditions only (refer reaction 1 of
this section).
(b) Grignard reagent cannot be prepared from organic compounds containing
groups such as carbonyl, epoxy, ester, nitrile, and acyl chloride as Grignard
reagent is reactive towards these functional groups (To get a good
understanding, the reader may refer to all the reactions of Grignard reagents
discussed).
25.3 ORGANOLITHIUMS
General preparation
Organolithium compounds are prepared by the reaction of lithium with organic
halides using dry ether, THF, or hexane as a solvent.

Lithium and halogen can be exchanged in the reaction to form new


organolithium compounds.

The most commonly used organolithiums are methyllithium, butyllithium, and


phenyllithium where alkyl or phenyl groups behave as carbanions as –CH3Li+ ,
–C H Li+ and –PhLi+.
4 9
Unlike magnesium, lithium being more electropositive reacts with CH2=CHCl (vinyl
chloride) and ArCl readily to form vinyl and aryllithiums.

25.3.1 Reactions of Organolithiums


The reactivity of organolithium compounds is similar to that of Grignard reagent
towards functional organic compounds. Its reactions are similar to Grignard
reagent as discussed in the previous section. The reactions of organolithiums
with various classes of organic compounds are generally carried out in inert
solvents like dry ether and THF and have been summarized, by taking RLi as a
representation for alkyl and aryllithiums, as follows:
(1) With active source of proton (formation of hydrocarbons)
General reaction
(2) With carbonyl compounds, epoxides, esters, and acid chlorides
(formation of alcohol)
General reaction

(3) With nitriles, isonitriles, and carboxylates (formation of aldehydes and


ketones)
General reaction
(4) With carbon dioxide, chloroformate esters, and carbondisulfide
(formation of corresponding carboxylic acids and esters)
General reaction: These reactions are also similar to the reactions of Grignard
reagent.
25.4 LITHIUM DIALKYLCUPRATES—GILMAN
REAGENT
General preparation of Gilman reagent (Section 11.4)
Gilman reagent is prepared from organolithium compounds by reaction with
copper (I) iodide and using dry ether as a solvent. The reactions involved in the
process are as follows:

25.4.1 Reactions of Lithium dialkylcuprates (Gilman


Reagent)

Lithium dialkylcuprates are less reactive compared to Grignard reagent and


organolithiums, however, they are more selective in their reaction with other functional
groups.
• Lithium dialkylcuprates couple readily with halogenated compounds.
• Lithium dialkylcuprates do not react with carbonyl compounds, esters, amides, nitriles,
and the like. Also they do not react with isolated double and triple bonds.

Lithium dialkylcuprates (Gilman reagent) are also used for the formation of
carbon–carbon bond. It is the most versatile and selective reagent used for the
preparation of unsymmetrical alkanes (Corey–House synthesis) thus overcoming
the limitation of Wurtz synthesis (which cannot be used to synthesize
unsymmetrical alkanes, refer Section 5A.2.2). Besides this, it reacts with organic
halides, acid chlorides, epoxy compounds and α, β-unsaturated carbonyl
compounds.
The reactions of Gilman reagent (Lithium dialkylcuprates) have been
summarized in this section. All reactions are discussed taking lithium
dimethylcuprate as the representative example:
(1) Reaction of lithium dimethylcuprate with halogenated organic
compounds. Lithium dialkylcuprates readily couple with halogenated
compounds and replace the halogen with alkyl group (from reagent). This
coupling reaction as well as the low reactivity towards carbonyl compounds is
used extensively to prepare higher alkanes, higher acids, unsaturated
compounds, and ketones. The reactions are as follows:

(2) Reaction with epoxides (formation of alcohols)

In case of unsaturation at α-carbon (vinylic epoxides), the reagent attacks at β-carbon


and the alkylation occurs with allylic rearrangement (C-alkylation).

(3) Reaction with α,β-unsaturated carbonyl compound (β-alkylation). In case


of α,β-unsaturated carbonyl compounds, the reagent does not attack the carbonyl
group (1,2-addition) rather conjugate addition (1,4-addition) occurs that results
in the formation of β-alkylated carbonyl compounds. The reaction occurs in the
following manner:
Comparing the reactivity of Grignard reagent and organolithiums with Lithium
dialkylcuprates
In case of the reaction of Grignard reagent with α,β -unsaturated carbonyl compounds,
the reaction occurs preferably at carbonyl group (1,2-addition) to yield α,β-unsaturated
alcohols besides the reaction at olefinic carbon (1,4-conjugate addition).
There is a competition between 1,2- and 1,4-addition reaction in case of Grignard
reagent (refer EM problems 112, 113, 114), however, Gilman reagent exclusively gives
1,4-addition product.
The organolithiums on reaction with α,β-unsaturated carbonyl compounds exclusively
gives 1,2-addition product (p. 611).

PART II: OXIDATION–REDUCTION


REACTIONS
25.5 INTRODUCTION
The loss of electron is referred to as oxidation process while the gain of electrons
is a reduction process. In the organic reactions, several oxidizing and reducing
agents are used for the transformation of one functional group to another
functional group.
The organic compounds can have more than one functional group present in it
and an oxidizing or reducing agent may interact with all the functional groups.
Sometimes, it is necessary to choose such an oxidizing or reducing agent, which
can bring the desirable changes in a particular functional group selectively
without affecting the other functional groups present in it.
In most of the chapters, a number of oxidation and reduction reactions have been
detailed while dealing with the chemistry of functional groups. This review
offers a concise and lucid information about general oxidation-reduction
reactions, that are studied and used frequently in organic chemistry.
25.6 REDUCTION REACTIONS

25.6.1 Reduction by Catalytic Hydrogenation


Catalytic hydrogenation is carried out by passing hydrogen, in the presence of
metals such as Pt, Pd, or Ni to the unsaturated organic compounds to yield
corresponding reduced products. The reduction reactions by catalytic
hydrogenation can be summarized as follows:

The reduction of alkynes to alkenes under controlled catalytic hydrogenation


results in the formation of cis-alkenes. But reduction of alkynes by dissolving
metals results in the formation of trans-alkenes (Section 6.2.1).
CATALYTIC REDUCTION: AT A GLANCE
25.6.2 Reduction by Metal Hydrides

(a) Lithium aluminium hydride [LiAlH4]


Preparation
Lithium aluminium hydride [LiAlH4] is prepared by the reaction of lithium
hydride and aluminium chloride

General characteristics
(i) LiAlH4 is a dark grey solid (m.p. 125°C, density 0.917 g/cc) and is highly
inflammable. It is soluble in ether. It is highly reactive towards water and
protonated solvents like alcohols.
(ii) The reaction of water with LiAlH4 is highly exothermic and the liberated
hydrogen may catch fire. Thus, LiAlH4 is stored away from water and its
reactions are carried out in perfectly anhydrous conditions.

• It is a strong reducing agent and reduces the compounds by hydride transfer


(H–) and thereby behaves as a nucleophilic reagent.

Reactions of LiAlH4
The reduction reactions of LiAlH4 are summarized as follows:
(1) Reduction of oxygen containing functional groups
General reactions:
(2) Reduction of nitrogen containing functional groups
General reactions
Selective reduction of nitrile to aldehyde
DIBAL (diisobutylaluminium hydride) adds only one hydride to nitrile to
give imine derivative, which on hydrolysis results in the formation of
aldehyde.

(3) Reduction of halogenated hydrocarbons


General reaction
DECREASING ORDER OF REDUCTION WITH LiAlH4: AT A GLANCE

(b) Sodium borohydride [NaBH4]


Preparation
Sodium borohydride is prepared by the reaction of sodium hydride and
trimethoxyborane in the manner as shown:

General characteristics
(i) It is less reactive than LiAlH4. It is a colourless solid (m.p. 400° C, density
1.074 g/cc).
(ii) It is soluble in water but gets decomposed by water with liberation of heat
to form diborane gas.
(iii) It is a mild reducing agent and selectively reduces the compounds by
hydride transfer. Similar to LiAlH4, it behaves as a nucleophilic reagent.
(iv) The reactions of sodium borohydride are generally carried out in alcohol as
a solvent unlike LiAlH4 where alcohol cannot be used as solvent.

Reactions of NaBH4
The reduction reactions of NaBH4 are summarized in this section. Functional
groups, which cannot be reduced by NaBH4 have also been listed.

FUNCTIONAL GROUPS NOT REDUCED BY NaBH4

(c) Diborane [B2H6]; Boron Hydride [BH3]


Preparation
Diborane is prepared by the reaction of NaBH4 with boron trifluoride as

General characteristics
(i) Boron hydride (BH3) exists as a more stable dimeric structure and is
known as diborane (B2H6).
(ii) It is highly reactive and is stored in ethers like tetrahydrofuran (THF). Its
reactions are carried out in nitrogen atmosphere.
• Unlike LiAlH4 or NaBH4, diborane behaves as an electrophilic reagent.

Reactions of B2H6
In the reactions, the reagent is written as BH3 and its reactions are generally
carried out in inert solvents like THF. The reduction reactions of B2H6 are
summarized as follows along with the list of functional groups, which cannot be
reduced by B2H6.
Functional groups not reduced by BH3

*Reduction occurs in the presence of catalysts (like BF3) only.


25.7 OXIDATION REACTIONS
(1) Oxidation of alkenes and alkynes
(a) with KMnO4 and Na2Cr2O7
(i) Oxidation of alkenes in cold alkaline KMnO4 results in the formation of
glycols (hydroxylation reaction). The reaction is also referred to as
Baeyer’s test for unsaturation.
(ii) Oxidation of alkenes in hot KMnO4, acidified KMnO4, or acidified
dichromate results in the formation of carboxylic acid and (or) ketone
depending upon the nature of the alkene.
(iii) Oxidation of alkynes in alkaline KMnO4 results in the formation of 1,2-
diketo compounds. The reactions are illustrated as follows:

(b) with ozone (ozonolysis)


Alkenes, on ozonolysis followed by reductive cleavage result in the formation of
two molecules of carbonyl compounds. The oxidative cleavage results in
the formation of carboxylic acid and (or) ketones.
Alkynes on ozonolysis followed by hydrolysis result in the formation of two
molecules of carboxylic acids. Alkynes having terminal hydrogen result in the
formation of acid and carbon dioxide.

(2) Oxidation of alcohols and carbonyl compounds


(a) with KMnO4 and Na2Cr2O7 (or K2Cr2O7)
Primary alcohols result in the formation of aldehyde, which further oxidizes to
carboxylic acids. Secondary alcohols result in the formation of ketones but
ketones do not oxidize further.

Jones reagent (solution of chromic acid and sulfuric acid in water) is also used to
oxidize
1° alcohols to acid and 2° alcohols to ketone. The oxidation of alcohol is indicated by a
change in the colour of the solution from orange to green.

(c) with periodic acid [HIO4] and lead tetraacetate [Pb(CH3COO)4]


(Oxidation of glycols)
Glycols, on oxidation with HIO4 (or lead tetraacetate) result in the formation of
acid and (or) ketone depending upon the substitution.
(3) Oxidation of alkylbenzenes
(i) Oxidation of toluene with chromyl chloride results in the formation of
benzaldehyde.
(ii) Oxidation of toluene with hot alkaline KMnO4 results in the formation of
benzoic acid. In general, all the alkyl benzenes except tert. butylbenzene
(due to nonavailability of benzylic hydrogen) result in the formation of
benzoic acid.
Following reactions summarize the oxidation of alkylbenzenes.

(4) Oxidation with Selenium dioxide (SeO2)


Selenium dioxide is used for oxidation of methyl or methylene group present α
to a carbonyl group, to give 1,2-dicarbonyl compounds. In the process, SeO2 is
reduced to metallic selenium (Se8).
Chapter26
Heterocyclic
Compounds

26.1 INTRODUCTION
The cyclic organic compounds where one or more carbon atoms of the ring are
replaced by other atoms (referred to as hetero atom) such as those of nitrogen,
oxygen, sulfur, and so on are classified as heterocyclic compounds. Nitrogen
heterocyclic compounds are most commonly found in nature and are involved in
several biological functions in plants and animals.
The heterocyclic compounds generally have five or six membered rings with one
or more hetero atom(s). Further, heterocyclic compounds may have fused ring
structure with other cyclic organic compounds and are known as condensed
heterocyclic compounds. The heterocyclic compounds may exhibit aromaticity
depending on the structure. Some important heterocyclic compounds are better
known by their common names and are widely accepted in IUPAC system. In
IUPAC nomenclature, the heterocyclic compounds are named in accordance with
the size of ring and the hetero atom present in the ring system (Table 26.1). For
example, In case of 5-membered heterocyclic ring system for nitrogen, sulfur,
and oxygen the prefixes used are aza-, thio- and oxa-, respectively. In case of
more than one hetero atom, the priority order followed while naming is oxa >
thio > aza. When used as suffix, the names of the compounds are written so that
they end with -ole. A few illustrative examples are as follows:
Numbering the position in heterocyclic compounds. The numbering in
heterocyclic compounds begins from the hetero atom in a molecule. The 2-
position is also referred to as α-position and similarly 3-position as β-position.
The numbering in pyrrole, furan, thiophene, pyridine, quinoline, and
isoquinoline is as follows:
26.2 FIVE MEMBERED HETEROCYCLIC
COMPOUNDS

26.2.1 Pyrrole, Furan, and Thiophene


The pyrrole ring system is largely found in several biomolecules such as vitamin
B12, alkaloids, chlorophyll, auxins (plant hormone), and so on. Thiophene
occurs in coal tar in small quantity along with pyrrole. Furan is obtained from
the distillation of pinewood.
General characteristics: Pyrrole, furan, and thiophene are five membered
heterocyclic compounds containing nitrogen, oxygen, and sulfur as heteroatoms
in the corresponding ring systems, respectively. They are colourless liquids. The
boiling point of pyrrole, furan, and thiophene are 126°, 32°, and 84°C
respectively. Pyrrole has a relatively high boiling point as compared to furan and
thiophene due to the presence of intermolecular hydrogen bonding through the
hydrogen present on the nitrogen of the ring.

26.2.2 Structure and Aromaticity

In pyrrole, the carbon and nitrogen atoms are sp2 hybridized. Nitrogen utilizes
its sp2 hybrid orbitals in making two N–C bonds (sp2-sp2 overlap) and one N–H
bond (sp2–s overlap). The p-orbital of nitrogen carries a lone pair of electron.
Similarly, the p-orbital of each carbon carries a single electron. The five p-
orbitals (one of nitrogen and four of carbon) being in the same plane overlap
with each other, resulting in the delocalization of six π-electrons (1 lone pair + 4
single electrons = 6 π-electrons) in the cyclic ring system. This sextet of
delocalized π-electrons follows Huckel’s rule (4n + 2 delocalized π-electrons;
refer Section 9A.4) and imparts aromatic character to pyrrole.

In furan, the carbons and oxygen are all sp2 hybridized. Oxygen utilizes its sp2
hybrid orbitals in making two O–C bonds (sp2–sp2 overlap) while the third sp2
hybrid orbital carries an unshared pair of electrons. The pure p-orbital on
oxygen is in the same plane as p-orbitals of carbon and carries a pair of
electrons. Thus, overlap of five π-orbitals results in delocalization of six π-
electrons (1 lone pair + 4 single electrons = 6 π-electrons). The sextet of
delocalized π-electrons follows Huckel’s rule (4n + 2 delocalized π-electrons;
refer Section 9A.4) and imparts aromatic character to furan. In furan, the sp2
hybrid orbital of oxygen carrying unshared pair of electrons is orthogonal
(perpendicular) to the plane of p orbitals and does not participate in
delocalization.
In thiophene, the aromaticity is explained in a similar manner as furan. The
carbons and sulfur are all sp2 hybridized. Sulfur uses sp2 hybrid orbitals to form
two S–C bonds (sp2–sp2 overlap) and the third sp2 hybrid orbital carries an
unshared pair of electrons. The p-orbital of sulfur carries two electrons. The p-
orbitals on sulfur and on each carbon are in the same plane and their overlap
causes delocalization of six π-electrons and follows Huckel’s rule (4n + 2
delocalized π-electrons; refer Section 9A.4). Thus, thiophene exhibits aromatic
character. The sp2 hybrid orbital of sulfur carrying unshared pair of electrons
does not participate in delocalization since it is orthogonal (perpendicular) to p-
orbitals of carbons.

Pyrrole, furan, and thiophene are best represented by a hybrid of contributing


structures I (a–e),
II (a–e), and III (a–e) respectively. as shown in Fig. 26.1.

Fig. 26.1 Various contributing structures of pyrrole I(a–e), furan II(a–e), and thiophene III(a–e).
26.2.3 Preparations of Pyrrole, Furan, and Thiophene
From 1,4-dicarbonyl compounds (general method)
Pyrrole, furan, and thiophene can conveniently be synthesized using 1,4-
dicarbonyl compounds (as the initial reactant) as follows:
(i) Heating 1,4-dicarbonyl compounds in presence of ammonium carbonate
results in cyclization to form pyrrole (Paal–Knorr Synthesis).
(ii) Heating 1,4-dicarbonyl compounds in the presence of phosphorous
pentoxide (P2O5) results in the formation of furan.
(iii) Heating 1,4-dicarbonyl compounds in the presence of phosphorous
trisulfide (P2S3) results in the formation of thiophene.
These reactions can be summarized as follows:

This method can also be used for the preparation of number of alkyl derivatives
of pyrrole, furan, and thiophene by taking suitable 1,4-dicarbonyl compound.
Preparations of pyrrole
Following methods are used for the preparation of pyrrole.
(i) Pyrrole is obtained by distillation of succinimide over zinc dust.

(ii) Knorr–pyrrole synthesis: This involves the condensation of α-amino


ketones with a β-diketone or β-ketoester to give a substituted pyrrole.

(iii) Heating a mixture of furan, ammonia, and steam over alumina catalyst
results in the formation of pyrrole.

(iv) Pyrrole is obtained by passing a mixture of acetylene and ammonia over


red hot tube.

Preparations of furan
(i) The catalytic decomposition of 2-formylfuran popularly known as furfural
(or furfuraldehyde) in steam, in the presence of calcium oxide as a catalyst,
yields furan.
Furfural is prepared by distilling pentoses (carbohydrates) with dilute sulfuric
acid.
(ii) Dry distillation of mucic acid forms furoic acid. Decarboxylation of furoic
acid using copper and quinoline results in the formation of furan.

Preparations of thiophene
(i) The reaction of butane and sulfur at 600°C results in the formation of
thiophene.

(ii) Commercially, thiophene is prepared by passing a mixture of acetylene


and hydrogen sulfide over heated alumina at 400°C.

(iii) Heating sodium succinate with phosphorous trisulfide results in the


formation of thiophene.

26.2.4 Chemical Properties of Pyrrole, Furan, and


Thiophene
The heterocyclic compounds being aromatic in nature undergo electrophilic
substitution reactions such as nitration, sulfonation, acetylation, halogenation,
formylation, reaction with organometallic compounds, and so on. Besides
electrophilic substitution reactions, these compounds also undergo reduction
reactions.
Electrophilic substitution reactions
General reactivity: In five-membered heterocyclic compounds, the heteroatoms
donate their electron pair to aromatic sextet. Thus, the heterocyclic rings are
highly activated (Fig. 26.1) and their reactivity towards electrophilic substitution
is more compared to benzene. In fact, the reactivity of aromatic heterocyclic
rings is at par with phenol and aniline.

The overall order of reactivity of five membered heterocyclic compounds, as


compared to benzene, towards electrophilic substitution reactions is as follows:

The high reactivity of pyrrole is due to the ease with which nitrogen
accommodates the positive charge compared to oxygen and sulfur, which results
in an increase in the electron density of the ring. Thus, the nucleophilic
character of the ring increases and electrophile readily attacks it. In case of
thiophene the sulfur utilizes its 3d orbitals in delocalization and as a result, the
ring bears positive charge (Fig. 26.2) and the reactivity towards electrophile
decreases.
Attack of electrophile (E+) is preferred at C2. The electrophile can attack
either at C2 or at C3 of the heterocyclic ring. In general, the electrophilic
substitution occurs preferably at C2.
(i) Also, C2 is relatively electron rich as evident from the contributing
structures I (c–d),
II (c–d), and III (c–d), which are more stable, compared to I (b and e), II (b
and e), III (b
and e) (Fig. 26.1).
(ii) The intermediate carbocation formed by the attack of electrophile at C2, is
more resonance stabilized (three contributing structures) compared to
carbocation formed by attack of electrophile at C3 (two contributing
structures).

(1) Nitration. Pyrrole, furan, and thiophene undergo nitration at C2 position to


form corresponding 2-nitro derivatives.
The nitration of pyrrole is carried out at low temperature in the presence of nitric
acid and acetic anhydride. At high temperature, the highly activated pryrrole ring
breaks down under reaction conditions.

The nitration of furan is carried out with acetyl nitrate (CH3COONO2).

The nitration of thiophene is carried out with fuming nitric acid in the presence
of acetic anhydride.

(2) Sulfonation. The sulfonation of pyrrole, furan, and thiophene also occurs at
C2 position of the heterocyclic ring to yield corresponding sulfonic acids.
The sulfonation of pyrrole and furan is carried out with sulfur trioxide in
pyridine.

The sulfonation of thiophene is carried out with cold sulfuric acid.


It should be noted that the sulfonation of pyrrole and furan is not carried out with
sulfuric acid since it can destroy the activated ring system.
(3) Acetylation. The acetylation of pyrrole, furan, and thiophene occurs at C2
position of the heterocyclic ring. The acetylation is carried out with acetic
anhydride in the presence of boron trifluoride (a Lewis acid), which results in the
formation of corresponding 2-acetyl derivatives (a methyl ketone).

Reaction of thiophene with benzoyl chloride in the presence of tin (IV) chloride
results in the formation of 2-benzoylthiophene (a phenyl ketone).

(4) Halogenation. Halogenation of furan and thiophene is carried out at low


temperature and results in the formation of 2-halo substituted derivatives as the
major products. Along with this, some amount of 2,5-disubstituted product is
also formed.
Pyrrole being highly reactive on halogenation results in the formation of 2,3,4,5-
tetrahaloderivatives. For example, chlorination of pyrrole, furan, and thiophene
is carried out as follows:
(5) Mercuration. Furan and thiophene react with mercury (II) chloride in the
presence of sodium acetate to form corresponding 2-mercury chloride
derivatives. The mercuration of pyrrole is a difficult reaction.

The mercury group can be easily replaced by halogen and acyl group and is
therefore used as an intermediate to prepare 2-substituted derivatives.
(6) Formylation. Formylation is carried out using Vilsmeier–Haack reaction
where dimethylformamide (DMF) in presence of phosphorous oxychloride
(POCl3) is used as the formylating reagent. The formylation reaction with
pyrrole, furan, and thiophene results in the formation of corresponding 2-formyl
derivatives.

Formylation can also be carried out using Gatterman reaction in the presence of
Zn(CN)2/HCl.

2-formylfuran known as furfural (or furfuraldehyde) is


prepared by distilling pentoses (carbohydrates) with dilute sulfuric
acid (Section 26.2.3).
Furfural resembles benzaldehyde in its chemical reactions. It
undergoes reactions like benzoin condensation, benzilic acid
rearrangement, Perkin, Cannizzaro’s, and Claisen reaction similar to
benzaldehyde.

(7) With organometallic compounds. Pyrrole is an active ring system and


reacts readily with Grignard reagent to form pyrrolyl-2-magnesium bromide.
The reactivity of furan and thiophene is relatively less and thus they react with
more electropositive metals. Thus, furan and thiophene react with butyllithium to
form corresponding 2-lithium derivatives.
The organometallic derivatives of these heterocyclic compounds are used as
intermediates to prepare various 2-substituted derivatives. This is depicted in the
following reactions:

(8) Reaction with diazonium salts. Pyrrole couples with diazonium salts in
weakly acidic medium to give azo compounds. The coupling occurs at 2-
position. In alkaline solution, it couples at 2- and 5-positions to give bis-azo
compounds.
Furan reacts with diazonium salts in alkaline medium and undergoes substitution
to give
2-arylderivative. The reaction involves the replacement of diazo group and is
known as Gomberg’s reaction.

Reduction reactions
Pyrrole and furan on reduction with hydrogen (hydrogenation) in presence of Ni
(at 200–300°C) form corresponding tetrahydro derivatives.

But thiophene on reduction with hydrogen and nickel results in the opening of
the ring to form butane. Tetrahydrothiophene is formed by the hydrogenation of
thiophene in the presence of palladium (Pd) as a catalyst. In this method, catalyst
Pd is used in relatively large amounts to overcome the poisoning effect of sulfur
of thiophene.
Some other reactions
(i) Salt formation and Kolbes—Schmitt reaction in pyrrole: Pyrrole is a weak
base. The lone
pair of electrons on nitrogen is not available for reaction as it is a part of the
aromatic sextet
(Section 26.2.2). Pyrrole shows a weakly acidic character and reacts with
potassium hydroxide to form a potassium salt. The loss of proton from nitrogen
is favoured as pyrrolyl anion is resonance stabilized. Due to this acidic character,
its potassium salt readily undergoes Kolbes–Schmitt reaction (similar to phenol)
to form pyrrole -2-carboxylic acid.

(ii) Diels–Alder reaction of furan. Furan is the only five membered


heterocyclic compound which undergoes Diels–Alder reaction. Furan being least
aromatic in character behaves as a diene and reacts with maleic anhydride (a
dienophile) to form an adduct.
26.3 SIX-MEMBERED HETEROCYCLIC
COMPOUND

26.3.1 Pyridine
Pyridine is obtained from coal tar. It is a six membered heterocyclic compound
containing one nitrogen atom. Its molecular formula is C6H5N and a boiling
point 115°C.

26.3.2 Structure and Aromaticity

In pyridine, five carbon atoms and one nitrogen atom are all sp2 hybridized.
Nitrogen utilizes its sp2 hybrid orbitals in making two N–C bonds (sp2–sp2
overlap) and the third hybrid orbital carries a lone pair of electrons. The p-
orbital of nitrogen carries a single electron. Similarly, the p-orbital of each
carbon carries a single electron. The p-orbital of nitrogen lies in the same plane
as the
p-orbital of five carbons. The six p-orbitals (one of nitrogen and five of carbon)
overlap with each other, resulting in the delocalization of six π-electrons (1
electron of nitrogen + 5 single electron of carbons = 6 π-electrons) in the cyclic
ring system. This sextet of delocalized π-electrons follows Huckel’s rule (4n + 2
delocalized π-electrons; refer Section 9A.4) and imparts aromatic character to
pyridine.
The lone pair of electrons on nitrogen does not participate in delocalization as
the sp2 hybrid orbital of nitrogen carrying the lone pair of electrons is
orthogonal (perpendicular) to the plane of the p-orbitals of carbon.
Pyridine can be represented as a resonance hybrid of the following structures:

26.3.3 Basicity of Pyridine


Pyridine is basic in nature (pKa = 5.21) due to the presence of lone pair of
electrons on nitrogen. This lone pair does not participate in delocalization and is
readily available for reaction. Pyridine behaves as a typical tertiary amine. In
acidic medium, it forms pyridinium salts. Pyridine also reacts with haloalkanes
to form N-alkyl pyridinium salt (quaternary salt).

Pyridine (pKa = 5.21) is more basic than pyrrole (pKa = –


0.27).
In case of pyrrole as mentioned earlier (Section 26.2.2), the lone pair
of electrons on nitrogen participates in delocalization and forms a part
of aromatic sextet. Due to non-availability of lone pair of electrons for
reactions pyrrole behaves as a very weak base.
Pyridine is less basic than aliphatic amines.
In both pyridine and aliphatic amines, the lone pair on nitrogen is
available for reaction. In case of aliphatic amines, the nitrogen is sp3
hybridized whereas in pyridine, nitrogen is sp2 hybridized. The sp2
hybridized nitrogen is more electronegative than sp3 hybridized
nitrogen. The lone pair of electrons is held more towards
electronegative sp2 hybridized nitrogen and is relatively less available
for reaction. Thus, basicity of pyridine is less as compared to aliphatic
amines.

26.3.4 Preparations of Pyridine


This section discusses the methods used for the preparation of pyridine.
(a) A mixture of acetylene and hydrogen cyanide on passing through a red hot
tube results in the formation of pyridine.

(b) Heating pentane-1,5-diamine dihydrochloride salt results in the formation


of piperidine which on dehydrogenation with Pd/C yields pyridine.
(c) Hantzsch Synthesis. The condensation of a β-dicarbonyl compound with
an aldehyde and ammonia gives a dihydropyridine derivative, which on
oxidation with nitric acid yields a pyridine derivative.

26.3.5 Chemical Properties of Pyridine


Electrophilic substitution reactions
Pyridine undergoes electrophilic substitution preferably at 3-position (β-position)
as it results in the formation of a more resonance-stabilized intermediate (three
contributing structures). The intermediates formed by the attack of electrophiles
at 2- and 4-positions are less stabilized since in the contributing structures (refer
26.3.2) nitrogen carries positive charge. The nitrogen is electronegative in
nature and a structure with positive charge on electronegative atom is a high-
energy structure and least contributing towards stabilization. Thus, the
intermediates formed by attack of electrophile at 2- and 4-positions have only
two contributing structures.
Pyridine behaves as a highly deactivating ring and electrophilic substitution
reaction occurs less readily in pyridine as compared to benzene. The deactivation
is due to the presence of highly electronegative nitrogen, which decreases the
electron density in the ring system. It undergoes halogenation, nitration,
sulfonation, and mercuration reactions to give 3-substituted derivatives.
However, due to deactivation it does not undergo Friedel–Crafts reaction. The
electrophilic substitution reactions in pyridine are summarized as under:
Nucleophilic substitution reactions
Pyridine is a deactivated aromatic ring system. The electron deficient centres at
2- and 4-positions are highly susceptible to the attack of nucleophiles. The
nucleophilic substitution reaction in pyridine occurs preferably at 2-position.

(i) Chichibabin reaction. Pyridine when heated with sodamide results in the
formation of
2-aminopyridine and the reaction is known as Chichibabin reaction.

(ii) Pyridine reacts with organometallic compounds to form 2-alkyl or 2-aryl


derivatives. The alkyl group behaves as a nucleophile and attacks at 2-
position of pyridine.
(iii) The reaction of pyridine with potassium hydroxide results in the
formation of 2-pyridone. In this reaction, the attack of hydroxide ion results
in the formation of 2-hydroxypyridine, which tautomerises to a more stable
keto- form, that is, 2-pyridone.

Oxidation reactions
The oxidation of pyridine with hydrogen peroxide or peroxybenzoic acid results
in the formation of pyridine-N-oxide.
Reduction reactions
The reduction of pyridine under different reaction conditions results in the
formation of different products. Such as,
(i) The catalytic hydrogenation of pyridine using nickel results in the
formation of piperidine.
(ii) Reduction of pyridine with lithium aluminiumhydride gives 1,2-
dihydropyridine.
(iii) Reduction of pyridine with sodium in liquid ammonia (Birch reduction)
gives 1,4-dihydropyridine.
(iv) Pyridine on reduction with hydrogen iodide at 300°C results in ring
opening to form pentane and ammonia.
These reactions are summarized as follows:
26.4 FUSED HETEROCYCLIC SYSTEMS

26.4.1 Quinoline and Isoquinoline


In quinoline and isoquinoline, the benzene ring is fused with pyridine ring.
These compounds are also known as benzopyridines. Quinoline and isoquinoline
are obtained from coal tar and are high boiling liquids. These compounds are
generally the structural units of alkaloids.

26.4.2 Preparations of Quinoline


(a) Skraup’s synthesis. Quinoline is synthesized by heating aniline with
glycerol and concentrated sulfuric acid in the presence of nitrobenzene as a
dehydrogenating agent. The reaction is violent and is controlled by adding small
amount of ferrous sulfate. Following steps are involved in the synthesis:
Step 1. Dehydration of glycerol (formation of acrolein)
Step 2. Reaction of acrolein with aniline (an addition reaction)
Step 3. Ring closure at ortho position of aromatic ring (formation of 1,2-
dihydroquinoline)
Step 4. Oxidation of 1,2-dihydroquinoline by nitrobenzene (formation of
quinoline)
(b) Friedlander’s synthesis. The condensation of o-aminobenzaldehyde and
acetaldehyde in presence of alkali results in the formation of quinoline.

26.4.3 Chemical Properties of Quinoline


Quinoline is basic in nature and forms quaternary salts with acids and
haloalkanes. Being a fused ring system, it resembles in its reactions with
benzene and pyridine.
(a) Electrophilic substitution reactions. Quinoline undergoes electrophilic
substitution reactions where substitution of electrophile occurs in the benzene
ring. This can be explained by the fact that the pyridine ring is electron deficient
as compared to benzene ring and thus in the fused ring system electrophile
attacks preferably electron rich benzene ring.
The electrophilic substitution reactions occur at 8- and 5-positions. Quinoline is
more reactive towards electrophilic substitution reactions compared to pyridine.
The electrophilic substitution reactions are summarized here as follows:
(b) Nucleophilic substitution reactions. Quinoline undergoes the attack of
nucleophile at 2- and 4-positions, that is, in the electron deficient pyridine ring.
Thus, it resembles the pyridine ring in nucleophilic substitution reactions. The
reactions are summarized as follows:

(c) Oxidation. Quinoline is oxidized to pyridine 2,3-dicarboxylic acid in the


presence of potassium permanganate.
(d) Reduction. Catalytic hydrogenation of quinoline in the presence of nickel
results in the formation of 1,2,3,4-tetrahydroquinoline. Further reduction reduces
the benzene ring and results in the formation of decahydroquinoline.

26.4.4 Preparations of Isoquinoline


(a) Bischler–Napieralski reaction. Heating N-acetyl-2-phenylethylamine with
phosphorous oxychloride results in intramolecular cyclization to form 3,4-
dihydroisoquinoline, which on dehydrogenation over palladium gives
isoquinoline derivative.

(b) Isoquinoline is formed when vapours of benzylidine ethylamine are passed


over red hot tube.

26.4.5 Chemical Properties of Isoquinoline


Isoquinoline is basic in nature and resembles quinoline in its chemical reactions.
The electrophilic substitution reactions occur at 5- or 8-position. The
nucleophilic substitution reactions occur at
1-position. It also undergoes oxidation and reduction reactions. Few reactions of
isoquinoline are summarized as follows:
EXERCISES
1. What are heterocyclic compounds?
2. Discuss the aromaticity in pyrrole, furan, and thiophene.
3. Explain the following:
(a) Pyridine is more basic compared to pyrrole.
(b) Piperidine is more basic than pyridine.
(c) Pyrrole does not give Diels–Alder reaction.
(d) Electrophilic substitution in pyrrole occurs at C2 and not C3.
(e) Pyridine-1-oxide is more reactive than pyridine towards electrophiles and
nucleophiles.
(f) Pyridine behaves as deactivated benzene ring system.
(g) Furan undergoes Diels–Alder reaction.
4. Pyrrole behaves as phenol in its reactions? Explain giving two chemical
reactions in support of this statement.
5. How do pyridine and pyrrole differ in their reaction with sulfuric acid?
6. Arrange the following in increasing order of basicity:
Pyridine, methylamine, piperidine, and pyrrole
7. Give Skraup’s synthesis for quinoline.
8. Complete the following reactions:
9. Carry out the following conversions:
(a) Furan to furfural
(b) Pyrrole to pyrrole-2-carboxylic acid
(c) Pyridine to 2-bromopyridine
(d) Thiophene to 2-acetylthiophene
(e) Furfural to 5-nitro-2-furoic acid
(f) Pyridine to 4-nitropyridine
10. Write the product formed and mechanism involved in the following
reactions of furfural
(a) with acetone in presence of base
(b) with propanoic anhydride in presence of sodium propionate
11. Name the products formed when isoquinoline reacts with
(a) Butyllithium
(b) Potassium permanganate
(c) H2/Ni
(d) Peroxybenzoic acid
(e) Sulfuric acid
(f) Nitrating mixture
H
Chapter27
Carbohydrates

27.1 INTRODUCTION
Carbohydrates, also termed as saccharides, one of the basic components of
food, are composed of carbon, hydrogen, and oxygen. Structurally, they are
polyhydroxy aldehydes or polyhydroxy ketones. Lower molecular weight
carbohydrates are known as sugars and their names generally end in
characteristic ose. Some examples are sucrose (cane sugar), lactose (milk sugar),
glucose (also known as dextrose), and fructose (levulose).
Classification
Carbohydrates are divided into four major groups–monosaccharides,
disaccharides, oligosaccharides, and polysaccharides.
(i) Monosaccharides. These are the simple sugars, which cannot be hydrolyzed
further into simpler forms and they have a general formula CnH2nOn.
Monosaccharides can be further sub-classified in accordance with
(i) the number of carbon atoms in continuous chain. A three-carbon
monosaccharide is termed as triose; one with four carbons—a tetrose; one
with five carbons—a pentose; and that with six carbons—a hexose.
(ii) the aldehyde (–CHO) or ketone (–CO) group present in the saccharide.
The saccharides containing an aldehyde group are called aldoses while
those with a ketone group are referred to as ketoses.

Some examples of monosaccharides are as follows:


(ii) Disaccharides. These are the sugars, which yield two same or different
molecules of monosaccharides, on hydrolysis. In other words, a disaccharide is
the result of joining of two monosaccharide units. The general formula for
disaccharides is Cn(H2O)n–1. Examples of some common disaccharides along
with products obtained from their hydrolysis are as follows:
• Maltose yields two molecules of glucose.
• Lactose yields one molecule of glucose and one molecule of galactose.
• Sucrose yields one molecule of glucose and one molecule of fructose.
Monosaccharides and disaccharides are collectively called sugars.
(iii) Oligosaccharides. These carbohydrates yield three to about eight
monosaccharide units on hydrolysis. A few examples of oligosaccharides are
• Maltotriose is a trisaccharide, which on hydrolysis yields three molecules of
glucose.
• Raffinoses a trisaccharide, which on hydrolysis yields three different
monosaccharides, namely glucose, galactose, and fructose.
(iv) Polysaccharides (Glycans). These carbohydrates yield more than eight
monosaccharide units on hydrolysis, for example, starch and glycogen. The
general formula for polysaccharides is (C6H10O5)n.
27.2 D- AND L-SUGARS: RELATIVE
CONFIGURATION OF SUGARS
Glyceraldehyde has an asymmetric carbon (marked by *) and exists in two
enantiomeric forms. The enantiomers having –OH group on the right side in
Fischer projection are referred to as D-glyceraldehydes and the ones with the –
OH on left as L-glyceraldehydes.

All aldoses can be prepared by a complex series of reactions from one of the
enantiomeric glyceraldehydes. The overall procedure involves adding carbons
one at a time. By this method, the glyceraldehyde is converted to an aldotetrose;
the aldotetrose can in turn be used to synthesize an aldopentose. Further,
aldopentose can be used to synthesize aldohexose, and so on. When
glyceraldehyde is converted to a tetrose, the carbon atom of the aldehyde group
of glyceraldehyde is reduced, thus, becoming a new-asymmetric carbon atom.
The added carbon atom becomes the aldehyde group of aldotetrose produced.

Thus, two aldotetroses are formed from D-glyceraldehyde and two from L-
glyceraldehyde. Consequently, there are four aldotetroses (as would be predicted
by the formula 2n for the number of stereoisomers, where n is the number of
asymmetric carbon atoms; refer Section. 3.2.4). The relationship of the four
possible aldotetroses to D- and L-glyceraldehydes is shown in Fig. 27.1.
Fig. 27.1 The four aldotetroses and their relationship with enantiomeric glyceraldehyes. Aldotetroses (i and
ii) are derived from D-glyceraldehyde whereas (iii and iv) are derived from L-glyceraldehyde.

Note that the original asymmetric carbon atom of glyceraldehyde now becomes
the asymmetric carbon farthest from the aldehyde group in new aldose. This
asymmetric carbon atom is shown as shaded in Fig. 27.1. All aldoses in which
the –OH group of the asymmetric carbon farthest from the aldehyde group
is written on the right of the Fischer projection formula are said to be
related to D-glyceraldehyde and are known as D-sugars. Similarly the aldoses
where asymmetric carbon farthest from the aldehyde group has –OH group on
the left are termed as L-sugars. This is known as relative configuration of
sugars. The D-threose and L-erythrose (ii and iii) and D-erythrose and L-threose
(i and iv) in Fig. 27.1 are enantiomeric pairs while the other combinations are
diastereomers.
In a similar manner, ketoses exist as enantiomeric pairs, that is, D and L sugars.
Most of the naturally occurring monosaccharides are members of the D-family.
The D-family of aldoses is depicted in Fig. 27.2.
This chapter discusses monosaccharides through the study of glucose (aldose)
and fructose (ketose) as representative examples.
27.3 D-(+)-GLUCOSE: CHEMICAL PROPERTIES
AND STRUCTURE
Glucose is the most common and perhaps the most abundant naturally occurring
organic compound. In nature, it occurs in fruits, honey and in blood (as blood
sugar). Glucose is a monosaccharide and it belongs to the D-family of the
carbohydrates. It is an optically active compound, which is dextrorotatory and
also known as dextrose (+ Glucose). This is a white crystalline solid (melting
point 146°C) which is highly soluble in water. Commercially it is prepared by
hydrolysis of starch under high temperature and pressure conditions in acidic
medium.

27.3.1 Open Chain Structure


The molecular formula of glucose is C6H12O6. Glucose has six-carbon straight
chain with five hydroxy groups and an aldehyde group. It can be written as a
penta-hydroxyaldehyde (aldohexose) with the open chain structure as shown
below.

The various chemical reactions that support an open chain structure of glucose
are discussed as follows:
(i) Reduction. In presence of H2/Ni, the reduction of glucose results in the
formation of sorbitol (C6H14O6) which on heating with hydrogen iodide
completely reduces glucose to yield n-hexane. Reduction to sorbitol can also be
carried out using LiAlH4 or NaBH4.
This clearly indicates that the six carbon atoms in glucose are in a straight
chain.
(ii) Reactions with
(a) Hydroxylamine and phenylhydrazine. The reaction of glucose with
hydroxylamine results in the formation of oxime. On the other hand, the reaction
with one mole of phenylhydrazine yields glucose phenylhydrazone. This
indicates the presence of a carbonyl group in glucose.
(b) HCN. Glucose reacts with HCN to produce a cyanohydrin, which on
hydrolysis followed by treatment with HI yields heptanoic acid, thus indicating
the presence of a carbonyl group in glucose as an aldehyde.
(iii) Acetylation of glucose. Glucose on reaction with acetic anhydride gives a
very stable pentaacetyl derivative.

The penta-acetyl derivative indicates that it contains five hydroxyl groups and
since the derivative is very stable, it indicates that five hydroxyl groups must be
present on different carbons.
(iv) Oxidation
(a) With bromine water. This results in the formation of gluconic acid
(C6H12O7). Bromine water is a mild oxidizing agent, which oxidizes an
aldehyde group to carboxylic acid.
(b) With concentrated nitric acid. This gives a glucaric acid (a dicarboxylic
acid), which contains the same number of carbon atoms as glucose. Nitric acid is
a strong oxidizing agent, which oxidizes not only the aldehyde group but also
any 1° alcoholic group present in the molecule to carboxylic acid.

(c) With periodic acid (HIO4). Glucose consumes five moles of periodic acid
(HIO4) to yield formic acid (five moles) and formaldehyde (one mole). The
periodic acid cleaves the carbon–carbon bond containing –OH groups.

(d) With Fehling’s solution and Tollens reagent. Glucose reduces both
Fehling’s solution as well as Tollens reagent similar to aldehydes and itself gets
oxidized to gluconic acid.
(v) Reaction of glucose with excess of phenyl hydrazine (formation of
osazone). Glucose reacts with phenyl hydrazine to form phenylhydrazone.
However, the reaction occurs further and alcoholic group adjacent to aldehyde
group gets oxidized to a carbonyl group, which reacts with phenyl hydrazine to
form osazone. The overall process involves the consumption of three moles of
phenyl hydrazine (The mechanistic details of the reaction is discussed later in
this chapter).

27.3.2 Cyclic Structure of Glucose

With the formation of hemiacetal, a new asymmetric carbon is formed and a


mixture of two stereoisomers known as α-D-glucose and β-D-glucose is
produced. The two forms α and β have different optical rotations. However, α-D-
glucose and β-D-glucose are not mirror images but diastereomers as depicted in
Fig. 27.3.

Fig. 27.3 Open chain and cyclic hemiacetal forms of D-glucose (Fischer projection).
27.3.3 MUTAROTATION
Glucose is an optically active compound. However, a solution of glucose in
water undergoes a gradual change in its initial specific rotation till a constant
specific rotation is achieved. This phenomenon of change in specific rotation is
known as mutarotation.
Explanation. The D (+)-glucose exists in two isomeric forms, namely α-D-
glucose and β-D-glucose (that is, α-D-glucopyranose and β-D-glucopyranose)
having the following physical characteristics:

The ordinary crystalline glucose happens to be in α-form. When α-D-glucose or


β-D-glucose is dissolved in water, a change in their specific rotation takes place
till both attain a value of +52.7°.
The specific rotation of α-D-glucose decreases from +112° to +52.7° and that of
β-D-glucose increases from +19° to +52.7°.

This change in the specific rotation represents a conversion of α-D-glucose to an


equilibrium mixture that contains α and β forms. The process involves opening
of hemiacetal ring to form open chain (aldehyde form) compounds. The
aldehyde formed is extremely unstable and exists only as a transient
intermediate, which is then transformed into β-D-glucose. Thus, α-D-glucose
and β-D-glucose are in equilibrium with the open chain D-glucose (refer Fig.
27.3).
A very little quantity of glucose exists in the open chain form. A dynamic
equilibrium exists between three forms and the equilibrium mixture of glucose
contains 0.02% of open chain glucose 37% (~1/3) of α-isomer, and 63% (~2/3)
of the β-isomer.

27.3.4 Synthesis and Degradation of Aldoses


(1) Killiani–Fischer synthesis of aldoses
[Lengthening of carbon chain in aldoses]
An aldose can be easily converted to another aldose with one carbon atom more
than the parent aldose using Killiani–Fischer synthesis. The following example
illustrates the conversion of an aldopentose to aldohexose.
Conversion of D-arabinose to D-glucose and D-mannose
The synthesis involves the following steps:
(i) Addition of HCN (formation of diastereomeric cyanohydrin): Addition of
cyanide to aldehyde group lengthens the chain by one carbon and generates
a new stereocentre around which two configurations are possible. As a
result, two diastereomeric cyanohydrins are formed.
(ii) Hydrolysis (formation of two diastereomeric aldonic acids)
(iii) Dehydration (formation of lactone)
(iv) Reduction of lactone with Na–Hg (formation of D-glucose and D-
mannose): The acids [in (ii)] on dehydration followed by reduction with
sodium amalgam result in the formation of two aldoses with one carbon
more than the parent aldose. The two new aldoses formed differ only in
their configuration at second carbon (C2) and are termed epimers. This is
depicted in the following reaction sequence:
In Killiani-Fischer synthesis the yield of sugar is poor. The modified Killiani-
Fischer synthesis involves the direct reduction of cyanohydrin (IA and IB) with
hydrogen using Pd/BaSO4 catalyst and water as solvent. This method gives
better yield of monosaccharides.
(2) Ruff degradation
[Shortening of carbon chain of aldoses]
An aldose can be easily converted to another aldose with one carbon less than
the parent aldose using Ruff degradation. The following example illustrates the
conversion of an aldohexose to aldopentose.
Conversion of D-glucose (an aldohexose) to D-arabinose (an aldopentose)
The degradation involves the following steps:
(i) Oxidation of D-glucose with bromine water (formation of gluconic acid).
(ii) Oxidation of calcium salt of gluconic acid (formation of D-arabinose):
The calcium salt of gluconic acid is prepared by treating it with calcium
carbonate. The calcium salt is then oxidized by hydrogen peroxide in
presence of ferric salts* as catalyst to form D-arabinose (new aldose with
one carbon atom less than the parent molecule). [*Hydrogen peroxide and
ferrous salts [Fe (II)], are together also used as oxidizing agents and
collectively known as Fenton’s reagent]. In the process, carbonate ion is
eliminated.
Conversion of D-glucose to D-arabinose is illustrated in the following reaction:

(3) Wohl’s degradation of aldoses


[Shortening of carbon chain of aldoses]
An aldose can be easily converted to another aldose with one carbon less than
the parent aldose using Wohl’s degradation. The following example illustrates
the conversion of an aldohexose to aldopentose.
Conversion of D-glucose (an aldohexose) to D-arabinose (an aldopentose)
The degradation involves following steps:
(i) Reaction of D-glucose with hydroxylamine gives corresponding aldoxime.
(ii) The aldoxime on heating with acetic anhydride looses water molecule to
form a polyacetylated aldonitrile.
(iii) Further heating of the product with ammonical silver oxide results in
elimination of HCN and hydrolysis of acetyl group to produce D-arabinose
(a new aldose with one carbon less). This step is, in fact the reverse of
cyanohydrin reaction.
The process takes place in the following manner:

27.3.5 Epimerization and Ene–Diol Rearrangement


Inter-conversion of D-glucose, D-mannose and D-fructose
The α-hydrogen in aldose or ketose is acidic in nature. In presence of an alkali,
the abstraction of this α-hydrogen results in the formation of an enolate ion,
which is stabilized through resonance.
The enolate ion may undergo protonation—
(i) at carbon, resulting in the formation of epimers (epimeric aldoses) and
(ii) at oxygen, resulting in the formation of ketose through ene–diol
rearrangement.

As an example, we now discuss the interconversion of D-glucose, D-mannose,


and D-fructose.
(a) In the presence of an alkali, D-glucose loses the α-hydrogen (at C2) to
form an enolate ion. The protonation regenerates the stereocentre at C2,
which was lost during enolization. The attachment of proton can occur from
either face to produce two isomeric aldoses, namely D-glucose and D-
mannose.

(b) In the presence of alkali, D-glucose looses the α-hydrogen (at C2) to form
an enolate ion, which is resonance stabilized. The enolate ion may undergo
protonation at oxygen to form an ene–diol intermediate, which in presence
of alkali forms an enolate ion. The rearranged enolate ion on protonation
forms D-fructose as
Thus, interconversion of aldose and ketose may be carried out in alkaline
medium through an ene–diol intermediate. As all the steps involved are
reversible, one can easily write down interconversion of D-glucose, D-mannose,
and D-fructose (aldose to ketose and also ketose to aldose) as follows:
All monosaccharides are reducing sugars.
Oxidation with Tollens and Fehling’s reagent
The sugars, which reduce Tollens and Fehling’s reagent, are known as
Reducing sugars. Both aldoses and ketoses reduce Fehling’s solution
and Tollens reagent. Aldoses contain aldehyde group, which reduces
the Fehling’s solution and Tollens reagent and in turn, aldoses are
oxidized to carboxylic group containing compounds (Aldonic acids).
Fructose (a ketose) is a reducing sugar because of enolization property
of the ketonic group. In the open chain form, fructose forms an enol
by movement of α-hydrogen (on C1). When the enol reverts to
carbonyl form, an aldose (glucose) may be formed. Thus, ketoses (the
α-hydroxyketones), in presence of alkali undergo ene–diol
rearrangement to aldoses and hence, reduce Fehling’s and Tollens
reagents.
In general, all monosacchsarides are reducing sugars irrespective of
aldose or ketose.

27.3.6 Osazone Formation: Reaction with


Phenylhydrazine
Glucose, mannose (aldohexoses), and fructose (ketohexose) on reaction with
excess phenylhydrazine result in the formation of the same osazone.
The reaction of aldoses and ketoses with phenylhydrazine is one of the most
important reactions in carbohydrate chemistry. The carbonyl group of aldose or
ketose reacts with one mole of phenylhydrazine to produce corresponding
phenylhydrazone. In a weakly acidic solution and in the presence of excess
phenylhydrazine, the formation of osazone takes place. This formation involves
use of three moles of phenylhydrazine in the following manner:
Generation of same osazone—an explanation
The formation of osazone is explained on the basis of mechanism proposed by
Weygand as discussed here.
(i) The first step involves the reaction of carbonyl group with phenylhydrazine
to yield phenylhydrazone.
(ii) The second step involves Amadori rearrangement where –CHOH group
at C2 in glucose is oxidized to keto group. Aniline is eliminated in the
process from glucose phenylhydrazone.
Similarly for fructose phenylhydrazone, the rearrangement causes oxidation of
–CH2OH group (C1) to aldehyde group (–CHO). In this process also,
elimination of aniline occurs.
(iii) The third step is the reaction of newly generated carbonyl groups with
second mole of phenylhydrazine followed by a reaction with the third mole
of phenylhydrazine, which involves removal of ammonia. The net result is
formation of glucosazone and fructosazone. The osazone so produced
contains two phenylhydrazino groups at C1 and C2.
The mechanism for generation of osazone using glucose and fructose can be
summarized as follows:
Since glucose and mannose are epimers and differ only in their configuration at
C2, they result in the formation of same osazone.

Why does the reaction stop beyond C2, that is, why does
secondary alcoholic group at C3 not react with phenylhydrazine?
This has been explained by ‘Fieser’ that osazone is stabilized through
chelation which prevents further reaction at C3.
The osazones, on hydrolysis in acidic medium, form osones (α-ketoaldehydes).
Alternatively, heating osazone with benzaldehyde can carry out its conversion to
osone. Benzaldehyde phenylhydrazone is highly stable which facilitates the
formation of osone as depicted in the following reaction:

Applications of osazone:
(1) Monosaccharides have a tendency to form syrups. The osazones have
characteristic crystalline strctures with sharp melting points. Being solids,
they are easily isolated and purified. Thus, identification of
monosaccharides can be carried out through osazones.
(2) Osazones are helpful in determining configuration of monosaccharides.
For example, hexose sugars, glucose, mannose, and fructose yield same
osazone with phenylhydrazine. Since osazone formation involves C1 and
C2 of aldose or ketose, these hexose sugars have same configuration at C3,
C4, and C5 stereocentres.
(3) Epimeric aldoses (which differ in their configuration at C2) also give same
osazones.
(4) An aldose can be converted to a ketose easily through osazone formation.

27.3.7 Interconversion Involving Aldoses and Ketoses


Conversion of D-glucose to D-fructose
[Conversion of aldose to isomeric ketose]
Glucose on reaction with three moles of phenylhydrazine gives glucosazone,
which on hydrolysis results in the formation of osone. Reduction of osone with
sodium amalgam causes faster reduction of aldehyde as compared to keto group
resulting in the formation of D-fructose.

Conversion of D-glucose to D-mannose


[Conversion of aldose to its epimer]
Glucose, on oxidation with bromine water forms gluconic acid, which on
treatment with pyridine results in the formation of epimeric aldonic acid. The
aldonic acid is then separated and dehydrated to form a lactone, which on
reduction with sodium amalgam forms D-mannose.

Conversion of D-fructose to D-glucose


[Conversion of ketose to isomeric aldose]
Fructose on hydrogenation in the presence of nickel undergoes reduction of keto
group to –CHOH group. Further oxidation results in the formation of
monocarboxylic acid. The dehydration of acid forms a lactone, which on
reduction with sodium amalgam results in the formation of D-glucose.

27.3.8 Configuration of Glucose


Glucose can be represented as:

It contains four stereogenic (chiral) centres. Thus, the first task is to assign
configuration to four stereocentres. For glucose, sixteen stereoisomers are
possible (2n, where n is the number of stereogenic centres and for glucose it is
24 = 16), which means eight pairs of enantiomers. [Fischer established the
configuration of glucose for which he received the Nobel prize.]
Fischer arbitrarily retained the structure for only those isomers, which had –OH
group on the right hand side. The configuration was established on the basis of
an aldopentose, i.e. arabinose.
In arabinose, four stereoisomers were retained that had –OH on right side (on
chiral centre farthest from –CHO).

(i) Arabinose, on oxidation with nitric acid results in the formation of


optically active dicarboxylic acid. This rules out structures I and III that
have –OH group at C2 on right hand side. These structures form optically
inactive dicarboxylic acids.

It indicates that in arabinose, the –OH at C2 should lie on the left.


(ii) Arabinose, on reaction with HCN followed by hydrolysis and then
reduction (Killiani–Fischer synthesis) results in the formation of (+) glucose
and (+) mannose. (+) Glucose and (+) mannose are epimers differing in
their configuration at C2; the rest of the configuration is same as in case of
arabinose. Thus, glucose and mannose may form epimeric pairs as (V and
VI) or (VII and VIII).
(iii) Both (+) glucose and (+) mannose on oxidation with nitric acid yield
optically active dicarboxylic acids. Thus, structures for (+) glucose and (+)
mannose may be either VII or VIII. Structure V on oxidation gives optically
inactive dicarboxylic acid and thus epimeric pair (V and VI) is ruled out.
The results indicate that (–) arabinose has –OH on right side at C3 and has the
following structure:

(iv) Fischer synthesized an aldohexose, known as (+) gulose. He further


established that chemically interchanging the –CHO and –CH2OH groups
in glucose result in the formation of (+) gulose. However, (+) mannose does
not form (+) gulose on such chemical interchange (Note: mannose and
glucose are epimers and differ in their configuration at C2). Thus, structure
VII represents glucose and structure VIII represents its epimer mannose.
27.3.9 Writing Haworth Formulae and Conformations
for Sugars
(A) Haworth formulae for 6-membered rings: Pyranose ring structures
Fischer projection formulae for the cyclic hemiacetal forms of sugar do not
portray the structure very well. A better way of depicting the hemiacetal forms of
sugars are Haworth formulae. The conventional means of writing the Haworth
formula for a six membered ring is to show the oxygen atom of the ring in the
back right hand corner of hexagon; the remaining corners of hexagon are
considered to be occupied by carbon atoms.
Thus, for any six membered ring hemiacetal, the initial structure is written as
A. The hemiacetal is formed between the aldehyde group, at C1, and the
.OH group on C5.
For all the members of the D-family of hexoses, the sixth carbon atom is
written attached to the fifth carbon in an upward position, as shown in
structure B.
For all the members of the L-family of hexoses, the sixth carbon atom is
written attached to the fifth carbon in a downward position, as shown in
structure C.

To write the complete Haworth formula,


All groups written to right of the Fischer projection formula are written
downwards
Whereas all groups on the left of the Fischer projections are written
upwards.

Fig. 27.4 Diagramatic representation of writing Haworth formula from Fischer projection for D-sugars.

Writing Fischer projection and Haworth formulae for D-Glucose: As


explained above the Fischer projection and the Haworth formulae for α-D-
glucose and β-D-glucose are as follows:
Understanding correlation of Fischer projection and Haworth formulae
(Why for D-sugars the —CH2OH is written upward from C5 in Haworth
formulae)
The Fischer projection I for D-glucose can be written as an equivalent Fischer
projection II, to show the proper orientation of —OH and —CHO for
cyclization. The rotation around C4—C5 bond in I has been depicted in IA. This
rotation results in Fischer projection II, where both —OH (at C5) and —CHO
(C1) are present along vertical line. In other words, the —OH and —CHO are
behind the plane and properly oriented for nucleophilic attack of —OH on the
carbonyl carbon (see structure IIA and III) to form pyranose ring. In Fischer
projection (II) of D-sugar the —CH2OH (C6) is on left-hand side thus, in
Haworth formulae of D-sugars the —CH2OH (C6) is written in upward
direction from C5 of pyranose ring.
In L-sugars, the —OH at C5 is on the left side in Fischer projection. With same
analogy, the rotation in Fischer projection of L-sugars will bring —CH2OH (C6) on the
right-hand side. Thus, for L-sugars the —CH2OH (C6) will be written in downward
direction from C5 of pyranose ring in Haworth projection.

Writing Fischer projection and Haworth formulae for D-Galactose and D-


Mannose: On the same analogy as discussed for D-glucose the equivalent
Fischer projection can be written for D-Galactose and D-Mannose.
D-galactose and D-glucose are C4 epimers as they differ in their configuration
at C4 only.
D-Mannose and D-glucose are C2 epimers as they differ in their configuration
at C2 only.
The Fischer projection and Haworth formulae for D-Galactose and D-Mannose
are as follows:
(B) Haworth formulae for 5-membered rings: Furanose ring structure
The conventional means of writing the Haworth formula for a 5-membered ring
is to show the oxygen on the top of pentagon, the remaining corners of pentagon,
are considered to be occupied by carbon atoms. Thus for any 5-membered ring
hemiacetal or hemiketal, the initial structure is written as Furanose.
Further,
all groups or atoms written to the Right side of the Fischer projection are
written downwards.
all groups or atoms written to the Left side of the Fischer projection are
written upwards.
Writing Fischer projection and Haworth formulae for D-Arabinose
(Furanose structure): D-Arabinose is an aldopentose. The hemiacetal
formation involves intramolecular cyclization between C1 and C4 to
form a 5-membered furanose ring.
(C) Conformations of D-hexoses
From Haworth formulae the chair conformations for hexoses can be written
directly.
In drawing chair conformation, the ring oxygen is written on the upper right
vertex.
The carbons of chair are numbered in clockwise direction from oxygen.
The axial and equatorial positions on each carbon are drawn.
Recall: Axial positions are upward and downward as per the vertex of the chair.
Also on each carbon the axial and equatorial position point in opposite
direction.
The groups or atoms written downward in Haworth formulae are placed in
downward direction in chair conformation (position may be axial or
equatorial).
The groups or atoms written upward in Haworth formulae are placed in
upward direction in chair conformation (position may be axial or
equatorial).
Haworth formulae and chair conformations of α-D-glucopyranose and β-D-
glucopyranose are shown as:

Why β-D-glucopyranose is more stable than α-D-glucopyranose?


From conformations of D-glucose that is, α-D-glucopyranose and β-D-
glucopyranose, it is evident that:
In chair conformation of β-D-glucopyranose the CH2OH and all –OH groups
occupy equatorial positions. Thus β-D-glucopyranose is more stable
compared to α-D-glucopyranose.
In chair conformation of α-D-glucopyranose the –OH group at C1 occupies
axial position. Chair conformations having substituent at axial positions
experience 1,3-diaxial interactions and are less stable (refer sec. 3.7.2).
Conformations in D-mannose and D-galactose

27.3.10 Evidence for Cyclic Structure of Glucose


Limitations of open chain structure
So far, we have seen that glucose is an aldohexose with the configuration that
has –OH group at right hand side on C2, C4, and C5 and left hand side at C3.
However, this structure does not explain the following facts about glucose:
(i) Glucose gives negative Schiff’s test and does not form sodium bisulfite
addition product. [Aldehydes give positive Schiff’s test and form sodium
bisulfite addition product].
(ii) It reacts with one mole of methanol to form acetal, that is,
methylglucoside. [Aldehydes react with two moles of alcohol to form
acetal.]
(iii) D (+) glucose exists in two isomeric forms which undergo mutarotation,
namely α-D-glucose and β-D-glucose (see Fig. 27.3).
(iv) Glucose reacts with methanol to form methyl α-D-glucoside and methyl
β-D-glucoside. (Fig. 27.4) which do not show mutarotation and do not
reduce Fehling solution and Tollens reagent.
All these facts indicate that glucose exists not as an open chain aldehyde but as a
cyclic hemiacetal. The cyclic hemiacetal formation occurs by the intramolecular
nucleophilic addition of the hydroxy group at C5 to carbonyl carbon (C1).
The cyclic structure of glucose is best represented by Haworth projection as a
pyranose ring (name derived from pyran, a six membered cyclic ether).
Proof for pyranose ring structure of glucose
One of the important aspects of hemiacetal structure of glucose is that it may
exist as a cyclic six-membered ring (pyranose) or as cyclic five-membered ring
(furanose) structure. In pyranose form, C1 and C5 are involved in hemiacetal
formation whereas in furanose form C1 and C4 are involved. Oxidation with
periodic acid helps in determining the ring size.
Glucose on mono methylation with CH3OH and HCl gives methyl glucoside (α-
or β-). This methyl glucoside on treatment with periodic acid liberates formic
acid and a dialdehyde. The formation of formic acid is possible only if glucose
exists in pyranose ring structure and it serves as a proof for the same. The
dialdehyde on oxidation with bromine water followed by acid hydrolysis yields
1 mole of glyoxalic acid and glyceric acid.
If glucose were in the furanose ring form, methyl glucoside would have given
formaldehyde (not formic acid) on treatment with periodic acid as depicted in
the following reaction:
27.4 D(-) FRUCTOSE: CHEMICAL PROPERTIES
AND STRUCTURE
It is a ketohexose (molecular formula C6H12O6), which occurs in sweet fruits
and honey. It does not crystallize readily. Fructose is the sweetest of all sugars.
Fructose is an optically active laevorotatory (–) sugar and thus also termed as
laevulose.
Commercial ly, fructose is synthesized by hydrolysis of ‘Inulin’ (a
polysaccharide) in acidic medium. It can also be prepared by hydrolysis of
starch.

The chemical reactions of fructose are discussed as follows:


(i) Reduction of fructose. In presence of H2/Ni, the reduction of fructose results
in the formation of sorbitol and mannitol (C6H14O6) which on heating with
hydrogen iodide result in complete reduction of fructose to yield n-hexane. This
clearly indicates that six carbons in fructose are in a straight chain.
(ii) Reactions with
(a) Hydroxylamine and phenylhydrazine. The reaction of fructose with
hydroxylamine results in the formation of oxime. On the other hand, the
reaction with one mole of phenylhydrazine yields fructose
phenylhydrazone. This indicates the presence of a carbonyl group in
fructose.
(b) HCN. Fructose reacts with HCN to produce a cyanohydrin, which on
hydrolysis followed by treatment with HI yields 2-methylhexanoic acid.
This indicates the presence of a carbonyl group in fructose as keto group at
C2.
(iii) Acetylation of fructose. Fructose on reaction with acetic anhydride gives a
very stable pentaacetyl derivative.
The pentaacetyl derivative indicates that it contains five hydroxyl groups and
since pentaacetyl derivative is very stable that indicates five hydroxyl groups
must be present on different carbons.
(iv) Oxidation of fructose:
(a) With bromine water. Fructose does not undergo any reaction with
bromine water.
(b) With concentrated nitric acid. Fructose gives a mixture of tartaric acid,
trihydroxy glutaric acid, and glycolic acid on reaction with conc. HNO3.
The acids obtained contain lesser number of carbons than fructose. This
further confirms the presence of keto group.
The reactions discussed in (i)–(iv) establish that fructose has a six carbon
straight chain and is 2-ketohexose sugar. Its structure can be represented as:

The various reactions discussed above are summarized in the reaction sequence
shown in Fig. 27.5.
Fig. 27.5 Reactions of fructose.
(v) Reaction of fructose with excess of phenylhydrazine (formation of
osazone)
Fructose reacts with excess phenylhydrazine to form osazone, which is same as
the osazone formed with glucose. Thus, fructose has the same configuration at
C3, C4 and C5 carbons as that of glucose.

Since the configuration of glucose has already been discussed, the configuration
of fructose can be written on the same lines as follows:

27.4.1 Cyclic Structure of Fructose


Fructose, like glucose also occurs in cyclic form. The intramolecular cyclization
occurs between C2 and C6. Fructose occurs as six membered rings (that is,
pyranose form) as α-D-fructopyranose and β-D-fructopyranose. In free state,
fructose occurs in pyranose form, however, in combined state it exists as
furanose, namely α-D-fructofuranose and β-D-fructofuranose. The Fischer and
the Haworth projections for the α-D and β-D forms are shown in Fig. 27.6.
Fig. 27.6 (a) Six membered cyclic hemiketal formation in fructose (fructopyranose) and, (b) five membered
cyclic hemiketal form in fructose (fructofuranose). Both forms are shown in Fischer and Haworth
projection.
A Word About
Deoxy and amino sugars—The monosaccharide derivatives
The aldose sugars where one or more hydroxy groups are replaced by
hydrogen atoms are known as deoxy sugars. One of the important
deoxy sugars is 2-deoxy-D-ribose sugar, which is present in DNA.
The other examples include ‘Rhamnose’ and ‘Digitoxose’.

The monosaccharides in which an amino group replaces one of the


alcoholic groups are known as amino sugars. The two most common
amino sugars are D-glucosamine and D-galactosamine. The amino
sugars are present in the cell membrane. The first amino sugar to be
isolated was D-glucosamine and was obtained from lobster shells.
27.5 DISACCHARIDES
The disaccharides, on hydrolysis break down into two monosaccharide units,
which may be same or different. A disaccharide can be formed by the reaction of
two monosaccharide molecules of which one serves as the hemiacetal and the
other as alcohol. The two monosaccharide units are joined through glycoside
linkage. Glycosides are the cyclic sugar derivatives where the hemiacetal or
hemiketal –OH group at the anomeric centre of the free sugar is replaced by –
OR group (R may be an alkyl, aryl, etc.). The glycosidic linkage may involve:
(1) The hemiacetal –OH group (C1 in aldose and C2 in ketose) of one
monosaccharide unit and alcoholic –OH group of another monosaccharide
unit.
Disaccharides formed as a result of these linkages are reducing in nature. The
hemiacetal form exists in one of the units, which provides reducing
character. These disaccharides are named, systematically, as glycosyl
glycose. The non-reducing unit is cited as glycosyl where as the unit with a
free hemiacetal group is cited as glycose. The examples of reducing
disaccharides are maltose and lactose.
(2) The hemiacetal –OH group of one monosaccharide unit combining with
hemiacetal –OH of another monosaccharide unit.
Disaccharides formed by such linkages are non-reducing as none of the ring after
linkage can occur in hemiacetal form, for example, sucrose. These disaccharides
are named, systematically, as glycosyl glycosides. The parent unit is cited as the
‘glycoside’. For parent unit aldose gets preference over ketose.

27.5.1 Sucrose
Sucrose is the most common and familiar disaccharide (C12H22O11), also
known as table sugar. Commercially, sucrose is obtained from sugar cane and
sugar beets. It is also present in varying concentrations in most fruits, flowers,
seeds, and roots. On hydrolysis, sucrose gives a molecule of D-glucose and D-
fructose each.
Sucrose has α-D-glucopyranose ring attached through its C1 to C2 of β-D-
fructofuranose. It is a non-reducing sugar (1–2 linkage) and does not undergo
mutarotation.

Invert Sugar
Invert sugar is a mixture of glucose and fructose in equal amounts
resulting from the hydrolysis of sucrose. Sucrose is dextrorotatory
having specific rotation, αD= +66.5°. while its constituent
monosaccharides have specific rotation αD= +53° (for D-glucose) and
αD= –92° (for D-fructose). Since D-fructose has higher specific
rotation than D-glucose, the resulting mixture obtained by the
hydrolysis of sucrose rotates plane-polarized light to the left
(laevorotatory; opposite from the direction of rotation of sucrose).
Due to this, the mixture is known as invert sugar.

27.5.2 Lactose
Lactose is a disaccharide found solely in milk of mammals and is commonly
known as milk sugar. On hydrolysis, it yields D-glucose and D-galactose. The
D-galactose is present as β-D-galactopyranose and D-glucose as D-
glucopyranose. The lactose has β-D-galactopyranose ring attached through its
C1 to C4 of D-glucopyranose (α or β anomer). It is a reducing sugar.

The conformation and nomenclature of lactose is as follows:


27.5.3 Maltose and Cellobiose
Maltose is a disaccharide, which on hydrolysis yields two units of glucose. In
maltose, the α-D-glucopyranose ring is joined through its C1 to C4 of D-
glucopyranose ring (α or β anomer). It is a reducing sugar and undergoes
mutarotation.

The conformation and nomenclature of maltose is as follows:

Cellobiose is a disaccharide, which on hydrolysis yields two units of glucose. In


cellobiose, the β-D-glucopyranose ring is joined through its C1 to C4 of D-
glucopyranose ring (α or β anomer). It is a reducing sugar and undergoes
mutarotation.

The conformation and nomenclature of cellobiose is as follows:

BARFOED’S TEST
Distinguishing Between Monosaccharides and Reducing
Disaccharides.
Monosaccharides and reducing disaccharides can be distinguished by
Barfoed’s test. This test takes the advantage of the fact that copper (II)
ions are weaker oxidizing agents in acid solution than in basic
solution. The test solution is prepared by dissolving copper (II) acetate
in acetic acid. Monosaccharides yield a red precipitate of Cu2O
within 10 minutes of warming in the solution.
On the other hand, reducing disaccharides do not react under normal
conditions of Barfoed test. Upon prolonged heating, they produce a
positive test because of hydrolysis that yields monosaccharides in
acidic solution.
27.6 POLYSACCHARIDES
A polysaccharide consists of a number of monosaccharide units joined through
glycosidic linkages. The monosaccharide units may be same
(homopolysaccharides) or different (heteropolysaccharides). A polysaccharide
consisting of glucose units only is known as glucan. Few important examples of
polysaccharides are starch, cellulose, and glycogen.
Starch is one of the important polysaccharides, which is present in the roots ,
seeds, and tuber of plants. It is made of a number of D-glucose units and on
hydrolysis, results in the formation of amylose and amylopectin
(polysaccharides). In amylose, the C1 of one glucopyranoside ring is attached to
C 4 of another glucopyranoside ring. In this manner, thousands of
glucopyranosides rings are joined together linearly to form a compact molecule
(Fig 27.7a). Amylopectin also contains a number of glucopyranoside rings
joined through 1,4-linkage as in case of amylose. However, in this case
branching takes place between C1 and C6 of different glucose units (Fig. 27.7b).
In general, the branching occurs at an interval of 20 glucose units. The
hydrolysis of starch can be represented as follows:

Amylose is water-soluble component of starch whereas amylopectin is water


insoluble component. Starch is insoluble in cold water however on heating with
water it forms a gelatinous mass due to swelling up of amylopectin. Sarch gives
an intense blue colour with iodine. Amylose molecules are present as a helical
coil and iodine molecules get inserted within the coil to form a blue starch iodine
complex (inclusion complex) as shown in Fig. 27.7c.
Starch is easily hydrolyzed by dilute solutions of strong acids to form glucose.
Glycogen. Glycogen is a polysaccharide that is present in the liver and muscle
tissues of animals as a ‘reserve carbohydrate’. Its acid hydrolysis results in the
formation of D-glucose. Structurally, glycogen resembles amylopectin, however,
it is a highly branched chain polymer of glucose and branching occurs at every 8
to 12 glucose units. Glycogen serves as reserve carbohydrate in body. The
excessive glucose in the body is stored in the form of glycogen that has large
molecular weight. The stored glycogen, when required, gets converted to
glucose. Glycogen is also referred to as ‘animal starch’.
Cellulose. Cellulose is the most abundant and commonly occurring
polysaccharide. It is the major constituent of plant cell walls. The pure natural
sources of cellulose are cotton, jute, flax, and hemp. Cellulose on hydrolysis
yields D-glucose. It has a structure similar to starch but in cellulose the glucose
units are joined through 1,4-β-linkages (refer Fig. 27.7d).
Cellulose is used extensively in manufacturing paper, cellulose fibres, rayon
(fibre with silk like texture), celluloid, cellophane, lacquers, solvents, detergents,
adhesives, explosives (Gun Cotton-fully nitrated ester of cellulose [C6H7O2
(NO3)3]n), and so on.
Fig. 27.7 Different polysaccharides. (a) and (b) are starch amylose and amylopectin: (c) starch-iodine
complex: and (d) cellulose.

NOTABILIA 17
EXERCISES
1. What are carbohydrates? Give their classification.
2. Many carbohydrates are known by their common names. Give the name of
the carbohydrates whose common names are given below:
(a) Cane sugar
(b) Grape sugar
(c) Fruit sugar
(d) Malt sugar
(e) Milk sugar
(f) Table sugar
3. What are D- and L-sugars?
4. Define reducing and non-reducing sugars?
5. What is mutarotation?
6. How will you establish the structure of glucose?
7. Carry out the following conversions:
(a) Glucose to n-hexane
(b) Glucose to fructose
(c) Arabinose to glucose
(d) Glucose to sorbitol
(e) Arabinose to erythrose
(f) Glucose to mannose
8. Explain the following:
(a) Unlike aldehydes, glucose does not form a bisulfite addition product with
NaHSO3.
(b) Sucrose, a disaccharide is a non-reducing sugar while maltose (also a
disaccharide) is a reducing sugar.
(c) The penta-acetylated product of α-D-glucopyranose does not give Tollens
test.
(d) Glucose, fructose, and mannose yield the same osazone in excess phenyl
hydrazine.
(e) Glucose reacts with one mole of CH3OH to form an acetal.
(f) Starch does not reduce Benedict’s solution.
9. Complete the following:
10. Give the chemical reactions that support the fact that glucose has a cyclic
structure.
11. What is the difference between epimers and anomers. Explain with
examples.
12. What are glycosides? Describe a glycidic linkage?
13. Write the Haworth projections for the sugars whose Fischer projections is
given as follows:

14. How can one determine the configuration of glucose? How is the
configuration of D-glucose related to that of D-glyceraldehyde?
15. What is ‘invert sugar’?
16. How can one establish the structure of fructose?
17. Why do both glucose and fructose give positive Tollens and Fehling tests?
18. Write the structure, using Haworth formula, for
(a) α-D-glucopyranose
(b) β-D-glucopyranose
(c) α-D-fructopyranose
(d) β-D-fructopyranose
(e) α-D-fructofuranose
(f) β-D-fructofuranose
19. In sucrose, fructose is present as fructopyranose or fructofuranose?
20. Disaccharide ‘trehalose’ is found in mushrooms and has the following
structure.

(a) What monosaccharide units will be obtained on acid hydrolysis?


(b) Name the glycosidic linkage through which two monosaccharide units are
joined
(c) Explain whether ‘trehalose’ is a reducing or non-reducing sugar.
21. What are deoxy sugars? Give two examples along with their structures.
22. What are amino sugars?
23. How do starch and cellulose differ in their structures?
24. Why does starch give a blue colour with iodine?
Chapter28
Amino Acids,
Peptides and Proteins

28.1 INTRODUCTION
Amino acids are carboxylic acids having an amino group. The naturally occuring
prevalent amino acids are α-amino acids. These are the carboxylic acids where
‘amino’ functional group is present at α-carbon.
In general, all amino acids have an amino (–NH2) group*, carboxylic (–COOH)
group, and a hydrogen atom attached to carbon along with an side chain. Amino
acids differ in their side chains. The variation in the nature of side chain imparts
characteristic properties to amino acids. Amino acids form polymers known as
peptides and proteins.
Amino acids can be represented by the general formula RCH(NH2)COOH as.

Though a large number of amino acids (more than 500) exist in nature, however,
proteins in all living organisms mainly consist of twenty amino acids. A protein
molecule may contain several hundreds or thousands of amino acid units joined
in different combinations. As an amino acid contains an amine* and carboxylic
group, it exhibits both acidic as well as basic behaviour.
28.2 CLASSIFICATION OF AMINO ACIDS
Amino acids may be classified
(1) Based on the chemical reactions they undergo in solution form as
(i) acidic, (ii) basic, and (iii) neutral
(2) Based on their nutritional importance as
(i) essential, (ii) semi-essential, and (iii) non-essential
Table 28.1 enlists the natural amino acids along with their classification,
structure, and abbreviations (three letter and one letter code).
Table 28.1 Natural animo acids
Classification based on the chemical nature
Acidic amino acids. These amino acids contain, in their side chain, a carboxylic
or carboxamide group. Examples are—aspartic acid, glutamic acid, glutamine,
and asparagine.
Basic amino acids. These amino acids contain, in their side chain, a basic group
such as an amino or an imidazole ring (a heterocyclic group). Examples are—
arginine, lysine, hydroxylysine, and histidine.
Neutral amino acids. The largest number of amino acids belong to neutral
amino acids. These neutral amino acids are further classified as:
(i) Aliphatic neutral amino acids. The side chain of these amino acids
contains either hydrogen, or an alkyl group, or hydroxyalkyl group.
Examples of aliphatic neutral amino acids are glycine, alanine, valine,
leucine, isoleucine, serine, and threonine.
(ii) Aromatic neutral amino acids. The side chain of these amino acids
contains an aromatic ring. For example, phenylalanine, and tyrosine.
(iii) Sulfur containing neutral amino acids. The side chain of these amino
acids contains thiol group (–SH) such as in cysteine or methylthio group (–
SCH3) such as in methionine, or a disulfide linkage such as in cystine.
(iv) Heterocyclic neutral amino acids. The side chain of these amino acids
contains a heterocyclic group such as indole or pyrrolidine. For example,
tryptophan, proline, and hydroxypyroline. In fact, proline and
hydroxyproline are α-imino acids.
Classification based on nutrition importance
(i) Essential amino acids. The amino acids that are not synthesized by body and
are required to be taken externally through diet are termed as essential amino
acids. Valine, leucine, isoleucine, phenylalanine, threonine, tryptophan,
methionine, and lysine, are some of the essential amino acids.
The amount of these amino acids required by our body is 1.1–0.25 g per day.
These amino acids are synthesized industrially and added to foodstuff.
(ii) Semi-essential amino acids. These amino acids are not synthesized in
human body in sufficient amount during growth. However, these are essential in
pregnant woman, lactating mothers, and in growing children. These amino acids
have an important role as growth promoting nutrients. Arginine and histidine fall
under this category.
(iii) Non-essential amino acids. These amino acids can be synthesized by
human body from other materials through a series of biochemical reactions. It is
therefore, not necessary that they are consumed through diet. Ten amino acids
(see Table 28.1) have been classified under this category.
28.3 STEREOCHEMISTRY OF AMINO ACIDS
Except glycine, all amino acids contain a stereocentre (chiral centre) and are thus
optically active.

The amino acids may be dextro (+) or laevo (–) rotatory depending upon the pH
of the solution and nature of the side chain. Except glycine, rest of the amino
acids may occur in D and L forms.

Natural amino acids occur in L-form. The natural proteins obtained from
animals and plants, in general, contain L-amino acids. Bacteria contain D-amino
acids. The L-amino acids exhibit
S-configuration.

Exceptions to this are cystine and cysteine, where sulfur is present in the side
chain (R) and thus in assigning the configuration, the side chain (containing
sulfur) gets higher priority over –COOH group, (R > COOH).
28.4 PHYSICAL PROPERTIES
Zwitterions and Isoelectric Points
Amino acids exhibit the following properties:
(a) These are high melting, colourless crystalline substances.
(b) Highly soluble in water as compared to other polar solvents.
(c) Large dipole moment and high dielectric constant value.
Thus, they resemble ionic compounds. This occurs by an internal, self-
neutralization reaction so highly favoured that amino acids exist in the zwitterion
form or inner salt (Zwitterion: a compound containing equal numbers of positive
and negative charges).

In strong acidic solutions (low pH), the amino acid exists as ammonium
carboxylic acid. In strong basic solutions (high pH), amino acids exist as amino
carboxylate ions.

In zwitterion form, the molecule has no net charge and is said to be isoelectric.
Zwitterion formation is dependent on the pH of the solution. The pH at which
an amino acid exists in its zwitterion form is called the isoelectric point
(symbolized as pI) for that amino acid. Thus, each amino acid has a
characteristic isoelectric point depending upon the nature of its side chain.

28.4.1 Electrophoresis
Electrophoresis is a method used for the separation and analysis of amino acids.
The method is based on pH control and electric charge. The amino acids differ in
their isoelectric points. The mixture of amino acid is placed on the centre of a
paper strip (cellulose acetate) at certain pH. The pH is maintained by saturating
the paper strip with the buffer solution. The paper strip is attached to two
electrodes. On passing electric current through the strip, amino acids migrate
towards electrodes depending upon the net charge present on them. For example,
a mixture of lysine, alanine, and glutamic acid at pH = 6.0, when placed on a
paper strip, on passing current results in the movement of glutamic acid towards
anode and of lysine towards cathode. The alanine has no net charge and
therefore does not move (Fig. 28.1). The amino acid with isoelectric point
greater than the buffer pH gains a proton and becomes positively charged
(Lysine, pI = 9.7) and thus moves towards cathode. On the other hand, the amino
acid with isoelectric point lower than the buffer pH looses a proton and becomes
negatively charged (Glutamic acid, pI = 3.2) and thus, moves towards anode.
The amino acid with isoelectric point comparable to buffer pH does not migrate
towards any electrode, such as Alanine, pI = 6.0.

Fig. 28.1 Separation of amino acids by electrophoresis. (a) At pH = 6.0, Glutamic acid is negatively
charged and moves towards anode. Lysine at this pH is positively charged and moves towards
cathode. Alanine remains at the same position, as at pH = 6.0 it has no net charge (b) Separation of
mixture of amino acids after electrophoresis.
28.5 SYNTHESIS OF AMINO ACIDS
The amino acids can be synthesized by a number of methods. However, a single
method cannot be used for preparation of all amino acids. The different methods
used extensively for preparation of different kinds of amino acids are as follows:
(1) Direct ammonolysis of α-haloacids. The carboxylic acids can be converted
to α-chloro or
α-bromo acids by Hell–Volhard–Zelinsky method (HVZ reaction) using chlorine
or bromine in the presence of red phosphorous. The α-bromo or α-chloro acids
on treatment with excess of ammonia result in the substitution of chlorine or
bromine by –NH2 to give α-amino acid.
Glycine, alanine, valine, leucine, and aspartic acid can be prepared by this
method.

(2) Gabriel phthalimide synthesis. Phthalimide is converted to its reactive salt


by reaction with ethanolic KOH. The potassium salt of phthalimide on reaction
with α-halo acids (α-haloester is preferred), followed by hydrolysis results in the
formation of α-amino acids.
This method results in higher yield of amino acids as compared to that obtained
by the amination of α-halo acids. Glycine and leucine are best prepared by this
method.
(3) Strecker synthesis. An aldehyde on reaction with KCN in the presence of
ammonia or ammonium chloride results in the formation of α-aminonitrile.
Hydrolysis of α-aminonitrile in acidic medium yields an α-amino carboxylic
acid. The overall reaction is known as Strecker synthesis, which basically
involves conversion of an aldehyde to α-amino acid having one more carbon
atom than the parent aldehyde.

(4) Phthalimido malonic ester synthesis. This method is a modification of


Gabriel phthalimide method and involves reaction of potassium phthalimide
with α-bromomalonic ester to
produce phthalimidomalonic ester. The sodium salt of ester on reaction with an
α-haloester
followed by hydrolysis results in the formation of α-amino acid. This reaction is
used for the synthesis of acidic amino acids and hydroxy amino acids such as
serine, glutamic acid and aspartic acid.
The methods described so far yield racemic mixture of amino acids. To obtain
optically active enantiomers of amino acid, resolution of mixture is carried out in
the presence of a chiral reagent.
In nature, the synthesis of amino acid occurs with 100% enantioselectivity.
28.6 CHEMICAL PROPERTIES OF AMINO ACIDS
The amino acids show reactions of amino as well as carboxylic acid groups.
However, other functional groups present in side chain of amino acid may
exhibit their characteristic reactions.
This section briefly describes some important reactions of amino acids.

28.6.1 Reactions Due to Amino Group


(i) Reaction with mineral acid (formation of salts)

(ii) Reaction with acid chlorides or anhydrides (formation of acyl


derivatives). The amino group of amino acids reacts readily with acid chlorides
like benzoyl chloride to form N-benzoyl amino acids. This reaction takes place
as follows:

(iii) Reaction with nitrous acid (formation of α-hydroxy carboxylic acids).


Similar to primary aliphatic amines the amino group of amino acids on reaction
with nitrous acid liberates nitrogen to form α-hydroxy carboxylic acid.
(iv) Reaction with formaldehyde (formation of methylene amino acids). The
amino group
(–NH2) of amino acids reacts with formaldehyde with the elimination of water
molecule, resulting in the formation of methylene amino acids.

The reaction is used for estimation of amino acids by direct titration with alkali.
This method is known as Sorensen’s Formol Titration Method. The direct
titration of amino acid with alkali cannot be carried out because of the presence
of free –NH2 group. Reaction with formaldehyde converts amino acid to
methylene amino acid, which is a strong acid and can be titrated directly with an
alkali.
(v) Oxidation. Potassium permanganate or hydrogen peroxide oxidizes the
amino group to imino group. The hydrolysis of imino acid, so produced, results
in the formation of a α-keto acid. In this process, ammonia is evolved.

28.6.2 Reactions Due to Carboxylic Group (–COOH)


(i) Reaction with alcohol (formation of ester). The amino acids react with
alcohol to produce ester hydrochloride. The treatment with sodium carbonate
solution releases free ester.
The amino acid ester may react with ammonia to release alcohol and amino acid
amide.

(ii) Decarboxylation of amino acids (formation of amines). The dry


distillation of amino acids or heating with Ba(OH)2 results in the liberation of
CO2 from carboxylic group to produce
amine.

(iii) Chelation with copper. The salts of amino acids with heavy metals are
chelate compounds. An aqueous solution of glycine on heating with copper
oxide form deep blue colour chelate compound.

(iv) Reduction. The α-amino acid on reduction with lithium aluminium hydride
undergoes reduction of carboxylic group to alcohol group. The product of
reduction is an amino alcohol.
28.6.3 Reactions Due to Both –NH2 and –COOH
Groups
(i) Effect of heat. Two molecules of α-amino acids, on heating undergo
intermolecular
dehydration to form 2,5-diketopiperazine derivatives. For example, glycine on
heating yields
2,5-diketopiperazine.

However in case of β-amino acids, heating results in α,β-unsaturated acids with


elimination of ammonia. For example,

(ii) Ninhydrin reaction. This reaction is used for the identification of amino
acids as amino acids on reaction with ninhydrin give a blue violet colour.
Ninhydrin is a hydrate of indane-1,2,3-trione. The steps involved in the reactions
are as follows:
(iii) Reaction with isocyanates (formation of hydantoin). The reaction of α-
amino acids with isocyanates results in the formation of substituted urea
derivatives (carbamides), which on heating in acidic medium result in the
formation of hydantoin derivative.
28.7 PEPTIDES AND THEIR SYNTHESIS
The interaction of amino group of one amino acid with carboxylic acid group of
another amino acid results in the formation of an amide linkage. A number of
amino acid units are joined with each

other through linkages, also known as peptide linkage. In accordance


with the number of amino acid units present, a peptide may be classified as:
(1) Oligopeptide (2–9 amino acids units)
(2) Polypeptide (10–100 amino acid units)
(3) Protein (also referred as macro peptides; >100 amino acid units)
The peptide with two amino acids is termed as dipeptide. Similarly, the amino
acid with three amino acid units is called a tripeptide and with four amino acids
tetrapeptide, and so on.

In peptide formation:
• There is a free amino group at one end of the molecule known as N-
terminal end and a free carboxylic acid group at another end
known as C-terminal end.
• It is a convention to name a polypeptide starting from the N-
terminal amino acid to be followed by the sequence of other
amino acids and ending with the C-terminal amino acid.
• While writing the structure of a polypeptide, the N-terminal is
written at the left and C-terminal at the right.
Let us consider the synthesis of peptides by taking example of combination of
glycine and valine (without repetition of either amino acid) which can result in
the formation of two different dipeptides.
(a) Carboxylic group of glycine reacting with amino group of valine, that is,
amino group of glycine (as N-terminal) is free and carboxylic group (as C-
terminal) of valine is free. The dipeptide thus formed is named as
Glycylvaline.
(b) Carboxylic group of valine reacting with amino group of glycine, that is,
amino group of valine (as N-terminal) is free and carboxylic group (as C-
terminal) of glycine is free. The dipeptide thus formed is called as
Valylglycine.

In a similar manner, three different amino acids—alanine (Ala), valine (Val), and
leucine (Lys) can form six different tripeptides (without repetition of any amino
acid), which are as follows:

With an increase in the number of amino acids, the possible combinations of


polypeptides also increase tremendously. If an amino acid is used only once, 20
different amino acids can have
2.4 × 1018 combinations. Nature, in fact, uses these 20 amino acids as building
blocks to create variety of proteins, which perform diversified functions.
28.7.1 Use of Protecting Groups in Synthesis of
Polypeptides
In the synthesis of peptides, care must be taken to prevent the interaction of
amino group of an amino acid with carboxylic group of the same amino acid.
For example, when dipeptide glycylvaline is synthesized from glycine and valine
amino acids, the following possibilities are observed to exist:

To exculsively synthesize glycylvaline, the –NH2 group of glycine and –COOH


group of valine must be protected so that they are not freely available to react at
these junctions. This is discussed in the following sections:
Protection of amino group (–NH2)
The amino end is protected or blocked by using the following groups:
(i) Phenylmethoxycarbonyl group (Carbobenzoxy; Cbz). This group is
commonly written in abbreviated form as Cbz group. The carbobenzoxy group
is introduced into amino acid by reaction of amino acid with phenylmethyl
chloroformate (benzylchloroformate).

The deprotection of Cbz group is carried out by hydrogenolysis in the presence


of palladium (H2/Pd), which first forms a carbamic acid that readily undergoes
decarboxylation to give free amino group.
(ii) 1,1-Dimethylethoxycarbonyl group (tert-butoxycarbonyl; BoC). This
group is abbreviated as BoC and is introduced into amino acid (for blocking the
amino group) by reaction of amino acid with bis-1,1-dimethylethyl dicarbonate
(di-tert-butyldicarbonate).

The reverse process or deprotection of tert-butoxycarbonyl group (BoC) is


carried out by treating the protected amino acid with hydrogen bromide in the
presence of acetic acid or by treating it with trifluroaceticacid (CF3COOH).

(iii) Phthaloyl group. The phthaloyl group is introduced into amino acid (for
blocking —NH2 group) by the reaction of amino acid with phthalic anhydride.
The deprotection of phthaloyl group is carried out using hydrazine follwed by
treatment with HCl.
Protection of carboxylic group (–COOH)
Ester groups, for example methyl, ethyl, allyl, or benzyl ester protect the
carboxylic end of amino acids.

Treating the protected amino acid with base (aq. NaOH) carries out deprotection
at the carboxylic group. The benzylic esters are deprotected by catalytic
hydrogenolysis (H2/Pd) to give toluene and free carboxylic group.

28.7.2 Synthesis of Peptides Using Protected Amino


and Carboxylic Acid Ends
Formation of peptide linkage: use of DCC
Once amino group and carboxylic group of amino acids are protected, the
peptide formation is carried out by converting carboxylic group of N-terminal
protected amino acid to its acyl derivative. A number of such methods are
available which cause the activation of carboxy group. The most important and
widely used method is to involve DCC (dicyclohexylcarbodiimide). DCC
activates the carboxylic carbon so that the nucleophilic attack of amino group of
another amino acid occurs readily at carboxylic carbon. DCC acts as a
dehydrating agent.

Now peptide chain is obtained by carrying out the deprotection. (Same


procedure is used for the synthesis of peptides using more number of amino
acids.) Thus, we can summarize the synthesis of dipeptide glycylvaline (gly-val)
in the following steps:
Step 1. Protection of N-terminal and C-terminal of amino acids

Step 2. Peptide formation in presence of DCC


Step 3. Deprotection of terminals

28.7.3 Solid Phase Polypeptide Synthesis


[Merrifield solid phase polypeptide synthesis]
The synthesis of polypeptide chain by adding one amino acid at a time, in a
sequence, to the growing peptide chain is a time consuming process. This
involves the purification of peptide at each and every addition of amino acid.
This major hurdle can be overcome by using the technique of solid phase peptide
synthesis as developed by Robert B. Merrifield. Merrifield synthesized
Ribonuclease enzyme containing 124 amino acids in just six weeks.
The Merrifield method involves the use of a solid support, which consists of
beads of a copolymer. The copolymer is synthesized from reaction of styrene
with 4-(chloromethyl)
styrene.
The synthesis (Fig. 28.2) is carried out in the following steps:
Step 1. The N-terminal protected (BoC) amino acid is attached to the copolymer
through ester bond formation between –COOH of amino acid and –CH2Cl of the
copolymer.
Step 2. The copolymer attached to amino acid is washed to remove excess
reagents. The reaction with trifluoroacetic acid (CF3COOH) is then carried out
to remove BoC protecting group so that a free amino group is available.
Step 3. Another amino acid (amino group protected with BoC) is then added to
copolymer in the presence of DCC. The free amino group of amino acid attached
to polymer forms peptide linkage with free carboxylic group of second amino
acid.
Step 4. Copolymer and the attached dipeptide is again washed to remove excess
reagent for the addition of another amino acid. Steps 2 and 3 are now repeated.
These steps can be repeated a number of times depending upon the length of
polypeptide chain.
Step 5. Once the required polypeptide is synthesized by linking of different
amino acids the polypeptide is released from the copolymer by reaction with
hydrofluoric acid (HF). Hydrofluoric acid performs two functions:
(a) It cleaves the ester bond and leaves the peptide chain intact
(b) It removes the N-terminal protecting group, namely BoC.
The Merrifield solid phase peptide synthesis is now-a-days carried out with
automated peptide synthesizer which automatically undertakes the washing and
removal of protecting group at each addition step.
28.8 DETERMINATION OF SEQUENCE OF
AMINO ACIDS IN A GIVEN POLYPEPTIDE
One of the most challenging works in the study of amino acids is to determine
the constituent amino acids in a polypeptide and the sequence in which these are
arranged. The determination of sequence of amino acids in polypeptide involves
the following steps:
(1) A polypeptide (protein) is isolated and purified. Purification of a
polypeptide may be carried out by different techniques, for example,
dialysis, gel filtration chromatography, ion-exchange chromatography,
electrophoresis, and so on. The electrophoresis is highly efficient and
powerful technique for separation and purification.
(2) The constituent amino acids of a polypeptide are determined by hydrolysis
of polypeptide chain. The entire chain is degraded by hydrolysis in the
presence of an acid (6N HCl) and this results in the formation of a mixture
of amino acids.
(3) The qualitative and quantitative estimation of amino acids is carried out
with the help of an automated amino acid analyzer. The basic principle
involved here is ion-exchange chromatography. In the process, the acidic
amino acids are released first and the most basic amino acid are released in
the end.
(4) The next step involves the identification of N- and C-terminals, that is, end
group
analysis.
(5) Once the end groups are identified, stepwise degradation of peptide chain
is carried
out which helps in determining the order of amino acids (sequence) in a
chain.
The process involves the non-enzymatic hydrolysis of polypeptide into
smaller
fragments.
The last two steps, namely end group analysis and sequence determination of
polypeptides are now discussed in detail.

28.8.1 End Group Analysis


The N-terminal contains a free amino group and the C-terminal contains a free
carboxylic group. Thus, amino acids present at two terminals are entirely
different from each other as well as from other amino acids present in a chain.
N-terminal analysis
The N-terminal residue can be identified by following three methods:
(i) DNP method. This method was developed by F. Sangar. The peptide chain is
treated with
2,4-dinitrofluorobenzene (DNFB), which causes substitution of DNFB by free
amino group to give a N-Dinitrophenyl (DNP) derivative. The hydrolysis of
peptide chain results in the separation of labelled N-terminal amino acid (DNP-
amino acid) and a mixture of another amino acids. Thus,
DNP-amino acid so separated is identified to determine the N-terminal.

(ii) Dansyl method. In this method the reaction of peptide chain with 5-
dimethylaminonaph-thalene-1-sulfonylchloride (Dansyl chloride) occurs at the
N-terminal (amino end). The hydrolysis of peptide chain results in the formation
of Dansyl amino acid. This Densyl amino acid on irradiation with UV light
shows fluorescence. Thus, the N-terminal is identified. This method is 100 times
more sensitive than DNP method in detection of the N-terminal.
(iii) Edman degradation. One of the most important and widely used method
which depends on the selective removal of N-terminal amino acids from
polypeptide chain (proteins) is the Edman degradation. The method removes N-
terminal amino acid and leaves the rest of the polypeptide chain intact. Thus, the
method can be used repeatedly to identify the N-terminal of shortened peptide
chain.
Pehr Edman of Max Planck institute of biochemistry, Munich, introduced this
method. It involves treatment of polypeptide with phenylisothiocyanate. The N-
terminal of the polypeptide reacts with phenylisothiocyanate to form substituted-
thiourea, that on mild hydrolysis results in the selective separation of N-terminal
amino acid as phenylthiohydantoin, which is identified by HPLC (High
Performance Liquid Chromatography). The rest of the polypeptide chain remains
intact (of course with one N-terminal amino acid less). The method can be
repeated on this new N-terminal and the process may be repeated.
The automation of above process has enhanced the efficiency of this method to
continue sequencing the amino acids in polypeptides. The repeated Edman
degradation can be used for identification of polypeptides containing 50 or even
more amino acids. This method is not applicable for high polypeptide chains due
to accumulation of impurities (that is, amino acids) in each hydrolysis step.
C-terminal analysis
The C-terminal analysis is carried out even more efficiently by enzymatic
methods rather than chemical methods. The enzyme carboxypeptidase, cleaves
selectively the peptide linkage adjacent to C-terminal (free carboxylic group) in
polypeptide chain. The removal of C-terminal residue results in the formation of
shortened polypeptide chain, which can be treated with the carboxypeptidase
enzyme to determine the new C-terminals.
28.8.2 Sequence Analysis
The end group or terminal analysis can be used for selective removal of terminal
groups. However, it is a difficult process to identify a polypeptide chain by
stepwise continuous removal of the terminal residues. The sequence of amino
acids is identified by carrying out partial hydrolysis of the polypeptide chain.
Partial hydrolysis breaks the chain selectively into smaller fragments that can be
identified further. Specific enzymes are used which cleave the polypeptide at
specific sites only. The smaller fragments can then be identified by Edman
degradation.
Reagents/Enzymes Specific site of cleavage in polypeptide chain
Cynogen bromide Cleaves at carboxy end of methionine (Met)
Clostripain Cleaves at carboxy end of arginine (Arg)
Trypsin Cleaves at carboxy end of arginine (Arg) and lysine (Lys)
Chymotrypsin Cleaves at carboxy end of phenylalanine (Phe), tyrosine
(Tyr) and tryptophan (Trp)
Pepsin Cleaves at carboxy end of aspartic acid (Asp), glutamic acid
(Glu), leucine (Leu), phenyalanine (Phe), tyrosine (Tyr), and
tryptophan (Trp)

Identifying the sequence of amino acids in a given peptide:


a problem solving approach
(a) Identify the N-terminal or C-terminal from the information if
specified like Edmand degradation, DNP method, enzymatic
hydrolysis, and so on as discussed.
(b) Select the different hydrolysis fragments with common amino
acids portion.
(c) Align the fragments in such a way that the common amino acids
overlap each other.
(d) Write the complete sequence taking overlapping portion(s) once.
Illustrative Example 1. A tripeptide on partial acid hydrolysis gave two
dipeptides containing Glu, Phe and Gly, Phe respectively. Treatment of the first
with DNFB gave N-labelled glutamic acid and that of second gave N-labelled
phenylalanine. What will be sequence of amino acids in tripeptide? Also write
the sequence using one letter code for amino acids (refer Table 28.1).
Solution. The first dipeptide has glutamic acid (Glu) as N-terminal so its
sequence is Glu-Phe. For second dipeptide, the N-terminal is phenylalanine
(Phe) and thus, its sequence is Phe-Gly.
Now writing these dipeptides in such a way that common amino acids are
aligned, gives the following sequence:
The sequence of amino acid, in tripeptide, using one letter code can be written as
E-F-G.
Illustrative Example 2. Partial hydrolysis of a pentapeptide gave three
fragments (tripeptides) as follows:
(a) Gly-Val-Ala
(b) Phe-Gly-Val
(c) Val-Ala-Pro
Identify the sequence of the amino acids in pentapeptide.
Solution. The different hydrolysis fragments are aligned one below the other so
that the common amino acids overlap. For the given fragments, the alignment
gives the correct sequence for pentapeptide as follows:

The sequence of amino acid, in pentapeptide, using one letter code can be
written as F-G-V-A-P.
Illustrative Example 3. A heptapeptide on reaction with DNFB followed by
hydrolysis produces N-DNP valine. The hydrolysis of peptide by
carboxypeptidase enzyme releases glutamic acid. The partial hydrolysis of
peptide gives the following fragments:
(i) Pro – Leu – Val (ii) Ala – Tyr – Pro (iii) Leu – Val – Glu (iv) Val – Ala (v)
Val - Glu
Identify the sequence of amino acids in the heptapeptide.
Solution. The formation of N-DNP valine in DNB method indicates that N-
terminal of peptide is valine (Val). The hydrolysis of peptide by
carboxypeptidase enzyme releases glutamic acid, which indicates that C-
terminal of peptide is glutamic acid (Glu).
Now the two fragments have valine (Val) as N-terminal, that is, Val-Ala and
Val-Glu. But the fragment with glutamic acid (Val-Glu) cannot be N-terminal as
glutamic acid is C-terminal of the heptapeptide.
Thus, aligning the different fragment obtained by hydrolysis keeping the Val-
Ala at N-terminal and Val-Glu at C-terminal gives the sequence as:

Amino acid sequence using one letter code can be written as V-A-Y-P-L-V-E.
Illustrative Example 4. Metenkephalin, a brain peptide, on Edman degradation
gives phenylthiohydantoin derivative of tyrosine. The hydrolysis of same peptide
with carboxypeptidase enzyme releases methionine. The second, third, and
fourth Edman degradation yields glycine, glycine and phenylalanine
respectively. What will be sequence of amino acid in metenkephalin?
Solution. The carboxypeptidase enzyme releases the C-terminal amino acid of
the peptide chain. Thus, methionine (Met) is the C-terminal of Metenkephalin
peptide. The Edman degradation releases the N-terminal amino acid of a peptide
as phenylthiohydantoin derivative leaving the rest of the polypeptide chain
intact. The repeated Edman degradation of the remaining peptide chain releases
the successive N-terminal amino acids as phenylthiohydantoin derivative. The
first Edman degradation releases tyrosine (Tyr) as phenylthiohydantoin
derivative, which indicates the
N-terminal of metenkephalin peptide to be tyrosine (Tyr). The successive
Edman degradation yields glycine (Gly), glycine (Gly) and phenylalanine (Phe)
respectively. Thus the sequence of amino acids in metenkephalin peptide is:
Tyr–Gly–Gly–Phe–Met
Amino acid sequence using one letter code can be written as Y-G-G-F-M.
28.9 PROTEINS
The nitrogenous macromolecules composed of several amino acid units are
referred to as proteins. The name is derived from the Greek word pre-eminent, as
proteins are the most important and fundamental structural components of the
body.
Functions. Proteins perform a variety of functions in the body. Proteins are the
only source, which supply nitrogen and sulfur to the body. Nitrogen is lost from
body in the form of urea through urine. Enzymes, responsible for catalyzing
various biochemical reactions are proteineous in nature. Proteins, also known as
immunoglobins or antibodies protect the body against bacterial and viral
infections. They provide mechanical support to body being the major
components of skin, muscles, and bones. Proteins perform the role of receptors
(bind vitamins and hormones) and transporters (carry small molecules and ions
within the system), and aid in storage. Proteins exert osmotic pressure and thus,
help in maintaining the electrolyte concentration and water balance in the body.

28.9.1 Structure of Proteins


Primary structure
The sequence of amino acids in a protein (polypeptide chain) is known as the
primary structure if proteins. The determination of primary structure i.e.
sequence analysis is just discussed above. In primary structure, the peptide
bonds form the backbone and the side chains of amino acids project outside the
peptide backbone.

Secondary structure
The secondary structure involves the determination of spatial arrangement of
polypeptide chains. The peptide linkage is a rigid bond. However, free rotation is
possible between C–C single bonds and C–N single bonds in amino acid side
chain. This results in different conformations of polypeptide chains referred to as
secondary structure of the proteins.
Examples. The amino acids which commonly form β-pleated sheet are—
alanine, methionine, valine, and isoleucine. The β-keratin protein, which occurs
in spider’s web and reptilian claw, is a fully extended chain of β-pleated sheet.
The silk fibroin protein (occurs in silk worms) is also rich in β-pleated sheet
structure.
β-bend structure. In proteins, the polypeptide chains may change their direction
and result in the formation of a loop. The loop is stabilized by hydrogen bond
formation between carbonyl oxygen and amide hydrogen (of the third amino
acid residue).

Examples. The amino acids that are found in β-bends are glycine, aspargine and
proline.
Tertiary and quaternary structures
The polypeptide chain with secondary structure may be present as folded
structures that have the hydrophilic groups on the exterior and hydrophobic
groups on the interior of the polypeptide chains. This lowest energy
conformation is known as the tertiary structure. The tertiary structure of a
protein is its biologically active conformation and is also known as native
conformation.
The major stabilizing force responsible for compact three-dimensional (3-D)
form of tertiary structure is hydrophobic interactions between non polar side
chains of amino acids. Besides this, the polar side chains undergo hydrogen bond
formation and ionic interactions. Folding in tertiary structures of proteins bring
the amino acid residues close together compared to primary structure.
Example. Enzyme carbonic anhydrase exhibits tertiary structure of proteins.
A protein that consists of two or more polypeptide chains held together by non-
covalent forces or covalent bonds, is referred to as the quaternary structure of
protein.
The assembly of many polypeptide chains is known as oligomer and the
individual polypeptide chains are termed monomer units.
The monomeric units can be identical or different in their primary, secondary,
and tertiary structures.
Example. Haemoglobin has a quaternary structure and consists of four folded
polypeptide chains. Enzyme creatine phophokinase has two polypeptide chains.

28.9.2 Fibrous and Globular Proteins


Proteins are classified into two major groups:
(a) Fibrous proteins. The molecules of fibrous proteins have long thread like
structure, that at times are joined to each other through hydrogen bonding. A
solvent must overcome these strong intermolecular forces to dissolve such
proteins. As a result, fibrous proteins are insoluble in water. They act as main
supporting structural material of tissues.
Examples. Keratin, collagen, myosin, fibroin, and so on.
(b) Globular proteins. The molecules of globular proteins have folded structure
that resembling a spherical shape. The intermolecular attraction (between
different molecules) in this case are weak, which are easily overcome by
solvents. As a result, globular proteins are water soluble. They perform the
functions of maintenance and regulations of life processes.
Examples. Haemoglobin, myoglobin, insulin, albumin, fibrinogen, and so on.
Classification based on solubility and physical properties
Proteins are also classified on the basis of their solubility and physical properties
into three main group as:
(a) Simple proteins. Proteins which on complete hydrolysis result in the
formation of amino acids only are called simple proteins.
Examples. Albumin, globulins, keratin, collagen, elastin, and so on.
(b) Conjugated proteins. These are simple proteins, which apart from amino
acids contain a non-protein group known as prosthetic group. The protein part of
conjugated proteins is termed as Apoprotein.
Examples. Nucleoproteins (DNA and RNA are present as prosthetic groups),
Hemoproteins of haemoglobin (Heme is present as prosthetic group), and
Metalloproteins of ferritin and carbonic anhydrase (metal ion is present as
prosthetic group).
(c) Derived proteins. These proteins are formed from native proteins by action
of heat and chemicals.
Examples. Denatured proteins or coagulated proteins.
NOTABILIA 18
28.9.3 Denaturation of Proteins
The change in secondary and tertiary structures of a protein causes changes in
the functional or biological activity of proteins and is termed as denaturation of
protein. Proteins in native state have a folded (coiled) structure and denaturation
involves the unfolding of this naturally folded structure. Denaturation causes
precipitation of proteins. The action of heat, sunlight, and chemicals can cause
denaturation of protein. Depending upon the extent and type of changes in the
structure, the denaturation may be referred to as reversible or irreversible.
Effect of denaturation
(1) Decreases the rate of diffusion of protein molecules.
(2) Decreases the solubility of proteins drastically and as a result, maximum
precipitation (coagulation) occurs at isoelectric point of proteins.
(3) Some of the groups like –SH get exposed for reactions.
(4) The enzymal and hormonal activities of proteins are destroyed.
(5) In certain denaturated proteins, the digestibility of proteolytic enzymes has
been found to increase.
(6) Proteins become biologically inactive.
Examples
(i) Boiling of an egg: Here, tertiary structure of protein is changed and forms a
disorganized polypeptide chain.
(ii) Frying eggs: It causes denaturation of albumin. The egg white (albumin) is
a liquid, which on heating gets converted to a solid.
(iii) Sunburn: It is basically the denaturation of skin proteins occuring due to
the reaction of skin proteins with UV radiations.
(iv) Poisoning due to heavy metal ions like Hg2+, Pb2+, and Ag+: It causes
the denaturation of a certain enzyme by coordinating with carboxylate
groups and sulfur atoms (in sulfide linkage).
EXERCISES***
1. What are essential amino acids? How do they differ from non-essential amino
acids? Give two examples of each and write their structures.
2. Explain the classification of amino acids on the basis of their chemical nature.
3. Give two examples of each of the following, along with their structure:
(a) Neutral amino acids
(b) Acidic amino acids
(c) Basic amino acids
(d) Semi-essential amino acids
(e) Sulfur containing amino acids
(f) α-imino acids
4. In general, the natural amino acids exist in which configuration?
5. What are zwitterions?
6. Define the isoelectric point of an amino acid?
7. Write the structural formula for species that exist predominantly for the given
amino acids at pH 1.0 and 13 respectively. The isoelectric points are:
Amino acid Isoelectric point
Lysine 9.7
Glutamic acid 3.2
Isoleucine 6.0
Alanine 6.0
8. How is electrophoresis used for separation and analysis of amino acids?
9. How will you prepare the following amino acids using methods indicated in
the parentheses against every amino acid?
(a) Alanine (direct ammonolysis)
(b) Glycine (Gabriel phthalimide synthesis)
(c) Serine (phthalimido malonic ester synthesis)
(d) Valine (Strecker’s synthesis)
10. Complete the following reactions:
11. What is ninhydrin reagent? How is it used for identification of amino acids?
Explain giving chemical equations.
12. What is a polypeptide linkage? In a polypeptide, what do the terms ‘N-
terminal’ and
‘C-terminal’ refer to?
13. In the following peptides,
(a) Identify number of peptide linkages.
(b) Identify, number of amino acids present in each structure.
(c) Identify N-terminal and C-terminal ends.
(d) Classify these structures as di-, tri-, tetra-, peptides.
14. What do you understand by N-terminal and C-terminal protection? Write the
structure of the species obtained by the reaction of the following amino acids
and protective reagents.
(a) Valine + di tert. butyldicarbonate
(b) Leucine + benzylchloroformate
(c) Alanine + benzyl alcohol
15. What does DCC stand for? What role does it play in formation of peptide
linkages?
16. Starting from alanine and valine, what steps should be followed to get the
dipeptide alanylvaline as the only product?
17. Explain Merrifield solid phase peptide synthesis. What are the advantages of
this method?
18. For polypeptide Val-Phe-Gly-Lys, what product(s) will be formed on its
reaction with DNFB reagent?
19. Write the structure of the product formed by the Edman degradation of
polypeptide Leu-Gly-Ala-Phe-Tyr-Val. What fragment of peptide chain will
leave after the degradation reaction?
20. A tripeptide on selective hydrolysis yields two fragments A and B. Fragment
A contains glycine and alanine and fragment B contains valine and glycine.
The N-terminal of fragment A is alanine while that of B is glycine. Identify
the tripeptide.
21. A tetrapeptide on complete hydrolysis yielded Val, Leu, Tyr, and Glu. The
reaction on tetra peptide with DNFB reagent gave DNP derivative of Leucine.
Selective hydrolysis of tetrapeptide resulted in the formation of peptides as
Val-Glu, Leu-Tyr-Val, Tyr-Val-Glu, and Leu-Tyr. Identify the tetrapeptide.
22. A polypeptide on partial hydrolysis gave the following di and tripeptides:
Ser-Met-Ile; Ile-Tyr; Ile-Ser-Met; Met-Ile; Ile-Ser.
Identify the sequence of amino acids in peptide chain.
23. A hexapeptide on Edman degradation yielded thiohydantoin derivative of
valine. Hydrolysis with trypsin gave two fragments, one containing Lys, Val,
Leu and the other containing Phe, Ala, and Gly. Hydrolysis with
chymotrypsin gave glycine and pentapeptide containing Lys, Leu, Val, Phe,
and Ala. Hexapeptide on hydrolysis with carboxypeptidase gave glycine.
Identify the sequence of amino acid in peptide chain.
24. What are proteins? What do you understand by primary, secondary, and
tertiary structures of proteins?
25. What is denaturation of protein? Explain with suitable examples.
26. Why is cooked food easy to digest?

Answers to selected exercises


19. The product is phenylthiohydantoin derivative of Leucine and the remaining
fragment of the polypeptide is Gly-Ala-Phe-Tyr-Val.
20. Fragment A is Ala-Gly and fragment B is Gly-Val. The tripeptide is Ala-
Gly-Val.
21. On reaction of peptide with DNFB regent, the N-terminal amino acid is
obtained as DNP derivative. The tetrapetide is Leu-Tyr-Val-Glu.
22. Ile-Ser-Met-Ile-Tyr
23. N-terminal of the hexapeptide is Val (from Edman degradation) and the C-
terminal is Gly (from carboxypeptidase hydrolysis). Trypsin cleaves the
peptide chain at carboxy end of Lys. Chymotrypsin cleaves the peptide chain
at carboxy end of Phe. The sequence of amino acids in hexapeptide is Val-
Leu-Lys-Ala-Phe-Gly.
* An exception to this is proline where a secondary amino group ( NH) is attached to α-carbon. Thus,
proline is actually an α-imino carboxylic acid.
** Answers to some selected exercises problems are given at the end.
Note:
* Amino acids with an asterisk are essential amino acids, that must be supplemented through diet.
† At pH = 7, Asp and Glu have a net negative charge and exist as anions. At pH = 7, Lys and Arg have a net
positive charge and exist as cations. Rest of the amino acids at this pH exist in the neutral form.
‡ Structurally, in cystine, the two cysteine molecules are joined through sulfur (disulfide linkage).
@@ Proline is an α-imino acid
@ Except Glycine all other amino acids are optically active.
Amino carboxylate ion
* Answers to some selected exercises problems are given at the end.
MINI ESSAY
II
ENZYMES AND NUCLEIC ACIDS
A. ENZYMES
Enzymes are complex molecules containing proein molecule of high molecular
weight. In other words, enzymes are colloidal proteins that are produced in
living organisms and catalyze the chemical reactions of biological systems. For
this reason, the enzymes are also termed as biocatalysts.
Enzymes have tremendous catalytic power and exhibit high reaction specificity
as well as stereoselectivity.
Structure of enzymes
Enzymes consist of protein part, which is known as apoenzyme and a smaller
portion of a non-protein part known as prosthetic group or co-enzyme. A co-
enzyme is a low molecular weight covalent molecule. In an enzyme, the co-
enzyme is combined with apoenzyme. In some enzymes, the apoenzyme is
combined with metal ions such as K+, Mg2+, Ca2+, Zn2+, Cu2+, Co2+, and
Fe2+. The enzyme where metal ions are loosely attached and form easily
dissociable complexes are termed as metal activated enzymes. The enzymes in
which metal ions have a strong binding and are not dissociable, are termed as
metalloenzymes.
Nomenclature and classification
Enzymes are generally named by adding suffix “ase” to the substrate on which
they act. A systematic nomenclature for enzymes is laid down by IUB
(International Union of Biochemistry). In accordance of this, the enzymes are
classified into six main categories as given in the following table:
Enzyme Action
An enzyme (E) functions by combining with the reactant (termed as substrate) to
form an activated complex known as enzyme–substrate complex. The complex
dissociates further to form product (P) and enzyme (E). The enzyme once
released from the complex is free to combine with other substrate molecules.

A substrate can attach to a specific site at the enzyme. This is the reason that
enzymes show specificity to reactions, that is, an enzyme catalyzes only a
particular reaction and not other reactions (even closely related).
A small portion of an enzyme area which is responsible for its catalytic activity
is termed as the active site.
Factors influencing enzyme action
Different factors that affect the activity of an enzyme are:
(i) Temperature. For each enzyme, there is a specific temperature at which the
activity of enzyme is maximum. This is termed as the optimum temperature of
enzyme. In general, the enzymes in human system have an optimum temperature
in the range 35–40°C.
(ii) pH. The pH at which an enzyme has maximum activity is termed as its
optimum pH. For most of the enzymes, the optimum pH ranges from 4–9.
(iii) Enzyme concentration. The rate of an enzymatic reaction increases with an
increase in the concentration of enzyme.
(iv) Concentration of product. An increased product concentration lowers
down the enzymatic reaction. The products formed may either inhibit the active
site of enzyme or can make the reaction proceed in backward direction.
B. NUCLEIC ACIDS
Nucleic acids are macromolecules present in the living organisms. Nucleic acids
control the synthesis of proteins and are also responsible for storing the complete
description of living organism (genetic information) and passing the information
from one generation to another. The two nucleic acids that are found in living
organisms and are of much importance are:
1. Deoxyribonucleic acid (DNA)
2. Ribonucleic acid (RNA)
Structure of nucleic acids
The structural units of nucleic acids are nucleotides. Nucleic acids, on complete
hydrolysis, produce heterocyclic base (nitrogenous base), sugar residues and
phosphoric acid. The basic structure of nucleic acids consists of a polymer chain
(polyester) where phosphoric acid forms an ester linkage with sugar. In other
words, polyester chain has alternate phosphoric acid and sugar residues. To each
sugar residues, one group of nitrogenous base is attached.

For DNA:
Sugar is D-2-deoxyribose and correspoinding nitrogen bases can be thymine, cytosine, adenine, or guanine
For RNA:
Sugar is D-ribose and corresponding nitrogen bases can be uracil, cytosine, adenine or guanine
Fig. 1 Schematic representation of nucleic acids.

The heterocyclic bases are pyrimidine and purine derivatives. The pyrimidine
derivatives include cytosine, uracil, and thymine while adenine and guanine are
purine bases. In deoxyribonucleic acids (DNA), adenine, guanine, thymine and
cytosine are present. In ribonucleic acid (RNA), thymine is replaced by uracil.
Sugar in nucleic acids is a pentose sugar, which may be either D-ribose (in
which case, nucleic acid is called ribonucleic acid; RNA) or D-2-deoxyribose
(the nucleic acid is called deoxyribonucleic acid, DNA).
Figure 2 depicts the structures of nitrogen bases and sugars in nucleic acids.
Nucleosides
A sugar is attached to a heterocyclic base through a glycosidic linkage and this
whole unit constitutes a nucleoside.
In a pyrimidine nucleoside, the C1 of sugar is attached directly to N1 of
pyrimidine base.
In a purine nucleoside, the C1 of sugar is attached to N9 of purine base.
The naming of nucleosides is done based on the type of sugar and bases present.
Thus, adenosine (ribose + adenine), deoxyadenosine (deoxyribose sugar +
adenine), cytidine (ribose + cytosine), deoxycitidine (deoxyribose + cytosine),
uridine (ribose + uracil); deoxythymidine (deoxyribose + thymine) are some
examples of nucleosides.

Fig. 2 Nitrogen bases and sugars in Nucleic acids.

Nucleotides
A nucleoside which is attached through its sugar to phosphoric acid group is
termed as a nucleotide. It should be noted that
(i) A deoxyribose sugar of nucleoside can attach to phosphoric acid through
free –OH present at C3 or C5 to form ester linkage.
(ii) A ribose sugar in a nucleoside can attach to phosphoric acid through free –
OH groups at C2, C3, or C5 to form an ester linkage.
The structures of nucleosides and nucleotides are as follows:
Deoxryribonucleic acid (DNA)
Watson and Crick postulated that this nucleic acid has a double helical structure.
In DNA, the sugar is a deoxyribose sugar and heterocyclic bases are Adenine
(A), Guanine (G), Thymine (T), and Cytosine (C). The nucleotide chain in DNA
has alternate sugar and phosphate ester residues. The phosphoric acid on one
side forms ester linkage with 3′ –OH of one sugar and on other side it forms
ester linkage with 5′–OH of another sugar residue.
The helix are held together by hydrogen bonds between the bases on one strand
and those on the other. The bases in one strand are attached to complementary
bases on other strand through hydrogen bonding. Adenine pairs with thymine
while cytosine pairs up with guanine. The adenine and thymine base pairs have
two hydrogen bonds between them while cytosine and guanine base pairs have
three hydrogen bonds between them. Thus, two strands can fit together as a helix
properly only if given base in one chain has only a specific base as its nearest
neighbour in the other chain.

During cell division, the double helix of DNA unwinds into two strands each
acting as a template on which its complementary strand may be constructed.
This replication is the key to inheritance of character (genetic information). A
gene is a portion of DNA molecule.
Ribonucleic acid (RNA)
This nucleic acid exists as a single stranded helix. The structure of RNA is
similar to that of DNA except that the sugar in it is ribose sugar and uracil is
present instead of thymine as a heterocyclic base. There are three types of RNA,
namely messenger (m-RNA), transfer (t-RNA), and ribosomal
(r-RNA).
The DNA and RNA are involved in protein biosynthesis, which involves two
important processes of transcription and translation.
Transcription. The genetic information contained in DNA is incorporated into
m-RNA by this process. The DNA strands uncoil and the m-RNA having a
nucleotide sequence complementary to that of uncoiled DNA strand is
assembled. For example, for DNA strand with sequence A-T-G-C-T-A-C, the
complementary m-RNA assembly is U-A-C-G-A-U-G.
The sequence of three heterocyclic bases in m-RNA is known as codon and each
codon specifies the amino acids to be used in protein synthesis. For example, the
codons for glycine are GGG, GGA, GGC, and GGU.
Translation. It is the process by which the nucleotide sequences in m-RNA
(formed by transcription) are identified by t-RNA and accordingly the order of
amino acid in the protein being synthesized is determined. For each amino acid,
there is at least one specific t-RNA. The t-RNA serves as an amino acid carrier
as the well as the interpreter of genetic code.
Chapter29
Drugs and Pesticides

A. DRUGS
29A.1 Introduction
Drug is defined as any substance or product that is used or is intended to be
used, to modify or explore physiological systems or pathological states, for the
benefit of the recepient. In medicine, drugs are used for diagnosis, prevention,
treatment/cure of diseases. The origin of word drug can be traced to the French
word drogue which means a dry herb. The treatment of infections (or diseases)
with specific drugs that destroy the harmful organisms with minimal or no toxic
effects on the cells of the host is known as chemotherapy. Drugs are mainly
classified on the basis of their action and it is difficult and rather meaningless to
include all the classifications in this book. Here, we discuss only those categories
which are general in nature and are of common concern.
Analgesics
A drug that selectively relieves pain by acting on the central nervous system
(CNS) and on peripheral (external) pain mechanisms without significantly
altering consciousness is referred to as an analgesic. For example, aspirin,
ibuprofen, phenylbutazone, and morphine (a drug obtained from opium).
Remarks: Analgesics relieve pain only as a symptom, without treating its cause.
Antipyretics
A drug, which reduces body temperature in fever but does not lower the normal
body temperature is called an antipyretic. Examples of antipyretics are
paracetamol and aspirin.
Antibiotics
These are the substances produced by microorganisms which suppress the
growth of or kill other microorganisms. For example, penicillin,
chloramphenicol, and tetracycline. The first antibiotic was discovered by
Alexander Fleming in 1929, which was procured from penicillium mould, and he
named the substance as penicillin.
Other categories of drugs include antimalarials (for cure and prevention of
malaria such as chloroquine), antihistamines (as antiallergics), hypnotics (for
inducing and maintaining sleep such as diazepam), sedatives (to reduce
excitement without inducing sleep, though drowsiness is caused. Examples,
derivatives of barbiturates), tranquilizers (to reduce mental tension, for example,
reserpine—an alkaloid). Derivatives of barbiturates are also used as hypnotics
and tranquilizers.
The action of the drugs on bacteria may be (a) bacteriostatic, that is, inhibiting
the growth of bacteria without killing them. Chloramphenicol and sulfonamides
are two of the bacteriostatic drugs; (b) bacteriocidal, that is, inhibiting the
growth of bacteria by killing them. For example, penicillin and streptomycin.
Some primarily static drugs may become cidal at higher concentrations such as
sulfonamides. On the other hand, some cidals may only be static under certain
circumstances. For example, streptomycin.
Effects of drugs
Drugs can affect the functioning of the body through side effects, secondary
effects, and toxic effects.
Side effects. These effects are unwanted but often unavoidable that occur at
therapeutic doses (means prescribed doses). For example, some of the anti-
allergic drugs may produce sedation (drowsiness).
Secondary effects. These effects are indirect consequences of a primary action
of the drug. For example, antibiotics kill the friendly bacteria along with harmful
bacteria.
Toxic effects. These effects are the result of excessive pharmacological action of
the drug due to over dosage or prolonged use. For example, over dosage of
paracetamol causes liver damage.

Lethal Dose –50: The toxicity of the drugs is expressed in


terms of LD50, which stands for Lethal Dose –50. It is defined as the
dose required to kill 50% of the population of the test animal and is
expressed in mg/kg of the body weight of the animal. Smaller the
LD50 value, the more toxic is the drug (in general any chemical).
The following section gives brief introduction to the synthesis, uses, side effects,
and mode of action of some common drugs.
29A.2 ANALGESICS, ANTIPYRETICS AND ANTI-
INFLAMMATORY DRUGS
(1) Aspirin
Chemically, aspirin is acetylsalicylic acid and is conveniently synthesized by
acetylating salicylic acid (2-hydroxybenzoic acid). Synthesis of salicylic acid is
carried out by treating sodium phenoxide with carbon dioxide at high
temperature and pressure (Kolbe’s reaction). Further acetylation of salicylic acid
with acetic anhydride in the presence of acetic acid or concentrated sulfuric acid
results in the formation of aspirin. Aspirin is a white crystalline compound. It is
slightly soluble in water but highly soluble in organic solvents such as alcohol,
chloroform, and so on. The formation of aspirin can be understood by the
following steps.
Step 1. Synthesis of salicylic acid (2-hydroxybenzoic acid)

Step 2. Acetylation of salicylic acid (formation of aspirin)

(2) Phenacetin
Chemically, phenacetin is 4-ethoxyacetanilide. It can be synthesized from p-
nitrophenol using two routes discussed as follows:
(a) In the first method, p-nitrophenol is converted into p-aminophenol, which
on acetylation with acetic anhydride yields p-hydroxyacetanilide
(paracetamol). It is then converted into sodium salt, which on reaction with
bromoethane results in the formation of phenacetin.
(b) In the second method, p-nitrophenol is converted into ethyl ether by
treating with sodium ethoxide and iodoethane (or bromoethane), which
results in the formation of p-ethoxynitrobenzene. Reduction of p-
ethoxynitrobenzene yields p-phenetidine, whose acetylation results in the
formation of phenacetin.

Phenacetin is a banned drug.


Phenacetin, a general analgesic and antipyretic, is a banned drug.
Phenacetin was never used alone but it had been used in APC
tablets (combination of Aspirin, Phenacetin, and Caffeine) for
curing common cough and cold.
The use of phenacetin was claimed to produce sense of
relaxation, to reduce tension and fatigue, and to increase work
capacity. It was considered to be habit forming, overdosage of
which may lead to kidney damage. Therefore, this drug is banned
in many countries including India.
(3) Paracetamol
The synthesis of paracetamol has already been described with the synthesis of
phenacetin by route (a). The initial reactant for the synthesis of paracetamol is p-
aminophenol, which on acetylation results in the formation of paracetamol.

(4) Phenylbutazone
Chemically, phenylbutazone is 4-butyl-1,2-diphenylpyrazolidine-3,5-dione. It is
synthesized by the condensation of diethyl butylmalonate and hydrazobenzene in
the presence of sodium ethoxide.

General uses of aspirin, paracetamol, and phenylbutazone


1. These drugs are used as analgesics in headache, backache, arthritis, and so
on.
2. These are used as antipyretics to bring down fever.
3. Aspirin is recommended to the heart patients to reduce the chances of heart
attack.
4. Physiological activities of paracetamol as a general analgesic and
antipyretic are comparable to those of phenacetin. But paracetamol is a
much safer drug and has fewer side effects compared to phenacetin.
5. Phenylbutazone has more potent anti-inflammatory action as compared to
the analgesic and antipyretic actions. It is used in rheumatoid arthritis,
spondylitis, and gout.
General side effects of aspirin, paracetamol, and phenylbutazone
1. Nausea, vomiting, and abdominal pain
2. Peptic ulcer
3. In few cases, skin rashes
4. The phenylbutazone may cause edema (swelling in body due to water
retention), bone marrow depression which means decrease in the formation
of blood cells.
5. Overdosages of paracetamol (>150 mg/kg body weight) may lead to liver
damage and death.
General mode of action of analgesic, antipyretic, and anti-inflammatory
drugs. Most of the analgesic and antipyretic drugs act by blocking the
generation of prostaglandins.
29A.3 SULFA DRUGS—ANTIBACTERIAL
AGENTS
(1) Sulfanilamide
The synthesis of sulfanilamide, beginning from aniline involves the following
four steps:
Step 1. Acetylation of aniline (formation of acetanilide)
Step 2. Reaction with chlorosulfonic acid (formation of p-
acetamidobenzenesulfonyl chloride)
Step 3. Reaction with ammonia (formation of p-acetamidobenzenesulfonamide)
Step 4. Acidic hydrolysis (formation of sulfanilamide)
The reactions involved in these steps are as follows:

(2) Sulfaguanidine
In the synthesis of sulfaguanidine, first two steps are the same as in the synthesis
of sulfanilamide. This means that up to the formation of p-
acetamidobenzenesulfonyl chloride the reaction process is the same. Thereafter
the reaction of guanidine and p-acetamidobenzenesulfonyl chloride followed by
acid hydrolysis results in the formation of sulfaguanidine. The reactions
involved in the process are as follows:
(3) Sulfadiazine
Chemically, this drug is 2-(4-aminobenzenesufonamido)pyrimidine. It is
synthesized by the condensation of 2-aminopyrimidine with p-
acetamidobenzenesulfonyl chloride. Synthesis of 2-aminopyrimidine is carried
out by the reaction of guanidine with ethyl 3-oxopropanoate (ethyl ester of
formyl acetic acid). The reactions are as follows:
(i) Synthesis of 2-aminopyrimidine

(ii) Reaction of p-acetamidobenzenesulfonyl chloride with 2-aminopyrimidine

Synthesis of p-acetamidobenzenesulfonyl chloride has already been described in


the synthesis of sulfanilamide.
General uses of sulfa drugs
1. Treatment of urinary tract infections
2. Treatment of throat and gum infections
3. Treatment of eye infection
4. Silver salt of sulfadiazine is used for preventing the infection of burnt
surfaces
5. Sulfa drugs in combination with other drugs are used for the treatment of
dysentery, typhoid, malaria, rheumatic fever, and so on.
General side effects of sulfa drugs
1. Nausea, vomiting, and abdominal pain
2. 2–5% cases of skin rashes (hypersensitivity reactions)
General mode of action of sulfa drugs. Folic acid is essential for the growth of
bacteria. The bacteria synthesize folic acid in the presence of p-aminobenzoic
acid (PABA). It takes PABA from the medium on which it grows. In presence of
sulfonamides, the bacteria is not able to take up PABA. As a result, folic acid
formation does not take place and the growth of bacteria is inhibited.
Sulfonamides do not affect the human cells as they do not synthesize folic acid
themselves and utilize the preformed folic acid supplied through the diet.
29A.4 CHLORAMPHENICOL
(CHLOROMYCETIN)—AN ANTIBIOTIC
Chloramphenicol is a wide spectrum antibiotic (bacteriostatic) and was first
isolated from Streptomyces venezuelae in the year 1947. Chloramphenicol is an
optically active compound and is laevorotatory. The synthesis of
chloramphenicol is carried out by two methods:
(1) Baltz synthesis. It involves the synthesis of chloramphenicol using
benzaldehyde and nitroethanol as the initial reactants.

(2) Long’s synthesis. This synthesis involves p-nitroacetophenone as an initial


reactant. The following reaction sequence explains the formation:
Uses. Chloramphenicol is very effective in the treatment of typhoid fever, curing
diarrhoea, dysentry, pneumonia, and whooping cough.
Side effects of chloramphenicol
1. Nausea, vomiting, diarrhoea.
2. Bone marrow depression, that is, decrease in the formation of blood cells.
General mode of action of chloramphenicol (Chloromycetin). They act by
inhibiting the protein biosynthesis in bacteria, thereby inhibiting their growth.
29A.5 CHLOROQUINE—ANTIMALARIAL DRUG
It is a 4-aminoquinoline derivative and its synthesis involves the reaction of 4,7-
dichloroquinoline with 4-diethylamino-1-methylbutylamine (also called
Novaldiamine).

(i) Synthesis of 4,7-dichloroquinoline

(ii) Synthesis of 4-diethylamino-1-methylbutylamine (Novaldiamine)


Uses of chloroquine
1. It is a drug of choice for cure and prevention of malaria.
2. It is also used for the control of extra-intestinal amoebiasis.
Side effects of chloroquine
1. Nausea, vomiting, abdominal pain, headache, and loss of appetite.
2. Prolonged use of high doses may lead to loss of vision, loss of hearing, and
graying of hair.
General mode of action of chloroquine. The mode of action of chloroquine is
not clearly known. However it is believed that it accumulates in the matarial
parasite and causes retention of toxic substances in the parasite, thereby killing
it.
29A.6 ANTIHISTAMINES (ANTIHISTAMINICS)
Histamine is responsible for allergic phenomena in body and is generally present
in rich amounts in the tissues of skin, lungs, liver, placenta, gastric tissues, in the
blood and in venom. Histamine is produced in the body by the decarboxylation
of histidine, an important amino acid.

Histamine dilates the blood vessels, contracts smooth muscles, increases gastric
secretion besides behaving as a neurotransmitter.
The drugs that competitively neutralize or oppose (antagonize) the action of
histamine at the receptor site are known as antihistamines. For example,
diphenhydramine (benadryl), promethazine (phenergan), cetrizine, etc. The
structure of benadryl is as follows:

NOTABILIA 19
Besides the drugs discussed so far, the alkaloids and steroids are another class
of compounds which are of immense medicinal use. The physiological
importance of some alkaloids and steroids is discussed in ‘Mini Essay III’
subsequently.

B. PESTICIDES
29B.1 INTRODUCTION
Agrochemicals can be broadly defined as the class of chemicals, which are used
to increase the yield of the crops. This class includes fertilizers and pesticides.
Fertilizers increase the crop yield by supplying the nutrients and pesticides
protect the crop.
Pesticide is the broader term used for the chemicals, which are used to combat
the attack of various pests to the crops. The application of pesticides can be
further broadened and is not just restricted to the crops but also includes the
chemicals used to eradicate the community pests such as cockroaches,
mosquitoes, rats, flies, and so on. Pesticides can be classified into three major
classes—insecticides, fungicides, and herbicides (or weedicides). Besides this,
there are also acaricides (to kill mites), molluscicides (to kill snails),
nematocides (to kill microscopic worms), and rodenticides (to kill vertebrate
pests such as rats).
Further on, pesticides may be categorized under the classes—contact, systemic
(or stomach), and fumigants. Contact pesticides are those, which kill insects
primarily by contact. For example DDT, HCH, Pyrethrum, and so on. Systemic
pesticides kill the pests by entering into their biological system. Parathion is one
such pesticide. Fumigants are easily vaporizable chemicals with insecticidal
properties that easily spread over a large area when used. For example, methyl
bromide (boiling point 4.5° C) is used for fumigation of grains, fruits, and
vegetables.
In the present chapter, we will discuss the synthesis of some selected
insecticides, herbicides, and fungicides.
29B.2 INSECTICIDES
Insecticides are defined as the chemicals, which are used to destroy the insects.
The chemical class of insecticides mainly includes organochlorine,
organophosphorous and carbamate compounds.

29B.2.1 Organochlorine Compounds


(1) DDT (DichloroDiphenylTrichloroethane)
[IUPAC name: 1,1-Bis (4-chlorophenyl)-2,2,2-trichloroethane]
Many organochlorine compounds are commonly used as insecticides and
perhaps the best known insecticide is DDT. It was first prepared by O. Zeidler (a
German chemist) in 1874. However, its insecticidal properties were discovered
by Paul Muller (a Swedish chemist) in 1939. DDT was initially used for the
control of malaria by killing mosquitoes. It is effective against several other
pests, which attack crops and is also effective for flies.
Synthesis. DDT is synthesized by the condensation of two moles of
chlorobenzene with one mole of chloral (trichloroacetaldehyde) in the presence
of 99% sulfuric acid (acts as a powerful dehydrating agent) at about 30°C. The
reaction of chloral occurs preferentially at the para position of chlorobenzene to
give DDT.

On the other hand, commercial DDT is a mixture of various isomers along with
the desired one. For example, ~80% of para isomer along with ~20% ortho-para
isomer and some impurities (due to intial reactants) Purified DDT is a white
crystalline solid having a melting point 108°C.

Residual effect of DDT on environment


The excessive use of DDT results in its accumulation in the
environment, since it is nonbiodegradable. It tends to
accumulate in fat tissues and can cause harm particularly to birds
and aquatic life. Thus, use of DDT has been restricted in many
countries.

Another organochlorine compound similar to DDT is


methoxychlor, which is found to be equally effective in terms of
its insecticidal properties. It has an added advantage of being
biodegradable. Methoxychlor is prepared in a similar way as
DDT but the reactants used for the synthesis are methoxybenzene
and chloral.
(2) HCH (Hexachlorocyclohexane) or Benzene Hexachloride (BHC*)
[IUPAC name: 1,2,3,4,5,6-Hexachlorocyclohexane]
BHC, an abbreviation of benzene hexachloride* is the popular but incorrect
name for this organo-chlorine insecticide (since it does not contain any benzene
ring as indicated by the misnomer). The correct name for the compound is
1,2,3,4,5,6-hexachlorocyclohexane (abbreviated as HCH).
Synthesis. HCH is prepared by passing chlorine gas through benzene in the
presence of ultraviolet light without any catalysts.

Theoretically HCH has eight stereoisomers (conformational isomers), of which


five are found in crude HCH when it is commercially prepared. The five isomers
include α-isomer (~70%), β-isomer (~16%), γ-isomer (13%), and δ, and ε-isomer
(in traces only). Only γ-isomer possesses strong insecticidal properties and is
commonly known as Lindane (named after its discoverer Van der Lindane).
Another popular name the for γ-isomer is Gammexane (derived from its
stereochemistry, that is, γ-isomer of HCH).
The structure of Lindane or Gammexane possesses three chlorine atoms
occupying equatorial positions and rest three occupying axial positions. Its
conformational structure is depicted as follows:

Uses of gammexane: The crude mixture containing 13% γ-isomer (active


ingredient) is used for the control of soil pests and many other sucking and biting
insects. It is also used for the protection of stored grains against the attack of
insects.

29B.2.2 Organophosphorous Compounds


(1) Malathion [O,O-Dimethyl-S-1,2-dicarbethoxyethylphosphorodithioate]
Malathion is among few organophosphorous insecticides which were developed
and used in 1950s. It is the first member of the family of organophosphorous
compounds with a broad spectrum of insecticidal properties and very low
mammalian toxicity (can be used safely).
Synthesis. Malathion is synthesized by the reaction of O,O-
dimethylphosphorodithioic acid with diethylmaleate in one step. The O,O-
dimethylphosphorodithioic acid is synthesized from P4S10 and methanol and
added directly to the diethylmaleate without isolating it from the reaction
mixture. The reaction takes place as follows:
Uses of malathion. It is a contact insecticide with low mammalian toxicity. It is
used widely for the control of aphids, red spiders, mites, and leaf hoppers on a
variety of vegetables and crops. It is also used for eradication of anopheles
mosquitoes.
(2) Parathion [O,O-Diethyl-O-(4-nitrophenyl)phosphorothioate]
Synthesis. Parathion is synthesized by the reaction of O,O-
diethylphosphorothiochloridate with p-nitrophenol in the presence of an alkali.

Uses of parathion. It is a contact, stomach and respiratory poison with high


mammalian toxicity, which restricts its use as an insecticide. It is used against
biting and sucking pests, for example yellow fever mosquito larvae, aphids, red
spiders, mites, and house flies.

Other Insecticides of Common Interest


The most common examples of insects that are a problem for
everyone are mosquitoes, cockroaches, and flies. We shall now
introduce some of the most commonly used household
insecticides.
(A) Pyrethrin is a natural insecticide found in Chrysanthemum
flowers grown widely in Japan and Kenya and is in use since
nineteenth century for keeping household pests away. Now-a-
days synthetic analogues of pyrethrin called pyrethroids are used
for the purpose. Two important examples are as follows:
(a) Allethrin. It is an active ingredient used in mosquito repellent
liquids and mats, which when heated evaporates and eradicate
mosquitoes.
(b) Cypermethrin: It is an active ingredient in the mosquito repellent
coils. It is also a constituent of sticks (lookalikes of blackboard
chalk), which are mainly used to eliminate cockroaches.

(B) Baygon® (also called Propoxur), the most familiar name in


households, is used for killing mosquitoes, cockroaches, and flies.
It belongs to carbamate group of insecticides.
(C) Diethylphthalate is an active component used in formulation
of mosquito repellent creams/lotions that are applied on skin. It is
also used as a fly repellent.

The formulations, which are used for household purposes, contain


very low concentration of insecticides and can be used safely.
29B.3 HERBICIDES
Weeds are undesirable plants that grow with the crops and pose a big problem to
the crops by taking away their large share of nutrients and moisture. If weeds are
spread over large areas they also deprive the plants of sunlight and space.
Growth of weeds is an inevitable process. Besides farmlands, they grow on the
unattended railway tracks, playgrounds, roadsides, and so on. Manual
eradication of weeds from these places is a cumbersome and tedious task. With
the development of organic chemistry, a number of organic compounds are
available for the selective eradication of weeds. These chemicals used for the
removal of undesirable plants from any place are termed as herbicides.
2,4-Dichlorophenoxyacetic acid [2,4-D]
The 2,4-dichlorophenoxyacetic acid (abbreviated as 2,4-D) is one of the most
important herbicide that has revolutionized the research on the use of chemical
compounds as selective plant killers (herbicides).
Synthesis. 2,4-Dichlorophenol is synthesized by the chlorination of phenol.
Then, 2,4-dichlorophenol is converted to its sodium salt, which on reaction with
chloroacetic acid results in the formation of sodium salt of 2,4-D. Further
neutralization with dilute HCl results in the formation of pure 2,4-
dichlorophenoxyacetic acid.

Uses of 2,4-D. It kills the dicotyledonous plants selectively and is thus


conveniently used to protect the monocotyledons.
29B.4 FUNGICIDES
Fungicides are the chemicals which destroy the fungus and thus protect the
crops. There are a number of fungicides belonging to various class of organic
compounds. Here, we discuss one example belonging to dithiocarbamate class
of organic compounds.
Thiram (Tetramethylthiuramdisulfide)
Thiram is an organic compound having a thiocarbamate group. It is one of the
oldest known fungicides.
Synthesis. Thiram is synthesized by the reaction of dimethylamine and carbon
disulfide in sodium hydroxide solution. This reaction results in the formation of
sodium dimethyldithiocarbamate, which on oxidation yields thiram.

Uses of thiram. It is used for the protection of lettuce and strawberry. It is also
used to prevent seeds (by seed dressing) from soil fungi.

Toxic effects of pesticides and treatment.


• Organochlorine pesticides are nerve poisons and increase nervous
excitability, cause tremors, and convulsions. The victim of the
poisoning of organochlorine pesticides is subjected to stomach
washout and is treated with barbiturates, mainly phenobarbiturate.
No fat and oil should be used during the treatment, as
organochlorines are fat-soluble.
• Organophosphorous pesticides cause headache, giddiness,
restlessness, cold sweating, unconsciousness, and in extreme cases
death. The immediate remedy is the removal of the victim from
poisonous environment, and removal of contaminated clothes,
washing of the exposed skin with soap and water (because
organophosphorous pesticides are absorbed by skin) and stomach
wash. The specific antidote for poisoning is atropine.
EXERCISES
Drugs
1. What is a drug? What properties a drug must possess in order to be
considered effective?
2. Define the following terms giving a suitable example.
(a) Antipyretics
(b) Analgesics
(c) Hypnotics
(d) Anti-inflammatory
(e) Tranquilizers
(f) Anti-malarial
(g) Anti-histamine
3. What is LD50? Discuss its significance.
4. Complete the following:

5. How does the side effect of drug differ from its toxic effect?
6. What are sulfa drugs? What is their general mode of action?
7. Carry out the following syntheses:
(a) Sulfaguanidine from aniline
(b) Chloroquine from Novaldiamine
(c) Chloroamphenicol from benzaldehyde and nitroethanol
(d) Phenacetin from p-ethoxynitrobenzene
(e) Phenylbutazone from diethylmalonate.
8. How many stereoisomers are possible for chloroamphenicol? Write their
structures?
9. How would you differentiate between an antiseptic and a disinfectant?
10. Giving suitable explanation, differentiate among following terms:
(a) Toxic effect of drugs
(b) Secondary effect of drugs
(c) Side effect of drug
11. Penicillins are β-lactam ring containing compounds. Give general structure
of penicillins and indicate the β-lactam portion of the structure by encircling
it.
12. To what class of drugs do the following compounds belong?
(a) Aspirin
(b) Barbiturates
(c) Chloroquine
(d) Reserpine
(e) Paracetamol
(f) Phenylbutazone
(g) Sulfadiazine
(h) Penicillin
PESTICIDES
1. What are pesticides?
2. Define the following terms:
(a) Contact pesticides
(b) Systemic pesticides
(c) Fumigants
3. What are insecticides and how are they categorized into different chemical
classes?
4. Give the synthesis of DDT and methoxychlor. Why is methoxychlor a
preferred choice over DDT?
5. How many stereoisomers are possible for HCH? Which isomer of HCH
exhibits insecticidal properties?
6. How is HCH (formally BHC) synthesized?
7. Write the conformation of gammexane (i.e. Lindane).
8. To which class of chemical compounds do malathion and parathion belong?
Give their structures and uses.
9. Name one natural insecticide and its source. Discuss its insecticidal
properties.
10. Give an example (along with its structure) of an insecticide that belongs to
carbamate class of compounds.
11. What are herbicides and fungicides? Explain giving one example of each.
12. What are toxic effects of pesticides?
13. Name the class of pesticides to which the following compounds belong:
(a) HCH
(b) Cypermethrin
(c) 2,4-D
(d) Allethrin
(e) Thiram
(f) DDT
(g) Malathion
(h) Methylbromide
MINI
ESSAY III
ALKALOIDS TERPENES, AND STEROIDS
A. ALKALOIDS
Alkaloids are naturally occurring nitrogen compounds that are generally
obtained from plants. These naturally occurring amines exhibit basicity and for
this reason, they are termed alkaloids which means ‘alkali like’. The main
structural feature of alkaloids is that they contain one or more nitrogen
heterocyclic ring. The alkaloids have marked physiological impact in living
organisms. A majority of alkaloids are excellent pharmaceutical agents and are
widely used as medicine since ancient times.
The alkaloids are extracted from dry plant material by boiling in methanol
followed by acidification and extraction from ether. The addition of base to this
extracted acidic solution results in crystallization of alkaloids. The water soluble
alkaloids are extracted from a suitable solvent. The alkaloids are then purified
and their structure is determined from their synthesis.
Classification: Alkaloids are broadly classified on the basis of their structural
features, as derivatives of phenyl alkyl amine, pyrrolidine, piperidine pyridine,
quinoline, isoquinoline, condensed heterocyclic systems, and so on.
Without delving into the details of such vast field of naturally occurring
compounds, we list (Table 1) some selected and well-known alkaloids and their
physiological importance. There after follow the structures of these compounds.
B. TERPENES
The fragrance of rose, jasmine, lilies, lavender, camphor, sandalwood, and the
like and the flavour of orange, lemon, peppermint, cloves and so on, are ones we
all are well familiar with. The volatile compounds responsible for such flavours
and fragrances are associated with certain parts of the plants (flower, leaves,
fruits, and so on) and are known as terpenes. Terpenes are also the major
constituents of the essential oils, which are the oils distilled from plants and are
generally associated with such fragrance/flavour.
The term ‘terpene’ has originated from one of the constituents (isomeric
C10H16) of turpentine. Terpenes were initially defined as compounds containing
isoprene unit (2-methylbuta-1,3-diene). In a terpene, more than two isoprene
units may be present and for this reason they are also termed as isoprenoids.
Terpenoid is another more appropriate term used for these compounds in wider
sense and includes open chain or cyclic compounds having C5 skeleton units
(isoprene units). These may contain oxygen containing functional groups such as
–OH, C=O and so forth. Thus, the term terpenoids includes a wide range of
natural compounds from terpenes to steroids. Terpenes generally occur in plants,
however, some of the higher terpenes occur in animals too, for example,
lanosterol, and β-carotene.

Isoprene Rule
The relation of terpenes to isoprene (molecular formula C5H8) characterizes all
the members of this very large group of molecules. The isoprene rule initially
postulated by Wallach states that in terpenes a number of isoprene units are
joined together. Later on, Ingold suggested that these isoprene units are joined in
head–tail arrangement. Now instead of head–tail linkage, the term used is 1-4
linkage. In other words, C1 of one isoprene unit (head) is joined to C4 of another
isoprene unit (tail).
Limitations of isoprene rule. In some cases, the isoprene rule is not followed.
For example, in carotenes there is tail–tail linkage of isoprene units.
In accordance with the number of isoprene units in a compound, the terpenes are
classified into some categories, each one of which may contain open chain or
cyclic compounds. The classes of terpenes are summarized in Table 2.

Monoterpenes
Monoterpenoids have ten-carbon atom skeleton and are the simplest terpenes.
These are dimers of isoprene. The monoterpenes are further sub-classified as (a)
acyclic monoterpenes, (b) monocylic monoterpenes, and (c) bicylclic
monoterpenes. A few selected examples of monoterpenes along with sources and
linkages between them are given in Fig. 1.
Fig. 1 Some examples of common monoterpenes (along with their source) and type of linkages of two
isorene units.

Sesquiterpenes
Sesquiterpenes have three isoprene units and have fifteen carbon atoms in their
skeleton. The sesquiterpenes may have acyclic, monocyclic, bicyclic, and
several other structures. A few of the representative sesquiterpenes along with
the linkages of isoprene units are shown in Fig. 2.
Fig. 2 Representative examples of sesquiterpenes (along with their common source) and linkages of three
isoprene units in their structure.

Diterpenes, Triterpenes, and Tetraterpenes


The diterpenes, triterpenes, and tetraterpenes have 20, 30, and 40 carbons
respectively. A well-known alicylic diterpene is phytol, an alcohol that occurs in
the chlorophyll in green plants. Vitamin A is an example of monocyclic
diterpene. An example of triterpene is a hydrocarbon called Squalene (C30H50)
which is found in liver oils of some of the species of shark family.
Carotenoids are the examples of tetraterpenes. Carotenes are reddish yellow
pigments found in plants and animals. β-Carotene is a precursor of vitamin A in
the body. Xanthins are also examples of tetraterpenes and are found in plants
and animals as yellow and red pigments. The structures of representative
examples of these terpenes are given below in Fig. 3.
Diterpene
Fig. 3 Contd.

Fig. 3 Examples of some diterpenes, triterpenes and tetraterpenes.


C. STEROIDS
Steroids are tetracyclic triterpene derivatives that are widely present in plants
and animals. In tetracyclic skeleton of steroids, the rings are numbered as A, B,
C, and D. Cholesterol is one of the most important naturally occurring steroids
which acts as a precursor for a number of other steroids. The basic skeleton of
steriods and the structure of cholesterol is shown as follows:

• The terpenes and steroids resemble in their molecular make up. Surprisingly,
in their biosyntheses isoprene is not the biological precursor.
The biosynthesis of steriods involves isopentylpyrophosphate and
dimethylallylpyrophosphate as precursors. In presence of enzymes, the repetitive
combinations of these units results in the formation of mono-, sesqui-, di-,
triterpenes and so on. Squalene (C30H50) acts as a precursor for steroids. It
undergoes cyclization to form lanosterol which through enzymatic reaction is
converted to cholesterol. The steroids are classified into five major categories:
1. Sterols: These steroids are characterized by the presence of an alcoholic
group and are found in plant oils and animal fat, for example cholesterol.
2. Sex hormones: Unlike hormones sex hormones are not secreted by ductless
gland. But are produced in gonads (testes of male and ovaries of female).
These are termed as steroidal hormones and are classified in these major
categories:
(a) Androgens: Male hormones e.g. testosterone
(b) Oestrogens: Female hormone e.g. oestrone
(c) Gestogens: Corpus Luteum hormone e.g. progesterone
3. Cardiac glycoside: These steroids are extracted from plants and are used
for cardiac therapy, for example, digitoxigenin.
4. Bile acids: These are the organic compounds present in bile. The bile is a
liquid secreted by liver and is stored in gall bladder and is essential for
digestion.
5. Sapogenins: These are present in the plant glycosides and their colloidal
solution in water form foam similar to soap (Latin sapo means soap). Their
solutions are highly toxic as they destroy the red blook cells.
The structures of some commonly found steroids are:
Chapter30
Dyes

30.1 INTRODUCTION
Dyes are organic compounds, which impart colour to the fibre. It not only
adheres to the surface of the fibre but also penetrates deep into it. For it to be a
good dye, the colour of a dye should be resistant towards heat, light, moisture,
dilute acids, washing soaps, and so on. A dye that permanently fixes on the
surface of the fibre is known as a fast dye where as a dye that fades out, or is
washed off is known as a fugitive dye.
There is no definite procedure for the nomenclature of dyes and these are
commonly known by their commercial names, as given by the manufacturer.
This is the disadvantage since the same dye may have different names. Thus,
‘Society of Dyes and Colourists’ has proposed a colour index and each dye is
given a colour index number (abbreviated as C.I.No.).

30.1.1 Theory of Dyeing


Nature of the fibre and that of the dye are two important factors in the dyeing of
a fibre. The nature of fibre depends upon its origin. The origin of fibres may be
natural (plants and animals) or synthetic (man made fibres).
(a) From plants. The fibres obtained from plants are mainly cotton, linen, and
jute. These are neutral as they contain cellulose (which is a natural polymer
having hydroxyl groups).
(b) From animals. The fibres obtained from animal sources include silk, wool,
and leather. Silk and wool are basically proteins and are amphoteric in nature.
(c) Synthetic fibres (man made fibres). Synthetic fibres may be cellulose based
such as cellulose acetate or may be polyamides and polyesters like rayon, nylon,
and decron.
Depending upon the nature of the fibre, the dye forms a linkage through ionic
interactions,
van der Waals interactions, hydrogen bonding, or covalent bonding. In case of
fibres of the plant origin, dye adheres to the plant polymer (cellulose) through
hydrogen bonding. In case of animal generated fibre, there is a salt formation of
fibre (having peptide linkages) with the dye.

30.1.2 Classification of Dyes


Dyes are classified into two types based on (a) their mode of application on the
fibres and
(b) chemical structure of dyes.
Mode of application on fibres
This classification is based on the various methods of dyeing different fibres
with dyes. According to this, dyes can be classified into following seven types:
(i) Acid dyes
(v) Vat dyes
(ii) Basic dyes
(vi) Azo dyes (or Ingrain or Developed dyes)
(iii) Direct or substantive dyes
(vii) Sulfur dyes.
(iv) Mordant dyes (or Indirect or Adjective dyes)
(i) Acid dyes. These are the sodium salts of sulfonic acids or nitrophenols. These
can be used for dyeing animal fibres and synthetic fibres, for example, martius
yellow, naphthol yellow-S, and picric acid.
(ii) Basic dyes. These are the cationic dyes containing basic groups such as –
NH2, –NHR, –NR2, and their salts (mostly in the form of hydrochloride salts).
These are used for dyeing animal fibres and can also be used for dyeing fibres of
plant origin after mordanting with tannins. The dyes in this category include
malachite green, magenta, and para-rosaniline
(iii) Direct or substantive dyes. This class of dyes mainly includes various acid
or basic dyes and can be directly used for the fibres of plant origin. The dyeing
of the fibres is carried out in the presence of common salt, so these dyes are also
called salt dyes such as congo red.
(iv) Mordant dyes (or indirect or adjective dyes). These dyes are insoluble in
water and a third substance is used as a binding material of the fibre with dye.
This third substance is called mordant. In the dyeing process, fibre is dipped in
mordant, dried and then again dipped into the dye solution. Mordant forms a
complex with the dye and is deposited on the fibre giving it a permanent shade.
For acid dyes, a basic mordant [like metal salts Fe(OH)3, Al(OH)3 ] is used and
for basic dyes, an acid mordant such as tannic acid is used. Alizarin is an
example of mordant dyes.
(v) Vat dyes. These dyes are applied directly on the fibre and are generally
insoluble in water. These can be used only on cotton and not on silk and wool.
The dyeing, in this case, is a continuous process and is carried out in a large
vessel called vat. For this reason, these dyes are termed as vat dyes. Indigo is an
example of vat dye.
These dyes are first of all converted into water soluble form by reduction
(referred to as vatting) in alkaline medium. The fabric to be dyed is dipped in the
solution of the reduced dye which is called the dihydrodye solution. The fabric is
then dried in air where oxidation takes place and coloured fabric is obtained.

(vi) Azo dyes (or ingrain or developed). These are applied directly on the fibre.
The process includes the diazotization and the coupling reaction at low
temperature on the fibre itself.
(vii) Sulfur dyes. These are complex water insoluble dyes containing sulfur. But
these dyes are soluble in sodium sulfide (Na2S) solution and thus the dyeing
process is carried out in Na2S solution. These dyes are generally used for dyeing
cotton fibre.
Chemical structure of dyes
Due to the variation in the structures of dyes, no distinct classification can be
given but roughly their classification can be given depending on their chemical
constitution. Some selected chemical classes of dyes are listed here as follows:
(i) Azo dyes (v) Anthraquinone dyes
(ii) Diphenylmethane dyes (vi) Indigoid dyes
(iii) Triphenylmethane dyes (vii) Phthalocyanine dyes
(iv) Phthalein dyes (viii) Nitro and nitroso dyes
30.2 COLOUR AND STRUCTURE
An organic compound appears coloured only if it absorbs the radiations in the
visible part of the electromagnetic spectrum. The visible region extends from
400–800 nm. The white light in the visible region consists of seven colours,
namely Violet, Indigo, Blue, Green, Yellow, Orange, and Red (VIBGYOR).
When the white light falls on an object, it may be:
(1) totally reflected back, and in such a case the object appears white.
(2) totally absorbed, and in such case the object appears black.
(3) a part of light is absorbed and the rest is reflected back, and in such case,
the object appears coloured.
Now consider case (3). The colour of the object depends upon the waevlength of
the light reflected back (that is, not absorbed). For example, a substance appears
red if it absorbs all the light waves except that with wavelength λ = 700 nm, that
is, it reflects the light which appears red to eyes.
However, if the substance absorbs light of a particular wavelength, our eyes
perceive the complementary colour. For each colour in the visible region there is
a complementary colour. For example, a substance that absorbs light of
wavelength 700 nm basically absorbs the red colour from white light and reflects
the rest. Thus, it appears blue-green. For this reason, we consider colours blue-
green and red to be complementary.
The colours and complementary colours perceived by eyes for the wavelength of
light reflected or absorbed by the substance have been summarized in Table
30.1.
The different theories that have been proposed for correlation of colour and
structure of organic compounds are as follows:

30.2.1 Witt Theory


German chemist O.N. Witt (1876) suggested the Chromophore–Auxochrome
theory for coloured organic compounds. The various terms defined accordingly
are as follows:
Chromophores (In Greek, chroma means colour and phoron means bearer). The
groups with unsaturation and electron withdrawing nature have an appreciable
effect on the absorption of light and when present in conjugation, are responsible
for the colour of compound (due to absorption in visible region). Such groups
are known as chromophores. The compounds containing chromophore group are
known as chromogens. A group, which when present singly is not able to impart
colour to a compound is termed as a weak chromophore. On the other hand, if a
single group is sufficient to give colour to the compound, it is termed a strong
chromophore. Some examples of chromophore groups are:

Auxochromes. These groups are not responsible for colour but when present
along with chromophore groups are responsible for deepening of the colour.
These are electron-donating groups. Some examples of auxochrome groups are:

The effect of chromophore and auxochrome groups on the colour of the


compound is illustrated in the following examples:

Limitations of Witt’s theory. There is no explanation why some groups act as


chromophores and others as auxochromes. Further, there is no explanation for
deep colours of indigo and triphenyl methane dyes, which have only weak
chromophores.

30.2.2 Quinonoid Theory


According to this theory, ortho or para quinonoid structures in a compound are
responsible for its colour. The drawback of this theory is that it has no
explanation for deep colour of azo dyes, which have no quinonoid structure.

30.2.3 Modern Theory


The modern theory is basically a modification of Witt’s and Quinonoid theories.
It explains the colour of organic compounds based on resonance effect and its
correlation with absorption of light (refer Chapter 32).
The light waves in the UV and visible region have high energy. When light falls
on a compound, it gets absorbed and results in three types of excitations in the
molecule, namely electronic, vibrational and rotational.
The compounds with single or multiple bonds undergo electronic excitation in
UV–visible region. In case of multiple bonded compounds, the π-electrons are
responsible for selective absorption and the electronic excitation.
If E1 is the energy in ground state and E2 is the energy in excited state, energy
required (ΔE) for excitation,
ΔE = E2 – E1 = hν = hc/λ
This shows that the frequency of light (ν) absorbed by a compound is related
directly to ΔE. This means that,
(a) If ΔE has a high value, a high frequency (ν) is absorbed, that is, absorption
occurs at a shorter wavelength (λ).
(b) If ΔE has a low value, low frequency (ν) is absorbed, that is, absorption
occurs in a high wavelength (λ) region called the visible region.
In organic molecules with conjugated multiple bonded systems, the
delocalization of π-electrons occurs. This delocalization, known as mesomeric or
resonance effect, causes stabilization in excited state and thus decreases ΔE
value. As a result, longer wavelength absorption occurs (visible region) and the
compound appears coloured.
The chromophores (weak or strong) and auxochrome groups present in a
compound cause deepening of colour by increasing the number of charged
contributing structures during resonance effect. The increased conjugation (or
delocalization) in a system shifts the absorption towards the longer wavelength
region (lower energy) and is known as bathochromic shift. Similarly, shifting of
absorption towards shorter wavelength region (higher energy) is known as
hypsochromic shift.
In short, higher the number of contributing structures in mesomeric or
resonance effect, lower will be the ΔE value and thus deeper will be colour
observed for that compound.
The following examples will further clarify the various aspects of modern theory
in relation with colour and structure of dyes.
Example 1. Nitrobenzene is yellow in colour whereas p-nitroaniline is orange in
colour. The –NO2 group is a chromophore which imparts a yellow colour due to
resonance effect.
On the other hand, the –NH2 group is an auxochrome which causes deepening of
colour by increasing the number of contributing structures during resonance
effect. Similarly, picric acid has a dark yellow colour due to increased number of
contributing structures because of the presence of three –NO2 groups.

Example 2. The p-aminoazobenzene is yellow in colour but in acidic medium, it


appears violet in colour. The contributing structures in p-aminoazobenzene
depict charge separation.

In acidic medium, the presence of single charge in contributing structures


imparts stability (lowering of ΔE). Thus, absorption occurs at longer
wavelengths and deepening of colour is observed. The contributing structures in
this case are:
Example 3. Both o-quinone and p-quinone undergo resonance but p-quinone is
yellow in colour and o-quinone is deep red. The respective contributing
structures are depicted as follows:

Example 4. In geometrical isomers of stilbene, one of them, trans-stilbene is


coloured while
cis-stilbene is colourless. In cis-isomer, resonance is not possible due to steric
hindrance while
trans-isomer exhibits resonance. The delocalization causes absorption to take
place in higher wavelength region and thus the trans isomer is coloured (also
refer Chapter 32, p. 1119).
Beside the natural dyes mentioned in Notabilia, other natural dyes like alizarin
and indigo have been discussed in details in the text.
30.3 SYNTHESIS AND APPLICATION OF DYES

30.3.1 Azo Dyes


(1) Methyl orange [mode of application: acid dye]
Methyl orange is synthesized by coupling diazotised sulfanilic acid with N,N-
dimethylaniline at low temperature (0–5°C). Diazotised sulfanilic acid is
prepared by treating sulfanilic acid with nitrous acid (prepared in-situ from
NaNO2/HCl) at 0–5°C

It is a yellow coloured fugitive dye, which is used extensively as an indicator in


acid-base titrations (pH range 3.0–4.4). In alkaline solution, it gives a yellow
colour while in acidic solution it gives a red colour due to a quinonoid structure.

(2) Methyl red [mode of application: acid dye]


Methyl red is synthesized by coupling diazotised anthranilic acid with N,N-
dimethylaniline at low temperature (0–5°C). The diazotised anthranilic acid is
prepared by treating anthranilic acid with nitrous acid (generated in-situ from
NaNO2/HCl) at 0–5°C.
It is used as an indicator in acid-base titrations (pH range 4.4–6.3). In alkali
solution, it gives a yellow colour while in acidic solution, it gives a red colour
due to the presence of a quinonoid structure.

(3) Congo red [mode of application: direct dye]


The Congo red is a fugitive dye. It is synthesized by coupling tetra-azotized
benzidine with
1-naphthylamine-4-sulfonic acid (naphthionic acid) at low temperature. The
coupling with naphthionic acid occurs at 2-position with respect to –NH2. The
tetra-azotized benzidine is prepared by treating benzidine with nitrous acid
(generated in-situ from NaNO2/HCl) at 0–5°C.
This is a direct cotton dye of commercial importance. It is also used as an
indicator in acid base titrations. The dye imparts a red colour in alkaline medium
and a blue colour in strong acidic medium. This dye is used for distinguishing
weak and strong acids using congo-red papers. Deepening of colour in strong
acidic solution is due to resonance stabilization of the charged structure. This is
shown as follows:
30.3.2 Triphenylmethane Dyes
(1) Malachite green [mode of application: basic dye]
The synthesis of malachite green involves condensation of benzaldehyde with
two molecules of N,N-dimethylaniline in the presence of concentrated sulfuric
acid. This results in the formation of a substituted triphenylmethane derivative,
which is colourless and is known as a leuco base. The leuco base on oxidation
with lead oxide (PbO2) in presence of HCl gives a colourless carbinol base also
known as colour base of malachite green. This colour base on treatment with
excess of HCl gives malachite green as a dye salt as shown in [I].
The green colour of the dye is due to the delocalization of electrons (extended
conjugated system), as indicated by the three contributing structures [I–III] for
dye salt. These structures are—

It is used for direct dyeing of silk, wool, leather, and acetate rayon fibres. It can
also be used for dyeing cotton after mordanting it with tannins.
(2) Rosaniline [mode of application: basic dye]
[Other names: Fuschine, Magenta]
Rosaniline is synthesized by the condensation of an equimolar mixture of
aniline, o-methylaniline (o-toluidine), and p-methylaniline (p-toluidine) in the
presence of an oxidizing agent (nitrobenzene in presence of iron). This results in
the formation of leuco base, which on oxidation with lead oxide (PbO2) in acidic
medium gives a colour base. The colour base on treating with excess of HCl
gives rosaniline as dye salt [I].

The colour of the dye is due to the delocalization of electrons (extended


conjugated system), as indicated by the three contributing structures [I–III] for
dye salt, shown as follows:
The crystals of rosaniline possess a green metallic lustre and its solution in water
gives a violet-red (magenta) colour. The coloured solution on passing sulfur
dioxide over it gives a colourless solution known as Schiff’s reagent. This
reagent is used for qualitative detection of aldehydes. It is used for direct dyeing
of wool and silk, however, vegetable fibres are dyed after mordanting with
tannins.
(3) Pararosaniline [mode of application: basic dye]
Pararosaniline is synthesized by condensation of two moles of aniline and one
mole of
p-methylaniline (p-toluidine) in the presence of oxidizing agent (nitrobenzene
and iron). This results in the formation of a leuco base which on oxidation with
PbO2 in HCl followed by a treatment with excess HCl gives pararosaniline as
dye salt [I]. Pararosaniline is used an direct dye for wool and silk.
The colour of the dye is due to the delocalization of electrons (extended
conjugated system), which is indicated by the three contributing structures [I–
III] for dye salt as follows:

(4) Crystal violet [mode of application: basic dye]


The reaction of N,N-dimethylaniline with phosgene (carbonyl chloride) results
in the formation of Michler’s ketone (I) which further condenses with another
molecule of N,N-dimethylaniline to give a colour base (II). The treatment of the
colour base with excess of HCl gives crystal violet as a dye salt (III). This is a
direct dye and is used for wool and silk.
Its solution in water is violet in colour. The colour of crystal violet changes with
the pH of the solution in the following manner:
(i) In presence of dilute acids, it gives a purple colour due to the presence of
single charged ion (III).
(ii) In strong acid solution, it gives a green colour due to the presence of
doubly charged ion (IV).
(iii) In highly acidic solution, it gives a yellow colour due to the presence of
triply charged ion, which cannot undergo resonance. Thus, decrease in the
intensity of colour is seen in strongly acidic medium (V).
30.3.3 Phthalein Dyes
(1) Phenolphthalein [mode of application: acid dye]
The synthesis of phenolphthalein is carried out by condensation of phenol with
phthalic anhydride in the presence of zinc chloride.

It is a white solid, which in alkaline solution gives red colour and is colourless in
acidic solution. In alkaline medium, it exists as a disodium salt, which exhibits a
quinonoid structure that undergoes resonance in the manner as shown:
In strong alkaline solution, the red colour of the dye disappears due to formation
of a trisodium salt (loss of the quinonoid structure) that cannot undergo
resonance. Phenolphthalein is used as an indicator in acid–base titrations.
(2) Fluorescein [mode of application: acid dye]
Fluorescein is synthesized by condensation of phthalic anhydride with resorcinol
in the presence of concentrated sulfuric acid as:

It is an orange solid, which in alkaline solution gives a reddish brown colour.


This on dilution imparts yellow-green fluorescence. The sodium salt of
fluorescein is known as uranine.
30.3.4 Anthraquinone Dye
Alizarin (1,2-Dihydroxyanthraquinone) [mode of application: mordant dye]
Alizarin occurs in nature as glucoside in the roots of the madder plant. The
synthesis of alizarin involves the oxidation of anthracene (I) with potassium
dichromate to anthraquinone (II). Sulfonation of (II) with fuming sulfuric acid at
high temperature (200°C) followed by treatment with sodium carbonate gives
sodium salt of anthraquinone-2-sulfonic acid (III). The fusion of (III) with a
mixture of sodium hydroxide and potassium chlorate gives disodium salt of
alizarin (IV) that on acidification yields alizarin.

Alizarin exists as yellow flakes and in aqueous alkali gives violet colour. It is a
mordant dye and in presence of different metallic mordants, it produces different
colours. Table 30.2 gives the list of various colours imparted by alizarin in
presence of different mordants.
Following figure shows the interaction of alizarin with a fibre in the presence of
a mordant.

The alizarin with aluminium and iron mordants is used for dyeing and printing
cotton while with aluminium and chromium mordants, it is used for dyeing
wool.

30.3.5 Indigotin Dye


Indigo [mode of application: vat dye]
[Other name: Indigotin]
Indigo is the oldest known dye and is found in the plants of Indigofera group.
Indigo can be synthesized by a number of methods. This section discusses a few
important industrial syntheses.
Heumann synthesis (first method). This method involves heating of aniline
with chloroacetic acid, which forms phenylglycine (I), which on treatment with
sodium or potassium hydroxide forms corresponding salt (II). The fusion of this
salt with sodamide and a mixture of sodium and potassium hydroxide gives
indoxyl (III), which on atmospheric oxidation yields indigo (indigotin).
Heumann synthesis (second method). Anthranilic acid (I) is treated with
chloroacetic acid, which results in the formation of phenylglycine-o-carboxylic
acid (II). On fusion with mixture of
potassium hydroxide and sodamide, (II) undergoes ring closure followed by
decarboxylation to form indoxyl (III), which on atmospheric oxidation yields
indigo (indigotin). The sequence of reactions is as follows:

In view of the central carbon–carbon double bond, indigo is expected to exhibit


geometrical isomerism. However, only trans-form (E-isomer) has ever been
isolated from natural sources since it is stabilized by hydrogen bonding. It has
been suggested that the chromophore which is responsible for its deep blue
colour is a fraction of the structure where the –NH and C=O groups are attached
to carbon–carbon double bond as shown below:
Indigo is a deep blue coloured dyestuff, which is used for dyeing cotton and
rayon fabrics.
EXERCISES
1. What are dyes? What do you understand by the terms ‘fast dye’ and ‘fugitive
dye’?
2. Explain the following terms giving suitable examples:
(a) Acid dyes
(b) Basic dyes
(c) Mordant dyes
(d) Vat dyes
(e) Phthalein dyes
3. What is Witt’s theory of colour and constitution (structure) of dye and what
are its limitations?
4. How the ‘quinonoid theory explains the colour and its relationship with the
structure of molecules? What are its limitations?
5. Explain the Modern theory of colour and constitution, with suitable examples,
that explains the relationship of colours with the structural features of organic
compounds?
6. What are auxochrome and chromophores? Explain with suitable examples.
7. Label the auxochrome and chromophore groups in the following structures:

8. What is ‘leuco base’ and how it is converted into dye?


9. In alkaline solution phenolphthalein gives pink colour, however in excess of
alkali it gives a colourless solution. Explain.
10. ‘Crystal violet gives purple colour in weakly acidic solution, green colour in
strong acid, and in highly acidic solution it gives yellow colour’. Explain.
11. Malachite green belongs to which class of dyes? Name the other dyes that
have structural features similar to malachite green.
12. Give the synthesis of methyl orange and explain why its colour changes to
red in acidic medium?
13. Give the structure and synthesis of alizarin. To which class of dyes alizarin
belong and how alizarin imparts the colour to the fibres?
14. Give the structure and synthesis of indigo. In the structure of indigo, encircle
the chromophore portion responsible for its deep blue colour.
Chapter31
Synthetic Polymers

31.1 INTRODUCTION
A polymer (Greek poly means many, and meros means parts) is defined as a
large molecule of high molecular weight formed by combination of a number of
one or more types of molecules of low molecular weight. Being molecules of
high molecular weight, they are also known as macromolecules. The smaller
molecules, which occur as repeating units to make up a polymer, are known as
monomers.
In other words, a polymer may be defined as a number of repeating chemical
units (monomers) held together by covalent bonds. The process by which
monomers combine to form a polymer is known as polymerization. The extent to
which the number of monomer units combines to form a polymer is known as
degree of polymerization and is represented by ratio of the average molecular
weight of polymer to molecular weight of the monomer.
31.2 CLASSIFICATION OF POLYMERS
Polymers are classified in a number of ways depending upon their source,
structure, physical properties, and type of polymerization reaction. These are:
(1) On the basis of source. On the basis of source, the polymers are classified
as:
(a) Natural polymers. These polymers are obtained from natural resources.
For example, proteins, cellulose, starch, rubber, and so on.
(b) Synthetic polymers. These polymers are synthesized by chemical
reactions. For example, polyethylene, teflon, nylon, and so on.
(2) On the basis of structure. The structure of a polymer depends upon (i) the
composition of basic polymer unit and (ii) the arrangement of the polymer
chains. Polymers can be classified as follows under these categories:
[i] Based on the composition of basic polymer unit, polymers are classified as
(a) Homopolymers. These polymers are made from only one type of
monomer unit. For example, polyethylene is synthesized by the
polymerization of a single monomer, namely ethylene (IUPAC name—
ethene).
(b) Copolymers. These polymers comprise two or more monomers. An
example is synthetic rubber, which is a copolymer of two monomers—
styrene and 1,3-butadiene.
[ii] Based on the arrangement of the polymer chains, polymers can be
classified as:
(a) Linear polymers. These are also known as one-dimensional polymers where
monomer units are held together in a long chain without any lateral
linkage or branching.
(b) Cross-linked polymers. In these polymers, different linear chains of
monomer units are cross-linked with each other through covalent bonds.
They may be further categorized into
• Sheet polymers. These are also known as two-dimensional polymers
where linear chains of monomer units are joined with each other in such
a manner that the length of the chain is more than its width.
• Three-dimensional polymers. In these, the linear chains are cross-linked
to form a huge network, which results in the formation of a thick
polymer product, for example, bakelite.
The schematic diagrams for classification of polymers based on composition and
arrangement of polymer chains are depicted in Figs. 31.1 and 31.2 respectively.

Fig. 31.1 Schematic diagrams of (a) homopolymer; (b) Branched polymer, (c) cross-linked polymer; and
(d) copolymer (P and Q indicate different monomer units).

Fig. 31.2 Schematic diagrams of (a) linear polymer; (b) sheet polymer and (c) three-dimensional polymer.
31.3 PHYSICAL PROPERTIES AND
CLASSIFICATION
Polymers may be classified on the basis of their properties as crystalline and
amorphous polymers.
Crystalline polymers. Due to the arrangement of polymer chains in a linear
regular pattern, the polymer exists as a crystalline solid. The crystalline
polymers being hard can be drawn into fibres.
Amorphous polymers. In Amorphous polymers, there is an irregular
arrangement of polymer chains (coiled or random) and they exist as amorphous
solids. Amorphous solids may exhibit elasticity and such polymers are known as
elastomers, for example, rubber. The amorphous solids, which do not exhibit
elasticity, are termed as plastics.
Depending upon the structure of a polymer and its behaviour at different
temperatures, the polymers may be classified as thermoplastics and
thermosetting plastics.
Thermoplastics. These polymers soften on heating and can be moulded into
different shapes. They become hard on cooling and regain their physical
properties. The process of heating and cooling can be repeated a number of times
without causing any change in their physical properties. For example
polyethylene, polyvinyl chloride (PVC), and nylon.
Thermosetting plastics. These polymers can be moulded into different shapes
by heating and they become hard when heated. The heat treatment is given only
during the preparation of various articles from these polymers. Once solidified
and set, they cannot be softened or remoulded on heating. Bakelite and
melamine are two such thermosetting plastics.
31.3.1 Stereochemistry of Polymers
In the formation of a polymer, there may be generation of stereocentres (chiral
centres) depending upon the structure of monomer units. In general,
monosubstituted vinyl compounds on polymerization result in the formation of a
polymer where alternate carbons in the chain are chiral in nature. The
arrangement of atoms or groups at these stereocentres affects the physical
properties of polymers. Depending upon the configuration at stereocentres, the
polymers can be classified as atactic, syndiotactic and isotactic.
(a) Isotactic polymer. In these polymers, all the stereocentres in the chain have
same configuration. This regular arrangement results in the formation of an
extremely fine quality polymer.
(b) Syndiotactic polymer. In these polymers, the configuration of stereocentre
alternates, for example, R, S, R, S... This orderly arrangement results in the
production of a good quality polymer.
(c) Atactic polymer. In these polymers, the stereocentres have a random
configuration. This disorderly arrangement results in a very poor quality
polymer.
While isotactic and syndiotactic polymers are generated through Ziegler–Natta
polymerization, atactic polymers are produced in the presence of peroxides. As
an example, the stereoisomeric forms of polypropylene are illustrated in Fig.
31.3.
Fig. 31.3 Stereoisomeric forms of polypropylene.
31.4 POLYMERIZATION REACTIONS
(1) Addition polymerization. Addition polymerization occurs in monomer units
having double or triple bonds. It involves the combination of a large number of
monomer units by addition reaction. In general, compounds containing
(Z is substituent) undergo addition polymerization which is popularly known as
vinyl polymerization as shown in Table 31.2). This polymerization is also known
as chain growth polymerization. The monomer units may be joined in a head–
tail arrangement, or a head–head and tail–tail arrangement, or it may be a
random arrangement. The most favourable arrangement is head–tail
arrangement.

(2) Condensation polymerization. It involves the combination of a large


number of monomer units with the elimination of simple molecules like water,
ammonia, and so on. This polymerization is also known as step growth
polymerization. In condensation polymers the monomer units contain two or
more functional groups and generally the condensation occurs between two
different monomer units. For example, polyesters, polyamides, polyurethane,
and bakelite.
31.4.1 Addition Polymers
Mechanism of Addition Polymerization
In the formation of addition polymers (chain growth polymers), the reaction is
initiated by a catalyst, which results in the formation of a reactive intermediate.
This intermediate then adds on to the monomer unit to generate a new
intermediate, which adds on to another monomer unit and the process goes on.
Depending upon the reactive intermediate formed, the polymerization reaction
may follow:
(a) Free radical mechanism
(b) Cationic mechanism
(c) Anionic mechanism
(d) Ziegler–Natta polymerization
(a) Free radical mechanism. The addition polymerization reactions, which are
carried out in presence of peroxides or potassium perborate, follow a free radical
mechanism. The addition occurs through a free radical intermediate and is also
known as radical polymerization. The radical polymerization occurs in head to
tail manner. The mechanism involves following steps:
Step 1. Chain initiation

Step 2. Chain propagation

Step 3. Chain termination


(b) Cationic mechanism. The addition polymerization reactions, which are
carried out in the presence of strong acids like sulfuric acid, halogen acids or in
the presence of Lewis acids (BF3, AlCl3) follow a cationic mechanism. The
protonation of alkene results in the formation of carbocation intermediates. Thus,
the growing chains in polymerization are cations and the process is known as
cationic polymerization. For example, the cationic mechanism for
polymerization of
2-methylpropene (isobutene) involves following steps:
Step 1. Chain initiation

Step 2. Chain propagation

Step 3. Chain termination

(c) Anionic mechanism. The addition polymerization reactions, which are


carried out in presence of strong bases like sodamide or metal alkyls follow
anionic mechanism. The alkenes containing an electron withdrawing group
generally undergo anionic polymerization. The electron withdrawing groups
facilitate the attack of base on the olefinic carbon to which they are attached. The
mechanism involves following steps:
Step 1. Chain initiation

Step 2. Chain propagation

Step 3. Chain termination

(d) Ziegler–Natta polymerization (Coordination polymerization). The free


radical, cationic, or anionic polymerizations generally result in the formation of
addition polymers, which are atactic. In 1953, two scientists Karl Ziegler
(Germany) and Giulio Natta (Italy) independently synthesized “stereoregular”
(isotactic and syndiotactic) addition polymers. The stereoregular polymers are
high melting, crystalline solids. These have much better properties and wider
range of applications than atactic polymers.
The Ziegler–Natta polymerization is carried out in the presence of a mixture of
titanium tetrachloride (TiCl4) and triethylaluminium as catalyst, popularly called
Ziegler–Natta catalyst. The polymerization is also known as coordination
polymerization. The reaction is carried out at a low temperature (<100°C) and
low pressure (~10 atm) in a nonpolar solvent. The reaction takes place as
follows:

Mechanism. The catalyst has a complex structure. The alkyl titanium bond
undergoes insertion of monomer units. The π-electrons of monomer are
coordinated to a vacant site on metal (This being the reason that it is known as
coordination polymerization). The coordinated monomer then inserts into
titanium–carbon bond and the process goes on to form a polymer. Since titanium
is attached to different ligands, the coordination of monomer units follow a
stereoregular pattern. The various steps of reaction mechanism are given
schematically in Fig. 31.4, as follows:
Fig. 31.4 Ziegler–Natta polymerization: schematic and simplified representation of (a) formation of
polyethylene (b) polypropylene formation (further simplified); (c) mechanism for the formation of
polyethylene.

Characteristics of Ziegler–Natta polymers. The polymers manufactured by


using Ziegler–Natta catalyst have the following important characteristics:
(1) They are high melting, highly crystalline, isotactic polymers having high
molecular mass.
(2) They are tough and flexible, so can be moulded easily and are resistant to
acids and alkalis.
Plasticizers
Some of the polymers are found as hard resins. Plasticizers are substances,
which are added in the polymer formulation to alter the physical properties of
polymers. For example, they are added for softening of hard polymers. The
plasticizers are generally liquids with low volatility or may be solids, which
decrease the attraction between polymer chain thereby making them more
flexible. The commonly used plasticizers are tricresylphosphate [(p–CH3–
C6H4O)3P=O], dioctylphthalate (DOP), diisodosylphthalate (DIP),
alkanesulfonyl chloride (RSO2Cl; R=long chain alkyl group), glycerol, camphor,
and so on.
For example, the addition of a plasticizer to polyvinyl chloride (PVC), a hard
resin, converts its texture to a rubber like texture. The most commonly used
platicizer for PVC is dioctylphthalate (DOP) whose structure is as follows:

Synthesis and uses of vinyl polymers


The polymerization of vinyl monomers generally occurs in the presence of
peroxides and follows a free radical mechanism. However in case of
polyethylene and polypropylene an ionic or Ziegler–Natta mechanism are also
followed (refer Fig. 31.4). The preparation of polypropylene is carried out
exclusively through Ziegler–Natta polymerization to obtain a stereoregular
polymer of high quality. The general reaction for vinyl polymerization is
represented as follows:

Some of the vinyl polymers, along with their monomer units and common uses
are listed in
Synthesis of monomers of some vinyl polymers
1. Polyvinyl chloride (PVC). Vinyl chloride is a monomer of polyvinyl chloride
and is synthesized industrially from ethene or acetylene as follows:

2. Teflon (Polytetrafluoroethene). Tetrafluoroethene is a monomer of teflon


(polytetrafluoroethene) and is synthesized industrially from choroform and
hydrofluoric acid as follows:

3. Polyvinylacetate. Vinyl acetate, a monomer of polyvinylacetate, is


synthesized industrially by oxidative acetylation of ethene in the following
manner:

4. Polyacrylonitrile (orlon). Acrylonitrile is a monomer of polyacrylonitrile and


its synthesis is exhibited by following reactions:

5. Polystyrene. Styrene, the monomer of polystyrene, is synthesized by


dehydrogenation of ethylbenzene as
If air is blown through during the process of polymerization of styrene, a foamed
plastic popularly known as thermocole, is formed. It is used extensively for
thermal insulation (in cold storages, air conditioners, auditoriums, etc.), as
protective screens in radars, for packaging and decorative purposes.

31.4.2 Condensation Polymers


Preparations of some condensation polymers
1. Nylon 6,6. Nylon 6,6 is a copolymer and is prepared by condensation of
hexamethylenediamine and adipic acid. During condensation, loss of water
molecule takes place to form a number of amide linkages (polyamide) to yield
nylon 6,6. Each monomer unit (diamine and diacid) has six carbons each and
therefore the name, nylon 6,6. Adipic acid is obtained by oxidation of
cyclohexanone.
Synthesis of monomers

Condensation of monomer units (formation of Nylon 6,6)

2. Nylon 6. Nylon 6 is prepared from caprolactam, which is a cyclic amide


containing six carbons. On heating, the cyclic amide ring opens up and amino
group of one molecule reacts with carbonyl group of the other molecule to form
polyamide. During condensation, loss of water molecule occurs to form a
number of amide linkages (polyamide) to give nylon 6. Caprolactam is prepared
from cyclohexanone by a series of reactions. Nylon 6 is so named becasue only
one monomer (having six carbons) is used for its preparation.
Synthesis of monomer

Condensation of caprolacatam units (formation of Nylon 6)

3. Polyesters
(i) Dacron [terylene]. Dacron, a copolymer, is prepared by the condensation of
methyl ester of terphthalic acid with ethylene glycol. The condensation involves
loss of a methanol molecule, which results in the formation of a number of ester
linkages (polyester).
This polyester can be fabricated into a strong film, which is known by the name
Mylar. It is a light weight, high strength, transparent film, which is used for
protection of artwork and important documents.
(ii) Polycarbonates. These are polyesters formed by condensation of derivatives
of phenol and carbonic acid. For example, condensation of bis-phenol A with
phosgene results in the formation of a polycarbonate, known by the name of
lexan. The reaction takes place as follows:
4. Polyurethanes. The reaction of an isocyanate and alcohol results in the
formation of a carbamate, which is known as urethane.
General reaction

Polyurethanes are copolymers and are prepared by the reaction of alkyl or aryl
diisocyanates with diols. In general, the diol used in the preparation is a
copolymer of ethylene glycol and adipic acid, which has free hydroxy (–OH)
end groups. The commonly used diisocyanate is toluene-2,4-diisocyanate
The polymerization in the presence of water results in the
formation of polyurethane foam. Water reacts with the isocyanate
group to produce carbamic acid, which spontaneously looses carbon
dioxide. The carbon dioxide thus generates bubbles (giving porosity to
polymer) to yield polyurethane foam. The density of foam is
dependent on the amount of carbon dioxide evolved during the
process.

5. Bakelite (Phenol-formaldehyde resin). Bakelite is a copolymer of phenol


and formaldehyde. It is a thermosetting polymer and has highly cross-linked
structure. The methylene groups at ortho and para positions participate in cross
linkages. The condensation of phenol and formaldehyde can occur in acidic or
alkaline medium.
• In acidic medium, condensation results in the formation of a linear polymer
known as Novolak. This polymer further polymerizes in basic medium to
yield bakelite.

Here, polymerization is shown with o-methylolphenol (other type of linear


polymers are also formed).
• In alkaline medium, condensation results in the formation of a linear polymer
known as Resol. Heating resol polymer results in the formation of bakelite.
• Both novolak and resol are thermoplastic polymers but they result in the
formation of a thermosetting polymer, namely Bakelite.

6. Urea-formaldehyde resin. Urea on condensation with formaldehyde results


in the formation of urea-formaldehyde resin. Initially, the reaction results in the
formation of monomethylol, which further condenses with urea and
formaldehyde to give the resin. The polymerization may occur in a linear, three-
dimensional, or cyclic structure.
7. Melamine resin. The condensation of urea results in the formation of
melamine, which on polycondensation with formaldehyde results in the
formation of melamine resin.

Some condensation polymers along with their monomer units and common uses
are listed in
Table 31.3.
31.5 RUBBERS

31.5.1 Natural Rubber


Rubber is an elastomer, which is obtained from (rubber plant scientific name:
Hevea brasiliensis) in the form of a white fluid called latex. Natural rubber is an
unsaturated polymer of 2-methylbuta-1,3-diene (isoprene). The double bonds
present in the natural rubber have cis- configuration (Z-configuration). The
structure of natural rubber is as follows:

Natural rubber is a soft and sticky solid and is often associated with a peculiar
smell. It has low elasticity and low tensile strength. It softens in hot weather and
gets hardened in cold weather. It is soluble in organic solvents such as carbon
disulfide, petrol, ether, and so on. The properties of natural rubber are improved
by introducing cross linkages in polymer chains by heating rubber with sulfur.
This process is known as vulcanization. A polymer of isoprene (from natural
sources) that has trans-configuration at the double bonds is known as Gutta-
Percha.

31.5.2 Vulcanization of Rubber


Raw or natural rubber is associated with some undesirable properties such as low
tensile strength, low elasticity, low resistance to friction, softness, and stickiness.
It exhibits elasticity over a small range of temperature. Rubber is soluble in
certain solvents and undergoes auto-oxidation. It
becomes hard in cold weather and sticky in hot weather. In 1893, Goodyear
developed a method for modifying the properties of raw rubber which involves
the introduction of cross linkages in polymer chain by heating the rubber with
sulfur. This process is known as vulcanization. Vulcanization improves the
properties of rubber and makes it fit for commercial uses. Heating
a mixture of raw rubber with sulfur, accelerator, fillers and antioxidants results
in the formation
of vulcanized rubber. The functions of these reactants are discussed in the
remainder of the
section.
Accelerator. It is an organic/inorganic compound, which is used to introduce
sulfur into rubber molecules. It also accelerates the reaction between sulfur and
rubber so that the reaction can occur at low temperature. The accelerator is
added in an amount ranging from 0.5–1%. Some examples of accelerators are
diphenyl guanidine, mercaptobenzothiazole, and zinc dimethyldithiocarbamate.
Filler. Filler further improves the activity of accelerator and are used in small
amounts. For example, ZnO (0.5%) or carbon black (0.5%).
Antioxidants. The antioxidants are very easily oxidized by air and thus prevent
the auto-oxidation of rubber. Aromatic amines and phenols are used as
antioxidants and they improve the quality of rubber.
The vulcanization of rubber can be carried out at both high and low temperature
conditions—
(i) Hot sulfur vulcanization. In this process, rubber is mixed with 5–8% sulfur.
A higher percentage of sulfur, say 50%, may also be used depending upon the
quality of rubber required. A mixture of rubber and sulfur is heated in the
presence of an accelerator, filler, and an antioxidant.
(ii) Cold sulfur vulcanization. In this process, raw rubber is treated with sulfur
monochloride (S2Cl2) at a very low temperature (no heating) in the presence of
an accelerator, filler, and an antioxidant. The rubber used in surgeon gloves is
prepared by this method.
Properties of vulcanized rubber
Vulcanized rubber possesses lesser unsaturation as sulfur cross linkages are
introduced in it. In vulcanized rubber, mono sulfur linkages or disulphide
linkages may be present along with free sulfur.
Vulcanized rubbers have following improved properties over natural rubber:
(i) It is hard and non-sticky.
(ii) It has higher elasticity, which occurs over a higher range of temperature.
(iii) It has a high tensile strength.
(iv) It is insoluble in solvents.
(v) It is not oxidized readily and behaves the same in all weathers.

31.5.3 Synthetic Rubber


Synthetic rubber is produced by polymerization of conjugated dienes in the
presence of a catalyst or by co-polymerization of conjugated dienes with other
olefinic compounds. Synthetic rubber has superior properties compared to
natural rubber. The double bonds in synthetic rubber have a
trans-configuration.
A brief discussion of a few important synthetic rubbers is as follows:
1. Neoprene. It is a polymer of 2-chlorobuta-1,3-diene (chloroprene) and is
obtained by free radical addition mechanism. It is a linear themoplastic polymer.

Neoprene rubber is used in the manufacturing of flexible rubber tubes, industrial


belts, in printing rolls, etc. It cannot be vulcanized with sulfur.
2. Buna rubber. The polymerization of butadiene in presence of sodium results
in the formation of buna rubber. The two important types of buna rubbers are
Buna-N and Buna-S.
(i) Buna-N. It is prepared by co-polymerization of butadiene and acrylonitrile
in the presence of sodium metal. It is used in the manufacture of storage tanks.

(ii) Buna-S. It is prepared by co-polymerization of butadiene and styrene in the


presence of sodium metal. Buna-S is also known as SBR (Styrene-Butadiene
Rubber) and is used in the manufacture of tyres and other mechanical goods.
EXERCISES
1. What are polymers? How can they be classified on the basis of their source
and structure?
2. Explain the terms (a) linear polymers and (b) cross-linked polymers with the
help of schematic diagrams.
3. What do you understand by the term degree of polymerization? Calculate the
degree of polymerization of polymer of chloroprene with average molecular
weight 81000 (molecular mass of chloroprene is 88.5).
4. Define the following:
(a) Homopolymers
(b) Copolymers
(c) Chain growth polymerization
(d) Step growth polymerization
(e) Ziegler–Natta polymerization
5. Explain the stereochemical aspects of polymers. How does the
stereochemistry of polymers affect their physical properties?
6. Write the product and mechanism of the following polymerization reaction:

7. Write the mechanism for addition polymerization reaction of acrylonitrile in


presence of sodamide.
8. What is Ziegler–Natta catalyst? Explain the mechanism for the
polymerization of propene in presence of Ziegler–Natta catalyst.
9. What are the characteristics and advantages of Ziegler–Natta polymerization?
10. What are platicizers? Write the structure for any two of them.
11. Write the structure of following polymers and also indicate the monomer
units involved in the formation of these polymers.
(a) PVC
(b) Orlon
(c) Teflon
(d) Plexiglass
(e) Buna-S
(f) Thermocole
(g) Dacron
(h) Bakelite
(i) Lexan
(j) Melamine
(k) Neoprene
(l) Natural rubber
(m) Gutta percha
(n) Nylon-6
(o) Nylon-6,6
12. What is vulcanization of rubber and how it affects the properties of rubber?
13. Give the free radical addition mechanism involved in the polymerization of
chloroprene.
14. Write a short note on polymers as water softeners.
Chapter32
Spectroscopy and
Structure

32.1 INTRODUCTION
Throughout this textbook, the chemistry of various functional groups of organic
compounds has been discussed. In every reaction, one functional group is
transformed into another. The reactions are the indication of the chemical nature
of functional groups and sometimes the characteristic chemical tests are
employed for the confirmation of the structural features of organic compounds.
But it is a difficult and cumbersome task to identify the structure of organic
compounds through these chemical reactions or tests alone. This problem of
identification of the structure of organic compounds aggravates in case of
isomeric compounds or when compounds are virtually unknown.
The identification of compounds through chemical reaction may include the
following steps:
(1) Purification
(2) Detection of elements
(3) Estimation of elements
(4) Calculation of empirical formula
(5) Molecular formula
(6) Assigning of structure, which involves (a) functional group determination;
(b) degradation (mainly for unknown compounds) to simple compounds
and their characterization, through which structure is established; and (c)
confirmation of structure by synthesis.
These are the conventional methods for the determination of the structure of
organic compounds. A substantial amount of pure compound is required for
carrying out the analysis, and practically compounds are not recovered as all
methods are destructive in nature.
The purpose of this chapter is to show how information about the presence or
absence of a particular functional group or arrangement of groups and atoms in a
skeleton of an organic molecule, can be obtained using spectroscopy.
The spectral methods have the four major advantages over most of chemical
tests:
(1) Spectral analyses are easier and take very little time.
(2) Spectral analyses require very small quantity of compound (~1 mg or less).
(3) Spectral analyses generally provide more detailed and reliable information
about molecular structure, which makes the task of determining the
structure easy and less time consuming.
(4) Spectral analyses are nondestructive (that is, have no effect on the sample)
and the sample can be recovered, if required.
All organic compounds interact with electromagnetic radiations, that is, they
absorb energy. Spectroscopy is a technique based on the absorption of radiations
by molecules. Different molecules absorb different amounts of radiations and
this difference in absorption is used for the analysis and identification of
molecules. In the present chapter, we briefly introduce the three important
spectroscopic techniques, namely (i) Ultraviolet–visible spectroscopy, (ii)
infrared spectroscopy, and (iii) nuclear magnetic resonance.
In discussing any type of spectroscopy, it is necessary to have an idea of some
facts about the interaction of all types of electromagnetic radiation with
molecules.
Electromagnetic radiations
The various types of electromagnetic radiation or light (ultraviolet light, visible
light, infrared, microwaves, X-rays, radio waves, and so on) can be described in
one simple way, by assuming that electromagnetic radiations have a dual
character (of a wave as well as that of a particle). The light consists of small
packets of energy called photons and the energy of a mole of photon, is related
to the frequency of light by equation:
E = hν
Since frequency and wavelength are inversely proportional and are related by ν =
c/λ,
Thus,
The other important unit frequently used in spectroscopy is wave number ,
that is, the number of wave cycles per centimetre, which is the reciprocal of
wavelength and is expressed in cm–1.

The infrared absorption frequencies are generally reported in wave numbers. The
energy of light wave is directly related to its frequency. This means that higher
the frequency of radiation, higher is its energy. A summarized data of
wavelengths, frequencies, and energies of electromagnetic spectra is as follows:

Recording of spectra. The absorption of radiations by molecules results in a


transition from low energy ground state to high energy excited state. The
instrument that measures the change in intensity of light (radiation) is known as
spectrometer. It has three important parts, namely the
(i) Source: A source provides electromagnetic radiation of particular frequency
that pass through a given organic sample (mostly in dissolved form); (ii)
detector: It measures the change in the intensity of radiations as it is absorbed by
the sample. (iii) recorder: The change in the intensity of radiation as detected by
a detector is recorded on a paper as signals, which are also referred to as peaks
or bands depending upon the type of spectroscopy. The change in the intensity of
radiation may be recorded as radiation absorbed (absorption spectroscopy) or
radiation transmitted (emission spectroscopy).
32.2 ULTRAVIOLET–VISIBLE SPECTROSCOPY
The ultraviolet–visible (UV–visible) spectroscopy involves the absorption of
electromagnetic radiations of high energy (200–800 nm). The ultraviolet region
of electromagnetic spectrum extends from 200 nm–400 nm and the visible
region extends from 400–800 nm (red light). This type of spectroscopy is used
for determining the presence of unsaturation and the extent of conjugation in a
molecule.
Absorption of ultraviolet–visible radiation causes the promotion of electrons
from low energy level to higher energy level (electronic excitation). The
electrons in a molecule may be σ-bonded, π-bonded, or present as lone pairs
(non-bonding). The higher energy levels are antibonding molecular orbitals, that
is, σ* and π*.
• Electrons of σ-bonds are held strongly and energy in ultraviolet–visible
region is not sufficient to excite them to high energy level (σ*). Hence,
alkanes, cycloalkanes, and alkenes show no absorption in the range 200–800
nm due to absence of σ – σ* transitions.
• The π-electrons and non-bonding electrons (n) are relatively loosely held and
require lesser energy for promotion to high energy level (π*). Thus, in π-
bonded systems two types of electronic excitations are observed, namely π–
π* transition and n–π* transition.
The π–π* transitions are allowed transitions while n–π* transitions are forbidden
transitions. The n and π* are perpendicular to each other and transition disturbs
the geometry of the molecule. The n–π* transition requires less energy than π–π*
transition (refer Fig. 32.1). In UV–visible spectroscopy the most important
transitions are π–π* transitions.
Fig. 32.1 Electronic transition.

For recording the UV–visible spectrum, the given sample is dissolved in a


suitable solvent, which itself does not absorb light in that range. The commonly
employed solvents are hexane, cyclohexane, chloroform, carbon tetrachloride,
and so on. The positions of absorption peaks are slightly shifted with the change
of solvent. The spectrum is recorded as a plot of absorbance (A) versus
wavelength (λ) of radiations. The absorption peaks are plotted as maxima in the
spectrum. The peak is marked by λmax, which is the wavelength of maximum
absorption. The absorbance A at a given wavelength is represented by Beer
Lambert’s law which is expressed as
A = εcl,
where
ε is molar absorptivity,
c is molar concentration (M) of sample, and
l is length of the path of sample solution through which radiation passes.
The height of an absorption peak is given by ε value and equals A/c for a length
λ of 1 cm. The value of ε is characteristic for a particular molecular structure.
Conjugation results in the shift of absorption band towards higher wavelength
(high λmax and low energy) and this shift is known as bathochromic or red
shift. Similarly the term hypsochromic shift or blue shift is used when
absorption band shifts towards shorter wavelength (low λmax and high energy).
The ultraviolet–visible spectroscopy is used as a tool for the detection of
conjugation in an organic molecule. Greater the conjugation in a molecule,
higher is the λmax value and hence lower energy is required for transition. The
extent of conjugation in a system is easily detected from λmax values as
illustrated in the following examples:
The unit for ε is M–1cm–1 but generally not mentioned while writing UV
spectral data.
WOODWARD–FIESER RULES
R.B. Woodward (in 1945) formulated certain rules for correlating λmax with
structure of molecules which was further modified by L.F. Fieser (in 1959). The
modified rules are known as Woodward–Fieser rules and are used to calculate
the theoretical values of λmax for number of molecules by considering the
substituent and groups attached to it. It theoretically deals with barthochromic or
hypsochromic effect of various groups or substituents present in molecules. The
calculated values for λmax are found to be in good agreement with observed
values for λmax of given molecule.
For the theoretical calculation of λmax, the systems can be categorized as:
1. Conjugated dienes
2. α, β-unsaturated carbonyl compounds
3. Aromatic compounds
Each type of systems are having a certain fixed value of λmax at which
absorption takes place. This constitutes the Base value or Parent value. The
effect of presence of alkyl substituents (or ring residues), extended conjugation,
polar groups such as –Cl, –Br, solvent and various other groups is added as
increment to the base value of particular compound to calculate its λmax value.
1. Conjugated dienes
Acyclic conjugated dienes and their λmax values are described above. The
double bonds in cyclic systems are correlated as follows:

The Woodward–Fieser rules, for calculating λmax for conjugated dienes, are
summarized in
Table 32.1.
Few examples of conjugated dienes, implementing Woodward–Fieser rules, for
the calculation of λmax, are given below:
2. α, β-unsaturated carbonyl compounds
The Woodward-Fieser rules, for calculating λmax for α, β-unsaturated carbonyl
compounds, are summarized in Table 32.2.

In general, the UV spectra is recorded in ethanol solution. Use of other solvents


require correction* in observed λmax values as: Water (–8 nm), chloroform (+1
nm), ether (+7 nm) and hexane (+11 nm).
Few examples of α, β-unsaturated carbonyl compounds, implementing
Woodward–Fieser rules, for the calculation of λmax, are given below:
3. Aromatic compounds
In the aromatic hydrocarbons such as anthracene, naphthalene, and benzene,
anthracene has high λmax value. This is because it has a highly extended
conjugated system compared to naphthalene and benzene.

The Woodward–Fieser rules, for calculating λmax for aromatic compounds, are
summarized in Table 32.3.
Few examples of Aromatic compounds, implementing Woodward-Fieser rules,
for the calculation of λmax, are given below:
Further it has been noted that steric hindrance also has an impact on λmax value.
For example, cis-stilbene has lower λmax value than trans-stilbene. The π – π*
transition requires higher energy in cis-form compared to that in trans-form. The
steric hindrance of aromatic hydrogens in cis-stilbene prevents the coplanarity of
the molecule and disturbs the delocalization of π-electrons.
32.3 INFRARED SPECTROSCOPY
The detection of the type of bonds and functional groups present in a molecule is
carried out easily with the help of infrared or IR spectroscopy. It deals with the
region of electromagnetic spectrum that lies in the frequency range 500–6670
cm–1. For organic chemists, the important region of IR spectroscopy is 700–
5000 cm–1. The infrared frequency is expressed in units of wave number. A
molecule of a compound is made up of large number of atoms, which
continuously undergo vibration and rotation. The exposure of the compound to
IR radiations results in an increase in the already existing vibrations and
rotations. A very little energy (2–12 kcal/mol) is required to bring about such
vibrational excitations. A particular vibration absorbs
IR radiations of particular frequency and this absorption occurs only when there
is a change in the dipole moment. The vibrations can be of two types, namely (i)
bond stretching and
(ii) bond bending. The following figure depicts such vibrations for tetrahedral
carbon:

Fig. 32.2 Different vibrations around tetrahedral carbon: (a) stretching and (b) bending.

Stretching of a bond requires higher energy and thus, absorption occurs at high
frequency region (1500–5000 cm–1). The bond stretching vibrations are
generally recorded and are useful for interpretations. For recording the IR
spectra of a compound, the compound is taken in Nujol (liquid paraffin) to make
film or mixed with potassium bromide and used as KBr pallet. Each functional
group shows a characteristic band in this region of band stretching vibrations and
appears in the same range irrespective of the structure of molecule.
For example, the C=O bond shows a stretching band in the range of 1680–1750
cm–1 irrespective of whether this bond is present in an aldehyde, ketone, acid, or
ester molecule. The infrared frequencies of some bonds are listed in Table 32.4.
Table 32.4 Characteristic infrared absorption frequencies for some important covalent bonds of various
structural units of organic molecules and some typical carbonyl stretching frequencies of
different functional groups
The bending of a bond requires less energy and thus, absorption occurs in low
frequency region (700–1500 cm–1), which is known as fingerprint region. The
bands obtained due to bending vibrations are difficult to interpret. The
fingerprint region is used for establishing the identity of two compounds. Each
compound shows unique bands in this region and if two compounds are
identical, they have same bands in fingerprint region.
The stronger bonds generally show absorption in the high frequency region due
to higher energy required for stretching. For example, the absorption band for
N–H bonds in amines appears in the frequency range 3300–3500 cm–1 whereas
the relatively stronger O–H bond shows absorption in 3600–3650 cm–1 (refer
Fig. 32.3). Although the functional groups have a characteristic absorption value,
a shift in the values may be observed due to presence of conjugation or hydrogen
bonding in a molecule. The hydrogen bonding in a molecule causes lengthening
of bonds and thus decreases the stretching frequency. Thus, the hydrogen bonded
–OH group appears at low frequency compared to free –OH groups. The
intermolecular hydrogen bonding is affected by dilution. In dilute solutions, the
hydrogen bonds between the molecules break and thus absorption for O–H
stretching appears at high frequency as in the case of free –OH groups. However,
the intramolecular hydrogen bonding does not break even in dilute solutions.
Fig. 32.3 A comparison of IR absorption frequencies in the region of 3000–3600 cm–1; (a) Hexan-1-ol
(O–H stretching), (b) Butanoic acid (hydrogen bonded O–H stretching) (c) Propan-2-amine (N–H
stretching or 1° amines), besides other characterisitics stretching frequencies are also encircled.
The following illustrative examples explain the use of IR spectroscopy in
identification of functional group(s) present in a compound:
Illustrative Example 1. How will you distinguish between the following pairs
of compounds using IR spectra (for values, refer Table 32.4)

Solution:
(a) In case of ketones, a strong absorption band for C=O appears around
1705–1725 cm–1 but in case of ethers no absorption is observed in this
region. In ethers, C–O– stretching band appears at ~1100 cm–1.
(b) In case of benzoic acid, the –OH of carboxylic group is hydrogen bonded
(exists as dimer; intermolecular hydrogen bonding) and thus the absorption
band for O–H appears at ~3000 cm–1 and the carboxylic acid also shows
the absorption band (for C=O ) around 1700–1725 cm–1. On the other
hand, in case of p-hydroxyacetophenone, the hydroxy group is not hydrogen
bonded and therefore its absorption band appears at ~3500–3700 cm–1.
Another absorption band is observed (for C=O) at 1705–1725 cm–1. Thus,
the absorption values for hydroxy group help in differentiating between the
two compounds.

(c) The o-hydroxybenzoic acid and m-hydroxybenzoic acid can be


differentiated on the basis of O–H stretching frequencies observed in their
IR spectra. In case of o-hydroxybenzoic acid, the hydroxy group is
hydrogen bonded (intramolecular hydrogen bonding) with carboxylic group
and its absorpition band (–O–H stretching) appears at a low frequency
(~3000 cm–1) . In case of m-hydroxybenzoic acid, the hydroxy group
shows the absorption band at ~3500–3700 cm–1 (non-hydrogen bonded –
OH). The intermolecular and intramolecular hydrogen bondings can be
identified by dilution effect (refer earlier discussion in the text).

Illustrative Example 2. How will you distinguish between the following pairs
of compounds using IR spectroscopy (for values, refer Table 32.4)

Solution
(a) An absorption band around 3000 cm–1 appears in case of acetic acid (in
general, carboxylic acids) due to hydrogen bonded –OH stretching. No such
band is possible in case of ethyl acetate (in general, esters).
(b) The 3° and 2° amines can easily be distinguished by IR spectra. The band
around
3300–3500 cm–1 appears for –N–H stretching which is possible only in
case of 2°
amines.
32.4 NUCLEAR MAGNETIC RESONANCE (NMR)
SPECTROSCOPY
Nuclear magnetic resonance (NMR) involves absorption of radiations in the
radio frequency region. This spectroscopy is generally used for the
determination of the type and the number of protons present in a molecule. The
nuclei of certain elements behave like spinning charges, for example, 1H, 13C,
15N, 19F, and 31P. A spinning charge creates a magnetic field and therefore
behaves as a tiny magnet. (Fig. 32.4). The spinning nuclei generate a magnetic
field on exposure to an external magnetic field. If an external magnetic field is
applied, spinning nuclei can get oriented in two ways:
(i) The magnetic field due to nuclei is in the same direction as the external
field. In this case, the nuclei are said to be in a low energy spin state.
(ii) The magnetic field due to nuclei is in the opposite direction as the external
magnetic field. In this case, the nuclei are said to be in a high–energy spin
state.
By applying radiation in radio frequency range, the nuclei can be excited from
lower energy spin state to higher energy spin state. The energy required for this
change in spin state (known as flipping) depends on the strength of external
magnetic field. Higher the external magnetic field, higher is the energy (or
frequency) required for excitation. It is a general practice to supply a constant
radio frequency and vary the strength of external magnetic field.
Fig. 32.4 (a) Spining charge develops a magnetic field along the axis of spin and (b) behaves as a tiny
magnet. (c) In presence of applied magnetic field, hydrogen nucleus absorbs electromagnetic
radiations and shows transition from low energy state to high energy state.

Different protons in a molecule are present in different environments (chemically


different). The absorption of energy by proton nuclei occurs when the applied
field is same as the difference in the energies of the two spin states of nuclei.
This is a point when proton nuclei are said to be in resonance with the applied
frequency thus, we term this as nuclear magnetic resonance or simply as
NMR or more specifically as proton magnetic resonance (PMR or 1HNMR)
spectroscopy.
A bare proton absorbs radiations at a particular magnetic field strength. However
in a molecule, a proton is always in the influence of some other atom. The
electrons in a bond shield the proton from external magnetic field. As a result,
proton in a molecule absorbs energy at a higher magnetic field strength
compared to a bare proton. This difference in magnetic field strength is known
as chemical shift.
The chemical shift for protons in a molecule is measured in terms of δ units with
respect to a standard compound TMS (tetramethylsilane; (CH3)4Si). The twelve
protons in TMS are equivalent and are highly shielded. These protons appear
as a single peak and this peak is assigned a chemical shift value of zero,
which means that for TMS, δ = 0.
The peaks for all other protons in any organic molecule appear downfield (at
lower magnetic field strength) with respect to TMS and are thus said to have
positive chemical shift (δ) values. The δ values are measured as parts per million
(ppm) in frequency units downfield of the TMS peak.

Fig. 32.5 Chemical shift with respect to TMS as the reference peak.

For example, if a proton appears 360 Hz downfield from the TMS peak, and the
spectrum is taken at a radio frequency of 60 MHz, the chemical shift δ is given
as:
δ = (position of peak) – (position of TMS/radio frequency)
= 360 – (0.0/60 × 106) = 6.0 ppm
Recording the NMR spectrum. The sample is dissolved in deutrated solvent
like CDCl3 (chloroform-d3), CD3COCD3 (acetone-d6), and (CD3)2SO
(DMSO-d6). As a characteristic, the deutrated solvents do not show any peak in
the spectra and are thus non-interfering in spectral study of the given sample. For
the reference peak (δ = 0), TMS is added to the deutrated solvent. The sample is
then introduced into spectrometer and the NMR spectrum is recorded on a paper
as a plot between intensity of absorption and the chemical shift (δ). A peak or
signal is obtained when nuclei absorb energy.
Interpretation of NMR spectrum
The different peaks obtained in a NMR spectrum need to be considered in terms
of the following observations:
(i) Number of signals. The total number of signals obtained in a spectrum is a
direct indication of the number of different types of protons present in a
molecule. Same types of protons (present in the same environment) are known
as equivalent protons and give only one signal or peak. Non-equivalent protons
(present in different environments) give different signals.
(ii) Position of signals. Protons that are present in different chemical
environments have different δ values. There is a standard chart that predicts the δ
values for representative protons (refer
Table 32.5). Highly shielded protons appear at low δ values (or upfield). The
presence of electron withdrawing groups or atoms on a carbon makes the protons
attached to that carbon less shielded and these protons appear at high δ value (or
downfield). For example, CH3CH2CH2CH3 has two types of protons and shows
two signals in the spectrum—one signal for CH3 protons and the other for CH2
protons. Similarly, CH3CH2CH2Cl has different types of protons and shows
three signals—one signal for CH3 protons, one for CH2, and one for CH2
(attached to Cl) protons.
Table 32.5 Chemical shifts (approximate values) of representative types of protons relative to TMS
(iii) Intensity of signals. The intensity of a signal gives an idea of the relative
number of equivalent protons that are responsible for the signal. The intensity of
a signal is measured electronically and in a spectrum, it appears as a stepped
curve over each signal.
(iv) Splitting of peak (spin–spin coupling). The signals in an NMR spectrum
may appear as
singlets (a single peak), abbreviated as ‘s’
doublets (signal split into two peaks), abbreviated as ‘d’
triplets (signal split into three peaks), abbreviated as ‘t’
quartets (signal split into four peaks), abbreviated as ‘q’, and so on.
This multiplicity (splitting) of a signal arises due to the presence of non-
equivalent neighbouring, protons. In general, the signal for a proton on a
carbon splits into (n + 1) number of peaks where n is the number of protons
present on the adjacent carbon(s). The equivalent protons have same δ
value and do not cause splitting of each other’s signal.
The multiplicity of a signal basically arises due to spin–spin coupling. For
example, signals for protons Ha and Hb present on adjacent carbons appear each
as a doublet in the NMR spectrum.

As discussed earlier, a proton behaves as a magnet and has two spin orientations.
In the presence of an external field, a proton may exist either in a low or high
energy spin state. Thus, Ha proton appears both in low energy spin state and in
high energy spin state. The same is true for Hb proton.
(a) Proton (Ha) can have its spin opposite to Hb and thus Ha decreases the net
magnetic field experienced by Hb. The signal for Hb appears at high
magnetic field strength (upfield, low δ value).
(b) Proton Ha can have its spin aligned with Hb and in this case, Ha increases
the net magnetic field experienced by Hb. The signal for Hb appears at low
magnetic field strength (downfield, high δ value).
(c) As a result, the Hb observes resonance twice and its signal appears as a
doublet (split into two). In a similar manner, Ha also splits into two signals.
Figure 32.6 illustrates the NMR spectra of some compounds.
This phenomenon is known as spin–spin coupling. The distance (in Hz) between
the individual peaks in a doublet is known as coupling constant (J). The value of
coupling constant remains same even if there is a change in strength of external
magnetic field. Spin–spin coupling can occur between hydrogens present on
adjacent carbons (vicinal coupling) or between hydrogens present on the same
carbon (geminal coupling).
The following illustrative examples explain the application of NMR
spectroscopy in the identification of organic compounds.
Fig. 32.6 NMR (1HNMR or PMR) spectra of (a) 1,1-dichloroethane (b) phenylmethyl ethanoate [or benzyl
acetate] and (c) methyl propanoate. (Integral and splitting patterns are mentioned for 1,1-
dichloroethane.)
Illustrative Example 3. Assign the number of signals (peaks) that will be
observed in the NMR spectra (or PMR) of the following compounds:

Solution
(a) One signal—Because all the hydrogens on methyl groups are chemically
equivalent; (CH3)4C.
(b) Two signals—One signal for the hydrogens on three methyl groups
(chemically equivalent (CH3)3C–) and one for methylene group (–CH2–
Cl) hydrogens.
(c) Three signals—As the hydrogens on each carbon are chemically non-
equivalent
(CH3–CH2–; –CH2–O-; –O–CH3).
(d) One signal—All the hydrogens are chemically equivalent; (CH3)2C=O.
Illustrative Example 4. Give the splitting pattern of the signals for the
hydrogen(s) in the PMR spectra of following compounds:

Solution
(i) a = singlet; b = quartet (splitting due to three adjacent hydrogens on methyl
group ‘c’); c = triplet (splitting due to two neighbouring protons of ‘b’).
(ii) a = triplet (splitting due to two protons of ‘b’); b = quartet (splitting due to
three protons of ‘a’); c = singlet (the proton on oxygen i.e. hydroxy group is
a labile proton and does not affect the splitting pattern of ‘b’).
(iii) a = triplet ; b = quartet; c = singlet
(iv) a = singlet ; b = doublet; c = sextet (actually said to be multiplet due to
complexity of signal); e and f = both are in the same chemical environment
and give single peak as doublet.
Illustrative Example 5. Give the splitting pattern of the signal(s) for the
hydrogen(s) in the PMR spectra of the following:

Solution
(i) The signal for benzene appears as a singlet.
(ii) Two signals are observed—one signal for three hydrogens of the methyl
group (as singlet) and one for five hydrogens of the aromatic ring (a broad
peak appears).
(iii) In case of 1,3,5-trimethylbenzene, only two signals (as singlets) appear—
one signal for all the nine hydrogens of three methyl groups and another for
three hydrogens of the aromatic ring.
(iv) In case of 4-methylphenol, the three signals appear as follows:
— one singlet for methyl group.
— one doublet each for two sets of ortho protons.
— the phenolic –OH appears as a broad singlet.
Illustrative Example 6. How will you distinguish between the following pairs
of compounds using PMR spectroscopy. Give the splitting pattern, intensity of
the signal and approximate chemical shift values (δ) refer Table 32.5.

Solution. For these compounds, the number of signals, approximate chemical


shift, the splitting pattern, intensity of signal, and the type of protons in a
molecule are as follows:
(i) In case of propanal, the two signals, appear at δ = 1.0; (triplet; for 3H;
CH3–) and
δ = 2.5 (quartet; for 2H; CH2). For acetone, one singlet appears at δ = 2.7.
(ii) In case of ethanol the two signals appear at δ = 1.0; (triplet; for 3H; CH3)
and δ = 4.0 (quartet; for 2H–CH2). For methoxymethane, one signal as a
singlet appears at δ = 4.0.
(iii) In case of propanoic acid, the two signals appear at δ = 1.0; (triplet; for
3H; CH3–) and δ = 2.5 (quartet; for 2H; CH2). For methylacetate, two
signals appear as singlets at
δ = 2.7 (for 3H; CH3CO group) and δ =4.0 (for 3H; –OCH3 group)
respectively.

It is a convention to write NMR spectra as for example: (triplet for 3H, CH3) and not as: (triplet for three
hydrogens for CH3 group). Further to add, the more correct way of writing is by using abbreviation. For
splitting patterns (e.g. singlet as ‘s’ doublet as ‘d’, triplet as ‘t’ and so on). But it is more often used in
specialized books.
32.5 MASS SPECTROMETRY—AN
INTRODUCTION
Mass spectrometry is an important physical technique, which is used for
measuring the masses of atoms and molecules with great accuracy. It differs
from the other spectroscopic techniques that have been discussed so far, in the
sense that, it does not involve the absorption of radiation. It is based on the
principle that the charged particles (ion) moving in magnetic field follow curved
paths and in this process heavy ions are deflected the least while light ions are
deflected the most.
In a mass spectrometer, the vapours of organic compounds are bombarded with
beam of high-energy electrons (70 eV) that makes the neutral molecule loose an
electron and converts it to a radical cation known as molecular ion. Some of the
molecular ions in the process break into all types of possible fragments. The
molecular ions of varying mass (m) to charge (e) ratio are generated in the
process.

The molecular ions are then passed through an electric field, which accelerates
their velocity. These high velocity molecular ions are subjected to magnetic field
where they follow a circular or curved path depending on their mass to charge
ratio. A highly sensitive detector then detects the molecular ions. When a
molecular ion reaches the detector, it is automatically recorded on paper as a
line. The height of the line is proportional to the number of ions (abundance)
reaching the detector. The resulting mass spectrum is thus, obtained as a discrete
series of lines with increasing m/e values. The molecular ion peak with greatest
m/e value is called the parent ion and is used to find the relative molecular mass
of the original molecule. As an example the mass spectrum of butane is shown in
Fig. 32.7.
Fig. 32.7 Mass spectrum of butane (schematic representation).

Elucidating the structure of unknown compound using spectral techniques


So far we have discussed different spectroscopic techniques, however, it should
be noted that data from one spectral technique alone is not sufficient for structure
determination. Different spectral data are used in combination for the structure
determination of an unknown organic compound.
The type of information obtained from different spectral techniques can be
summarized as follows:
(i) Mass spectrometry is used as an important tool by organic chemists for
determining the molecular formula of organic compounds.
(ii) The IR spectrum helps in identifying the characteristic structural unit of
functional groups like C=O, –OH, NH2, and so on.
(iii) An UV spectrum indicates the presence of unsaturation and extent of
conjugation in an organic compound.
(iv) The NMR spectrum gives direct information regarding the different types
and number of protons present in a molecule, which correlates with the
basic skeleton of unknown compound.
The following illustrative examples explain how the structure of an unknown
compound is determined making use of information obtained from various
spectral data.
Illustrative Example 7. An organic compound (molecular formula C9H10O2)
shows in its
UV spectrum λmax at 268 nm and 262 nm. The compound shows absorption in
IR spectra at
1745 cm–1 and 1225 cm–1. In the 1HNMR spectrum, three signals were
obtained at δ7.22, δ5.0, and δ2.0 with intensity ratio 5:2:3 respectively. Identify
the structure of the compound.
Solution. In the molecular formula, presence of two oxygen atoms indicates a
carboxylic acid or an ester functional group.
UV is indicating the presence of C=O structural unit and aromatic ring system
(λmax at
268 nm and 262 nm.). Its IR spectrum, confirms the presence of C=O structural
unit (1745 cm–1) and absorption at 1225 cm–1 indicates the structural unit –O–
R (refer Table 32.4). This collective information indicates that the unknown
compound is an ester.
In 1HNMR spectrum (refer Table 32.5), the signal (peak) at δ7.22 (intensity 5)
indicates
an aromatic system with 5 equivalent protons, which is a phenyl ring C6H5. The
signal at δ5.0 (intensity 2) represents a deshielded –CH2– structural unit. Also
the absence of splitting pattern indicates that –CH2– unit does not have
negihbouring protons. The signal at δ2.0 (intensity 3) indicates the presence of –
CH3– structural unit without any neighbouring protons (since splitting of the
signal is not observed).

This data makes it clear that the compound is an ester – –O– with a phenyl ring
(C6H5),
a –CH2– group, and a –CH3 group and can have three possible structures (I, II,
and III as shown here). Out of the three structures, the structure III is correct, as
it does not show any splitting pattern
and chemical shift values are in accordance with the given data. (see its NMR
spectrum in
Fig. 32.6b).
Illustrative Example 8. An oxygenated organic compound shows the molecular
ion peak (parent ion) at 72 m/e. Its IR spectrum shows strong absorption at 1070
cm–1 but no absorption is observed at 1600 cm–1. In its NMR spectra, it shows
peaks at δ 1.8 and δ 3.7 each corresponding to a triplet (intensity ratio 1:1).
Identify the structure of the compound.
Solution. The given compound is not an alcohol or carbonyl compound as
evident from its IR absorption patterns. But the absorption at 1070 cm–1
indicates that it could be an ether (see
Table 32.4). The splitting pattern in its NMR spectra indicates that it contains
structural unit
–CH2–CH2–, where both the –CH2– groups are not chemically equivalent (δ1.8
and δ3.7 each). That means one of the –CH2 groups is attached to oxygen.
Further the mass spectrum of the compound reveals that its molecular mass is 72
and since it is an ether, the molecular formula must be C4H8O. This molecular
formula suggests that the compound is a cyclic ether with four carbons, namely
tetrahydrofuran.
EXERCISES
1. Define the term molecular spectroscopy.
2. Give the units in which spectral information is reported for the following:
(a) UV–visible spectroscopy (b) IR spectroscopy (c) NMR spectroscopy
3. What type of molecular transitions occur by the absorption of the following:
(a) Ultraviolet–visible radiations
(b) Infrared radiations
(c) Radiofrequency radiations
4. Arrange the following:
(a) Ultraviolet–visible radiations
(b) Infrared radiations
(c) Radiofrequency radiations
in order of their (i) increasing wavelength, (ii) increasing frequency, and (iii)
increasing energy.
5. How can one distinguish the following pairs of compounds using IR
spectroscopy?

6. Which of the following pairs of molecules is expected to absorb radiations in


the ultraviolet–visible region at higher wavelength?
7. Indicate the set(s) of chemically equivalent protons in the following
compounds:

8. Which of the compounds (given in Exercise-7) will show single peak in their
1HNMR spectra?
9. Predict the splitting pattern using n + 1 rule in each set of chemically
equivalent protons in the following molecules:
10. How is 1HNMR spectroscopy helpful in distinguishing between the
following isomers?

11. How many signals will be obtained from the 1HNMR spectra of the
following:

12. What is the principle of mass spectrometry? How is it different from other
spectral techniques?
* Solvent correction (source): “Applications of absorption spectroscopy of organic compounds”–John R.
Dyer,
p. 14 (1991), Prentice-Hall of India Private Limited.
Octa-1,3,5,7-tetraene
Glossary

Name Reactions, Reagents and Some Important


Terms
Adam’s catalyst (p. 172): The Adam’s catalyst is platinum (IV) oxide (PtO2)
and is used for the hydrogenation of alkenes and alkynes to form alkanes.
Aldol condensation (p. 599): Two molecules of -hydrogen containing aldehydes
(or ketones) react in presence of alkali to form β-hydroxyaldehyde (aldol).
Ketones result in the formation of
-hydroxyketone. These aldols on heating undergo dehydration to form ,-
unsaturated carbonyl compounds.
Amadori rearrangement (p. 971): The rearrangement involved in the formation
of osazone in glucose.
Arndt-Eistert reaction (p. 796): It is a conversion of carboxylic acid into next
higher homologue (one carbon is added). The reaction involves the treatment
of acid chloride with diazomethane and it proceeds via the formation of ketene
intermediate.
Baeyer strain theory (p. 200): The ring strain, as explained by Baeyer, which
results from the deviation of tetrahedral angle (109°28) in cyclic hydrocarbons
in their planar form.
Baeyer test (p. 240): It is a qualitative test for unsaturation due to double and
triple bonds in hydrocarbons. It involves the decolourization of cold alkaline
potassium permanganate solution when added to the unsaturated compounds.
Baeyer–Villiger rearrangement/oxidation (pp. 587, 633): The oxidation of
ketones by peroxyacids to form esters that occurs through an intramolecular
rearrangement.
Baltz synthesis (p. 1045): Synthesis of chloramphenicol from benzaldehyde and
nitroethanol.
Balz–Schiemann reaction (pp. 433, 834): The method for the preparation of
fluorobenzene by the reaction of benzenediazonium chloride with
hydrofluoroboric acid followed by heating.
Barfoed’s test (p. 991): A qualitative test for distinguishing monosaccharides
from reducing disaccharides.
Beckmann rearrangement (p. 635): The formation of N-substituted amide
from ketoximes in presence of acids like H2SO4, HCl, RCOCl, SO3, PCl5,
BF3 etc. It is an intramolecular rearrangement reaction and involves migration
of an alkyl or aryl group, which is present anti to –OH group of oxime. It is a
stereospecific reaction.
Benedict’s solution (p. 595): An alkaline solution containing copper (II) sulfate,
sodium citrate and sodium carbonate. It is a mild oxidizing agent and is used
for distinguishing aldehydes from ketones. On reaction with aldehydes it gives
a red precipitate of cuprous oxide.
Benzidine rearrangement (p. 820): In acidic medium hydrazobenzene
undergoes intramolecular rearrangement to produce benzidine.
Benzoin condensation (p. 643): Two molecules of benzaldehyde react in the
presence of potassium cyanide resulting in the formation of -hydroxyketone
(benzoin). This reaction is exclusively given by the aromatic aldehydes.
Bergius process (p. 198): The formation of alkanes by destructive
hydrogenation of coal under high temperature and pressure conditions.
Benzil-benzilic acid rearrangement (p. 644): Rearrangement reaction of 1,2-
diketones (like benzil) in presence of base results in the formation of -
hydroxycarboxylic acids (benzilic acid).
Birch reduction (pp. 389–91): Dissolving metal reduction by sodium (or Li or
K) in liquid ammonia, in presence of alcohol. Aromatic rings, nonterminal
alkynes, conjugated olefins and phenylated olefins undergo Birch reduction.
Bischler–Napieralski reaction (p. 951): Synthesis of isoquinoline derivative by
intramolecular cyclization of N-acetyl-2-phenylethylamines.
Biuret test (p. 683): A qualitative test for urea in which urea is heated above its
melting point followed by addition of copper (II) sulfate in alkaline medium
that gives a violet colour.
Blanc reaction (p. 444): It is a method for the introduction of chloromethyl
group in benzene ring by its reaction with formaldehyde and HCl in presence
of zinc chloride or aluminium chloride. The reaction is commonly known as
chloroformylation.
Bouveault Blanc reduction (p. 459): Reduction of carbonyl compounds to
alcohols using sodium and ethanol.
Brosnted and Lowry acids and bases (p. 31): It defines acids as proton donors
and bases as proton acceptors.
Bucherer reaction (p. 859): The preparation of naphthyl amines from naphthol
by reaction with ammonium hydrogen sulfide and ammonia.
Cadiot–Chodkiewick oxidative coupling (p. 290): The synthesis of dialkynes
from terminal alkynes. The reaction is not given by ethyne.
Cahn–Ingold–Prelog system (CIP system) (p. 75): The sequence rules for
assigning absolute configuration (R and S configuration) to chiral molecules.
Cannizzaro’s reaction (pp. 602, 641): Reaction of two molecules of aldehydes
containing no
α-hydrogen, in presence of a base, results in the formation of alcohol and
sodium salt of carboxylic acid. It is self-oxidation and reduction
(disproportionation) of aldehydes.
Carbylamine reaction (pp. 786, 798): Reaction of 1° amines with chloroform
and alkali (NaOH or KOH) results in the formation of isonitriles (common
name carbylamines). The isonitrile formed has a characteristic offensive
odour. This reaction is used for the qualitative identification of 1° amines.
Carius method (p. 24): A method for quantitative estimation of halogens in
organic compounds.
Chichibabin reaction (p. 946): Heating pyridine with sodamide results in the
formation of
2-aminopyridine. It is a nucleophilic substitution reaction of pyridine.
Claisen condensation (p. 712): Two molecules of -hydrogen containing ester
condense in presence of a base to form -ketoester. The reaction is used for the
synthesis of ethyl acetoacetate
(an active methylene compound).
Claisen rearrangement (p. 531): It is the thermal rearrangement of allylphenyl
ethers to form ortho allylphenols.
Claisen–Schmidt reaction (pp. 569, 642): Reaction of an aromatic aldehyde
having no -hydrogen with an α-hydrogen containing ketone in presence of a
base to form -hydroxyketone.
Clemmensen reduction (pp. 597, 639): Reduction of carbonyl compounds in
presence of
Zn-Hg/HCl to convert carbonyl group to methylene group ( C=O to CH2).
Acid sensitive carbonyl compounds cannot be reduced using this method (c.f.
Wolff–Kishner reduction).
Collins reagent (p. 924): It is dipyridine chromium oxide used for selective
oxidation of 1° alcohols to aldehydes.
Constitutional isomers (p. 28): Compounds having same molecular formula but
different bond connectivity (formerly structural isomers).
Corey’s reagent (see PCC)
Corey–House synthesis (p. 175): The reaction of haloalkanes with lithium
dialkylcuprate to form alkanes in high yields.
Coupling reaction (p. 836): The reaction of diazonium salt with aromatic
compounds containing activating groups results in the formation of azo
compounds. It is an electrophilic substitution reaction where diazonium group
behaves as an electrophile.
Crossed Cannizzaro’s reaction (p. 641): Two different aldehydes with no -
hydrogen in presence of a base react to form alcohol and sodium salt of
carboxylic acid. In the process a more readily oxidizable aldehyde forms
sodium carboxylate while the other aldehyde molecule gets reduced to
corresponding alcohol.
Curtius reaction (p. 778): Heating acid azide in alcoholic solution produces an
isocyanate, which on hydrolysis results in the formation of 1° amine.
Darzen glycidic ester synthesis (p. 604): The formation of , epoxy esters
(glycidic esters) by reaction of carbonyl compounds with -hydrogen
containing α-haloester in presence of a base.
Diastereomers (p. 67): The stereoisomers of a compound, which are not the
mirror images of each other.
Dieckmann condensation (p. 202): Reaction of -hydrogen containing diester of
dicarboxylic acid in presence of base results in the formation of cyclic -keto
ester. It is basically an Intramolecular Claisen condensation and is used for
the preparation of alicyclic hydrocarbons.
Diels–Alder reaction (p. 265): The cycloaddition reaction of conjugated dienes
with alkene (dienophile) to form cyclohexene derivatives. The product formed
is also known as Diels–Alder adduct.
DNP method (p. 1017): The method for determining the N-terminal in peptide
chain by treating it with 2,4-dinitrofluorobenzene.
Dow’s process (pp. 436, 528): Industrial preparation of phenol by heating
chlorobenzene and aqueous sodium hydroxide under high pressure.
DTNP reagent (p. 502): An abbreviation of 2,2-dithiobis-(5-nitropyridine) and
is used for the identification of thiol (–SH) group. It is a modified form of
Elman’s reagent which is 5,5-dithiobis-(2-nitrobenzoic acid).
Dumas method (p. 24): A method for quantitative estimation of nitrogen in
organic compounds.
Edman degradation (p. 1018): A method for determining the N-terminal of
peptide chain by treating with phenylisothiocyanate. The method removes N-
terminal amino acid and leaves the rest of the polypeptide chain intact.
Elman’s reagent (see DTNP reagent)
Enantiomers (p. 66): The stereoisomers of a compound, which exhibit non-
superimposable mirror image relationship.
Etard reaction (p. 622): Oxidation of toluene with chromylchloride to get
benzaldehyde.
Fehling’s solution/test (p. 595): It is a mixture of alkaline copper (II) sulfate
solution and sodium potassium tartarate (Rochelle salt) solution. It is a mild
oxidizing agent and is used for distinguishing aldehydes from ketones. On
reaction with aldehydes it gives a red precipitate of cuprous oxide.
Fenton’s reagent (p. 966): This reagent is hydrogen peroxide in presence of
ferrous salts and is an oxidizing agent.
Finkelstein reaction (p. 396): Method for the preparation of iodoalkanes from
chloroalkanes or bromoalkanes by their reaction with sodium iodide in
acetone.
Fischer projection (p. 67): It is a representation of three-dimensional structures
in two-dimensions using vertical and horizontal lines.
Fischer–Speier method (p. 463): The formation of ester from alcohol and
carboxylic acid in acidic medium by passing the HCl gas.
Fischer–Tropsch synthesis/process (p. 198): Preparation of fuel oil from
synthesis gas.
Fittig’s synthesis (p. 851): Synthesis of naphthalene from 4-phenylbut-3-enoic
acid.
Friedel–Crafts acylation (p. 337): Reaction of acyl chlorides or acid anhydrides
with benzene (in general aromatic compounds) in presence of Lewis acids like
aluminium chloride results in the formation of aromatic ketones. It is an
electrophilic substitution reaction.
Friedel–Crafts alkylation (p. 333): Reaction of haloalkanes with benzene (in
general aromatic compounds) in presence of Lewis acids like aluminium
chloride results in the formation of alkylbenzenes. It is an electrophilic
substitution reaction.
Friedlander’s synthesis (p. 949): Preparation of quinoline by condensation of
o-amino-benzaldehyde and acetaldehyde in alkaline medium.
Fries rearrangement (p. 540): It is the rearrangement of phenolic esters in
presence of AlCl3 to form ortho, and para acyl phenols. It is an electrophilic
substitution reaction where acylium ion (RCO+) is an electrophile. The
reaction may follow inter or intramolecular rearrangement.
Gabriel phthalimide synthesis (p. 778): Synthesis of 1° amines by reaction of
haloalkanes with potassium phthalimide.
Gattermann aldehyde synthesis (p. 624): It is a method used for the
introduction of aldehydic group (formylation) in the aromatic system. The
reaction is carried out in aromatic compounds (e.g. benzene, alkylbenzene,
phenol, ethers) with HCN and HCl in presence of AlCl3.
Gattermann–Koch reaction (p. 624): It is a method used for the introduction of
aldehydic group in the aromatic system. The reaction is carried out in aromatic
compounds like benzene (or alkylbenzene) with CO and HCl in presence of
AlCl3 as catalyst. In case of alkylbenzene ortho and para substituted
aldehydes are formed.
Gilmann reagent (Chapter 25, Part I): The lithium dialkylcuprate is referred as
Gilmann reagent and is used for the synthesis of a number of organic
compounds. The reagent is prepared from organolithium compounds by
reaction with copper (I) iodide in dry ether.
Gomberg reaction (p. 834): The arylation of diazonium salts with benzene in
alkaline medium, which results in the formation of biaryls.
Grignard reagent (p. 416, Chapter 25, Part I): The alkyl or arylmagnesium
halides (RMgX or ArMgX) formed by reaction of haloalkanes (or haloarenes)
with magnesium metal in anhydrous ether (see also Chapter 25, Part I).
Haloform reaction (see Iodoform reaction)
Hantzsch synthesis (p. 944): Synthesis of pyridine derivatives by condensation
of β-diketo compounds with an aldehyde and ammonia.
Haworth formulae (p. 976–79): Ring formulae, depicting hemiacetal forms of
sugars , based on Fischer projection formulae.
Haworth synthesis (p. 850): Synthesis of naphthalene from benzene and
succinic anhydride. Similarly, synthesis of phenanthrene can be carried out
from naphthalene and succinic anhydride.
Hell–Volhard–Zelinsky reaction (HVZ reaction) (p. 671): The synthesis of -
Chloro or
-bromocarboxylic acid by the treatment of carboxylic acid with halogens (Cl2
or Br2) in presence of red phosphorous. The second α-hydrogen may be
replaced by treating with excess of Cl2 or Br2.
Heumann synthesis (p. 1083): It is a synthesis of indigo dye.
Hinsberg method (p. 772): The method used for the separation of 1°, 2° and 3°
amines by treating the mixture of amines with benzenesulfonyl chloride
(Hinsberg reagent) in presence of alkali.
Hoesch condensation (p. 627): The formation of aromatic ketones by reaction
of polyhydric phenols and alkanenitriles (or benzonitrile) in presence of HCl
and anhydrous zinc chloride, using ether as solvent.
Hofmann degradation reaction [Hofmann rearrangement] (p. 776): The
reaction of amides with bromine in alkaline medium that results in the
formation of a primary amine having one carbon atom less than the parent
amide.
Hofmann elimination/rule (p. 217): -Elimination in quaternary ammonium
hydroxides, which results in the formation of a less substituted alkene as the
major product (c.f. Saytzeff’s rule).
Hofmann mustard oil test (p. 786): The reaction of 1° amine with carbon
disulfide in presence of mercury chloride results in the formation of an
isothiocyanate having a typical mustard oil odour.
Hofmann’s method (p. 771): It is a method for the separation of a mixture of
1°, 2° and 3° amines by treatment with diethyloxalate.
Huckel’s rule (p. 298): Cyclic, conjugated, planar systems exhibit aromatic
character if they have 4n+2 delocalized electrons (n is an integer).
Hunsdiecker reaction (p. 396): It is a method for the preparation of halogenated
hydrocarbons from the silver salt of carboxylic acid and halogen in presence
of solvent like CCl4. The reaction proceeds through a free radical mechanism.
Internal Cannizzaro’s reaction (p. 602): It is a reaction of dialdehyde or
ketoaldehyde, with no
-hydrogen, (both the C=O groups are present in the same molecule) in
presence of a base to produce α-hydroxycarboxylate. During the course of
reaction, one of the carbonyl group (preferably aldehyde) is oxidized and other
carbonyl group is reduced.
Iodoform reaction (p. 606): The carbonyl compounds containing CH3C(O)–
group on reaction with iodine in presence of an alkali result in the formation of
iodoform (CHI3; a yellow solid) and sodium salt of carboxylic acid. This
reaction is also given by other halogens and commonly referred as haloform
reaction. The reaction with iodine is used in the qualitative analysis of organic
compounds.
Isoprene rule (p. 1060): In terpenes a number of isoprene units are present
which are joined in head-tail arrangement (1,4-linkage).
Jones reagent (pp. 468, 924): It is an aqueous solution of chromic acid and
sulfuric acid, which is used for the oxidation of 2° alcohols to ketones.
Kharasch effect [Peroxide effect] (p. 238): In presence of peroxides the
addition of HBr to unsymmetrical alkenes results in the formation of product
where the bromine adds to the carbon of the double bond having maximum
number of hydrogen(s). It is an anti-Markovnikov’s addition (c.f.
Markovnikov’s rule/addition).
Killiani–Fischer synthesis (p. 964): A method for lengthening of carbon chain
in aldoses.
Kjeldahl method (p. 24): A method for quantitative estimation of nitrogen in
organic compounds.
Knoevenagel reaction (pp. 696, 725): Reaction of aldehydes (or ketones) with
malonic ester (a reactive methylene compound) in presence of organic bases
like pyridine result in the formation of , -unsaturated acids.
Knorr–Paal synthesis (p. 932): Synthesis of substituted pyrroles by
condensation of -amino ketones with -keto esters.
Koch reaction (p. 660): It is the synthesis of carboxylic acid from alkenes by
addition of carbon monoxide and water in acidic medium.
Kolbe’s electrolysis reaction (pp. 178, 218, 277): The electrolysis of aqueous
solution of sodium or potassium salt of carboxylic acid to form alkanes,
alkenes or alkynes through anodic decarboxylation.
Kolbe–Schmitt reaction (p. 542): Reaction of sodium salt of phenol and carbon
dioxide results in the formation of sodium salt of salicylic acid. It is an
electrophilic substitution reaction where electrophile is carbon dioxide. A
carboxylic group is introduced at ortho position due to stabilization of
intermediate stage through intramolecular hydrogen bonding.
Laderer–Manasse reaction (p. 546): Hydroxymethylation of phenol at ortho
and para positions by its reaction with formaldehyde in alkaline medium.
Reaction is used for manufacturing bakelite.
Lassaigne test (p. 23): A method for qualitative determination of extra elements
(N, S, Cl, Br, I) in organic compounds.
Lethal dose-50 (LD50) (p. 1039): The dose required to kill the 50% of the
population of test animal. A smaller value indicates high toxicity.
Lewis acids and bases (p. 32): A Lewis acid is an electron deficient species,
which accepts an electron pair. A Lewis base is an electron rich species, which
donates an electron pair.
Libermann nitroso test (p. 785): A qualitative test for aliphatic 2° amine.
Lindlar’s catalyst (p. 213): A deactivated catalyst having Pd/CaCO3 in lead
acetate poisoned with small amount of quinoline and is used for the reduction
of alkynes to cis-alkenes.
Long’s synthesis (p. 1045): Synthesis of chloramphenicol from p-
nitroacetophenone.
Lossen rearrangement (p. 777): The thermal rearrangement of hydroxamic acid
to isocyanate, which on hydrolysis result in the formation of 1° amine.
Lucas test (p. 465): It is used to distinguish 1°, 2° and 3° alcohols by reaction
with HCl and anhydrous ZnCl2 (Lucas reagent).
Mannich Reaction (pp. 603, 647): Reaction of formaldehyde with active
hydrogen containing compounds in presence of 1° or 2°aliphatic amines
results in the formation of amino methylated products. (Active hydrogen
containing compounds include phenol (o and p hydrogens), RCCH, HCN, R–
OH, R–SH, G–C–H (G = electron withdrawing group like CN, –NO2,
–COOR, C=O).
Markovnikov’s rule/addition (p. 226): In the addition of polar reagents to
unsymmetrical alkenes the negative part of the addendum (polar reagent) adds
to that carbon of double bond, which has minimum number of hydrogen(s). It
is a regioselective addition of reagent.
Meerwein–Ponndorf–Verley reduction (pp. 459, 639): It is a method for the
reduction of carbonyl compounds to alcohols, in presence of aluminium
isopropoxide. The reduction of carbonyl to alcoholic group occurs selectively
without reducing other functional groups (e.g. –C=C–, –NO2 etc.) present in
the compound. The reverse of this reduction reaction is known as Oppenauer
oxidation.
Merrifield solid phase peptide synthesis (p. 1015): The method for the peptide
synthesis, which makes use of solid support (copolymer of styrene with 4-
(chloromethyl) styrene) to which
C-terminal of amino acid is attached and then successive amino acids are
attached via peptide bond formation. Once the synthesis is complete the
polypeptide is removed from the support.
Meso compound (p. 67): A compound having stereogenic centres but optically
inactive due to presence of plane of symmetry.
Michael addition (p. 611): The compounds with reactive methylene groups
undergo nucleophilic addition to activated olefinic bonds, as in , -unsaturated
carbonyl compounds, in presence of a base as a catalyst.
NBS (p. 245): It is an abbreviation for N-bromosuccinimide and is used for
allylic bromination of alkenes.
Nef reaction (p. 794): Formation of aldehydes and ketones by acidic hydrolysis
of sodium salt of 1° or 2° nitroalkanes.
Newman projection (p. 91): It is a representation of the conformations of
molecules in which the attachment of two sigma-bonded carbons is indicated
through a dot and a circle.
Ninhydrin reaction/test (p. 1009): Ninhydrin is a hydrate of indane-1,2,3-trione
. It gives a blue violet colour with amino acids and is used as a qualitative test.
Oppenauer oxidation (p. 597): Oxidation of alcohol to a ketone in presence of
aluminium alkoxide as catalyst.
Paal–Knorr synthesis (p. 931): Synthesis of pyrrole by heating 1,4-dicarbonyl
compounds with ammonium carbonate.
PCC (pp. 468, 924): It is an abbreviation of pyridiniumchlorochromate and this
reagent is used for selective oxidation of 1° alcohols to aldehydes.
Perkin condensation (p. 645): Condensation of aromatic aldehydes with
anhydride of aliphatic carboxylic acid containing at least two -hydrogens, in
presence of sodium salt of same aliphatic acid results in the formation of , -
unsaturated acids. This reaction occurs only with aromatic aldehydes.
Pinacol–Pinacolone rearrangement (p. 517): The rearrangement of di-tert.1,2-
diols (pinacols) in acidic medium to carbonyl compounds (pinacolone).
Racemic mixture (p. 74): A mixture containing equal amounts of pair of
enantiomers and is optically inactive.
Raney nickel (p. 593): A nickel aluminium alloy used as a catalyst in
hydrogenation reactions.
Reformatsky reaction (p. 584): Reaction of carbonyl compounds and α-halo
esters in presence of zinc, which results in the formation of β-hydroxy esters.
Reimer–Tiemann reaction (p. 543): Reaction of phenol and chloroform in
presence of sodium hydroxide to form 2-hydroxybenzaldehyde
(salicylaldehyde). An aldehyde group is introduced at ortho and para
positions however, ortho substituted product predominates. It is an
electrophilic substitution reaction where electrophile is dichlorocarbene.
Rochelle salt (p. 595): It is sodium potassium tartarate (see also Fehling’s
solution).
Rosenmund reduction (pp. 577, 628): Catalytic reduction of acid halides in
presence of Pd/BaSO4 to corresponding aldehydes.
Ruff degradation (p. 966): A process for shortening of carbon chain of an
aldose.
Sabatier–Senderens reaction (p. 172): The hydrogenation of alkenes and
alkynes using nickel catalyst at a higher temperature.
Sandmeyer reaction (pp. 433, 834, 835): It is a reaction of arenediazonium salt
with CuCl, CuBr or CuCN which results in the formation of chloroarenes
,bromoarenes or corresponding benzonitrile derivatives.
Sanger method (see DNP method)
Saytzeff’s rule (p. 214): Whenever there is a possibility of the formation of two
or more alkenes during the process of -elimination reaction, the preferred
product (major product) is a more substituted alkene.
Schiff’s bases (p. 589): The condensation product of aldehydes and ketones with
1° amines.
Schiff’s reagent (p. 1078): An aqueous solution of rosaniline dye, which is
decolorized by passing sulfur dioxide. This reagent is used for qualitative
detection of aldehydes.
Schmidt reaction (p. 778): Formation of 1° amine from reaction of carboxylic
acid and hydrazoic acid in acidic medium through an isocyanate intermediate.
Schotten–Baumann reaction (pp. 535, 826): The introduction of benzoyl group
in phenol and aniline using benzoyl chloride and aqueous NaOH.
Simmons–Smith reaction (p. 201): The reaction of alkenes with diiodomethane
in presence of zinc-copper alloy to synthesize cyclopropane derivative.
Skraup’s synthesis (p. 949): Synthesis of quinoline from aniline and glycerol in
presence of nitrobenzene as a dehydrogenating agent.
Sommelet reaction (p. 628): Formation of benzaldehyde from benzyl chloride
and hexamethylene-tetramine.
Stephen reaction (pp. 578, 628): The method for the conversion of nitriles to
corresponding aldehydes using HCl and anhydrous SnCl2.
Stereogenic centre (p. 66): Carbon attached to four different atoms or groups
(formerly chiral or asymmetric centre).
Strecker synthesis (p. 1004): The reaction is used for the preparation of -amino
acids by reaction of aldehydes (or ketones) with HCN in presence of ammonia
followed by hydrolysis.
Tischenko Reaction (p. 604): Two molecules of aldehydes (with or without α-
hydrogen) in presence of aluminium alkoxide result in the formation of an
ester. In this process one molecule of aldehyde is oxidized to carboxylic acid
and another aldehyde molecule gets reduced to alcohol but they form ester as
the final product (c.f. Cannizarro’s reaction).
Tollens reagent/test (p. 594): It is ammonical silver nitrate solution. It is a mild
oxidizing agent and is used for distinguishing aldehydes from ketones. On
reaction with aldehydes silver gets precipitated and reaction is also known as
silver mirror test.
Ullmann Reaction (p. 442): It is a method for the preparation of biphenyl form
halobenzene in presence of Cu. The reaction occurs through the coupling of
phenyl radicals, which are generated during the course of the reaction.
Victor Meyer test (p. 471): Test for distinguishing 1°, 2° and 3° alcohols.
Wacker’s process (p. 656): It is an industrial process for the synthesis of
acetaldehyde by oxidation of ethene in the presence of palladium chloride and
copper chloride catalyst.
Walden inversion (p. 400): Inversion of configuration at a chiral centre during
the course of substitution reactions. It is characteristic of bimolecular
nucleophilic substitution reactions. Named after Paul Walden who developed a
reaction sequence where a chiral starting material was transformed to its
enantiomer by a series of stereospecific reactions.
Wagner–Meerwein rearrangement (p. 216): The 1,2 hydride or 1,2-methyl
shift in carbocations that result in the formation of more stable carbocation
which leads to the formation of rearranged product during reactions.
Wohl degradation (p. 966): A method for shortening of carbon chain of aldoses.
Williamson ether synthesis (p. 484): Method for the preparation of ethers from
alkoxides and primary haloalkane through SN2 mechanism.
Witt theory (p. 1068): The chromophore-oxochrome theory for explaining the
colour of organic compounds.
Wittig reaction (p. 582): Method for the synthesis of alkenes by the reaction of
carbonyl compounds with phosphorous ylides.
Woodward–Fieser rules (p. 1113–1119): Rules used for theoretical calculation
of lmax for conjugated systems like conjugated dienes, a, b-unsaturated
carbonyl compounds and aromatic compounds. It theoretically deals with
bathochromic or hypsochromic effect of various groups or substituents present
in molecules.
Wolff–Kishner reduction (pp. 597, 639): Reaction of carbonyl compounds with
hydrazine followed by heating with sodium ethoxide result in the reduction of
carbonyl group to methylene group ( C=O to CH2). Base sensitive carbonyl
compounds cannot be reduced using this method
(c.f. Clemmensen reduction).
Wurtz Reaction (p. 174): It is a method for the preparation of alkanes, using the
reaction of haloalkanes and sodium metal in presence of dry ether.
Wurtz–Fittig Reaction (pp. 315, 442): It is a method for the preparation of
alkylbenzenes, which involves the reaction of haloarenes with haloalkanes in
presence of sodium metal (c.f. Wurtz reaction).
Zeisel’s method (p. 489): Method for detection and estimation of alkoxy group
in an organic compound.
Ziegler–Natta catalyst (p. 1093): It is a mixed organometallic catalyst
consisting of titanium tetrachloride and triethylaluminium and is used for the
synthesis of stereoregular polymers.
Zwitterion (p. 1001): A compound containing equal number of negative and
positive charges. Amino acids exist in Zwitterion form where amino group is
in its protonated form and carboxylic acid group exists as carboxylate.
Index
Absolute alcohol, 473
Absolute configuration, 75
Absorption spectroscopy, 1111
Acetaldehyde (see also aldehydes),
commercial synthesis, 574, 656
polymers of, 609
Acetamide, 669
Acetanilide, 827, 828
Acetals, 524, 556
as protecting group, 891, 892
Acetic acid (see also carboxylic acids, aliphatic)
industrial preparation, 656, 661
Acetoacetic ester (see ethyl acetoacetate)
Acetone, 572, 576, 583, 598, 606 (see also carbonyl compounds)
Acetonotrile (see alkanenitrile), 49, 624
Acetophenone, 622–624, 626, 629–647
N-acetyl-2-phenylethylamines, 951
Acetylacetone, 710, 720
Acetylation, 535, 827, 828, 830, 831, 840, 842
as protecting group, 882, 895, 896, 897
Acetylcholine, 684
Acetylcholinesterase, 684
Acetylene (see also alkynes),
acidic character, 284, 382
hydration, 281
polymerization, 287–288
preparation of, 274–275, 276
reactions of, 258, 277, 279–282, 283, 286
structure, 11, 273
Acetylene-allene rearrangement, 288
Acetylsalicylic acid, 1040, 1042, 743
Acid anhydrides,
nomenclature, 49
physical properties, 674, 676
preparation of, 667
reactions of, 677
Acid chlorides, 577, 628, 778
Acid dyes, 1066, 1073, 1080, 1081
Acid halides,
nomenclature, 49
physical properties, 674, 675
preparation of, 666
reactions of, 676
Acid value, of fats and oils, 762
Acidic amino acids (see amino acids)
Acidic character; definition, 31, 32
acidic behaviour of,
alcohols, 461, 462
carbanions, 165, 166
carboxylic acids, 662, 663, 693, 734–
737
geometrical isomers, 694, 883
-hydrogens in aldehydes and ketones, 599
phenols, 531, 534
reactive methylene compounds, 711
sulfonic acids, 745, 746
terminal alkynes, 284, 382
Bronsted and Lowry acids, 31
Lewis acids, 32
problems based on, 165, 166, 382, 549, 685, 755, 883
Acids and bases,
Bronsted and Lowry, 31
definition, 31
Lewis, 32
pKa and pKb values, 32
Acrolein, 523
Acrylonitrile, 1096
Active methylene compounds , 712
Acyl phenols; ortho, and para, 540
Acylium ion, as electrophile, 337, 540
Adam’s catalyst, 172
1,2- and 1,4-addition, 269–264
Addition polymerization, 1090
anionic mechanism, 1092
cationic mechanism, 1091
free radical mechanism, 1091
Ziegler–Natta polymerization, 1093
Addition polymers, 1090
Addition reactions (see electrophilic, nucleophilic and free radical addition reactions)
polypropene, 247
Adenine, 1034, 1035, 1036
Adenosine, 1034
Adipic acid, 699, 718, 724, 1103
Alanine, 998, 1008
Alcohols, from molasses, 473
acidic and basic character, 461, 462
by fermentation, 472
conversion to amines, 467
dehydration of, 469, 473, 567
dehydrogenation of, 469
formation of esters, 463
hydrogen bonding, 452
interconversion of, 1°, 2° and 3° alcohols, 474, 476, 477
iodoform reactions, 472, 476
nomenclature of, 45, 452
nucleophilic substitution reactions, 463–467, 473
oxidation of, 467–469, 574, 924
physical properties, 452
preparations of, 229–233, 453–461
from Grignard reagent, 455–457
from organolithium compounds, 458
reactions of, 462–472
solubility, 453
structure of, 451
Victor Meyer test, 471
Aldehydes (aliphatic and aromatic) (see also carbonyl compounds)
distinguishing aldehydes from ketones, 594, 595
oxidation reactions, 593–595, 637
polymerization reaction, 608
preparations of, 573–578, 622–625
Aldohexoses, 955, 958
Aldol Condensation, 599, 873
Aldonic acid, 970
Aldoses, 955, 958
Alicyclic hydrocarbons (see cycloalkanes), 202
Alizarin, 1082
Alkadienes, classification, nomenclature and stability, 253–254
Alkaloids, 1058, 1059
Alkanedioic acids (see also dicarboxylic acid, saturated)
Alkaneisonitrile,
nomenclature, 50, 796
physical properties of, 797
preparations of, 797
reactions of, 798–802
Alkanenitriles,
basic strength of, 805
nomenclature, 49, 796
physical properties of, 797
preparations of, 798
reactions of, 578, 628, 798–800
Alkanes,
aromatization, 193
conformations, 90
Corey-House Synthesis, 175, 206
cracking/pyrolysis, 194
dehydrogenation, 193
free radical substitution reactions, 179–191
from alkylboranes, by hydrolysis, 179
from carbonyl compounds, 176–178
isomerism, 170
isomerization, 195
nomenclature, 38, 170
oxidation reactions, 191
physical properties, 170, 171, 205
preparation of, 171–179, 218, 277,
via Alkyllithium, 176
via Grignard Reagent, 173, 206
Wurtz Reaction, 174, 207
Alkenedioic acids, E- or Z- (see also dicarboxylic acid, unsaturated)
Alkenes,
allylic substitution reaction, 244
electrophilic addition reactions, 222–236
addition of carbene, 236
addition of halogens, 223
addition of hydrogen halides, 225
hydroboration–oxidation, 232
oxymercuration-demercuration reaction, 230, 251
E/Z nomenclature, 87
free radical addition reaction, 237–239
geometrical isomerism, 86, 87, 211
isomerism, 211
isomerization, 247
Kharasch effect (see peroxide effect), 238
Markovnikov’s Rule, 226
nomenclature, 41, 211
oxidation reactions, 239–244
ozonolysis, 241–243
peroxide effect, 237
physical properties, 211
polymerization, 246
preparation of, 212–218
by elimination reactions, 213–217
Alkenes (Continued)
cis and trans alkenes, 213
Hydroboration–protonation of alkynes, 283
stability of, 219–221, 249
structure of, 210
Alkoxymecuration–demercuration, 486
Alkylborane, 179, 232, 282, 283
Alkyl carbanion, stability of, 123
Alkyl carbocation, as electrophile, 334
Alkyl carbocations, stability of, 118, 119
Alkyl free radicals, stability of, 125, 126
Alkylation, of alkenes, 235 (see also Friedel-Crafts alkylation)
Alkylbenzenes, preparation of, 315, 333–336, 442
Alkylsulfides (see thioethers)
Alkynes,
acetylene-allene rearrangement, 288
acidic strength, terminal, 284
distinguishing terminal alkynes, 286, 291
electrophilic addition reactions, 278–282
higher alkynes from lower alkynes, 277, 285
hydrogenation, 172, 213, 278
isomerism, 273–274
isomerization, 288
nomenclature, 42, 273
nucleophilic addition reactions, 283
oxidation reactions, 289–290
physical properties, 274
polymerization reactions, 287
preparation of, 275–277
reduction to cis-alkenes, 213
structure of, 273
Allenes, 253
stereochemistry, 85
Allethrin, 1052
Allyl alcohol, 520, 523
Allyl carbanion, stability of, 123, 124
Allyl carbocations, stability of, 120, 560
Allyl chloride, 520
Allyl free radicals, stability of, 127, 245, 246, 375, 558
Allyl halides, 418, 419
Allyl iodide, 521, 522
Allylic bromination, 245, 375, 558
Allylic hydrogens, 146
Allylic substitution reactions, 244–245, 558
Allylphenols; ortho and para, 712
Aluminium chloride, as Lewis acids, 333, 337,
624
Aluminium isopropoxide, 459, 639
Amadori rearrangement, 971
Ambident nucleophile, 409
Amides,
basicity, 680
physical properties, 680
preparation of, 669, 680
reactions of, 677, 776
resonance in, 680
Amines, aliphatic,
basicity, 781, 782
hydrogen bonding, 769
isomerism, 769
nomenclature, 50, 768
nucleophilic substitution reactions, 783
optical activity in, 768
physical properties, 769
preparations of, 770–771
1° amine, 774–779
2° amine, 779–780
3° amine, 780-781
reactions of, 782–787
separation of, 1°, 2° and 3°, 771, 777
structure of, 768
-Amino acids, 997
analysis of, 1002
asymmetry of, 1000
classification as,
acidic, basic and neutral, 997, 1000
essential, semi- and non-essential, 998, 1000
configuration, 1001
identification of, 1009
isoelectric point, 1002
nomenclature of, 997, 998, 999
physical properties, 1001–1002
reactions of, 1006–1009
resolution of, 1005
sequence analysis of, 1017–1022
separation of, 1000
stereochemistry, 1000
synthesis of, 1003–1005
three and one letter code, 998, 999
Zwitterion, 1002,
-Amino ketones, 932
Amino sugars, 987
p-Aminoazobenzene, 1070
o-Aminobenzaldehyde, 949
2-aminopyridine, 946
Ammonia derivatives, of carbonyl compounds, 589, 635
Ammonical silver nitrate, 286, 594, 875–878, 885–887,
Ammonium hydrogen sulfide, 859
Amorphous polymers, 1088
Amylopectin, 992–993
Amylose, 992–993
Analgesics, 1038
Angle strain, 95, 200, 364
Anhydrides; of aliphatic carboxylic acid, 645
Aniline, 820
acetylation of, 827
as protecting tool, for organic trans-formations, 828, 830, 831, 840, 842, 894–897
basicity of, 822
isomeric substituted, 823
steric effect and basicity, 824
bezoylation, 826 (see also Aromatic amines)
electrophilic substitution reactions, 827–833
halogenation, 827
in polar solvent, 827–828
in non-polar solvent, 827–828
reactions of, 825–832, 949
resonance in, 822
nitration, 829
nitrosation, 831
nucleophilic substitition retractions, 825–827
sulfonation, 830
Anomeric carbon, 963
Anomers, 963
Annulenes, 255–256
Anthracene,
aromaticity, 306, 864
Diels-Alder reaction, 867
electrophilic substitution reactions, 865–866
numbering in ring, 864
oxidation and reduction, 867
preparations of, 864–865
structure of, 863
Anthraquinone, 865, 867
Anthraquinone dyes, 1082
Anti-Markovnikov’s addition, 238
Anti-aromatic, 307
Antibacterial agents, 1042
uses, mode of action and side effects, 1044
Antibiotics, 1039, 1044
Anti-bonding molecular orbitals, 255
Antihistamines, 1039, 1047, 1048
Antimalarial, 1039, 1046
Antipyretics, 1038
Antipyrine, 721
Antiseptics, 1048
APC tablets, 1041
Apoenzyme, 1032
Arenediazonium salt
preparation of, 833
reaction of, 433, 834–838, 888, 889, 895, 896
coupling reactions and conditions for diazo coupling, 836, 837
diazonium ion as electrophile, 836, 884
in which diazo group is retained, 835, 836
involving replacement of diazo group, 834, 839, 840, 841, 842
Arenium ion, 329, 330, 332, 334, 338, 340
Arginine, 999
Arndt-Eistert reaction, 796
Aromatic amines (see also aniline)
basicity of, 822, 823, 838
steric effect, 824
nomenclature of, 821
physical properties of, 821
preparations of, 776, 778, 819–820
reactions of, 1°, 2° and 3°, 825–832
Aromatic hydrocarbons, (see also individual compounds benzene, toluene, styrene, naphthalene, etc.)
nomenclature, 308–311
physical properties, 311
source of, 308
Aromatic sulfonic acid,
acidic character and effect of substituent,
745
comparison with carboxylic acids, 745
effect of substituents, 746, 755
electrophilic substitution reactions, 749, 750
nucleophilic substitution reactions, 749
physical properties, 744
preparations of, 744
reactions of, 746–752
resonance in, 745
Aromaticity, concept of, 298, 384
discussed for,
three membered ring system,
cyclopropene, 298
cyclopropenyl anion, 299
cyclopropenyl cation, 299
four membered ring system
cyclobutadiene, 300
five membered ring system
cyclopentadiene, 300
cyclopentadienyl cation (tropylium ion), 301
cyclopentadienyl anion, 301,
ferrocene, 302
six membered ring system
benzene, 302
seven membered ring system
cycloheptatriene, 302
cycloheptatrienyl cation, 303
cycloheptatrienyl anion, 303
eight membered ring system,
cyclooctatetraene, 304
cyclooctatetraenyl anion, 304
annulenes, 304–306
[10] annulene, 305
[14] annulene, 305
[16] annulene, 305
[18] Annulene, 305
bridged annulenes, 305, 306
dehydro[14]annulene, 306
Aromaticity, concept of (Continued)
fused ring system,
azulene, 307
anthracene, 306, 864
naphthalene, 306, 848
phenanthrene, 306, 868
heterocyclic compounds,
furan, 307, 929
pyridine, 307, 942
pyrrole, 307, 928
thiophene, 307, 929
Arrows, role in organic chemistry, 15
Artificial sweeteners, 994
Aryllithium, 442
Asparagine, 998
Aspartame, 994
Aspartic acid, 999, 1005
Asphalt, 196
Aspirin, 1040, 1042
Asymmetric centre, 66
Asymmetry (see Chirality)
Atactic polymer, 1089
Atomic radius, 4
Atropine, 685, 1058–1059
Aufbau principle, 2
Auxochromes, 1068
Axial position, 97, 98
Azadirachitin, 1054
Azide, 778
Azobenzene, 819, 820
Azo-dyes, 1067, 1073
Azoxybenzene, 820
Azulene, 307

Bacteriocidal and bacteriostatic, 1039


Baeyer reagent/test, 240, 248
Baeyer’s strain theory, 200
Baeyer-Villiger rearrangement, 587, 633
Bakelite, 609, 1101
Baltz synthesis, 1045
Balz-Schiemann reaction, 433, 834
Barbitone, 726
Barbituric acid, 683, 726
Barfoed’s test, 991
Basic dyes, 1067, 1075, 1077, 1078, 1079
Basicity; definition, 31, 32
of amides, 680, 681
of amines, 781, 782, 822–824, 838
of anions, 404, 558
of guanidine, 806
problems based on, 166, 805, 806, 838
of pyridine, 943
of pyrrole, 943
Bathochromioc shift, 1070, 1111
Baygon (trade name-propoxur), 1052
9-BBN, 577
Beckmann rearrangement, 635
Beer Lambert’s law, 1112
Bees wax, 759, 763
Benidict’s solution, 595
Benzal chloride, 443, 447
Benzaldehyde (see also aldehydes), 622–628, 629–647, 1045
Benzene,
aromatic character, 302
electrophilic substitution reactions (see also), 313–314,
heat of hydrogenation, 297
Kekule structure, 295
orbital picture, 295–296
oxidation, 315
preparation of, 193, 312
reactions of, 313–315
resonance energy and stability, 297
resonance structure, 295
source of, 308
2
sp hybridization and structure, 296
Benzenediazonium chloride
reactions of, 433, 834, 835, 836, 884
Benzenehexachloride, 315, 1050
Benzeneisonitrile, 798
Benzenesulfonic acid, 314, 339
Benzenesulfonyl chloride, 777
Benzidine, 820
Benzidine rearrangement, 820
Benzil, 644
Benzil-benzilic acid rearrangement, 644
Benzilic acid, 644
Benzoic acid (see carboxylic acids, aromatic), 446, 447
Benzoin, 643
Benzoin condensation, 643, 874
Benzonitrile, 309, 433, 627, 628, 732, 834, 835
Benzophenone, 622, 623, 626, 629–647
Benzopyridines (see quinoline and isoquinoline), 948
o-Benzoquinone, 548, 1071
p-Benzoquinone, 548, 549, 1071
Benzotrichloride, 443, 447
Benzoylation, 535, 826, 882
Benzoyl chloride, 535, 628, 826, 882
Benzyl alcohol, 443, 445
N-benzoyl amino acids (see also hippuric acid)
Benzyl carbanion, stability of, 124
Benzyl carbocation, 120
stability of, 121, 124
Benzyl chloride; reactions of, 627, 628
Benzyl free radicals, stability of, 127
Benzyl halides, 443–446, 564
Benzylamine, 445
Benzylic hydrogens, 146
Benzylisonitrile, 445
Benzylnitrile, 445
Benzyne, 436, 565
proof for, 438
Bergius process, 198
Betain, 582
BHC (see HCH)
Biaryls; preparation of, 834
Bifunctional carboxylic acids, 691–709
Bile acids, 1064
Biodegradable detergents, 764
Biphenyl, 86, 388, 442
Birch reduction, 213, 278, 314, 389–391, 948
Bischler-Napieralski reaction, 951
Bis-phenol-A, 1103
Biuret test, 683
Blanc reaction, 444
BoC; tert-butoxy carbonyl, 1012
Boiling points, 22
Bond angle, 5
Bond cleavage, 115
Bond connectivity, 27, 141
Bond dissociation energy, 115
Bond energy, 116
Bond length, 5
Bond line formula, 13, 14
Bond polarity, 6
Bonding molecular orbitals, 255
Borane (see boronhydride)
Boranes, 179, 232, 282, 283, 577
Boron hydride
functional groups not reduced by, 922
general characteristics of, 920
preparation of, 920
reactions of, 179, 232, 282, 283, 577
reduction of functional groups, 921
Bouveault Blanc reduction, 459
Bridged hydrocarbons, 144–145
Bromoacetic acid, 663
Bromoalkanes, 396
Bromoanisols (o,m), 448
-Bromocarboxylic acid; preparation of, 671
Bromoarenes; formation of, 433, 834, 835
Bromobenzene, 330 (see also haloarenes), 385
Bromoform, 422
Bromonium ion, as electrophile, 330
N-bromosuccinimide (see NBS)
Bromotrichloromethane, 264
Bronsted and Lowry acids and bases, 31
BTX (benzene, toluene and xylene), 197
Bucherer reaction, 859
Buna rubber; buna-N and buna-S, 268, 269, 1106
Z-But-2-enedioic acid (see Maleic acid)
Buta-1,3-diene,
1,2- and 1,4-addition, 260
bond length, 255
conformations; s-cis and s-trans, 256
cycloaddition, 265
delocalization, 254
electrophilic addition, 259–263
free radical addition, 263–264
molecular orbitals picture, 255
ozonolysis, 267
polymerization, 268
preparations of, 257–258
reactions of, 258–269
Butane, 171, 173, 174, 176, 177
conformation, 93, 157–158
conformational analysis, 94
isomerism, 195
isomerization, 170
Newman projection, 94
Butanedioic acid (see succinic acid)
Butanone, 578, 579
Butyllithium, 908

Cadiot-Chodkiewick oxidative coupling, 290


Cahn-Ingold-Prelog system (see CIP system)
Camphor, 1061, 1095
Cane sugar, 955
Cannizzaro’s reaction, 602, 641, 874, 843, 939
Caprolactam, a cyclic amide, 1099
Carbamates, 1100
Carbanion, formation, structure and stability of, 122–124, 165, 166
enolization and resonance stabilization, 711
Carbazole, 863
Carbenes, 127, 128
reactions of, 192, 200, 201, 236
Carbenium ions, 117
Carbobenzoxy group, 1011
Carbocation, formation, stability and structure of, 116–121, 166–168, 373, 375
nomenclature (IUPAC), 167
rearrangements and stability, 216, 227, 373, 568, 570
Carbohydrates, classification of, 955, 956
chirality, 956
Carbon black, 191
Carbon dioxide; as electrophile, 542
Carbon monoxide, 660
Carbon nucleophiles, 407, 582–585, 629–632
Carbonic acid, 681, 801, 1103
Carbonium ions, 117
Carbonyl compounds (see also aldehydes and ketones)
acidic character, 598,
electrophilic substitution reactions, 647
hydrogen bonding in, 572
Carbonyl compounds (Continued)
iodoform reaction, 606, 646
keto-enol tautomerism, 598
nomenclature of, 46, 47, 571, 621
nucleophilic addition reactions, 581–593, 629–637
oxidation of, 593–595, 637–638
physical properties of, 572, 621
preparation of, 573–579, 622–628
reaction with Grignard reagent, 584, 630
reactions via enolate ions, 598
reactivity of, 580, 629
reduction of, 595–598, 638–640
resonance in, 629
structure of, 571, 621
,-unsaturated, 610, 879, 880, 889
Carboxylic acids; aliphatic,
acidic character and effect of substituent, 662, 685
dimerization, 655
hydrogen bonding in, 655
nomenclature, 48, 655
nucleophilic acyl substitution (see also as individual heading), 665
physical properties, 654
preparations of, 656–661, 685
qualitative test for, 664
reactions of, 664–672
Carboxylic acids, aromatic,
acidic character and effect of substituent, 734, 735, 755
comparison with aliphatic acids, 734
comparison with isomeric substituted benzoic acids, 736
basicity, 737
hydrogen bonding in, 730
nomenclature, 729
physical properties, 730
preparations of 730–734
resonance in, 741
reactions of, 738–743
structure of, 729
Carboxylic acid, derivatives of, 674 (see acid anhydrides, acid halides, amides, esters, ketene)
reactivity of, 680
Carboxylic acids; substituted (see hydroxycarboxylic acids and halocarboxylic acids)
Carboxypeptidase, 1019
Carbylamine reaction, 786–787
Carbylamines (see isonitriles)
Cardiac glycoside, 1064
Carius method, 24
Carnauba wax, 763
-Carotene, 1063
Cartenoids, 1062
Catalyst, 134
aluminium alkoxide, 597
base as, 611
cobalt-thorium, 199
nickel, 172
palladium chloride and copper chloride, 656
Pd/BaSO4, for hydrogenation, 577, 628
titanium tetrachloride and triethylaluminium, 1093
uses in hydrogenation, 593
Catalytic hydrogenation of, 914
acid chlorides, 577
aldehydes, 596, 638
alkenes, 172,
alkynes, 172, 213
imines, 776
nitriles, 775, 799
ketones, 596, 638,
reactivity order, 915
Catechol, 547
Catechu, 1072
Cbz; abbrev. of Carbobenzoxy, 1011–1012
Cellobiose, 990
Cellulose, 992–993
CFC’s (see chlorofluorocarbons)
Chain growth polymerization, 1090–1095
Chain isomers, 28, 170
Chemical energetics, 132
chemical equilibrium, 133
energy diagrams (or energy profile) of chemical reactions, 135
rate of reaction, 134
thermodynamics and kinetics of chemical reactions, 132
Chemical reactions, types of, 129–132
Chemical shift, 1126, 1127
Chemical weapons, 684
Chemotherapy, 1038
Chichibabin reaction, 946
Chiral centre (see stereogenic centre)
Chirality, concept of, 64
in organic molecules, 66
molecule with no stereogenic (chiral) centre, 85
Chloral, 606, 872
Chloral hydrate, 423, 872
Chloroalkanes, 396
Chloramine-T, 752
Chloramphenicol; synthesis of, 1044–1046
Chloretone, 424
Chloroacetic acid, 663
Chlorobenzene (see haloarenes), 313, 329, 436, 528, 834, 835
Chlorobenzoic acids, 735, 736
2-Chlorobuta-1,3-diene (see chloroprene)
-Chlorocarboxylic acid, 709
preparation of, 671
Chlorofluorocarbons, 425
environmental hazards, 425
nomenclature, 425
Chlorofom, preparations and reactions, 423–424, 543, 786, 798
Chloroformylation, 444
Chloromethane, reaction of, 333
Chloromethylation, 444,
Chloronium ion, as electrophile, 329
Chloropicrin, 424, 791
Chloroprene, 271–272
Chloroquine, 1046–1047
Chlorpicrin, 791
Cholestrol, 1063
Cholic acid, 1064
Choline, 684
Chromogens, 1068
Chromophore–auxochrome theory, 1068
Chromophores, 1068
Chymotrypsin, 1020
Cinnamic acid, 725
Cinnamaldehyde, 638
CIP system (see R and S configuration), 75
Cis-addition, 213, 222, 232, 239
s-Cis and s-Trans, 257, 378
cis-trans isomerism (see Geometrical isomerism),
Citral, 1061
Citric acid, 702, 706, 707, 708
Claisen condensation, 712, 939
intramolecular, 202, 873
Claisen rearrangement, 531
Claisen-Schmidt reaction, 601, 639
Clemmensen reduction, 176, 387, 597, 639, 892
Coal, 198
Coal tar, 308
Cocaine, 1058–1059
Co-enzyme, 1032
Cold sulfur vulcanization (see vulcanization)
Collin’s reagent, 924
Colour and structure, 1067–1072
complementary colour, 1067 (see also Witt theory, 1068–1069)
modern theory, 1069
quinonoid theory, 1069
Colour; of organic compounds, 1068
Conjugate acid and base, 31, 143
Coulomb’s law, 2
-Complex, 332
-Complex, 329, 332, 333, 334, 338, 340
Commercial alcohol, 473
Condensation polymerization, 1090
Condensation polymers, 1098
bakelite, 1101
nylon 6, 1098
nylon 6,6, 1098
phenol–formaldehyde resin, 1101
polycarbonate, 1100
polyester, 1099
polyurethanes, 1100
urea-formaldehyde, 1102
uses of, 1103
Condensed formula, 13, 14
Configuration, 64
inversion of, 400
isomerism, 64–86
Conformations and conformational analysis of,
butane, 93
cycloalkanes, 94
ethane, 90
propane, 92
Conformations, representation of, 90, 91
eclipsed, 90
gauche, 94
skew, 93
staggered, 90
sugars, 980, 989–991
Congo red, 1074–1075
Conjugated dienes, 253
Constitutional isomers, 28 (see chain, position, functional isomers, metamers and tautomers), 141
Contributing structures,
rules for writing, 16
stability of, 17, 142, 143
Coordination polymerization, 1103
Copolymer; styrene with 4-(chloromethyl) styrene, 1015
Copolymers, 1087
Corey’s reagent (see PCC)
Corey–House synthesis, 175, 206
Coupling reactions, 836
Covalent bond, 4
Cresols (o-, m-,p), 316, 447
Crossed Cannizzaro’s reaction, 641
Cross-linked polymers, 1087
sheet polymers, 1087
two-dimensional polymers, 1087
three-dimensional polymers, 1087
Crotonic acid, 1008
Crown ethers, 492
Cryptands, 492
Cryptates, 492
Crystal violet, 1079–1080
C-terminal analysis, 1019–1020
enzyme carboxypeptidase, 1019
C-terminal end, 1010
Cumulated dienes, 253
Curtius reaction, 778
Cyanic acid, 683
Cyanoborohydride, 775, 780
Cyanogens chloride, 798
Cyanohydrins, 582, 629
Cyanuric acid, 683
Cyclic ethers (see epoxides )
Cyclic -keto ester, 202
Cycloaddition reactions, 132, 265–267
Cycloalkanes,
conformations, 94
geometrical isomerism, 95
nomenclature, 43, 199
physical properties of, 199
preparations of, 200–203
reactions of, 204–205
Cycloalkynes, 364
Cyclobutane, 95, 96, 201, 202, 204, 205
disubstituted 96, 160
Cycloheptyne, 364
Cyclohexane, 193, 201, 202, 204
boat form, 96
chair form, 96
conformational analysis, 97
conformations, 96
disubstituted, 100, 161–163
half chair, 97
heat of hydrogenation, 297
monosubstituted, 99
stability of conformers, 97, 102, 162
substituted cyclohexanes, 103, 161
twist boat, 97, 163
Cyclononyne, 364
Cyclooctyne, 364
Cyclopenta-1,3-diene, 376, 377
Cyclopentane, 95, 96, 201, 202, 204,
Cyclopropane, 95, 96, 200, 201, 204, 205
Cyclopropene, 298
Cypermetharin, 1052
Cystein, 999
Cystine, 999
Cytosine, 1035, 1036

D and L nomenclature (see relative configuration)


Dacron (see also polyester), 1099
Dansyl method, 1018
D-arabinose, 965
Darzen glycidic ester synthesis, 604
Dashed-wedged-line structure, 64
interconversion to Fischer projection, 69, 154, 155
interconversion to Newman and Sawhorse projection, 154, 155
DCC (dicyclohexylcarbodiimide), 1013
DDT (dichlorodiphenyltrichloroethane), 1049–1050
Decalins (cis and trans), 851, 852
Decarboxylation, 178, 218, 277, 697, 883, 884
Dehalogenation, 214, 215, 276
Dehydration reactions, 215, 216, 469, 473, 567, 599, 698, 742
Dehydrohalogenation, 213, 214, 275, 276, 412, 413, 562, 563
Dehydrohalogenating agents, 275, 276, 562
Delocalization of electrons, 18, 19
through p- overlap, 111, 298
- overlap, 109, 298
-p overlap, 114
- overlap, 112
Denaturation of proteins, 1026–1028
Denatured alcohol, 473
D-2-Deoxyribose, 1034, 1035
2-Deoxy-D-ribose, 987
Deoxy sugars, 987
Detergents, 763
Deuterated alkanes, 365
Dialkylcadmium, 578
Dialkynes, 290
Diastereomers, 67, 71, 75, 148, 155–157
1,3-Diaxial interactions, 99
Diazomethane, 795
in stepping carbon chain (Arndt Eistert synthesis), 796
preparation of, 795
reactions of, 795–796
Diazonium salt; as an electrophile, 836
DIBAH, 679
DIBAL, 918
Dibenzyl, 446
Diborane (see boron hydride)
1,4-Dicarbonyl compounds; in pyrrole synthesis, 931
Dicarboxylic acid, diester of, 202
Dicarboxylic acids; saturated (see also malonic acid, oxalic acid, succinic acid)
action of heat, 697–699
ester formation, 700
ionization constants (K1 and K2), 693
IUPAC names of, 692
oxidation, 699
physical properties, 692
preparations, 695
Dicarboxylic acids; unsaturated (see also fumaric acid, maleic acid)
action of heat, 698
ionization constants (K1 and K2), 694
IUPAC names of, 692
oxidation, 699
physical properties, 692
preparations, 696
1,2-Dichlorbenzene, 441
1,4-Dichlorobenzene, 441
Digitoxigenin, 1064
Dichlorocarbene; as electrophile, 543
DichloroDiphenylTrichloroethane (see DDT)
Dieckmann condensation, 202, 873
Diels-Alder reaction, 204, 265–267, 378–379, 942
Dienes, stability of, 254
Dienophile, 265
Diesel, 196
Diethyl malonate (other names; malonic ester, diethyl propanedioate),
acidic character, 722
keto-enol tautomerism, 710
enol forms, stability of, 710
intramolecular hydrogen bonding, 710
Knoevenagel reaction, 725
physical properties, 722
preparation of, 722
reactions of, 722–726
Diethyl malonate, application in synthesis of,
alicyclic compounds, 725
alkyl substituted diethyl malonate, 722
carboxylic acids, mono, di and substituted, 723–724
heterocyclic compounds, 725–726
higher dicarboxylic acids (glutaric, adipic, pimelic acid), 724
keto acids, 724
,-unsaturated acids, 725
Diethylphthalate, 1053
1,4-Digitoxigenin, 1064
Digitoxose, 987
Dihaloalkanes, preparations and reactions, 420–422
Dihedral angle, 91
Dihydroxyacetone, 523, 524
1,2-Dihydroxyanthraquinone (see Alizarin)
2,3-Dihydroxybutanedioic acid (see tartaric acid)
1,2-Dihydropyridine, 948
1,4-Dihydropyridine, 948
Diiodomethane, use of, 201
Diisocyanates, 1103
-Diketo compounds, 944
1,2-Diketones, 644
2,5-Diketopiperazine, 1008
Dimerization, of alkenes, 234
of anthracene, 867
1,1-Dimethylethoxycarbonyl group, see BoC
2,4-D- (a herbicide), 1053
2,4-Dinitrofluorobenzene, 1017
2,4-Dinitrophenol, 143, 439
Diols (see ethylene glycol and glycols)
Diphehydramine, 1048
Diphenic acid, 868
Diphenyl, 868
Diphenylmethane, 354
Diphenylmethane, 446
Dipole moment, 6, 7, 159–160, 561
Dipole-dipole interaction, 20
Dipyridine chromium oxide (see Collin’s reagent)
Direct or substantive dye, 1067, 1074
Disaccharides, 956, 987–991
Disinfectants, 1048
Diterpenes, 1062
Dithioacids, 498
Dithiocarbamate, 1055
DNA, 1034, 1036
DNP derivative, of carbonyl compounds, 591, 635
DNP method, 1017
DNP reagent (2,4-dinitrophenylhydrazine), 591, 635
Dow’s process, 436, 528
Drug, 1038
mode of action, 1042, 1044, 1046, 1047
secondary effects, 1039
side effects, 1039
toxic effects, 1039
Dry ether; as solvent, 174, 903
DTNP reagent; abbrev of 2, 2’-dithiobis-(5-nitropyridine), 502
Duma’s decarboxylation, 177
Dumas method, 24
Dyes, 1065–1084
Dynamite, 522

E, Z system (see geometrical isomerism),


E-But-2-enedioic acid (see fumaric acid)
Edman degradation, 1018
Elastomers, 1088
Electromagnetic radiations, 1110
Electromeric effect, 108
Electron releasing groups, as ortho, para director, 344, 346
Electron withdrawing groups, as meta director, 347
Electronegativity, 3
Electronic displacements, 107 (see also inductive, electromeric, resonance effect and hyperconjugation)
Electrophiles, 128, 129
Electrophiles, Acylium ion, 337, 540
Electrophilic addition reactions,
of alkenes, 222–236, 369–374
of alkynes, 278–282, 380–384
of conjugated dienes, 269–263, 375–378
proof for, 223–224, 371
stereochemical aspects, 225, 369–371
Electrophilic substitution reactions (in disubstituted benzene),
orientation, 349, 358, 359
steric hindrance, 352
Electrophilic substitution reactions (in monosub-stituted benzene), 341
aromatic ring activators and deactivators, 342–344, 349, 358
effect of group on the orientation, 344–352, 388, 389
effect of group on the reactivity, 343, 356
mechanism, 345
meta directors, 347
nature of groups, 343
orientation and reactivity, 341–348
ortho, para directors, 344, 346, 348
Electrophilic substitution reactions, Aromatic, 316, 440, 537, 647, 741, 749, 818, 836
comparison with electrophilic addition, 327
energy profile for, 330, 340
in polynuclear hydrocarbons, 853–857, 865–866
in heterocyclic compounds, 933–938, 950
mechanism of (in benzene), 329–341
Friedel-Crafts alkylation, 333, 354
Friedel-Crafts acylation, 337
halogenation, 329, 385
nitration, 331, 385, 386
sulfonation, 339
Electrophilicity, 134, 580, 882
Electrophoresis, 1002
Elimination Reactions, 213–218
carbocation rearrangement, 216
dehalogenation of dihaloalkanes, 214
dehydration of alcohols, 215
dehydrohalogenation of haloalkanes, 213
Hofmann elimination, 217
Saytzeff elimination, 214
Saytzeff elimination vs. Hofmann elimination, 218
Wagner-Meerwein rearrangement, 216
Elimination reactions, mechanism, 412,
bimolecular elimination (E-2) mechanism, 413
comparison of E-1 and E-2, 414
elimination unimolecular of conjugate base (E1cB), 414
substitution vs. elimination, 415, 428
unimolecular Elimination (E-1) mechanism, 413
-Elimination, 214, 217
-Elimination reaction, 214
Elman’s reagent; 5, 5-dithiobis-(2-nitrobenzoic acid); DTNP reagent, 502
Emission spectroscopy, 1111
Empirical formula calculation, 24
Enantiomeric excess, 164, 165
Enantiomers, 66, 71, 75, 148, 151, 155
End group analysis, 1017–1020
Endo-; position, 266
Ene-Diol rearrangement, 967
Energy of activation, 92, 133
Enolate ions, 711
Enzymes, 1032–1033
Epimerization, 967
Epimers, 963
Epoxides,
industrial preparation, 494
isomerism, 482
nomenclature of, 46, 494
physical properties, 494
preparations of, 234, 494–495
reactions of, 495–497
ring opening mechanism
in symmetrical epoxide, 495
in unsymmetrical epoxide, 495
uses of, 494
, -Epoxy esters, see glycidic esters
Equatorial position, 97, 98
Erythro (as prefix), 72, 370, 371
Erythrose, 958
Essential amino acids, 998–1000
Essential fatty acids, 761
Esters,
of phosphoric acid, 684
physical properties, 674
preparation of, 668, 676, 677, 738
reactions of, 679
Esterification, 463, 535, 668, 747, 881, 882, 894
Etard Reaction, 622
Ethanamine, 776, 787, 769 (see also amines, aliphatic)
Ethane, 171, 173, 175, 177
bromination, 183, 366
chlorination, 182
conformational analysis, 92
nitration, 188
sawhorse projection, 91
Ethanedioic acid (see oxalic acid )
Ethanenitrile (see alkanenitrile)
Ethaneisonitrile, 787, 798, 799
Ethanoic acid (see acetic acid)
Ethanol, 453, 469–472 (see also alcohols)
Ethene, 10, 656, 1096 (see alkene)
Ethers,
comparison with alcohols, 491
isomerism, 482
nomenclature of, 46, 482
physical properties, 482
preparations of, 483–486
reactions of, 486–491, 568
uses of, 491
Ether peroxide, 487
Etherates, 487
Ethyl acetoacetate (other names; acetoacetic ester, Ethyl 3-oxobutanoate),
acid hydrolysis, 716
keto-enol tautomerism, 710
enol forms, stability of, 710
intramolecular hydrogen bonding, 710
ketonic hydrolysis, 715
physical properties, 712
preparation by Claisen condensation, 712
reactions due to enolic form, 714
reactions due to keto form, 713
Ethyl acetoacetate, applications in synthesis of,
alicyclic compounds, cycloalkane derivatives, 720
alkyl substituted ethyl acetoacetate, 715
carboxylic acids, mono, di and substituted, 716–718
diketones, 719–720
heterocyclic compounds, 721
higher dicarboxylic acids (glutaric, adipic, pimelic acid), 718
keto acids, 718–719
methyl ketones, 719
,-unsaturated acids, 719
Ethylene glycol,
preparations of, 509–512
reactions of, 512–516
uses, 516, 1103
Ethylene oxide (see epoxides)
Exo-; position, 266

Fats, 759
Fats and oils, analysis of, 762
Fatty acids, 759
Fehling’s test, 595, 673, 875–878
Fenton’s reagent, 524, 707, 966
Fermentation, 472
Ferrocene, aromaticity in, 302
Fibrous Proteins, 1025
Ficher-Tropsch process, 198
Finger print region, 1123
Finkelstein reaction, 396, 556
Fischer projection, 67–68, 148–150, 155–157
from dashed-wedged-line structure, 69, 154, 155
from Newman and Sawhorse projection, 154–157, 369–371
of carbohydrates, 958, 963, 977–980
Fischer-Speier method, 463
Fischer-Tropsch synthesis, 198
Fittig’s synthesis, 851
Fluorescein, 1081
Fluoroacetic acid, 663
Fluoroalkanes, preparation of, 397, 561
Fluorobenzene, preparation of, 433, 834
Food preservatives, 182
Formal charge, 6
Formaldehyde (see aldehydes),
polymers of, 546, 608, 1101–1103
reactions, 444, 455, 603, 647
synthesis, 192, 574
Formalin, 574, 609
Formic acid,
characteristics, 672–673
industrial preparation, 661
Formylation,
Gatterman reaction, 938
in aromatic system, 624
Vilsmeier–Haack reaction, 938
2-Formylfuran (see furfural)
Free radical addition reactions, 237–239, 263–264
Free radical inhibitors, 182
Free radical substitution reactions,
energy calculation, 184
of alkanes, 179–191, 366, 367
of alkenes, 244–245
proof for, 181
Free radicals, formation, stability and structure of, 124–127, 166
Friedel–Crafts alkylation, 313, 321, 333, 354, 441, 446, 855
alkylating agents in, 335, 390, 391
comparison with Friedel-Crafts acylation,
338
limitations of, 335, 357
Friedel–Crafts acylation, 314, 337, 387, 855, 893
synthetic applications of, 339
Friedlander’s synthesis, 949
Fries rearrangement, 540
-D-Fructofuranose, 986
-D-Fructopyranose, 986
Fructose (levulose), 955, 983
D(-) Fructose,
conversion to D-glucose, 973
cyclic ketal form, 986
cyclic structure, 985
furanose and pyranose form, 986
open chain structure, 983
reactions of, 983–985
sweetness value, 956
Fumaric acid, 696, 698, 699
Fuming nitric acid, 331
Fuming sulfuric acid, 339
Functional groups, 26, 27
Functional isomers, 29, 142
Fungicides, 1055
Furan,
aromaticity, 307, 929
Diels-Alder reaction, 942
electrophilic substitution reactions, 933–938
mercuration, 937
orbital picture, 929
physical properties of, 928
preparations of, 931–933
reaction with diazonium salts, 940
reaction with organometalic compounds, 939
reactivity, 933, 934
reduction reactions, 940–941
resonance and contributing structures, 930
structure, 929
Furfural,
Cannizzaro’s reaction, 939
Claisen condensation, 939
furil furilic acid rearrangement, 939
furoin condensation, 939
Perkin reaction, 939
preparations of, 932, 938
Furfuraldehyde (see furfural)
Furil-furilic acid rearrangement, 939
Furoic acid, 933
Furoin condensation, 939

Gabriel phthalimide synthesis, 778, 1004


D-Galactose, 958
D-Galactosamine, 987
D-Glucosamine, 987
D(+) Glucose, 959–983
configuration of, 974
conversion to D-fructose, 973
conversion to D-mannose, 973
cyclic hemiacetal form, 963
cyclic structures, 962, 981
mutarotation, 964
open-chain structure, 959, 981
osazone formation, 962, 970–972
pyranose form, 982
reactions of, 959–962
sweetness, 994
Gammexane (see lindane)
Gasohol, 199
Gasoline (or Petrol), 196, 199
Gattermann aldehyde synthesis, 624
Gattermann–Koch reaction, 624
Geometrical isomerism, 86
cis-trans system, 87
E, Z system, 87
in alkenes, 95
in cycloalkanes, 95
interconversion of, 89
Gilman Reagent (see Lithium dialkylcuprates)
Globular Proteins, 1026
Glucan, 991
Glucaric acid, 961
Gluconic acid, 961, 970
-D-Glucopyranose, 963, 977
-D-Glucose, 963, 977
Glucose (dextrose), 955
Glutamic acid, 999
Glutamine, 998
Glutaric acid, 698, 718, 724, 984
Glycans, 956
D-Glyceraldehyde, 956
Glyceraldehyde, 523, 524, 957
L-glyceraldehyde, 956
Glyceric acid, 523, 524
Glycerol,
preparation, 519, 520
physical properties, 519
reactions of, 520–525, 702, 949
Glyceryltrinitrate (TNG), 522
Glycidic esters; formation of, 604
Glycine, 998, 1003
Glycogen, 992
Glycolic acid, 681
Glycosidic linkages, 987, 991
Glyoxalic acid, 696
Gomberg dimer, 363
Gomberg reaction, 834
Grain alcohol, 472
Grignard reagent,
general charactristic of, 902
in synthesis of deuterated alkanes, 365
limitations, 908
preparation of, 416, 417, 441, 442, 903
problems based on, 556, 565, 566, 879, 880, 887
reactions of with, 904–908
acid chlorides, 457
active source of proton, 173
alkylchloroformates, 906
carbon dioxide, 659, 733
carbon disulfide and sulfur dioxide, 907
carbonyl compounds, 454, 555, 630, 631
cyanogens chloride, 798, 907
epoxides, 456, 563
esters, 456
halogenated compounds, 486
isonitriles, 799, 800
metal halides, 578, 908
nitriles, 575, 576, 624
oxocycloalkanes, 563
sulfur, 498, 504
terminal alkynes, 277, 285
side reactions, 557, 878
Guanidine, 806
Guanine, 1035, 1036
Gutta-Percha, 271

Haemoglobin, 1025
Halide ions,
as leaving group, 404, 426
basicity, 404
comparison of basicity and nucleophilicity of, 403
nucleophilicity, 403
Haloalkanes,
bond strength, 393
classification of, 392
nomenclature, 43, 393
physical properties, 393
preparation of, 394–397
Hunsdiecker reaction, 396
Finkelstein reaction, 396
Via Grignard reagent, 396
reactions of, 397–417, 556 (see also, nucleo-philic substitution reactions, Elimination reactions)
structure of, 392
Haloarenes,
electrophilic substitution reactions, 440–441
nomenclature, 431
nucleophilic substitution reactions, 434–440
a comparision with SN1 and SN2
addition-elimination mechanism, 438
elimination-addition mechanism, 435, 447, 448
physical properties, 432
preparation of, 432–434
reactions of, 434–442
reactivity, comparison with haloalkanes, 434
resonance in, 434
structure of, 431
Halocarboxylic acids, preparations and reactions of; -,- and -halocarboxylic acids, 708–709
-Halo esters, reaction of, 584
-Haloester, 604
Haloform, 422
Haloform reaction (see also Iodoform reaction), 606–608
Halogenated hydrocarbons (see haloalkane), 396
Halogenation, of alkane, 179
energy calculation, 184
of aromatic compounds, 316, 345, 537, 647, 741, 749, 818, 827, 828, 855
of benzene, 313, 329
-halogenation, 599, 712, 790
of heterocyclic compounds, 937, 945, 949, 952
reactivity and selectivity, 186–188, 208
Halogens, as ortho, para director, 348
Halohydrins, 229
Halon, 426
Hantzsch synthesis, 944
Haworth projection formulae, for sugars, 976–980
Haworth synthesis, 850, 868
HCFC’s, abbrev. of hydrochlorofluorocarbons, 425
HCH; abbrv. of hexachlorocyclohexane, 1050
–HCH (see lindane)
Heat of combustion, 220
Heat of hydrogenation, 219, 250, 297, 368
Heavy oil, 196
-Helix, 1034
-Helix, 1034
Hell-Volhard-Zelinsky reaction (HVZ reaction), 671
Hemiacetal; forms of sugars, 977
Henna, 1072
n-Heptane, dehydrogenation, 194
Herbicides, 1053
Heterocyclic compounds, nomenclature, 926–927
numbering the position, 927
Heterocyclic compounds, five membered, 928–942 (see pyrrole, furan and thiophene)
Heterocyclic compounds, fused, 948–952
(see quinoline and isoquinoline)
Heterocyclic compounds, six membered, 942–948 (see pyridine)
Heterolytic bond cleavage, 115
Heumann synthesis, 1083
1,2,3,4,5,6-Hexachlorocyclohexane (see HCH)
Hexamethylenediamine, 1103
Hexamethylenetetramine; use of, 628
n-Hexane, dehydrogenation, 193
Hexaphenylethane, 363
HFC’s (see hydrofluorocarbons)
Hinsberg method, 772–774, 900
Hinsberg reagent, 777
Hippuric acid, 1006
Histidine, 999
Hoesch condensation, 627
Hofmann degradation reaction, 776
Hofmann elimination/rule, 217
Hofmann mustard oil test, 786
Hofmann rearrangement, 776
Hofmann’s exhaustive methylation, 781
Hofmann’s method, 771
HOMO (highest occupied molecular orbital), 255
Homolytic bond cleavage, 115
Homopolymers, 1087
Huckel’s rule, 298
Hund’s rule, 2
Hunsdiecker reaction, 396, 892
Hydrates, of carbonyl compounds, 585, 872
Hydration reactions, 229–233, 281, 585, 872
Hybridization, 7
effect on bond length and bond strength, 12, 13
hybrid orbitals, 9
sp hybridization, 11
2
sp hybridization, 10
3
sp hybridization, 8
Hydantoin, 1009
Hydrazine; use in reduction, 597, 639
Hydrazobenzene, 820
Hydrazoic acid, 778
Hydrazone derivatives, of carbonyl compounds, 591, 635
Hydride shift (see Cannizzaro’s reaction and also 1,2-Hydride shift)
1,2-Hydride shift, 216, 227
Hydroboration oxidation, of alkynes, 577
Hydrocyanic acid, 796
Hydrofluorocarbons, 425
Hydrogen bonding, 20, 452, 528, 534, 572, 655, 730, 782, 538, 1036
Hydrogenation of,
alkenes, 172
alkynes, 172
coal; destructive, 198
-Hydrogens; reactivity in,
aldehydes, 599
esters, 712
ketones, 599
nitroalkanes, 790
Hydrolysis of,
acetals and ketals, 586
acid anhydrides, 677
acid halides, 676, 882
active methylene compounds, 659
amides, 681
cynamides, 780
epoxides, 495, 496
esters, 460, 488, 658, 733
ethers, 460
geminal halides, 575, 628, 732
isonitriles, 776
neopentyl halide, 559
nitriles, 656, 657, 732
tert. butyl chloride, 562
trialkylborane, 179
triglycerides, 519, 760
2-Hydroxybenzaldehyde (see salicylaldehyde)
2-Hydroxybenzoic acid (see salicylic acid)
2-Hydroxybutanedioic acid (see malic acid)
-Hydroxycarboxylic acids, 644
-Hydroxyketone, 643
2-Hydroxypropane-1,2,3-tricarboxylic acid (see citric acid)
2-Hydroxypropanoic acid (see lactic acid)
2-Hydroxypyridine, 946
-Hydroxysuccinic acid (see malic acid)
-Hydroxy esters; formation of, 584
-Hydroxyaldehyde (see aldol),
-Hydroxyketone, 599, 601, 639
o-Hydroxybenzaldehyde (see salicylaldehyde )
o-Hydroxybenzoic acid (see salicylic acid)
Hydroquinone, 547, 548
Hydroxamic acid, 777
Hydroxycarboxylic acids (see also citric acid, lactic acid, malic acid, tartaric acid)
action of heat, 706
IUPAC names of, 700
oxidation, 706
physical properties, 700
preparations, 701–702
reactions of, 702–708
reduction, 707
Hydroxylamine, 960
Hydroxylation reactions, 239, 240
Hyperconjugation, 112–115, 119, 125
isovalent, 113
sacrificial, 113
Hypsochromic shift, 1070, 1111

Indane-1,2,3-trione; 1009
Indigo, 1083–1084
Inductive effect, 107, 561, 564
in acidic strength, 462, 662, 663, 693
in basicity, 781
in decarboxylation, 883, 884
in esterification, 881, 882
in reactivity of carbonyl compounds, 580, 598, 599
in reactivity of diazonium salts, 884
in stability of carbanions, carbocations, free radicals, 118, 123, 125
in stability of hydrates, 872
Infrared (IR) spectroscopy, 1120–1124
problems based on, 1123–1124
characteristics stretching frequencies, 1121
Inner salt (see Zwitterions)
Insecticides, 1049
Intermolecular forces, 19–22
Internal Cannizzaro’s reaction, 602
Intramolecular cyclization, 951
Invert sugar, 989, 994
Iodine value, of fats and oils, 762
Iodoalkanes; preparation of, 396, 556
Iodoform reaction, 422, 472, 606–608, 646
2-Iodopropane, 521, 522
Ion–Exchange resins, 1107
Ion-dipole interaction, 19
Ionic bond, 3
Ipso-substitution, a definition, 440
IR spectra, 1122
Isobutane, 170
Isobutene, 211, 234, 235
Isocyanates, 776–778, 801–803
nomenclature, 801
Iodoacetic acid, 663
Isoelectric point (symbolized as pI), 998, 999, 1002
Isolated dienes, 253
Isoleucine, 998
Isomerism, 27–30
constitutional isomers, 28, 141
stereoisomers, 28, 70
Isonitriles (see alkaneisonitrile)
Isooctane, 197
Isopentane, 170
Isophthalic acid, 319
Isoprene,
preparation of, 269
reactions of, 270–271
rule, 1060
Isoquinoline, 948, 951, 952
Isotactic polymer, 1088
Isothiocyanates, 786, 801–804
Isothiocyanic acid, 801
Isoxazolone, 721
IUPAC nomenclature, 36
common errors in writing names, 56–59
organometallic compounds, 51
polyfunctional organic compounds, 38, 51–55
principal functional groups, suffixes and prefixes, 36–38, 51
recommendations, 42
rules for naming functional groups, 36–51 (see also individual functional groups like alcohols and ethers
etc.)

Jones reagent, 468, 924


Juglone, 1072

Kerosene, 196
Ketals, cyclic, 524, 586, 891, 892
Ketenes, 677–678, 796
Ketones (aliphatic and aromatic), (see carbonyl compounds)
preparations of, 573–579, 622–624, 626, 627
reactions of, 579–594, 596–598, 627–640, 646, 647
Ketoses, definition of, 955
-Ketoester, 712, 932
Ketoximes, 635
Kharasch effect, 238
Kieselguhr (a clay type), 522
Killiani-Fischer synthesis, 964
Kjeldahl method, 24
Knocking, of engine, 197
Knoevenagel reaction, 696, 725
Knorr-Paal synthesis, 932
Koch reaction, 660
Kolbe’s electrolytic process, 178, 218, 277
Kolbe-Schmitt reaction, 542, 941

L-amino acids, 1001


Lactic acid, 582, 701, 704, 707, 708
Lactones, 705
Lactose (milk sugar), 956, 989
Laderer-Manasse reaction, 546
Laevulose, 955, 983
Lassaigne test, 23
Latex, 1104
Lawsone, 1072
Leaving groups, 404, 557, 566
Lethal dose-50 (LD50), 1039
Leucine, 998
Leuco base, 1075
Lewis acids and bases, 32, 143
Lewis structure, 2, 3, 13
Lexan, 1100
Libermann nitroso test, 785
Lindane, 1051
Lindlar’s catalyst, 213
Line angle formula (see bond line formula)
Linear polymers, 1051
Lipids, 759
Lithium aluminium hydride,
general characteristics, 916
order of reduction, of functional groups, 889, 890, 919
preparation of, 916
reduction of,
acid chlorides, 676, 917
amides, 681, 779, 780, 918
anhydrides, 677, 917
carbonyl compounds, 596, 638, 916
carboxylic acids, 670, 739, 917
epoxide, 917
esters, 679, 917
halogenated hydrocarbons, 173, 919
nitriles, 775, 918
nitro compounds, 774, 918
oxime, 775, 918
Lithium dialkylcuprates (Gilman Reagent)
in preparation of,
alcohols, 912
-alkylatated product, 611, 612, 889, 913
higher alkanes, 175, 912
higher acids, 912
ketones, 579, 890, 912
unsaturated compounds, 912
preparation of, 175, 417, 911
London forces, 21
Long’s synthesis, 1045
Lossen rearrangement, 777
Lubricating oil, 196
Lucas reagent/test, 465
Luminal, 676
LUMO (lowest unoccupied molecular orbital), 255
Lycopene, 1063
Lysine, 999

Malachite green, 1070–1075


Malathion, 1051
Maleic acid, 696, 698, 699
Malic acid, 701, 705, 707
Malonic acid, 695, 696, 697, 698
Malonic ester (see diethyl malonate)
Malonyl urea (see barbituric acid)
Maltose, 956, 990
Maltotriose, 956
Mannich Reaction, 603, 647
Mannitol, 984
D-Mannose, 958, 963, 965, 978, 979
Markovnikov’s rule/addition, 226, 374
Mass spectroscopy, an introduction, 1132
Meerwein-Ponndorf-Verley reduction, 459, 597, 639
Meisenheimer complex, 440
Melamine, 1103
Meliantriol, 1054
Melting points, 22
Mercaptans (see thiols)
2-Mercury furan chloride, 937, 938
2-Mercurythiophene chloride, 937, 938
Merrifield Solid Phase Polypeptide Synthesis, 1015–1016
Meso compounds, 67, 72, 75, 369–371
Mesomeric effect (see resonance)
Mesomerism (see resonance), 17
Mesoxalic acid, 523, 524
Meta directors, 347
Metaformaldehye (see trioxane)
Metal activated enzyme, 1032
Metaldehyde, 609
Metallic soaps, 766
Metalloenzyme, 1032
Metamers, 29
Metanilic acid, 842
Methanamine, 768–770, 781 (see also amines, aliphatic)
Methane, structure of, 8
chlorination, 180, 366
oxidation, 191
physical properties of, 171
preparation of, 176, 177
Methanenitrile, 49, 624, 625
Methaneisonitrile, 796, 799, 800
Methanoic acid (see formic acid)
Methanol, 472 (see formaldehyde)
reduction of, 178
Methionine, 998
Methylbenzene (see toluene)
Methylcyclohexane, 99, 194
Methoxychlor, 1050
2-Methylbuta-1,3-diene (see isoprene)
Methylisocyanate (MIC), 800, 801, 802, 803
Methyl orange, 1073
Methyl red, 1073–1074
Methyl-1-naphthyl ketone, 856
Methyl-2-naphthyl ketone, 856
2-Methylpropane (isopropane),
chlorination of, 187
preparation of, 195
1,2-Methyl shift, 216, 227, 560, 567
4-Methyluracil (see Thymine)
Metyl methacrylate, 1096
Michael addition, 611
Michler’s ketone, 1079
Migratory aptitude, of groups, 517, 518, 569
Milk proteins,
as antidote, 1027
denaturation of, 1027
Milk sugar, 989
Molecular formula calculation, 25
Monosaccharides, definition of, 955–956
Monoterpenes, 1060–1061
Monomers units for various polymers, 1096, 1097, 1103
Mordant dyes, 1066, 1082
Morphine, 1058–1059
Mucic acid, 933
Mustard gas, 505
Mutarotation, 964

Name reactions, typical of aromatic aldehydes


Cannizaro’s reaction, 641
Claisen-Schmidt reaction, 642
benzil Benzilic acid rearrangement, 644
benzoin condensation, 643
Perkin condensation, 645
reactions of arylmethylketones (Haloform reaction), 646
Naming organic compounds (see IUPAC nomen-clature)
use of Greek numerals, 56
Naphthalene, 814
addition reaction, 852
aromaticity, 848, 306
bond length, 849
electrophilic substitution reaction, 853–857
Kekule structure, 848–849
numbering in ring, 846
oxidation and reduction, 851, 852
ozonolysis reaction, 853
preparations of,
Fittig’s synthesis, 851
Haworth synthesis, 850
reactivity of, 849, 851
structure of, 847, 849
Naphthalene; 1- and 2-alkyl, synthesis of, 850
Naphthalene-1-sulfonic acid, 856
Naphthalene-2-sulfonic acid, 856
Naphthoic acids, 861
Naphthol, 859
Naphthols (1- and 2- or - and -naphthols), 884
preparations of, 857–858
qualitative detection of, 859
reactions of, 858–859
Naphthoquinones, 862
Naphthyl amines, 859–861
1-Naphthyl-N-methylcarbamate (see Sevin)
Natural dyes, 1072
Natural gas, 196
Natural polymers, 1086
Natural rubber, 271, 1103
NBS, 245, 375, 443, 558
Neem, 1054
Nef reaction, 794
Neopentane, 170
Neopentylhalides, 559
Neoprene, 272, 1105
Nerve gases, 684
Newman projection, 90, 91, 154–159
Nicotine, 1058–1059
Nimbidin, 1054
Nimbin, 1054
Ninhydrin reaction/test, 1009
Nitrenes, 128
Nitriles (see alkanenitriles)
o-Nitroaniline, 897
Nitro compounds, aliphatic (see nitroalkanes)
Nitro compounds, aromatic
benzidine rearrangement, 820
electrophilic substitution reactions, 818–819
nomenclature, 812
nucleophilic substitution reactions, 816–818
physical properties, 813
preparations of, 813–815
reactions of, 816–821
resonance in, 815
selective reduction, 820, 840
p-Nitroacetophenone, 1045
Nitroalkanes,
acidic character of, -hydrogen, 790
comparison with alkyl nitrite, 794
distinguishing 1°, 2° and 3° nitroalkanes, 792
isomerism, 788, 789
Nef reaction, 794
nomenclature, 50, 788
physical properties, 788
preparations of, 789–790
reactions of, 790–794
reduction of, 793
tautomerism, 789
Nitroalkanes; acidic hydrolysis, 794
Nitrobenzene, 313, 331
Nitrobenzene; as a dehydrogenating agent, 949
Nitrobenzoic acids (o-,m-,p-), 735, 736
Nitroethanol, 1045
Nitrogen bases, 1034, 1035
Nitrogen nucleophiles, 407, 588–591, 634–637
Nitronium ion, as electrophile, 331
Nitrophenols (o-,m-,p-), 533, 534, 538
Nitrosobenzene, 819
Nitrosomethylurea, 795
NMR spectra of:
benzyl acetate, 1129
1,1-dichloroethane, 1129
methyl propanoate, 1129
problems based on, 1130–1131
N-nitrosoamines, 784
Node, 255
Non-aromatic, 308
Non-essential amino acids, 1000
Novolak, 1101
N-substituted amides, 635
N-Terminal Analysis, 1017–1019
Dansyl method, 1018
2,4-dinitroflurobenzene (DNFB), 1017
DNP method, 1017–1018
Edman degradation, 1018–1019
N-terminal end, 1010
N-terminal of peptide; determining of, 1017, 1018
Nuclear magnetic resonance (NMR) spectroscopy, 1125–1131
equivalent protons, 1127
chemical shift, 1126, 1127
non-equivalent protons, 1127
signals, number of, 1127
spin-spin coupling, 1128
splitting pattern, 1128
problems based on, 1130–1131
radio frequency region, 1083
Nucleic acids, 1033–1037
structures of, 1034
Nucleophiles, 129, 402
Nucleophilic acyl substitution, in carboxylic acids, 665, 738
acid catalyzed, 666
addition elimination mechanism, 665, 738
base catalyzed, 666
formation of
acid anhydride, 667, 739
acid chloride, 666, 739
esters, 668, 739
Nucleophilic addition reactions, of carbonyl compounds,
a general discussion, on reactivity, 580, 629
general mechanism, 581
with carbon nucleophile, 582–585, 629–632
with oxygen nucleophile, 585–587, 633–634
with nitrogen nucleophile, 588–591, 634–637
with sulfur nucleophile, 592–593, 632–633
Nucleophilic substitution reactions, of haloarenes 434–440
a comparison with SN1 and SN2, 435
addition-elimination mechanism , 438
elimination-addition mechanism, 435, 447, 448
Nucleophilic substitution reactions; a general discussion, 397
comparison of SN1 and SN2, 405
effect of solvent, 401, 562
SN1 mechanism, 397, 426, 427, 559, 560
SN2 mechanism, 400, 427, 559
SNi mechanism, 466
Nucleophilicity, 134, 402
factor affecting, 402, 403, 426, 427, 558, 561
Nucleosides, 1034
Nucleotides, 1035
Nujol, 1120
Nylon 6, 1098, 1099, 1103
Nylon 6,6, 1098, 1103
Octane number, 197
Oestrone, 1064
Oligopeptides, 1009
Oligosaccharides, definition of, 956
Oils, 759–762
Oppenauer oxidation, 597,
Optical activity, 73–75, 160–162
Orbitals, 2
Organic compounds, definition of, 1
as acids and bases, 138
chemical bonding, 2
classification of, 26
electronic structure, 2
identification, 22–25
isomerism, 27
Lewis structure, 2, 3
purification, 22
structural formula, 13
Organochlorine Compounds (as pesticides), 1049–1051
Organolithium compounds, preparation of, 175, 416, 442
reactions of, 175, 513, 565, 566, 909–911
Organometallic compounds (see Grignard reagent, Organolithium compounds, Lithium dialkylcuprates)
Organophosphorous insecticides, 1049, 1051
Orlon, 1096, 1097
Osazone, 962, 970–972
Osones, 972
Oxalic acid, 695, 697, 698, 699, 700, 707
Oxalyl urea, 683,
Oxidation reactions,
of alkenes and alkynes, 234, 239, 240, 922
oxidation of alcohols and carbonyl compounds, 467–469, 731, 923
oxidation of aldehydes and ketones, 593, 594, 595, 637, 638, 924
oxidation of alkylbenzenes, 318–320, 622, 731, 925
oxidation of glycols, 515, 925
selective oxidation of 1° alcohol,
with PCC (pyridinium chlorochromate), 924
with ozone (ozonolysis), 241–242, 289, 923
Oxime, 590, 635, 775, 960
Oxonium salts, 463, 464, 487
Oxygen nucleophiles, 406, 585–587, 633–634
Oxymercuration-demercuration reaction, 230
Ozonolysis reaction, 241–243, 249, 267, 271, 289, 379, 573, 853

Paal-Knorr synthesis, 931


Paracetamol, 1041, 1042
Paraffins, 169
Paraformaldehyde, 609
Paraldehyde, 609
Pararosaniline, 1078
Parathion, 1052
Parbanic acid, 683
Pauli exclusion principle, 2
PCC; abbrev. of pyridiniumchlorochromate, 468, 924
Pentane, 195
Pepsin, 1020
Peptides, 1009
Pericyclic reactions, 132
Perkin condensation, 645
Peroxide effect (see Kharasch effect), 237, 238
Peroxyacids, 234, 587, 633
Pesticides,
toxic effects of, 1055
treatment of pesticide poisioning, 1055
Petrochemicals, 197
Petrol, unleaded (see also BTX)
Petroleum, refining, 195–196
Phase transfer catalysts, 493
Phenacetin, 1040–1041
Phenanthraquinone, 869
Phenanthrene, 868–869
aromaticity in, 265, 868
Phenobarbital (see luminal)
Phenol–Formaldehyde resin, 1101
Phenolic esters; rearrangement of, 540
Phenolic ketones, 627
Phenolphthalein, 1080–1081
Phenols
acidic strength, 531
effect of substituent, 532, 549
comparison with alcohols and carboxylic acids, 531
of isomeric phenols, 533
azo dye formation, 544
electrophilic substitution reactions, 537–544
hydrogen bonding, 528
industrial preparation, 529
nomenclature, 527
oxidation of, 545
physical properties, 528
preparation of, 528–531
qualitative detection, 536
reactions of, 534–546
resonance in, 531
structure of, 527
Phenoxide ion, 532
Phenylisothiocyanate, 1019
Phenyl carbocation, 836
Phenyl radicals; coupling of, 442
Phenylalanine, 998, 1020
Phenylethene (see styrene)
Phenylhydrazine, 590, 635, 960
Phenylhydrazones, 590, 635
Phenylhydroxylamine, 820
Phenylisocyanate, 785
Phenylisothiocyanate (see Edman degradation)
Phenyllithium, 442
Phenylmethoxycarbonyl group (see Cbz)
Phenylmethylchloroformate (see Benzylchlorofor-mate)
Phenylthiohydantoin, 1019
Phosgene (carbonyl chloride), 801, 803
Phosphorous ylides, 582, 583
Photo-Fries rearrangement, 540
Phthaladehyde, 853
Phthalein dyes, 1080
Phthalic acid, 319, 698, 742, 853, 864,
Phthalic anhydride, 853, 865, 1080
Phthalimido malonic ester synthesis, 1004, 1005
Phthalocyanine dyes, 1067
Phthalonic acid, 853
Phthaloyl group protection, 1012
Phytol, 1062
pI; symbol of isoelectric point, 1002
Picric acid, 439, 1070
Pimelic acid, 699, 718, 724
Pinacolone, 517
Pinacol-pinacolone rearrangement, 517
mechanism of, 517
in symmetrical pinacols, 517
in unsymmetrical pinacols, 518
migratory aptitude, 518, 569, 570
ring contraction, 871
ring expansion, 872
Pinacols, 516, 517, 871
Piperidine, 948
pKa and pKb (see acids and bases)
Plasticizers, 1095
Plastics, 1088
Platinum (IV) oxide (PtO2); (see Adam’s catalyst)
Plexiglass, 1096
Poisoned catalyst (see Lindlar’s catalyst)
Polyacrylonitrile (see orlon), 1096
Polyalkylation, in Friedel–Crafts alkylation, 335
Polyamides, 1098
Polycarbonate, 1100
Polyester, 1099
Polyethene, 246, 1096
Polyfluoroethene (teflon), 247, 1096, 1097
Polyhalogenated alkanes, 420–424
Polyhydric alcohols, 508 (see also ethylene glycols and glycerol)
isomerism, 508
nomenclature, 45, 508
physical properties, 509
Polyhydric phenols (see catechol, hydroquinone, resorcinol)
Polyisobutene, 1096
Polymerization reactions, 268, 1090
Polymers, definition of, 1087
as water softeners, 1107
classification of, 1086, 1087, 1088
stereochemistry of, 1088
Polymethyl methacrylate, 1096
Polynuclear hydrocarbons, 846
Polypeptides, classification, 1009–1010
synthesis, 1013–1016
Polysaccharides, 956, 991–993
Polystyrene, 1097, 1098
Polyurethanes, foam, 1100, 1101
Polyvinyl chloride (abbrev. of PVC), 247, 1096, 1097
Polyvinylacetate, 1096, 1097
Polyvinylidine chloride (see Saran), 1096
Position isomers, 28
Potassium cyanide, 643
Potassium phthalimide, 778
Progesterone, 1064
Proline, 998
Propane, 172–177
bromination, 183
carbene insertion, 192
chlorination, 182
conformational analysis, 92
Newmann projection, 92
Propanedioic acid (see malonic acid)
Propene (see alkenes), 237–238, 1096
Propylene (see propene)
Prosthetic group, 1032
Protecting and blocking groups, in organic transformations, 882, 891, 892, 895–897 (see also acetylation
and sulfonation)
Protection of, amino group, 1011–1012
Protection of, carboxylic group, 1013
Proteins, classification, 1022, 1025–1026
denaturation of, 1027–1028
derived proteins, 1026
structure of, 1°, 2°, 3°, quaternary, 1023–1025
Proteins, structures of,
-helix, 1024
-bend structure, 1023
-pleated sheet structures, 1023, 1025
tertiary and quaternary, 1025
Proton magnetic resonance spectroscopy (see Nuclear magnetic resonance)
PTS, abbrev. of p-toluenesulfonates, 748
Purines, 1034–1035
Purple benzene, 493
Pyrazolone derivatives, 721
Pyrethrin, 1052
Pyridine, 942
basicity, 943
electrophilic substitution reactions, 944, 945
nuleophilic substitution reactions, 946
oxidation reduction reactions, 946–948
preparations of, 944
reactions of, 944–948
structure and aromaticity, 942
Pyridine-N-oxide, 947, 948
Pyridinium salts, 943
Pyridiniumchlorochromate (see PCC)
2-Pyridone, 946
Pyrimidines, 1034–1035
Pyrrole,
aromaticity, 307, 928
electrophilic substitution reactions, 933–938
hydrogen bonding, intermolecular, 928
Kolbe–Schmitt reaction, 941
orbital picture, 928
physical properties of, 928
preparations of, 931, 932
reaction with diazonium salts, 940
reactivity, 933, 934
comparison of pyrrole, furan, thiophene and benzene, 934
comparison with phenol and aniline, 934
reduction reactions, 940–941
resonance and contributing structures, 930
structure, 928
Pyrrolidine, 941

Qualitative test for organic compounds


aliphatic 2° amine, 785
amino acids, 1009
distinguishing 1°, 2° and 3° alcohols, 465, 471
distinguishing aldehydes and ketones, 594, 595, 1078
distinguishing monosaccharides and reducing disaccharides, 991
extra elements, 23
identification of 1° amines, 786, 798
identification of thiol (–SH) group, 502
iodoform test, 606
problems based on, 885–887
unsaturation, 240
urea, 683
Quantitative estimation, in organic compounds, of
alkoxy group, 489
halogens, 24
nitrogen, 24
Quinine, 1058–1059
Quinol (see hydroquinone)
Quinoline, 948–951
Quinones (o- and p-), 548, 1071
Quinonoid theory, 1069

R and S configuration (see absolute configuration), 75


sequence rules, 75–77
assigning, 78, 148
writing structure from, 148–150
in cyclic compounds, 151–154
IUPAC names, 151–154
Racemic mixture, 74, 75
Radical anion, 213, 391
Raffinose, 956
Rancidity, 761
Raney nickel, 593, 633, 1047
Reactions mechanisms, types of, (see details in individual mechanism type; electrophilic addition reactions,
electrophilic aromatic substitution, free radical addition reactions, free radical substitution reactions,
nucleo-philic addition reactions, nucleophilic substitution reactions, rearrangements, oxidation and
reduction)
Reactive intermediates, 115
Reactive methylene compounds, 709–726
acidic character of, 711
enolate ions, chemistry of, 709
keto-enol tautomerism, 710
enol forms, stability of, 711
intramolecular hydrogen bonding, 710
solvents, impact on keto content, 711
reactive methylene group, 710
Reagent types, electrophiles and nucleophiles, 128, 129
Rearrangement reactions, 517, 518, 531, 540, 569, 570, 635, 776, 777, 820
Rectified spirit, 473
Reducing sugars, 969, 989, 990
Reduction of alkynes,
by dissolving metals, 213
by Lindlar’s catalyst, 213
Reduction of organic compounds (see catalytic hydrogenation, Lithium aluminiumhydride, Sodium
borohydride, boron hydride)
Reformatsky reaction, 584
Reforming, 198
Regioselective reactions, 222, 226
Reichert-Meisel value (RM value), of fats and oils, 762
Reimer-Tiemann reaction, 543
Relative configuration, 85
Reserpine, 1058–1059
Resol, 1101
Resolution, 74
Resonance effect, 109–112, 434, 531, 561, 647, 734–737, 741, 745, 815, 822
on stability of allyl and benzyl cations, 120–121
carbanion, 123–124
radical, 127
Resonance, concept of, 15–19
versus tautomerism, 30
Resorcinol, 547
Rhamnose, 987
D-Ribose, 958, 1034, 1035
Ring activator, in electrophilic substitution reaction, 342–344, 358
Ring deactivator, inelectrophilic substitution reactions, 342–344
Ring strain (see Baeyer strain theory)
RNA, 1034, 1037
Rochelle salt; sodium potassium tartarate, 595
Rosaniline dye, 1077, 1078
Rosenmund reduction, 577, 628
Rubbers, 268, 269, 271, 272, 1103
Ruff degradation, 966
Sabatier-Senderens reaction, 172
Saccharides, 955
Saccharin, 752, 994
Sachse Mohr theory, 200
Salannin, 1054
Salicylaldehyde, 543
Salicylic acid, 542, 742–743
Sandmeyer Reaction, 433, 834, 835
Sanger method, 1017
Sapogenins, 1064
Saponification reaction, 760
Saponification value, of fats and oils, 762
Saran, 1096
Sarin, 684
Sarsasapogenin, 1064
Sawhorse projection, 90, 154–158
Saytzeff’s rule, 214, 367, 368
Schiff’s bases, 589, 634, 779
Schiff’s reagent, 1078
Schmidt reaction, 778
Schotten Baumann reaction, 535, 783, 826
Sedatives, 1039
Semicarbazide, 591, 635
Semicarbazone, 591, 635
Semi-essential amino acids, 1000
Separation of mixture of organic compounds, 897–900
Sequence analysis; of amino acids, 1017–1022
Sequence rules, 75
Serine, 998
Sesquiterpenes, 1061–1062
Sevin(a pesticide), 803
Sex hormones, 1064
Silver mirror test (see Tollens test)
Simmons-Smith reaction, 201, 379
Skraup’s synthesis, 949
SN1; substitution nucleophilic unimolecular (see nucleophilic substitution reactions)

SN2;substitution nucleophilic bimolecular (see nucleophilic substitution reactions)

SNi, substitution nucleophilic internal (see nucleo-philic substitution reactions)


Soaps, 763
Soda-lime, 177
Sodamide, 946
Solid phase polypeptide synthesis, 1015–1016
Sodium borohydride [NaBH4]
functional groups not reduced by (list of), 920
general characteristics of, 919
preparation of, 919
reduction of,
acid chloride, 460, 920
aldehydes and ketones, 460, 596, 638, 920
haloalkanes, 920
in demercuration, 230
Sodium ethoxide, uses, 597, 639
Sodium metal; use of, 174, 315, 442
Sodium potassium tartarate (see Rochelle salt)
Solvation, 562, 782
Solvents (see also appendix)
aprotic, 137
effect on SN1 and SN2 reactions, 401
in organic reactions, 137
non-polar, 138
polar, 138
protic, 137
Soman, 684
Sommelet reaction, 628
Sorbitol, 959, 984, 994
Sorensen’s formol titration method, 1007
Specific rotation, 74, 163–165
Spectroscopy, 1109–1134
problems based on, 1123–1124, 1130–1131, 1133–1134
Spermaceti, 763
Spiral compounds, 141, 145, 146
Squalene, 1062
Starch, 992, 993
Step growth polymerization, 1090
Stephen reaction, 578, 628
Stereochemistry, 27, 63
absolute configuration, 75, 148–151
relative configuration, 85
geometrical isomerism, 86, 151
conformations, 90
of cycloalkanes, 94–103, 151–154, 160–163
Stereogenic centre, 66
Stereomutation, 89
Stereoregular polymers; synthesis of, 1093
Stereoselective reactions, 222
Stereospecific reactions, 222, 369–371, 380, 400, 635
Steric effect, 136, 221, 222, 367, 368, 559, 566, 629, 782, 824, 894
Steroids, 1063, 1064
Sterols, 1064
Strain, in ring compounds (see Baeyer’s strain theory)
Strecker synthesis, 1004
Structural formula, 13
Structural isomers (see Constitutional isomers)
Structure and bonding in,
ethane, 9
ethene, 10
ethyne, 12
formaldehyde, 11
methane, 9
Styrene-Butadiene Rubber (SBR), 1106
Styrene, 320, 321, 1097
Succinic acid, 695, 696, 698, 707, 717, 723, 724
Succinic anhydride, 850
Sucrose, 955, 956, 988, 989, 994
Sugars, 956
asymmetric carbons, 957
Fischer projections, 956–958
Haworth projection formulas, 976–980, 986
relative configuration of, 957
D- and L- sugars, 957
sweetness value, 994
Sulfa drugs, 1042–1044
Sulfadiazine, 1043
Sulfanilamide, 1042
Sulfaguanidine, 1043
Sulfinic acid, 498, 502
Sulfonation, 190, 314, 339, 539, 647, 830, 856
as protective and blocking group, 750, 896, 897
Sulfones, 498
Sulfonic acids (see aromatic sulfonic acid), 51, 498, 502
Sulfoxides, 498
Sulfur dioxide, 1078
Sulfur for vulcanization, 1104
Sulfur nucleophiles, 408
Sulfurtrioxide, as electrophile, 339
Sulfuryl chloride, 444
Surfactants, 766
Sweetness scale, 994
Syn-addition, 222
Syndiotactic polymer, 1089
Synthesis gas, fuel oil from, 199
Synthetic fuel, 199
Synthetic polymers, 1086
Synthetic rubbers, 271, 1103

Tabun, 684
Tannins, 1066
Tartaric acid, 641, 707, 708, 984
meso tartaric, 699
racemic mixture, 699
stereochemistry, 72, 702
Tartronic acid, 523, 524, 707
Tautomerism
amido-amidol, 30
keto-enol, 29, 141, 142
nitro-aci, 30
Tautomers, 28, 29, 384
Teflon, 247, 1096, 1097
TEL (abbrev. of tetraethyllead), 197
antiknocking agent, 197
environmental hazard, 197
Terminal alkynes, 290
Terpenes, 1059, 1063
Terphthalic acid, 319
Terylene (see polyester)
Testosterone, 1064
Tetrafluoroethene (see teflon)
Tetrahydrofuran, 941
Tetrahydrothiophene, 941
Tetralin, 851, 852
Tetraterpenes, 1062–1063
Thermocole, 1098
Thermoplastic plastics, 1088
Thermosetting plastics, 1088
Thioacetals, 501
Thioacids, 498
Thioalcohol (see thiols)
Thioaldehydes, 498
Thioethers,
bond angle, 503
nomenclature, 503
physical properties, 503
preparation of, 503–504
reactions of, 504–505
Thioketals, 501
Thioketones, 498
Thiols,
acidic strength, 500
detection of, 502
nomenclature, 498
physical properties, 499
preparation of, 499–500
reactions of, 500–502
Thiophene,
aromaticity, 307, 929–931
electrophilic substitution reactions, 933–938
mercuration, 937
orbital picture, 929
physical properties of, 928
preparations of, 931, 933
reaction with organometalic compounds, 939
reactivity, 933, 934
reduction reactions, 941
resonance and contributing structures, 930
structure, 929
Thiram, 1055
Threo (as prefix), 72, 370, 371
Threonine, 999
Thymine, 1035, 1036
Tischenko Reaction, 604
TMS (tetramethyl silane; (CH3)4Si), 1126
Tollens Reagent/test, 594, 673, 875–878, 887
Toluene, 446
preparation of, 194, 315–316
reactions of, 316–320, 628
p-Tolunesulfonates, 748
Toluic acids (o-, m-, p), 316
Torsional strain, 92
Tosylates (see p-toluenesulfonates)
Trans-addition, 222, 225
Transannular strain, 99
Transesterification, 679
Triacetin, 522
Tricresylphosphate (see plasticizer)
Trifluoroacetic acid, 1015
Trihaloalkanes, preparations and reactions, 422–424
2,2,4-Trimethylpentane (see isooctane)
Triols (see glycerol)
Trioxane, 609
Triphenylmethane; carbanions, carbocations, free radicals of, 121, 124, 127, 166, 568
Triphenylmethane dyes, 1075–1080
Triphenylmethyl radicals, 363
Triterpenes, 1062
Tropylium ion, 121, 303
Trypsin, 926
Tryptophane, 998
Turmeric, 1072
Tyrosine, 999

Ullmann reaction, 442, 534


Ultraviolet (UV)-visible spectroscopy, 1111–1119
aromatic hydrocarbons, 1118, 1119
a,b-unsaturated carbonyl compounds, 1116–1118
conjugated dienes, 1113–1116
Unsaturated halogenated hydrocarbons (see allyl halides, vinyl halides)
, -Unsaturated acids; formation of, 645, 696, 725
, -Unsaturated carbonyl compounds, 599, 610-612, 879, 880, 889
Uranine, 1081
Uracil, 1035
Urea, 1
as monomer, 1102, 1103
basicity, 681
Biuret test, 683
Haber’s process, 682
reactions of, 682
resonance, 682
uses, 682
Urea-formaldehyde resin, 1102
Urethane, 1100
Urotropine, 588

Valine, 998
van der Waals forces (see intermolecular forces)
van der Waals radius, 5
Vat dyes, 1066, 1083
Veronal (see barbitone)
Victor Meyer test, 471
Vinyl acetate, 1096, 1097
Vinyl chloride, as monomer, 1097, 1096
Vinyl halides, 418–419, 561
Vinylic hydrogens, 146
Vinyl polymers, 1095–1097
Vinylic borane, 282, 283
Vinylidine chloride (1,1-dichloroethene), 1096
Vitamin-A, 1062
Vulcanization, 1104
Wacker’s process, 656
Wagner-Meerwein rearrangement, 216, 227, 567
Walden inversion, 400
Walnut shell, 1072
Washing action of soap and detergents, 765
Water gas reaction, 199
Wave number, 1110
Waxes, 762
Williamson ether synthesis, 484, 485
Witt theory, 1066, 1068, 1069
Wittig Reaction, 218, 582, 631
Wohl’s degradation, 966
Wolff-Kishner reduction, 177, 597, 639, 892
Wood alcohol, 472
Woodward Fieser rules, 1113
Wurtz reaction, 174, 207, 442, 446
Wurtz-Fittig reaction, 315, 323, 442

Xanthins, 1062
Xylenes (o-, m-, p), 198, 310, 319

Ylides, 582

Zeisel’s method, 489


Ziegler–Natta catalyst, 1093
Ziegler–Natta polymerization, 1093
Zinc-copper alloy, use of, 201
Zwitterions, 830, 1002

Potrebbero piacerti anche